You are on page 1of 192

SOLUTION manual

n Solutions for Diagnostic Test


n Solutions for Language
Comprehension
n Solutions for Mathematical Skills
n Solutions for Data Analysis and
Data Sufficiency
n Solutions for Intelligence and
Critical Reasoning
n Solutions for Mock Tests (1 to 5)

01_Solutions 01.indd 1 10/30/2009 11:56:28 AM


08_Section.indb 2 10/30/2009 11:52:36 AM
2. 2 22. 2 42. 4 62. 2 82. 2 102. 3 122. 4 142. 2 162. 3 182. 3
3. 3 23. 1 43. 2 63. 2 83. 1 103. 1 123. 1 143. 1 163. 2 183. 3
4. 1 24. 3 44. 2 64. 4 84. 4 104. 3 124. 3 144. 3 164. 2 184. 3
5. 4 25. 2 45. 2 65. 2 85. 3 105. 2 125. 2 145. 2 165. 4 185. 4
6. 3 26. 3 46. 1 66. 2 86. 1 106. 3 126. 4 146. 3 166. 3 186. 3
7. 3 27. 3 47. 4 67. 1 87. 4 107. 3 127. 1 147. 3 167. 4 187. 4
8. 2 28. 2 48. 4 68. 4 88. 1 108. 3 128. 4 148. 4 168. 2 188. 1
9. 2 29. 1 49. 1 69. 1 89. 1 109. 2 129. 2 149. 2 169. 4 189. 4
10. 3 30. 2 50. 4 70. 3 90. 1 110. 2 130. 4 150. 3 170. 1 190. 4
11. 4 31. 4 51. 2 71. 4 91. 1 111. 4 131. 2 151. 4 171. 2 191. 3

1
12. 2 32. 1 52. 2 72. 3 92. 4 112. 1 132. 1 152. 4 172. 3 192. 3
13. 4 33. 2 53. 1 73. 4 93. 2 113. 2 133. 1 153. 3 173. 1 193. 4
14. 2 34. 3 54. 3 74. 3 94. 3 114. 2 134. 1 154. 4 174. 2 194. 2
15. 2 35. 3 55. 2 75. 2 95. 2 115. 4 135. 4 155. 4 175. 2 195. 2
16. 1 36. 4 56. 2 76. 2 96. 1 116. 2 136. 4 156. 4 176. 4 196. 3
17. 3 37. 1 57. 1 77. 1 97. 3 117. 2 137. 3 157. 2 177. 3 197. 3
18. 4 38. 2 58. 1 78. 3 98. 1 118. 2 138. 1 158. 4 178. 3 198. 2
19.
20.
2
1
Solutions for Diagnostic Test
39.
40.
2
3
59.
60.
2
1
79.
80.
2
2
99.
100.
2
4
119.
120.
3
1

Solutions for diagnostic Test


139.
140.
4
1
159.
160.
4
1
179.
180.
3
3
199.
200.
4
3

Solutions for questions 1 to 20: Total dis tan ce 3x u100


Average speed = = =
1. Let the length of the train be x metres. When the Total time 14x
train crossed a person then the distance covered 213/7 kmph. Choice (4)
is its own length. 6. C and D are two consecutive kilometer stones
So x = (66 – 12) u5/18 × 12 = 180 m Choice (4) ? CD = 500 m
2. Solutions
The for questions
price per 1 to 20:
orange when sold in dozens A By equating (1) and (2), b3 = 36 × 36 ×
= 72/12 = Rs 6
1. Let the length of the train be x metres. 216 × 36
The price per orange when sold in scores = (216)3 ⇒ b = 216
= 100/20When
= Rs 5the train crossed a person then the
distance covered is its6 own 36 × 36
5 length. ∴a= = 6. ∴ The larger number
? The required
So x =percentage
(66 – 12)=×5/18 u 100 60° 216
6 × 12 = 180 m 30°
D
2  cent. Choice (4) C B = 216 Choice (3)
= 16 /3 per
Choice (2) In ∆ABC, AB = 3 BC; In ∆ABD,
2. The price per orange when sold in dozens In 'ABC, AB = 3 BC; In 'ABD, BD = 3 AB
3. Let the sum be Rs x ? BDBD= 3= BC3 AB 11. p = m (3p + q + r) → (1)
= 72/12 = Rs 6
[x (1˜2)  7200] (1˜2) = 7200 Ÿ x (1˜2) (1˜2) ACD∴= BD
500 =m,3CB
BC= 125 m AND BD = 375m q = m (p + 3q + r) → (2); r = m (p + q +
The price per orange when sold in scores
= 7200 (2˜2)
? AB = 125= 3500 m m, CB = 125Choice (3) 3r) → (3)
= 100/20 = Rs 5
? x = 11000. Choice (3) ACD m AND
6−5 (1) + (2) + (3) ⇒ p + q + r = 5m
∴ The required percentage BDfirst
7. Let the = 375m
term be a.
4. The amount of profit that is distributed=among×the 100 (p + q + r)
common share holders = 640000 – 80000 6 –
Let the common difference be ‘d’.
∴ AB = 125 3 m Choice (3)
Ÿ 5 u a + 5d + 6d + 7d + 8d + 9d ∴m = 1/5 [∵ p + q + r ≠ 0] Choice (4)
100000 u= 1016u2/40/100
per cent.
= Rs 1,60,000 Choice (2)
3  [5 u a + 0d + d + 2d + 3d + 4d] = r50
1, 60, 000 7. Let the first term be a.
3. Let
? The the sum
required be Rs x =
percentage u(100 ? 25d = r 50 Ÿ d = r 2 12. Let the distance between his house and
6, 40, 000 ? t2 Let
= a the
+d= common
4 + 2 or difference be6 ‘d’.
4 – 2 that is, or 2. school be x km.
[x (1⋅2) −
per cent) = 25 per cent7200] (1⋅2) = 7200 ⇒ x (1⋅2) ⇒ 5 × a + 5d + 6d + 7d + 8d + Choice 9d (3)
(1⋅2)
Choice (1) − [5 × a + 0d + d + 2d + 3d + 4d] x x 15 x 1
8. Let the sum be Rs P. − = ⇒ = ⇒ x = 7.5 km
= 7200 (2⋅2)
5. Let the total distance be 3x km. =§±50 5 6 60 30 4
15 10 ·
∴ x = 11000. § 1 Choice (3) ? Pu ∴ ¨ 25d = ±
¸ =50450⇒ ?d P
= =± Rs
2 9000
· © 100 ¹  Choice (2)
Time taken to cover x km ¨? of 3x x ¸
© 3 that is ¹distributed ∴ t2 = a + d = 4 + 2 or 4 – 2 that is, 6 (2)
Choice
4. The amount of profit or 2. Choice (3) 13. Let the speeds of A and B be 4x m/s and
= x/25 hours
among
Time taken thethecommon
to cover remainingshare
2x km holders =
640000 – 80000 – 100000 × 10 × 40/100 3x m/s respectively.
= 2x/20 hours 8. Let the sum be Rs P.
= Rs 1,60,000 Time taken to meet anywhere on the
1, 60, 000  15 −10  Sol/764 track
∴ The required percentage = ∴ P×  = 450 ∴ P = Rs 9000
6, 40, 000   100  Choice (2) 600
= = 600/x seconds
4x − 3x
×(100 per cent) = 25 per cent Choice (1) 9. Probability of getting at most 2 heads when
3 coins are tossed = 1 – [Probability of In 600/x sec distance covered by
– 3 600
5. Let the total distance be 3x km. 1 7 A= × 4x = 2400 m
getting exactly 3 heads] = 1   = x
Time taken to cover x km  2 8
So A meets B after covering every
 1  Choice (2)
∴ of 3x = x  2400 m. So A
 3  crosses B twice. Choice (4)
10. Let the two numbers be a and b
= x/25 hours
Time taken to cover the remaining a 36 14. Let the number be x and k be any non-
∴ = (∵ 36 is their mean
2x km 36 b negative integer.
∴x = kd + 10 ⇒ x2 = k2d2 +20kd
= 2x/20 hours proportional)
Total dis tan ce + 100
Average speed = 362 Remainder of
Total time ⇒a=  ------ (1)
b
 x2   100 
3x ×100 a b   = 0 + 0 + Re mainder of  
= 21 /7 kmph.
3
 d   d 
= (∵ 7776 is their third
 14x Choice (4) b 7776
= 11 ⇒ d is a factor of 89.
proportional) and d is not less than or equal to 11. So
6. C and D are two consecutive kilometer
2 d = 89 as
stones b
⇒a=  ----- (2) 89 is prime
7776 X3 = k3d3 + 30kd(kd + 10) + 1000
∴ CD = 500 m

08_Section.indb 3 10/30/2009 11:52:37 AM


crosses B twice. Choice (4) ? The required percentage
320  50
14. Let the number be x and k be any non-negative = u 100 540% . Choice (1)
integer. 50
2 2 2
?x = kd + 10 Ÿ x = k d +20kd + 100
Remainder Solutions for questions 21 to 23:
§ x2 · § 100 · 21. The synonym of ‘Proliferating’ is ‘growing’.
of ¨ ¸ 0  0  Re mainder of ¨ ¸ = 11 Ÿ d Choice (1)
4  © nd  ¹ Solution Manual © d ¹
is a factor of 89. 22. ‘Integrate’ is ‘to become a part of’. Hence the
and d is not less than or equal to 11. So d = 89 synonym of ‘integration’ is ‘combination’.
as 89 is prime  x3  22. ‘Integrate’ is ‘to become a part of’. Hence(2) Solutions for questions 35 to 37:
Choice
X3 = k3Remainder of + 10)+=1000
d3 + 30kd(kd 0 + 0 + Remainder of the synonym of ‘integration’ is ‘com- 35. ‘Little did I know that I would be re-
 d  23. ‘Comprehensive’
bination’. means ‘includesChoice
every (2)
thing’.
 1000of ¨ § x3 · Hence the synonym of ‘comprehensive’ is turning home’ can be best replaced by ‘I
Remainder = 0 + 0 + Remainder
  © d ¸¹ 23. ‘inclusive’.
‘Comprehensive’ means ‘includes Choice
every(1) did not know that I would be returning
 d 
§ 1000 · § 1000 thing’.
for Hence the 24
synonym
to 26: of ‘compre- home’—with enduring images. The other
of ¨ = Remainder
¸ = Remainder
 1000  · = 21 Solutions questions
of of ¨ 89 =¸21 Choice (2) hensive’ is ‘inclusive’. options do not suit the context and the in-
© d ¹  89  ¹
© 24. The synonym for ‘conspicuous’ is ‘obvious’. The
Choice (2)  antonym is ‘obscure’. Choice (1)(3)
Choice tention of the sentence. Choice (3)
15. 15. Solutions for questions
25. ‘Tranquility’ means 24 and peaceful’. The 36. ‘A stint at an art college certainly helps
to 26:
‘calm
antonym is ‘tumult’. Choice (2) hone one’s skills’ can be best replaced by
24. The synonym for ‘conspicuous’ is ‘obvi-
50 26. ‘Prodigious’ means ‘very large, enormous or ‘studying at an art college helps to sharpen
ous’. The antonym is ‘obscure’.
impressive’. The antonym is ‘insignificant’. one’s skills (to hone means to sharpen).
 Choice (3)
T Sol/765  Choice (4)
50 3 25. ‘Tranquility’ means ‘calm and peaceful’.
37. ‘Federalism is the bedrock of our democ-
The antonym is ‘tumult’. Choice (2)
racy’ can be best replaced by ‘federalism
 50  –1  1  26. ‘Prodigious’ means ‘very large, enormous is fundamental to our democracy’. Bed-
θ = tan–1   = tan   = 30°.
 50 3   3 or impressive’. The antonym is ‘insignifi- rock means the main principles on which
 Choice (2) cant’. Choice (3) something is based, which means funda-
Solutions for questions 27 to 30: mental. Choice (1)
16. Let Balu join after x months.
∴ 12 × 60000 : (12 – x) × 48000 27. The error is in part 3 ‘Different’ is the ap- Solutions for questions 38 to 40:
=3:2 propriate word. Choice (3) 38. The error is in (2), as the reference is
12 × 5 3 28. ‘Economic’ is the right word. Choice (2) to an action that began sometime in the
= = ⇒ 12 – x = 10.
(12 − x ) × 4 2 29. ‘Inclined’ is the right word. Choice (1)
past is still continuing and hence there
has been (present perfect tense) a steady
∴x=2 30. ‘Prevail’ is appropriate. Choice (2) decline should be used in the place of
 Choice (1) there was. Choice (2)
Solutions for questions 31 to 34:
17. Let the number of apples with the girl 39. The error is in (2) as one succeeds in do-
be x 31. ‘Mount’ means to organize and begin an ing something and not succeeds to do
x (100 − 13)(100 − 75 ) activity or event, which goes with the first something.
∴ = 261 blank.
(100)(100) Hence ‘succeeded to project’ should be
‘Unearth’ means to discover proof or replaced by succeeded in projecting.
 100   100  some other information, especially after  Choice (2)
⇒ x = 261     = 1,200
 87   25  careful searching, which goes with the
second blank. The other option do not go 40. The error is in (3) as one learns the ways of
 Choice (3) life and not the way in life. ‘Ways of life’
with the context. Choice (4)
18. Let the average of the 16 members means the manner in which people live.
be A. 32. The first and second blanks have to go  Choice (3)
with the last part of the sentence which
15 × 200 + A + 750 says “because of the money that follows”. Solutions for questions 41 to 45:
=A Since “impoverished” means being very The given words are arranged in alphabetical
16
3000 + 750 = 15A ⇒ 3750 = 15A poor, the impoverished cricket board is order, in the following method.
only too happy (happiness is a positive In the first step, the word with the minimum
3750 emotion) to receive the team because it number of letters is shifted to the first position.
A= ; A = 250 ⇒ ∴ A + 750 = 1000 suits their monitory interests/requirement. If two words have the same number of letters,
 15 Choice (4)  Choice (1) then the one which occurs alphabetically first
19. Let radius of seed be r will be placed first.
33. The season in the first blank has to be
∴ radius of cherry = r + r + r = 3r one in which people are likely to spend 41. Input: all of you must follow these rules
∴ The ratio of their volumes good time. Hence, ‘festive’ season and Step Ι: of all you must follow these rules
4 4 ‘shopping’ would be the right options. It Step ΙΙ: of all you must rules follow these
= π(3r)3 : πr 3 = 27 : 1 Choice (2)
3 3 is not to be taken that people necessarily Step ΙΙΙ: of all you must rules these
do not spend time during other seasons follow Choice (1)
20. Let the cost of each orange be Rs x like summer and monsoon. The words in
The cost of all apples = 80 × 4x = 320x 42. Input: I do not know who he is
the other options do not fit the context.
The cost of all oranges = 50 × x = 50x Step Ι : I do he not know who is
 Choice (2)
∴ The required percentage Step ΙΙ: I do he is not know who
34. ‘Although’ indicates that the second half Step ΙΙΙ: I do he is not who know
320 − 50 of the sentence does not agree with the
= × 100 = 540% . Choice (1)  Choice (4)
50 first half. Although the ‘industries’ has its
core competence (adequately qualified or 43. From the choices, if the input is
Solutions for questions 21 to 23: capable) in one area, it claims that other (1) Input: going to be a king is he
21. The synonym of ‘Proliferating’ is ‘growing’. fields are not unrelated to it. Step Ι: a going to be king is he
 Choice (1)  Choice (3) Step ΙΙ: a be going to king is he

08_Section.indb 4 10/30/2009 11:52:38 AM


Solutions for Diagnostic Test  n  5

Step ΙΙΙ: a be he going to king is Hence, ΙΙ is implicit. If it is assumed that Solutions for questions 56 to 60:
Step IV: a be he is going to king the customers do not abide by the notice, 56. RI is not able to explain the reasons be-
Here, step IV is not the last step. then the notice would not have been is- hind stout’s arrest. By stating that the
(2) Input: he is going to be a king sued. Hence, Ι is implicit. Choice (4) police are corrupt we can not justify the
Step Ι: a he is going to be king arrest. RII gives the appropriate reason
Step ΙΙ: a be he is going to king Solutions for questions 51 to 55: behind the arrest. Choice (2)
Step ΙΙΙ: a be he is to going king
Step IV: a be he is to king going 51. The statement refers to the amount of 57. From the assertion, more crimes are com-
Here, step IV is the last step. leadership and the amount of manage- mitted by young people than by any other
 Choice (2) ment involved in a successful transfor- group of people. So, crime is becoming
mation. The word ‘only’ indicates that a choice for the young people which
44. From the choices, if the input is the statement is stressing on the signifi- will be the cause for the assertion. Hence,
(1) Input: who is the last person to cance of leadership over management in RI is the reason and RII is irrelevant.
come a successful transformation. Though the  Choice (1)
Step Ι: is who the last person to role of management is less significant as
58. RI is certainly the fact which violates hu-
come compared to the role of leadership in a
man rights and even corrupts the youth.
Step ΙΙ: is to who the last person successful transformation, its importance
So, it could be the reason. Human rights
come is not referred to in the statement. Hence,
group and the rise in charges of the MMS
Step ΙΙ is not the last step Ι does not follow. As the statement refers
are in no way related. So, RII cannot be
(2) he is the last person to come to leadership and management indepen-
the reason. Choice (1)
Step Ι: he is to the last person come dently of each other, ΙΙ can be concluded.
Step ΙΙ: he is to the come last per- From the numbers used in the statement, 59. RI is not the reason. RII is the reason,
son ΙΙΙ can be concluded. as the Indian government follows non–
Here, step ΙΙ is the last step. violent policies. Hence, India can be
 Choice (2)
 Choice (2) considered a non-violent country.
45. Input: all the students must be present 52. The statement refers to the creation of  Choice (2)
Step Ι: be all the students must present thousands and thousands of large organi-
60. RI, certainly is the reason because of
Step ΙΙ: be all the must students present sations for the first time. But it is not clear
which the ban should be imposed. If cel-
 Choice (2) whether the statement is about such huge
ebrations consume excessive amounts
numbers or to the large organization it-
of money and time, not only Valentine’s
Solutions for questions 46 to 50: self. Hence Ι does not follow. From the
day but also many more such celebrations
46. The word ‘Despite’ indicates that the statement it cannot be concluded, whether
should be banned. Choice (1)
statement assumes that an icon in Bol- there is anything else apart from good
lywood will be affected by the trapping managers which can keep the organisa- Solutions for questions 61 to 66:
of tinsel town. Hence, Ι is implicit. From tions functioning. Hence ΙΙ does not fol-
61. Using the first statement, we can find the
the statement ΙΙ, it appears that becoming low. The statement refers to the organisa-
expenditure on food as a fraction of the
an icon occurs before getting affected. tions as ‘those bureaucracies’. Hence ΙΙΙ
total expenditure. First statement alone
Hence, ΙΙ is not implicit follows. Choice (2)
is sufficient. Using the second statement,
 Choice (1) 53. According to the statement, China may as we do not know the earnings of this
become the largest economy sometime month, we cannot answer the question.
47. Unless the University assumes that some  Choice (1)
people want to do Hotel Management in the future. Hence, Ι follows. From the
statement it is clear that China has the 62. First statement alone is not sufficient, as
course, the advertisement would not have
highest growth rate, even though it is not we do not know the exact tip. Using sec-
been given. The advertisement is given
with an intention of attracting the people the largest economy. Hence, ΙΙ does not ond statement alone, as we determine the
to the University also. Hence, both Ι and follow, but ΙΙΙ follows.  Choice (1) total sum he spent, on lunch alone. There-
fore we can find the answer. Choice (2)
ΙΙ are implicit.
54. From the statement it cannot be concluded
63. First statement alone is not sufficient, as
 Choice (4) that either education or opportunity alone
we do not know the ratio of their efficien-
48. The statement assumes existence of the brings about inspiration. Hence, neither Ι
cies or the efficiency of Tinku. Second
subject and the other hypothetical things nor ΙΙ follows. According to the statement
statement alone is sufficient, as we know
referred to. Hence, both Ι and ΙΙ are im- education brings about opportunity and
the ratio of their efficiencies. Choice (2)
plicit. Choice (4) because of opportunity one get inspired,
hence ΙΙΙ follows. Choice (3) 64. First statement alone is sufficient, as we
49. The word ‘pity’ indicates that the state- know the total births from 1001 to 1005 and
ment expects the government to take 55. According to the statement ‘Lage Raho that from 1001 to 1004. From the second
charge of the situation. Hence, Ι is implic- Munnabai’ has exploded the myth that statement, we have the average number of
it. The statement has no reference to the sequels are bad. From this both Ι and ΙΙ annual births from 1001 to 1004 as 5 less
feelings of the government towards the can be concluded. From the statement, than 820. For the same reason as above, the
affected areas. Hence, ΙΙ is not implicit. it cannot be known whether ‘Lage Raho second statement alone is also sufficient.
 Choice (1) Munnabhai’ is a movie or something else.  Choice (4)
50. When a notice is given, it is assumed that Hence ΙΙΙ does not follow.
65. First statement alone is not sufficient, as
there is a necessity to give such notice.  Choice (2) it gives no information about the speed or

08_Section.indb 5 10/30/2009 11:52:38 AM


6  n  Solution Manual

the distance. Second statement alone is suf- rank. Brand value of HP in 2003 = $8.88 No. of kids who watch only cartoons (C)
ficient, as it gives the speed, that is, the dis- Bn. ⇒ 71st rank. = 760; Amount Collected from A = 2504
tance travelled in twenty minutes is given. ∴ Since Ralher's brand value is greater x 10 = 25040
than Oracle's but less than that of HP it Amount Collected from B = 1646 x 8 =
 Choice (2)
must be ranked somewhere between 71 13168; Amount Collected from C = 760 x
66. First statement alone is not sufficient, as and 87. Choice (3) 5 = 3800
we do not know the number of boys or Total amount collected = 25040 + 13168
73. Brand value of Cola Cola in 2002
total strength of the class. + 3800 = 42008 Choice (1)
70.3 x100
Second statement alone is sufficient, as = = $62.48 bn.
112.5 78. By observation we know that the most
we know the ratio of the number of boys popular programme is cartoon and the
and girls. Choice (2) Cola Cola’s Brand value was 16.66 per
cent (that is, 1/6) less than that of Micro- least popular is News.
Solutions for questions 67 to 69: soft in 2002 (that is Microsoft was 6/5 Total number of people watching cartoons
times the value of Cola Cola). = 760 + 525 + 311 + 440 + 590 + 200 +
67. The required percentage increase 370 + 68 + 892 + 222 + 151 + 127 + 63 +
∴ Microsoft's brand value in 2002
(24 + 42 + 60 + 90 )− (20 + 35 + 45 + 59 ) 132 + 59 = 4910
= 62.48 x 6
= ~ $75 bn. Choice (4) (count all 1st preference and 2nd prefer-
20 + 35 + 45 + 59 5
ence as well as only preference)
× 100
74. Brand value of top 5 brands in 2003 = Total no. watching News = 68 + 74 + 118
216 −159 $260.7 Bn.= x; Brand value of brands + 63 + 69 + 132 + 74 + 23 + 59 + 88 + 53
= × 100 ≅35.8 per cent ≅ 36
159 ranked 6-10 in 2003 = $125.7 Bn + 48 + 68 + 31 + 29 = 997
per cent 125.7 Difference in cartoons and News = 3913
 Choice (1) ∴ = 0.482 ∴$125.7 Bn. is 0.482x 3913
260.7 ∴ x 100 = 392.4. Approximately it
68. The number of people who filed the tax  Choice (3) 997
returns in 2003-04 in any income range is is 400 per cent Choice (3)
greater than those who filed the tax returns Solutions for questions 75 to 79:
from that range over the previous years. 79. 1st preference is cartoons = 3264; 1st
75. We need to find out the number of people
∴ Maximum number of people filed the preference is News = 399those whose 1st
whose 1st preference is either Games or
tax returns preference is news and cartoons = 3663
Quiz and whose 2nd preference is either
in 2003-04. Choice (4)  → (A)
Horror or Music. This means we need to
1st preference is movies = 1292 → (B)
90 − 70 count all those people whose 1st prefer-
69. The required percentage increase = Difference in (A) and (B) = 3663 – 1292
70 ence is Games and 2nd preference is ei-
= 2371
ther Horror/Music or all those whose 1st
× 100 ≃ 28.6 per cent ≃29 per cent  Choice (2)
preference is Quiz and 2nd preference is
 Choice (1)
Horror/Music. Solutions for questions 101 to 120:
Solutions for questions 70 to 74:
1st 2nd 101. As pipe P is at the middle of the tank, the
No of people
70. Cola Cola, GE, Nokia have improved preference preference water that is filled =3000/2 = 1500 litres.
their ranking. The value of these brands Games Horror 74 In this, the water filled by P and Q will be
together is $142 Bn Games Music 53 in the ratio of 2 : 3.
Microsoft, Intel, Mc. Donald’s and Mer- Q filled 3/5 × 1500 = 900 litres.
Quiz Horror 43
cedes have a ∴ part of the tank filled by Q = 900/3000
Quiz Music 68
deteriorated ranking. = 3/10 Choice (2)
The value of these brands together is Total number of people satisfying this crite-
$142.4 bn. ria is 238 102. The perimeter of the garden, P = 2 (l + b)
∴ Difference in these two values = 142.4 Total number of people whose 1st and = 2 (37.5 + 12.5) = 100 m.
− 142 only preference is Cartoon = 760 The total cost of fencing = 4 × 6 × 100 = Rs
= $0.4 bn Choice (3) ∴ 238/760 x 100 = 31.3 per cent 2400  Choice (3)
 Choice (2) 103. Let the two digit numbers satisfying the
71. Brand value of H.P. and Oracle
160 x (17.8 + 9) 76. Only one type of programme is only Car- given condition be of the form a, b. a + b
= = $11.9 bn toons, only Sports, … etc. The fastest is a perfect square.
360
way to solve it is to count all the numbers As a ≤ 9 and b ≤, a + b ≤ 18
The 10th ranked brand is valued at $21.4 along the diagonal i.e 760 + 214 + 152 + ∴ a + b can be 1, 4, 9 or 16.
bn. hence 112 + 112 + 72 + 83 + 23 If a + b = 1, (a, b) = (1, 0)
we cannot estimate the exact ranking of If a + b = 4, (a, b) = (1, 3), (2, 2), (3, 1) or
ORAHP = 1528 Choice (2)
(4, 0)
in 2003. Choice (4) 77. Number of kids whose 1st preference is car- If a + b = 9, (a, b) = (1, 8), (2, 7), (3, 6) …
toons (A) = 525 + 311 + 440 + 590 + 200 + (8, 1) or (9, 0)
72. Brand value of Ralher in 2003 =
370 + 68 = 2504 If a + b = 16, (a, b) = (7, 9), (8, 8)
32 + 11.1 No of kids whose 2nd preference is car-
x 18.9 = $8.15 bn or (9, 7)
100 toons (B) ∴ ab has a total of 17 possibilities.
Brand value of Oracle in 2003 =$ 4 bn = 892 + 222 + 151 + 127 + 63 + 132 + 59  Choice (1)
⇒ 87th = 1646

08_Section.indb 6 10/30/2009 11:52:38 AM


Solutions for Diagnostic Test  n  7

104. Distance covered by B when they meet = 110. Let the speed of A be s m/sec. Let its length 118. Given (A + B)’s 1 day work = 1/24
6.5 – 2 = 4.5 km be L m. (B + C)’s 1 day work = 1/36
Distance covered by A when they meet = (C + A)’s 1 day work = 1/48
6.5 + 2 = 8.5 km L + 300 ∴ 2(A + B + C)’s 1 day work
= 40. The required time =
Time taken by B to cover 4.5 km =
4.5  s =
1
+
1
+
1
18 1.2L + 360 24 36 48
1 2s = 13/144; ∴ (A + B + C)’s 1 day work
= hr = 15 min = 13/288
4 1.2[L + 300]
⇒ = 24 seconds.Choice (2) ∴ B’s 1 days work = (A + B + C)’s 1 day
B started from P, 2 minutes after A started 2s work
from P.
111. Let the new height be x cm. − (A + C)’s 1 day work
Time taken by A to cover 8.5 km = 15 + 2 =
17 min 4 7 7 7 13 1 7
Speed of A =8.5/17 × 60 = 30 kmph
[π (7) (7) (20 – x)] = (π )       = −
288 48 288
=
3 2 2 2
 Choice (3) ⇒ 20 – x = 7/6. Choice (4) ∴ B can complete the work in 288/7 that is,
411/7 days. Choice (2)
105. Let the numerator be x. 112. Profit = Investment x Time period
x ∴ Here ratio of profits = 3 × 2 : 4 × 3 = 1 : 2 119. L
 et b be the height of raised field
Form of the fraction = 2 . Now equating volumes
x −2  Choice (1)
π . (49) (1.46) = (300 – π49)h ⇒ h = 1.54 m
x+2 113. P(winning a prize for atleast on one  Choice (3)
Given, =1 / 2 ⇒ 2x + 4 = x² + 1
x2 − 2 + 3 ticket) =
120. Let the expenditures of A and B be 5k and
⇒ x2 − 2x − 3 = 0 ⇒ x = 3 or −1 1 – P(“Losing on all tickets”)
4k respectively.
If x = 3, original fraction is 3/7 = 1 – (0.8)4 = (1 + (0.8)²) (1 – (0.8)²) =
Let A’s savings be a, ∴B's income = a.
If x = −1, original fraction is −1/−1 = 1. Hence (1.64)
Given (5k + a) − (a) = 5,000 ⇒ 5k =
x ≠ –1. Choice (2) (0.36) = 0.5904 Choice (2)
5,000
106. Each ring can be positioned in 10 ways. 114. Each question can be answered in 3 ⇒ k = 1,000.
Hence, the three rings can be positioned in ways either mark true or mark false or ∴ B's expenditure = 4k = Rs 4,000.
103 ways. Of these, 1 combination is suc- leave blank. Hence, total ways are 320.  Choice (1)
cessful. Hence, unsuccessful attempts are  Choice (2) Solutions for questions 121 to 140:
103 – 1. Choice (3) 115. L
 et the scores of the five students be a, b, 121. Option (4) – from paragraph 2.
107. A c, d, e such that a > b > c > d >e. Then, a
 122. Option (4) – from paragraph 1.
+ b will be the highest sum.
O a + c will be the next highest sum and b + 123. Option (1) – from paragraph 3.
 
C1 C2 c or a + d may be the third highest sum 124. Option (3) – from paragraph 5.
B
∴ a + b = 187; a + c = 179 125. Option (2) – from paragraph 6.
b + c = 177 and a + d = 176 ……case (1) There is a failure of application of safety
‘O’ is the mid point, ∴ C1O = OC2 or a + d = measures, because of negligence on the
= 4 cm. 177 and b + c = 176 ……case (2) part of human subjects. Hence, human
AOC, is a right angled triangle Case (1): a = 94.5, b = 92.5, c = 84.5, d participation is reduced, whenever pos-
= 81.5 sible.
∴ AO = (AC1 ) 2 − (C1O) 2 = 52 − 42 Case (2): a = 95, b = 92, c = 84, d = 82
So, the required average cannot be 126. Refer to the last sentence of para 1.
(α + β) 2 − 4αβ2 = 3 cm. uniquely determined.  Choice (4)  Choice (4)
∴ AB = 2(AO) = 6 cm. Choice (3) 127. Refer to the last sentence in para 2. Op-
116. GFEH is a trapezium where GF = 3 cm
tions (2) and (3), though true, have been
108. Total numbers using all the 6 digits minus and HE = 4 cm and the distance between
cited by the author to contrast greenhouse
those starting with 3 and those starting them, h = 12 cm
gases and aerosols. Choice (1)
with 43 = 120 – 24 – 6 = 90 Choice (3) ∴ The area of the shaded region is (1/2)
h (a + b) = 128. O
 ption (1) is rendered false by the first
109. Sum of roots = −bn/am = α + β
6 (3 + 4) cm2 = 42 cm² Choice (2) sentence of para 7 which implies that
Product of roots = cl/am = αβ
South Asia and the northern Indian Ocean
117. Let the length of the race be x m. Let the have higher concentrations of aerosols.
Difference of roots, = (α + β) 2 − 4αβ2
speed of B be y m/s. Speed of A =4y/3 The last line of para 6 renders option (2)
=α−β
m/s. A and B finish the race simultane- false. Para 6 and para 7 lead us to under-
b 2 n 2 − 4clam
= b 2 n 2 / a 2 m 2 − 4cl / am = ously. So time taken by A to cover x m stand that option (3) is true. The phrases
a 2m2
is the same as that taken by B to cover “soot particles do not just block in the last
Difference = Product (x − 120) m. line of para 6 and “aerosols over south
b 2 n 2 − 4clam (cl) 2 x x − 120 3x x −120 Asia / the northern Indian Ocean” render
⇒ = = ⇒ = ⇒ 3x
a 2m2 (am) 2 4y / 3 y 4y y option D to be true. Choice (4)
⇒ b2n2 − 4clam = c2l2 =4 129. Options (1) and (4) can clearly be ruled out
⇒ b2n2 = c2l2 + 4clam Choice (2) (x – 120) ⇒ x = 480 Choice (2) as there is neither a difference of opinion

08_Section.indb 7 10/30/2009 11:52:39 AM


8  n  Solution Manual

expressed nor a supporting stance taken on 1010, 1111, 10110, 11111, 101010, 149. Shravya Choice (2)
a particular line of thought. Between (2) _________
150. White Choice (3)
and (3), the words like ‘however’, ‘nor’ The decimal equivalents of the above bi-
in sentences 1/3 of para 2 respectively, nary numbers will be 151. Green Choice (4)
‘unlike’ in line 1 of para 3 have been men- 10,15,22,31,42
Solutions for questions 152 to 154:
tioned to give information on something ⇒ 10+5.15+7,22+9,31+11,42+13,55.
that is commonly believed. Hence, option Hence, the binary form of 55 which is 152. The path traversed by David is as fol-
(2) is the right answer choice. Choice (2) 110111 is the next number. Choice (1) lows:
130. Refer to sentence 3 of the last para. 144. Given, 35 : 143 : : ____ : 667
F
 Choice (4) 35 can be represented as. 5 × 7 7 km
143 can be represented as. 11 × 13 D 2 km E
131. Refer to line 4 of para 1. Choice (2)
These are the products of consecutive
132. Refer to lines 5 and 6 of para 1. prime numbers. 11km
 Choice (1) Here, 667 can be represented as. 23 × 29.
B 3 km C
133. The author mentions “men of highest Two previous consecutive prime numbers of
type”, eg. statesmen like Lincoln, etc., to 23 are 17 and 19. 5 km
motivate. Thus, the passage does not only ∴ The missing term = 17 × 19 = 323.
A
merely preach but also motivates. Hence,  Choice (3)
option (a) Choice (1) The vertical distance is
145. The angle θ, between the hands of the
(AB + CD + EF) = 5 +11 +7 = 23 km
134. Refers to the last line of para 2. clock at 20 minutes past 7 is given by the
 Choice (4)
 Choice (1) formula
θ = 30h − 11/2 m. 153. Kareena is Karishma’s mother’s only
135. Refer to the lines 12 – 14 of para 2. ⇒ θ = 30 × 7 – 11/2 × 20 = 100° son’s daughter. Hence, Karishma is Kar-
 Choice (4)  Choice (2) eena’s father’s sister. Hence, Karishma is
136. The drastic reduction in the population of the aunt of Kareena. Choice (3)
146. After replacing the symbols, the expres-
the Andamanese can be attributed to all the sion becomes 25 – 32 ÷ 2 + 3 × 8 = 25 – 16 154. In total, Ajay’s birthday will be 26 days
reasons. [Refer para 2]. + 24 = 33 Choice (3) before Pradeep’s birthday.
 Choice (4) Solutions for questions 147 to 151: ∴ 26 days = 5 odd day.
From (iii) and (iv), the girl wearing the green Hence Ajay’s birthday will be on Saturday.
137. The great Andamanese lacked resistance
coloured dress is sitting to the right of girl wear-  Choice (4)
to diseases like pneumonia and measles.
ing the blue coloured dress.
[Refer para 1]
From (i), (iii) and the above result, the girl wear- Solutions for questions 155 to 159:
 Choice (3) ing the red coloured dress is sitting to the right Let us tabulate the data given.
138. T
 he first contact was made when the of the girl wearing the yellow coloured dress and
British established a penal colony. [Refer the girl wearing the white coloured dress is sit- Name Place Month
para 1]. Choice (1) ting to the right of the girl wearing the orange Naresh Hyderabad March
coloured dress. Kishan Not January
139. The British hired the Andamanese to trace Also from (vi), the arrangement is Siddharth Not January
escaped convicts and to fight the Jarawas.
 Choice (4) red Prasant March
Shravya yellow Lucknow March
140. The increase in population can be (Orange) Bibhas Not January
attributed to influx of Indian settlers.
 Choice (1) Ritesh Ahmedabad
Ankur Delhi
Solutions for questions 141 to 146: white green
Mumbai Not February
141. The given sequence is blue
Chandigarh March
Bhavya
BL2MZ4AQR3F7ER8AZ13C
MZTOP47 From (v) and (ii), and the above results, Atmost three people celebrate their birth-
8th letter/digit to the 17th from left = days. So, for two months, three birthdays
(red)
(17 – 8)th or 9th element from left which Kavya are celebrated in each month and in the other
is R. Choice (3) month, two birthdays are celebrated. that is,
(orange) (yellow)
Shravya Divya birthdays of Kishan and Siddarth, which can-
142. A’s sister’s grandfather is A’s grandfa-
notbe in March or January. So, that must be
ther. A’s grand father’s only son is not
in February.
A’s father, because he has only one child
∴ Bibhas is from Chandigarh and he celebrates his
whereas A’s parents have at least two chil- (green)
(white) birthday in March.
dren that is, A and A’s sister. Hence, the Navya Ramya
(blue) ∴ Ritesh, Ankur and Charan celebrate their
person is A’s mother’s brother, who is the
Bhavya birthdays in January.
uncle of A and the uncle’s only child is
∴ Charan is from Mumbai.
the cousin of A. Choice (2) 147. Yellow Choice (3) Prasant is from Chandigarh.
143. The given sequence is 148. White Choice (4) The final distribution table is as follows

08_Section.indb 8 10/30/2009 11:52:39 AM


Solutions for Diagnostic Test  n  9

A - 330 4000 100 200 8 Solutions for questions 166 to 170:


Name Place Month
Ankur Delhi January B - 767 6000 100 300 18 166. Total imports = 17 + 18 + 13 + 8 + 21 + 13
Ritesh Ahmedabad January B - 777 4500 100 400 18 + 17 + 15 + 16 + 15 = 153; Total exports
B - 777 1000 100 600 6 = 18 + 22 + 10 + 10 + 19 + 14 + 14 + 12
Charan Mumbai January
− 400 + 17 + 16 = 152
Kishan Indore / Pune February Trade deficit = 153 – 152 = 1; Average
Con- 100 100 350 3.5
Siddharth Pune /Indore February corde imports = 153/10 = 15.3
1
Bibhas LucknowDelhi March BA - 41 ∴Required percentage = x 100 =
Ankur January Logic: + 6 + 6 + 6 + 6 + 6 + 6 + 6 + 6 6.5 per cent 15.3
Ritesh Ahmedabad January Bom- 6000 200 100 12
Prasant Chandigarh March  Choice (3)
Charan Mumbai January bardier
Naresh Hyderabad
Kishan Indore / Pune March
February EJ - 200Code: G S H O Z O U T
Siddharth Pune /Indore February 152 ×1010
155. Charan lives in Lucknow
Mumbai. Choice (4) Similarly, 167. Total tonnage of exports =
Bibhas March ∴Code:
Total VCarrying
X O Capacity
T Z U ofA allZ Planes 2000
Prasant
156. Kishan lives inChandigarh
either PuneMarch
or Indore.
Naresh Hyderabad March in 2003 = 77 Mn passengers = 76 x 107; Total tonnage of imports
 Choice (4) 60 Mn
∴Logic:
Total –Occupancy Rate = – 6 – 6x–100
6 – 6 – 6 – 6 – 6 77 6 153 ×1010
155. Charan lives in Mumbai. Choice (4) Mn
= 78 = = 51 x 107; ∴ required per-
157. The person from Pune celebrates his 3000
156. Kishan lives in either Pune or Indore.
birthday in February. Choice (2)(4)
Choice
per cent Choice (3)
Word: P R I N T O U T 76 − 51
158.  Bibhas—Lucknow—March
157. The person from Pune celebrates is the hiscorrect
‘PRINTOUT’ is coded as ‘VXOTZUAZ’.
birthday in 163. From the previous solution maximum num- centage ≅ x 100 ≅ 49 per cent
February. Choice (2)
Choice (1) 51
combination. Choice (4) ber of passengers
Solutions that161
for questions cantobe165:
transported by more Choice (4)
158. Bibhas—Lucknow—March is the correct Airbus = 19.5 mn.
159. Allcombination.
the given choices are true.Choice (4)
Choice (4) 161. This question can be best solved by simple
Maximum
inspection.number
We findofthat
passengers
only in thethatyearcan1991 168. The highest exports are to USA and the
159. All
Solutions forthequestion
given choices 160:are true. Choice (4) bethere was a doubling in the number of
transported by Concorde = 3.5 mn. least imports are from Australia. Imports
passengers travelling that is, a 100 per cent
Solutions for question 160: ∴ increase.
Maximum number of passengers that from USA = 18; Exports to Australia = 10
160. 160.
Word: Word: A M B I T I O N can be carried by Airbus and Concorde Choice (3)
18 − 10
in 2000 = 19.5 + 3.5 = 198.5 mn → (A) ∴ Required percentage = x 100
10
Maximum number of passengers that can
= 80 per cent Choice (2)
January Logic:
Logic: + 6 + 6 + 6 + 6 Total + 6 +Passengers
6 + 6 + 6Travelling be carried by Boeing = 180 + 180 + 60 =
dabad January
162. Total Occupancy Rate =
TotalSeats Available 420 mn → (B) 169. Statement (1): As calculated before, the
ai January
January Logic: + 6 + 6 + 6 + 6 + 6 + 6 + 6 + 6
(A) (B) (C) (A) as a percentage of (B) = company’s imports are Rs 1 crore more
/
abad Pune February
January Code:  G   S   H   O   Z   O   U   T
Code: G S H O Z O U T
Indore January January
February
Logic:
Similarly,
Aircraft +6 +
Type
Similarly,
Number
6 + 6 of + 6Number
+ 6 + 6of+Passengers
6 +6 198.5 Total Carrying Capacity than the company’s exports, hence a trade
dabad March January Planes Trips per trip x 100 = 47.26%
= A x B x C (in mn)
ow
Pune February Code:
Code:
A - 3xx G V SX 1000 O T
H O ZZ 100 U A
O U TZ 400 420 4  Choice (2) deficit (not trade surplus). Hence, this
bai
igarh January
March
ndore February
e / Pune March
abad February ACode:
- 340 G S 2500
Similarly, H O Z 100 O U T 300 7.5 statement is false.
w March
/Indore February Code:
A - 330 V X 4000
Code:
Similarly, O T Z 100 U A Z 200 164. Aircraft with maximum 8 carrying capacity Statement (2): The cumulative trade deficit
ai.
garh MarchChoice (4) Logic:
B - 767 – 6 – 6
6000 – 6 – 6 –
1006 – 6 – 6 –
300 6 18
now
bad
March
March Code: V X O T Z U A Z is the Boeing 777−400. is Rs 1 crore and the total imports of the
digarh
Pune or Indore. March B - 777 4500 100 400 18
rabad March Choice(4) (4) BWord:
- 777 –P6 –R6 I– 6 N– 6T– 6O– 6U – 6T – 6
Logic:
Total passengers transported by B 777 − company is Rs 153 crores. 1/153 ≠ 1/15,
ai. Choice 1000 100 600 6
400
Logic:
‘PRINTOUT’
Logic: – 6 –is6 coded – 6 –as 6 ‘VXOTZUAZ’.
–6 –6 –6 –6
400 in 2000 operating at 100 per cent ca- hence this statement is false.
ebai.
unecelebrates
or Indore. hisChoice
birthday(4) in
Concorde
Choice(4)
Choice (2) 100 100 Choice
350 (1) pacity = 60 mn 3.5 → (A) Statement (3): The trade deficit with China
Pune or Indore. BA
Word: - 41 P R I
Solutions for questions 161 to 165: N T O U T
March is his theChoice (4)in
correct ‘PRINTOUT’
Word: P Ris coded
Bombardier I N asT‘VXOTZUAZ’.
O U T
Total passengers travelling in 2003 = 600 is (15 – 12) = 3, which is only 200 per cent
celebrates birthday 6000 200 100 12
ne celebrates his Choice
Choice
(4) in
(2)
birthday
Word:  
161. EJ This
‘PRINTOUT’ P   R  
- 200questionis coded I   as
can N   T   O  
be‘VXOTZUAZ’.
best solvedU   byT simple
Choice (1) mn → (B) more than the cummulative deficit.
Solutions for questions
inspection. We find161 thattoonly
165:in the Choiceyear(1)1991
are
archtrue.is
Choice (4)
theChoice
(2)
correct
‘PRINTOUT’
? Total
Solutions thereforCarrying
was isa coded
questions Capacity
doublingas 165:
161 toof
‘VXOTZUAZ’.
all Planes
in the number in 2003 of (A) as a percentage of (B) in 2003 is
= 77 Mn passengers Statement (4): The difference between the
March is Choice (4)
the correct  This
161. questiontravelling
passengers can be that best60is,
Mn aChoice
solved 100 byper (1)cent
simple
60 highest exports and the lowest imports
0: 161. Total
?inspection.
This Occupancy
question
increase. We can findRate
be =onlysolved
thatbest in thex 100
year
by = 78 per cent
1991
simple = x100 = 10%
Choice (4) 77 Mn
re true. Choice (4) there was
inspection. We a doubling
find that in
only inthe
the number
Choice (3)of
year 1991 600  Choice (2) = 22 – 8 = Rs 14 crores.
T I O N Solutions for questions
passengers
there was
Choice (3) travelling
161
a doubling to 165:
thatinis,the a 100numberper centof
0:
are true. Choice (4) Average exports to Brazil and Germany
passengers travelling that is, a 100 per cent
increase.
60:
T I O N
161.  This
163. From question
the previous
increase. cansolution
be best maximum solved number
Choice by
(3) of 165.  Number
164. of passengers
Aircraft with maximum carryingexpected to be
capacity is the 14 + 14
T I O N simplepassengers that can
inspection. Webefind transported
thatChoice
onlyby (3)Airbus
in Boeing 777400.
flying in 2001 = 90 mn (50 per cent in- = Rs 14 crores
e=
Total Passengers Travelling = 19.5 mn. Total passengers transported by B 777  400 in = 2
TotalSeats Available theMaximum
year 1991 there was
number a doublingthat
of passengers in can
the be crease)
2000 operating at 100 per cent capacity = 60 As both the figures are equal, hence only
Total Passengers Travelling number transported of passengers
by Concordetravelling = 3.5 mn. that is, a Total
mn o available
(A) capacity in 2000 of all
(B) Passengers
= Total (C) Travelling ? Maximum
100 per Total number
cent Carrying
increase. of passengers that
Choice (3) can be Total passengers travelling in 2003 = 600 mn o statement (4) is definitely trueChoice (4)
= TotalSeats
fteNumber Available
of Passengers carried by Airbus and Capacity
Concorde in 2000 aircraft put together = 77 Mn; ∴ Deficit
TotalSeats Available (B)
s
(B)
Trips per trip
(C) 162. Total = 19.5 = A x B
+ 3.5 =Rate
Occupancy x C (in
198.5 mn)
= mn o (A) Maximum capacity
(A) as = a13 percentage
Mn of (B) in 2003 is 170. Total imports from Brazil, Japan, S.A ,
100
(B) 400
(C) number of passengers 4 that can be carried by
Number100 ofofPassengers
300 Total Carrying 7.5 Capacity This 60 deficit is to be filled with the aircraft Russia and China = 13 + 16 + 13 + 21 +
of Number
Trips
Passengers
per trip Total Passengers
Boeing Total
= Carrying
180
=AxBxC + 180Travelling
Capacity
+ 60
(in mn) = 420 mn o (B) = x100 10% Choice (2)
es 100 Trips 200
per trip (A) =as
A x B xaC 8 (in mn)
percentage of (B) with600 the least carrying capacity which is 15 = Rs 78 crores
100
100 400
300 TotalSeats Available 4
418
100
100 400
300 198.5 7.5 the Bombardier EJ-200.
165. Number of passengers expected to be flying in Total exports to the other five countries
100
100 400
300 = x 100 7.5 1847.26% Choice (2)
100100 200
200 420 88 Number
2001 = of90 passengers
mn (50 per centthatincrease)
can be carried = 18 + 22 +
100
100 600
300 (A) (B) 18 186 (C)
100 300 byTotal
one available capacity
Bombardier in 2000
EJ-200 in of200
all aircraft
= 100 put 10 + 14 +16 = 80
100100 400
400 18
18
Aircraft Num- Num- 3.5 Passen- Total Carry- x 200 = 20000
100
100
350
600
Sol/772 Ratio = 78/80 = 0.975 Choice (1)
100 600 Type ber of ber of66 gers per ing Capacity ∴ number of Bombardier EJ-200 to be
200 100 12
100100 350
350 Planes Trips3.5 3.5 trip =AxBxC purchased Solutions for questions 171 to 173:
city of all Planes in 2003 = 77 Mn passengers (in mn) The data is represented in the Venn Diagram
200200 100
100 12
12 161.5 mn
60 Mn
te = x 100 = 78 perAcent - 3xx 1000 100 400 4 = = 8075  Choice (4) below.
ity of77
acity of Mn
allall Planesinin2003
Planes 2003==77 77Mn
A - Mn
340 passengers
passengers
2500 100 300 7.5 20000
6060
MnMn
eate
== x x100
100==7878per
percent
cent
7777
lution Mn
Mn
maximum number of 164. Aircraft with maximum carrying capacity is the
be transported by Airbus Boeing 777400.
olutionmaximum
maximumnumber
numberofof Total passengers
164. Aircraft
Aircraft with maximumtransported
carryingby B 777 is 400
capacity the in
ution
passengers that can be 164. with
2000 operating maximum
at 100carrying
per centcapacity
capacityis =the60
nbebetransported
transportedbybyAirbus
Airbus Boeing 777400.
de = 3.5 mn. Boeing
mn o 777
(A) 400.
Total passengers
Total passengers transported
transported by
byBB777
777 400
400 inin
of
of passengers that can
passengers that can be
be Total passengers travelling in 2003 = 600
2000 operating at 100 per cent capacity = 60 o mn
passengers that can be 2000
Concorde in 2000
rde = 3.5 mn. 08_Section.indb 9 (B) ooperating
mn (A)
at 100 per cent capacity = 60
10/30/2009 11:52:40 AM
e = 3.5 mn. mn
8.5 mn o (A) Maximum
of passengers that can be (A) a percentage of (B) in 2003 is
(A)o as
10  n  Solution Manual

172. Choice (3) The highest was in 1999. Choice (2)


B(42) C(48)
173. Choice (1) 176. Revenue earned in 2000 = 60 × 28 + 46 ×
6 35 = 3290
Solutions for questions 174 to 178:
Similarly revenues earned in 2001, 2002, 2003,
174. The revenue in 1998 = [24 × 50 + 30 × and 2004 are 3474, 3886, 4221 and 4718.
S (45 − 5)] × 106 = 240 × 107 ∴ It has the highest in 2004.
The revenue in 1999 = [25 × 48 + 32 × (50
The figures are the percentages. Alternate method:
− 2)] × 106 = 273.6 × 107
Percentage of students who watch all three = 6 273.6 − 240 As the production and the sales outside India
The required percentage =
Percentage of students who watch exactly 240 are both highest in 2004 along with the indi-
× 100
two = 24 vidual prices, the revenue is the highest in 2004.
= 14 per cent Choice (2)
Percentage of students who watch exactly one  Choice (4)
= 64 175. Year Required Percentage
177. The trend changed in 1999 and 2002.
∴ Percentage of students who do not watch any  Choice (3)
channel = 6 42 − 7 35
1997 × 100 = × 100 = 43.75
6 per cent is 18 ⇒ 100 per cent is 300 ….. (1) 80 80 178. Except in 1999 and 2000, in all oth-
Let s per cent be the percentage of students who er years, the loss due to storage was
40 more than 10 per cent of the storage.
watch Star News. 1998 × 100 = 42.10
95  Choice (3)
Total percentage of students who watch some
channels = 94 = (42 + 48 + s) – (percentage of 48 Solutions for questions 179 to 181:
1999 × 100 = 48.97
students who watch exactly two channels) – 98
2(percentage of students who watch all three 179. Viewership of DD Sports during second
channels) 46 half of February = 60 x 2/3 = 40°; ∴ Re-
2000 100 = 41.81;
= (90 + s) – (24) – 2(6) ⇒ s = 40 ⇒ 120 students 110 quired ratio = 40° : 45° = 8 : 9Choice (3)
watch 180. Given, 80 per cent x (Actual value of DD
49
Star News ….. (2) 2001 × 100 = 43.36 sports) = 60°
113
If 10 per cent watch only CNN and Star News, ∴ Actual value of DD sports = 75°. The
(24 – 10) per cent watch only BBC and CNN or 55.5 total share is 360 + (75 – 60) = 375 per cent
2002 × 100 = 45.86
only BBC and Star News/ 121 share of Set Max = 150/375 x 100 = 40 per cent
∴Of the 42 per cent who watch BBC (14 + 6)  Choice (3)
per cent watch other channels. 59.5
2003 × 100 = 47.98 181. Given 45° – 90,000
∴22 per cent watch only BBC, that is, 66 stu- 124
dents watch only BBC ……(3) 360°– ?
60
2004 × 100 = 46.15 360 × 90000
171. Choice (2) 130 ?= = 7,20,000 Choice (4)
45

08_Section.indb 10 10/30/2009 11:52:40 AM


Solutions for Language
Comprehension 2
AN OVERVIEW CORRECTION OF 10. order (x)
(order: The way in which people or things
SENTENCES
are arranged)
GRAMMAR AND SENTENCE Practice Exercise 1 (orders: instructions)
CORRECTION The Magistrate has passed orders ……
1. “Advice” is an uncountable noun; so many
advices is (x) 11. Scissor (x)
Practice Exercise 1
Much, a lot of, or plenty of, are used with Names of instruments with two identical
1. The train    2.  He uncountables. parts are always used as plural.
Answer: a lot of advice. Answer: This scissors ………….
3. There     4.  Birds
5. The Prime Minister 2. Many informations (x) 12. Cannon: a large, heavy piece of artillery
information - uncountable noun
(plural form is cannon or cannons)
Practice Exercise 2 Hence a lot of information (or) much in-
formation. Hence the sentence is correct.

1. fly over ---------- 3. a summon (x) 13. alm (x)


2. are -------- Summon is a verb - to call The beggar is begging for alms ()
Summons is a noun - Magisterial orders. ("alms" used as a plural noun).
3. is suffering ------
It is a singular noun
4. is equipped with ---- 14. luggages (x)
The Magistrate has issued summons
5. have been discussing ----- ()……. He has lost all his luggage ……. ().
(luggage is used as a singular noun.)
4. hairs (x)
Practice Exercise 3 Should not be used in plural number. 15. The given sentence is correct.
Answer: My hair has turned grey. (Note: dispose is followed by the preposi-
1. C   2.  P   3.  C
‘Hairs’ should be used only when counted tion ‘of’ and not ‘off’)
4. P   5.  C separately, but not when in a bunch.
16. pain (x)
Practice Exercise 4 5. furnitures (x) “pain” means suffering.
It is used as a singular noun.
“pains” mean care.
1. Adverb clause 2. Adverb clause Hence answer: Our office has purchased
new furniture …….. In the statement; she took great pains …..
3. Adverb clause 4. Adjective clause ()
5. Noun clause 6. Adverb clause 6. troubles - maker (x)
“One of the” takes a plural noun and sin- 17. The given sentence is correct because “in-
7. Adverb clause 8. Adverb clause nings” is used as a singular as well as a
gular verb.
9. Adverb clause 10. Adverb clause ∴Answer: One of the trouble - makers plural noun.
……. 18. Sceneries (x)
Practice Exercise 5 The subject here is ‘one’, which is always Scenery is always used as a singular
singular. The sentence means ‘one among
1. Complex 2. Simple noun. Hence the answer is : These scenes
many’. So trouble-makers.
of Kashmir ….. ()
3. Compound 4. Simple
7. thiefs (x) 19. brother-in-laws (x)
5. Simple 6. Simple
plural of thief is thieves The plural form of brother-in-law is broth-
7. Complex 8. Complex Answer: Four thieves ………
ers-in-law.
9. Simple 10. Complex
8. machineries is wrong usage. Hence my brothers-in-law ……. ()
(machinery: machines collectively)
Practice Exercise 6 Answer: latest machinery. 20. These golds (x)
This gold ……… ()
1. to come 2. to reach, to start 9. many mischiefs (x)
Gold is ‘material’ noun so uncountable.
3. use 4. to see Many acts of mischief () or
A lot of mischief () Hence singular.
5. die, tell

08_Section.indb 11 10/30/2009 11:52:41 AM


12  n  Solution Manual

Practice Exercise 2 “Neither of the” is always followed by a 3. ………. is better than Chennai (x)
singular verb and singular pronoun. The comparison is between the climate of
1. Between you and I (x) Hence answer: Neither of the boys …… two places. Hence it is correct to say: The
Rule : When a pronoun is connected by has ……. his record. climate of Hyderabad is better than that
a conjunction (and, or) with some other 11. availed of (x) of Chennai. ()
word in objective case, it must also be in The verb “avail” always takes a “self” 4. The given sentence is correct. Many a +
the objective case. pronoun and preposition ‘of’. Singular noun + Singular verb is the cor-
Hence answer: “Between you and me Hence she availed herself of ……….. rect structure.
……..”
[Note: “The secretary” is the subject here 12. one another (x) 5. little sugar (x)
and ‘you’ is the ‘object’.] “one another” and “each other” are recipro- Little – Not at all.
cal pronouns “one another” refers to three or a little – some
2. “do your duty …….” (x) more whereas ‘each other’ refers to two per- (‘Little’ is used for uncountable nouns)
one is an indefinite pronoun. The pronoun sons or things. A little sugar ()
following ‘one’ should be one’s (not his/ Hence answer: Those two companies al-
her) ways help each other () 6. Older (x)
Ans: One should do one’s duty ……. Older is used only when we compare peo-
() 13. “one another” (x) ple belonging to different families.
answer: each other; follows the same rule Pooja is my elder sister ()
3. Like you and he (x) mentioned in the previous statement.
Follows the same rule as in question 1 7. much fine (x)
‘you’ is the object. 14. ……… enjoy during holidays (x) very fine ()
Hence Ans: Good students like you and enjoy takes a reflexive (self) pronoun as 'much’ is generally used in a negative
him ….. () the object because it is a transitive verb. sense. (much worse)
Dear students, enjoy yourselves ……. ()
4. You, he and I (x)—wrong ordering of 8. Senior than (x)
words. 15. Committee is used as a plural noun here,
The comparative adjectives which end in
While expressing a negative idea or since each one has a different opinion
‘or’ are always followed by ‘to’.
guilt, hence the pronoun following it also should
My friend is senior to me by 4 years. ()
First person—first be in plural.
The committee were divided in their 9. Preferable than (x)
Second person—second and
……..() Preferable is followed by ‘to’
Third person—third
Ans I, you and he ……… () Answer: Death is preferable to disgrace.
16. The team, ………. its bath (x) “team” is a
5. Only you and him (x) collective noun, but when the reference is 10. Worth-seeing city (x)
‘You’ is in subjective case, as ‘you’ is do- to “bath”, every individual is taken sepa- City worth-seeing ()
ing the action. rately. So, the correct sentence is the team The adjective “worth-seeing” qualifies the
A pronoun in subjective case, when con- ….. their bath () noun “city”, but has to be used after 'city’.
nected with another pronoun by a con- 17. My opinion is the same as your (x) 11. Both sisters are doctors (x)
junction (and, or), the second pronoun My opinion is the same as yours (posses- Both of my friend’s sisters are doctors. ()
also must be in subjective case. sive case of the pronoun)
Hence only you and he ……….. () 12. The two first …… (x)
18. ……….. only you and she (x) Answer: The first two chapters…….. ()
6. If I were him …….. (x) ‘You’ is in objective case.
“If I were ………” states impossible con- Hence the second pronoun also must be in 13. Greater than any student (x)
dition, the pronoun ‘I’ is in subjective objective case (her) Suvarna is to be excluded from the stu-
case hence it should be followed by the Refer to the rule stated in question (1). dents with whom she is being compared.
subjective case of the pronoun. Ans: We were told to let …… you and Hence the correct sentence is Suvarna is
If I were he …….() her enter. greater than any other student. ()
7. Everyone announced one’s …….. (x) 19. which (x) 14. better than any drama (x)
Everyone—indefinite pronoun ‘who’ is the right word to be used, be- The rule followed is the same as in the
“Everyone” is followed by the pronoun cause here the reference is to a person. previous sentence.
“his”. Further, if Shakuntala is the name of a
20. His problems are the same as my (x)
Everyone announced his ………. drama or a woman is not clear, in which
His problems the same as mine ()
8. She helped everyone of those boys in doing (possessive case) case, it becomes wrong comparison.
their work (x) 15. is further from (x)
“everyone of the boys” should be fol- further is used to mean “additional”.
lowed by the pronoun “his” Practice Exercise 3
farther is used to express distance.
Hence answer: …….. in doing his work. Hence the right sentence is, His house is
1. more wiser (x)
9. Every teacher and every student …….. their usage of dual comparatives is wrong. farther…..()
duty (x) Answer: I am wiser than…….. () 16. Later suggestion (x)
[teacher and student are common genders].
2. ……… larger than you (x) Later refers to time, while latter refers to
Hence answer: Every teacher and every
The comparison should be between two position.
student …….. his or her duty.
similar things Hence later suggestion is erroneous in
10. Neither of the boys ….. have ………. Answer: My house is larger than yours () this context.
their records. (Yours → Your house) The latter suggestion ……..()

08_Section.indb 12 10/30/2009 11:52:41 AM


Solutions for Language Comprehension  n  13

17. Smell sweetly (x) The answer is: No sooner did I…. than 8. Bread and milk are (x)
Smell sweet () the train left. () “Bread and milk” is treated as singular
noun as they reflect a single idea, hence
18. Little learning (x) 6. Completely finish your homework before
takes a singular verb.
A little learning () lunch. (x)
Bread and milk is ……….()
(Rule is the same as the rule mentioned …… finish your homework completely be-
in Q.5) fore lunch () 9. Sanjay as well as his brothers have gone
Adverb should follow verb and when ob- home (x)
19. the most happiest ……. (x)
ject is present, adverb should follow that. Rule: When the subject of the sentence
Dual superlatives should not be used.
consists of two nouns joined with “as
The right sentence is ……… “the happi- 7. He always is ….. (x)
well as”, then the verb agrees with the
est” He is always ………()
first subject.
Rule: The adverb should be placed after
20. The rich people hate poor (x) Sanjay as well as his brothers has gone
the 'be’ verb in the sentence.
the rich hate the poor. ……()
the rich indicates → rich people 8. The given sentence is correct.
10. ……… with his followers have escaped.
the poor indicates → poor people
Rule: When the subject of the sentence
21. Concerned clerk (x) Practice Exercise 4
consists of two nouns joined with “with”,
The usage is wrong. It conveys the mean- the verb agrees with the first subject.
1. Measles have (x)
ing worried clerk. ……… with his followers has escaped.
The names of diseases take singular
The appropriate answer clerk con-
verbs.
cerned……… () Practice Exercise 5
Measles has ()
22. dear to me than…….. (x)
2. ‘Arabian Nights’ are (x) 1. Sugar sells in (x)
dearer to me than ()
‘Arabian Nights’ is the title of a book Sugar sells at ()
23. Little act ……… (x) hence a singular verb should be used.
A little act …………() 2. Congratulate upon (x)
‘Arabian Nights’ is ……… ()
(Refer to Q.5 for the rule) Congratulate on ()
3. Five miles are (x)
24. the ferocious of all animals …… (x) 3. time with (x)
When a plural noun denotes some specific
The correct structure is: the most fero- time by ()
quantity taken as a whole, it takes a sin-
cious of all animals () gular verb. 4. travel with a bus (x)
When “of all” or “of any other” are used Five miles is () travel by bus ()
in the sentence, the adjective is in its su-
4. The cost of all essential commodities have 5. Among you and me (x)
perlative.
….. (x) When two persons or things are referred
25. the sharpest of the two (x) Rule: The error of proximity. The verb to, we use between.
Rule: When selection of one of the two must agree with the actual subject of the Between you and me ()
persons or things of the same kind is sentence. 6. among his two sons (x)
meant the comparative degree is preceded ‘Cost’ is the subject in the sentence (sin- between his two sons ()
by “the” and followed by ‘of’. gular)
Hence the verb it takes is also singular. 7. suffering with (x)
Practice Exercise 3 The cost of all essential commodities suffering from ()
has…………() 8. working since five years (x)
1. The given sentence is correct. Since indicates a point of time.
5. The construction of these buildings have
2. “Only wants’ (x) taken ….. (x) For indicates a period of time.
Rule: An adverb must be placed as near as The subject of the sentence is “the con- Working for five years. ()
possible to the word it modifies. struction”. It is singular. Hence it takes a 9. from morning (x)
Here ‘only’ modifies rupees ten thou- singular verb. since morning ()
sand. The construction of the buildings has
10. above eight years (x)
3. Worked hardly (x) taken …….()
for eight years()
“Hardly” conveys a negative meaning. 6. There are a bunch of keys (x)
Worked hardly → did not work at all. bunch of keys – collective noun. Practice Exercise 7
Hence worked hard () Hence it takes a singular verb.
4. hardly entered …….. than (x) ∴ There is a bunch of keys……..() 1. …… raining as he left the house (x)
“Hardly” is always followed by “when” 7. A pious man and good neighbour have …… raining when he left the house ()
……… hardly entered…….. died (x) 2. Your action was either just or fair (x)
when……… Rule: Neither just nor fair ()
5. No sooner did I ……. when the train left When two nouns are joined by ‘and’ and
3. As I was ill, so I did not go to college (x)
(x) they refer to the same person or thing, the
‘So’ should not be used when the sentence
The expression verb is singular.
begins with ‘As’.
“No sooner did” is always followed by A pious man and good neighbour has died
As I was ill, I did not go to college. ()
than. ()

08_Section.indb 13 10/30/2009 11:52:41 AM


14  n  Solution Manual

4. Both of you as well as ……….. (x) 6. The correct sequence is 1, 4, 3, 2. The er- security. You exchange one thing ‘for’
“Both” takes ‘and’; Hence the correct ror is in (3). The conjunction ‘but’ is right another and exchange something ‘with’
sentence is at the beginning of the part in the context of somebody. Hence, ‘for’ is apt in the con-
Both you and your brother are at fault. () the sentence. Hence, the correction is – but text of the sentence. Choice (3)
5. I shall not come …. I am not invited (x) rather an accumulation of …. in (3), since it
17. The right sequence is 1, 3, 2, 4. There is an
“Unless” means – “If not” is preceded by (4) which says – there was no
error in (3). The correction is – vision of
It does not take another negative word. single defining moment. Choice (3)
himself, Vision here would mean ‘view’.
I shall not come unless ….. I am invited 7. The correct sequence is 1, 3, 2, 4. The error is It is followed by ‘of’ and not ‘about’.
() in (3). The correction is – conceptions ‘of’,  Choice (3)
6. Unless you have no objection, I shall see ‘conceptions’ is not followed by ‘about’.
18. The correct sequence is 1, 3, 4,2. There
you tomorrow (x)  Choice (3)
is an error in (3). The correction is child
Unless you have any objection, I shall see 8. The correct sequence is 1, 4, 3,2. 1 is ‘into’ a dangerous monster. To transform
you tomorrow () linked to 4 as 4 mentions the conse- is to change somebody ‘into’ something
Rule: Same as in 5th sentence. quences of ‘more destructive weapons’. It else. Hence, it is ‘into’ and not ‘to’.
7. …… lest you should not fall (x) is followed by 3 as it presents the result  Choice (3)
“lest” means ”in order that ….. not” (oth- of having that perception. It ends with 2.
19. The correct order is 1, 4, 3, 2. There is an
erwise) There is an error in (3). The correction is
error in (4). ‘Vent’ is not followed by ‘out’.
“lest” always takes “should” with it. – as ‘did’ a sense that, … A comparison is
Hence, the correction is – vent their anger.
………lest you should fall. being made and hence it has to be parallel
 Choice (4)
8. He works hard because …… (x) in structure. Choice (3)
20. The right sequence is 1, 4, 3, 2. There is
He works hard so that …… () 9. The right sequence is 1, 3, 4, 2. There is an
an error in 4. Because it is an intellectual
9. I am glad when …….. (x) error in (1). Solidarity, which means unity
discipline based on ……., it ‘should’ meet
I am glad that ………..() or agreement of feeling or action, espe-
…. integrity is the argument. Hence the
cially among individuals with a common
10. So (x) correction is – history must/should meet.
interest, is followed by ‘with’ and not ‘for’.
and ()  Choice (4)
 Choice (1)
21. The right sequence is 1, 4, 3, 2. There is
10. The right sequence is 1, 4, 3, 2. There is
an error in (2). The correction is -question
STYLE an error in (3). The correction is evidence
mark ‘over’ and not ‘about’ guilt.
‘of’ damaging not ‘about’. Choice (3)
 Choice (2)
11. The correct sequence is 1, 3, 2, 4. There
22. The right sequence is 1, 3, 2, 4. There is an
ERROR IDENTIFICATION is an error in (3). The correction is–agree
error in (2). The correction is - but ‘to en-
‘on’ the need to ….. Agree ‘with’ is used
courage’, according to the rule of parallel-
Practice Exercise 1 when two people have the same opin-
ism ‘not just to prevent _ _ _ to encourage’.
ion on something. If people agree ‘on’
Solutions for questions 1 to 35:  Choice (2)
something, they all decide to accept it.
1. The right sequence is 1, 3, 2, 4. The error  Choice (3) 23. The right sequence in 1, 4, 3, 2 there is an
is in (4). The correction is – ‘they belong error in (3) the correction is - rather than,
12. The right sequence is 1, 3, 2, 4. There is an er-
to that race or ethnic group’ because it is a since there is a contrast being presented
ror is (2). Since the sentence clearly refers to
reference to the ‘particular race or group’, - you focus on the individual, rather than
the future, the correction is – superhighway
mentioned in 2.  Choice (4) on the structure of the game. Hence, apart
‘will be’ an Choice (2)
from - which means ‘as well as’ does not
2. The right sequence is 1, 3, 2, 4. The er-
13. The right sequence is 1, 3, 4, 2. There is suit the context. Choice (3)
ror is in (4). The correction is – which
an error in (2). The correction is ‘at differ-
is characteristic of, as it refers to ‘only 24. The right sequence is 1, 4, 3, 2. There is an
one taxonomy’ and not to the organisms. ent stages’. Choice (2)
error in (1). The connection is - threatened
 Choice (4) 14. The right sequence is 1, 4, 3, 2. There is with suspension. You threaten somebody
3. The correct order is 1, 4, 3, 2. The error an error in (4). The correction is – as ‘with’ something. If something or some-
is in (1). Reluctance is followed by to. though it ‘were’ a personal …… right. We one threatens a person or thing, they say or
Hence, the correction is – reluctance to use ‘were’ and not ‘was’, in a hypothetical imply that they will do something unpleas-
accept social arguments. Choice (1) situation, even though the subject singular. ant - threatened ‘by’ is used in this sense.
 Choice (4) Hence, threatened ‘with’ is appropriate in
4. The right sequence is 1, 4, 3, 2. The er-
the context of the sentence. Choice (1)
ror in (3). The correction is – coordinated 15. The right sequence is 1, 3, 4, 2. There is an er-
the dozen democracies. Coordinate is fol- ror in (2). The correction is –assigned ‘itself’. 25. The correct sequence is 1, 3, 2, 4. There is
lowed by ‘with’ not ‘between’.Choice (3) The sentence indicates that America is plan- an error in (3). The correction is - that it
ning to do something – a task. Hence, assign will. Since ‘so’ is already there in part 1,
5. The correct order is 1, 4, 2, 3. The error is
should be followed by the reflexive pronoun its repetition in 3 is erroneous.
in (2). The correction is – by way of. ‘By
‘itself’. Choice (2)  Choice (3)
way of’ means via, by means of, which
is suitable in the context of the sentence. 16. The right order is 1, 3, 2, 4. There is a 26. The right sequence is 1, 4, 3, 2. 3 and 2
‘By the way’ means ‘incidentally’. grammatical error in (3). The correction form a pair ‘person of good sense _ _ _’
 Choice (2) is –– personal integrity ‘for’ economic 1 and 4 form a pair and is followed by 3.

08_Section.indb 14 10/30/2009 11:52:41 AM


Solutions for Language Comprehension  n  15

There is an error in (1). Emphasize is fol- 2. ‘To drop my plate’ means that the action not indicate a period of time. Here ‘for’
lowed by ‘that’ and not ‘on’. The correction was done on purpose. It has to be ‘and should be used to indicate a period of
is - ‘emphasized the importance’ ‘which is dropped my plate _ _ _’ which suggests three years. Choice (2)
connected with ‘that’ is (3). Choice (1) that it happened by mistake. Choice (3)
18. Here the meaning is the black man is
27. The correct order is 1, 3, 4, 2. There is an 3. The use of ‘that’ is incorrect. It was com- somewhat bitter, so the first part of the
error in (4). Since 4 and 2 form a pair - ‘_ pared with ‘those’ in other countries. sentence should read ‘a little bitter’ in-
_ _ _ extracted from them’, indicates that  Choice (3) stead of ‘little bitter’ which has a negative
there has to be a reflexive pronoun in 4 4. ‘Which’ is used when we talk about small connotation. Choice (1)
-contain within ‘themselves’. Choice (4) number of possibilities whereas ‘what’ is
19. ‘Told’ is a transitive verb that needs an
28. The right sequence is 1, 4, 3, 2. 4 is a continu- more general. Choice (1)
object. In the absence of an object we
ation of 1. 4 and 3 are linked – ‘evidence’ and 5. As ‘one’ is the subject, it has to be used must use ‘said’ as it is an intransitive
that are connected. 3 and 2 form a pair, ‘one throughout. The use of ‘he’ in place of verb. ‘--- told us that -----’ Choice (1)
of the features’, it is the result of experimen- ‘one’ in part 4 is incorrect. Choice (4)
tation. There is an error in (3). The correction 20. The error lies in subject-verb agreement.
6. The third part of the sentence should be In this sentence the subject is singular
is ‘on behalf of’ Choice (3)
‘and crystallizes it’. Crystallizes is a verb hence the verb should be ‘recognizes’.
29. The right sequence is 1, 4, 3, 2. 1 is linked which takes an object. Choice (3)  Choice (2)
to 4 as what they have ruled (pronounce
7. Part 2 of the sentence is faulty as the prep-
authoritatively and legally to be the case) is 21. Since the sentence does not speak of a
osition used should be ‘of’ and not ‘in’.
stated in 4. Then follow 3 and 2. There is an comparison, ‘lesser’ which is in the com-
One is the ‘master of the art’ not in the art.
error in (4) the correction - legal right ‘to’ on parative degree should be replaced by
 Choice (2)
abortion. Choice (4) less. Choice (1)
8. The fourth part of the sentence is faulty,
30. The correct order is 1, 3, 4, 2. 4 and 2 form 22. Since the subject Norway’s problems is
as ‘drifting into winter’ is correct - it indi-
a pair ‘whose’ in 2 refers to statistics in 3. plural, in the second part of the sentence
cates motion. ‘In’ does not indicate motion.
1 and 3 form a pair as 3 is a continuation. instead of ‘that’ the plural form ‘those’
 Choice (4)
There is an error in (4). The correction should be used. Choice (2)
is - ‘a’ healthy skepticism, since it refers 9. In the first part of the sentence ‘its’ is in
to ‘skepticism’ in a particular situation. the genitive case, so there need not be an 23. In the second part of the sentence ‘slid’
Hence, the article is required. Choice (4) apostrophe, ‘Because of its hardness’ is should be followed by the preposition ‘by’
correct. Choice (1) as the phrase ‘slid by’ means ‘passed by’.
31. The order is 1, 4, 3, 2. The error lies in  Choice (2)
part 3. The use of ‘so’ is wrong here be- 10. The third part of the sentence should read
cause the sentence begins with ‘having’. ‘devote yourself unswervingly and un- 24. In the last part of the sentence ‘dare explor-
 Choice (3) flinchingly’ to maintain parallelism in con- ers’ is faulty. It should have the adjective
struction. Choice (3) ‘daring’ qualifying the noun ‘explorers’.
32. The order is 1, 3, 2, 4. Part 3 is erroneous.
 Choice (4)
‘Such’ in the main clause should be fol- 11. In an ‘if clauses’, if the conditional clause
lowed by ‘that’ in the subordinate clause contains present tense of the verb the 25. ‘Eminently’ means notably whereas ‘im-
(‘such - - - - that’). ‘Hence’ is incorrect. main clause should have ‘will’ and not minently’ means ‘impending or about to
 Choice (3) would. Hence the third part is incorrect, happen’. Hence it should be read ‘emi-
‘---that will lead to -----’ Choice (3) nently cautious’. Choice (4)
33. The order is 1, 3, 2, 4. ‘India is the land of
- - - -‘ is an incorrect expression. The defi- 12. The phrase ‘at variance with’ means dis- 26. ‘Phenomenon’ is singular, since the sen-
nite article, in this case, should be replaced agreeing or opposing. The first part of the tence talks about many unusual happen-
by ‘a’ because ‘a’ is used to represent one. sentence which says ‘in variance’ with is ings it should be replaced by phenomena,
 Choice (1) incorrect. Choice (1) plural. Choice (2)
34. The order is 1, 3, 4, 2. Part 4 is erroneous. 13. Here ‘regulated’ is in the active voice it 27. The positioning of the adverb ‘reason-
The sentence speaks about an action that will should be ‘have been regulated’ in the ably’ is wrong, it should produce ‘well
be completed before a certain point of time in passive voice. Choice (2) established. It is ‘reasonably well-estab-
the future. Hence, the verb should be in the 14. ‘Integrated by law’ should be followed by lished….’ is correct. Choice (1)
future perfect tense that is, ‘will have com- ‘segregated by practice’ to maintain parallel-
pleted’. Choice (4) 28. ‘Smartly’ should replace ‘smart’ as it is
ism in construction. Choice (3) the appropriate adverb following the verb
35. The order is 1, 4, 3, 2. The error lies in part 15. In the third part of the sentence… ‘ and step. Choice (1)
4. The verb ‘have’ does not agree with the marginalize teaching’ does not make sense.
singular subject ‘one’. The correction is 29. The verbal phrase ‘look at’ means’ con-
It should be ‘marginalize his teaching’.
‘has presented - - - - ‘. Choice (4) sider or examine’ whereas ‘look to’
 Choice (3) means ‘rely on’. In this context ‘look to’
Practice Exercise 2 16. Two ideas are mentioned in the sentence is appropriate. Choice (4)
‘morally right’ and ‘politically right’ so 30. In the ‘Not since…..’ construction the
Solutions for questions 1 to 35: the word used should be ‘between’ and auxiliary verb ‘has’ should follow the
not among. Choice (1)
1. ‘Birds’ is given in plural and the pronoun subject. Hence the sentence should read
to correspond this is ‘their’ and not ‘its’. 17. The error lies in the second part, in the ‘Not since Gandhi has the world seen….’.
 Choice (4) use of the preposition ‘from’ which does  Choice (2)

08_Section.indb 15 10/30/2009 11:52:42 AM


16  n  Solution Manual

31. ‘Fall back’ means ‘withdraw’ or ‘retreat’ be ‘--- go overboard resorting to ---’. must have an apostrophe since it means
whereas ‘fall off’ means, decrease.  Choice (1) the minds of everyone. Choice (2)
Hence ‘… begin to fall’ is suitable here. 10. It is the impact ‘of’ the tobacco industry 10. Sentence B must have ‘a British soldier’
 Choice (4) and not ‘by’. Hence part 4 is incorrect. since we are referring to one person.
32. ‘Concerned’ means ‘anxious’. Here the  Choice (4) Sentence C must have the definite article
authorities are not anxious. Hence con- before the superlative − the finest. In sen-
cerned should follow ‘authorities’ which Practice Exercise 4 tence D the intended word is ‘site’ (noun
would then mean the relevant authorities. meaning an area of ground) not ‘cite’ (verb
 Choice (4) meaning quote as evidence).
Solutions for questions 1 to 20:
33. ‘Reputed’ should be followed by ‘to +  Choice (4)
1. The first sentence makes a statement that
……’ He is reputed to be a great musi- is in the nature of a generalized truth. 11. Sentence A must have ‘environmental’
cian. Here it should be ‘a college of great So it has to be followed by the present (adjective) not ‘environment’ (noun) since
repute. Choice (4) tense. The past in B makes it specific. It it modifies ‘cost’. Choice (1)
34. The expression ‘The Britishers’ is faulty has to be ‘has’ not ‘had’. Similarly C has 12. Sentence A must have cells (plural) since
it should be ‘the British ruled over India’. an “if clause” and so must be followed we are talking of 100 billion. In sentence
 Choice (3) by ‘should’ or ‘would’ in D, but not by D it should be ‘involved in’ not ‘involved
‘are’. The correction is ‘we would be sav- with’. (‘involved in‘ means make someone
35. When there are more than two people
ages----’. Hence B and D are incorrect. take point in something whereas ‘involved
‘each other’ should not be used. It should
 Choice (4) with’ means to have a close personal rela-
be ‘one another’. Choice (4)
2. ‘Finances’ (plural) refers to the money tionship with someone). Choice (2)
Practice Exercise 3 available to a person, an organization or a 13. You ‘discuss something’ not ‘discuss
country. This is the meaning intended in A. about something’. Hence A is incorrect.
Solutions for questions 1 to 10: Similarly B must have ‘assets’ as it refers In sentence D it should be ‘blurted out’
to the property a person owns. Choice (1)
1. The time phrase ‘until the other day’ sug- meaning to say something suddenly with-
gests a past time. Hence, the verb should 3. Sentence A has ‘its’ (possessive) in place out thinking. Choice (3)
also be in the past. It should be ‘that was of ‘it’s’ (it is). In sentence C the verb 14. Sentence C should begin ‘on account of
derisive’ and not ‘that is’….. Choice (3) should be ‘say’ not ‘says’ since the sub- ….’ (meaning because of) not ‘on an ac-
ject is ‘researchers’, which is in the plural
2. As the adverb should be positioned be- count of …’ In sentence D it should be
number. Choice (3)
tween the auxiliary and the main verb ‘… ‘Chilka is home to,’ meaning the large
(‘is’ and ‘courting’), the correction is ‘… 4. Sentence B must have … people looking birds consider it their home.  Choice (4)
is assiduously courting …’ Choice (1) at it …’. Similarly in sentence C it should 15. Sentence A should read ‘at the earliest
be ‘a place like Easter Island’.Choice (2)
3. That clause lacks the correct verb form. It stage‘ – ‘at’ rather than ‘in’ is preferred in
should be ‘that cater to….’ since the noun 5. Sentence B must have ‘ruins’, since it referring to time. In sentence B the stem
that precedes that is plural. (Places and refers to the remains of destruction and cells develop ‘into’ (not for) the different
countries). Choice (3) damage. In sentence C the word should blood cells. In C, they are released ‘into’
be ‘symbol of’, (a thing that represents or (not on) the bloodstream. Finally, in D
4. The adverb hardly should be placed after
stands for something else) not ‘symbol in’. the stem cells can be collected ‘from’ (not
‘are’. Choice (4)
 Choice (4) ‘for’) the bone marrow. Choice (3)
5. It is not ‘chip with’ but ‘chip in with
6. Sentence B should read ‘blessed with’ but 16. Sentence A says flowers have always
something’ which means to join with
not ‘by’ −, ‘blessed with something’ is an played a part. Hence the reference can
something. Choice (4)
idiom which means ‘to have something not be to one religious ceremony so, it
6. A group of scientists is the subject which good’. Sentence D must have ‘flower’ must be ceremonies (plural). In sentence
is singular and hence the use of the plural (singular not plural) since the verb is B, their perfumes have a distinct ‘effect’
verb ‘are conducting’ is incorrect in 4. It singular and the reference is to a single on the mind. The omission of the definite
has to be ‘is conducting’. Choice (4) flower.  Choice (2) article is incorrect. Choice (2)
7. The positioning of ‘each’ and ‘would’ in 7. Sentence D must have ‘to do’, …. He was 17. Sentence B must have ‘a consistent stream
the first part of 4 is incorrect. It conveys supposed ‘to do’ ….. (be supposed to do of cash’. Sentence C must read ‘to have’
the intended meaning only if it is posi- something means be required or expected not ‘having’ since the latter implies that
tioned as it is in the second part of 4. (i.e) to do something). As given the sentence is the newspaper already has what is stated.
it has to be ‘----that would each sell a mil- incomplete. Choice (1)  Choice (1)
lion ------’ the adverb placed between the 18. You can not ‘help’ polio vaccine but you
8. Sentence B should end … fondly imagine
verbs. Choice (4) can ‘help with’ polio vaccines. Hence
him to be. Sentence D should not have
8. Part 3 is erroneous. The use of the gerund ‘from’ − suffer fools gladly. The idiom ‘not sentence A should read ‘I helped with the
‘in triggering’ is absurd. It does not con- suffer fools gladly’ means ‘to have very …..’ Sentence D must also have the modal
vey the meaning intended. The correc- little patience with people that you think are auxiliary ‘would’ − ‘Then I’d check ….’
tion is ‘----which triggered the war ----’. stupid.’ Choice (3) Since the previous two sentences narrate
 Choice (3) in that manner. Choice (4)
9. In sentence B the conjunction should be
9. The use of the preposition ‘about’ after ‘but’ not ‘since’ as two contrasting things 19. Sentence C should read to find the bal-
‘overboard’ in incorrect in 1. It has to are connected. In sentence D ‘everyones’ ance. In sentence D parallelism requires

08_Section.indb 16 10/30/2009 11:52:42 AM


Solutions for Language Comprehension  n  17

that ‘by’ is repeated .. not by sermons but 9. Sentence A must read …. no other drug for the.………’. Sentences A, and C are
by working….. Choice (3) enforcement agents … In the absence of correct sentences. Choice (2)
‘other’ it includes themselves. Sentence
20. Sentence B should read ‘At the core of 17. The present tense (makes) in B makes it
C must have ‘into’ not ‘in’. because the
this Rs 20-crore project ……’ It is es- a generalized statement but what follows
intended idea is to bring him into the open
sential to have ‘this’ or ‘the’ here because in ‘since then …..’ makes it specific. So
(means not hidden). Sentence D should
the reference is to a specific project. Sen- the tense in B should be past (made). Sen-
be … more than a match. (meaning to a
tence D must have ‘across’ not ‘through’. tences A, C and D are correct.Choice (2)
greater degree). Sentence B is a correct
 Choice (4)
sentence. Choice (3) 18. In sentence B it should be ‘reserve of’ not
‘in’, In sentence D it will rise over the
Practice Exercise 5 10. In sentence A it should be ‘reputation for’ coming years. A and C are correct sen-
not ‘of’. In sentence B it should be ‘hit tences. Choice (1)
Solutions for questions 1 to 20: the headlines’ − an idiom which means
‘to be an important item of news in the 19. Statements B, C and D are incorrect. In
1. Sentence A should have ‘boasts of’ or only
newspaper’. Sentence C and D are correct statement B the reference is to ‘a system
‘boasts’ not ‘boasts about’. Sentence C should
sentences. Choice (4) of railways’ hence ‘Indian railways’ is the
have the phrasal verb ‘account for’ (to be the
correction. In statement ‘C’ the pronoun
explanation or cause of something). Sentence 11. Sentence A must have the plural ‘cre- ‘they’ is incorrect. The correction is ‘It’
B and D are correct sentences. Choice (2) ations’ − ‘one of’ is always followed by because the reference is to ‘ the railways’.
the plural noun. In sentence B it should
2. It is the dream of an anthropologist (i.e) The correction in D is ‘superiority over’
be ‘a group of crystals’. Sentences C and
‘anthropologist’s dream’. Hence A is in- but not ‘on’. Hence choice is (1).Only
D are correct.  Choice (3)
correct. Since sentence A is in the past sentence A is correct. Choice (1)
tense, sentence B which talks of some- 12. Sentence A must have ‘use of’ not ‘use 20. Sentence A must have the superlative
thing that happened earlier, should be in for’. If we lose the ‘use for’ something (biggest) not the comparative (bigger).
the past perfect tense − ‘… had been dis- then it is no longer useful to us. (I have Sentence D has a comparative ‘more
covered ….’ not ‘were discovered’. C and no use for school books now) but when aggressive’, so the question arises ‘than
D are grammatically correct. Choice (4) we lose the ‘use of’ something we are what’? So the sentence must read ‘more
no longer able to use it (I have no use of
3. In sentence B ‘a healthy threat’ is wrong; aggressive than before’ or ‘becoming
my right hand after the stroke). Sentence
it is ‘a health threat’ (a threat to health). more aggressive’. B and C are correct
C must be – ‘He is also a music teacher
Sentence C should end … ‘it may be an sentences. Choice (2)
…….’, linking it to his being an expert
addictive’ but not ‘it may addict.’ A and D harmonium player. As given it implies
are correct sentences. Choice (4) that he is a music teacher at the munici-
4. Sentence A must have ‘seen as’ − the pal school in addition to being a music ERROR CORRECTION
phrase means ‘to imagine somebody or teacher somewhere else. This is neither
something as something’. In sentence C stated nor implied. Sentences B and D are Practice Exercise 1
the right conjunction is ‘but’ not ‘and’ correct sentences. Choice (4)
since ‘brilliant’ and ‘obscure’ are quite 13. In the context of what follows (predic- Solutions for questions 1 to 45:
contradictory. B and D are correct sen- tions about the future) sentence A must 1. Option 1 is incorrect because the preposi-
tences. Choice (2) have the simple present (promises) not tion ‘with’ should be replaced by ‘of’. In
5. Sentence A must read ‘nearly fell’ not the past (promised). In sentence C it must option 3 the phrase ‘sterner stuff’ should
‘fell nearly’ because the adverb ‘nearly’ be the plural ‘industries’ not the singular not be preceded by the article ‘a’. In op-
qualifies ‘fell’. Sentence B must be …. ‘industry’ since a number of industries are tion 4 the noun ‘easy’ should be changed
soaked up to my thigh − ‘up to’ means referred to. Sentences B and D are correct to ‘ease’, because ‘a life of ease’ is the
‘as far as’ here. C and D are correct sen- sentences. Choice (4) correct expression.
tences. Choice (3) 14. Sentence C must read ‘workspaces are  Choice (2)

6. In sentence C the punctuation is wrong increasingly being designed …….’ The 2. Options 2, 3 and 4 are incorrect because
− the question mark should be inside the adverb must be placed between the prin- the conjunction ‘since’ and ‘because’ do
quotation mark. A, B and D are correct cipal verb and the auxiliary or between not explain the ‘paradox’, which is being
sentences. Choice (2) two auxiliaries. In sentence D, however, referred to. The ‘paradox’, is best stated
the adverb ‘internationally’ must be at the by the conjunction ‘yet’. Choice (1)
7. Sentence B must have … gazed down at beginning of the sentence since it quali-
the …. Sentence D should read ‘plant and 3. Option 1 is incorrect because the words
fies ‘design’ but not ‘taken’. A and B are
animal life’ not plants. A and C are correct ‘nineteenth century’ should be preceded
correct sentences. Choice (3)
sentences. Choice (2) by the article ‘the’. Besides, the words,
15. Sentence A must be either ‘in myriad ‘world commonwealth’ should be pre-
8. Sentence B must have ‘as exciting as’ not ways’ or ‘in a myriad of ways’. Sentence ceded by the article ‘a’, because in this
‘so exciting as’. Sentence C must have D must have ‘their’ (plural) not ‘its’ (sin- context a particular common- wealth is
the adjective ‘monosyllabic’ not the noun gular) since the reference is to ‘others’. being referred to. Choices 3 and 4 are
‘monosyllable’ since it qualifies response. Sentences B and C are correct sentences. incorrect because the verb ‘is’ does not
Sentence D should be ‘out of the ques-  Choice (3) agree with both ‘cosmopolitanism’ and
tion’ (idiom meaning impossible) not ‘off ‘nationalism’. Choice 4 is incorrect be-
16. In sentence B it should be ‘open to’ not
question’. Choice (3) ‘for’. In sentence D it should be ‘it was cause is should be replaced by ‘are’ and

08_Section.indb 17 10/30/2009 11:52:42 AM


18  n  Solution Manual

the word freedom should not be preceded 12. The infinitive “to win” and “to get” is The sentence is of a general nature and
by ‘the’. Choice (2) appropriate since the sentence speaks the phrase “having acted” rules out (2).
of attributes which helped John Wright Only option (3) is free of these errors.
4. Option 1 is incorrect because here the cor-
to achieve something (that is, to win the  Choice (3)
rect conjunction to be used is so ………
confidence and to get them to listen to
as. Option 2 is incorrect because the word 19. The phrasal verb “go by” something
him). So option (3) is ruled out, the other
literature should not be preceded by the which means “be guided by something”
error is in the positioning of ‘enough’
definite article. Option 3 is incorrect be- or “take it as a basis” is appropriate in the
which should be placed before the infini-
cause the usage of ‘so ……. that is incor- context. Options (1) and (2) are ruled out
tive. Thus option (1) and (4) are ruled out.
rect in this context. Choice (4) as ‘from’ in option (1) and ‘owing to’ in
 Choice (2) option (2) distort the meaning in the sen-
5. Choice 1 is erroneous because the usage
13. The phrasal verb “look for” meaning tence. In option (3) the phrase “is made to
of the article ‘a’ before the words ‘con-
search for something is appropriate in the appear making” is grammatically incor-
stant consternation’ is incorrect. Besides,
context. Hence “looking into” in (4) is rect. Hence only (4) is right. Choice (4)
in choices 1 and 4 the word ‘consterna-
incorrect. Further “to get around” mean-
tion’ should be followed by the preposi- 20. The adverb ‘powerfully’ modifies how the
ing “to avoid” is contextually relevant.
tion ‘to’. Option 2 is incorrect because poet (Shakespeare), expresses the decline
Hence option (3) is the answer choice.
the words ‘many’ and ‘most’ should be of youth and the inevitability of ageing. So,
 Choice (3)
preceded the article ‘the’.  Choice (3) the adjective “powerful” rules out options
14. The verb ‘is’ ‘whispered’ is incorrect in (1) and (3). The phrasal verb “muse over”
6. Choice 2 cannot be the right answer be-
option (3). The post of “special envoy” is meaning think carefully about something
cause the words ‘electorate’, and ‘opinion
a front with a purpose i.e to hide the se- is appropriate in the context and so options
polls’ should be preceded by the article
cret activities. Hence the preposition ‘for’ (1), (2) and (3) are ruled out. Choice (4)
‘the’. Option 3 is erroneous because the
is correct. The preposition ‘on’ rules out
verb ‘are’ does not agree with the word 21. Options (2) and (4) are ruled out as “in-
option (4). The relative pronoun ‘which’
‘electorate’ which is taken as singular. justice” should be in the plural as we have
being the subject of the clause requires a
Option 4 is incorrect because the adverb ‘reports’. The second error is in the phrase
verb. Hence, in option 2, the verb ‘is’ is
mostly is incorrectly used in place of “opportunities to basic education”. The
in the appropriate place and so renders it
‘most’  Choice (1) correct form would be “opportunities for
incorrect. Choice (1) something”. This rules out options (2) and
7. Political parties can be replaced by the
15. The past tense of ‘lie’ being ‘lay’ rules (3). The specific article ‘the’ is needed be-
pronoun ‘which’ not ‘who’. Hence choice
out (1). The word “oppressive” refers to fore “exploitation of children”.Choice (1)
4 is ruled out. Choice 3 is incorrect be-
conditions which oppress people and so
cause it should be ‘the’ sun. Choice 1 is 22. We ‘portray’ someone ‘as’ someone else;
the adjective ‘oppressed’ which refers
incorrect because we are ‘weary’ ‘of’ not ‘with’ renders options (1) and (4) incor-
to people render options (1) and (2) in-
‘about’ something. Choice (2) rect. The adverb ‘more’ should be placed
correct. Since the sentence is in the past near the verb “matters” and so option (3)
8. The positioning of ‘too’ is correct only in tense, ‘has’ or ‘have’ is incorrect and so is ruled out. ‘roll up’ meaning fold some-
choice 4. We do not say ‘a’ too complex, only option (3) is appropriate. Choice (3) thing is correct. The right answer choice
hence choices 1 and 3 are ruled out. The
16. Concerns can “grow” or “sharpen” but is option (2). Choice (2)
structure here is ‘too……to’ which is not
not ‘grown’ or ‘sharpened’. So option
followed in choice (2). Choice (4) 23. The verb ‘rising above’ is correct because
(1) and (2) are eliminated. The sentence it means that a building is tall. “Arising”
9. The sentence is a generalized statement clearly implies that the concerns are al- is inappropriate because it refers to some-
as indicated by the latter part of the sen- ready existing and so the conjunction ‘if’ thing that begins to exist. ‘Raising’ would
tence. Therefore ‘has been’ in options (2) is incorrect. ‘As’ gives the reason why refer to someone else doing it. Hence op-
and (3) renders them incorrect. Option (1) “the concerns have grown and will further tions (1) and (3) are ruled out. ‘Unless’
does not convey proper sense, hence is sharpen”. Thus the conjunction ‘if’ in op- expresses condition and ‘until’ time. The
ruled out. Choice (4) tion (4) rules it out.  Choice (3) former is right here. Choice (4)
10. The verb ‘weathered’ means to come 17. The ‘be’ form of the verb renders the ad- 24. When something is “the modern face”
safely through a difficult period or ex- jective ‘enough’ meaningful. ‘Does’ is of something we mean to say that it is
perience. Hence, “has been weathered” meaningless. Option (1) is ruled out. The in contrast to that in a particular aspect.
being in the passive is in apt. Thus (1) adverb ‘often’ follows the verb ‘is’ Option Hence, the preposition ‘to’ is incorrect.
is ruled out as incorrect; the words “this (2) is also ruled out. “Keeping the goal” Between options (3) and (4), option (4)
time” shows that “finds” is correct and would refer to a presentation while, in the is eliminated because the adverb ‘soon’
hence options (1) and (2) rules them out. context, the infinitive “to keep” is more should be placed before the verb ‘dis-
Further, we say “many a crisis”, and so appropriate as it suggests a future action. abuses’ hence option (3) is the right an-
option (4) is the correct answer choice. This rules out (4). ‘To move’ again is er- swer choice. Choice (3)
 Choice (4) roneous as the action is in progression.
Hence ‘moving’ makes it grammatically 25. The sentence states that something “is a
11. Both ‘reflect’ and ‘depict’ are followed by
correct and renders it more meaningful. successful technique”. So, the verb ‘is’
an object and hence options (1) and (3)
 Choice (3) here, indicates that it is of a general na-
are ruled out because of the preposition
ture. ‘To tie’ is in the future tense while
‘of’. Option (2) is also ruled out for the 18. Since the subject is ‘two molecules’ the ‘tying’ is in keeping with the general na-
same reason above, Hence (4) is the right pronoun should be “their”. The pronoun ture of the sentence. This rules out options
answer.  Choice (4) “its” in options (1) and (4) is incorrect. (3) and (4). The adjective ‘successive’

08_Section.indb 18 10/30/2009 11:52:42 AM


Solutions for Language Comprehension  n  19

doesn’t go with the phrase “com- 33. The correction is ‘one and a half centu- finitive, choice (1) is wrong. Choice (2)
pletion of a task”. Hence option (2) is ries’. Hence the right answer is choice is wrong because hear him preaching…
ruled out.  Choice (1) (2). Choice (3) is wrong because when distorts the meaning of the sentence.
we have two proper nouns and both of  Choice (3)
26. It is clear from the phrase “conform with EU
them own the same thing, we should use 43. The underlined part has one error, ie., too
regulations” that Sweden joined the EU and
apostrophe and ‘s’ to the second noun.
not vice-verse. The statement gives examples great that. It should be. So…that, ‘across’
 Choice (2)
of what Sweden did since joining the EU. and ‘of’ in choices (2) and (3) make them
Thus, options (3) and (4) are ruled out. The 34. The underlined part has the following er- wrong. Choice (4)
year 1995 is taken as a point of reference, ror. When we say having been + done, we 44. The underlined part has two errors. The
therefore, the preposition “during” is erro- mean a huge time gap. Since the context verb ‘inform’ does not take any preposi-
neous and its correction is “in”. So, option suggests no time gap between turning tion, hence it should be… informed…
(1) is ruled out. The “lifting of bans” is valid off the file encryption and shifting the (not informed to) The ‘concerned authori-
even now; hence ‘has had’ is correct. These systems, we should say just turned off. ties’ would mean the worried authorities.
corrections are seen in option (2).Choice (2)  Choice (2) But in fact, what we mean here is the right
27. Since there are a number of days we refer 35. The underlined part has the following er- authorities (people). To get that meaning
to the duration as a “five day period”. ‘Five ror. The verb instructed implies that the we should say the authorities concerned.
days period’ is erroneous. This rules out NGO taught the locals how to raise chick- Thus choice (3) is the right answer.
options (1) and (3). In option (2), “from” is en, hence it takes the preposition ‘on’.  Choice (3)
always followed by ‘to’. “Hence”, …. and  Choice (4) 45. The underlined part has an error. When
“between” is correct. Hence only option 36. Since the collective noun ‘government’ we use an adjective with the definite ar-
(4) is correct. Choice (4) refers to all people in the government as ticle the, the adjective refers to all the
28. ‘Which’ rules out option 3 and ‘in develop- a group and not to each individual sepa- people of that class, e..g., The rich = all
ment’ rules out option 4. The preposition rately, its equivalent pronoun is ‘its’ and rich people similarly, the downtrodden =
‘for’ is appropriate as the intention is that not ‘their’. Choice (3) is the right answer. all downtrodden people. Therefore, it al-
such kind of people will develop India. We  Choice (3) ways takes a plural verb. Choice (1) is the
say “a greater height” or “greater heights”. 37. When the conjunction as is used in the
right answer. Choice (1)
Hence between (1) and (2), the former is combination of Adjective + as + sub-
grammatically correct. Choice (1) ject + be-form verb, it means though or
Practice Exercise 2
29. The prepositional phrase ‘ahead of’ although. Therefore, the right answer is
Solutions for questions 1 to 45:
means further forward or advanced than choice (4). Choice (4)
– ‘Ahead’ is always followed by ‘of’. Op- 1. The phrase many + a + noun always takes
38. The underlined part has the following er-
tion (1) is ruled out. The adjective ‘big’ a singular verb, hence we should change
ror, ‘to be wrenched’. We should not use
modifies the task and the tone in which it ‘have’ to ‘has’. Thus choice (3) is the an-
‘to + verb’ after 'copes with'. However we
has been used is negative. Hence the word swer. Choice (1) is wrong because the noun
can say…being wrenched. Choice (2)
‘very’ or ‘so’ (in (1), (3) and (4)) should ‘loyalty’ takes either ‘to’ or ‘towards’.
be replaced by ‘too’, which is in tune with 39. Since the verb grow takes to + infinitive or  Choice (3)
what is stated. So, option (4) is ruled out. to + be + part participle (in passive voice)
2. As per rules of grammar, we should not
Since there is no comparison, the adjec- and not being + part participle, choice (1)
use two comparatives together. However,
tive ‘smaller’ is erroneous is erroneous in is the right answer. Choice (3) is wrong be-
we can use one positive and one compara-
(3) should be ‘small’. Choice (2) cause the verb fascinate takes 'by' in pas-
tive. Therefore, choice (1) is the answer.
sive voice and not 'at'. Choice (1)
30. Both the conjunctions ‘as’ and ‘if’ appear  Choice (1)
correct and both make sense in the sen- 40. The underlined part has an error in tense.
3. The idiomatic phrase ‘cannot be bothered
tence and therefore “growth rate of 10 per Since the context suggests that after the
to do something’ means that you do not
cent” and “growth rate to 10 per cent con- first action (that is, Turning’s publishing)
want to spend your time or, energy doing
sequently also respectively appear cor- had happened, the second action (that is,
it. The correction is …could not be both-
rect. It’s only the correlative conjunctions Cullough suggesting) happened, the first
ered to nail on. Choice (1)
“not only … but also” in the sentence action should be in the past perfect tense,
which decides the appropriate answer i.e, had published. Therefore, choice (4) is 4. The underlined part is awkward. What
choice. ‘need to’ precedes ‘not only’ and the answer. Choice (3) is wrong because is said in the part between commas just
so only choice 3 is right. Choice (3) few years mean hardly any years but gives us some additional information
the context suggests after some years. about ‘Eddoes’. The correction is to re-
31. Since the verb ‘singled out’ is not paral- place being that with ‘who’. Choice (2) is
 Choice (4)
lel to ‘highlights’, it should be changed the answer. Choice (3) is wrong because
to ‘singles out’. Choice (1) is therefore 41. Since none in this context means not one, we should not use ‘that’ in Non-defining
the right answer. Choice (4) is wrong be- i.e, no city, it takes a singular verb ‘from’ clauses. Choice (4) distorts the meaning
cause it should be ‘as being’ not ‘as to be’. is wrong in choices (1) and (3), ‘rise’ is of the sentence. Choice (2)
 Choice (1) wrong in choice (4). Choice (2)
5. The correction is….wiped the decision
32. The underlined part has the following er- 42. The comparative adjective… the + com- off…. and not had wiped….because
ror. “Smoother” as the verb does not ex- parative adjective is the standard expres- wiping something off something means
ist. It should be smother (stifle or put out). sion. Hence, choice (3) is the answer. removing something from something.
 Choice (3) Since hear does not take 'to' with its in-

08_Section.indb 19 10/30/2009 11:52:42 AM


20  n  Solution Manual

Choice (4) is the answer. Choice (3) is wrong look upon, it should be placed before the temples predate the Dravidian structures.
because in case of plural nouns ending in preposition ‘as’. Therefore, choice (2) is The best choice 4 makes the point of com-
s, we should not use s after apostrophe. the right answer. Choice (2) parison clear. It also uses the correct adjec-
Since off itself means away from, choices 12. Since the noun ‘world’ is singular, the tive supposed, rather than supposedly (an
(1) and (2) as wrong. Choice (4) verb should be singular, that is, devotes. adverb) to modify the noun phrase Dravid-
ian predecessors. Choice (4)
6. The underlined part has two mistakes. And, ‘devote’ always takes ‘to + ing verb’
One, ‘boldly’ as it is placed suggests that and not ‘to + infinitive’. Thus, choice (3) 22. In this sentence, Ben Jonson the dramatist
the philosopher is ‘bold’. But the sen- is the right answer. Choice (3) can be compared to Shakespeare the dra-
tence actually intends conveying that the 13. Since the verb phrase ‘has been’ is com- matist. His plays can not be. Only choice D
speaker is ‘bold’ (here confident) when he mon, we need not use it in both the places; brings out the correct comparison. Choices
defines the philosopher. Hence, it should we can use it just before the correlative 1, 2 and 3 make illogical comparisons.
be placed after the verb ‘said’. Two, ‘in- conjunction ‘not only’. The conjunction  Choice (4)
terested’ always takes ‘in’ and not ‘on’. ‘not only . . . but also’ takes the same part 23. Each choice but 3 contains errors of
Therefore, choice (2) is the right answer. of speech. In addition, the preposition ‘in’ agreement. The singular subject each
 Choice (2) is redundant. Thus, the right answer is takes a singular verb has and not have.
7. The underlined part has two mistakes. choice (4). Choice (4) Choice 4 is very awkwardly constructed
One, the time phrase ‘for the past. . .’ 14. Since we are referring to all debt collectors, and convey a very absurd idea. Choice
indicates a situation that has continued we should use the plural verb, that is, ‘are’. 2 is wordy and contains the unnecessary
over a period of time and still continues. Since we mean ‘delayed’ and not ‘recently’, ‘each of’. Choice 3 maintains subject–
Hence, we should present it in the present we should use ‘late’ and not ‘lately’. Thus verb agreement (sons have made) and
perfect tense; we should use been, that is, the right answer is choice (1). Choice (1) provides a clear structure. Choice (3)
. . . who has been . . .’ Two, ‘links’ cannot 24. In this sentence, “benefits of exercises
be ‘alleging’; they can only be alleged by 15. Since the noun ‘scientists’ is plural, we
should use the plural verb, that is, ‘claim’. such as………and aerobics” should be
someone. Hence, ‘alleged links’ is cor- followed by a plural verb ‘are’ and not
rect. Hence choice (3) is the right answer. Choice (2) is the right answer. Choice (3)
is wrong because it suggests that the find- ‘is’ (that are strenuous). Similarly, in the
Choice (4) is wrong as we cannot use the second part of the sentence, underesti-
relative pronoun ‘that’ in non-defining ing is ,not over. But the tone of the sen-
tence (hunting . . . ) suggests otherwise. mate that is incorrect. The reference is to
clauses. Choice (3) the plural “benefits of exercises” and the
 Choice (2)
8. The non-finite verb ‘having stoked’ sug- pronoun to be used is ‘those’ not that. In
16. As ‘the majority’ suggests ‘many’, the choices 3 and 4 the use of when nonsensi-
gests two ideas which the sentence does
verb should be plural, that is, are. Hence, cally suggests that benefits of strenuous
not convey. One, it implies that the cost
choice (4) is the right answer. Since ‘at- exercises are underestimated when they
encouraged the fuel prices. Two, it shows
tributable’ take ‘to’ and not ‘for’, choice are derived from lighter exercises Choice
that encouraging the prices happened
(2) is wrong. Choice (4) 2 uses the correct pronoun those. Besides,
long back and the cost’s sparking a vio-
lent protests has happened now, that is 17. Since the context suggests that in the ‘strenuous exercises’ is more concise and
there is a huge time gap between the two. presence of the circumstances, a skill solves the Subject Verb agreement prob-
Besides, it is not the cost of one food item comes into existence, we should use ‘in’ lem. The phase ‘strenuous exercises’ is
but many that has instigated violence. before ‘which’. So, choice (3) is the right also parallel to lighter exercises. Choice
Hence, food staple should be plural. And answer. Choice (2) is wrong because we (2)
the non-finite clause should have only should use a singular verb with a ‘skill’. 25. The use of the pronoun ‘it’ at the end of
the past participle, that is, ‘stoked’. Thus, Choice (4) is also wrong as ‘many a skill’ the sentence gives room for confusion
choice (1) is the right answer. Choice (1) takes a singular verb. Choice (3) and ambiguity. We are not sure whether
9. When two nouns are connected by either 18. As ‘restrict’ is a verb that requires a re- the book was on his study or on magnets
‘either . . . or’ or ‘neither . . . nor’, the flexive pronoun, we should use ‘itself’. or on their properties. So, Choices 1 and
verb agrees with the second noun in num- And as it takes ‘to + ing verb’, ‘ to hit- 2 are eliminated. The inappropriate po-
ber. Since the second noun ‘officials’ is ting’ is correct. So, choice (4) is the right sitioning of the adverb first in Choice 3
plural, the verb should also be plural, that answer. Choice (4) eliminates this option also. It suggests
is, ‘have’. Choice (4), thus, is the right an- that he made the study first in 1600 and
19. When we define a noun with ‘some of’,
swer. Choice (4) followed it up with another study later.
we should use the objective relative pro-
Choice 4 is the best choice because it
10. Since the three verbs fund, conduct and nouns. Hence, the correction is ‘some
uses the term ‘magnetism’ and not ‘it ‘ or
write are parallel, the third verb ‘writ- of whom’. Choice (2), therefore, is the an-
‘they’. ‘A detailed study’ is preferrable to
ing’ should also be in the past participle swer. Choice (2)
did a detailed study. Choice (4)
form, that is, ‘written’. Moreover, we al- 20. The context suggests something that will
26. The original sentence contains no error
ways ‘raise’ an issue (mention or bring to not happen since we express this idea
and so Choice 1 is the best. It uses the
someone’s notice) we do not ‘rise’, ‘rise’ with ‘wish’. We should use the past tense
idiomatically correct expression ‘distin-
cannot take an object. Therefore, choice verbs. As the past equivalent of ‘will’ is
guish between ‘x’ and ‘y’. In choice 2,
(3) is the right answer. Choice (3) ‘would’, choice (1) is the right answer.
the use of from instead of and is incor-
 Choice (1)
11. When we begin a clause with ‘nor’, we rect. In Choices 3 and 4 farmers learn
should use a helping verb immediately 21. The expression as old as indicates equality distinguishing and distinguishing of are
after it. Since ‘primarly’ qualifies the verb of age, but the sentence indicates that the ungrammatical. Choice (1)

08_Section.indb 20 10/30/2009 11:52:43 AM


Solutions for Language Comprehension  n  21

27. The comparison in this sentence is between 43. The use of ‘has’ or ‘had’ is incorrect. 14. Scarcely ……….. when. Correlative con-
a fixed-interest housing loan and a floating ‘Handed over, apologized and explained’ junctions. Hence, the right sentence would
interest housing loan. Choice 1 illogically is correct.  Choice (1) be “when it broke’ as it has to be in simple
compares loan with loan seeker and loan past, corresponding to ‘did’. Choice (3)
44. ‘Many’ indicates number, quantity. So
seekers respectively. In choices 2 and 4, 15. ‘would visit’ indicates an action in the
many memories is the correct usage.
the comparison is again illogical and im- future. As “informed’ is also in past ‘will’
 Choice (3)
precise. Prepositional phrases ‘with float- should be changed to would’. Choice (1)
ing interest loan seeking’ in 2 and ‘floating 45. It is an imaginary situation hence the
interest loan seekers’ in 4 make the choices structure is ‘… as though I were….’ The 16. ‘So’ indicates comparison. Here no com-
incorrect. Choice 3 makes the correct com- given sentence is grammatically correct. parison is intended. The man spent a lot
parison between loans and also provides  Choice (1) because he had a lot of money. This idea
the active verb form ‘does not require’ is best conveyed by the clause ‘had a
which is parallel to requires.  Choice (3) Practice Exercise 3 large amount of money’.  Choice (4)

28. The verb ‘calls’ is in simple present tense. 17. The sentence is in the past tense (knew
Solutions for questions 1 to 30: that ….). Hence, to have uniformity the
Hence ‘approach’ is correct. It is not
1. day dreaming …… and ….. someone verb ‘is’ should also be in the past tense.
followed by ‘to’. Hence 4 is incorrect.
clinging …… ‘ing’ form should be used Also, the comparative form ‘greater than’
 Choice (3)
throughout. Choice (4) should be followed by ‘other’. Hence,
29. Centuries of servility ‘has’ (already) made  Choice (4)
him tame. Choice (3) 2. The correct past tense to be used is
‘ground’. Maintain the past tense through- 18. The verb ‘believed’ is in the past tense.
30. An act has been stated simply. Hence con- out. Heart broken and ignored, she ground Hence ‘is’ should also take the past tense
tinuous tense should not be used. Verbs of her way…. Choice (2) form ‘was’. Choice (4)
possession are not used in the progressive
3. The word ‘most’ in the given sentence 19. Whenever we have ‘neither …..nor’ in
tense.  Choice (3)
indicates the superlative degree. Hence, a sentence the verb must agree with the
31. A habitual action should be in the simple the correct form would be the one shown number (singular/plural form) of the sec-
present tense. All the more, ‘leaves’ is in option 4. Vithal is the most meticulous ond subject. Since ‘members’ is plural we
present and hence ‘eats’ should be used. man that I have ever met.  Choice (4) must use ‘have’ and not ‘has’. Choice (1)
 Choice (2) 20. ‘Verb + if’ should be followed by ‘could’
4. The entire sentence talks of an incidence
32. quitting (it) seems to be ……. Here we that took place in the past. Therefore, the (past tense form). Hence, the sentence
should use the Gerund form.  Choice (3) should read - The master tested the new
correct form of the sentence would be –
boy to see if he could read English.
33. ‘cope’ is followed by ‘with’.  Choice (1) Hardly had Vina scolded her daughter,
 Choice (3)
when she started crying.  Choice (2)
34. Christmas is an event and not a person.
21. The original sentence gives the feeling
Therefore, ‘that’ is the right choice and 5. ‘Informed’ indicates past tense. Therefore,
that my marks are being compared with
not ‘who’.  Choice (3) the usage of ‘has’ is incorrect. ‘Had sent’ is
all the boys (and not their marks). Hence,
the right form.  Choice (3)
35. Past continuous tense ‘was just setting’ our choice would be the one where ‘my
is appropriate. The given sentence is cor- 6. The correct expression would be ‘…….. marks’ are compared with the marks of
rect. Choice (3) suggested that we should not talk about the other boys. In option 4 ‘…….. than
…….’  Choice (2) those of ……’ means the marks of the
36. “I can scarcely remember”, “I can hardly
other boys.  Choice (4)
remember” indicate that the person finds 7. In ‘either …. or’ the verb agrees with the
it difficult or impossible to recall. ‘Rarely’ latter. It should be ‘I am’. Choice (4) 22. ‘Has’ indicates present tense. ‘I shall wait
has the tone of ‘occasionally’ which is not …… here’ indicates that we should have
8. The correct idiomatic expression is ‘cut
intended.  Choice (4) future tense throughout the sentence. There-
out for’. Choice (2)
fore, the correct form of the sentence should
37. Out of the choices (3) is the correct choice 9. ‘to’ infinitive should be used here. Hence, be ‘I shall wait here until the bus arrives.’
structurally, because ‘would’ cannot be ‘hard to get work’ is the right. Choice (3)  Choice (2)
followed by ‘wanted’.  Choice (3)
10. Parallelism requires that ‘”rather work” is 23. This poem is ‘one of the many’. Hence
38. To denote a completed action we use the followed by ‘than sit and watch’. Choice the plural of poem (that is) ‘poems’ must
simple past.  Choice (2) (4) be used. Choice (2)
39. ‘as normal as possible’ is intended here. 11. The past tense should be applied uniformly 24. The first part of the sentence, the main
 Choice (3) “……… he hadn’t had ……. “, because clause, is in past tense as ‘called’ is used.
40. The past tense is suggested by ‘talked’. ‘looked’ is part.  Choice (1) Hence what follows in the subordinate
‘Had’ is not required, as all the other clause should also be in past tense. ‘has
12. Here the simple present tense should be used.
verbs ‘crept’, ‘hugged’ are also given in seen’ is present perfect and hence is wrong.
Option 1, “…….. strongly desires …….” is
simple past.  Choice (2) ‘saw’ is the correct usage. Choice (3)
the answer.  Choice (1)
25. The adjective ‘complete’ does not take
41. ‘Became’ suggests past tense. ‘Had’ is not 13. The future tense is best represented by
the comparative degree or the superlative
required.  Choice (4) ‘will be’ and not ‘is’. Also holiday ‘for’
degree. Hence the use of ‘most’ is inap-
42. ‘I always had’ …… past tense. ‘Have’ is is the correct preposition to be used.
propriate. The use of ‘his’ is incorrect in
not needed. Choice (3)  Choice (2)
options 1 and 3. Choice (4)

08_Section.indb 21 10/30/2009 11:52:43 AM


22  n  Solution Manual

26. He has taken ‘leave for two days’. Hence it history hence it should be denoted by the 9. The original sentence is erroneous because
becomes ‘two days’s leave’ where ‘s’ after adjective ‘much’, and not ‘most’. Choice the word ‘life’ should be in the plural be-
the apostrophe need not be used. The use 2 is a repetition of the original. Options cause the word ‘human’ indicates that the
of ‘his’ is incorrect in option (1) and (3). 3 and 4 are incorrect because of the reference is to many people. In option 3
 Choice (1) inappropriate placement of the adverb apart from the word ‘life’ not being in plu-
27. The error is in the usage of the question ‘mostly’. Since the reference here is to a ral, the pronoun ‘his’ does not agree with
tag. A negative sentence takes a positive vast period of history it should be denot- the subject ‘one’, hence option 3 is gram-
tag and vice versa. Also the same auxil- ed by ‘much’. Hence choice 1 is correct. matically inconsistent. In option 4 there is
iary verb (are) that has been used in the  Choice (1) no agreement between the pronoun and
sentence is repeated in the question tag. the subject. Only option 1 is grammatically
4. The original sentence is incorrect because
 Choice (3) correct. Choice (1)
the expression ‘which are among the
28. The appropriate usage is ‘to arrest some- world’s oldest’ is a parenthetical expres- 10. The original sentence is erroneous be-
one on a charge of something’. sion and therefore should be preceded and cause the apostrophe and s after the word
 Choice (2) followed by commas. Options 1 and 3 are chomsky is redundant – the phrases ‘that
incorrect because the word ‘home’ should of’ indicates possession and hence the
29. ‘None of the employers’ is the subject of apostrophe is not required. Option 3 is er-
not be preceded by the article ‘a’. Only
the given sentence, which is plural. Hence roneous due to the superlative adjective
option 4 is correct in terms of grammar
‘have’ should be used and not ‘has’. ‘In- ‘most’. Here a comparison is being made
and usage. Choice (4)
vest in’ is more appropriate that ‘invest hence it should be denoted by the com-
on’ in this context. Choice (4) 5. The original sentence is incorrect because parative adjective. Choice (4)
the verb ‘possesses’, which is in the plural,
30. ‘A lot of money’ is taken as a singular 11. The word argument is either followed by
does not agree with the subject ‘individu-
noun and so ‘is’ should be used in place ‘for’ or ‘against’. Hence the original sen-
als’. Choice 2 is repetition of the original.
of ‘are’. Choice (2) tence is erroneous. Option 3 is erroneous
Options 3 and 4 are incorrect because of
the use of the word ‘that’. Choice (1) because the word ‘argument’ should be in
bEST SENTENCE the plural because it should agree with the
6. The given sentence is erroneous because subject ‘one of the’. Option 4 is errone-
of the pronoun ‘it’ which is unnecessary. ous because ‘sometimes’ should precede
Practice Exercise 1 Option 2 which is a repetition of the origi- ‘imposed’. Choice (1)
nal is therefore erroneous. In option 1 the
Solutions for questions 1 to 40: 12. The original sentence is erroneous be-
expression ‘which is’ and the pronoun ‘it’
cause the word ‘so’ should be followed by
1. The original sentence is erroneous be- is not required. Therefore option 1 does
‘as’ because ‘so _ _ _as’ is the correct con-
cause the action denoted by the verb re- not conform to standard english usage. In
junction. In option 1 the use of ‘such as’
quires the use of the possessive pronoun. option 3 the word ‘not’ which is used after
is incorrect. Option 4 is incorrect because
Hence the correction is ‘_ _ _ will depend the word ‘seldom’ is not required. Hence
of the error in parallelism. The word ‘un-
on our doing something’. Option 2 which 3 can be ruled out. Only choice 4 is most
fulfilled’ should be preceded by the word
is a repetition of the original sentence is appropriate. Choice (4)
‘so’ in keeping with the preceding phrase.
therefore incorrect. Option 1 is incorrect 7. The original sentence is erroneous be- Hence, the correction is ‘so persistent, yet
for the same reason. Further, in option 2 cause of the preposition ‘in’. The word so unfulfilled. Therefore only option 3 is
the pronoun ‘us’ cannot be followed by ‘toll’ should not be followed by ‘in’. The correct in terms of grammar and usage.
the to infinitive. Option 4 is incorrect correct preposition to be used here is ‘on’.  Choice (3)
because of the incorrect tense. Since the In option 1 the preposition ‘to’ is incor-
reference is to the future the verb should 13. The given sentence is erroneous because
rect after the word ‘adopt’ as it changes the word perceived should be followed by
be in the future tense ‘_ _ _survival will the meaning. Option 4 is incorrect be-
depend on _ _’. Choice 3 is grammati- ‘to be on’. Option 1 is erroneous because
cause the sentence should be preceded ‘perceived’ is followed by ‘as being on’.
cally correct. Choice (3) by ‘with’. In the given sentence the first Option 4 is incorrect because the expres-
2. The original sentence given in the ques- part of the sentence talks about the condi- sion ‘the previous decade’ should be pre-
tion is incorrect because the word century tion which results in the second. Hence ceded by ‘that of’ because the comparison
should be in the plural as we are talking the sentence should begin with ‘with’. is between the growth trajectory of the
about two centuries the seventeenth and  Choice (3) present and the previous decade. Hence
the eighteenth. Option 1 is erroneous be- only option 3 is grammatically correct.
8. The given sentence is erroneous because
cause of the preposition ‘in’. Since the  Choice (3)
parallelism is not maintained in the sen-
reference is to an occurrence throughout a
tence. The word ‘lead’ is in the simple 14. The original sentence is erroneous be-
particular period, it should be denoted by
present while the verb ‘adhering’ is in the cause the expression ‘_ _ _seeing things
‘through’ and not ‘in’. Option 3 is incor-
present continuous tense. Option 2 is a in their roundness and wholeness’ should
rect because of the adjective ‘most’ which
repetition of the given sentence. Option be preceded by the preposition ‘of’. As
is generally used with quantity. Since the
3 is erroneous because although both the the reference is to two different powers
reference, here, is to period ‘much’ is the
verbs are in the present continuous tense the power of seeing things as they re-
appropriate adjective and not ‘most’.
the verb ‘are’ is incorrect before the word ally are and the power of seeing things
Only option 4 is right. Choice (4)
‘consistency’. Option 4 is also incorrect in their roundness and wholeness, hence
3. The given sentence is incorrect because because parallelism is not maintained in ‘of’ should be used before both the ex-
the reference is to a period in human the sentence. Choice (1) pressions. Option 1 is erroneous because

08_Section.indb 22 10/30/2009 11:52:43 AM


Solutions for Language Comprehension  n  23

the word failure’ should be in the plural of ‘can’ before ‘or’ also. Only option 4 is ever much’. Since the reference here is to
because the preceding word ‘success’ is correct in terms of grammar and usage. reading something, it should be denoted
in the plural. Further, the reference is to  Choice (4) by ‘no matter how much one has read’.
many successes and failures in a persons Hence among the given options only
20. The given sentence is erroneous because the
life. Option 4 is incorrect because the option 1 is correct in terms of grammar
conjunction ‘not only’ should not be placed
verb ‘see’ is in the simple present in the and usage. Choice (1)
after the verb ‘involve’ as the verb applies to
first part of the sentence and in the present both the noun phrases ‘minimizing the im- 25. The given sentence is erroneous because of
continuous tense (seeing) in the second pact _ _ _’ and ‘enhancing positive feelings_ the preposition ‘to’ which follows the expres-
part. Therefore only option 3 is the right _ _’. Hence ‘not only’ should be placed after sion ‘racial history’ which does not agree
answer. Choice (3) the verb ‘involves’. Option 1 is erroneous be- with the word equates. Option 3 is erroneous
15. The original sentence is erroneous be- cause of the inappropriate positioning of not because of the similar reason (the expression
cause the word ‘hypothesis’ should be in only and ‘but’ should be followed by ‘also’ ‘equate native Africans to blacks’ is errone-
the plural because the word ‘ideas’ is in because ‘not only _ _ _ but also’ is the correct ous), the correction is ‘equate native Ameri-
the plural. Option 4 is erroneous because conjunction. Option 4 is incorrect because cans with blacks’. Option 4 is erroneous be-
the word ‘life’ should be in the plural be- ‘not only’ should be followed by ‘but also’ cause of a similar reason. Choice (1)
cause the reference, here, is to the lives and not ‘so also’. Choice (3) 26. The given sentence is erroneous because of
of human beings in general. Only option 21. The given sentence is incorrect because the inappropriate positioning of the adverb
1 is right. Choice (1) ‘as’ cannot be followed by ‘like’. The cor- ‘hitherto’. The correction is ‘_ _ _hitherto
16. The given sentence is erroneous because rection is ‘as dangerous _ _ _as’. Option unknown stores of knowledge’. Statement
of the preposition ‘among’. In this sen- 3 is erroneous because of the conjunction 2 is a repetition of the original. Option 3 is
tence the comparison is between 2 entities ‘so _ _ as’. The correction is ‘as _ _ _as’. incorrect because the word ‘opened’ should
(that is individuals and groups of individ- Similarly in option 4 the use of ‘as and be followed by ‘up’. The phrase ‘open up’
uals) hence this should be denoted using like’ is incorrect. The comparison is best means to remove restrictions. The sen-
the proposition ‘between’. Option 1 is in- brought out by ‘as _ _ _as’. Only option 1 tence implies that knowledge of English
correct because the verb ‘depend’ should is grammatically consistent. Choice (1) has made certain areas accessible which
agree with the noun ‘functioning’ which were earlier difficult to access. Hence the
22. The given sentence is erroneous be-
is the subject here. Option 2 is a repeti- phrasal verb ‘opened up’. This is denoted
cause the word ‘plants’ cannot be in the
tion of the original sentence. Option 4 is by the phrasal verb ‘opened up’ which is
plural because the word ‘animal’ is also
incorrect because the word ‘functioning’ précise in the context. Option 4 is incorrect
singular. Hence the correction is ‘plant
should be followed by ‘of’. Only option 3 because of the phrasal verb ‘opened out’
and animal species’. Option 3 is errone-
is correct in terms of grammar and usage. which is inappropriate. Therefore only op-
ous because of the preposition ‘between’
 Choice (3) tion 1 is grammatically consistent.
since the reference here is to several spe-
 Choice (1)
17. The given sentence is erroneous because cies of plants and animals it should be
the word control should not be followed denoted by ‘among’ and not ‘between’. 27. The given sentence is erroneous because
by ‘in’. The correction is ‘_ _ _control Option 4 is erroneous because the word the word ‘forbidding’ should be followed
over’. Also ‘mankind’ is singular. Op- edible should not be followed by to, the by ‘from’ and not against. Option 1 is in-
tion 3 is incorrect because the relative correction is edible for (that is fit for hu- correct because the word discriminated
pronoun ‘who’ does not agree with the man consumption). Hence only option 1 should be followed by ‘against’. Option
subject ’being’. Option 4 is incorrect be- is right. Choice (1) 4 is erroneous because of the preposition
cause the word ‘mankind’ is always taken ‘with’ which follows the word ‘villages’.
23. The original sentence is erroneous be-
as singular. Hence, only option 1 is gram- The correction is ‘_ _ _leaving their vil-
cause the verb ‘emphasizes’ does not
matically correct. Choice (1) lages in traditional dress’. Choice (3)
agree with the subject ‘studies’ which is a
18. The original sentence is erroneous be- plural word. Hence the correction is ‘_ _ 28. The original sentence is incorrect because
cause the word ‘price’ should be in the _studies emphasized’. Option 3 is errone- the verb ‘is’ does not agree with the subject
plural because there is a rise in the price ous due to the inappropriate positioning ‘living conditions’ which is a plural verb.
of not only wheat but also other staple of the adverb ‘almost’. The adverb should Choice 1 is erroneous because the word
crops. Option 1 is erroneous because the be placed before the words ‘all studies’. ‘billion’ should be preceded by the article
word ‘pose’ does not agree with the sub- Option 4 is incorrect because the word ‘a’, in order to mean ‘one billion’. Option
ject ‘rise’ which is singular. Option 3 is emphasize cannot be followed by ‘on’. 4 is erroneous because of the inappropriate
not right because the word threat should Only option 1 is correct in terms of gram- placement of the adverb ‘today’. The word
be followed by ‘to’ and not ‘for’. Choice mar and usage. Choice (1) ‘today’ qualifies living conditions hence it
(4) should follow ‘living conditions’, there-
24. The original sentence is grammatically
fore only option 3 is correct. Choice (3)
19. The word ‘associate’ should be followed wrong because the pronoun ‘his’ does not
by the preposition ‘with’. Hence the given agree with the subject ‘one’. Hence his 29. The given sentence is erroneous because
sentence is erroneous. In option 1 the word should be replaced by ‘ones’. Option 3 the expression ‘western culture’ should
dissociate should be followed by ‘from’. is erroneous because of the inappropriate be preceded by the article ‘a’ because
The correction is ‘_ _ _associate themselves positioning of the adverb - ‘beforehand’. the reference here is to a particular cul-
with or dissociate themselves from’. Option The correction is ‘no matter how much ture. Option 1 is erroneous because the
3 is incorrect because of the use of ‘can’ one has read before _ _ _’. Option 4 is verb ‘seem’ does not agree with the sub-
before ‘either’ which necessitates the use erroneous because of the words ‘what- ject. Option 3 presentation is erroneous

08_Section.indb 23 10/30/2009 11:52:43 AM


24  n  Solution Manual

because of the positioning of the word Option 4 is incorrect because of the usage ‘- - - - has had a mixed reaction’. Option 4 is
‘Into’. Only option 4 is correct in terms of of the present continuous in the last part erroneous because the word ‘agreed’ should
grammar and usage. Choice (4) of the sentence which does not make be followed by ‘on or upon’ but not ‘with’.
sense. Only option 3 is grammatically  Choice (1)
30. The original sentence does not make sense
consistent. Choice (3)
because the expression ‘have proved’ is 40. ‘Concern’ is an uncountable noun, there-
ambiguous as it is not clear as to what 35. The original sentence is erroneous be- fore it cannot be used in the plural, hence
the humans have proved. The expression cause the verb ’insists’ does not agree the original sentence is erroneous. Option
‘have proved’ should be followed by ‘to with the subject ‘pessimists’ which is 1 is incorrect because the tense of the
be’ in order to mean that they have proved a plural word. Further, the word ‘cope’ verb should be in the present perfect be-
themselves to be. Option 1 is incorrect cannot be followed by ‘up’. Option 1 is cause the effect of the past action is still
because the word ‘hundred’ should be incorrect because of the same reason. In being felt. Hence the correction is ‘- - - -
preceded by ‘a’ in order to mean one hun- option 3 the word ‘billion’ is given in the has risen’. Option 3 is incorrect because
dred years ago. Option 4 is erroneous. The plural, which makes it erroneous. Option ‘- - - - far from being new’ is an incorrect
word ‘systematic’ should be preceded by 4 is grammatically correct in terms of expression. Choice 4 is grammatically
the adjective ‘more’ in order to maintain grammar and usage. Choice (4) correct. Choice (4)
parallelism in the sentence as the preced- 36. The given sentence is incorrect because
ing adjective is also in the comparative. of the conjunction ‘yet’, the contrast in
 Choice (3) the sentence is best brought out using the VOCABULARY EXERCISES
31. The original sentence is erroneous due conjunction ‘but’, therefore ‘yet’ is erro-
to the inappropriate placement of the neous. Choice 3 is incorrect because the Practice Exercise 1
adverb ‘sufficiently’. The correction is word ‘perform’ does not agree with the
‘- - - - seem to be sufficiently met - - - subject ‘each’. Hence the correction is ‘- 1. 1 2. 2 3. 3 4. 3
‘. Option 1 is erroneous because of the - - each of which performs - - - ‘. Choice 5. 2 6. 4 7. 2 8. 3
inapt positioning of the adverb. Further, 4 is erroneous due to the phrase ‘- - - yet 9. 2 10. 1 11. 3 12. 3
the expression ‘his ideals’ should be pre- by being’. Only choice 1 is grammatically 13. 1 14. 2 15. 2 16. 4
ceded by the preposition to. Option 3 is consistent. Choice (1) 17. 1 18. 3 19. 3 20. 1
incorrect because the word ‘met’ should 37. The original sentence is erroneous be-
be followed by the preposition ‘by’. Only 21. ‘Farrago’ is a confused mixture of differ-
cause the word ‘marker’ cannot be fol-
option 4 is correct in terms of grammar ent state of things. ‘Motley’ collection too
lowed by the preposition ‘to’, it has to
and usage. Choice (4) is a collection of things that are all very
be followed by ‘of / for’. If something different. Hence, the pair AB in option (1)
32. The given sentence is erroneous because is a marker of or for something it means is the correct choice. Caveat, a warning is
the word ‘improvement’ should be fol- that it is a sign that shows the position of unrelated to havoc. Choice (1)
lowed by ‘of’. Option 1 is erroneous be- something. Option 1 is incorrect because
cause the word ‘two third’ should be in the word ‘held’, in this context, should be 22. A ‘genius is a very clever person. A ‘mas-
the plural. Further the word ‘seem’ does followed by ‘up’. The phrasal verb ‘hold termind’ is also a clever (or cunning)
not agree with the subject ‘aim’ which is up’ means to use or present somebody or person, who is responsible for planning
singular. Hence ‘- - - seems to be - - - ‘. something as an example. Option 3 is in- and organizing something. The others are
Option 4 is incorrect because the pronoun correct because the word ‘brutish’ should unrelated. Choice (4)
‘its’ does not agree with the subject ‘man- be followed by ‘ones’ or states in order to 23. ‘Virtue’ and ‘miscalculation’ are unre-
kind’ which, in this context, is not taken make sense. Only option 4 is grammati- lated. ‘Clamour’ is din or a racket. So, is
as a single entity but the reference is to cally consistent. Choice (4) ‘clangour’. Hence A and B are synony-
different individuals of a group. Hence mous. Choice (2)
38. The positioning of the adverb ‘fiercely’
‘- - - - mankind - - - - their’. Choice (3) makes the given sentence erroneous. The 24. A‘crown’is worn on the head. So, is a coronet,
33. The incorrect tense ‘are aware’ in the correction is ‘- - - - fiercely anti-Ameri- A pebble and a jewel are unrelated.
original sentence, makes it erroneous. can’. Option 1 is incorrect because of the  Choice (4)
The correction is ‘have been aware’. The word ‘it’s’. Here, the use of the apostrophe
25. A ‘scoffer’ is someone who thinks some-
reference, here, is to an action which be- is incorrect. Option 4 is incorrect because
thing is ridiculous or inadequate. A
gan in the past and is still continuing (that the use of the pronoun ‘they’ in the latter
‘doubter’ also is a cynic who does not
is, they had been aware in the past and part of the sentence is redundant. The cor-
think something is adequate or true. The
they are still aware) the present perfect rection is ‘- - - - aspiring democrats who if
other two are unrelated. Choice (2)
tense should be used. In option 1 the use allowed to vote freely, would - - -’. Hence
of the past perfect tense is incorrect. The only option 3 is grammatically consistent. 26. A ‘lanky’ person is ‘tall’. One who is bil-
correction is ‘- - - - have been aware - - -‘  Choice (3) ious is bad-tempered. Choice (2)
Option 3 is also incorrect due to the use of 39. The original sentence is incorrect because 27. Something that is foreign is not familiar
simple past tense. Only option 4 is gram- the word ‘agreed’ should be followed by and therefore it is ‘strange’; ‘dire’ mean-
matically consistent. Choice (4) the preposition ‘on or upon’. Option 3 is not ing dreadful and ‘puny’ are not related to
34. The original sentence is erroneous be- correct because the verb should be in the any of the other words. Choice (4)
cause of the preposition ‘into’ after ori- present perfect tense because the reference, 28. A room that is ‘dinky’ is dark, dull and
gin ‘driving to’ not ‘into’. Option 1 is in- here, is to a past action which has a bearing ‘gloomy’. A ‘’dinghy’ is a small, boat.
correct because of the preposition ‘on’. even in the present. Hence the correction is Choice A and B are synonyms.Choice (3)

08_Section.indb 24 10/30/2009 11:52:44 AM


Solutions for Language Comprehension  n  25

29. To ‘collate’ is to collect or ‘assemble’. Practice Exercise 3 31. ‘Ennui’ is boredom. ‘Animation’ is ‘live-
‘Vend’ is to sell. Choice (2) liness’ or something that is full of life and
hence is its antonym. ‘Hauteur’ meaning
30. A ‘firebrand’ is a radical or revolutionary. 1. 3 2. 1 3. 2 4. 1
arrogance and mirthless ness with out
A ‘touchstone’ is a test or criterion for 5. 2 6. 4 7. 1 8. 4
cheer. Choice (3)
genuineness or quality. A benchmark is a 9. 1 10. 3 11. 2 12. 1
standard for quality. Choice (1) 13. 3 14. 1 15. 3 16. 4 32. ‘Fruition’ is fulfillment and is the end re-
17. 3 18. 1 19. 2 20. 4 sult of something. It’s opposite is some-
31. Something, that is dank is ‘damp’ or wet. thing that is still at the beginning stage.
‘Lank’ mean limp or lifeless. A person 21. A ‘polymath’ is a scholar and an ‘ignora- Hence ‘inception’ is its antonym. Both
who is ‘forlorn’ is sad. Choice (2) mus’ is a daft or a stupid person. B and D ‘serendipity’, which is a chance and ‘de-
are antonyms. Choice (2) tachment’ are unrelated. Choice (1)
32. ‘Trash’ and ‘chaff’ are junks. So, A and
D are synonymous. ‘Twist’ and ‘style’ are 22. To ‘harrow’ is to torture, that is subject to 33. ‘Frumpy’ is dull and old fashioned and is
unrelated. Choice (3) physical or mental pain or make one suf- opposite to modern. Thus option (D) is its
fer. Its opposite is to ‘console’ or calm the antonym. A ‘chivalrous’ person is a well
33. ‘Feral’ is undomesticated so is ‘unbro- mannered person and to be ‘farcical’ is to
person. B and C are antonyms. To ‘extol’
ken’. Choice (1) be hilarious. Choice (2)
is to praise someone and ‘allude’ is to re-
34. ‘Philippic’ is a ‘tirade’ or verbal on- fer to something covertly. Choice (4) 34. A loafer and a desperado who is a bandit
slaught. So, A and C are synonymous. 23. Something that is ‘disparate’ is heterog- are unrelated. A ‘demagogue’ is one who
The others are unrelated. Choice (1) enous or essentially different in kind. is a troublemaker. Hence its opposite is
Hence, its opposite is ‘cognate’ or some- troubleshooter one who resolves disputes.
35. Something that is ‘germane’ is ‘relevant’.
thing that is related. So B and C are ant-  Choice (2)
One who is compliant is one who is will-
ing to do what he / she has been asked to. onyms. Something that is ‘quotidian’ is 35. ‘Elan’ is style. Lack of style or clumsiness
Something that is ‘galling’ is annoying. something that is very ordinary. Choice is its opposite. ‘Dogmatism’ is opinion-
 Choice (2) (3) ated ness  Choice (2)
24. Something that is ‘incorporeal’ is not 36. ‘Avalanche’ is a flood. Hence its opposite
36. Sorority is a social organization for wom-
composed of matter, and hence cannot is trickle. Anticlimax and a faction are un-
en. An ‘association’. Choices B and C are
be touched. Therefore, ‘palpable’ which related. Choice (4)
similar. The others are unrelated. Choice
(4) means something that can be touched or 37. Compact is dense or brief and is opposite
felt is its antonym. Choice (4) to ‘capacious’ or roomy. ’Vast’ is being
37. ‘Hallowed’ is something holy and ‘sa- large and is unrelated to the above two
25. ‘Aboriginal’ is native or indigenous.
cred’. Thus ‘sacred’ is close to ‘hallowed’. words as is tranquil or peace. Choice (1)
Hence, its opposite would be something
The other two are unrelated. Choice (3)
that is foreign or ‘imported’. When some- 38. Traditional. is opposite to innovative. To
38. ‘Pett’ is to throw and so is ‘lob’. Choice one, or something is ‘exalted’, it is famous be ‘raddled’ is to be haggard. Choice (1)
(3) or important. Something that is prescient
39. To ‘quash’ something is to annul or can-
is one that is predictable. Thus A – D is
39. ‘Treachery’ is a specific crime and ‘per- cel it. Hence, its opposite is to validate or
the required pair. Choice (1)
fidy’ is also a specific crime. Crime is a legalize. To ‘chivvy’ is to nag and to ‘im-
much broader concept and too general to 26. One who is ‘natty’ is smart or well pale’ is to cut. Choice (4)
be close to either of them. Hence, choices groomed. Therefore, one who is unkempt 40. ‘Obloquy’ is condemnatory While ‘beni-
A and B are closer to each other. Parox- is its opposite. One who is ‘diabolical’ is son’ is benediction or blessing. Felony is
ysm is spasm or convolution and is unre- cruel and one who is ‘intrepid’ is fearless. crime and ‘nonentity’ is non-existence.
lated. Choice (2)  Choice (3)  Choice (3)
40. Something that is ‘noisome’ is unpleasant 27. Something that is ‘pedestrian’ is dull.
Practice Exercise 4
or ‘obnoxious’. The pair A − C is syn- Hence ‘exciting’ in (C) is its antonym.
onymous. Agile and noisy are unrelated. ‘Pallid’ is colourless. Choice (4)
1. 3 2. 4 3. 1 4. 3
 Choice (4) 28. To ‘disburse’ is to pay out and so it does 5. 1 6. 4 7. 2 8. 3
not belong to one. Hence, its opposite is 9. 2 10. 1 11. 3 12. 1
Practice Exercise 2 to ‘claim’ something as one’s own. Hence 13. 2 14. 2 15. 3 16. 1
A – B is the required pair. To ‘impute’ is 17. 2 18. 3 19. 2 20. 1
1. 2 2. 3 3. 1 4. 4 to allege. Choice (1) 21. 1 22. 2 23. 4 24. 4
5. 1 6. 2 7. 3 8. 1 25. 2 26. 2 27. 3 28. 2
29. Morbidity is related to death or what is
9. 2 10. 3 11. 4 12. 2 29. 1 30. 4 31. 1 32. 2
gloomy. Its opposite is cheerfulness.
13. 3 14. 4 15. 1 16. 1 33. 3 34. 3 35. 3 36. 2
‘Dotage’ is feebleness of mind and ‘ca-
37. 1 38. 2 39. 2 40. 3
17. 3 18. 2 19. 4 20. 3 dence’ is intonation or rhythm.Choice (2)
21. 1 22. 2 23. 2 24. 4
30. To ‘efface’ something is to erase or wipe Practice Exercise 5
25. 3 26. 1 27. 2 28. 4 out something. The opposite is ‘embalm’
29. 3 30. 1 31. 1 32. 2 or preserve something. To bestow is to Solutions for questions 1 to 30:
33. 3 34. 4 35. 2 36. 4 give something and to ‘interdict’ is to ban 1. ‘Facilitate’ is to make easy while ‘fe-
37. 3 38. 2 39. 3 40. 1 something. Choice (1) licitate’ is to congratulate. Option (A)

08_Section.indb 25 10/30/2009 11:52:44 AM


26  n  Solution Manual

is appropriate. ‘Addenda’ means to add honouring some one or entertaining lav- 10. ‘Anoint’ means apply oil as a religious
something. ‘Agenda’ is a list of things to ishly. Option (B) is appropriate. ABAB ceremony. Not an appropriate choice. So
be done. Option (B) is appropriate ‘Shot  Choice (3) (A) ‘Override’ means to intervene and
at’ is the act of trying to do or achieve make ineffective. ‘Over run’ is to con-
6. ‘Deprecate’ is to express disapproval. ‘De-
something. (A) is therefore appropriate. quer. Option (A) is correct. ‘Step down’
preciate’ is diminish in value. Option (A)
‘Faucet’ is a tap while ‘facet’ refers to any which conveys the desire to resign is the
is appropriate. The context shows that it
of several sides or aspects of something. appropriate option (B). ‘Raise’ refers to
is urgently needed. ‘Badly’ is appropriate.
In this context option (A) is appropriate. increasing consciousness, hopes, aware-
Option (B) is correct. ‘Principal’ refers to
ABAA. Choice (1) ness etc. ‘Rise’ is to increase gradually.
a person first in rank or importance while
In the context option (B) is appropriate.
2. ‘Faint’ is to lose consciousness. ‘Feint’ ‘principle’ refers to a fundamental truth or
AABB Choice (2)
is to pretend, especially in sport which law. Option (A) is appropriate. ‘Propor-
confuses the opponent. Option (A) is ap- tional means in proportion ‘Equipoise’ 11. ‘Turbid’ is generally used with a liquid
propriate ‘Vise’, refers to a tool where as refers to equilibrium. Proportional is ap- and means ‘cloudy or opaque’ ‘Turgid’
‘vice’ means defect, or fault. Option (B) is propriate. ABAA Choice (3) means tedious. Turgid is appropriate
appropriate, ‘Deceased’ is dead. ‘Diseased’ ‘Entice’ -A means to persuade some one
7. ‘Opinion’ is what we think about a particu-
is seriously ill. Option (A) is appropriate. to do something by offering them some-
lar subject, situation, problem etc. ‘View’
‘Snap’ is to break something suddenly. thing pleasant if they do it. ‘Persuade’ is
is what we think or what we believe about
‘Slash’ is to make a long cut or reduce appropriate in the given context. Option
something, especially an official matter. In
something by a large amount. Option (B) (A) Exacerbate’ meaning to make a bad
this context ‘opinion’ is appropriate. Op-
is appropriate. ABAB. Choice (2) or difficult situation even worse. ‘Exag-
tion (A) is correct. A ‘record’ is informa-
gerate’ is to make it seem larger or greater
3. ‘Investiture’ means a ceremony at which tion that is collected over a period of time
than it actually is. (A) ‘Antisocial’ means
somebody formally receives an official by an official organisation. A ‘dossier’ is a
contrary to social customs. ‘Unsocial’
title or special powers. ‘Investment’ is the set of papers containing a lot of informa-
means socially inconvenient since the
act of investing money. Option (B) is ap- tion, especially secret official information,
sentence speaks about laws, antisocial
propriate. ‘Pray’ refers to speaking to God, about some one or something. In the given
is a more appropriate word (A). AAAA
especially to give thanks or ask for help,. context dossier is appropriate. Option (A)
 Choice (3)
‘prey’ refers to being harmed. Option (B) is correct. ‘ Nourishment’ does not collo-
is appropriate. ‘Dessert’ which refers to cate with ‘light’. ‘Out of range’ is too for 12. Although both the words denote how fast
sweet food is appropriate. ‘knock down’ is away to be hit. ‘Out of reach’ is too for something or some one travels or moves,
to make somebody fall to the ground. To away to pick up. Out of reach is appropri- ‘momentum’ is a technical word meaning
‘knock’ at is to draw attention. Option (B) ate. AABA Choice (4) the force or power contained in a mov-
is appropriate. BBBB Choice (4) ing object, so it is more appropriate. Op-
8. ‘Universal’ is something used by every-
tion (B) is correct. ‘Antique’ refers to old
4. ‘Draught’ refers to a flow of cool air in a one. ‘Unanimous’ is something which ev-
and valuable where as ‘ancient’ is used to
room. ‘Drought’ refers to a long period of ery one agrees with. Unanimous (Option
describe something that has existed for a
time when there is little or no rain. Since B) is appropriate. ‘World wide’ is happen-
long time. ‘Ancient’ is more appropriate.
the sentence speaks about food crisis ing, existing or having an effect in all parts
Option (A) is correct. ‘Successive’ is com-
‘drought’ is appropriate. ‘Speculate means of the world. ‘All over the world’ means
ing or following one after the other. ‘Serial
to form an option about something without every part of the world, used especially to
murders’ would mean killing one after an-
knowing all the details. ‘Evaluate’ means say how much a particular idea, organisa-
other by the same killer. Option (B) is ap-
to form an opinion after thinking about tion etc. has spread. ‘All over the world’
propriate. ‘Exciting’ means arousing great
it carefully. Evaluate is a strong word in (Option B) is appropriate. ‘Compulsive’
interest. A ‘Nail biting’ situation makes
the given context. Option (A) is correct. means difficult to control. Complicated
people extremely nervous and excited,
‘Wary’ means cautious. ‘Weary’ means is difficult to understand. Complicated is
especially because they are waiting for a
very tried. It is more appropriate to use the appropriate choice. A ‘feverish’ activ-
result or decision. Option (A) is correct.
‘wary’ when we talk about the unexpected ity refers to an activity which people are
BABA. Choice (1)
shift the generative PC produces ‘Contem- hurrying to finish. A ‘Frantic’ activity is
plative ‘ means thinking quietly and seri- when people are rushing around in a con- 13. ‘Presumptive’ is to offer grounds for pre-
ously about something. ‘Contemporary’ is fused way, especially because they are sumption. ‘Presumptious’ unduly or over-
belonging to the same period. Option (B) worried that they will not have time to do bearingly confident. Option (A) is correct.
is appropriate. BAAB Choice (3) something or get something. ‘Feverish’ ‘Shimmering’ is to shine with a faint or dif-
option (B) is appropriate. Choice (1) fused light. ‘Simmering’ suggests a state of
5. ‘Set down’ refers to record in writing
suppressed anger or excitement Option (B)
while ‘set in’ means become established. 9. Lassy found it difficult to forget John.
is appropriate ‘Sedulous’ means persever-
Option (A) is appropriate. The correct Option (B) is correct. ‘Affect’ means to
ing. ‘seductive ‘ refers to alluring. In the
preposition in this context is ‘up’, as take produce an effect on something or some
context option (B) is correct. ‘In sanitary’
up means become engaged or interested one, so that they change in some way.
means dirty while ‘insanity’ refers to
in while ‘take on’ will mean undertake ‘Effect’ is to have influence on some one
foolish, irrational. Option (A) is correct.
work. Option (B) is appropriate. ‘luxuri- or something. ‘Effect’ (option B) is ap-
ABBA. Choice (2)
ate’ means’ to relax in comfort. ‘Luxury’ propriate. ‘Hanging is more appropriate
is choice or costly surrounding. In the in the context. ‘Native land’ refers to the 14. ‘Chastise’ means reprimand severely
context ‘luxury’ is appropriate. ‘Fetid’ country where one is born. This is the ap- ‘Chasten’ refers to being subdued or re-
means stinking while ‘feted’ refers to propriate choice. BBBA. Choice (2) strained. Option (B) is correct. ‘Hubris’

08_Section.indb 26 10/30/2009 11:52:44 AM


Solutions for Language Comprehension  n  27

refers to being arrogant. ‘Debris’ refers to text. Option (B) is correct ‘edgy’ which mind, mad. Option (B) is correct. ‘Prac-
scattered fragments, especially of some- means irritable’ is appropriate (B). ‘Cha- ticable’ is something that can be done.
thing wrecked or distorted. In the given rade’ means an absurd pretence. ‘façade’ ‘Practical’ is concerned with practice
context ‘hubris’ is more appropriate. A mans an outward appearance or front rather than theory. Option (A) is appro-
‘Luxuriant’ is the appropriate adjective as which is a deception one . In the context priate. ‘Venal’ means corrupted ‘Venial’
the reference is to vegetation. Option (A) option (B) is appropriate. ABBB Choice means pardonable B. So AAAB. Choice
is correct. ‘Confirm’ means to provide sup- (2) (2)
port for the truth or correctness. ‘Conform’ 20. ‘Hedge’ means a protection against pos- 26. ‘Inventors’ is appropriate option. Option
refers to complying with rules or general sible loss or diminution. In the context (A) is correct. ‘Humane’ means benevo-
custom. Option (B) is correct. Choice (1) option (B) is appropriate. ‘Responsive’ lent. ‘Human’ is more appropriate. Option
15. ‘Inequities’ is bias ‘Inequality’ refers to means sympathetic. In the context re- (B) is correct. ‘Personnel’ refers to a body
lack of equality in any respect. Option sponsible is more appropriate. (B) ‘Per- of employees. ‘Personal is the right op-
(A) is correct ‘Equable’ is not varying eq- formance’ is a noun so ‘performing’ tion. ‘Delicious’ means tasty. Option (A)
uitable is appropriate. Option (A) is cor- which is an adjective is more appropriate. is correct. ABAA Choice (2)
rect. ‘Ravenous’ refers to being famished. (A) ‘cannon’ refers to a large gun where
27. ‘Teeming’ means full of or swarming
‘Raving’ is used as on intensifier. Sanc- as ‘canon’ means rule or a general law.
with. Option (B) is correct. ‘Allusion’ is
tion is authoritative approval ‘Sanctions’ Option (B) is correct. BBAB Choice (1)
an indirect or passing reference. ‘Illusion’
refers to enforcing obedience to a rule. 21. ‘Embezzle’ means to divert company or is deception. Option (B) is correct. ‘Cope’
Option (B) is appropriate. AABB.Choice public funds fraudulently to one’s own is used without a preposition. Option (B)
(4) use. Option (B) embraced is appropriate. is correct. ‘Suddenly’ refers to something,
16. The sentence speaks about rude behaviour ‘Dramatically’ is unexpectedly. ‘Vulner- unexpected while ‘immediately’ refers
by the clerk so insolent option (A) is ap- able’ suggests that some thing or some to something done without delay (A).
propriate. The clerk was rebuked for his one may be wounded or harmed. Option BBBA. Choice (4)
insolent attitude. ‘Reproach’ is therefore (B) is correct. ‘Spirit’ is more appropriate
appropriate. ‘Rapprochement refers to re- to maintain the subject + verb agreement. 28. ‘Coup’ refers to a successful move.
sumption of harmonious relations. Option Option (A) is more appropriate. ‘Adapt’ ‘coupe’ refers to a car with a hard roof.
B is appropriate. ‘Palaver’ suggests fuss and is to adjust. ‘Adopt’ which means choose Option (A) is correct. ‘Justly’ means ac-
bother especially prolonged. ‘Pallor’ refers to follow is the appropriate choice (B) cording to justice. ‘Justifiable’ something
to paleness. Option (A) is appropriate ‘Go- BBAB. Choice (4) that can be defended or justified. Option
ing an steam’ suggests moving with vigour. (B) is appropriate. ‘Trail’ is a track left
22. ‘Comforting’ means consoling. ‘Com- by a thing or person. ‘Trial’ is a judicial
Option (A) is appropriate. ABAA. Choice fortable’ meaning at case is the appropri-
(4) examination and determination of issues.
ate choice. Option (A) is correct. ‘Art’ is Option (B) is correct. In the context ‘re-
17. ‘Insipid’ is lacking vigour. ‘Intrepid’ is human creative skill. When used in plural sisting’ is logical. Option (B) is correct.
brave. In the given context insipid (option ‘Arts’ it is preceded by ‘the’ and refers to ABBB. Choice (2)
B) is appropriate. The middle class are those branches of learning like languages,
badly affected because of soaring prices. literature etc. In the context art is appro- 29. ‘Routes’ which refers to methods or ways
‘Soar’ is rise high. Option (A) is appro- priate. A ‘Intelligence’ is a noun. So in- is appropriate. Option (B) is correct. ‘Ex-
priate ‘Prevailing’ suggests existing, tellectual (adjective) is more appropriate ude’ means display an emotion freely or
‘pertaining’ which means related is more (B). ‘Revelled’ means to have good time. abundantly. ‘Exult’ is to be greatly joyful.
appropriate. Servitude means ‘slavery’ So ‘revealed’ (show) is more appropriate. ‘Exuded’ is the appropriate option (B)
servility is being like a slave. Both the AABB Choice (2) ‘Back drop’ is the appropriate choice, op-
words have similar meaning. the context tion (B) A ‘facilitator’ is one who makes
23. To maintain the parallel structure of the
of the sentence makes option (B) more thing easy. Option (B) is correct. BBBB
sentence ‘to unite’ is appropriate. Option
appropriate. BABB. Choice (3)  Choice (1)
(B) is correct. ‘Abrogate’ is to abolish a
18. ‘Beautify’ is to make it beautiful. ‘Be- law or a custom. ‘Abominate’ is to defect. 30. ‘Differentiate’ refers to constituting a
atify’ is to formally declare a dead person Option (A) is appropriate. ‘Annex’ means difference. Option (A) is correct. ‘Be-
‘blessed’, a step towards canonisation op- to add as a subordinate part. ‘Annexe’ re- havioural’ is the correct option as we
tion (B) is correct. ‘Reckoned’ is to count fers to a separate or added building. Op- have operational. Option (B) is correct
or compute. Option (A) is correct. ‘Con- tion (B) is appropriate. Choice (2) ‘Neighbourhoods’ is correct A ‘Oblivi-
temptuous ‘means scornful. Contemptible ous’ means unaware so not relevant in the
24. ‘Aural’ is the appropriate choice. ‘Ficti-
is something that deserves contempt. Op- context. ‘obvious’ means easily seen. Op-
tious’ is imaginary. ‘Fictional’ is an in-
tion (A) is appropriate. ‘Hype’ means ex- tion (B) is correct. ABAB. Choice (2)
vented idea. Option (B) is correct. ‘De-
travagant or intensive publicity. Hyperbole fuse’ means reduce tension. The correct
is an exaggerated statement. In the given Practice Exercise 6
option is (A) ‘Averse’ is opposed and
context option (A) is appropriate. BAAA ‘adverse’ means contrary. ‘Averse’ is ap-
 Choice (3) 1. 3 2. 4 3. 2 4. 1
propriate (B). BBAB. Choice (1)
5. 3 6. 2 7. 2 8. 4
19. ‘Inception’ suggests from the beginning.
25. ‘Gambit’ means an opening move. ‘Gam- 9. 1 10. 4 11. 3 12. 3
Option (A) is appropriate ‘Genius’ is the
ut’ refers to range or scope of something. 13. 1 14. 1 15. 3 16. 2
right choice as ‘genus’ which means taxo-
Option (A) is appropriate. ‘Inane’ refers 17. 3 18. 3 19. 3 20. 1
nomic grouping is not relevant to the con-
to senseless ‘Insane’ refers to unsound 21. 3 22. 4 23. 2 24. 3

08_Section.indb 27 10/30/2009 11:52:44 AM


28  n  Solution Manual

25. 2 26. 1 27. 3 28. 4 16. An agency is an organization. A mother is 39. ‘Suave’ means sophisticated, polished,
2 9. 1 30. 4 31. 3 32. 4 a parent. Choice (2) urbane.  Choice (2)
33. 2 34. 3 35. 1 36. 3 17. Fur keeps the animal warm. Sweat- 40. An exacting task is very demanding.
37. 4 38. 2 39. 3 40. 2 er is used by humans to keep warm.  Choice (2)
 Choice (1)
Practice Exercise 2
GENERAL VERBAL ABILITY 18. Proverbially every rose has a thorn as ev-
ery cloud has a silver lining. Choice (3) Solutions for questions 1 to 40:
19. Cane comes from bamboos. Timber 1. ‘Import’ and ‘export’ are antonyms as
Practice Exercise 1 comes from trees.  Choice (2) also are all the choices except ‘inspire’
20. Bells jingle. Anklets tinkle.  Choice (2) (fill with an urge) and ‘expire’ (die).
Solutions for questions 1 to 40:  Choice (3)
1. The given words are ANTONYMS. Op- 21. Cogent and convincing are synonyms as
are laconic and pithy.  Choice (2) 2. A milkmaid delivers milk. A postman
tions 1 and 2 and are antonyms. We, can, delivers letters. A newspaper boy deliv-
eliminate these choices because they don 22. Retrospection is analysis of something ers newspaper while a vendor could de-
not indicate a negative or positive tone related to the past. Prognostication means liver sweets. All these people – milkmaid,
like the key words. ‘Create’ is a positive prediction, which is related to the future. postman and a vendor are mobile people
word while ‘destroy’ is negative. The tone  Choice (3) who deliver an article. The exception is a
is positive in ‘advance’ and ‘retreat’ (go-
23. Eulogise means praise, whereas lambast baker who bakes biscuit but is not known
ing back) is slightly negative. Hence, op-
indicates criticism. Hence, option (3) is for delivering biscuits.  Choice (2)
tion (3) is the best fit.
the answer as this choice also has a pair 3. A pigeon does not stay in a burrow.
 Choice (3)
of antonyms. ‘Invigorate’ means ‘to ener-  Choice (4)
2. The ‘hand’ is part of the ‘body’ Like- gise’ and ‘debilitate’ means to ‘weaken’.
wise, the wheel is part of an automobile.  Choice (3) 4. Lair or den is the place where a wild ani-
 Choice (3) mal (including a lion) stays.  Choice (4)
24. A group of lions is referred to as a ‘pride’,
3. ‘Mercenary’ is a person who fights be- whereas a group of pups (puppies) is 5. ‘Calf’ is the young of a ‘cow’, ‘sheep’ is
cause he wants ‘money’. Likewise, Midas called ‘litter’.  Choice (2) not the young of ‘goat’. Choice (2)
is a mythical character who wanted gold. 6. ‘Galaxy’ is a collection of ‘stars’.
25. Antonyms  Choice (3)
 Choice (4) ‘Row’ is not a collection of ‘soldiers’.
26. A machiavellian person indulges in de-  Choice (3)
4. A polymer has cells. A chain has links.
ceit. A naïve person shows gullibility.
 Choice (3) 7. Hustling indicates jostling, forcing or
 Choice (4)
5. Property can be mortgaged. Money can hurrying. All the other relationships are
27. Antonymous relationship  Choice (2) sound oriented. Choice (4)
be lent. Choice (1)
28. Relationship of degree. Petrify indicates 8. All the pairs are the masculine and
6. A cheque may be negotiable. An asset
extreme fright. Mortify indicates extreme feminine combination except ‘dear’ and
may be frozen. Choice (1)
humiliation.  Choice (4) ‘darling’. Choice (2)
7. A sceptre symbolizes sovereignty. A de-
29. A munificent person is generous (not 9. All the pairs give the person and the thing
gree symbolizes knowledge.  Choice (3)
stingy) Likewise, any argument which he studies except (4). Ichthyology is the
8. The dead body of an animal is called a is articulate is very clear (not obscure). study of fishes not fossils.  Choice (4)
cadaver. The dead body of a man (human  Choice (3)
being) is called corpse.  Choice (1) 10. All the pairs have synonymous meaning
30. Cryptic and enigma are also synonymous. except ‘articulate’ and ‘obfuscate’ which
9. A pistol has a trigger. A motor has a In both the first is an adjective and the are antonymous.  Choice (4)
switch. ‘Trigger’ and ‘switch’ are used to second a noun. Choice (4)
start a ‘pistol’ and a ‘motor’, respectively. 11. All the pairs are masculine and feminine
31. Tenuous is not substantial just as fringe in except ‘lion’ and ‘tiger’. Choice (4)
 Choice (3)
not central. Choice (2)
10. Cabin is a room in a ship. Galley is a 12. ‘Palm’ is connected to ‘fingers’, head to
32. A coward is cravenly. A cheat is duplici- hairs, chain to hook and watch to strap.
kitchen on a ship.  Choice (2)
tous. Choice (4) While computer is connected to technol-
11. Poetry has rhyme. Mystery has suspense. ogy it is not a physical connection or rela-
33. A diffident person is shy. A lacklustre per-
 Choice (2) tionship. Choice (3)
formance is dull.  Choice (1)
12. Computer has RAM, while a book has 13. A whale is in the water, clouds are in the
34. Antonymous relationship.  Choice (2)
pages. Choice (1) sky. Hangar is the place where planes
35. Synonymous relationship.  Choice (2) are kept. Kennel is the place where dogs
13. Chromosome is a part of a cell. Likewise,
oxygen is a constituent of air. Choice (4) 36. Rigor is onerous (burdensome). Tedium are kept.  Choice (1)
is boring. Choice (2) 14. Each pair gives the designation of people
14. Radium was discovered by Madam Cu-
rie. Graham Bell invented the telephone. 37. Synonymous relationship.  Choice (4) who work together in a hierarchy except
 Choice (3) employee and employment. Choice (2)
38. ‘Veracity’ (‘truthfulness’), is opposite of
15. Ocean gives saline water whereas rain ‘chicanery’ (‘trickery’). So also loyalty 15. Philology (not philosophy) is the study of
gives fresh water.  Choice (2) and perfidy.  Choice (3) language. Choice (3)

08_Section.indb 28 10/30/2009 11:52:45 AM


Solutions for Language Comprehension  n  29

16. Bacteria causes leprosy. Evaporation is the 34. The words in each pair are antonymous 5. ‘Discerning’ means having good insight.
cause for cloud formation. Anger could but boisterous (noisy, energetic) is not the The other words carry no relevance to the
lead to violence. The sun is the cause for opposite of ‘boasting’. Choice (1) context. Choice (4)
the occurrence of the day. Choice (1) 35. All the pairs have synonymous words 6. The second blank needs a word which
17. Each pair gives something made from the but ‘consumable’ (intended to be used suggests ‘excess’. ‘Surfeit’ which suggests
preceding word. But ‘rain’ is not made up and then replaced) has nothing to do ‘tiredness or boredom because of exces-
from ‘rainbow’. Choice (3) with ‘portable’ (easily carried) though sive use’ is the appropriate choice. ‘Milieu’
portable may be confused with potable refers to environment. Choice (1)
18. Each pair gives the head of a place.
(drinkable). Choice (3) 7. The words ‘coarse’ ‘grotesque’ and
But Buddha is not the head of Burma.
 Choice (4) 36. Somnolent means sleepy, drowsy; induc- ‘square’ do not match the words ‘modest
ing drowsiness. It has no relation with tone’ so can be ruled out-self deprecating
19. Each pair gives antonyms. But Africa is
sobriety. Somber means dark, gloomy, refers to the ‘act of belittling oneself. The
not the antonym of America. Choice (2)
dismal.  Choice (4) word serves well in bringing out a contrast
20. After a kidnap, ransom is demanded. So, to the meaning vainglorious. Choice (4)
37. The words in each pair are antonymous
we can say “ransom follows a kidnap.”
except ‘pernicious’ (having harmful 8. ‘Juxtaposes’ which means place things
Likewise, after a rain we notice a rain-
effect) and ‘relevant’ (connected or ap- side by side is appropriate in the context.
bow. After, autumn we get winter. After
propriate). Choice (4) ‘Licentiousness’ means immoral, espe-
adolescence comes youth. Choice (3)
cially in sexual relations. Choice (2)
38. Apocryphal means ‘of doubtful origin’.
21. ‘Saturnalia’ is associated with ‘mer- 9. The context is about the candidate not de-
Hence, it is not genuine. Similarly, op-
rymaking’ just as ‘Bacchanalian’ with testing the nature of his job. The words
tions 1, 3 and 4 are antonyms. In option
‘drunkenness’ ‘Enervated’ is ‘weak’ ‘meandering’ nomadic’ and ‘cushy’ are out
(2), we have synonyms.  Choice (2)
and ‘ecstatic’ is ‘joyous’. But ‘altruistic’ of context. ‘Peripatetic’ which means going
and ‘alternating’ have no connection. 39. Each pair has words that are synonymous. from place to place is appropriate for the
 Choice (4) But ‘parry’ (word off) does not mean second blank. Choice (4)
‘fight’. Choice (1)
22. Each pair shows antonyms except ‘in- 10. ‘Scarce’ is the appropriate word for the
undate’ and ‘flood’ which are synonyms. 40. Inveterate means long established or second blank because only then the value
 Choice (1) habitual. Hence, option 2 which is not of the land will increase. Choice (4)
related to orthodox is the answer.
23. A prudent person is judicious. He is not 11. The context of the sentence needs a nega-
 Choice (2)
injudicious (rash).  Choice (4) tive word in the first blank. On this count
24. Each pair shows antonymous relationship Practice Exercise 3 option 4 can be ruled out. The word which
except ‘foppish’ and ‘affected’ which is is used to fill the second blank must con-
synonymous.  Choice (2) Solutions for questions 1 to 50: vey disregard to judiciary. The word ‘im-
punity’ is therefore appropriate.Choice (2)
25. Each pair has a class and a member of that 1. ‘Polemical’ refers to getting involved in
class except choice 4. Choice (4) a dispute or controversy. In the context it 12. ‘Tenet’ which means principle is the ap-
is an appropriate word. The other options propriate word. ‘Accord’ is the right word
26. Each pair has antonyms except ‘intrepid’
do not collocate well. ‘Propriety’ which for the second blank. Choice (4)
and ‘undaunted which are synonyms.
 Choice (1) means rightness fits well in the second 13. ‘Expurgate’ which refers to removing
blank. Choice (4) matter thought to be objectionable from
27. Sacrilege has nothing to do with hones- a book etc is the appropriate word to fill
ty. It means violation of what is sacred. 2. The words ‘whiled away’ in the sentence
indicate that the first blank needs a nega- the first blank. ‘Salacity’ means lustful.
 Choice (3)  Choice (1)
tive word. ‘Feckless’ which means inef-
28. Felicity has no relation with ‘fall’. Felic- fective is appropriate. ‘Meretricious’ 14. The context of the sentence is that he
ity indicates capacity for apt expression; which refers to being ‘falsely attractive’ is a strong advocate of pragmatism. In
great happiness. Choice (4) also fits the second blank appropriately. this context the words ‘deft’ ‘adept’ and
29. Each pair has antonyms but ‘impropriety’  Choice (2) ‘mettlesome’ do not convey that meaning.
has nothing to do with ‘property’.Choice 3. ‘Maudlin’ means tearfully sentimental. ‘Staunch’ which means strong is the ap-
(4) Since the sentence refers to the well- propriate word. ‘Entrenched’ means ‘not
wishers being careful, ‘maudlin’ is an easily modified’. Choice (3)
30. ‘Quixotic’ has nothing to do with ‘quick’.
It means ‘imaginative’ not practical. appropriate choice for the first blank. The 15. The second blank needs a word which will
 Choice (2) other words do not convey the right mean- suggest the meaning of ‘measuring some-
ing. ‘Torrent’ which means rushing water thing’. The word ‘gauged’ which refers to
31. The pairs are synonymous but ‘compas-
also conveys the appropriate meaning ‘standard measure’ is appropriate. ‘Profun-
sionate’ is not ‘lustful’. Choice (1)
as the reference is to overflow of tears. dity’ which refers to ‘depth’ is also appro-
32. While all the paris are synonyms, seques-  Choice (4) priate in the first blank. Choice (3)
trate is to take legal possession not align. 16. The reference is to purity. So ‘Pristine’
4. The meaning conveyed is that – he is an-
 Choice (3) which means ‘fresh as if new’ is the right
gry with his sons for wasting his wealth.
33. ‘Despotic’ has nothing to do with ‘de- The word ‘chagrin’ conveys this idea. word for the first blank. ‘Quagmire’ refers
pot’ while all the other pairs have related  Choice (3) to a soft area that gives way underfoot.
words. Choice (4)  Choice (3)

08_Section.indb 29 10/30/2009 11:52:45 AM


30  n  Solution Manual

17. ‘Cloture’ which means closed is not ap- 27. Energies cannot flow ‘illusorily’. So op- 42. The blank needs a word which means
propriate in the context because political tion 1 is not appropriate. From the given ‘hate’ or dislike. ‘Abhor’ means detest,
parties cannot be closed. ‘Downfall’ is options ‘disowned’ and ‘freely’ fit into the regard with disgust and hatred. Option (2)
appropriate. Choice (4) context appropriately. Choice (4) is appropriate. Choice (2)
18. The fact that Brooks had to face and over 28. ‘Abdication’ is the appropriate choice for 43. The appropriate words are ‘shimmer-
come two handicaps, suggests that the the second blank as it means to give up or ing’ and ‘flopped’. ‘Shimmering’ means
word for the first blank must be something renounce the throne. ‘Off-guard’ blends to shine with a faint or diffused light.
which will demote her strength. ‘Braved’ with the idea of a ‘surprise invasion’.  Choice (4)
which means ‘encounter bravely’ is an  Choice (1) 44. The word ‘advanced’ indicates that ‘at-
appropriate word. Choice (2)
29. ‘Global’ and ‘despotic’ are appropriate titudes’ and ‘infrastructure’ ‘ are inappro-
19. In the given context ‘inevitably’ and words. ‘World wide’ also means ‘global’ priate options. ‘Serendipitous’ means to
‘dominating’ are appropriate words. but the word ‘righteous’ makes option (2) become happy because of an unexpected
 Choice (4) inappropriate. Choice (3) discovery. It has no relevance to the con-
20. The word ‘glueing’ suggests a technique text. ‘whimsical’ which means ‘odd or
30. The clue lies in the words ‘separated into
for remembering something permanently. quaint is appropriate. Choice (1)
elements’. According to this ‘analytical’
‘Evanesce’ refers to things which ‘fade is the appropriate word. Choice (4) 45. ‘Triumph’ and ‘support’ are appropriate
from sight’. It is the appropriate word as it words. The words blend into the given
suggests that glueing’ helps to remember 31. The word ‘sustains’ which means sup-
context. Choice (4)
words which fade from sight. Choice (1) port, bear the weight of is the right word
for the first blank. ‘Moral’ is the appro- 46. The words ‘breeding’ ‘behaviour’ and
21. The word ‘foreseen’ indicates something priate word for the second blank as the ‘barriers’ do no collocate with the context
that is about to happen. Hence “imminent” context is about rejection of dishonor. given. the right words are ‘boundaries’
suits the first blank. In the second blank,  Choice (4) and ‘transgression’. Choice (4)
we should use a word that describes bad
weather. Choices (3) and (4) are elimi- 32. The words ‘peripheral’ and ‘sensational’ 47. ‘Sentient’ means to have the power of
nated as the weather is not ‘impending’ are appropriate words. Choice (1) perception by the senses ‘Abeyant’ refers
or ‘dangerous’. ‘Imminent’, which means to state of temporary disuse. In the given
33. The sentence is about the importance of
‘soon to happen’, and ‘inclement’, which context option 4 is appropriate.Choice (4)
folktales in Eskimo culture. ‘Sedentary’
means ‘severe/stormy’ completes the refers to much setting and little physical 48. The words ‘non-physical and ‘genes’ fit
meaning of the sentence. Choice (1) exercise since it is a folktale the narra- into the context well. Choice (2)
22. ‘Asphyxiants’ interfere with oxygen sup- tor’s, ‘imaginative’ skill is important. 49. ‘parlance’ means to speak in a particular
ply. In the second part of the sentence,  Choice (2) way. Option (1) is appropriate.Choice (1)
the clause “gases that are chemically in- 34. The context draws a difference between
active” should take a word that conveys 50. The words ‘feebly’ and ‘succumb’ are ap-
the reader and the listener. The options
a similar meaning. ‘Inert’ fits the blank. propriate words to fill the blanks in the
‘unlike’ and ‘ in-acquainted’ are suitable
 Choice (3) given context. Choice (2)
words. Choice (3)
23. The words “against” and “encourages” 35. ‘Speculative’ converting’ and ‘prosecut- Practice Exercise 4
are clues which make it imperative that ing’ are irrelevant words. ‘Proselytizing ‘
the blanks should be filled with contrast- is to convert a person from one opinion, Solutions for questions 1 to 50:
ing words. We can eliminate choices (1), creed to another. In the context it is an ap-
(2) and (3), as the words match. Only op- 1. Option (3) is the right choice. To adopt a
propriate word. Choice (4) new attitude, plan or way of behaving is
tion (4) has contrasting words.Choice (4)
36. The words ‘participatory’ and ‘intimately’ to begin to have it ; espouse. To grovel is
24. The word ‘sycophants’ in the sentence to crawl on the ground, especially to find
are appropriate words. Choice (2)
points directly to ‘obloquy’ which means something. It means that we should not
bad reputation, and ‘obsequious’ in op- 37. ‘Exiguousness’ means scanty ‘extant’ re-
assume superiority and look down on oth-
tion (1). The others are irrelevant. Hence fers to still surviving. In the given context
ers. Option (1) is ruled out because of the
only option (1). Choice (1) these words convey the right meaning.
second word loitering, which means to
25. A ‘mendicant’ meaning a beggar is out of  Choice (1)
stand or wait somewhere especially with
context. The word “liar” after the blank 38. ‘Kinesthetic’ refers to movement. The no obvious reason. Option (2) – to endorse
makes ‘mendacious’, meaning untruth- word ‘intelligible’ which means mean- is to support, approve of. To stutter is to
ful, redundant. Both ‘nefarious’ meaning ingful is appropriate for the second blank. have difficulty speaking because you can-
wicked and ‘nebulous’ meaning vague are  Choice (4) not stop yourself from repeating the first
not appropriate to describe a liar. ‘Invet- sound of some word several times ; stam-
39. ‘Posterity’ refers to ‘all succeeding gen-
erate’, meaning compulsive, fits the first mer. Option (4) is ruled out as merely ‘ac-
erations’ ‘couched’ is to express in words
blank. The second word fallacious which cepting a philosophy is not what is being
of a specified kind. Choice (1)
means ‘false’’, in option (3) fits the sec- talked about, in the sentence. To ‘adopt’
ond blank. Choice (3) 40. The physical components of environ- is one step ahead. Also, to flounder is to
26. The words ‘secession’ and ‘resistance’ ments are relatively ‘predictable’. Option struggle to move. Choice (3)
do not fit into the context. For the second (2) is appropriate. Choice (2)
2. ‘Shallow’ and …… indicates that the
blank the word ‘holding’ is more appro- 41. The appropriate words are ‘abstract’ and word in the first blank is synonymous to
priate than anchoring. Choice (2) ‘imitate’. Choice (3) it and ‘but’ indicates that the word in the

08_Section.indb 30 10/30/2009 11:52:45 AM


Solutions for Language Comprehension  n  31

second blank is antonymous to it. Hence, ficult situation. Option (4) also is not apt. 10. While indicates that there is a contrast.
frivolous, which means superficial fits To rationalize is to find a logical reason ‘Similarities’ and ‘which’ indicate that
into the first blank and profound, which to explain. It does not collocate with ‘in’. the word in the first blank refers to the
means deep, weighty fits into the second  Choice (1) differences. Hence, diverse, which means
blank. Hence, choice (1) is the right op- 6. The fact that functioning is resumed ‘made up of a wide variety of things’ ;
tion. Option (2) is ruled out as upright, the shows that it is temporarily suspended. various, divergent, different fits into the
word in the second blank does not fit into Moreover, ‘from within’, suggests that first blank. Also, striking, which means
the context. Upright – honest, respect- it happened naturally. Hence, spontane- obvious, marked collocates with similari-
able.. option (3) is ruled out. Volatile – ously, which means – happening natu- ties. Hence, option (4) is the right option.
unpredictable, variable. The second word rally, without being made to happen is Option (1) is ruled out as diffuse means
is not antonymous to volatile. Cogent the right word in the second blank. spread out, scattered. Option (2) is also
means compelling, forceful. Option (4) is Therefore, choice (3) is the right option. ruled out as diverting which means amus-
also ruled out even though the two words Option (1) is ruled out as thwart, which ing, delightful does not fit into the blank.
are antonymous to each other as they do means prevent does not fit in because of Option (3) does not fit in even though
not gel with the sentence. Choice (1) ‘resume….’ Option (2) is also ruled out. divergent, which is a synonym of diverse
To inhibit also means the same as thwart. fits into the first blank, because analo-
3. The sentence clearly talks about the im-
Also, the second blank cannot accommo- gous, which means – parallel or similar
portance of language. It says that it would
date prompt which means done or acting becomes redundant when used with simi-
be wrong to hold a view that people can
without delay. Option (4) – To terminate larities. Choice (4)
do without language. Hence, illusion and
incidental fit into the blank. An illusion means to make something end completely
11. Option (2) is the right choice. The root of
is a false idea or belief. Something that and hence cannot fit into the first blank,
something is the basis, core, essence. The
is incidental is less important acciden- even though instinctively fits the second.
‘essential’ ideas control our intellectual
tal. ‘Mere’ in the sentence points to the  Choice (3)
life that is, they are very important (es-
word in the second blank. Option (1) is 7. Far from being ‘weakened’ indicates that sential ideas). Option (1) is ruled out as
ruled out. Delusion is synonymous with the laws gained strength. Hence, vigour, facilitate which means to make smooth,
illusion but fictional means imaginary which means energy, force or enthusiasm easier does not collocate with intellectual
and does not fit into the second blank. fits into the first blank. The tendency to life. Option (3) is also ruled out as regu-
Option (2) is also ruled out. Misconcep- favour ‘oldness’ is also brought out in the late means to control, by means of rules
tion can also be considered synonymous sentence. Hence, prejudice which means and hence does not fit into the second
with illusion but it does not collocate with bias fits into the second blank. Hence op- blank. Option (4) – determined means to
imagine. Something is a misconception. tion (3) is the right choice. Option (1) is decide and it does not gel with ‘intellec-
Trifling – unimportant. Overstatement – ruled out as esteem collocates with ‘for’. tual life’. Crux, pivot and core, all fit into
exaggeration. Customary – usual, normal. Option (2) is ruled out as ‘leaning’ is used the first blank. Choice (2)
 Choice (3) with towards and hence does not fit into
4. To ‘hollow something out’ means to re- blank two. Option (4) is ruled out as a facet 12. ‘Degrading’ indicates that the word in the
move the inside part of it. The sentence is a particular part or aspect of something second blank is negative. Also, science
suggests that the content or the essence and is used with ‘of’.  Choice (3) and technology points to the answer. The
has been removed, without disturbing the right option is (1). To relieve somebody of
8. The right option is (2). The key word in
framework or skeleton. Hence, choice (2) something is to help, by taking something
the sentence is ‘perform’. Hence, only
is the right answer since substance means heavy or difficult from them. Drudgery
‘function’ – a special activity or purpose
essence and something that is intact is is hard boring work. Hence it fits into the
of a person fits in, as it collocates with per-
complete and has not been damaged. Op- context perfectly. Option (2) is ruled out as
form. To eject which means ‘to oust, ex-
tion (1) is ruled out as the second word ‘freed’ does met collocate with of and also
pel’, also fits In-Judgements are made and
– steadfast means loyal ; staunch ; firm. weariness – tiredness does not fit into the
not performed, obligations are met. Op-
Option (3) is ruled out as the foundation second blank. Option (3) is not the answer
tion (4) is ruled out because of ‘evinced’.
is not hollowed out. Option (4) does not as redeem – save does not collocate with
To evince, means to reveal or indicate.
fit in as ‘effects’ does not fit into the first ‘of’. To deprive someone of something is
 Choice (2)
blank, even though untouched can fit into to take away things that are necessary and
9. The correct option is (2). To resist means hence does not fit in. Labour and toil fit
the second blank. Choice (2) to oppose. The words ‘dared’ and ‘non- into blank two. Choice (1)
5. The right choice is option (1). It means confirmist’ point to the answer. Proclaim
that those values that were nurtured in is to declare. That means that they were 13. Option (1) is ruled out as zeal, which
the past are no longer relevant in an age looked upon, as the enemies of people. means enthusiasm collocates with ‘for’
deeply influenced by science etc. To Option (1) is ruled out as to reproach is to and not towards. Option (2) is ruled out
‘nourish’ means to foster, maintain. To blame or criticize. Merely criticizing will as ‘proclivity’ which means inclination
be steeped in something is to have a lot not make them non-confirmists. Option collocates with for. Option (3) will not
of a particular quality. Option (2) is ruled (3) is also ruled out for the same reason, work as motivation ‘to’ do something or
out as ‘deluged’, does not collocate with though both declared and dubbed can fit ‘for’ something is the right usage and not
‘in’, it is used with ‘with’. Also, it does into the second blank. Option (4) is ruled towards. Option (4) fits in best as ethic
not fit into the context. Option (3) is also out as to entitle is to give a title to a book, means principle or concept and drive,
ruled out. To embroil oneself is to become play etc. Hence, it does not fit into the which means impulse, motivation, fits
involved ; entangled especially in a dif- context. Choice (2) into the second blank. Choice (4)

08_Section.indb 31 10/30/2009 11:52:45 AM


32  n  Solution Manual

14. Option (1) is the right choice. If some- 18. The right option is (4). If something clearly ruled out. Though the words in
thing is derived from another thing it gains ground, it becomes more power- options (1) and (2) for the first blank can
is developed from that or the first thing ful or known. Hence, it fits into the first fit in, the phrase ‘disputes and mistrust’ im-
resulted in the second. To institute is to blank – the manifestations are increasing. plies the need for a consensus in the political
introduce a system, policy etc; to estab- Also, racism etc. are forms of intolerance arena. Hence, ‘political’ renders option (2)
lish. Option (2) and (4) are ruled out as or bigotry. Option (1) is ruled out .If the more apt than option (1). Choice (2)
merely examining or suggesting new manifestations are put to an end, there is 24. The clue is the word ‘fear’ the word for
forms would not lead to any progress by no need to renew efforts. Option (2) is the first blank should match with it. ‘In-
itself. It is only when it is acted upon, that ruled out as racism etc are not types of in- credulity’ in option (1) meaning ‘disbe-
progress takes place. To emanate from is dignity – shame, loss of pride. Option (3) lief’ does not go with ‘fear’ (which can
to come from. To fabricate is to invent or is ruled out as they are not representations only stem from some belief that some-
create. It is ruled out for the same reason and also of savagery – violence. Choice thing will affect the person). Option 3 is
as (2) and (4). Choice (1) (4) ruled out for the same reason. ‘Discre-
15. The right option is (2). Orientation is the 19. The right option is (2). Option (1) – ‘ad- tion’ meaning ‘tact’ or caution ruled out
act of directing your aims towards a par- vocated’ and as a ‘human right’ do not fear. Option (2) is ruled out for the same
ticular thing ; attitude. It means that his go together. Also, it is not mandatory or reason ‘Uncertainly’ goes with ‘fear’ be-
perspective is broad and that he does not compulsory, it is necessary or required, cause fear may be caused by uncertainly.
depend on specific or particular circum- that is the basis. Also mandatory prereq- Thus, only option (4) is appropriate.
stances as suggested by the word ‘univer- uisite is redundant. Option (3) is ruled  Choice (4)
salistic’ and also the conjunction ‘than’ out as it is not a ‘natural’ gateway. Also, 25. It is clear that the second blank needs a
which suggests that there is a contrast. gateway ‘to’ something is a means of get- negative word. ‘Eulogised’ in option (1),
Option (1) is ruled out as ‘limited’ does ting or achieving something. It does not having a positive connotation, rules it
not gel with circumstances. Option (3) fit into the context. Option (4) is ruled out out. ‘Impute’ is always followed by the
is ruled out as ‘structure’ cannot be used as a premise is a statement or idea that preposition ‘to’. This rules out option
since we are referring to a person and not forms the basis for a reasonable line or ar- (2). ‘Accused’ in option (4) is followed
to organisation. Option (4) is ruled out gument ; a hypothesis. Hence, universally by the preposition ‘of’. Thus, only option
as ‘external’ is irrelevant to the context. acknowledged or recognised …. condi- (3) renders the sentence meaningful and
 Choice (2) tion fits in  Choice (2) grammatically correct. Choice (3)
16. Option (2) fits in best. Subsequent learn- 20. The right option is (1). The ‘fact’ makes 26. The word ‘covetous’ (which is disapprov-
ing would mean all the other levels of it inevitable…. everyone has to …… con- ing) is the clue. Hence, option (2) though
education that follow primary education duct. Option (2) is ruled out as ‘placed’ it gives sense to the sentence, is ruled out
as subsequent means ‘happening after …. should be followed by ‘on’. Same as it’s not in tune with the tone of the sen-
something else’. Also, ‘mastery of’ is an is the case with confers. Also, ‘exhibit’ tence. The word ‘media’, in option (4),
objective or purpose of primary educa- does not gel with line of conduct. Hence, gives a neutral sense. Hence this too is
tion. Hence, goal is the right word. Con- option (3) is ruled out. Option (4) is ruled out for the same above given reason.
sequently would mean ‘happening as a also ruled out as to ensue is to resut or ‘Logic’ in (3) does not collocated with
result of something’. Hence, option (1) follow. It does not fit into the sentence. ‘beautiful’. Choice (1) is apt. Choice (1)
is ruled out. Most important factor ‘in’ Also ‘principle’ does not suit the context
would be right. Hence, (3) is ruled out.  Choice (1) 27. ‘Literary work’ does more than just re-
Option (4) is ruled out as to say ‘reading porting. This rules out option (1) while
21. The word required for the first blank ‘scientific’ work is for the sole purpose
skills’ are the basis of influential theories
should be positive. Hence, ‘seditious’ in of reporting thus it makes sense. In op-
is too far-fetched.  Choice (2)
option (1) and ‘drastic’ in option (3) are tion (3), ‘poetic’ is ruled out by the word
17. The right option is (3). ‘different areas’ ruled out. Between options (2) and (4), ‘report’. Option (4) is clearly ruled out.
points to the word distinct. Option (1) is the latter is ruled out as ‘peace’ in option The purpose of a rhetoric is to emphasise
ruled out because of the word ‘concur- (4) is contradictory and so distorts the or exaggerate and not summarise. This
rent’, which means parallel, as opposed meaning of the sentence. Thus, only op- makes option (2) correct.  Choice (2)
to distinct. Option (2) is ruled out as ‘hu- tion (2) is appropriate. Choice (2)
man rights’ cannot be called a system and 28. The conjunction ‘and’ in the sentence
22. Option (1) is ruled out because ‘in the requires that the two words for the blank
also because of the word ‘expression’.
name of’ suggests something derogatory should be matching. ‘The word ‘enlight-
One is a matter of… and the other …,
hence praise cannot fit in the second blank. enment’ in option (3) does not collocate
means that the second blank also requires
We can say someone has achieved or earned with the verb ‘rules’ and so is eliminated.
a word that means issue or matter. Hence
fame but not ‘drawn’ fame. This rules out Though the first word in options (2) and
‘expression’ does not fit in. Option (4)
option (2). So is option (3) as splendour too (4) seems to fit in the blank, the word
is ruled out as something is ‘conceived
does not collocate with the verb ‘drawn’. ‘fashion’ renders ‘designing’ and ‘de-
of’ as.. and not just conceived as. Also,
Option (4) is the appropriate answer choice. velopment’ irrelevant. Only option (1)
‘consideration’ – something that has to
 Choice (4) makes sense in the sentence.  Choice (1)
be thought about, especially when you
are planning or deciding something does 23. The clue is “by disputes….”. Hence, the 29. ‘Correlation’ is totally irrelevant. ‘Rec-
not fit into the second blank. A ‘question’ second blank is a negative word. So, ‘pal- tifying’ is rendered incorrect by the
in option (3) means the same as matter, liated’ meaning alleviated or improved in word repeal. So is ‘circumvention’. It is
issue. Choice (3) option (3) and ‘nullified’ in option (4) are obvious that in order to decide to change,

08_Section.indb 32 10/30/2009 11:52:46 AM


Solutions for Language Comprehension  n  33

repeal or perpetuate a law, one needs to The remaining three are irrelevant in the and 4 we can say the person can be ‘wor-
‘understand’ it. Thus, option (3) is the apt context. Choice (1) ried’, or chagrined’ is more appropriate
answer choice. Choice (3) than the other two words. Choice (3)
36. ‘To crib’ is to grumble. Here people al-
30. The sentence talks of people who pass ways grumble saying that life has become 49. ‘Dextrous’ is skill in handling and ‘clum-
“destructive laws”. Hence the word re- stressful. Choice (2) sy’ is lack of it option (4) is appropriate.
quired should be a negatively connoted  Choice (4)
37. Learning or studying the earth is no doubt
word. Further the other word for the sec-
interesting. But it is also a ‘challenge’ or 50. Option (2) is appropriate. ‘Emaciated’ is
ond blank should be a positively connot-
a demanding task. Choice (4). thin and weak. Choice (2)
ed word. Hence, ‘Potentates’ meaning
kings being a neutral word is a misfit for 38. Doping is ‘prevalent’ or ‘common’ or
‘widespread’ in certain fields of athletics
Practice Exercise 5
the first blank. The remaining three are
possible. ‘Narrow-minded’ being a nega- in India. But some of these athletes have
Solutions for questions 1 to 40-:
tively connoted word is ruled out. So is been wrongly ‘accused’ or ‘charged with
‘light-hearted’ which is out of context. this crime’. Choice (2) 1. The view that is talked about in the sen-
Option (1) is appropriate in the context. tence is that of a person. Hence the sen-
39. The hunger made me consume the food
 Choice (1) tence should start with ‘according to _ _
quickly and greedily or ‘devour’ the food.
_’. Therefore C, which says ‘for singer’
31. The two clues are ‘but’ and “uncertain-  Choice (3)
starts the paragraph. D follows as it in-
ties” which make it imperative that both 40. A person becomes ‘debilitated’ (weak- troduces the view. A form a pair as A
the words required for the blanks should ened) by a long bout of fever but not comments on the view. E comes after
be negative. So, ‘promoting’ in option (1) ‘apprehensive’ (fretful), ‘salubrious’ them as it is an extension of the previous
rules it out and in option (3), ‘escalating’ (healthy), ‘flummoxed’ (confused) or statement. The sentence ends with B as it
rules it out. “Maturity” in option (4) ren- ‘sprightly’ (lively). Choice (2) finally states the author’s opinion. Hence
ders the sentence incorrect. Also, the word CDAEB is the right sequence.Choice (2)
‘but’ renders ‘maturity’ incorrect. Only 41. Only the word ’artlessness’ (sincerity,
option (2) is apt as if renders the sentence straight forwardness) conforms with the 2. It is clear that A and E cannot start the sen-
meaningful and correct.  Choice (2) phrase ‘childlike simplicity’. The words tence. C is general in nature and it starts
‘charisma’ (charm) ‘impudence’ (disre- to talk about an idea which is elaborated
32. Option (1) is eliminated as the word ‘vi- spect), ‘inanity’ (foolishness) and ‘frivol- in the rest of the sentence. B states the
sion’ in it does not collocate with the word ity’ (childishness) are not logically appro- idea and is followed by E which says that
‘garner’. The clues are ‘lack of _ _ _’ and priate in the given context. Choice (4) the idea seems particularly relevant to _ _
‘absence of _ _ _’ hence the word for the _ and hence, E should follow CB. A is an
first blank should be negative. This rule out 42. Only in a ‘secular’ (non-religious) society
people belonging to different religions extension of E since it is relevant to two
‘simple’ in option (2), option (3) is the an- things. It is relevant - because the present
swer choice that is logical and makes sense live in mutual harmony and ‘tolerance’.
The other choices are not logically appro- situation is stated in D and hence, it con-
in the context. ‘Pertinacious’, being a posi- cludes the sentence. There fore CBEAD
tive word, is ruled out. Option (3) is apt.. priate in the given context. Choice (4)
is the right sequence. Choice (1)
 Choice (3) 43. The woman was attractive. So the man
went to her ‘actually’ to take her phone 3. B and D clearly form a pair as one talk
33. The phrases ‘will not be free of” and “ex- about a debate and the other starts with
cessive power” shows that the tone of the number and address but pretending to ask
for the time or ostensibly’ asking for the ‘over’ - debate over something. Also, D
sentence is negative. This rules out ‘guid- and A also form a pair because D ends
ance’ which is a positive word, the sen- time. Choice (1)
with augmenting water supply and A talks
tence talks of statisticians who provide 44. The first words of all the four choices can about how that can be done. Hence BDA
statistics. In the context, ‘repel’ in option fit into the first blank. But choices 2, 3 and form a sequence. C starts the sentence as
(1) is made irrelevant by the word ‘en- 4 can be eliminated because the words it talk about when the debate exploded. E
hance confidence”. ‘Follow’ in option (3) ‘relinquished’ (surrendered), ‘expended’ should precede D as episode here refers
would distort the sentence as ‘confidence (consumed) and ‘abandoned’ (forsaken) to the debate. Hence, the right order is
is enhanced’ only if what the statisticians do not logically fit into the second blank. CEBDA. Choice (3)
do is bettered and not replicated. Option  Choice (1)
(2) would mean that even legislation will 4. C starts the sentence as it talks about an
45. A person feels ‘groggy’ (dazed) only if he ‘idea’ which is elaborated in the rest of
not make statistics free from interference
has had a ‘fitful’ (intermittent) sleep the sentence E has to follow as A as ‘which’
and the statisticians can still be directed
previous night. The remaining options are refers to the idea. It is followed by D as
(that is, dictated). Choice (2)
inappropriate. Choice (3) which here too refers to the idea. B comes
34. It is understood from the sentence that
46. Only option (1) brings the contrast between next as what the idea does is stated in it. A
the word for the first blank should be
the herbs that grow in excess and palatable concludes the sentence as it finally states
negative. Hence, all the words except for
vegetables which are in shortage.Choice (1) the idea. Hence, CEDBA is the right se-
‘slower’ in option (4) are misfits. Thus
quence. Choice (1)
only ‘slower’ and ‘disaster’ in option (4) 47. The word ‘pass’ shows that he tried to
are appropriate in the context. Choice (4) use something that is not real. Option (2) 5. B is opening part as BE together talk
brings out this idea effectively.Choice (2) about the negative impact of enmity and
35. It is understood from the sentence
conflict. DAC form a set as they state
that Ivan Illich is a writer. From the op- 48. The word ‘ridicule’ shows that the blank
what happens inspite of the enemity. D to
tions, only ‘critic’ in option (1) is close. needs a negative word. From option 2, 3
A is a link - integrate _ _ _ in _ _ _ also

08_Section.indb 33 10/30/2009 11:52:46 AM


34  n  Solution Manual

A to C is a link - intimate realms of _ _ _ sequences of ‘pressure of population’. E 18. B opens the sentence as it introduces the
marriage _ _ _ hence the right sequence is follows as it talks abut another aspect it sentence - the large number of street chil-
BEDAC Choice (2) neglected. Hence BDCAE is the right se- dren. A is a continuation. It is followed by
6. D is the opening part of the sentence as it quence. Choice (1) D as it states what it in the present con-
talks about the particular setting in which 12. The sentence starts with D as it talks dition of the kingdom. E and C together
the argument is relevant. It is followed about one’s view. It is followed by C and form the conclusion. C is followed by E
by A, which talks about some exceptions E. C is lined to E as E talks about the con- as ‘deserve praise _ _ _ and ‘for a pre-
(isolated cases). It is followed by C which sequence. It is followed by A as ‘under- server’ go together. Therefore, BADEC is
continues to talk about the cases. It is then stand’ is linked with ‘an opponent’s ‘point the right order Choice (2)
followed by E and B as both talk about of view’ in A. It ends with B as ‘our expe- 19. B opens the sentence as it introduces the
the general trend The words ‘and punish- rience’ shows a solution. Hence DCEAB focus of the sentence - the Great War. It
ments’ indicate that B is a continuation of is the right sequence. Choice (2) is followed by D as ‘which’ in D refers
E. Hence EB form a pair. The right se- to the war. C follows as it is a continu-
13. The sentence starts with C. It states one pos-
quence is DACEB.  Choice (3) ation, E is linked to B (D and C provide
sibility and indicates that there is another, by
7. The sentence starts with B as it introduces the use of ‘along with’. E follows as it states an additional piece of information), E is
the subject of the sentence. E follows as it the reason for extinction. It is followed by followed by A - ‘denied’ is linked to ‘by
states what the values the person firmly A, which states that there is another im- any _ _ _ student’. Hence BDCEA is the
believes in. It is followed by C which portant problem which is mentioned in D. correct sequence. Choice (3)
states the name of the person and through It is followed by B as there in a link be- 20. C should begin the sentence as the name
a conjunction of contradiction provides a tween contamination and ‘with substances of the lady who gave a particular warn-
link to A and D. nevertheless believed _ _ _ _’ hence, the right sequence is CEADB. ing is introduced. A follows as it provides
_ _ ‘that’. Hence C and A form a pair A  Choice (1) the warning and also when E follows as
and D are linked as ‘crucial _ _ _ _ for’. how fast it is decreasing is talked about in
14. B is the opening part as it introduces sub-
Hence, BECAD is the right sequence. E, in the form of a comparison. It is then
ject of the sentence. D follows as ‘in what
 Choice (1) followed by D, which is a continuation,
sense’ is described in D. It is followed by C
8. The sentence starts with B as it introduces as ‘even though’ is linked to what is stated ‘win back’, suggests that the statement
the topic - increase in power shortage. It in C. It is followed by E as it is linked to which talks about the decrease in func-
is followed by D as it states the reason for ‘those’ in B. In between, how they realized tions should come before D. It is followed
that. A follows as ‘while’ indicates a con- themselves is described in E is followed by by B, which talks about the consequence
trast - capacity _ _ _ targets. It is followed A as ‘service of a human purpose _ _ _’ is if they do not win back _ _ _ work (in
by C and E which talk about the failure of linked to ‘larger _ _ _ themselves’. Hence, D). Hence CAEDB is the right sequence.
the plans in ensuring that targets are met. BDCEA is the right sequence. Choice (2)  Choice (4)
Hence BDACE is the right sequence. 21. The sentence starts with C as it is intro-
15. B begins the sentence as it introduces the
 Choice (4) ductory in tone. It talks about how wom-
view. D and A cannot follow as both have
9. C opens the sentence as ‘lack of informa- ‘himself’ and ‘he’, which refer to ‘human en are judged. It is followed by A - there
tion about women’s lives’ is the subject. being’, in E. Hence BE forming a pair. E is a link between ‘regard’ and ‘as a - - ‘.
The consequence of this is stated in the is followed by D ‘squarely’ is linked to Why it is a mistake is talked about in DE
rest of the sentence. ED go together to ‘the fact that’ in D. That is followed by and B. D is linked by E. What is stated
give one consequence (perpetuates- - - C as it talks about what else a human be- in D and ‘to be sure’ are connected. E
contribution). A is the second (implies - - ing should do. The sentence ends with also presents a contrast - ‘but’ - - which
-) and B the third (and promotes - - -). The A. Hence, BEDAC is the right sequence. is stated in B. The right sequence is
sequence therefore is CEDAB.Choice (3)  Choice (1) CADEB. Choice (2)
10. The sentence begins with D “as the habit 16. B introduces the view; hence it is the first 22. The sentence should start with C as it in-
of obedience” is what is discussed in the part of the sentence. It is followed by troduces the topic - our experience being
rest of the sentence. A follows as ‘upon’ D as that states the view. C follows as it psychological. It is followed by A, which
in D is linked with, ‘the impressible na- explains what is said in D. Next comes is the beginning of the explanation. A is
ture of a child’. E talks about what the A, which talks about what you are likely followed by E. E is followed by D. ‘It’ in
habit does not develop, hence it follows to do, if you hold that view. AE form a D refers to what we receive by the sense.
it is followed by C as it talks about what pair as judge as inferior _ _ _ a work in The sentence ends with B - which is a
it develops instead. The paragraph ends _ _ _ hence, BDCAE is the right order. continuation of ‘the sense - mind’ in D.
with B which is a continuation or a con-  Choice (4) The right order is CAEDB. Choice (1)
sequence of what is stated in C. Hence
17. C is the only part which talks about ‘an ar- 23. C is the introductory part as it talks about
DAECB is the right order Choice (2)
gument’ in general terms. Hence it opens the a particular judgment and what it did not
11. B is the opening part of the sentence as sentence. It is followed by D as ‘are’ refers do. It is followed by E, which talks about
it introduces the view of the Malthusian to arguments. It is followed by A, which is a what it did ‘It’ in E refers to the judgment.
league. It is followed by D as ‘point out’ continuation. E follows as ‘it’ refers to ‘the It is followed by B as B has the result of
and ‘that’ are linked. It is followed by C arguments’ in A. The sentence ends with what is stated in E. The sentence ends
as ‘take account’ is linked with ‘of’ and B. hence, CDAEB is the right sequence. with D, ‘which’ in D refers to ‘universal
what was neglected is stated in C. It is  Choice (1) attention’ in B. hence, the right sequence
followed by A as it talks about the con- is CAEBD. Choice (2)

08_Section.indb 34 10/30/2009 11:52:46 AM


Solutions for Language Comprehension  n  35

24. E is the introductory part, as indicated by 31. The verb ‘survived’ requires an object. that the sequence in option (1) does not
its tone. It is followed by B which gives That is, what is that which something (or make sense if D follows E. C after E is
the reason or the setting in which they ap- someone) has survived. The words ‘the meaningful. This rules out option (1).
pear redundant. It is followed by D as it is ravages’ in A provide it. BA as an opening From the remaining options, we see that
a comparison - ‘as _ _ _ ‘so _ _ _ ‘. It is pair makes the sentence logical. The con- AD and EC are pairs. Further, B can-
followed by C, which takes this worldly junction ‘and’ joins the two phrases − ‘as not precede AD as it would then refer to
pleasures’ forward. Also, ‘along with’ is a legend’ and ‘as a window’. So D should the clause ‘it would mean’ and alter the
an indication that it is the continuation. follow E. Only option (3) is logically cor- meaning of the sentence. From the con-
The sentence ends with A. Hence, EBD- rect. Choice (3) stitution point of view, “supremacy over
CA is the correct order. Choice (1) parliament” is ‘unacceptable’. Hence B
32. The verb ‘passed on’ should be near the
25. B opens the sentence as it talks about an modifies AD and so should follow it. This
phrase ‘to the next generation’ so B must
‘act’, which is elaborated in the rest of rules out option (4). EC is better as the
follow C. makes CB a pair. This rules out
the sentence. It is followed by D, which opening pair than as a concluding pair.
option (1). The conjunction ‘or’ makes
gives a condition. It is then, followed Thus, between options (2) and (3), the
DE a pair. The position of A is appropriate
by A which states the act. C and E ex- latter is logical. Choice (3)
after DE as it has the pronoun ‘it’. Thus
plain it further and the sequence is CE DEACB is meaningful as well as gram- 39. It is understood from the sentence that
as E has a concluding tone - it states matically correct. Choice (3) the welfare of the rural population is as-
the intention. Hence, the right order is sociated with a faster pace of growth of
33. It can be seen that the conjunction ‘if’ in
BDACE Choice (1) rural economy. Thus, we se that A modi-
A needs a clause to make it meaningful as
26. C opens the sentence as it talks about a fies D and so DA is a pair. This rules out
well as grammatically correct. Either C or
‘conviction or belief’. E follows as it option(2) and (3) as AD makes the sen-
D can follow it. This rules out option (1)
states the conviction. A follows as it talks tence incorrect because of its awkward
and (4). Option (2) is a logical sequence.
about how it is contrary to religious be- construction In option (1), E after DA
 Choice (2)
lief, and is followed by D. The structure renders the sentence grammatically incor-
34. Option (3) is ruled out as DA as a pair rect. Only option (4) is both logical and
not only to _ _ _ but also to _ _ _. B con-
is grammatically incorrect. We see that correct. Choice (4)
cludes the sentence. The right sequence is
B and C as a pair does not convey any
CEADB. Choice (3) 40. It is illogical for authorities to murmur
meaning, while CB does. This rules out
27. C is the opening part - it is the introduc- their protests. This rules out (3) and (4).
option (2). Option (1) is ruled out as the
tion is followed by E and A. E is an ad- Option (2) is an awkward construction.
sequence does not make any sense. Op-
ditional piece of information. C and A are Only Option (1) makes sense logically.
tion (4) is meaningful, hence is the correct
linked. Hence, CEA form a group. It is Neither the drivers nor the passengers
choice. Choice (4)
followed by B - which qualifies what is could do anything more than murmur
stated in A. It ends with D ‘Demonstra- 35. We see that A cannot follow C. A ‘ten- their protests as the authorities (presum-
tion’ and ‘of’ form a link. Hence, CEABD dency’ can be ‘natural’ but not a ‘key’ to ably the traffic authorities) noted down
is the correct sequence. Choice (2) anything. Thus CA, being a pair, rules out their vehicle numbers.  Choice (1)
option (2) and (4). Option (3) is incorrect
28. C is the beginning of the question which as it is an incomplete sentence. Thus, only Practice Exercise 6
is talked about in the rest of the sentence. option (1) which is logical is the correct
It is followed by a - actions of _ _ _ parts Solutions for questions 1 to 100:
choice. Choice (1)
and E, which is a continuation as it talks 1. Commitment of employees is a critical
about the result. D comes next as it says 36. A cannot begin the sentence as it is awk-
factor for corporate ‘success’ the word ‘
‘we still are clearing’ about what is stated ward. Option (3) is therefore ruled out. It
commitment’ makes it mandatory that the
in the first part of the sentence. The sen- can be seen that AB is a pair joined by
blank needs to be filled with a positive
tence ends with B. The correct sequence ‘and’. The preposition ‘on’ requires that C
word. Choice (1)
is CAEDB. Choice (3) follows D. DC is missing in option 1, so
it is ruled out. E makes the sentence logi- 2. Work atmosphere should not be tiring.
29. B is the opening part as it introduces the
cal if it precedes DC while the sentence Infact it should be such that the employ-
topic. It is followed by D ‘its’ refers to
is rendered meaningless if it follows DC. ees are involved in what they are doing.
the child. It is followed by C, which gives
Thus option (2) appropriate. Choice (2)  Choice (3)
an example to illustrate what is stated be-
fore. It is then followed by A which talks 37. Both A and C can open the sentence. 3. Employee needs to be connected with the
about what B and D tell us. The sentence We see that the pronoun ‘it’ in B stands goals and objectives of the organization.
ends with E. Hence BDCAE is the right for the rainforests. Further ‘species’ in  Choice (4)
sequence. Choice (2) C, which is in the plural should be fol- 4. Office environment must be in the right
lowed by the verb ‘rely’ in B. Therefore,
30. The sentence starts with B - it introduces sprit. For this the management must take
it is clear that CB is a pair. This ruled out
the subject - tyranny of man. It is fol- the necessary steps to create such an en-
options (1) and (2). Between options (3)
lowed by E - arguments _ _ _ forward. vironment. Choice (3)
and (4), the latter is ruled out as the word
It has a link with D. ‘Prove that _ _ _
‘renders’ makes the sentence grammati- 5. The appropriate word is ‘motivate’.
the two sexes _ _ _ ‘. It is followed by
cally incorrect. Option (3) is logically as  Choice (4)
C, which states that the same thing can
well as grammatically correct.Choice (3) 6. When the employees are motivated in a
be put more straight forwardly and that is
talked about in A. Hence BEDCA is the 38. The verb ‘ignores’ in E requires an object. proper direction human ‘expertise’ can be
right sequence. Choice (1) While either C or D can follow E, we see retained. Choice (1)

08_Section.indb 35 10/30/2009 11:52:46 AM


36  n  Solution Manual

7. The context is about retention so ‘com- 28. ‘Fundamental’ which means basic or pri- enrichment, ‘veneration’ – respect, ‘per-
pensation’ is the appropriate option. mary is the right option. Choice (4) version’ – wickedness/ ‘corruption’ – are all
 Choice (1) inappropriate in this context. Choice (2)
29. ‘Personality’ is the right option. Choice
8. The context is about a positive force’. So (4) 45. The words ‘distorted’ which means ‘mis-
the word ‘driving fits in well. Choice (4) construed, ‘stripped’ which means deprived
30. The voice of a person indicates his physi-
and ‘depraved’ which means perverted, are
9. ‘Switch’ means ‘change’ so an appropri- cal, emotional and mental health. ‘Indica-
all irrelevant in the given context. Only ‘de-
ate word in the given context. Choice (2) tor’ is the right option. Choice (3)
clined’ which means deteriorated is appro-
10. Several organization follow the policy of 31. ‘Residents’ which refers to people who priate.  Choice (1)
equating the pay with the performance, of live in a home or area is the right option.
46. The keywords here are ‘twenty-first cen-
the employee. It is called as ‘pay for per-  Choice (3)
tury’ and ‘historians’. The word that best
formance’ model. Choice (2)
32. The blank needs a word which will con- fits is ‘tagged’. Choice (2)
11. The tone of the sentence suggests sur- vey the meaning ‘progress’. ‘Advance’ is 47. The statement here is trying to establish
prise. ‘Dismay’’ which means the same is the right option. Choice (1) the link between terrorism and Islamic
the appropriate option. Choice (3) groups and hence the right word is ‘in-
33. With advancement, physical activity in
12. A city is fighting for its ‘survival’. The developing nations takes a downword volvement’.  Choice (1)
other words do not collocate with the sen- trend. the word ‘declines’ is appropriate. 48. The statement says that the alarming
tence. Choice (4)  Choice (2) frequency of such terrorist attacks could
13. The words ‘acceptable’, ‘accessible’ and 34. ‘Influences’ is the appropriate choice. negate all other achievements of this cen-
‘admissible’ do not hold any relevance to  Choice (1) tury. ‘Overshadow’ is the word that best
the context. ‘Vulnerable’ which suggests conveys this and hence it is our answer.
35. Cars take the place of walking in develop-  Choice (3)
susceptibility is the right word. Choice
ing nations. ‘Replace’ conveys this idea.
(3) 49. The blank here takes a negative word that
 Choice (1)
14. ‘Surrounded’ is the appropriate word. best expresses the mindless acts of vio-
36. The words ‘pleased’, ‘engaged’ and ‘re- lence being perpetrated; hence ‘abomina-
 Choice (1)
habilitated’ are not relevant. ‘Occupied’ is tion’ is the right choice.  Choice (3)
15. ‘Detained’, ‘maintained’ and ‘prevented’ the right choice. Choice (4)
are not relevant. Protected is the right 50. The statement says that western leaders
37. ‘Incidence’ which means frequency with assert that Islam is a religion of peace
word. Choice (1)
which something occurs is the appropri- and that extremists have gone against
16. ‘Breach’ is to make an opening in a bar- ate word. Choice (4) this basic tenet. The word that conveys
rier the right word. Choice (2) this meaning is ‘usurped’ which means to
38. ‘Implications’ is the appropriate choice.
17. ‘Filling up’ is the right phrase.Choice (3)  Choice (2) wrongly assume or seize. Choice (2)
18. It may take months before it becomes 39. ‘Daunting’ means afraid of something. 51. Both ‘resorts’ and ‘haunts’ fit in the blank.
normal. ‘Gets’ is the right choice .Choice The right option. Choice (4) ‘Resort’ is a holiday place. ‘Haunt’ is a
(1) place of frequent visit since it is enjoy-
40. ‘Access’ is the appropriate choice. able. Keeping in mind the tone of the
19. ‘Fled’ is the appropriate word. It means to  Choice (2) passage and the words ‘clubs of colonial
escape from something. Choice (1)
41. The word ‘depicts’ is inappropriate in this affectations’, ‘haunts’ will be the most ap-
20. Options 1, 2 and 3 have no relevance. The context because to ‘depict’ something propriate word. Choice (3)
right option is (4). Choice (4) means to portray by drawing. The word 52. ‘Galling’ and ‘exasperating’ can be ruled
21. Option (3) is appropriate ‘Ingredients’ ‘delineates’ is irrelevant. The word ‘extir- out since they mean ‘annoying’. The pas-
refers to things that are used to make up pate’ which means root out or destroy is sage is not discussing, ‘erudite’ or learned
something. Choice (3) irrelevant. Therefore the word ‘manifests’ people but it refers to the ‘privileged’ peo-
which means ‘reveals’, is the most appro- ple which means respected and admired,
22. To draw someone’s attention to something
priate in this context. The clue is “in dif- and ‘hallowed’ conveys this meaning.
is to make them aware of it or make them
ferent forms” which means “manifests”.  Choice (3)
think about it. The appropriate choice is
 Choice (3)
‘attention’. Choice (1) 53. After the word ‘hallowed’, the only word
23. The appropriate word is ‘making’.Choice 42. The word ‘apathy’ which means lack of that would suit the blank is ‘urbane’. It
(1) concern is the most appropriate in the means polished and high-class. The other
context. The other words are irrelevant. three words do not pertain to the para-
24. ‘Selling’ is the appropriate word. In the  Choice (1) graph. Choice (4)
context it refers to gathering support.
 Choice (2) 43. The words preceding the blank talk about 54. The word that suits the context is ‘iner-
the hunger for wealth, and misplaced tia’. It means to be unwilling to move or
25. The clue word is ‘interview’ ‘Appearing’ prestige, hence the word ‘competitive’ is be active. It is in keeping with the idea of
is the right choice. Choice (4) more appropriate than the rest of the op- ‘haughty resistance to criticism’. It also
26. ‘Infuse’ which means to fill with is the tions. Choice (3) takes the preposition ‘against’ after it.
right choice. Choice (4)  Choice (2)
44. The word ‘degeneration’, which means
27. ‘Charm’ which suggests attractive is the deterioration, is the most appropriate in 55. ‘Abjuring’ and ‘abdicating’ can easily be
right option. Choice (4) this context. The words –‘fortification’ – ruled out. They mean renounce or give up.

08_Section.indb 36 10/30/2009 11:52:46 AM


Solutions for Language Comprehension  n  37

‘Spurning’ means to reject. The clubs are suitable. The word ‘economy’ means to ‘seething crowd’ but it is inappropriate to
averse to reform and continue their old use something that is available in a way say ‘seething country’. ‘Populist means a
practices, unmindful of others opinions. that avoids waste. Text message helps to person who express populist views. Not a
‘Ensconsed’ conveys this idea.Choice (2) improve children’s skills in economy of right option. A country or area which has a
expression. Choice (3) lot of people living in it is ‘populous’. This
56. What follows the blank (touch the hearts)
is the right option. Choice (3)
makes ‘ennoble’ (give greater dignity to) 64. Text message gives children an oppor-
the apt word.  Choice (1) tunity to learn spellings and sounds of 68. Labourers who live in Goru Gotu struggle
words in a playful way. The word ‘il- hard to keep their families alive. The word
57. The paragraph refers to the outstand-
lustrious’ means very famous and much ‘revile means to hate something intensely.
ing performance of Pele, hence tran-
admired. The word is inappropriate in If this word is used it will mean labour-
scends (surpasses) is the right word here.
the context. The word calculating means ers hate something but continue to till the
 Choice (3)
‘crafty’. Not a suitable word because the land. It gives no meaning. The word ‘rev-
58. ‘Infectious’ is the only word that can word has a negative connotation. ‘Can- el’ is inappropriate here. It means to en-
be used to qualify ‘joy’ since it means did’ means to say what you think openly joy something very much. The labourers
‘likely to spread to or influence others’. or honestly. This is not an appropriate op- have nothing to enjoy. The word ‘shud-
 Choice (2) tion. Imaginative’ is having or showing der’ is inappropriate. ‘Shudder’ means to
59. You ‘triumph’ over a team. None of new and exciting ideas. Children learn to shake with fear, horror or disgust. It also
the other words can go with ‘over’. use imaginative ways in learning sounds means to shake in cold. The word ‘sur-
 Choice (4) and spellings of the language. Choice (4) vive means to manage to live or continue
to live inspite of difficult circumstances.
60. He is a phenomenon, so he defies ‘emu- 65. It is interesting to know that the more one
Labourers survive in that place tilling soil
lation’ (attempt to match or surpass). uses text message the better is his / her lit-
by hand and bear all the problems to keep
 Choice (3) eracy score. The word ‘preparing’ means
their families alive. Choice (1)
to make something ready to be used.
61. It was common to use rebuses or word ‘Cunning’ is to look for an opportunity 69. The fury of nature has left Goru Gutu
puzzles with picture even during the to attack. ‘Predicting’ is to say that some- starving. All their efforts to keep their
Victorian times. ‘Universal’ means in- thing will happen in future. These options families alive did not bear fruits. It is in-
volving all the people in the world. Since are not appropriate. ‘Beginning’ means appropriate to use the word ‘doldrums’.
we are talking about Victorian times in to start doing something. It is therefore It means it is very quiet and nothing new
particular to use this word will be inap- not surprising that studies are beginning or exciting is happening. ‘Amiss’ means
propriate. ‘Fabled’ means famous, often to show that the more you text the better something wrong. There is nothing wrong
talked about, but rarely seen. This is not your literacy scores will be. Choice (4) with the labourers efforts. It is the fury
a suitable option. ‘Dominant’ means no- of nature which causes problems to the
ticeable. This is not an appropriate option. 66. The paragraph is about people residing at
labourers. ‘Sham’ means not real or not
‘So not an appropriate choice. ‘Common’ Goru Gotu in Ethiopea. They suffer the
really what it seems to be. Their efforts
means happening often. It is an appropri- effects of famine. They are unable to stave
are not a sham. So it is inappropriate to
ate choice. Choice (2) off (stop something bad from happening)
use this word. ‘Vain’ is used to describe
starvation. ‘Anathema’ means a strong
62. Text messaging gives children a chance an attempt or action that fails to achieve
dislike. This option is inappropriate be-
to practise reading and writing. ‘Informa- what was intended. All their efforts have
cause the paragraph is not about likes or
tion’ gives facts or details of something. It been in vain because of the fury of nature.
dislikes. ‘Anorexia’ is a kind of illness in
is inappropriate to use this word. The word  Choice (2)
which a person has an over whelming fear
‘mode’ which means a particular way of of becoming fat, so refuses to eat enough 70. A famine affects human beings and ani-
doing something. To say text messaging and becomes thinner and thinner. This mals alike. ‘Warped’ is when something
gives children a ‘mode’ is inappropriate. option is illogical in the present context. gets damaged by bending or curving of-
The word ‘performance’ means the act of People living in Ethiopa may become ten because of the effect of heat or water.
performing a play, concert or some other anaemic due to lack of food. ‘Anaemia’ This is not an appropriate option. To use
form of entertainment. To use this word is is a medical condition in which there are the word ‘replete’ is irrelevant because it
inappropriate. ‘Opportunity’ is a time when too few red cells in the blood causing one means full of food and drink. The animals
a particular situation makes it possible to to feel tired and look pale. This is not a are not in an ‘aggressive’ mood. Not a cor-
do or achieve something. Text messaging right option. ‘Starvation’ means extreme rect option. Because of starvation animals get
gives children an opportunity to practise suffering or death caused by lack of food. thinner.  Choice (3)
reading and writing. Choice (1) The word ‘famine’ in the next sentence 71. The paragraph is about the Israel Philhar-
63. Text messaging helps to use words eco- further confirms that this is the right op- monic Orchestra. The orchestra is very
nomically. It improves the skill of expres- tion. Choice (4) well known. Israel is the home for many
sion. The word ‘Overflow’ means to be 67. Ethiopia is thickly populated. ‘Crowded’ classical musicians. There is a competi-
so full that the contents go over the sides. means a lot of people. While talking about tion for funds, though. The orchestra is
This is an inappropriate choice. ‘Locale’ the density of a particular country it is not called as one of the country’s cultural
means a place where something happens. appropriate to use the word ‘crowded’. jewels. ‘Legitimate’ means that which is
It is not an appropriate choice. ‘Pletho- We can say a ‘crowded’ place but not a approved by the society or acceptable to
ra’ means amount that is greater than is ‘crowded’ country. ‘Seething’ means full the society. This is not the right option.
needed or can be used. This is not a suit- of people. This adjective is appropriate to ‘Off stage’ is used to describe the behav-
able word in the present context. It is not talk about a huge gathering. We can say iour of actors or entertainers in real life.

08_Section.indb 37 10/30/2009 11:52:47 AM


38  n  Solution Manual

The paragraph is not about the behaviour do what they want. ‘Inevitable’ means it This is not an appropriate adjective to de-
of the musicians. It is not an appropriate is certain to happen and cannot be pre- scribe the popularity of the organisation.
choice. The Israel Philharmonic Orchestra vented or avoided. Israel has many clas- The organization has definite goals. To say
is not a privileged one. It does not enjoy sical musicians so competition for funds it is ‘Wanton’ will mean ‘it is motiveless’.
an advantage or opportunity that the other is inevitable and it is increasingly bitter. So it is not a correct option. ‘Obsequious’
orchestras do not have. The orchestra is  Choice (1) means obedient or attentive. TMTC is a
called as a cultural jewel. The option ‘glit- famous and celebrated organisation. The
75. The management at the Philharmonic is
tering’ which means something that is very option ‘renowned’ which means famous
not satisfied with the funds allotted be-
impressive or successful is effective to is the right choice. Choice (1)
cause it was sanctioned less money than
describe the popularity of the orchestra. in the previous years. So it decided to 79. ‘Transpersonal’ means transcending the
 Choice (4) launch a protest. ‘Condemnation’ is the personal. It is not a right option. ‘Uni-
72. The orchestra travelled to the battle fields act of saying that something or some one lateral’ means performed by or affecting
in war time to keep the spirits of the sol- is very bad and unacceptable. The man- only one person or party. TMTC is not
diers high. The word ‘rebellion’ is inap- agement has no intention of condemning a unilateral organization. ‘Statuesque’ is
propriate. The orchestra has no intention the government. It only wants to demand having the dignity or beauty of a statue.
of organizing a violent action to change mere funds. ‘Divergence’ which means a These options are not correct. TMTC has
the country’s political system. During the difference between two or mere things, become famous because of the respect
war time the orchestra has not travelled to attitudes or opinions is not an appropri- derived from its achievements. So the
the battle fields to discuss the principles ate option. The management at the Phil- option ‘prestigious’ is the correct choice.
and values concerning people’s behav- harmonic has no intention of creating a  Choice (4)
iour. So to use the option ‘morality’ to fill situation that interfere with the rights of
80. TMTC provides professionals with the right
in the blank is inappropriate. ‘Decorum’ the government. So to use the option ‘in-
opportunities. ‘Orgeat’ is a cooling drink. So
means the behaviour that people consider fringement’ is incorrect. It wants to plan
irrelevant option. ‘Eirenicon’ is a proposal
to be correct, polite and respectable. This a set of activities over a period of time
made for achieving peace. ‘Siege’ means a
is not an appropriate choice. The orches- in order to achieve more funds from the
military operation. Not a right operation. It
tra wishes to entertain the soldiers and government. ‘Campaign’ therefore is the
is not a right choice. ‘Expedience’ means a
keep their ‘morale’ high. ‘Morale’ means right option. Choice (4)
good chance of attaining goals. This is an ap-
the amount of confidence and cheerful- 76. The paragraph is about the Tata Manage- propriate choice. Choice (3)
ness that a group of people have. This is ment Training centre, its objectives, and
the right option. Choice (1) 81. ‘Obsession’ is the appropriate word. The
achievements. The Tata Management
first two lines of the passage which talk
73. The Israel Philharmonic Orchestra is well Training centre was set up by the famous
about mass murder of unborn girls sug-
known. ‘Milieu’ means a group of people or JRD Tata. ‘Catchpenny’ means superfi-
gest that from the age-old days, people
activities that one lives among or is famil- cially attractive and there is no substance.
are ‘obsessed with’ the idea of the male
iar with. This is not an appropriate choice. To say this about JRD Tata is inappropri-
child (sons). Choice (3)
‘Scrummage’ is a group of people who are ate. JRD Tata is not ‘dissentient’, because
close together and push against each other. he is not disagreeing with a majority or 82. The passage talks about Manu’s treatise
Not a right option. The orchestra has not official view. It s not correct to call him an which has become ‘acceptable’ through tra-
built a false outward appearance. It has a ‘ignoble’ person because he is not dishon- dition and belief. Hence ‘legitimized’ is the
genuine fame and name. To use the word ourable, mean or base. He is ‘legendary’. right word. Choice (1)
‘façade’ is therefore not correct. So to use This is the right option. Choice (2) 83. The passage says that obsession with sons
the word ‘chauvinism’ which means that is 77. The main objective of TMTC is to func- has ‘passed through’ or ‘percolated down’
not correct. The orchestra is well known tion as a learning institution and to from the past ages to 21st century India.
and is remembered for its performance. contribute towards the economic devel-  Choice (3)
The right option is ‘reputation’. The or- opment of the country. ‘Debilitating’ is 84. ‘Torn apart’ is the right option. The pas-
chestra built a reputation as one of the to weaken or enervate something. TMTC sage says that India which is said to be
world’s leading orchestras. Choice (2) has no intention of weakening the econo- in the process of growth is actually ‘torn
74. Israel is a country which is stuffed to the my of the country. So it is a wrong option. apart’ that is it is ‘ripped apart’ or ‘torn
rafters with classical musicians. Because ‘Curtailing’ is to cut short or reduce. This is into pieces’ from within. Choice (4)
of this there is always a stiff competition not an appropriate choice. When an event
is judged or determined even before the 85. The passage finally says that when is-
for funds. ‘Principled’ means some one
evidence is examined it is ‘fore judging’. sues like dowry and atrocities against the
with strong principles. This is an incor-
This option will not be appropriate in this minorities in the society have not been
rect option as we are not talking about
context. The practice of making things less taken to court. Then there is no surprise
principles to be maintained. ‘Punitive’
difficult is known as ‘facilitating’. TMTC if the ‘perpetrators’ that is the people
means actions that are intended to punish
as a learning institution is helping to groom who carry out the crime are not punished.
people. This is not a right option. ‘Inflam-
individuals into professional leaders. The  Choice (2)
matory’ means to say something which is
likely to make people react very strongly. right option is ‘facilitating’.  Choice (4) 86. ‘Exhortation’ is the appropriate word.
It is inappropriate to use this option. To 78. TMTC is one of the foremost management ‘Exhortation’ is a piece of advice given
use the word ‘constricted’ to fill the blank training facilities in the country. It is a fa- earnestly. The context in the passage talks
is also not correct. The word suggests that mous organization. To say it is ‘timorous’ about Christ’s advice to his followers.
actions are controlled and people cannot would mean it is easily alarmed or timid.  Choice (3)

08_Section.indb 38 10/30/2009 11:52:47 AM


Solutions for Language Comprehension  n  39

87. ‘Stoically’ is the appropriate word. The being ‘active’ and the other being ‘calm’. That is linked to neurological processes
context conveys the idea that Mahatma Further the word ‘even’ implies compari- mentioned in D. C, continues A as it talks
Gandhi submitted himself to the beat- son in degree of intensity. Hence, collo- about modification, which is an explana-
ings of white aggressors that is “he en- cation – wise, ‘quite active’ is the most tion of the alteration, mentioned in A.
dured pain and hardship without showing appropriate. Choice (2) Hence, BDAC is the correct sequence.
his feelings”. This is in keeping with the 100. ‘And’ in the sentence speaks of continu-  Choice (3)
words ‘turn the other cheek’. The other op- ation of the idea, that is the responses 5. B is an opening sentence as it is general in
tions are not suitable. Though ‘unemotion- of the people which were ‘active’ and nature. It also introduces the topic. As B
ally’, ‘insouciantly’, and ‘dispassionately’ ‘calm’. Therefore, ‘calm’ and ‘sleepy’ in talks about equality, D follows it as it also
are synonymous, they are not suitable in option 1 is the only other response that is talks about overcoming divisions within
the context. ‘Unemotionally’ means not appropriate in the blank. Choice (1) the human family that is, becoming equal.
‘expressing any strong feelings’; ‘insou- C follows D as optimism – hopefulness or
ciantly’ means ‘casually unconcerned’; Practice Exercise 7 confidence about the future, refers to what
‘dispassionately’ means ‘not influenced by is stated in D (overcoming divisions ……).
strong emotions’. Choice (3) Solutions for questions 1 to 40: A concludes by saying that instead of help-
88. Further, the passage says that one can not 1. ‘D’ is the introductory sentence as it talks ing dissolve divisions, progress has added
change man’s ‘convictions’. A ‘convic- about the concerned topic and the people to them or has made the situation worse.
tion’ is a firmly held belief or opinion. The who are related to it – Americans. B con- Hence BDCA is the correct sequence.
passage earlier refers to aggressors. But tinues what is said in D, both mention  Choice (3)
it does not talk about ‘man’s behaviour’ time periods and B talks about the pen- 6. C is the opening sentence as it talks about
or ‘outlook’. Hence they are ruled out. dulum swinging to one side, the reference the ‘selfish gene notion’. The rest of the
 Choice (2) being made to ‘weight loss’ mentioned in paragraph explains that concept. ‘selfish
89. Violence ‘embitters’ (that is makes bit- D. A further explains what that attitude gene’ indicates the place of the gene in
ter) its victims. It does not infuriate, or mentioned in B has caused. C concludes the scheme of things and that it is of great
agitate, exacerbate or worsen its victims. the paragraph by summing up the ‘culture importance. Only D can follow C as the
of slimming’. Hence, DBAC is the right rest of the sentences do not continue the
 Choice (1)
sequence. Choice (2) idea. D also talks about the ‘gene’. D says
90. ‘Reconcile’ is the most appropriate word. 2. A and D clearly form, a pair. ‘The re- that only the ‘gene’ exists for a long time.
It is used in the sense of bringing people sults of the restrictions imposed’ in A, It has to be followed by A, which talks
together or to agree with each other. The are talked about in D, that is cause and about how everything else is temporary.
strength that would enable them to do effect relationship. B is a concluding The paragraph concludes with B, which
this is the strength of God. ‘Propitiate’ is sentence as it says ‘if all relevant con- states that the ‘gene’ is immortal as op-
to please; ‘persuade’ is to force. ‘Reunite’ siderations.……….’ The considerations posed to other things, which are mortal
and ‘reassure’ also do not fit in the context. are mentioned in A, C and D. They are or impermanent (as mentioned in option
 Choice (3) weighed and a conclusion is arrived at, in A). Hence CDAB is the right sequence.
91. ‘Accords’ is the right word which means option B and is also stated in it. Hence,  Choice (2)
‘assigns’ or ‘designates’  Choice (2) C has to be the opening sentence. It in-
troduces the topic of discussion – distress 7. C is the opening sentence as it introduces
92. ‘Revered’ or respected is the right word in of work ….. on animal subjects. Hence, the topic – how the past influences be-
the blank. Choice (4) CADB is the right sequence.  Choice (1) haviour. Stone age here represents the
93. ‘Enables’ which means ‘assists in’ is the past. Hence, D, which explains the topic
3. D is the opening sentence as it talks about or states it clearly is what follows C. D
right word. Choice (2) science programmes and what they should
and B are clearly a pair as both talk about
94. Repose’ which means place is the right include. C follows as it says, unfortunate-
the past. This in B refers to the ‘past’; men-
word. Choice (3) ly, the current programme content is not
tioned in D. A concludes the paragraph
up to the mark. Hence DC form a pair.
95. ‘Imparts’ which means to pass or convey and this notion in it refers to the statement
How they a should be taught is given in A
knowledge is the right word in this blank. D. Hence, CDBA is the right sequence.
and it also provides a specific example. B
 Choice (1)  Choice (2)
concludes the paragraph by saying that it
96. One can not ‘device’, develop’, or ‘detest’ the is a good idea for children to learn about 8. C is the opening sentence as it introduces
‘effect’ while we can “determine” the effect. the environment since it is an educational the topic – new borns and their eyesight.
Thus collocation wise, only option 3 is pos- experience according to A. Hence, DCAB Also, it specifically talks about their in-
sible. Choice (3) is the right sequence.  Choice (3) ability to focus well. D follows C as it in-
4. B is the opening sentence as it is gen- troduces another problem when it comes to
97. For the same reason given in Q 41, only eyesight their visual acuity being limited.
option 2 fits in the blank as the energy eral in nature and talks about science in
a broad sense. B is followed by D as it A follows D as it illustrates that through
can only be either ‘mental’ or ‘physical.’ an example. B follows A as it also talks
 Choice (2) supports B by talking about the fact that
something new has been learnt about about ‘seeing unclearly’ across distances.
98. Except for ‘behaviour’ in option 4, which is neurological process – ‘identifying…… (A mentions distances). Hence, CDAB is
appropriate, all the others are irrelevant in processes’. It also talks about ‘science’, the right sequence. Choice (3)
the context. Hence option (4). Choice (4) which provides a link with B. A, which 9. D is the opening sentence as it talks about
99. The last sentence compares the responses talks about stimulants follows D. How ‘piece of land and a house.…….’, introduc-
of the two groups of people, that is one they alter brain cells is talked about in it. ing to us, indirectly, the topic. A follows D

08_Section.indb 39 10/30/2009 11:52:47 AM


40  n  Solution Manual

and clearly states the topic – realty. It talks talks about poverty in Africa ad E says Also, A has to follow C. D follows A.
about the craze for realty, hence proving to ‘the continents’. Hence the right sequence  Choice (3)
be forming a link with D, once again (pen- is BDCAE. Choice (2) 21. C cannot start the argument because of
chant in D and craze in A). B follows A as it 14. C is the opening sentence as it says – ‘sugar’ the word ‘these’. Therefore, options 2 and
elaborates on how the craze is all-pervasive. for the ‘first time’, which is introductory 3 are ruled out. Also, D must precede B.
C concludes as it says ‘such’ is the predomi- in nature. It talks about how ‘sugar’ got a  Choice (4)
nance ………. Hence, the right sequence is bad name. A also is on similar lines, it is
DABC. Choice (2) 22. The choices indicate that we have to look
an extension of C. Hence CA is a pair. B at B or C as the opening statement. B can-
10. B is the opening sentence. The whole talks about how foods high in sugar were not start the argument. The words ‘that is
paragraph talks about managing work and considered bad and that they were ‘dubbed why’ show it as an explanation. B follows
home. This sentence introduces the concept. as ……...’. D follows B as these term’s C. D must precede A. Choice (1)
It says that it is not just juggling parenting refers to “junk foods etc” of the previous
and work, it is much more than that. C fol- sentence. Hence, BD is clearly a pair. E 23. C must precede B. Also, the topic under
lows B as it talks about what else ‘it means’. connects CA and BD as it takes what C and discussion is ‘compulsory sports’ Hence,
C also talks about ‘care and women’. A fol- A state forward by saying attacks escalated D follows B as ‘disinclination’ in option
lows C as it talks about caregivers. C also or increased, which is what, is explained D springs from ‘compulsory sports’ in ‘C’.
talks about the impact this has on women or in B and D. Hence CAEBD is the right se-  Choice (2)
the consequences (costs) of all that is stated quence. Choice (4) 24. A gives the example of a farmer to elabo-
above. D completes the paragraph as it lists rate what is stated in ‘C’. ‘B’ is conclud-
15. C is the opening sentence as it brings out
the costs, specifically to women, who are ing in tone.  Choice (2)
the topic of discussion – that technology
the primary caregivers. Hence, BCAD is alone is not enough to find solutions to ev- 25. C must follow A linked by ‘infrastruc-
the right sequence. Choice (3) erything. A follows as it also talks about ture’. B must follow C. This happens in
11. D is the opening sentence as it draws at- ‘purely technological solutions’, which is choice 4. Also, D is suitable as the open-
tention to the topic of discussion - science. what C talks about. E follows A as it also ing sentence.  Choice (4)
It says that science has disturbed our uni- talks about the argument of scientists -
26. The question posed in statement A is be-
verse. It has to be followed by B. B talks that sole reliance on technology will give
ing answered in D. Therefore, D must fol-
about how eminent people in the filed and us all the answers. B follows E as it gives
low A. This happens in choices 1 and 2. B
their theories have constantly disturbed an alternative way of finding solutions. D
and C do not logically precede A. There-
us (relocated ……). It is followed by A, follows B as ‘that would mean’ is an ex-
fore, we rule out option 1. Choice (2)
in which ‘such disturbance’ refers to what tension of the alternative way mentioned
is stated in D and B. Also, it is clear that C in B. Also B says – ‘better understanding 27. A, C and D cannot be the opening
and E form a pair. C - ‘No period …..’ and of humans’ and D says - ‘emphasis on statements. Therefore, the Choice is 2.
E - ‘yet, no period’. Even though the pe- psychology …….’. Hence CAEBD is the  Choice (2)
riod was so dependent on science, it was right sequence. Choice (1) 28. C must follow D. The subject ‘animation
most uneasy about it. Hence, the right se- 16. Look at the choices and notice that either films being overlooked’ in D is followed
quence is DBACE. Choice (1) A or B is the likely ‘opening’ sentence. by ‘In a bid to revitalize’ ….in C. Option
12. E is the opening sentence as it introduc- We can rule out D as the concluding state- 1 is the answer since BA cannot precede
es two areas which are discussed in the ment. Therefore, option 1 is ruled out. D DC. Choice (1)
paragraph – ‘different cultural outlooks cannot follow A. So options 2 and 3 are 29. Either B or C can start the argument be-
or orientations’ and ‘views of the world’. also ruled.  Choice (4) cause they have the proper noun ‘Sanjana’
B should follow E as it talks about giv- 17. The choices indicate that either C or D in them. Also, A follows D. Hence, option
ing meaning to the world or ‘how people is the first sentence. However, the word 3 is the answer. Choice (3)
view the world’. It also says it is easy to ‘the’ in D indicates that this is not the
be critical of people who have a view dif- 30. One reading of the sentences makes it
first time that leaders are being men-
ferent from that of yours. D should follow clear that Jesse Owens is the subject
tioned. So, only C can start a sentence. A
B as these differences in D refer to what here. Therefore, D would be the first
follows C. Also D follows A because of
is stated in B. A follows D as A also talks statement. B follows D. Two choices – 2
the ‘check upon the people in power’. So,
about ‘respect’. A says that all the differ- and 4 – have this order. We now have to
 Choice (2)
ences are in ourselves and C says we see determine whether A comes before C or
the world only as we are. So AC is a pair. 18. The choice start with B or C. However, vice versa. A precedes C because you first
Hence C concludes the paragraph. The only C can start the argument as it states the ‘see’ someone/something which makes
right sequence is EBDAC  Choice (4) subject – the system of dowry. B follows you ‘nervous’ thereafter.  Choice (4)
C. Only, choice 3 has this combination.
13. B is the opening sentence as it intro- 31. B and D cannot be the opening statements
 Choice (3)
duces ‘Kenya’ and the maladies that because of ‘adults’, ‘sometimes’ in these
plague it. D follows as it states one 19. The choice indicates C or D as the first sentences which indicate a continuation.
of those maladies - poverty. The rest sentence. B has to follow A. Therefore, Therefore, we narrow down to two choic-
of the paragraph talks about ‘poverty’ either 1 or 2 is the right answer. D is a bet- es 2 and 3. C follows A logically. (smile
C follows D as it also talks about poverty. ter opening statement while C is a good …… happy-looking). Therefore, choice 3
E cannot follow D as it says ‘the conti- closing statement.  Choice (1) is the answer. Choice (3)
nents’, which is a reference to Africa and 20. The choices start with B or C. D cannot 32. Science is the general theme of the para-
not to Kenya. A and E form a pair as A follow B. Therefore, option 4 is ruled out. graph. “Inventions, Information technolo-

08_Section.indb 40 10/30/2009 11:52:47 AM


Solutions for Language Comprehension  n  41

gy and the Computer” are examples to the 5. ‘B’ states the subject. ‘D’ expands what option (2) is ruled out. Option (3) is more
general theme that science has brought a is stated in ‘B’ followed by ‘A’ and ‘C’ logical. In E, the word ‘they’ refers to
lot of changes in our life. Moving from which comment on the subject. . people. Hence, B can precede E. The idea
general to specific the correct answer  Choice (1) of moving slowly is further explained in
choice is 1. Choice (1) 6. Sentence ‘B’ describes the word ‘gloss- C. A is a perfect sequel to C. And D can
be the closing statement. Choice (3)
33. A cannot be the opening line because of ing’, followed by ‘A’ which adds to its
‘these words …… ‘ B too cannot open the meaning. D and C are linked by the word 17. A is the opening statement since it men-
paragraph because of ‘this is ….. ‘. This ‘meaning’. Choice (3) tions an event that is discussed in the
quotation must come first. Hence, choice 7. B raises a question. D and C follow as other sentences. C follows A – ‘it’ in C
2 which has the quotation in the begin- rhetorical questions. A concludes.Choice refers to the broken arm in A. Hence, we
ning is the right answer. Choice (2) (1) narrow down to options (1) and (2) only.
E explains the effect of the injury. D fur-
34. A, B and D cannot open the paragraph. 8. D follows B logically. A follows C as it ther explains it and B is a good closing
Hence, option 4 starting with C is the right answers the question raised in C. Choice statement. Choice (1)
answer.  Choice (4) (3)
18. The adjective ‘earlier’ in ‘D’ and the
35. D cannot be the first sentence of the 9. ‘B begins the para as it describes the sub- conjunction ‘but’ in ‘C’ state that these
paragraph. Also, the central theme of the ject. ‘A’ describes the man followed by sentences are continuations of ‘B’ which
paragraph is “genuine democracy”. All more description in D and conclusion in introduces the topic ‘textile industry’ in
the other statements follow this central C. Choice (4) Gujarat. Sentence ‘A’ is just an explana-
theme. Therefore, option 1 with A as the
10. Sentence ‘B’ sets the tone of the passage, tion of ‘B’ citing Ahmedabad, one of the
first sentence, is the right answer. Choice
‘A’ and ‘C’ describe the scene while D is cities of Gujarat, where the first compos-
(1)
a comment on it. Choice (1) ite mill was established in 1958. ‘Earlier’
36. Sentence ‘B’ sets the tone of the passage, of ‘D’ refers only to ‘dates even further
hence choices ‘2’ and ‘3’ can be elimi- 11. Only C can begin the paragraph. D fol-
back’ of ‘C’ which refers to ‘as early as
nated. A follows C ‘plumbed’ in A and lows – ‘it’ in D refers to ‘redecorating’ in
1958’ of ‘A’. Hence the correct sequence
‘downward’ in C. Choice (4) C. B follows linked by ‘But’. A logically
is 3. Choice (3)
follows B. Choice (2)
37. Sentence ‘B’ sets the tone of the passage. 19. The pronoun ‘they’ in ‘A’ and the open-
‘A’ precedes ‘D’ and ‘C’ since they elabo- 12. BD go together logically and linked by
ing words - ‘on the other hand’ and ‘on
rates on the ‘unthinkable’ in A.. Choice ‘But’. AC are linked by ‘Consequently’.
the one hand’ of the sentences ‘C’ and ‘D’
(3)  Choice (1)
respectively suggest that statements ‘A’,
38. ‘D’ tells us what the passage is about and 13. Sentence E introduces us to the idea of ‘C’ and ‘D’ cannot open the paragraph.
‘B’ summarises it. ‘D’ is followed by ‘C’, technological progress and economic Hence ‘B’ is the appropriate sentence to
B by ‘A’ as C and A answer the points growth. The result of this is given in sen- open the paragraph. Choice (4)
raised in D and B.  Choice (3) tence A. The effects of this growth are
20. ‘the atrocities’, ‘the all-Dalit village’, ‘the
given in sentences B and C. Sentence D
39. A raises a question which is reiterated in caste related clashes’ are nothing but the
concludes the passage. While B could
D. C and B answer the questions rhetori- references to ‘black chapter’ and ‘a re-
draw attention as a possible opening
cally.  Choice (2) mote village’ of ‘C’. Hence the opening
sentence, B and A have no link between
sentence is ‘C’ but not ‘B’. ‘It seeks legal
40. Sentence ‘C’ sets the tone of the passage. them and hence would be inappropriate.
proceedings’ of ‘D’ is a further explana-
The word ‘and’ in sentence ‘A’ is indica-  Choice (2)
tion of ‘particularly of the district’ ‘losing
tion that it is a continuation, followed by 14. Options (2), (3) and (4) are ruled out be- faith in the impartiality of police and the
‘B’ and ‘D’. B precedes D as it is linked cause B cannot follow D, nor can A come administration’. Choice (3)
to A in the question it raises. Choice (4) after D. Also, there is no logical connec-
21. D is the introductory sentence as it is gen-
tion between A and D. B can be the open-
Practice Exercise 8 eral in nature – it talks about a language,
ing statement. A can follow B because the
meaning any language. B follows as ‘such
theme remains ‘Russia’ and the ‘space
Solutions for questions 1 to 30: grammars’ refer to the ‘grammar’ of all
programs’. Also, the idea of the collapse
languages, ‘quite a number of languages’
1. AB is about patience. CD about rusting. of the Soviet Union is further explained in
is also general in nature. E follows as hy-
AB goes together as also CD. Choice (1) D. Hence, D comes after A and C follows
potheses ……, there is something else.
2. Sentence ‘B’ is the subject of the para- D. Choice (1)
C follows as here too’ would refer to
graph. ‘A’ ‘C’ and ‘D’ follow logically. 15. Statements C and D go together, because the new insight. Here ‘too’ there is ……
 Choice (1) the ‘them’ in D refers to the Britons who progress as the hypotheses first referred
3. A is the opening sentence as it introduces are not going out and spending money. D to are well-founded (B). A is the conclud-
the subject WTC. B, C, D follow giving and B go together because both of them ing sentence. It also talks about ‘universal
different features of the building. Choice deal with ‘the government’s take’ on the grammar’ mentioned in E. E and A are
(1) situation. Therefore option (3) is the an- linked and C is just an additional piece of
swer. Choice (3) information. Hence, DBECA is the right
4. Sentence ‘C’ states the subject of the para- sequence. Choice (3)
graph, followed by ‘A’ ‘B’ and ‘D’ which 16. Option (1) is ruled out because C and E
elaborate on it.  Choice (4) have no co-relation. C refers to a time 22. E is the introductory sentence as it talks
while E talks about people. Similarly, about what a corporation wanted to do.

08_Section.indb 41 10/30/2009 11:52:48 AM


42  n  Solution Manual

B follows as ‘the Beijing -based utility’ and C. D and F form a pair as they both Practice Exercise 9
refers to State Grid Corp. Now, the focus talk about the future of public education.
is on electricity. A follows as how it did F comes after D as it says ‘or do we ……. Solutions for questions 1 to 30:
good (by providing free electricity) is ‘, which means it is an alternative way of
1. The key points in the paragraph are
stated in it. D talks of the consequences looking at something. Hence, EACBDF
1. We are unique in some ways
or results of the project. C concludes the is the right sequence. Choice (2)
2. Our very uniqueness has landed us in
paragraph by proving a quote by the proj- 27. B is the introductory sentence - it talks the mess
ect chief. Hence, EBADC is the right se- about the mental status of a patient who 3. Over exploitation of resources has led
quence. Choice (4) is incapacitated and the question is - how to extinction of some species.
23. C is the opening sentence as it is a gen- can we assess. D follows as assessment
eral statement introducing the topic of is the main task talked about in the para-  he points are captured in Choice 2. Choice
T
discussion – connectivity, with special graph. The task is not impossible is what 1 and Choice 3 are not right because they
reference to ‘air’. E continues the idea. is stated. Hence, how to go about is elabo- say that extinction of species is 60 per cent
A follows as it takes what is said in E rated on, in the rest of the paragraph. E whereas as per the passage 60 per cent of
forward - it will be audible even if you which says - first of all …… comes first. the ecosystem services are being degraded.
do not want to listen to it. B follows as It talks about prejudicial notions. A which Choice 4 is wrong because the Millennium
it gives more details about ‘connectivity states what the notion is follows. F con- Ecosystem Assessment does not show that
during air travel’. D concludes the para- tinues the idea in A (notion). ‘He’ refers we are unique. Choice (2)
graph by providing an additional piece to the patient. C concludes the paragraph. 2. The main points covered are :
of information. Hence, CEABD is the Hence, BDEAFC is the right sequence. 1. Format change of a newspaper needs
right sequence. Choice (1)  Choice (1) consensus within the organisation.
24. E is the opening sentence as it is general 28. D is the opening sentence as the idea that 2. Managers must explain the need for
in nature. It helps us determine that the “birth order” determines personality traits change and the potential for growth.
tone of the passage, (negative). The fo- is introduced here. Also, age and sibship 3. The tone must be positive.
cus is - racial America. It states that it is size are mentioned. A follows as it also  hoice 1, 2, and 4 leave out the fact that
C
outdated. C gives another point against it takes about Shyness (reserved) and birth we are talking of a newspaper. Choice
and continues that idea. D and B form a order. E is an example to illustrate that 2 gives examples which are generally
pair. D talks about voting based on race age is related to shyness and lack of cour- avoided in summaries. Choice 4 leaves
and how it leads to separation. B follows age. B follows as it provides a reason for out valid information. Choice (3)
as ‘it’ in B refers to what is described in what is stated in E. F follows as it again
D. ‘Electoral’ in B is related to voting in talks about how age spacing determines 3. The main points covered are
D. B talks about how it fosters ideas of behaviour. C wraps up by saying _ _ _ 1. Immigration is the key issue in the
separation. A concludes the paragraph ‘the same age disparity or spacing _ _ pre-election scenario.
as it also talks about separation and also _ affects other traits also’. The right se- 2. People, including Labour party sup-
takes it forward by talking about its vari- quence is DAEBFC. Choice (2) porters, want tougher laws on immi-
ous repercussions. Hence, ECDBA is the gration.
29. B introduces the concept and hence is
right sequence. Choice (3)  hoices (3) and (4) do not mention the
C
the opening statement. B provides one
25. C is the opening sentence. ‘More’ in- reason for immigrants overtaking natives elections. Hence they are incorrect.
dicates that the realization is recent. B in the race. D which talks about another Choice (2) is factually wrong since it is
logically follows as it also talks about the reason is the continuation. A and F are a the supporters of the Labour party and not
‘new’ general agreement about the danger pair as both talk about self-employment. the party itself that wants tougher laws.
involved in the possession of handguns. E E follows as it says ‘also’, that is apart  Choice (1)
supports what is stated in B, by provid- form representing a major part of self-em- 4. The main points are:
ing evidence. A talk about the results of ployed people; ‘they also are…..skilled 1. France has suggested global tax to
the poll. Hence EA form a pair. D con- jobs’. C follows as it gives examples of raise funds to meet the Millennium
cludes the paragraph as ‘This’ refers to ‘highly skilled ……. and technical jobs’. Development Goals.
the change in opinion, which was talked Hence BDAFEC is the right sequence. 2. The developed and developing coun-
about in the rest of the paragraph. It is  Choice (1) tries agreed to a small tax on trade and
also linked to the surprising results of the air transport.
30. B is a general statement - it talks about the
poll …… ‘first time …..’. Hence CBEAD importance of a patron. It states that the  hoice 1, 2, and 3 do not mention what
C
is the right sequence. Choice (1) role is major. It is followed by D which are to be taxed. Choice 2 says it is the
26. E begins the paragraph by introducing the strengthens what is stated in B. F should only way, which may not be true. Choice
topic - the growing realization that the follow as it talks about how patronage is 3 does not mention France which is the
quality the public education is not good a part of even the political culture. It is key player. Choice (4)
enough. A continues as it talks about followed by A which gives a specific ex-
school - related news and how it is nega- ample. Now, E and C form a pair. E talks 5. The main points are:
tive in nature. C follows A as it talks about about the relationship between a patron 1. Rajiv Gandhi’s passion for modernisa-
another concern, when it comes to public and a beneficiary and C strengthens the tion of minds and attitudes.
education. B follows as ‘what then’, is in statement made in E. Hence BDFAEC in 2. The things he cared for are the chal-
the light of the concerns expressed in A the correct sequence. Choice (2) lenges facing Indian society.

08_Section.indb 42 10/30/2009 11:52:48 AM


Solutions for Language Comprehension  n  43

 hoice 1 can be discounted because its


C  hoices 1 and 4 make the mistake of in-
C 2. Large neighbouring countries usually
conclusion (he was passionate about the cluding Sykes among those that have lost have huge bilateral trade.
challenges facing Indian society) changes business to Asia Pacific region whereas 3. India and China with their huge popu-
the originally intended meaning. Choice Sykes is the call centre that has cut its work lation have a lot of scope for coopera-
3 leaves out the second main point while force in India. Choice 2 does not talk about tion.
choice 4 concludes wrongly that he tried the possible end of BPO in India. Choice 3
 hoice 1 misses out points 1 and 3. The
C
to find answers to the challenges facing captures the points accurately. Choice (3)
conclusion in Choice 2 (must increase
Indian society. Choice (2) 11. The main points are: trade as they also have huge population)
6. The main points are: 1. The liver has sensors that can detect is not in tune with the tone of the original.
1. Giving up smoking is not just about sugar and amino acids in the blood. Choice 3 captures the points. Choice 4
will power. 2. It sends signals to the brain through misses out point 1. Choice (3)
2. It may be genetically influenced. nerve cells.
3. Finding the gene will help in finding 16. The main points are:
3. The information contributes to the
the right quitting programme. 1. High number of air crashes.
feeling of hunger.
2. VIP deaths have not changed safety
 hoice 2 leaves out the 3rd point Choice
C Choice 1 captures all the points standards.
3 is wrong because it says nicotine de- Choice 2 omits point 1 3. Crashes in general aviation are far
pendence cannot be overcome. Choice 4 Choice 3 omits point 2 more than those in civil aviation.
is totally skewed. Choice (1) Choice 4 omits point 1 Choice (1)
 hoice 1 is wrong as it says air crashes
C
7. The main points are: 12. The main points are:
have been increasing which is not stated
1. Decision of the government of EU to 1. The Manas river enters India at
in the passage.
review the Bolkestein directive. Mathanguri and splits into two.
Choice 2 captures all the points.
2. What the Bolkestein directive means 2. It has a variety of fish depleted through
Choice 3 omits point 3.
3. The decision is a victory for social poaching.
Choice 4 omits point 2. Choice (2)
model against liberalisation. 3. It is the source of water for the fauna.
17. The main points are:
 he points are captured in Choice 3.
T Choice 1 does not include point 3. 1. India’s security policies have changed
Choice 1 leaves out point 3. Choice 2 Choice 2 misses out point 2. as a response to change in environ-
does not explain the Bolkestein directive Choice 3 is too long ment.
and Choice 4 does not talk of the success Choice 4 is appropriate. Choice (4) 2. External factors – end of cold war,
of social model. Choice (3)
13. The main points are: collapse of Soviet Union, emergence
8. The main points made here are: 1. Subir Raha is ONGC’s Chairman. of USA as the sole superpower.
1. You can not argue with a person whose 2. ONGC is India’s largest profit making 3. Internal factors – economic and politi-
belief is based on faith. company and largest in terms of mar- cal crises.
2. Science establishes truth by appealing ket capitalisation.
 hoices 1 and 2 leave out the changes in
C
to evidence and logic. 3. He has moved from control to free
the internal conditions. Choice 3 misses
market economy.
 hoice 2 captures the point. Choice 1 says
C out point 2. Choice (4)
4. He has made a global presence through
faith will lead to heinous crimes, which is
acquiring property in other countries 18. The main points are:
wrong. Choice 3 does not talk about sci-
under its subsidiary, ONGC Videsh. 1. Per capita-related development is
ence. Choice 4 says only a person with
linked to level of literacy.
faith commits crimes, which is wrong. Choice 1 leaves out point 4. 2. India has 200 million adult illiterates.
 Choice (2) Choice 3 misses out point 3. 3. The development of computers based
9. The main points are : Choice 4 does not include point 4. functional literacy methods to tackle
1. Claymation has been around for a cen-  Choice (2) the problem has given positive re-
tury. 14. The main points made are: sults.
2. The winning of Oscars brought it to 1. Spirituality is no longer a taboo in the
prominence.  hoice 1 fails to cover point 2 and the fact
C
world of science and medicine.
3. It has been in India for 6 years and has that the new method has given positive
2. A preliminary study says belief in god
a long way to go. results. Choice 2 talks of the ‘short com-
may improve a person’s health
ings’ of the missions which is not men-
3. The involvement of a major university
 hoice 1 is wrong as it says India has not
C tioned in the passage. Choice 4 misses
shows the importance of spirituality.
achieved quality in claymation. Choice point 3. Choice (3)
2 leaves out what the scenario in India.  hoice 2 is too long, misses out point 2
C
19. The main points are:
Choice 3 wrongly says that the BBC film and states erroneously that universities
1. Industries cause pollution.
was on claymation. Choice 4 captures the are ‘prepared’ whereas they are already
2. India does not have comprehensive
points appropriately. Choice (4) into it. Choice 3 is wrong in saying that
laws on environmental protection.
link between belief in God and good
10. The paragraph raises the question: Is the 3. Nevertheless companies have woken
health has been established. Choice 4 has
end of the BPO industry near at hand? As up to the need to do their bit:
a flippant tone. Choice (1)
evidence it points to
1. Absenteeism and high attrition that 15. The main points are:  hoice 1 fails to mention that companies
C
have always troubled the industry. 1. India and China are two of the fastest are doing what they can to reduce pollu-
2. Even reputed BPO losing business. growing economies of the world. tion. Choice 3 makes it more specific to

08_Section.indb 43 10/30/2009 11:52:48 AM


44  n  Solution Manual

SAIL whereas the original is generalised  nly choice (1) has these points. Choice
O incorrect because it says that coalitions
with SAIL given as an example. Choice 4 (2) is wrong because it says capitalism were unknown in the past. Choice (4) is
is wrong as it says companies do not both- physically a minority, which distorts the incorrect because it does not mention the
er about their bottom lines. Choice (2) meaning of the passage because capital- fact that coalition formations are helpful
ism as a concept cannot do that. Choice in monitoring vital issues pertaining to
20. The main points are:
(3) is ruled out as it leaves out important the stability of a nation. Choice (1)
1. Expenditure on education – quantity
points. Choice (4) is also wrong because
versus quality. 28. Main points – it is man’s attitude that
it says the means of production help a
2. Spending not reaching the intended makes success possible by enabling him to
minority to control others but the passage
beneficiaries. use the power of the mind. Choice (1) is
says it is minority’s capture of the means incorrect because it repeats the word ‘atti-
 hoice 1 covers the points. Choice 2 does
C of production that helps. Choice (1) tude’, three times and fails to mention im-
not mention quality, says it is only wast- 24. The most important points expressed in portant details like the superiority of man
ed. Choice 3 quotes figures, which are not the passage are: due to his thinking abilities. Choice (2) is
required in a summary. Choice 4 misses 1. Real India does not any more reside in incorrect because it distorts the meaning of
on some points. Choice (1) villages, it resides in cities. the original text by saying that the passage
21. The given passage has the following im- 2. Cities offer many opportunities and a depicts the partial nature of God. Choice
portant points: variety of attractions. (3) cannot be called a best summary be-
1. Dacher Keltoer is specialized in the 3. Urban India contributes most to the na- cause it mentions the words caste, creed
study of facial expressions. tions’s GDP. and race, which are redundant after “all
2. A smile is one of the most useful tools successful people”. Choice (4)
 he focus of the passage is essentially on
T
of human behaviour because it helps real India residing in cities now. Only choic- 29. Main points – Bullying is unethical but
build kinship, strengthen social rela- es (3) and (4) reflect this idea. Again choice is increasingly being seen as appropriate
tions and makes one feel good. (4) can be eliminated because it has repeti- behaviour in some business situations.
tive statements. All the above mentioned Choice (1) is incorrect because it distorts the
 nly choice (1) has these points. Choice
O
points are focused in option (3).Choice (3) meaning of the original text by mentioning
(2) distorts the meaning of the passage as
25. Option (1) does not qualify as the best that bullying in commercial establishments
it says that we can build kinship because
summary because it mentions trivial de- is justified. Choice (2) is incorrect because
a smile releases positive brain chemi-
tails and examples. Choice (2) cannot it states that bullying one’s entire staff is
cals but the passage says it helps release.
be the best summary because it does not better than victimising one or two persons,
Choice (3) cannot be a summary because
mention that doctors are unable to extend which is incorrect. Choice (4) is incorrect
it is verbose; choice (4) is also ruled out as
the best treatment to all because modern because here it is erroneously stated that
it leaves out the aspect that smile makes
medicine is expensive and best treatment bullying is permissible in commercial
one feel good. Choice (1)
cannot be available to all due to paucity establishments. Choice (3)
22. The important points that the passage of funds. Option (3) is not a best summary 30. Main points – workers have always been
deals with are: because it mentions that the best treat- treated poorly and this continues in some
1. Realty is thriving in Singapore ment is available to only those who are Third World countries. Choice (2) cannot
2. Millionaires are queuing up to buy wealthy, thereby distorting the original qualify as the best summary because of
high-end properties. passage.  Choice (4) the emphasis on words like capturers, ven-
3. Of them, Indian millionaires are be-
26. Main points - different countries and re- dors, etc., and it is also distorted. Choice
coming dominant with their rising
gions grow different things that are used (3) is incorrect, because it focuses only on
contribution.
together thus leading to interdependence. slavery and fails to capture the other ideas
 hoice (2) has these points. Choice (1)
C Choice (2) does not qualify as the best in this para. Choice (4) is incorrect be-
is wrong because it says millionaires are summary because examples have been cause it distorts the meaning of the text by
“queuing up… themselves” but the pas- used. Choice (3) is incorrect because the mentioning that employers shared a close
sage says that they also hire people to buy meaning of the text has been distorted rapport with their employees. Choice (1)
properties for them. Choices (3) and (4) here, since it has been erroneously men- conveys the idea completely. Choice (1)
are ruled out because it is not clear wheth- tioned here that nature has been ‘partial’.
er ‘contrary to…’ refers to queuing up or Choice (4) cannot be considered as the
acquiring high-end properties.Choice (2) best summary because the main point, Practice passagEs
23. The important points in the passage are: which is ‘interdependence’ has been men-
tioned in terms of trade. Solutions for questions 1 to 5:
1. Production and distribution are basic
economic activities.   Hence, choice (1) is the best summary. 1. The author mentions the incident of his
2. They change their institutional features  Choice (1) going into Manchester with a bourgeois
under the changing social conditions. 27. Main points - coalitions were earlier to give an example that the bourgeois is
3. Under feudalism, agriculture was the considered incongruous. However they motivated by self-interest and money
most dominant productive activity and are now viewed as something that can gain. [Refer para 2, line 12-13]. Choice
distribution played more important provide stability. Choice (2) does not (2)
role than ownership over land. qualify as the best summary because 2. It is unlikely that an English bourgeois
4. Under capitalism, a small section of it only mentions that coalitions help will set up a charitable asylum and run
the people capture means of produc- monitor vital issues but it fails to men- it with his own money. [Refer para 2
tion and control others. tion what those issues are. Choice (3) is line 3-5]. Choice (1)

08_Section.indb 44 10/30/2009 11:52:48 AM


Solutions for Language Comprehension  n  45

3. The author is averse to those who are 16. In the passage, the author quotes the defi- is the wrong answer as it does not address
interested in making money alone. nition to poetry as many as twenty times. the question. It is the reason why there is
 Choice (3) Majority of the times he comes up with a deviation from the classification. Option
an expression of disapproval placed ei- (4) is ruled out as it merely adds to option
4. The bourgeois is likely to turn the power
ther before the definition or after. Hence, (2). The word ‘gains’ renders option (1)
of the government against the proletariat
option 2 is the correct one. Choice (2) irrelevant. Choice (3)
to keep the latter out of its way. [Refer the
last line of para 2]. Choice (3) 17. At the beginning of the second paragraph 28. Refer to the first para. It can be understood
of the passage, the author cautions the that option (1) is the answer choice (with
5. The tone of the passage is likely to appeal
reader not to get swept away by the defi- which the author disagrees). Though op-
to a philanthropist. Choice (1)
nitions given by various people. Choice tion (3) also seems to be correct, it is only
Solutions for questions 6 to 10: (2) as a consequence of the belief that “man-
kind can survive….” and is not the focus.
6. According to the passage leasing out un- 18. At the very beginning of the second para-
Hence option (3) is ruled out. Option (2)
used land was strongly resisted by vari- graph of the passage, the author says
is what the author believes and here, he
ous government departments which led to “Apart from such formal definitions. . .”
disagrees with Eugene Rabinowitch Op-
further problems for the Delhi Transport This should mean that all the definitions
tion (4) is again the result of option (2)
Corporation. [Refer para 6]. Choice (3) mentioned in the first paragraph of the
 Choice (1)
passage are formal ones. Therefore the
7. The author’s experience taught him that
correct choice is 3. Choice (3) 29. Refer to the last sentence of para 3, sen-
essence of financial rectitude involved in
tence 4 of para 4 and the third sentence of
distancing oneself from any plan that may 19. Option 1 ‘excitedly’ is acceptable, as
the last para. They point to option (4). The
conceivably benefit private entrepreneur. hysterical refers to a state of uncontrolled
other options distort the author’s view.
[Refer para 3]. Choice (3) excitement. Choice (1)
 Choice (4)
8. The author is referring to the family silver Solutions for questions 20 to 25: 30. Sentence 3 of the last para renders op-
of the public institutions. [Refer para 4].
20. It is understood from para 3 that option 3 tion (1) and (4) and sentence 4 of para
 Choice (1)
is the answer choice because it means that (4) renders option (2) correct. The author
9. According to the passage, when a pub- within the society rules change from time would not agree with only option (3)
lic system suffers from financial cri- to time option 1 is out of context. So are  Choice (3)
sis, the situation calls for painless and options 2 and 4. Choice (3)
quick disposal of all white elephants. Solutions for questions 31 to 35:
21. Refer to para-5. It renders option (4) cor-
 Choice (3) 31. Refer to the first sentence of para-5
rect. Choice (4)
10. The author suggested that they have an  Choice (2)
agreement with the private operator al- 22. Refer to the last sentence of para 2 and the
32. It is understood from para 4 that both op-
lowing him to use the owned surplus ma- first 2 sentences of para 3. These point to
tion (1) and (2) are necessary for an in-
chines in exchange for price concession. option (3). Choice (3)
dustry to achieve more. (i.e its growth)
[Refer para 1]. Choice (1) 23. Through out the passage the author tries But as the choices do not have both fac-
to analyse the sense of right and wrong tors, the question then obviously, refers
Solutions for questions 11 to 14:
prevailing in society. Choice (4) to achievement in terms of efficiency if
11. Refer to the first sentence of para -4. In the not in terms of more production and less
24. Refer to the first and last sentence of para
context, to safeguard’ means storing the usage of raw materials. Refer to the first
7 On the tablets of stones are the objec-
seeds so that they will come handy in case sentence of para 5-Efficiency can only
tive morality that has been practiced for
of a disaster. This is brought out in option be improved, if option (3) takes place.
generations. The word ‘traditions’ in op-
(1), Option (4) is incorrect. The purpose is  Choice (3)
tion (1) brings that out. Choice (1)
to ‘safeguard’ and not to ‘experiment.’ The
25. In the sixth paragraph of the passage 33. Refer to the first sentence of the last para.
words ‘to control’ in option (2) renders the
the author himself states that he is from Choice (1) is the answer. There can be
first sentence of para (4) incorrect. The pas-
Europe. Hence, option 3 is the correct economy with resources and production
sage states that the more the diversity of the
choice. Choice (3) as well but though true, the focus of the
plant life, the better it is in different ways.
passage is on ‘growth’. Option (1) is stat-
Option (3) is incorrect for the same reason.
Solutions for questions 26 to 30: ed in the passage. Such an economy need
 Choice (1)
26. Refer to the last sentence of the passage not necessarily be successful. In order for
12. Refer to para (4). Option (3) is a distor- an economy to be successful, growth is
of the last para.
tion of ‘resistant to diseases’. Choice (3) necessary. Choice (1)
It implies that man is inclined to treat
13. Refer to para (7). Because of plausible things of his own making in a casual way. 34. Though it can be understood from the
climate changes, the vault for storing In this context the first sentence of the passage that option (1) and (2) together
seeds is necessary as there might be an last para clearly points out that land too are true, the given options do not provide
extinction of plant life. Choice (1) cannot be treated similarly. This makes for such a choice. Refer to the last sen-
14. From question (4), it can be understood choice (3) as the only correct option. Op- tence of para 3. Option (3) is stated and is
that option (3) is the closest. Choice (3) tions (1) and (2) are against the author’s the answer choice. Choice (3)
views. So is option (4) Choice (3)
35. While option (1), (2) and (4) are implicit
Solutions for questions 15 to 19: 27. Refer to sentences 2 to 5 of para (4) which and most definitely true, they are not the
15. Refer to paragraph one Choice (4) means the same as option (3). Option (2) focus.

08_Section.indb 45 10/30/2009 11:52:49 AM


46  n  Solution Manual

 efer to the first sentence of para 5. The


R Solutions for questions 41 to 45: possible. Option 2 is polar opposite of the
focus of the passage is that the most 41. Refer to the first sentence of the passage. requirement. Option 1 distorts the informa-
advanced economy (that of US) is inef- Option (1) is the correct answer choice. tion given in the passage. Option 3 is un-
ficient because it has failed to make the Option (2) is not the focus of this passage. related to the context of the question. The
population happy, to guarantee its well- Though the disadvantages of dieting have correct choice is 4. Choice (4)
being, etc. Refer to the word “behind” in been mentioned they are not high lighted. 52. In the second paragraph, the author de-
the inverted commas which means that This rules. The author only explains the scribes rhythm of socialization to be a
the 94.4 percent of mankind are at least in urge to over-eat after a period of dieting process in which an individual changes
some other aspects if not the materialistic but does not support it. This rules out op- from being a supporter of individualism
aspect, better. Choice (3) tion (4). Choice (1) to becoming a part of the society support-
Solutions for questions 36 to 40: 42. Option (1) is rendered incorrect by the ing social solidarity. Option 3 captures
third and fourth sentence of para (2). The this correctly. Choice (3)
36. It can be understood from the passage
that options (1) and (3) are cited to show last sentence of para (2) renders option 53. The phrase ‘two movements’ is men-
that we have not understood cognition (2) incorrect. Hence, Option (4) is accept- tioned in the third paragraph. It refers to
to its full extent. This renders option (2) able. Option (3) is irrelevant. Choice (4) the movements of individualism and so-
the correct answer choice. Option (4) in 43. The last para and Keys’ experiment both cialization mentioned in the immediately
itself is not a problem rather it has caused point to option (2). Once dieting period preceding paragraph. Therefore, the cor-
dilemmas which, in order be to the re- is over, the over-eating or the yo-yo syn- rect choice is 2. Choice (2)
solved, needs further information. The drome sets in because of the physiologi- 54. The role of the fourth paragraph is to
author says that this will happen in half a cal and psychological changes. The last prove that science arises out of practice.
millennium from now. Refer to sentence but fourth sentence of the last para ren- Choice 1 is the correct choice.Choice (1)
last but 3 of the passage. Choice (2) ders option (4) incorrect. Choice (3)
55. In the fifth paragraph the author mentions
37. Refer to sentence 4 and 6 of para-1. Op- 44. In anorexic patients, diet restriction is self that a given individual is convinced about
tion (1) is the answer choice. Option (2) is – imposed. The last line of the 4th para, the facts formulated through his/her ex-
ruled out as the author cites option (2) as ‘These had been submerged .............. self perience. But unless they are tested and
an example to explain option (1). Option imposed. ‘renders option 1 to be correct. judged to become generalizations, they
(3) is ruled out as nowhere in the passage  Choice (1) are not valid. The author explicitly men-
is the author confused about mind and
45. Both A and B are stated in the fourth and tions in the last sentence of the paragraph
matter. Both are clearly distinct. Option
fifth sentences of para (1). C contradicts as to what is required to be done to get the
(4) is actually option (2). The cause of
the fifth sentence and so is incorrect. The prestige of generalization to one’s conclu-
the ‘failure’ mentioned in option (4) is
word ‘permanent’ renders sentence D sions. Option 4 talks about the process.
because of option (1). Choice (1)
incorrect. Thus option (3) is the right an-  Choice (4)
38. It is clear from the passage that the con- swer choice. Choice (3)
flict is between ‘mind’ and matter, that Solutions for questions 56 to 63:
is whether both can coexist. Refer to Solutions for questions 46 to 50: 56. Refer to the second sentence of para one.
sentence 6 of para 1. This renders option 46. According to the author the lesson that ’…… a scientist working anywhere…….
(3) the correct choice which means that excellent companies can teach their em-  Choice (2)
a robot, being an object of matter, would ployees is to make them feel that they are
57. Refer to the third sentence of para one.
be able to perceive, that is be conscious. winners. [Refer para 2, line 3].Choice (2)
‘In addition to…… detailed genom-
 Choice (3) 47. In the passage the author takes an ex- ic………’. Choice (5) is wrong because
39. Refer to sentence (3) of para-2. They ample of employees solving puzzles to it helps in predicting the functions of pro-
could not comprehend because earlier, prove that employees are motivated by a teins. Choice (1)
their knowledge was limited in the first self-perception that they are indeed doing
place (in the form of Aristotelean philoso- 58. Paragraph 4 suggests that there are no
well. [Refer para 5]. Choice (3)
phy) and also because knowledge did not means of preventing the use of genetic
48. The reason for poor performance accord- data to create novel pathogens.  Choice
change as fast or as much as it did in the
ing to the author lies in the designing of (4)
later decades. This is aptly expressed in
systems in a manner which demoralizes
only option (5). Choice (4) 59. Refer to the first sentence of para three.
the employees. Choice (3)
40. The last sentence means that the dilemma  Choice (3)
regarding Data will, by then, become a 49. According to the author our barest emotions
60. Refer to the last sentence of para 4.
moral issue. Hence the dilemma of a dif- are attributed to an underdeveloped right half
 Choice (4)
ferent nature, that is one of moral aspect of the brain. [ Refer para 7]. Choice (1)
61. Refer to the last sentence of para 4 and
will arise. This is expressed in option (3). 50. According to the passage, attribution
the first sentence of para 5 which led to
Option (1) and (2) are stated in the pas- theory attempts to explain the manner
the commissioning of such U.S. agencies.
sage. Refer to sentence (7) and (8) of para in which we assign causes for success or
 Choice (2)
(2) respectively. With further change in failure. [Refer para 4]. Choice (2)
knowledge, the issue will not disappear 62. Refer to para 5 which says that it was
Solutions for questions 51 to 55: established to advise the government on
but in half a millennium, it will either be
supported or dismissed. Option (1) and 51. In the last sentence of the first paragraph technical issues. Choices 2 and 3 can be
(2) are actually extentions of option (4). the author explicitly mentions as to what eliminated as it says it would ‘convene
 Choice (3) is necessary to make a transition period a scientific panel to evaluate and recom-

08_Section.indb 46 10/30/2009 11:52:49 AM


Solutions for Language Comprehension  n  47

mend but not directly ‘evaluate the risk Solutions for questions 76 to 80: variable interest rate investments. [Refer
and recommend policies’. Choice (1) 76. The author is of the viewpoint that the para 9]. Choice (4)
63. Choice 1 is true. Refer to the first sen- variations in societies or individuals in Solutions for questions 89 to 96:
tence of the last paragraph. Choices 3 and the visible physical features are not a sure
4 are true according to the second and guide to differences in ancestry. In other 89. According to the passage rhythm is the
third sentences of the last paragraph. Ac- words, racial classifications are a myth. creative force in the hand of the artist.
cording to the penultimate sentence of the Thus options (1) (2) and (3) are complete- [Refer para 3]. Choice (2)
passage, choice 2 is not true. Choice (2) ly contradictory. Only option (4) supports 90. According to the passage all the state-
the above view. Choice (4) ments mentioned (1, 2 and 3) are true.
Solutions for questions 64 to 70: [Refer para 2, 4]. Choice (4)
77. Refer to the first sentence of para (5). The
64. The degradation of natural resources will phrase ‘biologically alien’ means those 91. Statements 1, 2 and 3 are all true state-
lead to poor economic utilization of re- societies or individuals who do not have ments. Choice (4)
sources. [Refer para 1]. Choice (1) a common ancestor and it refers to races, 92. The difference between a king and his
65. According to Neeri seventy percent of total thus, it means that race was usually not subjects is in the different metres of their
water available in the country is polluted. the reason for any antogonism between situation and circumstances. [Refer para
[Refer para 2]. Choice (2) political parties or religions. Choice (2) 2]. Choice (3)
66. Municipal sewage pollutants account for 78. Refer to the first sentence of para (4). Op- 93. ‘The meaning of rhythm’ is an appropri-
70 per cent of Ganga’s water pollution tion (4) is not the causative factor for cul- ate title. Choice (3)
load. Choice (2) tural variations. Choice (4)
94. According to the author, life is an inces-
67. All the three statements are correct state- 79. The reason for strife between religions sant explosion of freedom. Choice (1)
ments. Choice (4) or between political bodies in the past
95. When a rose is pulped the image is lost.
68. The cost of the Clean-the Ganga Popula- could be because of culture and tradition.
 Choice (1)
tion Project Action Plan is likely to be  Choice (2)
96. In one’s effort to capture life as expressed
sourced from the US, U.K, Netherlands, 80. According to the passage, cultural varia-
in living tissue we literally find carbon,
Poland, France, the World Bank and India. tions did not arise from physical inheri-
nitrogen and many other things utterly
[ Refer para 7]. Choice (4) tance from their ancestors. [Refer para 4].
unlike life, but never life itself. [Refer
69. Both statements 1 and 2 are correct.  Choice (4)
para 1]. Choice (4)
[Refer para 6]. Choice (3) Solutions for questions 81 to 88:
Solutions for questions 97 to 100:
70. Option (3) since the point the author is 81. As per the passage, the Indian industry so
trying to make is best encapsulated here. 97. The hard work put in to meet security
far has thrived in a protected environment.
 Choice (3) need is short lived because once the se-
[Refer para 2]. Choice (1)
curity need is satisfied, employees tend to
Solutions for questions 71 to 75: 82. It is true that globalisation of economy relax. [Refer para 1]. Choice (4)
would bring in frontal global competition.
71. Option (3) since the author is clearly 98. The achievement need of employees be
[Refer para 3]. Choice (1)
sounding a warning and is not ridiculing, satisfied by providing autonomy to them.
accusing or pointing fingers. Choice (3) 83. Investment decisions should be based on [Refer para 2]. Choice (3)
quality of management, productivity of 99. Choice (3) is the best one. Choices (2)
72. ‘The perils of interdependence’ would
organization, innovation of organization. and (4) are tangential. While choice (1)
be apt as the author is trying to explain
[Refer papa 4]. Choice (4) is tempting, the actual reason is clearly
the dangers that accompany ‘interdepen-
dence’. Choice (2) 84. Of the companies given in the passage, mentioned in (3) alone. Observe lines 5
the productivity level of a worker is high- and 10 of para 1. Choice (1)
73. Option (4) since the author (according
est in Toyota. [Refer para 5]. Choice (4) 100. The best choice is (2) Line 3 – 5 in para 2.
to the passage) does not agree with any
of the statements that are mentioned. 85. It is not true that the choice available to  Choice (2)
 Choice (4) investors in terms of kinds of instruments
Solutions for questions 101 to 105:
available in capital and money markets
74. Sentence 2 and 3 of para (4) point to op-
could get limited. Choice (3) 101. The best choice is (3). The last 2 lines in
tion (2) as the answer choice. It can be
para 3 bear evidence to this. Choice (3)
understood from the passage that option 86. The number of components produced in
(1), (3) and (4) are incorrect. Choice (2) house by GM and Toyota is more than 50 102. The best choice is (1). Observe the lines
per cent and 25 per cent respectively. [Re- 6-10 of para 4. Choice (4) contradicts it.
75. The historical fact is that people did not  Choice (3)
fer para 6]. Choice (3)
in the past perceive themselves as dif-
ferent from others. The view was that of 87. Indian investors have always had the 103. The best choice is (4). Read Lines 2 and
universal brotherhood or origin from a fancy for companies manufacturing im- 3 in para 1 and lines 1 and 2 in para 2.
common ancestor and that the differences port substituting products. [Refer para 3].  Choice (4)
were only geographical. The author has  Choice (3) 104. The significance of Einstein’s special
cited options (1), (2) and (4) to explain 88. Money markets will see changes in principle of relativity is that there are lim-
the above fact. Only option (3) is a dis- terms of introduction of credit risk re- its to which experimentation can be used
tortion of sentence of the last of para (4) lated debt investments, more challenge to understand some physical phenomena.
 Choice (3) entering the market, introduction of [Refer para 4]. Choice (3)

08_Section.indb 47 10/30/2009 11:52:49 AM


48  n  Solution Manual

105. The statement ‘New knowledge about of institutions and the faith and trust that Solutions for questions 134 to 140:
natural phenomena builds on existing they can generate among the people. 134. According to the passage anything self-
knowledge’ captures the theme of the  Choice (1) referential is paradoxical. [Refer para 5].
passage. Choice (4) 120. The capital of Vijayanagar illustrates that  Choice (2)
Solutions for questions 106 to 110: it takes centuries to recover from a war. 135. From the passage it can be inferred that
 Choice (1) to understand a system, we should
106. A computer virus spread infection when
it actually replicates itself on to other pro- 121. The author does not believe in revolution investigate both within and beyond its
gramme disks or memory. [Refer para 3]. because it is not a tea party. Choice (1) boundaries. Choice (4)
 Choice (1) Solutions for questions 122 to 125: 136. According to the Humpty Dumpty one
107. All antivirus programmes look for codes imparts to a word the meaning one in-
122. Third generation robots are characterized tends. Choice (3)
with suspicious patterns, cannot offer by built-in intellectual capacity. Choice
complete protection and need to be up- (4) 137. According to the passage when some
dated continuously. Choice (4) people say ‘bad’, they mean the opposite.
123. ‘Artificial intelligence’ is about under-  Choice (1)
108. A suspicious looking code can be called standing the situation and reacting suit-
a virus only when it carries instructions ably. [Refer 5th paragraph] Choice (2) 138. According to the passage, words are
that copy the virus code to another host. packets of meaning. Choice (2)
[Refer para 1]. Choice (1) 124. From the passage we can conclude that
mechanical arms are a part of any genera- 139. Crawling/walking by a kid was used by
109. We may unknowingly activate camou- tion of robots. [Refer para 2]. Choice (2) the author as an analogy to counter some-
flaged virus along with the mail when we body who may say “why give the word a
open one of our email messages. [Refer 125. The mechanical arm of a robot is com- meaning at all if accepting it is suspect”?
para 3]. Choice (2) parable to a human arm, has a grip-  Choice (2)
per which most probably does work
110. A proactive approach is better because similar to that done by our fingers., has 140. The author meant it as a paradox. Choice
no antivirus programme can be termed many joints, each of which is provid- (4)
successful even in eliminating the listed ed with an independent motor. [Refer
viruses completely. Choice (4) Solutions for questions 141 to 145:
para 2]. Choice (4)
141. A primary motivation for the economic
Solutions for questions 111 to 115: Solutions for questions 126 to 133: cooperation with the EC is Europe’s de-
111. The author preaches truthfulness. Choice 126. If the managing agents handled firms cline in world economic standing and fear
(1) and industries on the companies under of conflict among its member nations.
112. The belief that life is all about material- their management the companies will be  Choice (3)
ism is a lie. Choice (4) milched and exploited to the greatest ex- 142. The most important outcome of the Single
tent. [Refer para 4]. Choice (3) European Act is likely to result from lower
113. The most important task to be done as we
start on our spiritual journey is to try be- 127. ‘Going public’ means to become public costs, improved efficiency, exploitation
ing true to our own higher nature.Choice limited. This is to rise funds from the gen- of economics of scale, reallocation of re-
(1) eral public in the open market.Choice (1) sources and innovations. [Refer the last
paragraph]. Choice (4)
114. It can be found in Mahanirvana Tantra 128. Carefully investigating the viability and
that truthfulness is the highest moral prospects of new projects require ‘origi- 143. The sense of mission of the European
virtue. Choice (3) nation’ according to the passage. [Refer Community is expressed by activities in
para 6]. Choice (4) many other fields, concern for welfare of
115. The author accuses politicians of lying and disadvantaged people , concern for the
cheating, business sector of justifying being 129. According to the passage ‘private place- needs of future generations and creation
untrue as a way of making profits and the ment’ means not offering the issue to the of new programmes. Choice (1)
tendency towards materialism of meaning general public for subscription, but plac-
as away from the spiritual path. Choice (4) ing it privately with big financiers. [Refer 144. The European Community felt that the
the last para]. Choice (2) best way to tackle its economic problem
Solutions for questions 116 to 121: was to integrate the vast resources of its
130. Providing liquidity to existing securities member countries. Choice (4)
116. The central thrust of the passage lies in the is not a function of the primary market.
belief that India is gearing up for a new 145. The economic standing of the European
 Choice (4)
awakening. [Refer para4]. Choice (2) Community has been in decline for most
131. The passage confirms that managing of the early 1980s compared to that of the
117. According to the passage, it is seldom un- agencies are owned by both European and US and Japan. [Refer para 1]. Choice (4)
derstood that development is difficult to Indian business houses. Choice (3)
manage. Choice (4) Solutions for questions 146 to 150:
132. According to the passage, the distributive
118. The idea of a splendid Indian past is a 146. The main reason for promoting renew-
capacity of stockbrokers normally deter-
heady intoxicant that appeals to Indians able and alternate sources of energy in
mines the extent of putblic participation
but is not firmly grounded in historical India is the severe impact of oil prices on
in new issue. [Refer last para].Choice (2)
realities. Choice (1) the economy. [Refer para 1]. Choice (3)
133. It is not true of a rights issue that it closes
119. The author is of the view that civilized 147. The central issue discussed in the pas-
the way for companies to convert some of
behaviour is a function of the emergence sage is that EDA should market solar
their reserves to their capital. Choice (1)

08_Section.indb 48 10/30/2009 11:52:49 AM


Solutions for Language Comprehension  n  49

cookers in rural areas rather than in urban subject and gives extra information on the 17. ‘Xenophobia’ is fear of ‘foreigners’. ‘Py-
areas. Choice (2) subject. Choice (4) rophobia’ is fear of ‘fire’. The words in
the pairs (2), (3) and (4) do not match
148. Rural households are more likely to use 8. Option (2) can be ruled out with reference
correctly. Choice (1)
solar cookers because they are not pres- to paragraph 3, last sentence Option (3) is
surized for time and there is plenty of ruled out since Devils rely on communal 18. A ‘Geologist’ studies the ‘earth’s crust’ and
private open space with direct sunshine. scavenging, the spread of the disease can- an ‘ornithologist’ studies birds. The relation
[Refer the last para]. Choice (1) not be curbed. Option (4) is also a distor- is that of a professional to the subject of
tion since ‘dubbed himself’ means ‘called his study. ‘Science’ is the subject of study
149. Carrying of food utensils around is a ha-
himself’. Choice (1) can be inferred as the of a scientist but it is too broad in context.
bitual practice in rural areas because the
correct choice from paragraph 1, 5 and 6.  Choice (4)
kitchen is detached from the living rooms.
[Refer last para]. Choice (2)  Choice (1)
Solutions for questions 19 to 22
150. The author recommends that EDA should 9. A ‘Veterinarian’ cannot be a potential 19. ‘Cognizance’ means knowledge or aware-
target rural agricultural households for reader since the reference to the disease ness. The phrase ‘take cognizance of’
marketing solar cookers. Choice (2) is just a subset to the main idea in the means to take note of. Hence, choice (2)
book. Option (2) can be ruled out since is the answer. The other words given in
Practice tests the author himself remarks that the book the choices are inapt.  Choice (2)
has ‘far wider appeal’ in the second para-
20. ‘Accentuated’ means made prominent.
graph. Choice (3) is just a reference and
TEST PAPER 1 ‘Contour’ means an outline, esp. repre-
is not relevant to the readers of the book.
Solutions for questions 1 to 10: senting the shape or form of something.
Archeology does not relate itself to living
 Choice (4)
1. Referring to the first paragraph we can things. Option (4) is the correct choice
since ‘Natural History’ refers to an ac- 21. The underlined word ‘alleviation’ means
deduce that the choice (1) is the answer.
count of the plant and animal life in a par- reduction and it conveys the meaning of
Choice (2), (3) are the consequences of
ticular place. Choice (4) the sentence correctly. ‘Destruction’ and
their success in business. Choice (3) is
‘prevention’ do not collocate with ‘pov-
out of the context. Choice (1) 10. Option (1) can be inferred as true from erty’. Banks cannot eradicate ‘poverty’.
2. While choice (1), (2) and (3) portray the paragraphs 1, and 6, option (2) from 1,  Choice (1)
contrast, choice (4) talks about ‘the way’ 5 and 6, option 3 from 3, Option (4) is
22. ‘Niche’ means a comfortable or suitable
which is a distortion of information. distorted since Tasmanian aboriginals are
position in life or employment. This is
 Choice (4) not alive anymore. Choice (4)
the most appropriate word in the context.
3. From the first and last lines of second Solutions for questions 11 to 14:  Choice (2)
paragraph we can conclude that the an- 23. 1   24.  2   25.  3   26.  4
11. The error is in part 4. We obey some-
swer is choice (3), While choice (1) and
one or something. We need not use ‘to’. Solutions for questions 27 to 30:
(4) are subsets to it choice (2) is a wrong
 Choice (4) 27. (1) Incorrect – The statement contradicts
inference. Choice (3)
12. The error is in part 2. Despite is not fol- what goes in the earlier part. “Disin-
4. Choice (4) is out of the context of clined to believe the worst” means an
lowed by ‘of’. ‘The rains came despite
the question, as it is not connected to the optimist.
the firing - - - - ‘ Choice (2)
responsibilities of American business. It (2) Incorrect – “believe the best” again
is more connected to the worry about the 13. The error is in part 3. The use of prep- is contrary to the tenor of the state-
power of the corporations. Choice (4) osition ‘up’ with cope is incorrect. ment.
 Choice (3) (3) Incorrect – gives the notion of mag-
5. Referring to the last paragraph, we find
that “the process of change and the out 14. The adjective ‘enlightened’ is misplaced. nanimity which is erroneous.
come are tinged with a high degree of un- It must be placed before the noun it (4) Correct – Blends with the rest of the
certainity” – is restated in choice 2. The qualifies ‘fisherman’. The error is in (1). statement. Choice (4)
other 3 choices are distortions of the ideas  Choice (1) 28. (1) Incorrect – Freedom of mind can
in the paragraph. Choice (2) never be “not comfortable”. Also
Solutions for questions 15 to 18: “exhaustive” means thorough and
6. Option (1) is just a perception of the Tas-
15. ‘Rivulet’ is a small stream. ‘River’ is a is not appropriate here. It should be
manian Devil. Option (3) is eliminated
copious stream of water. The relation is “exhausting”.
since it is an opinion of the author, thus
that of small to big. ‘Ringlet’ is a little (2) Incorrect – “not comforting” is in-
cannot be categorized under ‘true state-
ring. Choice (1) is correct. ‘Trench’ and compatible. Also “aspires for” means
ments’, option (4) is a distortion of the
‘moat’ are synonyms. ‘Torrent’ is a great aiming for an object. The correct
information given in the first paragraph,
downpour, whereas ‘drizzle’ is light rain preposition should be “aspire to”.
option (2) is true as can be inferred from
falling in fine drops. We eliminate this as (3) Incorrect – “not a comfort” is absurd.
the second paragraph. Choice (2)
the relation is in the reverse. Choice (1) Also “fight” means with some exter-
7. While the choices (1) and (3) are just sup- nal forces.
porting ideas, choice (2) can be eliminated 16. ‘Datum’ is the singular form of ‘data’. (4) Correct – All the words harmonize,
since it is a distorted piece of information. ‘Axis’ is the singular form of ‘axes’. including “struggle” which means an
Choice (4) is the proper choice since the The relation is that of plural to singular. internal conflict with one’s own self.
reviewer himself seems interested in the  Choice (4)  Choice (4)

08_Section.indb 49 10/30/2009 11:52:50 AM


50  n  Solution Manual

29. (1) Incorrect – “temporary regularity”, is 38. Something that is ‘noxious’ is ‘harmful’. 4. In paragraph 1 the author is critical of
not consonant with “modern machine ‘Contumacious’ means ‘insubordinate’ economists as the latter believe that only
civilization”. Also “punctual by the or ‘stubbornly’ or ‘wilfully disobedient’. material things have value whereas there
clock” is meaningless. ‘Outlandish’ means ‘strange’, ‘bizarre or are many things like nature, which are
(2) Incorrect – “punctured by the clock” unfamiliar’. Choice (2) invaluable. But while he is critical his
is absurd. tone is not sharp. Hence he is ‘sarcastic’
Solutions for questions 39 and 40:
(3) Correct – The message, that the clock but not scathing or indignant. Choice 4
governs the life of the modern man 39. The key points in the paragraph are can be ruled out as the author does not
and is relentless in its rule, is brought 1. We are unique in some ways act superior. He is sure of his position
out clearly. 2. Our very uniqueness has landed us in and is critical of the economists’ stand.
(4) Incorrect – “temporal frequency” is the mess  Choice (2)
again an absurd phrase. Also “punc- 3. Over exploitation of resources has led 5. Paragraph 9 together with Paragraph 8
tilious describes a person and here it to extinction of some species. shows the author ridiculing the rich who
is inappropriate. Choice (3) are ready to go to any length (however
 he points are captured in Choice 2.
T
30. (1) Incorrect – “Economical” is inappro- Choice 1 and Choice 3 are not right be- foolish) to maintain their exclusive status.
priate here as it means frugal, sparing cause they say that extinction of species Choice 1 – (To say that he is ‘overly criti-
and ascetic. Also “societal” pertains is 60 per cent whereas as per the passage cal’) would imply that there is some logic
to something within society. 60 per cent of the ecosystem services are in what the rich are doing – which is not
(2) Incorrect – The use of “compelled” being degraded. Choice 4 is wrong be- the case. Choice 3 is ruled out because the
for environment is erroneous. Also cause the Millennium Ecosystem Assess- author is not ‘advocating rationality’ – he
the word “sociological” is out of ment does not show that we are unique. is pointing to the folly of their behaviour.
place.  Choice (2) The author is neither obsequious (fawn-
(3) Correct – “Physical & social tinder- ing, excessively attentive) nor contemp-
40. The main points covered are : tuous (scornful). Choice (2)
box is appropriate and correctly con-
1. Format change needs consensus with-
veys the meaning.
in the organisation. Solutions for questions 6 to 10:
(4) Incorrect – “transformed” generally
2. Managers must explain the need for 6. At the beginning of the passage, the au-
means something positive and hence
change and the potential for growth. thor is sad, gloomy, despondent but not
is incorrectly used. Choice (3)
3. The tone must be positive. pessimistic critical or desperate (rules out
Solutions for questions 31 to 34: 1, 3 and 4). At the end he is philosophic,
 hoice 1, 2, and 4 leave out the fact that
C
31. The idiom ‘root and branch’ means com- optimistic and accepting but not euphoric.
we are talking of a newspaper. Choice
pletely.  Choice (2)  Choice (2)
2 gives examples which are generally
avoided in summaries. Choice 4 leaves 7. The author’s recounting is poignant
32. ‘With open arms’ means with a warm
out valid information. Choice (3) (moving). It is not pathetic because if we
welcome. Choice (4)
say that his recounting is pathetic it would
33. The idiom ‘Tooth and nail’ means with TEST PAPER 2 mean that the author hasn’t expressed
great energy. Choice (1) himself well. Choices 3 and 4 are obvi-
Solutions for questions 1 to 5:
34. ‘Spread like wild fire’ means spread ously incorrect. Choice (1)
1. The passage shows that the author is
rapidly. Choice (3) 8. When the author asks ‘Why should this
concerned about what is happening in
the world today. There is no bitterness – happen to us?’ he is dejected or in low
Solutions for questions 35 to 38
hence it is not cynical. The author does spirits. He is far from arrogant or abusive;
35. The most appropriate choice is ‘amnesty’ he is not hopeless. Choice (3)
not express any hatred – .so it cannot be
because the given question stands as a
called hostile. The author is not desperate 9. The author refers to Indianapolis 500-car
definition for that. ‘Solicitation’ means
nor is he stubborn or arrogant.Choice (4) race to show that these children are as in-
asking for something from someone, trepid as the drivers of the car race and the
‘arbitration’ means ‘the settlement of a 2. Paragraph 5 shows that even the poorest
audience enjoyed the event as much. He
dispute by an arbitrator’. ‘Allusion’ re- place has all the material affluence (be-
is not belittling the race, nor is he exag-
fers to an indirect reference to something. yond wants). So the question at the end
gerating. Choice (4)
 Choice (2) of the Paragraph is rhetorical (does not
require an answer as the answer is obvi- 10. The passage may be described as motivat-
36. The appropriate choice is ‘morphology’; ing because though the author is gloomy
ous). When the author asks the question,
‘philology’ is ‘the science which studies at first, by the end he has learnt to ‘cel-
the answer implied is that the rich nations
the history and development of languag- ebrate life’ and this motivates the readers
have successfully met the basic needs of
es’. ‘Anthropology’ is ‘the study of the to feel the same. Choice (1)
the people. Choice (1)
origin and physical and cultural develop-
ment of mankind’. ‘Geology’ is ‘the sci- 3. Paragraph 8 makes it clear that the rich Solutions for questions 11 to 14:
ence that studies the crust of the earth’. buy products not for the benefit they 11. In the first sentence the usage of ‘of’ is
 Choice (2) would get from it, but as a means to set redundant because of and about cannot
themselves apart from the crowd. This is be used together. Option 2 is erroneous
37. The given statement defines the word
why they are never satisfied and the desti- because of the same reason. Options 4
‘mollycoddle’. Choice (1)
nation keeps shifting. Choice (4) is incorrect because the relative pronoun

08_Section.indb 50 10/30/2009 11:52:50 AM


Solutions for Language Comprehension  n  51

‘whom’ is not used. Thus option 3 is the 2 uses the correct pronoun those. Besides, ‘ Tiny’ is small while ‘tinny’ is high-
correct sentence. Choice (3) ‘strenuous exercises’ is more concise pitched voice. Option (B) is appropriate.
and solves the Subject Verb agreement ‘prelate’ is a member of the clergy hold-
12. The expressions ‘go swimming’, go shop-
problem. The phase ‘strenuous exer- ing a high rank. ‘Prelude’ is an introduc-
ping’, etc. are always used without ‘for’.
cises’ is also parallel to lighter exercises. tion to something that is going to happen.
The correction is ‘go swimming’ or ‘go
 Choice (2) Option (B) is appropriate.
for a swim’. Hence only choice 2 is right.
A ‘scallywag’ is someone who behaves
 Choice (2) 19. 2   20.  2   21.  4   22.  1
badly. ‘Scqpe goat’ is someone who is
23. 2   24.  3   25.  1   26.  3
13. When we mean ‘at the end of a period punished although he may not be respon-
27. 3   28.  2   29.  4   30.  4
of time; we use ‘in’ and not ‘after’. Op- sible for it. Option (A) is appropriate.
tion 3 is incorrect because the ‘s’ in the Solutions for questions 31 and 32: Choice (4) is the correct option. ABBA.
word ‘months’ should be followed by an  Choice (4)
31. The main points covered are
apostrophe – months’. Hence option 4 is
1. Immigration is the key issue in the pre
grammatically correct. Choice (4) 35. ‘Pared’ is to back down or reduce. Option
election scenario. (B) is appropriate.
14. If we feel sorry about something that has 2. People, including Labour party sup- ‘Adherents’ refers to supporters. ‘Adher-
already happened, we regret doing it. The porters, want tougher laws on immi- ence’ is to behave according to a particu-
to infinitive is used after ‘regret’ when gration. lar rule. Option (B) is appropriate.
we are about to give someone some bad
 hoice (3) and (4) do not mention the
C A ‘tabloid’ is a newspaper with small pag-
news. Hence only option 4 is correct.
elections. Hence they are incorrect. es and usually has short articles and a lot
 Choice (4)
Choice (2) is factually wrong since it is of pictures. A ‘tableau’ is a scene showing
Solutions for questions 15 to 18: the supporters of the Labour party and not events and people from history. Option A
the party itself that wants tougher laws. is appropriate.
15. The expression as old as indicates equal-
 Choice (1) A ‘questioner’ is a person who asks
ity of age, but the sentence indicates that
questions, especially in a broad cost pro-
the temples predate the Dravidian struc- 32. The main points are: gramme. A ‘questionnaire’ is a written list
tures. Statement 4 makes the point of 1. France has suggested global tax to of questions. Option (B) is a appropriate.
comparison clear. It also uses the correct raise funds to meet the Millennium The correct choice is (2). BBAB.
adjective supposed, rather than supposed- Development Goals. Choice (2)
ly (an adverb) to modify the noun phrase 2. The developed and developing coun-
Dravidian predecessors. Choice (4) tries agreed to a small tax on trade and 36. ‘Queer’ is to feel strange. Queasy is feel-
16. In this sentence, Ben Jonson the dramatist air transport. ing sick with a sensation of vouching op-
can be compared to Shakespeare the dra- tion (B) is appropriate.
 hoice 1, 2, and 3 do not mention what
C
matist. His plays can not be. Only choice ‘Systemic’ is to affect the whole of some-
are to be taxed. Choice 2 says it is the
4 brings out the correct comparison. thing. ‘Systematic’ is do something ac-
only way, which may not be true. Choice
Choices 1, 2 and 3 make illogical com- cording to a plan. Option (A) is correct.
3 does not mention France which is the
parisons. Choice (4) ‘Uncommitted’ is not promising support.
key player. Choice (4)
‘Unclassified is not good enough. In the
17. Each choice but 3 contains errors of Solutions for questions 33 to 36: context option (A) is suitable.
agreement. The singular subject each
33. ‘Jinx’ refers to some thing or some one A ‘simile’ is a figure of speech. It drows
takes a singular verb has and not have.
that is considered to be unlucky. ‘Gibe’ comparisons. A ‘homily’ is a speech
Choice 4 is very awkwardly constructed
refers to a rude or insulting remartk. Op- or piece of writing giving advice on
and conveys a very absurd idea. Choice
tion (B) is appropriate. correct way to behave. Option (B) is
2 is wordy and contains the unnecessary
‘Dates back’ refers to something that was correct. Choice (3) is correct. BAAB.
‘each of’. Choice 3 maintains subject verb
started or made at that time. Option (A) is  Choice (3)
agreement (sons have made) and provides
a clear structure. Choice (3) appropriate. Solutions for questions 37 to 40:
A ‘grater’ is a kitchen tool which has a
18. In this sentence, “benefits of exercises 37. ‘Adulation’ which means uncritical admi-
rough surface and is used to cut food into
such as………and aerobics” should be ration. Choice (3)
very small pieces A ‘grate’ is a framework
followed by a plural verb ‘are’ and not of iron bars option (A) is appropriate. 38. Anecdote is the meaning story about a
‘is’ (that are strenuous). Similarly, in the ‘Imaginings’ are things that you think you person or event. Choice (4)
second part of the sentence, underesti- have seen or heard. Imaging is a process
mate that is incorrect. The reference is to 39. ‘Barricade’ is a time of objects placed
of forming images. Option (A) is appro-
the plural “benefits of exercises” and the across a road to stop people from getting
priate. The correct choice is (1) BAAA.
pronoun to be used is ‘those’ not that. In past. Choice (1)
 Choice (1)
choices 3 and 4 the use of when nonsensi-
34. ‘Stifled’ means prevent from continuing. 40. A ‘souvenir’ is kept as a reminder of a
cally suggests that benefits of strenuous
‘Stiffened’ is to be firm. Option (A) is ap- place, occasion etc. Choice (4)
exercises are underestimated when they
propriate.
are derived from lighter exercises Choice

08_Section.indb 51 10/30/2009 11:52:50 AM


Solutions for Mathematical Skills 3
Equations 4. 2x – y + 3z = 9 → (1); x + y + z = 6 → 11. Let the number of Rs 5, Rs 10 and Rs 50
(2) notes be x, y and z respectively.
(1) + (2) 3x + 4z = 15 → (3); x − y + z 5x + 10y + 50z = 125 and x + 2y + 10z
Practice Exercise = 2 → (4) = 25
Solving equations, we get x = 1, y = 2 and If z = 0, x + 2y = 25
Solutions for questions 1 to 40:
z = 3 Choice (1) The possible values of x and y are (1, 12),
1. (a) 6x + 14y = 2 and 6x + 27y = 15 ; (3, 11), (5, 10), (7, 9), (9, 8), (11, 7), (13,
a1 3 b −6
Solving, we get y = 1 and x = –2 5. (a)  = and 1 = ; For a unique 6), (15, 5), (17, 4), (19, 3), (21, 2), (23, 1)
 Choice (4) a2 4 b2 P and (25, 0).
(b) 2007x + 2008y = 10038 → (1); a1 b1 If z = 1, x + 2y = 15. The possible values
2008x + 2007y = 10037 → (2) solution, ≠ ⇒ P ≠ –8 of x and y are (1, 7), (3, 6), (5, 5), (7, 4), (9,
a 2 b2
(1) + (2) ⇒ 4015 (x + y) = 20075 3), (11, 2), (13, 1) and (15, 0). If z = 2, x +
 Choice (3)
⇒ x + y = 5 → (3); (2) − (1) ⇒ x − y 2y = 5, the possible values of (x, y) are (1,
= –1 → (4) a1 7 1 b1 P a b 2), (3, 1) and (5, 0).
(b)  = = , = ; 1≠ 1 ⇒
Solving (3) and (4), we get x = 2 and a 2 14 2 b 2 11 a 2 b 2 ∴ Total numbers of possibilities are 24.
y=3  Choice (4)
∴ x = 2 and y = 3. Choice (2) 11
P≠  Choice (1) 12. Given u + v + w = 6720 → (a) and
2 u = 1/3 (v + w) → (b)
1 r 1+ a p + q + r
2. = ⇒ = 6. ∴ For 3x – 4y = 2 and 51 x – 68y = 34 as substituting (b) in (a) we get, 3u + u =
a p+q a p+q
3 4 2 1 . 6720 ⇒ u = 1680 Choice (2)
a p+q b q+r = = =
= ; = ; 51 68 34 17 13. Let the two-digit number be xy. 10x + y +
1+ a p + q + r 1+ b p + q + r 10y + x = 110
∴ the given pair of equations will have
p + q )+ infinite ⇒ x + y = 10 → (1) and Given x − y =
c
=
r+p
   
a
+
b
+
c
=
( (q + r )+number
(r + p ) =of2 solutions Choice (4) 4 → (2)
1+ c p + q + r 1+ a 1+ b 1+ c 7. (pLet + r ) be a total of x cows and goats
+ q there Solving (1) and (2), we get x = 7, y = 3,
and y hens in the farm.
a b c (p + q )+ (q + r )+ (r + p ) x × y = 21
+ + = =2 From the data, 4x + 2y = 184 → (1) and x
 Choice (3)
1+ a 1+ b 1+ c (p + q + r ) + y = 66 → (2)
Solving (1) and (2), we get x = 26 and y = 14. Possible numbers are 108, 801; 207, 702;
 Choice (2) 40 Choice (1) 306, 603; 405, 504 and 900.
3. (a) a1 = 3, b1 = 4, a2 = 2, b2 = 3 504 – 405 ≠ 297; 603 – 306 = 297
8. Let the number of monkeys on T1 be x
3 2 a a 702 – 207 ≠ 297; 801 – 108 ≠ 297
a1/b1 = and a2/b2 = ; 1 ≠ 2 , and the number of monkeys on T2 be y
4 3 b1 b 2 900 – 009 ≠ 297
x – 1 = y + 1 ⇒ x = y + 2 → (1); x + 1 =
∴The number is 603, as 306 is not given
the given equations are consistent. 2(y – 1) ⇒ x = 2y – 3 → (2)
in the choices. Choice (2)
Solving (1) & (2) we get x = 7 and y = 5
 Choice (4) 15. Let the present ages of Tarun, Aakash and
a1 3 a2 3 c1 9
(b)  = , = and = ; x x + 2 2 Rohit be T, A and R respectively.
b1 5 b2 5 c2 11 9. Let the fraction be ; = ⇒ 3x = (T − 8) + (A − 8) = 2 (R − 8) ⇒ T + A =
y y+4 3
a1 a 2 c1 2R → (1) and (T + 12) + (A + 12) = 80 ⇒
= ≠ 2y + 2 T + A = 56 → (2)
b1 b 2 c2 x −1 3
= ⇒ 4x = 3y + 1; By solving x = From (1) and (2), R = 28
∴ The given equations are inconsis- y −1 4 ∴ T + A + R = 3R = 84 Choice (2)
tent. 4, y = 5 Choice (1) 16. Let the three consecutive numbers be x –
(c) a1 = 1, b1 = 3 and c1 = 12; a2 = 3, b2 = 10. Let the present age of the man be x 1, x, x + 1
9 and c2 = 36 years. Sum of three consecutive number x − 1 +
a1 b c x + 25 = 11(x – 25) ⇒ 10x = 300 ⇒ x x + x + 1 = 63
 = 1 = 1 ∴ The given equa-
a 2 b2 c2 = 30 ⇒ 3x = 63 ⇒ x = 21. Product of extremes
∴The present age of the man is 30 years = (x − 1)(x + 1) = x² – 1 = (21)² − 1 = 440
tions are consistent. Choice (4)
 Choice (3)  Choice (4)

08_Section.indb 52 10/30/2009 11:52:51 AM


Solutions for Mathematical Skills  n  53

17. Let the present ages of Amar’s father, moth- 24. Total distance travelled by Narad = 120 30. 3x + 4y + 2k = 20 → (1) x + 2y + K = 8 →
er, and Amar be f, m and s respectively. km. Distance travelled by him on walk = (2) Multiplying equation (2) by 2 and sub-
When Amar was born, the ages of his fa- 3 tract it from (1) we get x = 4. Choice (1)
ther and his mother were (f − s) years and × 120 = 36 km
10 31. Let the number of mango trees and ba-
(m − s) years respectively. Let distance travelled by boat = b. Let dis- nana trees in his garden be m and b re-
3m – f = 40 → (1); m + f = 80 → (2); tance travelled by him by bicycle = c spectively. m = b + 1.
f – s = 2(m – s) → (3) ∴ b = 3c Total number of fruits = 2 (m + b) = 62 ⇒
by solving (1), (2) and (3) we get s = 10 ∴ 36 + b + c = 120 ⇒ 36 + 3c + c = 120 m + b = 31 ⇒ b + 1 + b = 31 ⇒ b = 15
 Choice (3) ⇒ c = 21 km Choice (1)  Choice (4)
18. Let the age of the father and the son be 25. Le there be x chocolates ⇒ Each friend 32. Let the fraction be x/y
x and y years respectively. Father was y x +1 1
years old (x-y) years ago. Son’s age (x – got x chocolates. = ⇒ 2x + 1 = y → (1)
4 y +1 2
y) years ago = y – (x – y) After eating 3 chocolates each, the to-
given x = 4 (y – x + y) ⇒ 5x = 8y → (a) tal number of chocolates with them = x 9 x 9
sum of the ages of father and son = 130  = ⇒ = ⇒x=
y − x 10 2x + 1 − x 10
∴x + y = 130 ⇒ 5x + 5y = 130 × 5 → (b) x 
4  − 3  = x – 12.
substituting (a) in (b), we get y = 50 years, 4  9, Substituting x = 9 in equation (1) y =
x = 80 years Choice (2) ⇒ x = 24 19 Choice (3)
∴ Total number of chocolates with them 33. Let the larger part be Rs x and smaller
19. Let his monthly salary be Rs x; rent = initially = 24. Choice (3)
x part be Rs (200 − x).
Rs 26. Let the number be xy x
3 Given, + 3(200 − x) < 200 ⇒ x > 160
10x + y + 10y + x = 99 ⇒ x + y = 9 → 2
Amount spent towards clothes and enter-
(1) ∴ Larger part is always greater than
1 x x x – y = 3 → (2) y – x = 3 → (3) Rs 160. Choice (2)
tainment =  x −  = Rs.
2 3 3 If x – y = 3 then solving equations (1) and
34. A + B + C + D = 110 → (1), A = 2D →
Amount spent towards food = (2) we get x = 6 and y = 3
(2), A + C = B → (3), C = D + 20 → (4)
1 x x If y – x = 3 then solving equations (1) and
Substitute (2), (4) in (3) we get 2D + D +
 x −  = Rs. (3) we get x = 3 and y = 6.
4 3 6 20 = B ⇒ D = (B – 20)/3 ⇒ 2B + (B –
So the number is 36 or 63. Choice (3)
3x 20)/3 = 110
Amount spent towards travelling = 27. Let the number of students in each row be ⇒ B = 50
20 x and the number of rows be y. Substitute value of B we get A = 20, B =
x x x 3x ∴ Total number of students = xy 50, C = 30, D = 10
⇒ + + + + 3900 = x ⇒ x = 39000 According to the problem,
3 3 6 20 Share of A is Rs 20, B is Rs 50, C is Rs 30
 Choice (2) (x + 3) (y − 2) = xy ⇒ –2x + 3y = 6 ------ and D is Rs 10. Choice (1)
(1) and (x − 3) (y + 4) = xy ⇒ 4x − 3y =
20. Given (a + b) = 5/4c → (a), a = 2/3b and 35. Let the cost of each pen, each sharpener
12 ------ (2)
→ (b); a + b + c = 90 → (c) and each eraser be Rs p, Rs s and Rs e
Solving (1) and (2), we get x = 9 and y
substituting (a) in (c) we get c = 40 years respectively.
=8
∴ a + b = 90 – 40 = 50 → (d) 9p + 11s + 14e = 107 → (1);
∴ Total students in the class = 72
substituting (b) in (d), we get a = 20 years 8p + 13s + 16e = 119 → (2);
 Choice (2)
 Choice (3) 11p + 7s + kr = 83 → (3)
28. Let the present age of father and son be f Subtracting 2 × (2) from 3 × (1),
21. Let the number of marbles with Ramesh and s years respectively. Given f = 2s + 10 11p + 7s + 10r = 83 → (4)
and Suresh be x and y respectively. --------- (a) it follows that if k = 10, the equations (1),
From the data, x + 3 = y − 3 ⇒ x − y = –6 Let after ‘x’ years, father’s age be twice (2) and (3) would not be independent of
------ (1) and 4 (x − 3) = y + 3 ⇒ 4x − y son’s age each other.
= 15 ------ (2) ∴ f + x = 2 (s + x) --------- (b) Solving (a) ∴ They would not yield an unique solu-
Solving (1) and (2), x = 7 and y = 13 and (b) ⇒ x = 10 years Choice (2) tion. Choice (4)
 Choice (3)
29. Let there be n persons in the group. 36. Let the numbers be x – 1, x, x + 1
22. Middle digit of a 3 digit number is zero so Amount received by each person = Rs 3x = 45 ⇒ x = 15; ∴ the numbers are
the remaining digits be represented by x
2640 14, 15, 16
and y .
n (16² − 14²) − (16 − 14)2 ⇒ (256 – 196) – 4
x + y = 9 → (1) ⇒ 100x + y + 99 = 100y
= 60 – 4 = 56 Choice (2)
+x If 3 more persons were there, amount
x − y = −1 → (2) 2640 37. Let the number of questions left unan-
Solving equations (1) and (2) we get x = received by each person = ; From swered by Vinod be x. Number of ques-
n +3
4, y = 5. The number is 405 Choice (2) tions attempted by him = 200 – x
2640 2640 Number of questions he marked wrong = x.
23. Let the costs of an apple, an orange and the given data, − =8 ⇒
n n +3 Number of questions he marked correct =
mango be Rs x, Rs y and Rs z respective-
(200 − x) − x = 200 − 2x
ly.  3 
4x + 5y + 7z = 115 → (1) and 2640  =8 1 1
(200 − 2x) 2 − x   − x   = 134 ⇒
 n (n + 3)  2 4
9x + 7y + 3z = 161 → (2) x = 56
(1) × 2 + (2) ⇒ 17x + 17y + 17z = 391 ⇒ n (n + 3) = 30 × 33; ∴ Number of questions attempted = 200
⇒ x + y + z = 23 Choice (4) ∴ n = 30 Choice (4) − 56 = 144 Choice (1)

08_Section.indb 53 10/30/2009 11:52:51 AM


54  n  Solution Manual

38. Let the distance A and B travel in 1 hour On solving (1) and (2), we get x = 3n and 13. Let the numbers be 2k and 3k. When 5
be x and y respectively; 3x = y → (1) 5x y = 2n 2k + 5 5
= 2y – 1 → (2) ∴ Total number of chocolates = 4x + 3y = is added to both = ⇒ 2k = 20
3k + 5 7
Solving (1) and (2) we get x = 1, y = 3 4(3n) + 3(2n) = 18n
In 15 hours A can walk 15 km whereas B Total number of biscuits = 3x + 2y =  Choice (1)
can walk 45 km 3(3n) + 2(2n) = 13n 14. Let the numbers of boys, girls be 2x, 3x
∴ Difference between the distance trav- Required ratio = 18n : 13n = 18 : 13. respectively
elled by A and B = 30 km Choice (4)  Choice (2) 2x − 5 1
= ⇒ x = 15;
39. Let the number of vehicles capable of car- 6. 2a2 + 8b2 + 5c2 = 2c (a + 6b) ⇒ 4a2 – 4ac 3x + 5 2
rying 10 and 15 voters be x and y respec- + c2 + 16b2 + 9c2 – 24bc = 0
∴Number of boys = 15 × 2 = 30Choice (2)
tively. Given, x + y ≤ 90 → (1) and 10x + ⇒ (2a – c)2 + (4b – 3c)2 = 0;
15y = 1200 ∴ 2a – c = 0 and 4b – 3c = 0 15. Total amount = Rs 5625.
⇒ 2x + 3y = 240 → (2) a : c = 1 : 2 and c : b = 4 : 3; ∴ a : b : c = 1
A + B + C = Rs 5625 → (1); A = (B
Using (2) in (1) we get y ≥ 60 2 : 3 : 4 Choice (4) + C) → (2) 2
To get maximum votes by second candi-
7. The ratio of the ages of two persons is 5 1
date, y = 60 B = (A + C) → (3); From (1) and (2)
: 7. Let the ages of the two persons be 5k 4
∴ Number of voters of second candidate =
and 7k. ⇒ A = Rs 1875; From (1) and (3) ⇒ B
60 × 15 = 900
⇒ Number of voters of first candidate = 5k −18 8 = Rs 1125
So = ⇒ k = 90; The age of A − B = 1875 – 1125 = Rs 750.Choice (2)
1200 − 900 = 300 7k −18 13
The minimum difference = 900 – 300 = 16. 9 + 11 = 20 is not a factor of 75. So we
the younger one = 5k = 5 ×10 = 50 years
600 Choice (4) cannot divide medals in the ratio of 9 : 11.
 Choice (1)
40. 2p + 3s + 4e = 35 and 4p + 6s + 7e = 66  Choice (4)
2(35 − 4e) +7e = 66 ⇒ 70 − e = 66 8. (a) 22 : 32 = 4 : 9  Choice (2)
(b) 23 : 13 = 8 : 1 Choice (2) 17. Let Ram gave Rs 5x and Rs 7x to Lava and
⇒ e = 4 Choice (3) Kusha respectively; Given share of Lava =
(c) 16 : 9 = 4 : 3 Choice (3)
1
5x + (5x + 7x) = 560 ⇒ x = 80.
Ratio, Proportion (d) 4 / 9 × 16 / 25 = 8 / 15 = 8/15 6
 Choice (1)  If Lava gets 5x + 2x = Rs 7x, then

and Variation Kusha will get Rs 5x that is 5 × 80 = Rs
9. Let the monthly income of A and B be 4x
and 3x. 400. Choice (3)
Practice Exercise
Savings of each of them = Rs 600 18. A B
Expenditure of A = 4x − 600 and Expendi- Incomes 5x 4x
Solutions for questions 1 to 40:
ture of B = 3x − 600 Expenditures 2y 3y
1. A : (B + C) = 2 : 5; A : (A + B + C) = 2 : 7 4x − 600 3 savings 5x – 2y 4x – 3y
2 2 ∴ = ⇒ x = 600
∴ A = (A + B + C) = × 350 = 2 × 50 3x − 600 2 Given 5x – 2y = 3y ⇒ x = y
7 7 ∴ ratio of savings of A and B =
∴Monthly income of A is 4x = 4 × 600 =
= 100 Choice (2)
Rs 2400. Choice (1) 5x − 2y 3
(1 / a + / b) a+ b =  Choice (2)
2. = ; Substitute a = 3x 10. a : b = 1 : 2 = 3 : 6; b : c = 3 : 5 = 6 : 10 4x − 3y 1
(1 / a − 1 / b) b − a a : b : c = 3 : 6 : 10; c : d = 5 : 4 = 10 : 8
a : b : c : d = 3 : 6 : 10 : 8 = 9 : 18 : 30 : 24 19. Let the amount B gets be Rs x.
3x + 4x
and b = 4x; = 7 Choice (3) ∴The amount received by A is x + 600
4x − 3x d : e = 6 : 5 = 24 : 20;
a : b : c : d : e = 9 : 18 : 30 : 24 : 20 2 2
3. A : B = 100 : 75 = 4 : 3; B : C = 2 : 1
 Choice (2)
C = (A + B ) = (x + x + 600 ) ; B =
3 3
8
A : B : C = 8 : 6 : 3; Earnings of A = × 11. Let the density of metal z be h. 3(A − C);
3
4200 = 11,200 Choice (2) ∴ Density of metal x = 15h and density  2 
of metal y = 8h x = 3  x + 600 −  (2x + 600 )  ⇒ x =
4. 21x2 − 28y2 = 16x2 − 8y2; 5x2 = 20y2 Let the metals x and y be mixed in the ra-  3 
x2/y2 = 4/1; (x/y)2 = 4/1 ⇒ x/y = 2/1 tio k : 1⇒ 15h(k) + 8h (1) = 13h(k + 1) ⇒ 300
5x + y 11 5 ∴A’s share is 300 + 600 = Rs 900
Let x = 2k, y = k, = Choice (1) 15k – 13k = 13 – 8 ⇒ 2k = 5 ⇒ k = ;
3x − y 5 2  Choice (4)
5.
The required ratio = 5 : 2 Choice (4) 20. Let the ages of the three persons in years
White box Red box
p q r 1 be 5x, 7x, and 9x
Number of chocolates 3y 4x 12. Given = = =
a(q + r) a(p + r) a(p + q) b Given (7x)2 + (9x)2 = 3250 ⇒ x = 5; The
Number of biscuits 3x 2y ages in years are 25, 35, 45 Choice (4)
a p q r
 otal number of chocolates and biscuits in
T  = = = 21. The ratio of 1 rupee, 50 paise, 25 paise
b q+r p+r p+q
the white box = 3x + 3y and the total num- coins is 5 : 6 : 8
ber of chocolates and biscuits in the red a p+q+r a 1
⇒ = ⇒ =  Let the number of 1 rupee coins be 5x.

3x + 3y 15 b q+r+p+r+p+q b 2 So, the number of 50 paise that is ½ rupee
box = 4x + 2y; Given, =
4x + 2y 16 coins = 6x and the number of 25 paise that
3x + 3y = 15n → (1); 4x + 2y = 16n → (2) ⇒ a = b /2 Choice (3) is ¼ rupee coins = 8x

08_Section.indb 54 10/30/2009 11:52:52 AM


Solutions for Mathematical Skills  n  55

1  1 30. Speed of A to that of Horse B = (7 × 6 : 5 36. x ∝ y ⇒ x ∝ 1/z2 ⇒ x ∝ y/z2; x1/x2 = y1/y2



So, 5x +  6x ×  +  8x ×  = 200 ⇒ × 9) × z22/z12; 60/x2 = 4/8 × 6 × 6/3 × 3 ⇒ x2=
 2  4
⇒ 42 : 45 = 14 : 15; 30 Choice (1)
x = 20 ∴Ratio of time = 1/14 : 1/15 ⇒ 15 : 14
37. Given, B α t, C α t2; B = k1t, C = k2t2
 Choice (4)
∴Total number of coins = 19x = 380. A α (B + C), A = k1t + k2t2 when t = 5;
 Choice (1) 31. Let the fixed charge be Rs x and the 5 = (5k1 + 25k2); 1 = (k1 + 5k2) → (1)
charge per kilolitre be Rs y. Given, x + When t = 3, 9 = (3k1 + 9k2); 3 = (k1 + 3k2)
22. m1/m2 = w2/w1 × r2/r1; x = 1600 × (18/16) 100y = 300 → (1) → (2)
× (25/20) = 2250 Amount charged when the consumption Subtracting (2) from (1), − 2 = 2k2 ⇒ k2
∴650 more persons joined the garrison. is 300 kilolitres = Rs (x + 300y) and the = −1
 Choice (1) amount charged when the consumption is Putting k2 = −1 in equation 2, 3 = k1 − 3
23. Let the three numbers be 2k, 3k and 5k 400 kilolitres = (x + 400y) ⇒ 6 = k1;
∴4k2 + 9x2+ 25k2 = 608 ⇒ k = 4 x + 300y 5 9
∴The smallest of the three numbers is 2k = ; ⇒ y = 1 and x = 200. ∴ A = 6t − t2. Now, when t = ,
x + 400y 6 2
= 8 Choice (3)
 Choice (1) 2
9 9 27 3
24. m = s + 24, s = d – 6 ⇒ m = d + 18 A = 6. −   = = 6 Choice (1)
32. Let the capacity of the three casks be 3x, 2 2 4 4
Let the present ages of mother and daugh-
2x, 6x
ter in years be 2x and x.
So 2x – x = 18 ⇒ x = 18. Present ages of
Quantity from third cask = (2/3) × 6x = 38. Given xα y2 and x ∝ 1
4x; Quantity from first cask = (1/3) × 3x = x z
mother & daughter are 36 years and 18
4x − x = 18 ⇒ x = 6 y2
years respectively. Choice (2) Combining both, we have x ∝ ⇒ xz
∴Capacities of original three casks is 18,
25. Let the number of students in A be 2x. z
12, 36 litres. Choice (3)
Number of students in B and C are 3x and ∝y2
33. E ∝ mv2 ⇒ E = kmv2
4x respectively. x1z1 y2 8×3 4
Where E is the energy possessing by a ⇒ = 12 ; ⇒ = ⇒ x2 =
moving body. m is the mass v is the ve- x 2 z2 y2 x 2 × 6 16
3x + 10 4
= ⇒ x = 10
4x − 10 3 locity, and k is constant proportionality. 16 Choice (4)
Number of students in A = 2x = 20 When E = 40, m = 10 kg, v = 2 2 K2
 Choice (4) 39. S ∝ R ⇒ S = K1R and T = ; Where R
1 R
26. Let the number of 20 paise coins be 5x, ∴40 = k × 10 × (2 2) 2 ⇒ k =
2 = 20,
Total values of 20 paise coins 5x × 20 = K2
100x when E = 4000 and mass is 80 kg; S = 40 and T = 10; 20 = ⇒ K2 =
1 T
25 paise coins be 6x;  6x × 25 = 150x 4000 = × 80 × v2 ⇒ v = 10 m/s
50 paise coins be 4x;  4x × 50 = 200x 2 200
 Choice (4) 40 = K120 ⇒ K1 = 2
⇒ 150x – 100x = 100 ⇒ x = 2
Total value = 10 × 20 + 12 × 25 + 8 × 50 34. Let the weight of each piece be 2x, 3x and When S = 20; 20 = 2 × R ⇒ R = 10 ⇒
= 200 + 300 + 400 = 900 paises = Rs 9 5x respectively. K2
 Choice (1) Total weight (w) of the stone = 2x + 3x + T=
R
27. Let the capacity each glass be x. Total 5x = 10x; Value ∝ (weight)2 ⇒ V = Kw2
volume of milk is 2x. 20000 = K (10x)2; 20000 = 100 Kx2 ⇒ 200
= = 20 Choice (4)
Kx2 = 200 10
2
Volume of milk filled in 1st glass = V1 = K(w1)2 = K (2x)2 = 4 Kx2 = 4 × 200
14 Volume
= 800, 40. Height ∝ ⇒ H ∝ V ⇒ Hr 2 ∝ V
× 2x V2 = K (w2)2 = K (3x)2 = 9 Kx2 = 9 × 200
(radius) 2 r2
4x 1 = 1800, H1r12 V
= < x; Volume of milk in 2nd glass ⇒ = 1
14 2 V3 = K (w3)2 = K(5x)2 = 25 Kx2 = 25 × H 2 r2 2 V2
200 = 5000
3 6x 1 ⇒ Given V : V = 1 : 2 and r : r = 1 : 3 ⇒
= × 2x = < x Total value of the broken stone is 800 + 1 2 1 2
14 14 2 1800 + 5000 = Rs 7600; Loss = 20000 H1 (1) 2 1 H ×1 1 H 9
= ⇒ 1 = ⇒ 1 =
Volume of milk in 3rd glass − 7600 = Rs 12400 Choice (2)  H 2 (3)
2
2 H2 × 9 2 H2 2
4 × 2x 8x 1 35. E = k1N + k2 where E is expenditure Choice (3)
= > x ; Volume of milk in
14 14 2 and N is the number of inmates. k1 is the
5 × 2x 10x 1 expenditure of each inmate and k2 is the
4th glass = > x constant expenditure. When inmates are Percentages
14 14 2
200, 1300 = 200k1 + k2 → (1)
∴2 glasses are filled with less than 50 per
When inmates are 250, 1600 = 250k1 + Practice Exercise
cent of their capacity. Choice (2)
k2 → (2)
28. 4 × 9 = 6 Choice (4) Subtracting (1) from (2) We get k1 = 6; Solutions for questions 1 to 40:
Substitute k1 = 6 in equation (1), 1300 = 1. Let A’s salary be 100x. B = 80x and c =
29. Let ‘x’ be added to 14, 17, 34, and 42 to
1200 + k2. 120x
make it proportional.
∴k2 = 100
14 + x 34 + x 20x
∴ = ⇒ (14 + x) (x + 42) = Now, when the inmates are 300 the ex- The required percentage = ×100 =
17 + x 42 + x 120x
penses are = 300 × 6 + 100 = 1800 + 100
(x + 17) (x + 34) ⇒ x = −2 Choice (4) = 1900 Choice (4) 16 /3 per cent
2
Choice (2)

08_Section.indb 55 10/30/2009 11:52:53 AM


56  n  Solution Manual

2. Let length = 10 units, breadth = 10 units; 9. Let the number of students be 100. 15. Since the winner has won by the least
Area = 100 units ∴ Number of boys = 30 per cent of 100 percentage of votes and x, y and z being
Later, length = 11 units, Breadth = 9units; = 30 integers, winner should win by 1 per cent
New Area = 99 units. ⇒ Number of girls = 70 of the total votes. The two contestants
1 The number of boys passed = 40 per cent should get 33 per cent each votes. Winner
Decrease in area = × 100 = 1 per cent of 30 gets 34 per cent of votes. 1 per cent of
100
 Choice (4) votes = 2432. 34 per cent of votes = 34 ×
2
= × 30 = 12 2432 = 82,688
3. Let total number of votes be 100; 5
 Choice (1)
∴Number of valid votes = 65 The total number of students passed = 70
Let the number of votes polled in favour per cent of 100 = 70 16. Given A got 40 per cent of total marks and
of X = x. Let the number of votes polled got 10 marks more than the pass mark and
∴ The number of girls passed = 70 – 12 =
in favour of Y = y B got 50 per cent of total marks and got
x−y 60 58 50 marks more than the pass mark. Let
⇒ x + y = 65 → (1); = ⇒ 5x 58; The required percentage = × 100 =
y 100 70 total marks be x,
6 ⇒ pass mark in both the cases is
x 8 82 per cent Choice (1)
− 5y = 3y; = → (2) 7 ∴ 40 per cent of x − 10 = 50 per cent of
y 5 10. Let the total number of votes be ‘x’. Can-
x
From (1) and (2), x = 40 and y = 25 45 x − 50 ⇒ 10 per cent of x = 40 ⇒ =
didate who was defeated got = ×x 10
100
40 votes 40 ⇒ x = 400
∴The required percentage = × 100 = ∴Pass Mark = 40 per cent of 400 – 10 =
100 55
40 per cent Choice (3) Candidate who won got = × x votes; 160 −10 = 150 Choice (4)
100
4. Let 1st number be 100; 2nd number is
55 45 17. l × b = Area =
(100 + x )(100 + 20 ) =
125 and 3rd number is 150; x per cent of Given x− x = 321 ⇒ x = 3210 100
125 = 150 ⇒ x = 120 per cent 100 100
 Choice (4) 150
 Choice (4) ⇒ 100 + x = 125 ⇒ x = 25 per cent
11. Let the income of Mahesh be Rs 100; The
5. Let the number of units be 100, the cost of  Choice (2)
amount spent on house rent = 20 per cent
each unit be Rs 100. of 100 = Rs 20 18. Let Z = 100, Y = 90, X = 108; X is 8 per
∴Total expenditure = Rs 100 × 100 = Rs The amount spent on food = 30 per cent cent more than Z. Choice (2)
10000. Let the number of units to be pur- of (100 – 20) = Rs 24 19. Number of questions attempted = 683/4
chased be x. The amount spent on education = 40 per per cent of 160 = 110
∴125 per cent of (Rs 100) × x = 90 per cent of (80 – 24) = Rs 22.4
cent of (Rs 10000) ∴Number of questions which are left
The amount spent on miscellaneous = 50 = 160 − 110 = 50; Number of incorrect
9000 per cent of (56 – 22.4) = Rs 16.8 questions = 91/11 per cent of 110 = 10.
⇒ x = = 72 ∴The consumption
125 Savings = 33.6 – 16.8 = Rs 16.8; The re- ∴Marks scored by the student = (100 × 5)
must be reduced by 28 per cent. Choice (2) 16.8 − (50 × 2) − (10 × 4) = 360
quired percentage = × 100 = 16.8 per
100 Maximum marks = 160 × 5 = 800;
6. Let the cost of the article be Rs 100. Cost
cent Choice (2)
of the article after increasing by 25 per 360
cent and decreasing by 25 per cent is 100 12. Number of persons voted “For” the reso- The percentage of marks = × 100 =
800
 25  25  80 45 per cent Choice (2)
1 + 1 −  = 93.75 lution = × 1500 = 1200
 100  100  100
20. Total Telephone connections as a percent-
Total number of voters after increase is
∴Net change = 100 – 93.75 = 6.25 per age of population = 40 per cent
3000.
cent less Choice (2) Illegal telephones form 12.5 per cent of
∴ For the resolution to be passed, total
7. Let the maximum marks be x. In case of the total connection (as 87.5 per cent are legal
number of voters required are 2/3rd of
first student pass marks = 30 per cent of x + connections).
3000 = 2000
30 → (1) Also, pass marks in case of second ∴ 800 new votes should vote “For” the ∴Illegal telephone connections of total
student = 40 per cent of x – 10 → (2) resolution; 800/1500 = 53.33 per cent population = 12.5 per cent of 40 ⇒ 5 per
Solving (1) and (2); x = 400; Pass mark =  Choice (3) cent Choice (1)
30 per cent of 400 + 30 = 120 + 30 = 150. 21. Let the quantity of solution be 100 litres.
13. Let B = 100 then A = 120; D will be 25
 Choice (2) Acid content is 70 litres and water content
per cent less than B. ∴ D = (0.75)100 =
8. a per cent (a + b) + b per cent ( b + a) = 75; If D is 162/3 per cent less than C, then is 30 litres.10 litres of solution is taken
(a2 + b2) per cent of 1 + ab per cent of x C will be 20 per cent more than D; ∴C out, Now, Quantity of acid = (70 – 10 ×
= 1.2(75) = 90; C as a percentage of A = 70
a
(a + b) +
b
(b + a) =
(a 2 + b2 ) + abx 90 100
) litres = 63. Quantity of water = (30
× 100 = 75 % Choice (1)
100 100 100 100 120 30
a
(a + b) +
b
(b + a) =
( a 2 + b 2 ) abx
+ ; 14. Number of students who passed = 240;
– 10 ×
100
) litres = 27
100 100 100 100 who failed = 60, Required percent =
If 10 litres of water is added, then it be-
a 2 + b 2 2ab a 2 + b 2 abx 180 comes 37 litres.
+ = + ;x=2 × 100%
100 100 100 100 240 ∴ The required percentage = 37 per cent.
 Choice (2) = 75 per cent Choice (2)  Choice (3)

08_Section.indb 56 10/30/2009 11:52:53 AM


Solutions for Mathematical Skills  n  57

22. Rs 750 pm should come from commis- 30. Difference of the population be- ∴D got the least hike. Choice (4)
sion tween 3rd year and 2nd year =
38. Given 30 per cent of x – 30 per cent of 30
⇒ 9000 p.a. 10000000 10000000
− = 30
5 per cent – 9000, 100 per cent – (1 + 25 / 100) 2 (1 + 25 / 100)3
1,80,000 30 30
= 12,80,000 Choice (2) ⇒ x− × 30 = 30 ⇒ x = 130;
∴Annual sales should be Rs 1,80,000 100 100
 Choice (2) 31. Since Mandara spends 20 per cent towards 130
37.5 rent, remaining income = 80 per cent. 130 per cent of x = × 130 = 169
23. Reduction on the whole = × 2000 = 100
100 Amount spent towards food = 75 per cent
 Choice (3)
Rs 750 of 80 per cent; 80 × 75 × 90 × 72 × 39. Let, A = 2B =3C = 4D = 5E = k
∴After reduction, one can buy 6 dozens 100 100 100 100
for Rs 750. x = 97,200 where x is the total salary ⇒ x k k

Price per dozen after reduction =
750
=
= Rs 2,50,000 Choice (4)  he required percentage = 3 4 × 100
T
k
6 32. Girls passed = 70 per cent of Total Girls
4
Rs 125. ∴42 = 70 per cent of Total Girls ⇒ Total
∴Price per dozen before reduction = Girls = 60; Boys failed = 4 (Girls passed) 4 1
per cent = × 100 per cent = 33 per
125 × 100 125 × 100 = 168; 12 3
= = Rs 200. Boys failed = 40 per cent of Total Boys; cent Choice (3)
100 − 37.5 62.5
168 = 40 per cent of Total Boys ⇒ Total 40. Let the length and the breadth be 100 m
 Choice (3) Boys = 420 each initially. Increase in length = 40 m;
Total number of students = 60 + 420 =
24. Given 75 per cent of 5x – 25 per cent of 480 Choice (3)
New length = 140 m. Decrease in breadth
4 = 20 m; New breadth = 80 m
33. In 2005, let Roberts income, expenditure Original area = 10000 m2;
144 = 24; 75 × 5x − 25 × 144 = 24 ⇒ x and savings be Rs 100x, Rs 50x and Rs Final area = 11200 m2
100 4 100 Percentage increase in the area =
50x.
= 64; 25 per cent of 64 = 16 Choice (2) In 2006, let his income, expenditure and 11200 −10000
×100 = 12 per cent
25. The numbers that end in 4 or 6 have the savings be Rs 100y, Rs 60y and Rs 40y 10000
last digit 6 in their squares. The number respectively.  Choice (2)
of such cases from 1 to 60 are 12. The Given that 100y < 100x ⇒ y < x; Percent-
age increase in Robert’s expenditure =
required percentage = 12 × 100 = 20 per
cent 60 Choice (2) 60y − 50x Profit and Loss
× 100; If y = x, the answer is
50x
26. Let the initial price be Rs x; Price after Practice Exercise
20 per cent, y < x, the increase is less than
increase = Rs 1.3x
Decrease in price = Rs 0.26x; Price after 20 per cent. Choice (2) Solutions for questions 1 to 40:
decrease = Rs 1.04x 34. Expenditure of X = Rs 36000; Savings of 100 840 × 100
2nd increase in price = Rs 0.416x; X = 60,000 − 36000 = Rs 24,000 1. C.P. = S.P. × = = Rs
(100 + P) (100 + 20)
Final price = Rs 1.456x; 1.456x = 2912 ⇒ If the ratio of expenditure and savings to
x = 2000 Choice (3) 700 Choice (2)
be reversed, X should reduce his expendi-
27. Let the required number of matches be x. ture by Rs 12,000. Choice (2) 2. Let the cost price of the article be Rs C; C
The number of matches won by the team = 35. Let the cost of one pencil be Rs x.; ∴ The  25 
1 −  = 243 ⇒ C = Rs 324
90 per cent of 40 = 36 cost of one pen is Rs 2x.; The cost of 20  100 
The required success rate = 80 per cent pencils = 20x; The selling price of the article as to gain
The cost of 30 pens = 30 × 2x = 60x;  25 
36 + x 36 + x 4 25 per cent profit = 324 +  × 324  =
∴ × 100 = 80; ⇒ = 20x  100 
90 90 5 The required percentage = × 100 per
80x Rs 405 Choice (3)
 36 + x = 72 ⇒ x = 36; ∴ It has to

cent = 25 per cent Choice (3) 3. Let the cost prices of two articles be Rs
win 36 more matches from the remaining
100x and Rs 100y.
matches. Choice (4) 36. Required percentage
The selling prices of the two articles are
50 2
28. Let the number of cows in the farm be x; = (100)% = 14 %  Choice (2) Rs 120x and Rs 95y. Given that, 120x =
400 − 50 7 95y
60 5 5
(hens) = x ; hens = x ; x + x = 48
100 3 3 37. Let initial salary of A, B, C and D be Rs x 95 19
⇒ = = ; Let x = 19k, y = 24k
⇒ x = 18 Choice (2) 100; Let x = 10 per cent, then x − 1 = 9 y 120 24
per cent
 P1 = 1900k, CP2 = 2400k; Profit on first
C
29. Let Krishna’s original income be Rs 110 × 109
A’s salary after hike = = 119.90; article = 380k and Loss on the second =
100x.
100 120k
His original expenditure = Rs 70x. Given 111 × 108
B’s salary = = 119.88; Overall profit = 380k − 120k = 260k;
70x = 17,500 ⇒ x = 250 100
Krishna’s new income = Rs 120x; His 260k
112 × 107 Overall profit percent = × 100 =
new expenditure = 70x + 7x = Rs 77x C’s salary = = 119.84; D’s salary 4300k
his new savings =120x − 77x = Rs 43x = 100 260 = 6 per cent
113 × 106 Choice (2)
43 × 250 = Rs 10,750. Choice (3) = = 119.78; 43
100

08_Section.indb 57 10/30/2009 11:52:54 AM


58  n  Solution Manual

4. Let number of pens bought be LCM of 9, 12. MP = 360, S.P. = 288, Loss = 4 per cent. 20. Let the marked price and cost price of the
6 = 18 pens ∴ C.P. = 300 article be Rs m and Rs c respectively
∴C.P of 18 pens = 10 × 2 = Rs 20 ∴Marked up percentage = 60/300 × 100  2
∴ S.P of 18 pens = 7 × 3 = Rs 21 = 20 per cent Choice (1)  16 3 
∴Gain percentage = 1/20 × 100 = 5 per cent ⇒ m 1 −  = 840 (given) ⇒
13. Let the CP be Rs 100x. SP = 65x = 1755  100 
 Choice (2)  
100
⇒ 100x = × 1755; ∴ CP = Rs 2700
5. Selling price of each article = Rs 7 ; Selling 65
In order to recover the loss, the second m (1 –
1
) = 840
6 6
article should be sold at Rs 945 more than
price of 20 articles = 20 × = Rs. 70
7
Rs 2700 = 2700 + 945 = Rs 3645. B sell- 50
6 3 m = Rs 1008 ⇒ c (1 + ) = m = 1008
3645 − 1755 100
70 ing price is more than A by (given)
CP = 18; SP = ; Profit percent = 1755
3 ∴c = Rs 672 Choice (1)
1890
70 × 100 = × 100 = 107.4 per cent
− 18 1755 21. Let the marked price fixed by Amarender
3 × 100 ≅ 30 per cent Choice (1)  Choice (3) be Rs 100.
18 80
14. Let cost price be Rs 100 ⇒ S.P. = 114.75 SP = Rs 80. Cost price = = Rs 64.
6. Given S.P of 10 articles = C.P of 11 ar- 1.25
= M.P.(0.85) × (0.9)
ticles. Cost price for Parmesh is the selling price
Let C.P. of 1 article be Rs 1 11475 × 10
M.P. = = Rs 150 ⇒ Mark up for Amarender.
∴S.P of 10 = Rs 11; ∴ S.P of 1 article 85 × 9 ∴C.P. = Rs 80; Selling price for Parmesh
= Rs 1.1 percentage = 50 per cent Choice (3) = Rs 80(1.2) = Rs 96
 1.1 − 1  Cost price for Maninder = Rs 96. The re-
∴Gain percentage =  × 100  = 15. Let the original cost price be Rs 100x.
 1  Original SP = Rs 110x. New CP = Rs 96 − 64
10 per cent Choice (1) 90x quired percentage = × 100 = 50
64
7. Let the CP be Rs 100x. Due to under valu- 1 per cent Choice (1)
New SP = 90x + (90x) = 105x;
ation, 75 (SP) = 120x ⇒ SP = Rs 160x 6 22. Let the cost price of the article A and B be
100 But 105x = 110x − 20 ⇒ x = 4
Rs x and Rs (6000 – x) given 20 per cent
If it is over valued by 25 per cent; SP = Original CP = Rs 100x = Rs 400
 Choice (3) of article A = 40 per cent of article B.
125 20 40
× 160 x = Rs 200x; Profit = Rs 100x; 16. 10 per cent loss + 7 per cent profit = Rs 85 ∴ ×x= (6000 – x) ⇒ x = Rs
100 100 100
⇒ 17 per cent of C.P. = Rs 85 ⇒ CP = Rs
CP = Rs 100x; 4000 Choice (3)
500 Choice (1)
Profit is 100 per cent. Choice (1)
17. Cost price of the article for Ramesh = 23. SP of walkman = Rs 810; Loss per cent =
8. Cost per dozen = x/12, S.P. per dozen = 10 per cent
x / 6 − x / 12 5000 × 85 × 75 = Rs 3187.50
x/6; Profit percentage = × 100 100 90 per cent of CP = 810 ⇒ CP = Rs 900;
x / 12 Gain per cent = 10 per cent ∴ SP = 110
Cost price of the article that Mohit pur-
100 =100 per cent. Choice (2) per cent of 900 = Rs 990 Choice (2)
90 85 85
9. Overall gain = 20 per cent of 30 = Rs 6. chased = 5000 × × × = Rs 24. 25 dozen at Rs 12 per dozen ⇒ Total cost
100 100 100
Selling price of 80 per cent of fruits = Rs price Rs 300
3251.25
36 The required difference = 3251.25 − Sales : 10 dozen at Rs 18/dozen = Rs 180
36; Selling price per kg = × 100 = Rs
80 3187.50 = Rs 63.75 Choice (2) As 3 dozen are rotten, remaining 12
45 Choice (4) dozens at Rs 1.25 per banana. 12 dozen
18. CP = Rs 100; Loss = 10 per cent;
10. Cost = 90000 + 30000 = Rs 120000, After at (1.25 × 12) = Rs 180; Total = Rs 360.
∴SP = Rs 90; 3/4 (Actual SP) = 90;
Depreciation cost = 75/100 × 1,20,000 = ∴Actual SP = Rs 120 (360 − 300)
Profit percentage = × 100 =
Rs 90000; SP = Rs 1,00,000 300
Profit percentage = [(120 − 100)/100] 100
∴Profit = 10000, percentage of profit per 20 per cent Choice (4)
= 20 per cent Choice (3)
cent = (10000/90000) × 100 = 11.11 per
19. Let the cost price of the article be Rs 25. Let the cost of each article be Rs x. Cost
cent Choice (3)
100. of 25 articles = Rs 25x.
11. Let the CP be Rs 100. Given that, 3 × SP Marked price = 100 + 40 = Rs 140;  2 5
4 SP of 25 articles = 1 +  × 25x = ×
Selling price = 100 + 12 = Rs 112  3 3
= (100 − 19) ⇒ SP = 108
140 − 112 125x
2 Discount percent = × 100 = 25x =
(SP) = 72; Loss incurred on A and B = 140 3
3
19 + 28 = 47 28  hen the remaining 15 articles are sold,
W
× 100 = 20 per cent
In order to gain 10 per cent on the whole, 140 there is a loss, which is equal to the SP of
on each of the three articles there should If the article is marked 50 per cent above 10 articles.
be 10 per cent gain. that is, 3 × 10 = 30 the CP., then MP = 150. CP of 15 articles − SP of 15 articles = SP
And the loss of 47 has to be recovered. of 10 articles. CP of 15 articles = SP of
80 25 articles.
Overall gain should be 30 + 47 = 77 SP = 150 × = 120; Profit = 120 − 100
C should be sold at 77 per cent gain. 100 CP of 15 articles = Rs 15x. SP of 25 ar-
 Choice (4) = 20 per cent Choice (4) ticles = Rs 15x.

08_Section.indb 58 10/30/2009 11:52:54 AM


Solutions for Mathematical Skills  n  59

15  P of 30 apples = Rs 36 (∵ gain = 20 per


S 40
SP of 15 articles = × 15x = 9x; CP of cent of 30); The number of apples sold at 50x − × 50x = 2000 ⇒ x = 66.66
25 100
40 articles = Rs 25x + Rs 15x = Rs 40x 1 ∴ At the maximum he can buy 66 articles.
20 per cent loss = × 90 = 30
125x 3  Choice (3)
SP of 40 articles = Rs + 9x =
3 For the next 30 apples, CP = Rs 30, SP 38. Loss percentage = 202/100 = 4 per cent
= Rs 24 loss Choice (4)
152 152x For the next 30 apples, as the apples are
x ; Overall profit = − 40x = (100 − x) (100 − x)
3 3 spoiled 39. Selling price = 250 × ×
CP = Rs 30; SP = Rs 0. Total CP = Rs 90; 100 100
32x
30 (100 − x) 2
3 Total SP = Rs 60; Loss per cent = × = ; Discount = MP − SP = 97.9
32x 90 40
(100 − x ) = 97.9
2

Overall profit percent = 3 × 100 = 1 ⇒ 250 −


40x 100 per cent = 33 per cent 40
 3 Choice (3)
32x ⇒ x = 22 per cent Choice (4)
× 100 = 262/3 per cent Choice (1)
120x 32. Let the CP be Rs 100. MP = 150; SP = Rs
135 (at 10 per cent discount). SP1 = Rs 40. C.P. = 360 × 100/90 = Rs 400, S.P. = 550
26. Let CP be Rs 100. MP = 130; – 550/11 = Rs 500 (91/11 per cent Dis-
Credit SP = 90/100 × 130 = 117; Cash 120 (at 20 per cent discount).
The difference of Rs 15 is equal to Rs count)
SP = 90/100 × 117 = 105.3; x = 100 × Profit percentage = (500 – 400)/400 × 100
106/5.3 = 2000 100.
105 × 100 = 25 per cent  Choice (3)
CP of the product is Rs 2000 Choice (1) The difference of Rs 105 = =
15
27. Let the C.P of 1 gm of goods be = Re.1
Total C.P of 960 gm = Rs 960; Total of Rs 700; CP = Rs 700 Choice (2) Partnerships
S.P of 960 gm = Rs 1000 33. Let the SP of each article be Re.1.
1000 − 960 SP of 25 articles is Rs 25; Gain = S.P of 5 Practice Exercise
Gain percentage = × 100 articles = Rs 5; CP of 25 articles is Rs 20
960 Solutions for questions 1 to 25:
5
40 1 Profit percent = × 100 = 25 per cent 1. Ratio of Profits = 5 × 4 : 6 × 5 : 8 × 6 = 10
= × 100 = 4 per cent Choice (1) 20
960 6  Choice (2) : 15 : 24; Given that 24x = 8880 ⇒ 5x =
28. Let SP be Rs 100. Profit percentage = 25 34. Let the cost prices of articles A and B be a 8880
× 5 = Rs 1850 Choice (4)
per cent and b respectively. Given, 125 per cent of 24
P = Rs 25 ⇒ CP = 100 − 25 = Rs 75. His a + 87.5 per cent of b = 12000 → (1) 2. A’s Share = (6/21) × 6300 = Rs 1800;
correct profit percentage = 25/75 × 100 = 75 per cent of a + 112.5 per cent of b = B’s Share = (7/21) × 6300 = Rs 2100;
331/3 per cent 12000 → (2) C’s Share = (8/21) × 6300 = Rs 2400
 Choice (3) Solving (1) and (2), we have b = Rs 8000  Choice (4)
29. Cost of 25 kg of sugar = 25 × 35 = Rs  Choice (3) 3. Ratio of profits of Mukund Rao and Nag-
875 35. Let the marked price be Rs 100; SP af- nadh = 357300 × 12 : 460000 × 9 = 1191
Cost of 15 kg of sugar = 15 × 15 = Rs ter three successive discounts = 100 × : 1150.
95 90 85 = Rs 72.675 Net profit = Total profit − Total expens-
875 + 225 × × es = Rs 5,13,200 − Rs (9 × 5000) = Rs
225; C.P. per kg of mixture = 100 100 100
25 + 15 Selling price after single discount of 30 4,68,200
1150
1100 per cent = Rs 70 Nagnadh’s share of profit = ×
= = Rs 27.5 2341
40 selling price = 130 per cent of 27.5 Given that 2.675 per cent of M.P. = Rs 468200 = Rs 2,30,000
Required
149.80; Total income of Nagnadh = 2,30,000 +
130 149.80 × 100 45,000 = Rs 2,75,000 Choice (3)
= × 27.5; S.P of each kg of sugar = Rs M.P. = = Rs 5600.
100 2.675 4. (6 : 5 : 4) × (x : y : z) = 4 : 5 : 8;
 Choice (2) Ratio of time periods = 4/6 : 5/5 : 8/4
35.75 Choice (3)
36. Let the cost price = Rs 100. ∴M.P = Rs (2/3) : 1 : 2 = 2 : 3 : 6; ∴ x : y : z = 2 : 3 : 6
6.30 × 100  Choice (3)
30. Cost price = = Rs 7, 160
90 Given discount percentages are 20 and 10 5. The total investment of Sainath for the 1st
 100  year = 1,00,000 + 95,000 + 90,000+…. +
S.P. = 7 × 1.2 = Rs 8.4, M.P. = 8.4  160 × 80 × 90
 ∴Selling Price = = Rs 50,000 + 45,000
 84  100 × 100
= Rs 10 Choice (3) 12
115.20 = (1,00,000 + 45,000) = Rs 8,70,000
2
31. Let the total number of apples be 90. The 15.20 The total investment of Sekar for the
Profit percentage = × 100 = 15.2
number of apples are sold at a gain of 20 100 first year = 5,000 + 10,000 + 15,000 +.…+
1 per cent Choice (2) 60,000 + 70000 × 12
per cent = × 90 = 30
3 37. Let he can buy x articles each of Rs 50. 12
= (5000 + 60000) = Rs 3,90,000 +
Let the CP of one apple be Rs 1. CP of 30 Total cost price = Rs 50x. Discount = 40 2
apples = Rs 30 per cent of 50x. 840000 = 1230000

08_Section.indb 59 10/30/2009 11:52:55 AM


60  n  Solution Manual

Ratio of investments = 87 : 123 = 29 : 41; 5


29 ∴ The share of Q = (1.26) lacs = 0.45
Share of profit of Sainath = × 140000 14 100 × 12
⇒ 100 × 12 = x × 15 ⇒ x = = 80
70 lacs = Rs 45000 Choice (2) 15
= Rs 58000 Choice (4) 13. Y withdrew an amount of Rs 2000 from  Choice (2)
6. 10000 × 12 = 15000 × x ⇒ x = 8 his investment after 6 months. ⇒ His in- 22. x investment for four quarters = 1000 +
∴Ajay left the business after 8 months. vestment is Rs 8000 for 6 months and Rs 3000 + 5000 + 7000 = Rs 16000,
 Choice (4) 6000 for remaining 6 months. Also given, y investment for four quarters = 4000 +
4 Z joined the business after 6 months with 3000 + 2000 + 1000 = Rs 10000,
7. Shyam’s profit share = (81000) = Rs an investment of Rs 6000.; Ratio of their
9 Ratio of investment of x and y is 16000 :
36000. investments = [12 × 5000] : [(8000 × 6) + 10000 = 8 : 5
Shyam’s profit share if his salary is ex- (6000 × 6)] : [6000 × 6] = 60 : 84 : 36 = x’s share = 8/13 × 39000 = Rs 24000; y's
5:7:3 share = 5/13 × 39000 = Rs 15000
3 7
cluded = (45000) = Rs 27000. ∴ Y’s share = × 9615 = Rs.4487  Choice (3)
5 15
Shyam’s salary = 36000 − 27000 = Rs  Choice (3) 23. Face value of 1 share = Rs 100; Dividend
9000. Choice (3) 14. ∴ The share of Bal is 1/3 of total profit as = 12 per cent
he invested 1/3 of capital. So Bal’s share ⇒ Income from 1 share = Rs 12
8. Given P, Q and R’s capital ratio = 3 : 2 : 4
∴Total income from 140 share = Rs 140
and profits ratio = 6 : 3 : 4. = 1 × 75000 = Rs 25000 Choice (2) × 12 = Rs 1680 Choice (1)
We know that Profit α (Time) (Amounts); 3
24. Profit after salary and commission =
Pr ofit 15. Market value of 1 share = Rs (25 + 20 per
∴Time = 87000 – 12000 – 7500 = Rs 67500
Amount cent of 25) = Rs 30; Income from 1 share
Ratio of incomes of earth, moon and sun
∴ P, Q and R’s time periods ratio = = 20 per cent of 25 = Rs 5
=3:5:7
Number of shares to be bought to get Rs
Moon’s share in the profit of Rs 67500
6 3 4 1200
: : 1200 income = = 240; ∴Amount to = 5/15 × 67500 = Rs 22500
3 2 4 5 Income of Moon = 22500 + 7500 = Rs
= 4 : 3 : 2 Choice (1) be invested = Number of shares × Market 30,000 Choice (4)
value of 1 share = 240 × 30 = Rs 7200
9. The ratio of their profits at the end of the 25. We have, Face value × Rate of dividend =
 Choice (3)
year = 4500 × 12 : 5500 × 12 : 7000 × 6 = Market value × Rate of return; Let rate of
9 : 11 : 7 16. 50000 × 12 : 60000 × (12 – x) : 70000 return be r per cent p.a.
The total profit share of Sushma and Hari- (12 – x) = 20 : 18 : 21; Simplifying Market value = Rs (100 + 60) = Rs 160
(9 + 11) 27000 = Rs 20000 50000 × 12
=
12 ⇒ 100 × 12 = 160 × r ⇒ r = 7.5 per cent
ka = ( ) 60000 × (12 − x) 8  Choice (2)
27
 Choice (3) ⇒ x = 3 Choice (2)
10. The ratio in which Chiru and Pavan share 17. Let Tushar’s rate of return be r1 per cent Simple Interest and
the total profit = 15000 : 45000 = 1 : 3; and Bhupesh rate of return be r2 per cent.
Let the total profit be Rs x 25 × 16 = 40 × r1 ⇒ r1 = 10 per cent; 10 Compound Interest
× 20 = 16 × r2
20
(x) = 4800 (given) ⇒ x = Rs 24000; ⇒ r2 = 12.5 per cent Practice Exercise
100 ∴r2 > r1, Bhupesh’s investment is better.
The profit that is to be shared between Chiru Solutions for questions 1 to 40:
 Choice (2)
and Pavan = 24000 – 4800 = Rs 19200 1. Let the sum be Rs P, N = R; Simple inter-
18. X’s investment in 12th month = 250 + (12
∴ The difference in the shares of the prof-
– 1) 250 = Rs 3000  R  1
3 1 Y’s investment in 12th month = 3000 + (12 – est = P   (R) = P ⇒ R = 5
its of Chiru and Pavan =  −  19200  100  4
4 4 1) (–250) = Rs 250  Choice (1)
= Rs 9600 Choice (3) As their capitals for the year are same,
they share the profits in 1 : 1 ratio. 2. P × 5/2 × 19/100 = 9500; P = Rs 20,000
11. Given that, Investment made by Subash =  Choice (2)  Choice (2)
9 (x) = Rs 9x
19. Face value of 1 share = Rs 100; Market 3. The difference in the amount in one year
Investment made by Harsha = 12 (y) =
value of 1 share = Rs (100 + 15 per cent is Interest for one year = 783 – 702 = Rs
Rs 12y; profits earned are equal ⇒ in-
of 100) = Rs 115 81; Interest for two years = 2 × 81 = Rs
vestments must be equal ⇒ 9x = 12y ⇒
Face value × dividend rate = Market val- 162; Sum = 702 − 162 = 540. Now 540
x 12 ue × Rate of return ⇒ 100 × x = 115 × 10 R
= ⇒x:y=4:3 ×1× = 81 ⇒ R = 15 per cent p.a.
y 9 Choice (3) ⇒ x = 11.5 per cent Choice (4) 100
 Choice (2)
20. A’s total investment = 50000 × 6 + 60000
12. Let the investments of P, Q and R be Rs 4. Amount after 6 years = Principal + 6
× 6 = Rs 660000
(x – 9) lacs, Rs (x – 3) lacs and Rs x lacs years interest = 20400 → (1)
B’s total investment = 50000 × 6 + 60000
Given, x – 9 + x – 3 + x = 42 ⇒ x = 18
× 6 = Rs 660000 Amount after 3 years = Principal + 3years
lacs
Ratio of profits = 660000 : 660000 = 1 : 1 interest =17700 → (2)
∴ The ratio in which P, Q and R share the
 Choice (2) (2) – (1) ⇒ 3I = 2700 ⇒ I = 900;
total profit is (x – 9) : (x – 3) : x = (18 – 9)
21. We have, Face value × Rate of dividend = P = 17700 – 2700 ⇒ Rs 15000;
: (18 – 3) : 18
= 9 : 15 : 18 = 3 : 5 : 6 Market value × Rate of return. Let market r = 100 × 900/15000 = 6 per cent
value be Rs x  Choice (2)

08_Section.indb 60 10/30/2009 11:52:55 AM


Solutions for Mathematical Skills  n  61

5. Let the sum borrowed be Rs n. 13. Let the sum be Rs x; S.I1 = 19. Let x be the amount paid at the end of the
25 x × 3× 6 18x ; third year.
Rate of interest = per cent = Rs S.I2 = 25,000(1.3) = 10,000(1.2) + 10,000(1.1)
4 100 100
n
+ x ⇒ x = Rs 9500. Choice (3)
 17  x × 3× 8 24x
Let the time be n years ⇒ 2 =   = Rs 20. Let principal be Rs 100; 100 × 1 × 10/100
 16  100 100 = 10 (Interest for 1st six months)
∴ Interest is 2x – x = x Given that S.I2 – S.I1 = 750 ⇒ 110 × 1 × 10/100 = 11 (Interest for 2nd
x × 25 × n 24x 18x 6x six months); Total interest = 10 + 11 = 21
∴x= ⇒ n = 16 Choice (1) − = 750 ⇒ = 750 ⇒ x =
400 100 100 100  Choice (1)
6. (Interest at 16 per cent) – (Interest at 12 21. The sum borrowed by Raju = Rs 15,000;
750 × 100
per cent) = 500 = 12500  Choice (3) After 3 years, Raju has to pay an amount
6
 3 × 15 
P × 2 × 16 P × 2 × 12 2 equal to = 15000 1 +  = Rs
− = 500 ⇒  r   r   100 
14. Given P 1 +  − P 1 +  = 2000
 100 100
 100   100  21,750
P×2×4 500 × 50 Interest gained by Mahesh = 21750 −
= 500 ⇒ P = = Rs 2
100 4  25   25  15000 = Rs 6750
⇒ P 1 +  − P 1 +  = 2000
6250 Choice (3)  100   100  Let the sum lent by Raju at 20 per cent
7. Let the sum borrowed be Rs x. Let rate of p.a. be Rs ’x’. Then the sum lent at 12 per
⇒ P = Rs 6400 Choice (2)
interest = number of years be n. cent p.a. would be Rs (15000 − x).
15. Total amount A = x × 20 × 3 (15000 − x) × 12 × 3
9 + =
Interest for n years is x.  TR TR TR  100 100
16 P 1 + 1 1 + 2 2 + 3 3  = 50000
 100 100 100  6750
9 x×x×x 15 1 ⇒ x = 5625 Choice (1)
∴ x = ⇒ x = = 7
16 100 2 2  3 × 10 2 × 5 3 × 8 
1 + 100 + 100 + 100  = Rs 22. Interest = 12 per cent per year; Interest
years Choice (1)
8. Let each instalment be Rs 100. Present 82000. Choice (1) per quarter = 12 = 3 per cent; Interest for
4
values of Rs 100 each at the end of the 16. P becomes 2P in 9 years. 2P becomes 4P 2 quarters or six months = 3 per cent + 3
1st, 2nd and 3rd years will be equal to Rs in another 9 years. 4P becomes 8P in an-
120, Rs 110 and Rs 100 respectively. An- other 9 years. per cent + 3 per cent of 3 per cent = 6 +
nual instalment of 100 discharges a debt 8P becomes 16P in another 9 years. 0.09 per cent = 6.09 per cent
of Rs 330. ∴P becomes 16 times in 36 years.
Annual instalment of × discharges debt of 6.09
 Choice (4) 6.09 per cent of 5000 = × 5000 = Rs
Rs 1650. 100
∴x = 1650 × 100/330 = Rs 500 Choice 17. Cost price of scooter = Rs 25000; 304.50 Choice (3)
(1) Down payment = Rs 9600; Balance = Rs

9. Interest paid at the end of the five years =


15400 23. P × r × t = 2 P ⇒ r ×15 = 2 ⇒ r = 200
Let Rate of interest be = x per cent 100 100 15
10 10 10 10 10 shopkeeper’s side, total value =
(2,00,000)+ (1,80,000)+ (1,60,000)+ (1,40,000F)+rom (1,20,000 ) = 131/3 per cent Choice (3)
100 100 100 100 100
10 10 10 10 10 10 x
(2,00,000)+ (1,80,000)+ (1,60,000)+ (1,40,000)+ (1,20,000) 15400 + 15400 × × = 15400 + 24. Difference between second and third
100 100 100 100 100 12 100 years interest = Rs 216 ⇒ 216 = interest
= 80,000 Choice (3) 1540x on Rs 1440 for 1 year
→ (1)
I ×100 1050 ×100 12 1440 × r
10. P = = ⇒ P = 1000. 216 = ⇒ r = 15 per cent
N×R 5 × 21 From Buyer’s side, total value = 1650 × 100
 Choice (3)
∴Rs 1000 will fetch an interest of Rs 1050 x 1
10 + 1650 × × × 45 = 16500 + 165 25. Let the rate of interest be R per cent p.a.
 Choice (2) 12 100
11. Let the rate of simple interest be R per  R 
3 Extra simple interest = (10000)  
cent p.a. Let the sum be Rs p. Let the re- × → (2)  100 
8
quired time be t yeaRs 165 × 3x (2) = 2000 ⇒ R = 10 Choice (3)
Equating (1) and (2), 16500 +
 R  8 26. SI for 1 year at 4.5 per cent pa. =
2p – p = p   (10) ⇒ R = 10;
 100  1540x 50000 × 4.5 = Rs 2250; C.I per annum
= 15400 + ⇒ x = 16.5 per cent
 R  12 100
3p – p = p   t ⇒ t = 20 Choice (4)  Choice (4) = 4 per cent;
 100 
18. Let the sum be 100. 5 per cents at the end C.I. every six months = 4/2 = 2 per cent
12. Let each instalment be Rs 100. Present val- of 1st year Rs 100 amounts to Rs 105 Total C.I. for the period = 2 per cent + 2
ues of Rs 100 each at the end of the 1st, 2nd 6 per cents at the end of 2nd year Rs 105 per cent + 2 per cent of 2 = 4.04 per cent
and 3rd years will be equal to Rs 120, Rs amounts to Rs 111.3 CI for 1 year compounded every 6 months
110 and Rs 100 respectively. An annual pay- 10 per cents at the end of 3rd year Rs
50000 × 1 × (4.04)
ment of 100 discharges a debt of Rs 460 111.3 amounts to Rs 122.43 = = Rs 2020
What annual payment discharges a debt of 100 amounts to Rs 122.43. ?(x) amounts 100
Rs 2300; Annual payment = (2300/460) × to Rs 12243 ⇒ x = 12243/122.43 × 100 B loses 2250 − 2020 = Rs 230
100 = 500 Choice (3) ⇒ 10,000 Choice (1)  Choice (1)

08_Section.indb 61 10/30/2009 11:52:56 AM


62  n  Solution Manual

27. Let the difference be R per cent; (2000) 2 From (1), P(1.25)2 [1.25 − 1] = 5000 ⇒
 R  p 1 + R  = 4320 → (1); P = 12800
(2)   = 20 ⇒ R = 0.5 Choice (1)  100 
 100  The total interest for first two years =
2
28. Let the Individual shares of X & Y be x1  R + 10  25 
2
p 1 +  = 5070 → (2) 
and y1. x1(1 + 4/100)5 = y1(1 + 4/100)3 100  12800 1 + − 12800 = Rs 7200.
  100 
x1/y1 = (100/104)² ⇒ 625/676;
x1 = 13010 × 625/1301 = 6250 2  Choice (2)
 R + 10 
y1 = 13010 × 676/1301 = 6760Choice (2) 1 +  5070 38. Let the sum invested at C.I be Rs x
100 
 iving (2) by (1), 
D 2
=
29. We have, A =  R  4320 2
1 +   10  121
R  R   R   100  ∴ A = x 1 +  =x+ = 1.21x.
  100  100
P 1 + 1 1 + 2  1 + 3 
 100  100   100  R + 10   30 
2 2
Let the sum invested at simple interest

⇒ 162864 = 125000 1+   1 + 100 
169  100 be y.
= ;   =  ; Amount received after 2 years at 10 per
 4  8   R3  144  1 + R   1 + 20 
1 + 1 +  1 +     
20
 100  100   100   100   100 
cent = y + y = 1.2y
100
162864 26 27  R 3  R = 20 Choice (2)
⇒ = × × 1 +  ⇒ R3 = It is given that the amounts he received
125000 25 25  100  35. P = 80,000; R = 7 per cent p.a. Amount from both the places are equal.
16 per cent Choice (2)  7  x 1.2
at the end of first year, 80000 1 + = ∴1.21x = 1.2y ⇒ = ;
30. Let Rs 100 be principal. Interest for half-  100  y 1.21
year = 5 per cent Rs 85600
At the end of first year let the amount paid 1.21
A = 100(1 + 5/100)2 = Rs 110.25; Interest ∴y = × 84,350 = 42350 Choice (2)
by him be Rs ’k’. 2.41
for the year = Rs 10.25. Effective rate of 2
∴ Amount to be paid after second year is  10 
interest = 10.25 per cent. Choice (4) 39. Amount after 2 years = 25,000 1 + 
 7   100 
R 
3 (85600 − k) × 1 + = 48792 ⇒ k =

31. 2315.25 = 2000 1 +  ⇒
231525  100  = Rs 30250
 100  200000 40000 Choice (2) Amount paid = Rs 15000; Balance = Rs
3 3 3 15,250
 R   21   R  36. P becomes 2P in 3 years. 2P becomes 4P Amount to be paid to clear the loan
= 1 +  ⇒   = 1 +  ⇒ in another 3 years.
 100   20   100  = 15,250 (1.1)2 = 15250 × 1.21 = Rs
4P becomes 8P in another 3 years. 18,452.50 Choice (4)
R = 5 per cent Choice (3)
∴We get 8 times the sum in 9 years.
32. A’s interest = PTR = 5840 × 3 × 1 = Rs  Choice (4) 40. I = 9 P ; A = principal + Interest = P +
100 100 3 2 16
 k   k 
1752 37. Given, P 1 + − P 1 + =
 100   100  9
P =
25
P
5000 16 16
730 × 12.5 × 20
B’s interest = = 73 × 25 2 2 2
100  k   k    r  25  r 
⇒ P 1 + 1+ − 1 = 5000 A = P 1 +  ⇒ P = P 1 +
= Rs 1825, Total interest = 1752 + 1852 =  100    100    100  16 

100 
Rs 3577. Choice (2)
k r 25
33. Let the sum borrowed be Rs x. 31,360 Let 1 + = T ⇒ PT2 [T − 1] = 5000 → ; 1+ = ⇒ r = 25 per cent p.a.
100 100 16
2
 R   Choice (2)
= x 1 +  → (1); 35,123.20 = x  k  
4
k 
3

 100  (1); P 1 + − P 1 + = 6250


 100   100 
3
 R  Numbers
1 +  → (2) ⇒ PT3[T − 1] = 6250 → (2)
 100 
1 5000 Practice Exercise
R 
1 (1) ÷ (2) Gives, = ⇒ T = 1.25
 T 6250 Solutions for questions 1 to 30:
Dividing (2) by (1), 1 +  =
 100  k
where T = 1 + 1. 2562 + 1442 + 2 × 256 × 144 – 256 ×
35123.20 3512320  R  100 144
= ; 1 +  =
31366 3136000  100  = (256 + 144)2 – ((16) (12))2 = 160000 –
k
1 + = 1.25 ⇒ k = 25 per cent. 36864 = 123136 Choice (1)
100
112
⇒ R = 12 per cent
100 2.
Now, substitute R = 12 in equation (1),
2
Number Sum of odd place Sum of even Difference No, to be added
 112  digits(1) place digits(2) between the to make it
31360 = x   ⇒ x = Rs 25,000
 100  sums (1) and (2) divisible by 11.
 Choice (1) (a) 35126 10 7 3 8 ∴Choice (3)
34. Let the sum be Rs p. Let the rate of inter- (b) 256345 13 12 1 10 ∴Choice (4)
est be R per cent p.a.

08_Section.indb 62 10/30/2009 11:52:57 AM


Solutions for Mathematical Skills  n  63

3. 53 is a prime number. ∴ All numbers (ii) If the factors are distinct, number of 20. 615 × 925 × 546. Here 6 is an even number.
between 67 and 87 are co prime to 53. 1 even number × 5 is a multiple of 10.
 Choice (2) ways = [(6 + 1) (2 + 1) – 1] = 10 ∴The units digit is zero. Choice (2)
2
4. (a) xn – yn is always divisible by x – y.  Choice (2) 21. 24 = 3 × 23. Largest power of 24 = Small-
Hence 9n – 1 is always divisible by 8 est common power of 3 and 23. As the
12. 6 5 3 Number = [(1 × 5 + 1) 6 + 1] =
(= 9 – 1), for all values of n. Choice power of 23 in 400! will be less than the
37.
(3) power of 3,
1 1 1
(b) xn + yn is divisible by x + y, when n required power = largest power of 23.
37 when divided by 18 leaves the remain-
is odd. 2397 = 2(23)132.
der 1. Choice (1)
hence 2n + 1 is divisible by 3(= 2 + ∴ Largest power of 24 = 132. Choice (2)
1) for odd values of n. Choice (2) 13. Let a = 0.48, b = 0.42 and c = 0.4.
22. A pair of numbers is relative prime if their
(c) xn – yn is divisible by x + y, when n The required value is
HCF = 1
is even.
a 3 + b3 + c3 − 3abc (a) For 36, 35, HCF = 1
hence 3n – 1 is divisible by 4 (= 3 + 1) =
a + b 2 + c2 − ab − bc − ca
2 ∴They are relative primes.
for even values of n. Choice (1)
(b) For 119, 255, HCF = 17.
5. The given number is divisible by both 16 (a 2 + b 2 + c2 − ab − bc − ca) (a + b + c) ∴They are not relative primes.
= (a
and 3. (a 2 + b 2 + c2 − ab − bc − ca) (c) 636, 54324 are divisible by 4,
56y6 is divisible by 16. ∴ y must be 1 + b + c) = 1.3 Choice (2) ∴They are not relative primes.
or 9 (d) For 221, 300; HCF = 1.
2 + 3+ 4 + 5 + x + 4 + 5 + 6 + y + 6 = 14. 560 = 24 × 51 × 71. For a number to be a ∴They are relative primes.
35 + x + y perfect square all the prime factors should (e) For, 65, 93 HCF = 1
y ≥ 1. ∴ 35 + x + y ≥ 36 + x. This must be have an even exponents. Hence multiply ∴They are relative primes.
divisible by 3. 24 × 51 × 71 with 51 × 71 to make the  Choice (4)
∴Minimum possible value of x = 0. powers even. Hence 560 should be mul- 23. 54N – 44N = 625N – 256N. It is divis-
∴ Required sum = 1. Choice (2) tiplied with 35(= 51 × 71) to make it a ible by 625 – 256 = 369 = 9 × 41. So it is
6. 3odd, when divided by 8, remainder is 3 perfect square. Choice (3) divisible by 9 and 41. Choice (3)
 Choice (1) 15. Let f(x) = x40 + kx39 + x38 + 2x37 – 2; f(x) 24. When 50 is successively divided by 3, we
7. All perfect squares have an odd number is divisible by x – 1. get the quotients as 16, 5 and 1 respectively.
of factors. There are 9 positive perfect ∴ f(1) = 0 ∴ Highest power of 3 in 50! is 16 + 5 + 1
squares less than 100. Choice (1) f(1) = 2 + k = 0 ⇒ k = –2 Choice (2) = 22 Choice (2)
16. (a) 66 × 435 + 66 × 565 = 66 × 1000 = 25. Three digit perfect cubes are 125, 216,
287 (2 )
3 29

8. = 3 . Let 23 = x; By Remainder 66000  Choice (4) 343, 512 and 729.


9 2 +1 (b) (3 + 2 – 5 + 2) + (6/24 + 18/24 – As ABC as well as CAB are perfect cubes,
theorem, when x29 ÷ (x + 1), remainder is 20/24 + 20/24) = 2 + 24/24 = 2 + 1 = ABC can be 125 only. Choice (4)
f (−1) 3 Choice (2)
26. Let the number be qk + 16. Twice the
f (−1) = (−1)29 = −1 ⇒ −1 + 9 = 8Choice 1.05 9.1
(c) × = 1.225 Choice (3) number is 2qk + 32
(3)
3 2.6 ∴ Divisor is (32 – 5) = 27. Choice (4)
x (x + 1) 17. Units digit of 5342712345 = units digit of
9. T = ∑x = 2
⇒ 120 = 712345
27. N (N2 − 1) = (N − 1) N (N + 1) = product
of three consecutive natural numbers. The
(15) (16) = units digit of 71 + (4) (3086) = units digit of product of any three consecutive natural
71 = 7 (∵(Powers of 7 follow a) cycle of numbers is divisible by 6, but not neces-
2
4) sarily by any greater number.
(20) (21) (30) (31) Units digit of 7612968 = units digit of 2(4)  Choice (4)
210 = ; 465 = (242)
2 2
As the power of 2 is divisible by 4, units 28. Number of consecutive zeroes is the high-
; Choice (4) cannot be expressed in the
digit of 7612968 = 6. est power of 5 in 200! = 40 + 8 + 1 = 49
x (x + 1) ∴ Required units digit = 1. Choice (4)  Choice (1)
form of . Choice (4)
2 29. 120 = 31 × 23 × 51; The different number
10. By using pattern method when 21, 22, 23 18. 0.479 = 479 − 4 = 475 = 95 of rectangles = Number of ways 120 can
and 24 are divided by 5, the remainders 990 990 198
obtained are 2, 4, 3 and 1 respectively. be represents as a product of two numbers
 Choice (3)
275 ≅24 × 18 + 3 = 23 = 8, 1 1
8 ÷ 5, R = 3. Choice (2) 19. Number of times the keys have to be = [(1 + 1) (3 + 1) (1 + 1)] = × 2 × 4 ×
2 2
pressed to type the single digit natural
11. 576 = 242 = (23 × 3)2 = 26 × 32 2 = 8 ways Choice (4)
numbers = 9. Number of times the keys
(i) Number of different ways of express- have to be pressed to type the two digit 30. 2940 = 22 × 3 × 5 × 7². For a number to be
ing 576 as a product of two factors = natural numbers = (90) (2) = 180. There a perfect cube the powers of all the prime
1 are 101 three-digit natural numbers. Num-
[Number of factors of 576 + 1] = factors should be divisible by 3. Hence,
2 ber of times the keys have to be pressed multiply 22 × 3 × 5 × 72 with 2 × 32 ×
to type the three digit natural numbers = 52 × 7 to make the power divisible by 3.
1 (101) (3) = 303. Hence 2940 should be multiplied with
[(6 + 1) (2 + 1) + 1] = 11.
2 ∴ Total number of times the keys to be 3150 (= 2 × 3² × 5² × 7) to make it a per-
 Choice (2) pressed = 492. Choice (3) fect cube. Choice (3)

08_Section.indb 63 10/30/2009 11:52:58 AM


64  n  Solution Manual

∴ Total number of resulting pieces = ∴ N = 26


L.C.M. and H.C.F.
3.6 + 3.9 + 4.2 + 4.5 General
 form of the number = KLCM
= 54. Choice (2) (9, 7) + 26 = 63K + 26 where K is a natu-
Practice Exercise 0.3
ral number.
Solutions for questions 1 to 30: Largest three digit number is obtained
8. LCM × HCF = product of numbers ⇒ 14
1. Let the two numbers be 12a and 12b × LCM = 2352 ⇒ LCM = 168 when 63K + 26 < 1000 and K is maxi-
where a and b are co-prime.  Choice (3) mum.
∴144ab = 12 × 1800 ⇒ ab = 150 9. The general form of the natural number = 29
K < 15 ⇒ K = 15. Largest three digit
The possible pairs are (1, 150) (2, 75) KLCM (6, 8) – 2 = 24K – 2 63
(3, 50) (6, 25) The largest three-digit number must satisfy, number = 971. Choice (1)
Only four possible pairs can exist. 24K – 2 < 1000 and K must have the larg-
 Choice (1) est possible value. 16. As difference between the divisors and
remainders is same that is (12 – 1, 15 – 4)
2. (a) 24 = 23 × 3, 36 = 32 × 22, 54 = 33 ×
501 3 = 11
2 K< = 41 ; ∴ Largest possible value
∴ LCM = 23 × 33 = 27 × 8 = 216. 12 4 LCM of (12, 15) = 60; Smallest number =
HCF = 2 × 3 = 6 Choice (2) of K = 41 60 – 11 = 49 Choice (2)
(b) 99 = 3² × 11; 165 = 3 × 11 × 5 ∴ Largest number = (24) (41) – 2 = 982 17. By HCF model 2, greatest number = HCF
HCF = 11 × 3 = 33, LCM = 32 × 5 ×  Choice (2) (556 – 6, 888 – 8, 1110 – 10) = HCF (550,
11 = 495 Choice (3) 10. The least number = (LCM of 15, 17) + 880, 1100) = 110 Choice (2)
LCM (5, 4, 6) 60 (Remainder) = 255 + 3 = 258 Choice (2)
(c) LCM= = = 60 18. The difference between any divisor and
HCF (6, 7,11) 1 11. Let the least four digit number be N. its corresponding remainder is one.
Each of the first two divisors exceeds the ∴ Required number = LCM of (1 to 10) – 1
HCF of numerators
H C F = = remainder it leaves by the same value = 2520 – 1 = 2519 Choice (2)
LCM of deno min ators which is 3.
∴ N = k LCM (10, 11) − 3 = 110k − 3 19. Let the minimum number of chocolates
HCF (5, 4, 6) 1
=  Choice (2) ≥ 1000 with Ajay be N.
LCM (6, 7,11) 462
Least possible value of k = 10 N must be of the forms of LCM (60, 72,
(d) LCM = 15/1, HCF = 1/140 ∴ If k = 10, 110k − 3 = 1097. Choice (4) 90) + 14 = 17b
 Choice (1) N = 360 a + 14 = 17b, where a and b are
12. LCM of 18, 24, 27, (18 = 32 × 2, 24 = 23
(e) 18 = 32 × 2, 35 = 7 × 5 3a + 14
× 3, 27 = 33) natural numbers. b = 21a +
LCM = 9 × 2 × 7 × 5 = 630, HCF = 1
∴ LCM = 33 × 23 = 216; 17
 Choice (3)
∴ Least number = 216 + 9 = 225.
3a + 14
LCM (numerators) Largest 3-digit number = 216 × 4 + 9 = 873 For b to be a natural number,
3. LCM = = 17
HCF(deno min ators)  Choice (4)
must be natural number. Minimum value
LCM (7, 41, 71) 13. By LCM model 2,
= 20377 smallest number = LCM (8, 6) – 2 = 22 3a + 14
HCF (8, 7, 9) of a for which is a natural num-
General form of the number = KLCM (8, 17
HCF (numerators) 6) + 22 (where K is a whole number) = ber is 1.
HCF = =
LCM (deno min ators) 24K + 22 ∴ Minimum number of chocolates he had
1 = 374. Choice (2)
HCF (7, 41, 71) 1 100 < 24K + 22 < 1000 ⇒ 3 <K<
= 4
LCM (8, 7, 9) 504 20. 32009 is an odd number, ∴ 32009 – 1 and
3 32009 + 1 are even numbers. But they differ
1 40
∴ LCM − HCF = 20377 – = 20376 4 by 2
504 ∴ K has 37 possibilities. Choice (3) ∴ 32007 – 1 and 32009 + 1 are consecutive
503 even numbers.
 Choice (4) 14. The number which would be of form
504 ∴ HCF of 32009 – 1 and 32009 + 1 is 2.
L.C.M. (27, 15, 18) k + 6 = 270 k + 6;
4. Let the numbers be 15a and 15b where a  Choice (2)
if k = 9, then the number would be = 270
and b are co-prime ⇒ 15a × 15b = 2700 (9) + 6 = 2430 + 6 = 2436. But the given 21. Let the numbers be KA and KB where A
⇒ a × b = 12 = 1 × 12 = 2 × 6 = 3 × 4 number is 2434. and B are co primes with A < B. Sum = K
⇒ The numbers could be (1 × 15, 12 × (A + B) and LCM = KAB
∴The least number that is to be added is 2.
15) and (3 × 15, 4 × 15). Hence only 2 K (A + B) = 100 → (1); KAB = 120 → (2);
 Choice (4)
pairs are possible. Choice (2) Dividing (1) by (2),
15. Let the least number satisfying the given
5. Let the largest number be N. N must di- A+B 5 2+3
conditions be N. = =
vide 122 − 2 = 120 and 134 − 2 = 132.
Let the quotients obtained when N is di- AB 6 2.3
As N is the largest, N = H.C.F. (120, 132)
vided by 9 and 7 be q1 and q2 respectively.
= 12.  Choice (4) Comparing both sides, A = 2 and B = 3.
N = 9q1 + 8 = 7q2 + 5; 9q1 + 3 = 7q2 ⇒
6. HCF of (121 – 63), (208 – 121) that is, From (1) or (2), K = 20.
3(3q1 + 1) = 7q2
HCF of 58, 87 = 29 Choice (2) ∴ Numbers are 40 and 60.
The LHS of the above equation must be
∴ Sum of the digits of the smaller num-
7. Weight of each piece must be maximum divisible by 7.
ber = 4. Choice (1)
and also a factor of 3.6, 3.9, 4.2 and 4.5. ∴ 3q1 + 1 must be divisible by 7.
∴ it must be the HCF of 3.6, 3.9, 4.2 and Least value of q1 satisfying this condition 22. Let the two numbers be a and b and their
4.5 that is, 0.3 is q1 = 2. LCM and HCF be l and h respectively.

08_Section.indb 64 10/30/2009 11:52:58 AM


Solutions for Mathematical Skills  n  65

Given, l = 12h, we have, l × h = a × b ⇒  otal score of all = (12) (55) = 660; 7th
T
12h2 = 84b ⇒ h2 = 7b → (1) Averages − Mixtures − highest score = (420 + 300) – 660 = 60
Alligations  Choice (3)
Also, l + h = 364 ⇒ 13h = 364 ⇒ h = 28;
14 × 57 + 3 × 63 − 66 − 51
28 × 28 Practice Exercise 9. New average cost =
Substituting in (1), b = ⇒b= 14 + 3 − 2
7 Solutions for questions 1 to 40: = 58
112 Choice (4) 1. Taking averages as 15, we get deviations ∴New average cost is Rs 58 Choice (1)
23. 3078 – 3 = 3075, 3906 – 6 = 3900. –4 – 3 – 2 – 1 + 0 + 1+ 2 + 3 + 4 + 5 = 10. Let the number of boys be b.
∴The greatest number is the HCF of 5/10 = 0.5 (or) ∴ Average = 15 + 0.5 = Total weight of all students = (56.8) (b) +
3075 and 3900 = 75. Choice (2) 15.5 Choice (2) (53.2) (462) = (54.6) (b + 462) ⇒ b = 294
24. LCM of (8, 12, 16) = 48, 1854/48 remain- 2. Third number = (3)(20) – 22 – 26 = 12  Choice (2)
der = 30;  Choice (3) 11. Let x be the strength of the class. Due to
∴Smallest number that must be added to 12 correction total marks increased by 150.
3. Total Sales = [2 × 5000 + (12 – 1)
1854 = (48 − 30) + 3 = 21. Choice (1) 2 Now they have 45x + 150 = 50x ⇒ x = 30
1000] = 126000  Choice (2)
25. Let the general form of the number be N.
N = K[LCM (11, 13, 39)] + 6 where K is Average monthly sales = 126000/12 = 12. Total cost of 15 items = (15) (56) = Rs 840
a natural number = 429K + 6; Required 10500 units (or) Alternatively, his sales in
Total cost of the purchased items = (3) (52)
number has the minimum possible value February is 6000 units,
= Rs 156
of K satisfying 429K + 6 > 10000 In December is 16000 units.
Total cost of the returned items = (6) (36)
5000 + 16000 = Rs 216
127 ∴ Average = = 10,500
K > 23 2 New total cost = 840 + 156 – 216 = Rs
429 units. Choice (4)
780
∴ Minimum value of K = 24 4. Let the temperatures in the city from 780; New average cost = = Rs 65
12
∴ Required number = (429) (24) + 6 = Sunday to Saturday be x°C, (x + 1)°C,
10302. Choice (2)  Choice (3)
(x + 2)°C,…, (x + 6)°C.
26. Final dividend = (18) (2) = 36 Average temperature in the city 13. New total score = 27 × 54 – 6 × 60 + 9 ×
from Sunday to Wednesday = 58 = 1620
Final but one divisor = Final dividend = 36
New strength = 27 – 6 + 9 = 30
 inal but one dividend = (Final but one
F x + x +1+ x + 2 + x + 3
= 30.5 C New total score
divisor) (Final but one quotient) + (Final 4 New average points = =
divisor) = (36) (1) + 18 = 54 ⇒ x = 29 New strength
x + 4 + x +5+ x + 6 1620
Final but two divisor = Final but one divi- ∴Required average = = 54 Choice (3)
3 30
dend = 54. = 34°C Choice (2)
 inal but two dividend = (Final but two
F 14. Let the strength of the class be x. Total
5. Sum of the ages of the students = (18 × 18) mark of the class = (x) (80) = 80x.
divisor) (Final but two quotient) + (Final = 324 years.
but one divisor) = (54) (4) + 36 = 252; First Total mark of the failed students = (8)
Sum of the ages of the students and the (35) = 280; Total mark of the remaining
divisor = Final but two dividend = 252. class teacher = (19 × 19) = 361 years. students = (x – 8) (90)
 irst dividend = (First divisor) (First quo-
F Class teacher’s age = 361 – 324 = 37
80x = 280 + (x – 8) 90 ⇒ x = 44
tient) + Final but two divisor = (252) (1) years. Choice (4)
 Choice (4)
+ 54 = 306 Choice (1)
6. The sum of all the 11 members is 330.
15. Average earnings of the first 3 months =
27. Given numbers are 5-1, 5, 5-2, 52, 5-3 The sum of the first 6 members is 198.
Rs 6000
that is, 1/5, 5, 1/25, 25, 1/125 ∴The sum of the last 5 numbers is 132
and their average is 132/5 or 26.4. We Earning in the 4th month = Rs 7500, earn-
Required L.C.M = ing in the 5th month = Rs 10500
should find out the minimum value, the
LCM of (1, 5, 1, 25, 1) 25 greatest of these numbers can have (the Average earnings of the first 5 months = Rs
= = 25 7200, Earning in the 6th month = Rs 9000
HCF of (5, 1, 25,1, 125) 1 6th number would have to be greater than
that value). The greatest of the last five Average earnings of the first 6 months =
 Choice (4) 45000/6 = Rs 7500 Choice (2)
numbers (that is, the 7th number) has to
be greater than 26.4. If it is 27 or 28, we 16. Let the cost of the cheapest pen be Rs c.
28. From the given conditions, 1, 5, 7, 11, 13, can not have 5 numbers whose average is
17, 19 and 23 are co-prime to 24. Cost of the costliest pen = Rs 1.5 c
26.4. It has to be 29. (The numbers could
 Choice (4) 1.5c
be 22, 26, 27, 28, 29) Cost of the third pen = = Rs
25
29. Let the required least number be N. Least ∴The least value of the 6th number is 30. 1+
number which is exactly divisible by 6 1.2c 100
The first 6 numbers could be 30, 31, 32, 34,
and 8 = LCM (6, 8) = 24. 35, 36. Choice (4) ⇒ c = 60 Choice (4)
∴ N must be 1 more than this least num- 7. Batsman’s new average 17. Let x be the average consumption on the
ber in order to yield 1 as the remainder
18 × 36 + 12 × 35.75 last 2 days of the week.
when divided by 6 or 8. Choice (4) = = 35.9 Choice (2)
18 + 12 250 × 3 + 400 × 2 + x × 2 = 350 ⇒ 1550 +
30. LCM of 3, 4, 5. 6. 7, 8 is 840 seconds = 14
minutes. They will toll together after 14 8. Total of the top 7 scores = (7) (60) = 420; 7
minutes that is, at 11.29 am. Choice (2) Total of the least 6 scores = (6) (50) = 300 2x = 2450 ⇒ x = 450kg Choice (4)

08_Section.indb 65 10/30/2009 11:52:58 AM


24. Let S.P. of 1 litre of milk and water = Rs 1
1 u 100 ratio = (4  4) : (5  6) : (9  8) = 8 : 11 : 17
? C.P = = Rs 5/6; Using alligation rule, Choice (3)
120
the ratio of milk and water is 5 : 1 31. Using alligation rule, the ratio of the quantities of
100 milk and water is 80 per
per cent of water contained in the mixture = 100
6 t
2
= 16 /3 per cent 85
66  n  Solution Manual Choice (1)
15 : 5 = 3:1
25 u 3  35 u 6  55 u 1 3 parts --- 60lt 15 per 5 per
25. Resultant concentration =
18. Let the number of students in the class 26. Using alligation rule, 1 part --- ? (x); 1 part cent
10 the ratio of solu- --- ? (x); t
x = 20lt Choice (4)
be N. 75  210  55 tions of alcohol is
340 x = 20lt Choice (4)
= = = 34 per cent
Let d be the decrease in average. 10 5 →1075 32. Let the quantity of milk to be added be x litres by
32. Let the
x 15  10 quantity of milk to be added be x
Sum of the ages of the students = 15 NChoice (2) 4 → ? (x) allegation, 1/2 Ÿ x = 10
20 10  0 x 15 − 10
years 26. Using alligation rule,4 × the =
litres by allegation, = 1/2 ⇒
75ratio of solutions of 20 (3)
Choice 10 − 0
15N − 23 − 17 alcohol is x= = 60 30 21
= 15 – d ⇒ 10 = d (N 5– o 2)75 5 x = 10 Choice (3)
N−2 4 o ? (x) ∴ 60litres of alcohol should be added 25
d is prime and a factor of 10; d = 2 or 5; If4 u 75 = 60  Choice (1) 33. Cost of the mixture = 3 × (100 / 120) = Rs
x=
d = 2, N = 7; If d = 5, N = 4 Choice (4) 5 2.50
5
27. Ratio  Milk part  Water part  Qty 4
? 60litres of alcohol should be added Using alligation rule, the ratio of two
19. New average = Ι  3 : 2  3/5 Choice (1)
2/5   33.1 Cost of the mixture
quantities is / 120) = Rs 2.50
= 3 u (100
18 × 36 + (5 × 36 + 63) − 33 − 41 − 37 27. ΙΙ  4 : 1  4/5 1/5   3 Using alligation rule,
= 39 Ratio Milk part Water part Qty 2.25 3.25
20 , 3:2 ΙΙ 
3/5 2 : 1  2/52/3 1 1/3   2 the ratio of two
quantities is
 Choice,, (2)4 : 1 Ratio of capacities 
4/5 1/5 5:6:8
3
,, 2:1 2/3 1/3 2 2.50
20. Total score of 30 students = (30) (60) Ratio = of capacities 5:6:8 1 1 1
Ratio of quantities = × 5 : × 6 : ×
1800 1 1 1 5 2 4 (3.25 – 2.50) : (2.00 – 2.25)
Ratio of quantities = u 5 : u 6 : u 8 = 1 : 3 : 2
Let the top and bottom scores be t and b 8 =51 : 3 : 22 4
0.75 : 0.25 = 3 : 1 Choice (4)
respectively. 3 4 2 365 4 2 0.75 : 0.25 = 3 : 1  Choice (4)
Resultant Milk = Resultant
+ u3 +Milk u 2== + ×3 + 34. × 2Applying
=
1800 – (t + b) = (28) (60 – 1) = 1652 ⇒ t 5 5 3 515 5 3
the rule of alligation,
+ b = 148 → (1) 34. Applying the rule
Quantity of wheat cos ting Rs.12 15  13 2 of alligation,
65
2 1 1 25
The difference in the highest and lowest Resultant water = u 1 + u 3 + u 2 = Quantity of wheat Quantity
cos tingof wheat13cos
Rs.15  12ting1 Rs.12 15 − 13 2
155 5 3 15 = =
scores is given by Resultant ratio = 65 : 25 = 13 : 5 Choice (1) 25 2 Quantity of wheat cos ting Rs.15 13 − 12 1
25
2 1 Ÿ1 Ÿ x 12.5 Choice (1)
t – b = 90 → (2); Solving (1) and28. = to form Resultant
(2),Int order water = ×1+ × 3 + x 1Quantity 2 of wheat cos ting Rs.12 = 15 − 13 = 2
1 kg of the third alloy,50.5 kg of 5 3
119 and b = 29. Choicealloy
(4) A must be mixed with 0.5 kg of alloy B. Quantity of wheat cos ting Rs.15 13 − 12 1

35. Ratio of quantities = 2 : 3 : 1; Concentrations of
25
21. SP of the mixture is Rs 24 per kg Weight and of copper×in20.5= kg of A = 3 (0.5) = 3 kg. milk in these three vessels are 3/5, 8/15 and 7/10
15 7 14 respectively. 25 2 25
profit per cent = 20 ⇒ CP of the mixture ⇒
Overall concentration = milk
of ⇒ xin= the =mixture
12.5  = Choice (1)
is 24 x 100 / 120 = Rs 20 4 : 25 2= 13 : 5 x 1 2
Weight of copperResultant ratio
in 0.5 kg of B = = 65(0.5) = kg. 2 u3/ 53 u8/15  1 u7/10
= 35/10 u 1/6 = 35/60
Using alligation rule, the ratio of quanti-  9 9 Choice (1) 35. 6 Ratio of quantities = 2 : 3 : 1; Concen-
ties is (x − 20) : (20 − 18) trations
Ratio of milk to water = 35of milk
: 25 = 7 in
: 5 these three vessels are
x – 20 : 2 :: 1 : 1 ⇒ x = 22 Choice (4) 28. In order to form 1 kg of the third alloy, 0.5 3/5, 8/15 and 7/10 Choice respectively.
(2)
kg of alloy A must be mixed with 0.5 kg Overall concentrationSol/847 of milk in the mix-
22. Let the initial quantity of mixture be x of alloy B. Weight of copper in 0.5 kg of
4x 2 × 3 / 5 + 3 × 8 /15 + 1 × 7 /10
litres. Initial quantity of alcohol = 3 3 ture = = 35/10 ×
7 A = (0.5) = kg. Weight of copper in 6
7 14
3 1/6 = 35/60
litres. Final quantity of alcohol = (x + 7) 4 2
7 0.5 kg of B = (0.5) = kg. Weight of Ratio of milk to water = 35 : 25 = 7 : 5
4 3 9 9  Choice (2)
litres x = (x + 7) ⇒ x = 21 3 2
7 7 copper in 1 kg of the third alloy = + 36. Using alligation rule, the ratio of milk and
 Choice (4) 14 9 water is 3 : 1
23. Resultant concentration of spirit = 55 4/5
= kg. Choice (1) 0
0.5 × 2 + 0.75 × 3 126
2 +3 29. Concentration of milk in the resultant so- 3/5
n 3
3.25  A − B  200 − 20 
= = 0.65; lution =   =  ×100 1/5
5 3/5
 A   200 
∴ Concentration of water = 0.35.
= 72.9 per cent Choice (2) 1/(3 + 1) = 1/4 or 25 per cent of mixture
∴ Ratio of spirit and water = 0.65 : 0.35 =
should be removed Choice (2)
13 : 7. Choice (1) 30. Weight of Ι bag = 2k + 3k + 4k = 9k;
Weight of ΙΙ bag = 4k + 5k + 9k = 18k. As 37. Using alligation rule, the ratio of quanti-
24. Let S.P. of 1 litre of milk and water = Rs 1 they mixed in 1:1 ratio; Weight of Ι bag = 3 4 3
ties mixed is − 0 : − = 3 : 1
1 × 100 weight of ΙΙ bag. 5 5 5
∴ C.P = = Rs 5/6; Using alli- ∴ Ratio of Ι bag = 4 : 6 : 8; ∴ Resultant Total Quantity = 100; ∴ Quantity of
120
mixture ratio = (4 + 4) : (5 + 6) : (9 + 8) =
gation rule, the ratio of milk and water is 8 : 11 : 17 Choice (3) 1
5:1 water to be added = × 100 = 25 litres
4
per cent of water contained in the mixture 31. Using alligation rule, the ratio of the
 Choice (3)
100 quantities of milk and water is
= = 162/3 per cent Choice (1) 38. Let the capacities of A, B and C be 3x, 4x
6 80 per 100 and 5x.
25. Resultant concentration = 3x 2 3 
85
Quantity of milk in =  x = x
25 × 3 + 35 × 6 + 55 × 1 75 + 210 + 55 2 3 2 
= =
10 10 3 4 
15 per 5 per Quantity of milk in B =  x  = x ;
340 4 3 
= 34 per cent Choice (2) 15 : 5 = 3:1
10 3 5  3
3 parts --- 60lt Quantity of milk in C =  x  = x
5 4  4

08_Section.indb 66 10/30/2009 11:52:59 AM


Solutions for Mathematical Skills  n  67

Total quantity of milk in the vessels = x + 8. The student who copied the co–efficient = (50 − x)
3x 11 of x in correctly must have copied the Cost of 1 dozen of bananas = Rs x.
x+ = x constant term correctly. Cost of 1 dozen of apples = Rs (50 − x)
4 4
For the equation, x2 + bx + c = 0, the con- Given, x (50 − x) + x (50 − x) = 1050 ⇒
Required ratio =
stant term = c = Product of the roots = 16 x = 15 or 35
11  3x 4x 5x 11  × 6 = 96, the student who copied the con- As the price of an apple is more than that
x : + + − x  = 33 : 16
4  2 3 4 4  stant term in correctly must have copied of a banana, the number of bananas are 35
 Choice (4) the coefficient of x correctly. The coeffi- dozen. Choice (3)
39. Resultant concentration of spirit = cient of x = b. 16. The given quadratic expression is 4x2 –
0.3 × 3 + 0.4 × 3 + 0.8 × 4 = 0.53 As sum of the roots = – (coefficient of x) 3x + 4.
10 = 11 + 9 = 20 a = 4 (> 0) The minimum value of the
∴ ratio of spirit and water = 53 : 47 Hence the correct equation is x2 – 20x +
 Choice (2) 96 = 0. Choice (2) 4ac − b 2
given quadratic expression = =
4a
40. When 1/10th of mixture is removed, 9. As per the given equation, the sum of the
1/10th of milk is removed. When the 4 × 4 × 4 − (−3)2
2m
same quantity of water is added, there is roots = ; product of the roots = 4(4)
k+m−
no change in milk. Again 1/10th of mix- 64 − 9 55
ture is removed. +m−k = =  Choice (2)
16 16
Let the initial quantity of milk be M0. Let k + m −
17. Sum of the roots = 3/a. Product of the
the quantity of milk after 1 and 2 replace- f rom the above, we observe that (k + m − roots = k/a. Given that 3/a = k/a; ∴ k = 3.
ments be M1 and M2 ℓ) is the denominator. Only choice (2) has  Choice (2)
∴ M1 = 0.9M0 and M2 = 0.9M1 = 0.81M0 (k + m − ℓ) as the denominator.Choice (2)
= 65.61 (given) q r
10. Let the roots be α and α2. Then, α + α2 = 18. ℓ + m = ; ℓm = . The equation
∴M0 = 81. As this is 90 per cent of p p
the vessel’s capacity, V. V = 90 litres. –p and α3 = −q.
 Choice (1) ∴(α + α2)3 = −p3 ⇒ α3 + (α3)2 + 3α3 (α + whose roots are 1 1
and 3 is x2 −
α2) = −p3 ⇒ −q + q2 + 3pq = −p3 ⇒ p3 + q2 3 m
= q(1 – 3p) Choice (2)
1 1  1
Quadratic Equations 11. α + β = −1 and αβ = 2;  3 + 3 x + 3 3 = 0
 m  m
(m +  ) − 3m ( + m ) x +
3
Practice Exercise α10 + β10 α10 + β10 (αβ)10 1
= = x2 − = 0;
Solutions for questions 1 to 25:
−10
α +β −10 10 10
(1 / α ) + (1 / β ) 1 3m3 3m3

1. x2 + x − 6 = 0 ⇒ (x + 3) (x − 2) = 0 ⇒ x = (2)10 = 1024  Choice (3)  q 3 − 3r qp  p3


x2 −  x+ 3 = 0
= −3 or 2 Choice (4)  r 3
 r
12. Let 56 + 56 + 56 − −∞ = p ⇒
2. x = 2 2 satisfies the equation 3x2 + 2 ⇒ r3 x2 – (q3 – 3rqp) x + p3 = 0
2 x – 32 = 0. Choice (3) 56 + p = p  Choice (4)
3. Let the number be x. x − 1/x = 35/6 p2 – p – 56 = 0 ⇒ p = 8 or p = – 7; p is 19. Let the price of each chair be x and the
⇒ 6x2 − 35x − 6 = 0 ⇒ x = 6 or −1/6 positive; number of chairs Ramesh bought for Rs
∴ The integer is 6. Choice (3) ∴p = 8 Choice (1) 2.400 be ‘y’. According to the problem x
. y = 2400; (x − 20) (y + 10) = 2400
−m n 13. Let the roots of the equation be aα and bα.
4. α + β = and α β = ;The equation Sum of the roots is α (a + b) = −r/p. a + b  2400 
  ⇒  − 20  y + 10 = 2400 ⇒ y2 + 10y
= −r/p × 1/α y
 
with (α + β) and α β as roots is x2 – (α + β Product of the roots is α2ab = r/p
+ αβ)x + (α + β) αβ = 0 − 1200 = 0 ⇒ y = 30 or −40; y > 0; ∴ y = 30
⇒ (b / a) + (a / b) = (a + b) / (ab)  Choice (2)
 −m n   −m  n 20. Let α and β be the roots of the equation
x2 –  +  x+  =0 = −r/p × 1 / α  (r / p) × 1 / α  = − (r / p)
       2 2
 1  1
⇒ ℓ2x2 – (n – m) ℓx – mn = 0 Choice (4)  Choice (2) Given, (α + β)2 =   + 
α β
5. α + β = 3/2 and αβ = −1; 14. Sum of the coefficients = p (q – r) + q (r –
p) + r (p – q) = 0. β2 + α 2 (α + β) 2 − 2αβ
α3 + β3 = (α + β)3 − 3αβ(α + β) = = ⇒
α3 + β3 = (3/2)3 − 3(−1) (3/2) = 27/8 + 9/2 ∴ 1 is the root of the equation. Since the (αβ) 2 (αβ) 2
= 63/8 Choice (3) roots are equal the other root is also 1.  1 
2αβ
= (α +β ) 2  −1 
(αβ )  
 (αβ )
2 2
6. If x − k is substituted in place of x in ax2 + r (p − q) 
∴ Product of the roots = =1⇒
bx + c = 0, the values of x satisfying this p (q − r)
equation obtained will be k more than the 2  1  2P
pq – pr ⇒ = S 2  2 −1  ⇒ S2 =
roots of the original equation. Choice (3) P P  1− P 2
1 1 2
7. Let number of days = x, Cost per day = y = pr – qr ⇒ pq + qr = 2pr ⇒ + = .  Choice (1)
r p q
xy = 1200 → (1); (x − 30) (y + 20) = 1200 21. Let one root be α and the other root be α2.
→ (2)  Choice (1)
Given; α × α2 = –64 ⇒ α = –4
By solving (1) & (2), we get x = 60, y = 20 15. Let the ‘x’ be the number of dozen’s of Also α + α2 = k; (–4) + (16) = k ⇒ k = 12
 Choice (2) apples. ∴Number of dozen’s of bananas  Choice (3)

08_Section.indb 67 10/30/2009 11:53:00 AM


68  n  Solution Manual

22. Since the given equation has equal roots. 20. Let the first term and the common ratio be
b2 – 4ac = 0 ⇒ 4 (k + 2)2 = 4 × 9k ⇒ k2 – 8. S33 = 33 [2a + 32d] = 1089 ⇒ 2 a + 32 a and r respectively.
2 2
5k + 4 = 0 a. ar. ar2 ar3 .......... a. r11 = 4096
k = 4 or k = 1 Choice (1) d = 33 a12 r66 = 46 ⇒ (a2 r11)6 = 46 ⇒ a2 r11 =
Second term + thirty second term = 33 4 → (1);
23. Let the total number of apes be ‘x’. num-
∴ Thirty second term = 33 – 2 = 31 ar5 + ar6 = 5 → (2)
ber of apes playing + remaining apes =
 Choice (3) Let ar5 = x; ar6 = y, then x + y = 5; xy = 4
total number of apes.
⇒ (x/8)2 + 12 = x ⇒ x = 48 (or) x = 16 9. The given series can be writ- ⇒ x = 4 or 1 Choice (4)
 Choice (3) 1 1 1  1  1 1  1 21. Let x1, x2,…………..x6 be the six harmonic
ten as  − +  − +
4 4 8  4  8 12  4
24. Now α2 + β2 = (α + β)2 – 2αβ = b2 − 2c
2
means. Then 3, x1, x2,….., x6, 6 are in
 1 1 23
2 a a  − + −−−−
= b −22ac  12 16  1 1 1 23
a H.P. ⇒ , , ,......, are in A.P. d =
3 x1 x 2 6
1 1 1 1 1 1 1 
Now α4 + β4 = (α2 + β2)2 – =  − + − + − + − − − − =
4  4 8 8 12 12 16  23 1
b−a −
(b − 2ac ) = 6 3 = 21 = 1
2 2
2
2c 1 1 1
2α2β2 = − 2 = n +1 6 +1 6.7 2
a4 a  =  Choice (3)
4  4  16
2a 2 c 2 + b 2 (b 2 − 4ac) 1
∴ The fourth harmonic mean is =
10. Let a − 3, a, a + 3 be three terms. a + 4d
a4
 Choice (2) (a −3)a + a(a + 3) + (a − 3) (a + 3) + 32
= 243 1 3
25. Let Eswar’s present age be ‘x’ years. =  Choice (3)
3a2 = 243 ⇒a2 = 81 ⇒ a = 9. Choice (1) 1 1 7
(x − 3) (x + 3) = 27 ⇒ x2 − 9 = 27 ⇒ x2 + 4 
= 36 ⇒ x = ±6 3 2
11. a = 147, d = −7; Tn = a + (n − 1)d = 0
Since number of years cannot be nega- ⇒ 147 − 7 (n − 1) = 0 ⇒ n = 22 22. Given a1 = h1 = 3 and a7 = h7 = 39; We
tive, x = 6 Choice (2)  Choice (2) have a7 = a1 + 6d; 39 = 3 + 6d; d = 6
12. In the given series, tn = (3n – 2) (5n) = ∴ a4 = a1 + 3d = 3 + 3 × 6 = 21; Now,
15n2 – 10n. h7 = 39
Progressions Sn = ∑ (15 n2 – 10 n)
1 1 −2
15(n (n + 1)(2n + 1)) 10n (n + 1) ⇒ + 6d1 = ⇒ d1 = ⇒
Practice Exercise = − ; 3 39 39
6 2
Solutions for questions 1 to 25: 1 1 −2 7 39
If n = 15 ⇒ S15 = 17400 Choice (3) = + 3× = ⇒ h4 =
1. a = 3, d = 7 − 3 = 4; tn = a + (n − 1)d = 63 h4 3 39 39 7
⇒ 3 + (n − 1)4 = 63 ⇒ n = 16Choice (3) 13. The terms x, y, z are in G.P. ⇒ y2 = xz
⇒ y2 = x.(2.56)x ⇒ y = 1.6x Choice (2) 39
∴ a4 . h4 = 21 × = 117 Choice (4)
2. Given 19 [2a + 18d] = 190 a 15 7
2 14. Given, = , 4a + 15r = 15 → (1), 23. Let the numbers be P and Q.
1− r 4
a + 9 d = 10 ( ∵ a = d) a + r = 3.2 → (2) 2PQ 24
∴ 10 d = 10; ⇒ d = 1.  Choice (1) Solving (1) and (2) we get a = 3 and ∴PQ = 36 → (1) = ;P+Q=
P+Q 5
3. t5 =a + 4d = 15 → (1), t9 = a + 8d = 23 → 1 15 → (2)
(2) r= = 0.2 Choice (1)
5 Solving (1) and (2); we have P, Q = 12, 3
Solving (1) & (2), we get a = 7, d = 2
15. a = 5; r = 2; sixth term = ar = 5(2)5 = 5 ×
5  Choice (3)
∴t14 = a + 13d = 7 + 13 x 2 = 33.
32 = 160. Choice (1) 24. a, b, c are in arithmetic progression
 Choice (2)
4. Let the maximum number of terms be N. 16. Let the numbers be a, a r and a r2 where r b = a+c
Nth term = 60 + (N – 1) (– 3) = 63 – 3N > 1. 2
63 – 3N ≥ 0 for the sum to be maximum a + a r + a r2 = 117; a (1 + r + r2) = 117
b, c, d are in geometric progression. ∴c2
∴ N ≤ 21. → (1)
= bd;
∴ N = 21 Choice (3) If r = 2, a is not a natural number. If r = 3,
a = 9 Choice (2) c, d, e are in harmonic progression
5. Series is 11, 16, ………. 96; Sum = 18(11 2ce
+ 96)/2 = 963 Choice (1) 17. 1 + 16 + 256 + 4096 + ……>1000. Thus, d=
in the above series, the sum to four terms c+e
6. The number of terms which are divisible 2+c 36c
is greater than 1000 Choice (2) If a = 2 and e = 18, b = ,d= ;
by 6 from 400 to 900 is 84. The number of 2 c + 18
terms which are divisible by 8 is 63. The 18. Given, (a) (a r) (a r2)…………a r10 = 2048
a 11 r1 + 2 +………10 = 2048  2 + c   36c 
number of terms which are divisible by 6 c2 =  ⇒ c2 – 36 = 0
and 8 is 21. a11 r55 = 211 ⇒ (a r5)11 = 211 ⇒ a r5 = 2.  2   c + 18 
The number of terms which are divisible ∴ 6th term = ar5 = 2. Choice (2) ⇒ c = ±6
by either 6 or 8 = 84 + 63 – 21 = 126 5
3(r − 1) For c = 6, b = 4 and d = 9
 Choice (4) 19. a = 3; Sum to 5 terms = = 33333 For c = –6, b = –2, d = –18 Choice (2)
(r − 1)
7. tn = Sn − Sn − 1 = 6n2 − 2n − (6(n − 1)2 − 5
r −1 25. Since x, y and z are in harmonic progression
2(n − 1)) = 12n − 8 ⇒ = 11111; r = 10, satisfies the 1 1
d = tn − tn − 1 = 12n − 8 − (12(n−1) − 8) =
r −1 , and 1 are in arithmetic progression.
above equation. Choice (1) x y z
12 Choice (4)

08_Section.indb 68 10/30/2009 11:53:01 AM


Solutions for Mathematical Skills  n  69

1 1 1 1 x−y y−z 9. 30 women - 36 days 16. Let the time taken by X to complete the
∴ − = − ⇒ = ⇒ ? women - 24 days job be x days. Time taken by Y to com-
y x z y xy yz
Number of women = 30 × 36/24 = 45 x
x−y x plete it = days.
⇒ =  Choice (1) 45w = 15w + 30w. 15 women and 15 men 2
y−z z (30w) are required to do work in 24days.
Women's earnings = (1/3) × 60000 = Rs And time taken by Z to complete the job
20000; Men’s earnings = (2/3) × 60000 x
Time and Work = Rs 40000 is
3
days
Difference between the shares of men and
Practice Exercise women is Rs 20000 Choice (2) 1 1 1 1
+ + = ⇒ x = 120
x  x   x  20
Solutions for questions 1 to 40: 10. Let the numbers of units completed by a    
women and a man in a day be w and m 2 3
M1D1H1 M 2 D 2 H 2 respectively.  Choice (4)
1. =
W1 W2 Job = (2w + 4m) 8 = (5w + 1m) 5 ⇒ 3m =
17. Let the work be completed in x days.
30 × 24 × 6 × 1800 1w. Job = [2(3m) + 4m]8 = 80m
⇒ D2 = = 36 days. ∴ A, B and C works for x, (x − 1) and
1500 × 18 × 8 80m 80m (x − 5) days respectively.
Required time = = =
 Choice (1) 6w + 2m 6(3m) + 2m x (x − 1) (x − 5)
2. Let the number of men in the group be m. + + = 1 ⇒ x = 11.
4 days Choice (3) 18 36 54
(m) (12) = (m + 5) (10) ⇒ m = 25
11. 4(6m + 9w) can do the work in 1 day. ∴They have to work for 11 − 5 = 6 days
 Choice (2)
3. Men Days Time Similarly, 8(4m + 3w) can do the work more.
in 1 day  Choice (4)
15 30 1
8(4m + 3w) = 4(6m + 9w) ⇒ 2m = 3w
12 ? 1 1/4 18. Let B’s one day work be 1 ; A’s one day
15 × 30 × 1 = D × 12 × 5/4 ⇒ D = ∴ Work done by two men = Work done x
by 3 women.
15 × 30 × 1 × 4
= 30 days Choice (4) (6m + 9w) × 4 days = (20m + 6w) × x work = 1 × 120 = 6 ⇒ 5 + 6 = 11 ⇒
12 × 5 x 100 5x 5x 120
8 × 42
days ⇒ 18w × 4 = 36w × x ⇒ x =
MDH MDH 36 x = 24 Choice (2)
4. We have 1 1 1 = 2 2 2 = 2 days Choice (1)
W1 W2 19. Let A takes x days to complete the work.
12. Let the numbers of units completed by a 3C × x
⇒ 120 × 25 × 9 = 150 × d 2 × 6 = d2 = 36 man, a woman and a machine in a day be B takes 2x days = 2x ⇒ C = 2x
1500 1800 (C + x)
 Choice (1) m, w and c respectively.
Job = (3m + 4w + 5c)50 = (6m + 24c)25 (1/x) + (1/2x) + (1/2x) = 1/30 ⇒ x = 60.
5. 1500 × 30 = 1500 × 26 + 300 (26 – x) ∴A, B, C take 60, 120 & 120 days re-
⇒ x = 6. w 7 4 spectively to complete the work.
⇒ = ; Time taken by a woman =
∴ The new men joined after 6 days. c 4 7  Choice (2)
 Choice (2) (700) 20. Let time taken by A and B to complete the
6. Let the number of men be x and the num- = 400 days Choice (2) work be x days.
ber of days in which they can complete
13. Part of work finished by all of them in one 1 1 1
the job be y. ∴ + = ⇒
day = (1/16)+ (1/20) + (1/30) = 35/240. x + 18 x + 32 x
Job = xy mandays = (x – 5) (y + 20) man-
days = (x + 5) (y – 10) mandays. 240 7
They can do the new work in = × 2x + 50 1
xy = (x – 5) (y + 20) ⇒ 4x – y = 20 → (1); 35 2 =
xy = (x + 5) (y – 10) ⇒ 2x – y = –10 → (2) (x + 18)(x + 32 ) x
= 24 days. Choice (3)
Subtracting (2) from (1), x = 15Choice (1)
⇒ x2 = 576 ⇒ x = 24 Choice (3)
14. Let the number of days required to com-
M1D1H1 M 2 D 2 H 2
7. = ⇒ plete the work be x days. 21. (1/P) + (1/Q) = 1/24 → (1);
W1 W2 Work done by P in (x − 3), days + work (12/P) + (42/Q) = 1 → (2) ⇒ P = 40 and
1200 × 30 × 1 600 × D 2 × 4 done by Q in (x − 2) days + work done by Q = 60 Choice (1)
=
1500 1800 × 3 R in x days = 1

1200 × 30 × 18 × 3 ∴
x −3 x −2 x
+ + = 1 ⇒ 11x = 132 ⇒ 22. Work done by Suresh = 1 – 15 = 3 of the
D2 = = 54 days 60 40 20 24 8
15 × 600 × 4 x = 12. So R worked for 12 days.
 Choice (1)
 Choice (2) total work. Share of Suresh = 3 × 2400 =
8. 100 men completed the remaining work 8
in 18 days. 15. Let the work be completed in x days. A, B Rs 900 Choice (3)
As the work remains the same. 80 men & C work for x, (x − 2) and (x − 1) days
23. Work done by P in one period of two days
respectively.
100 × 18 3+ 2 th
would have taken = 22.5 days to x x − 2 x −1 = 1/15 + 1/10 = = 5 = 1 of the
80 + + = 1 ⇒ x = 4. 30 30 6
do that work. 6 12 18 work
If additional men had not joined they ∴They can complete the job in 6 peri-
∴The work has been completed in 4
would have taken 22.5 − 20 = 2.5 days ods. Working on alternate days they take
days.
more. Choice (4)  Choice (2) – 6 × 2 = 12 days. Choice (1)

08_Section.indb 69 10/30/2009 11:53:02 AM


70  n  Solution Manual

24. Part of the job completed by P and Q in 4−3 39. A, B and C fills in 10, 15 and 20 minutes
one day if they work together would be 1/3 – 1/4 = = (1/12)th of tank. It respectively.
12
completed in two days if they work on Part of tank filled by A, B and C all to-
takes 12 hours for the tank to be emptied.
alternate days. 1 1 1 13
 Choice (3) gether in 1 minute = + + =
1 10 15 20 60
∴ of the job will be completed by P
15 32. Let Q be opened for x hours. 12 + x = 1 ∴ In 5 minutes, 5 × 13/60 = 65/60 is
15 30 filled
and Q in 2 days.
⇒ x = 6 hours
∴ P and Q can complete the job in (2) ∴ volume of water overflown = [(5/60)/1]
∴Q has worked only for 6 hours and
(15) = 30 days.  Choice (2) × 100
hence it was closed at 4:00 a.m. + 6 hours
= 81/3 per cent Choice (3)
25. Let the work be completed in x days. = 10:00 a.m. Choice (1)
P, Q and R work for x , (x – 1) and (x – 5) 40. Time taken by C to empty the tank
33. Both pipes together can fill the cistern
days respectively.  
in 1/10 + 1/15 = 5/30 = 1/6 = 6 minutes.
x/18 + (x – 1)/36 + (x – 5)/54 = 1 ⇒ x =  1 
Time taken for full cistern to be emptied =2   = 48 minutes → (1)
11 days Choice (2)
 1 + 1 
= 1/5 – 1/6 = 6 − 5 = 1/30; Answer is 30  40 60 
26. Let the time taken by leakage to empty 30
the tank be x minutes. In one minute the minutes. Choice (1) Part of the tank filled by A, B and C in a
water in the tank is  1  1 1
34. Let the times taken by P and Q to fill the minute = 2   – = ;
1 15 5
∴ 1 −1 = 1 ⇒ = = ⇒x tank be p hours and q hours respectively.  48  48 48
12 x 192 x 192 64 ∴ The tank will be filled in 48 minutes.
1q 1p
= 12 /5
4
Choice (4) Part of the tank filled =   +   =  Choice (3)
p 2 q 2
27. Let tap B empty the tank in x hours.
∴ tap A will fill the tank in (x − 3) hours. q 2 + p 2 (p − q) 2
= +1
2pq 2pq Time and Distance
1 1 1
− = ⇒ x = 6. 2
(x − 3) x 6 (p − q)
⇒ ≥ 0 ∴ Part ≥ 1 Choice (4) Practice Exercise
∴The second tap can fill the tank in 6 hours. 2pq
 Choice (1) 35. As A is taking least time to fill and D Solutions for questions 1 to 50:
28. Let the capacity of the tank be x litres. maximum time to empty it, their combi- 1. Let the speed of Vinay be x kmph.
In one minute the part of the tank filled by nation would be optimum. 300 300 30
Combination of A and D = (1/12) – (1/28) − = ⇒ x = 37.5Choice (2)
 1 1  8x x x + 2.5 60
the taps = x  +  = 7−3 4 1
15 25  75 = = = = 21 minutes. 2. Let the total distance travelled be 2d km.
84 84 21
Total water filled in 150 minutes = 150 × (d/30) + (d/50) = 8 ⇒ d = 150 ⇒ 2d = 300
∴ Combination of A & D is the best.  Choice (2)
8x  Choice (4)
→ (1) 3. Let the usual time taken be x minutes.
75 36. Part of tank filled by two inlet pipes in At 5/4th of usual speed Raju would have
 ater discharged by the outlet pipe in 150
W
minutes = (80 × 150) litres → (2) one minute = 1 + 1 = 4 + 1 = 1 . taken 4/5th of his usual time.
10 40 40 8 4x
∴ 8x × 2 − 80 × 150 = x ⇒ x = 800 litres. ⇒ x – = 20 ⇒ x = 100. Walking
So inlet pipes were open for 8 minutes. 5
 Choice (2)
Part of tank filled by both the inlet pipes at 3/4th of his usual speed, Raju would
29. Amount of water filled in the cistern in 5
2 1 have taken 4/3rd of his usual time. So he
 1 1  4+3 7 in 2 minutes = = ; So the outlet pipe
minutes =  5 +  = 5× = 8 4
 15 20  60 12 4
1 would be late by x − x = 331/3 minutes
 1 – 7/12 = 5/12 of the cistern is empty
∴ empties of tank in 8 minutes 3
4  Choice (3)
when pipe P is closed, which has to be
4 4. Average speed = 5 × 60 × 40/(60 × 2 + 40
filled up by pipe Q alone. It can empty the tank in × 8 = 32 minutes
1 × 3) = 50 km/hr Choice (3)
5
Time taken to fill the tank is × 20 =  Choice (4) 5. Total distance between Tirupati and
12
37. Clearly pipe A is open for 6 minutes, and Hyderabad = 50x km
81/3 minutes Choice (1)
B for 3 minutes.  2
30. Let the time taken to fill the tank be x Remaining distance = 50x 1−  = 50x/3
Let C is open for x minutes  3
minutes. (6/12) + (3/18) + (x/24) = 1 ⇒ x/24 = 1 −
Volume of water filled by the three taps is (1/2) − (1/6) ⇒ x/24 = 2/6 ⇒ x = 8 min- 3x x
km; Remaining time = x − = hours
x x −1 x − 3 utes. Choice (4) 4 4
+ + = 1 ⇒ 37x = 222;
12 15 18 38. Part of the tank filled by A and B in an 50x / 3 200
∴x = 6 Choice (2) Required speed = = kmph =
1 1 1 x/4 3
31. If there had been no leak, the amount of hour = 1 ; = + ⇒ 2t2 + 20t = 662/3 kmph Choice (4)
water filled in one hour would be (1/3)rd of t t t + 2 t +18
tank. But, because of the leak, in one hour, t + 20t + 36
2
6. (x/50) − (x/60) = 30/60 ⇒ x = 150 km
only (1/4)th of tank is full. The difference ⇒ t2 = 36. ∴The distance between his house and office
gives the effect of leakage which is ∴ t = 6 Choice (2) is 150 km. Choice (1)

08_Section.indb 70 10/30/2009 11:53:02 AM


Solutions for Mathematical Skills  n  71

7. Let the forward speed be s kmph. Return first. But it is given that both reached B the ratio of their distances covered is 1 : 2.
speed = 3s kmph. simultaneously and one person halted for The time taken for the first train to meet the
2(s)(3s) 30 minutes. second train after 5:00 p.m. is (1/3) × 4 =
Its average speed = = 24 ⇒ s = 140 140 11/3 hours = 1 hr.20 minutes.
s + 3s ∴ Pavan must have halted −
X 1.4X ∴ The two trains meet at 5:0 p.m. + 1 hr.
16. Choice (1) 20 min = 6.20 p.m. Choice (3)
30
8. Let x be the speed of the car be x. (300/x) =
60 22. Distance covered by the car in 3 minutes
– 300/(x + 10) = 1 ⇒ x = 50 km/hr = 200 + Distance covered by the man in 3
⇒ X = 80 kmph. Pavan’s speed = 1.4 × 80
 Choice (2)
= 112 kmph Choice (4)  3 
minutes = 200 +  × 6  ×1000 m = 200
Speed of A 15. Let the distance be D km ⇒ (D/9) –  60 
9.
Speed of B (D/12) = 40/60 ⇒ D = 24 km. Using D = + 300 = 500 m.

=
Timetakenby B to reach P from the meeting po int 24km in D = t + 1 , we find t and then s =  
Timetakenby Ato reach Q from the meeting po int 9 3  500 
 102/3 kmph Choice (2) Its speed =   kmph = 10
8 4   3 
= ⇒ x = 12 kmph Choice (4)  (1000)  
16. Let the speed of the train be s m/sec.   60  
x 9
Let its length be L m
10. Let his usual speed be x kmph Let his kmph Choice (2)
usual time taken be y hours. xy = (x – 10) L + 370
= 51 ⇒ L + 370 = 51s → (1);
(y + 2) s 23. (189/x) − [189/(x + 36)] = 4 ⇒ x = 27
⇒ 2x – 10 y = 20 → (1); xy = (x + 10) km/hr Choice (1)
L + 480
(y – 1) = 62 ⇒ L + 480 = 62s → (2),
s 24. Let the distance travelled at 50 kmph be x
⇒ 10y − x = 10 → (2)
Solving L = 140. Choice (3) km
Solving the equations (1) and (2), we get
Distance travelled at 68 kmph = (x + 44)
x = 30 and y = 4; Total distance = xy = 30 17. Speed of first train = 150/10 = 15 m/sec. km
× 4 = 120 km Choice (4) Let the length of the second train be l mts.
15 = (570 + l) / 58 ⇒ l = 300 m. Totaldis tan ce
11. (a) In the same direction, Relative speed Average speed =
= 20 km/hr; t = (300 + 150)/15 = 30 sec. Choice (2) Total time taken
Distance covered in 31/2 hrs = 20 × 18. When it stops in one hour, it travels for 2x + 44 50 × 68(2x + 44)
3.5 = 70 km 100/120th of an hour. ⇒ = 58 ⇒
x x + 44 68x + 50x + 2200
(b) In the opposite direction, Relative  100  +
speed = 180 km/hr; ∴ Stoppage time = 1−  th of an 50 68
 120 
Distance covered in 31/2 hrs = 630 km = 58
 Choice (4) 1 ⇒ x = 500 km.
hour = hours = 10 minutes. Choice (4)
12. When the owner started, the thief was 6 Total distance travelled = 2x + 44 = 1044
60 × 2 = 120 km away from the theft 19. Let ‘x’ be the length of the each train and y km. Choice (2)
point. Time taken by them to meet = be the speed of train B; 2x/(30 − y) = 72 →
25. Distance covered by the first train in one
(1) and 2x/(30 + y) = 24 → (2), Solving (1)
Dis tan ce initially between them hour = 60 km. Distance covered by the
and (2) we get x = 150 mts, y = 15 km/hr
Relative speed second train over the first train in one
 Choice (2)
hour = 75 − 60 = 15 km. The second train
120 20. Distance between the two persons when will gain 60 km in 4 hours (for both trains
= = 6 hours. So they met at 4:00
80 − 60 the train crossed the second person = to be together). They will meet after 4
 10   10  hours from 7:00 a.m. ∴They will meet at
p.m. Choice (3)  72 ×  −  6 ×  = 11 km
 60   60  11 a.m. Distance travelled in 4 hours = 4
13. Actual time between the two shots being × 75 = 300 km. Choice (4)
fired = 25 minutes If the car were sta- At 6:10 a.m., the distance between the
tionary, the person in the car also would two persons = 11 km 26. Let all the three trains meet t hours after
have heard the shots with an interval of 8:00 a.m. 120t = 80 (t + 2) ⇒ t = 4. In 4
Time taken by both the persons to meet
25 minutes. But since the car is moving, hours distance covered by B = 120 × 4
distance 11 11 11 = 480 km. This 480 km was covered by
he heard the second shot after 24 minutes = = = hours =
itself. That is, the distance travelled by relative speed 9 + 6 15 15 480
C in 3 hours. So speed of C = = 160
the car in 24 minutes = Distance travelled × 60 = 44 minutes. 3
by the sound in 1 minute (or 60 seconds). kmph. Choice (3)
Both persons will meet at 6:54 a.m.
Therefore, distance travelled by the car in  Choice (4)
24 minutes = 60 × 330 metres. Distance 27. Speed of Train T1 (S1) = 45 km/hr = 45 ×
travelled by the car in 60 minutes (that is, 21. They will meet at = First starting time + 5/18 = 25/2 m/sec
Time taken by first (2nd arrival time − 1st starting time) Let the Length of second train be T2 m.
speed of the car) = 60 × 330 × 60 = 99 Sum of times Speed of T2 (S2)= 27 km/hr = 27 × 5/18
24 1000 2 = 15/2 m/sec
= 49.5 kmph Choice (2) 6 (8 − 3) Time taken to cross each other in opposite
= 3 pm + = 3 pm + (30/9) hrs
14. Let Prasad ‘s speed be X kmph. Pavan’s (6 + 3)
300 + T2
Speed = 1.4X = 6.20 pm; Alternately the ratio of time direction ⇒ = 35 ⇒ T2
(25 / 2) + (15 / 2)
 ince Pavan was faster than Prasad, it
S taken by the two trains is 6 : 3 = 2 : 1.
would be expected that he reached B ∴ The ratio of their speeds is 1 : 2, hence = 400m Choice (4)

08_Section.indb 71 10/30/2009 11:53:03 AM


72  n  Solution Manual

28. Let the speed of the slower train be s m/ 36. Let speed of John be x kmph. Wind speed The ratio of the speeds of Prakash and
sec. = 5 kmph 1000
The speed of the faster train = 1.5 s m/ S1 = distance covered in t sec when going
Suresh = t = t + 60 = 1000 = 5
sec. against the wind. S2 = distance covered in 1000 t 1000 − 200 4
t sec with the wind.
300 + 200 t + 60
= 50 ⇒ s = 20. The required Since S2 = 2S1 ⇒ 2(x – 5)t = (x + 5)t
1.5s − s ⇒ x = 15 kmph. Choice (1) ⇒ t = 240

 Choice (2)
300 + 200 500 37. Let the speed of the person in still water
time = = 10 seconds. 43. A beats B by 100 m in a 1000 m race
1.5s + s 2.5(20) be x kmph. Then, speed upstream = (x – 3)
kmph; speed downstream = (x + 3) kmph If A = 1000m, B is at (1000 – 100) = 900 m
 Choice (3) A : B = 1000 : 900 = 10 : 9; If B covers
If he has travelled for “t” hrs, upstream
29. Required time = and “t” hours downstream we have Down 1000 m, C covers (1000 – 150) = 850 m
stream distance = 2 × Upstream distance. B : C = 1000 : 850 = 20 : 17; A : B : C =
Length of Train1 + Length of Train 2
= Therefore, t(x + 3) = 2 × t × (x – 3) ⇒ x = 9 20 × 10 : 20 × 9 : 9 × 17 = 200 : 180 : 153
Relative Speed If A covers 200 m, C covers 153 m. If A
 Choice (2)
(250 + 350) covers 1000 m, C covers = (1000 / 200)
= 24 seconds 38. Let its downstream journey time be t hours. × 153 = 765 m
(108 − 18) × 5 / 18 Its upstream journey time = (t + 1) hours. ∴A beats C by (1000 – 765) = 235 m
 Choice (3) Downstream speed = 7 + 1 = 8 kmph. Up-  Choice (4)
30. P would stop at the stations it come across stream speed = 7 – 1 = 6 kmph. Distance
between the two points = 8t = 6(t + 1) ⇒ 44. It is given that Akhil is 11/4 times as fast
after travelling 10 km, 20 km, 30 km, 40 as Anil.
km, and 50 km. Hence between the two t = 3 = 8t = 24 km Choice (1)
It means Akhil will cover 5 metres for ev-
stations the number of stations is 5. So P 39. Let the length of the race be x m. Let the ery 4 metres Anil covers. So, Akhil will
train stops for a total of 50 minutes. Time
speed of B be y m/s. Speed of A = 4y have an advantage of 1 metre for 5 metres
taken to travel 60 km = 60 = 6 hours. So 3 he travels. To cover up 45 metres, Akhil
10 m/s. A and B finish the race simultane- must travel (45 × 5) metres = 225 me-
A reaches B in 6 hours 50 minutes after ously. So time taken by A to cover x m tres. ∴At a distance of 225 metres from
5:00 a.m. that is at 11:50 a.m. is the same as that taken by B to cover the starting point Akhil will meet Anil.
 Choice (2) (x − 120) m.  Choice (4)
31. Downstream speed = x + y = 12/3 = 4 km/ x x − 120 3x x −120 45. A B C
hr. = ⇒ =
4y / 3 y 4y y 1200 1100 1000
Upstream speed = x − y = 12/6 = 2 km/ 5500 5000
hr. Speed of man in still water = 3 km/hr. ⇒ 3x = 4 (x – 120) ⇒ x = 480
Speed of C = 500/25 = 20 sec
Speed of stream = 1 km/hr  Choice (2)
∴Time taken by C to complete the race =
Difference in distances = 3 × 5 − 2 × 5 = 40. 5500/20
Time taken= by
275B sec
to complete the race = 275  25
5 km Choice (3) 40. R Q Time
=T taken
imesec
250 takenbybyBBto to
complete the the
complete race = 275
race
Choice  25
=(2)
40. A R Q B = 250 sec Choice (2)
A B 275 − 25 = 250 sec Choice (2)
32. The upstream speed of the boat = 6 = 3 46. Let the length of the race be Lm. When Alok finished
2 46. the
46. Letrace,
Let the
thelength
Bala of the
must
length racerun
ofhave
the be (L
raceLm. When
Lm. Alok
– 200)m
be finished
and Dinesh
When
C D the race,
must have Bala must
run (L have run
– 400)m. (L – 200)m
InBala
order andtoDinesh
for Bala finish
kmph. The downstream speed of the boat C
P
D
Alok
must
finished
have
the race, must have
P the race, he run
must(Lhave
– 400)m. In order200
run another form.
Bala to finish
run (L – 200)m and Dinesh must have run
= 15 = 5 kmph. ?the race,
For thehe musttime
same haveDinesh
run another
would200 m. run less
have
PQ =PQSpeed (m per
= Speed (mmin) in stillinwater
per min) = 30 = 30
still water (L – 200
400)m. In Bala
order for Bala to finish the
? For
than them.same
? timebeats
Dinesh would
Dinesh byhave
more runthan
less
3 PQ==Speed
QR QR
Speed(m(mper
= Speed
permin)
min)ofinriver
(mmin)
per min)
still water
of==river
= 30
x (say)
= x (say)
QR = Speed (m per of river x (say) race,
200 he must
thanm.200 m. ?haveBalarun another
beats Dinesh200by m.more(4)
Choice than
30 30 PR = Speed (m per min) w.r.t bank = 32.5
The required time = + = 16 hours. PR =PR = Speed
Speed (m min)
(m per per min) w.r.t bank
w.r.t bank = 32.5= 32.5 200For
∴ m. the same time Dinesh wouldChoice have (4)
5 3 R Q 47. A and B meet for the first time after 450/10 =
Q
run
47. 45sec
A andlessBthanmeet200form.the Balatime
∴ first beatsafter
Dinesh
450/10
Choice (3) =
 Choice (3) R
32.5 by more than 200 m.
45sec Choice (4) (3)
Choice
=32.5
13 u 2.5 30 48. Let the length of the track be L m. Let the speeds
33. s/(x + y) = (2/3) [s/(x − y)] ⇒ x = 5y → (1) 47.
= 13 u 2.5 =30
2.5 u 12 48. ofA
Letand BPavan
meet
the length
Aman, forSekhar
ofand
the the be
track first
beLam.time after
Let the
m/sec. b speeds
m/sec
Given xy = 45 → (2); Solving (1) and (2) = 2.5 u 12 450/10
of Aman,
and = Pavan
c m/sec45sec and Sekhar
respectively. be aChoice
Required time to(3)
m/sec. bmeet
m/sec
=
we get x = 15 km/hr; y = 3 km/hr P and c m/sec respectively. Required time to meet =
P LCM
48. Let the length of the track be L m.LLet the
 Choice (1) §LCM
L L L· L
Ÿ PQ = 2.5 u 5 = 12.5 m/min is the speed of
⇒ PQ = 2.5
u 5× =5 = 12.5m/min
m/min is is thespeed
speedof §aL, bL cof
¨speeds L¸·=Aman, Pavan
L and SekharL be a =
34. Let the speed of the boat upstream be x Ÿ PQ
River. = 2.5 12.5 the ©¨ , ¹¸ =H.C.F(a, b, c) H.C.F(2b, b, 6b) =
of River.
River.
m/sec. b m/sec and c m/sec respectively.
© a b c ¹ H.C.F(a, b, c) H.C.F(2b, b, 6b)
kmph and its speed downstream be y kmph. ?Option (2) Choice (2) L
∴Option
?Option (2) (2) Choice
Choice (2) (2) L = 2 minutes  L L L  (2)
24 36 40 24 Required time to meet = LCM  ,Choice
b = 2 minutes 
+ = 6 → (1); + = 7 → (2)
41. By the time A completed the race, B completes
41. By the time A completed the race, b  a Choice
b c  (2)
x y x y 41. 900
By mts
the time
and CAcompletes
completed810 themts.
race, B completes
Therefore inBa
900 completes
500 mts
mts and 900 mts and
C completes
race, C covers 810 Cmts.
(500 completes
u Therefore
810)/900 810
in=a 49. A and B meet for the first time after 200/8 =
A andABL
49. 25sec; meet =for the
L first
first time L 200/8
after
Solving (1) and (2), x = 8 and y = 12. 500mts
450 mtswhen
mts. race,B completes
Therefore C covers
in a 500the(500
mts u 810)/900
race,
race. C cov-C=
B beats = and C meet for the time =after =200/5=
450 mts when B completes the race. B beats H.C.F(a,
=25sec;
40sec A andb,c)C meetH.C.F(2b,
for the b,6b)
first time after
b 200/5
1
Speed of the stream = (12 − 8) = 2
by 50ers
mts.
by 50 mts.
(500 × 810)/900 = 450 mts when(2)BC
Choice
Choice (2) ?A= 40sec
and B meet for the first time after 25sec
2 42. Let the completes
time taken thebyrace.
Prakash B beats
to run C the
by race
50 mts.be t 2?A
?A minutes
and
will B meet
meet for 40
C after thefirst
25 =time Choice
after
15sec. it(2)
25sec
After met B.
kmph Choice (3) 42. seconds.
Let the
 time Time taken
takenbyby
Prakash
Rakeshtotorun the
therace
runChoice race be= t
(2) ?A will meet C after 40  25 = 15sec. After it met (1) B.
(tseconds. Time taken
+ 90) seconds. Timeby Rakesh
taken to run to
by Suresh therunrace
the= 49. A and B meet for the first time afterChoice 200/8
Choice (1)
35. With the help of the tide, time taken to 42. Let
(t + 90)
race +the90time
= (t seconds. taken
– 30)Time by
60) Prakash
= (t +taken by Suresh
seconds. to to
runrunthe
the = 25sec;
50. Let the speedA andof BCbemeetx m/s.for the first
Speed of A time
= 3x m/s
raceratio
= (t of
+be90 50. When
Let the
after Aspeed
met=Bof
200/5 B be
for
40sec thex m/s.
first Speed
time atofthatA = time
3x m/s A
cover 10 km = 1 hr The race t –seconds.
the 30) = (t of
speeds +Time
60) seconds.
takenand
Prakash bySuresh
Rakesh= When Aone met B formorethe than
first B.
time at that time A
The
1000ratio of the speeds of Prakash and Suresh = covered round
With the reversal of the direction of the to run the race = (t + 90) seconds. Time ∴A and B meet for the first time after 25sec
1000 5covered
(3x) – 5one round
(x) = 100 more
Ÿ x =than B. Time taken by
10 m/s;
tide, time taken = 5 hr. Time he would t by
t taken 60Suresh
1000 5 race = (t + 90 –
to run the ∴A will
5 (3x) – 5meet
(x) =C100
afterŸ40x600−= 25
10 =m/s;
15sec.
Time After
taken by
1000t30) =t (t60 1000 5Ÿ t = 240
have saved = 4 hrs. Choice (1) t + 60)
1000  200 4 Ÿ t = 240
seconds. Bitto complete
met B. the race = 600= 60Choice sec (1)
1000
t  60 t 1000  200 4 B to complete the race = 10 = 60 sec
10 Choice (2)
t  60 Choice (2) Choice (2)
Choice (2) Exercise 14
43. A beats B by 100 m in a 1000 m race Exercise 14
(Geometry)
43. IfAAbeats B by B
= 1000m, 100 m (1000
is at in a 1000
– 100)m race
= 900 m (Geometry)
AIf :AB==1000m,
1000 : B900
is at= (1000
10 : 9;– If
100) = 900 1000
B covers m m, Solutions for questions 1 to 40:
08_Section.indb 72 10/30/2009 11:53:04 AM
CAcovers
: B = (1000
1000 :–900
150)==10 850: m
9; If B covers 1000 m, Solutions for questions 1 to 40:
PQ PR 3.3 7.8 perpendicular sides =
So, DE is not parallel to QR. 12
cm
1 2
? DE z QR Choice (2) C
2 B 2
D
1 § 12 ·
Area = u¨ ¸
7. ‘8  ‘1 = 100° 2 ¨© 2 ¸¹
‘1 + ‘8 = 180° (sum of exterior angles on the = 36 cm .
2

same side of theSolutions


transversal)for Mathematical
ChoiceSkills 
(2) n  73
----------------------
2‘8 = 280° 15. ‘ BOC = 90° + (1/2) ‘A Ÿ ‘A/2 =120°  A
50. Let the speed of B be x m/s. Speed of ∠1 = ∠3 =----------------------
∠5 = ∠7 = 40° A right angled 90°
∆le has the maximum area
= 30° Ÿ ‘A = 60°
A = 3x m/s ∠2 = ∠4 =Ÿ ∠6 ‘8==∠8 140° Ÿ ‘1 =Choice
= 140° 40° (3) when it is isosceles.
ChoiceEach
(3) of the perpen-
When A met B for the first time at that ‘1 = ‘3 = ‘5 = ‘7 = 40°
8. We know that‘2 = the
‘4sum= ‘6 of=any‘8 two sides of
= 140° Choicedicular
(3) 12the bisectors of ‘P and ‘Q meet at the point
16. Let
time A covered one round more than B. cm
sides =T on SR.
a triangle is greater than the third side. 2 P
5 (3x) – 5 (x) = 100 ⇒ x = 10 m/s; Time 8. We know that the
AP + BP > 3.5 cm and CP + DP > 4.8 cm sum of any two sides of a Q
x y
600 ⇒ AP + BPtriangle
+ CP +is DP greater
> 8.3than
cmthe third side. 1  12 
2
taken by B to complete the race = AP + BP > 3.5 cm and CP + DP > 4.8 cm Area = × x y
10  Choice
Ÿ AP + BP + CP + DP (4)
> 8.3 cm Choice (4) 2  2 
= 60 sec Choice (2) 2 2
9. PR = 9.12 PR
+ 16 = 20
2 cm2
= 12  16 = 20 cm = 36 cm2. Choice (2)
Area of triangle PQR by = 90° + (1/2) ∠A ⇒ ∠A/2 y
x =120°
P 15. ∠BOC Let ‘PS= 2x and ‘Q T
= 2y; PS = QR =Ra cm.
Geometry X
taking QR and PR
− 90°
12 1 § ___ ___
·
separately as = 30°u ⇒ ∠A =‘60°
bases ¨' PQ ||(3)
QPT = ‘PTS Choice SR ¸ ; ‘PQT = ‘QTR
2 © ¹
Practice Exercise Q R
16 1 16. Let the bisectors ___ ∠P___
§ of · ∠Q meet at
and
Solutions for questions 1 to 40: 2
u PR uthe
QR u PQ = ¨' PQ || SR ¸
QXpoint T on SR.
 rea of triangle PQR by taking QR and
A © ¹
1. AB II GF and CB II DE ⇒ ∠BCD = Ÿ (1/2) u 12 u 16 = (1/2) u 20 u QX Ÿ QX P = From the figure, PS
Q = QR = ST = TR = a
9.6cm Choice (1) x Ÿ PQ = SR =y 2a
∠CDE = 60° (alternate angles)
of 1two 1 are proportional
x
10. Theas
PR separately areas
bases similar
× QR triangles
× PQ = y
Perimeter of PQRS = 13.5 cm (given)
⇒ CD II GF ⇒ ∠CDE + ∠DEG = 180° to the squares2 of their corresponding
2 sides. Ÿ 6a = 13.5 cm Ÿ a = 2.25 cm. Choice (2)
⇒ ∠DEG = 180° − 60° = 120°Choice (2) Area (PAB) : Area (PQR) = PA2 : PQ2 = 1 : 2
× PR × QX 17. By ASA congruency, the areas of both the
2. ∠ADE = ∠ABC ( DE || BC) → (1) and PA× 161= (1/2)PQ 2
⇒ (1/2) × 12 Ÿ × 20 × QX ⇒ Choice x triangles
y are equal.
∠AED = ∠ABC QX = 9.6cmPQ  2 QA Choice
2  1(1) S area
T of each triangle R = 1/2 (Area of quadrilateral)
2
From (1), ∠ADE = ∠AED ⇒ ∠DAE + = (1/2)(36) = 18cm Choice (1)
10. The areas (4)
of two similar triangles are Let ∠P = 2x and ∠Q = 2y; PS = QR =
∠ADE + ∠AED = 180° PA PC
proportionalADto the1 squares
AD of1 their AE
cor- 1 AEa cm.
1 18. AC parallel to SR. In triangle PSR, o
∠DAE + ∠DAE + 30° + ∠DAE + 30° 11. Ÿ and Ÿ ;
responding DB
sides. 5 AB 6 EC 5 AC 6 PS PR
= 180° (1)  ___ ___

Area (PAB)In : Area (PQR)
triangles = PA
ADE and : PQ
2
ABC,= two sides∠QPT
2
are = ∠PTS ∵ PQ || SR  ; ∠PQT =
⇒ ∠DAE = 40°  Choice (2) By the basic proportionality
 theorem, 'APC is
1:2 proportional.
3. AC = 5 cm, In a right triangle, median drawn DE 1 similar to 'SPR. PA : PS = 2 : 5
PA 1 As perPQ 2
basic proportionality theorem. ;  ___ Area___
 triangle APC : Area of triangle SPR = 22 :
of
to the hypotenuse, is half the hypotenuse. = ⇒ =  Choice (4) BC ∠QTR 6 ∵ PQ 5||2SR
= 4: 25
PQ 2 QA 2 −1 
∴BD = 1/2 × Hypotenuse = 5/2 = 2.5 cm 1 Area of 'APC : Area of SACR = 4 : 21

Choice (4) ? DE = u 18 = 3 cm. ChoiceFrom
(4) the figure, PS = QR = ST = TR = a Choice (4)
11. AD 1 AD 6 1 and AE 1
4. Let AG = AF = 1 = ⇒ = = ⇒ ⇒ PQ = SR = 2a
DB 5 AB 6 EC 5 Perimeter of PQRS = 13.5 cm (given) Sol/857
In triangle PGA, ∠G = 90°, ∠A = 60°,
∠P = 30° AE 1 ; In triangles ADE and ABC, ⇒ 6a = 13.5 cm ⇒ a = 2.25 cm.
=  Choice (2)
Since, AG : GP : PA = 1 : 3 : 2 AC 6
two sides are proportional. 17. By ASA congruency, the areas of both the
GP = 3 ; GD = 2 and PD = 2 − 3 ⇒
As per basic proportionality theorem. triangles are equal.
GP 3 area of each triangle = 1/2 (Area of quad-
=  Choice (1) DE 1
PD 2 − 3 = ; rilateral) = (1/2)(36) = 18cm2 Choice (1)
BC 6
5. ∠AEC + 50° = 180°; ∠AEC = 130° 18. AC parallel to SR. In triangle PSR,
∠ACD + ∠EAC = 240° ⇒ ∠ACD + 1
∴ DE = × 18 = 3 cm. Choice (4) PA PC
130° = 240° 6 = → (1)
PS PR
⇒ ∠ACD = 110°; ∠ACD + ∠ACB =
12. AB = AC = 2 → (1) AB = 2DC and AC  y the basic proportionality theorem,
B
180° ⇒ 110° + ∠ACB = 180° ⇒ ∠ACB DC BD
= 70° ∆APC is similar to ∆SPR. PA : PS = 2 : 5
∠A + ∠B + ∠C = 180° ⇒ 50° + ∠B + = 2BD Area of triangle APC : Area of triangle
AB AC SPR = 22 : 52 = 4 : 25
70° = 180° ⇒ ∠B = 60° Choice (3) AD bisects ∠BAC, ∴ = → (2);
BD DC Area of ∆APC : Area of SACR = 4 : 21
6. PD = 3 cm, PQ = 3.3 cm; PE = 7 cm, PR  Choice (4)
Dividing (1) by (2), BD2 = DC2 ⇒ BD
= 7.8 cm. GP = 3 ; GD = 2 and PD = 2  =3DC AB AC
By the basic proportionality theorem, 12. = = 2 o (1) AB = 2DC and AC 19. = 2BD
In triangles ABC and DEC, ∠C is common
GP 3 From (2), AB DC= ACBD = BC Choice (1)
Ÿ Choice (1) ∠CDE = ∠CAB (corresponding angles)
PD 2 to 3BC, PD = PE ⇒
if DE is parallel 13. AB = 12 cm,AD BCbisects
= 5 cm‘BAC,
AB AC
and AC?= 13 =cm; o (2); Dividing ∠CED = ∠CBA ( -do- )
PQ PR BD DC ∴ Triangles ABC and DEC are similar
3 5. 7‘AEC + 50° = 180°; ‘AEC = 130° ΔABC is a right angled triangle.
2 2
(1) by (2), BD = DC Ÿ BD = DC ⇒ DE = 1/2 AB.
≠ ∴Length ofFrom the median
(2), AB = BD
AC =(1/2)
= BC (13) Choice (1)
3.3 7.8 ‘ACD + ‘EAC = 240° Ÿ ‘ACD + 130° = 240°
= 6.5 cm; Area of Δ ABC = 64 cm2;
Ÿ ‘ACD = 110°; ‘ACD + ‘ACB = 180° Ÿ 110° 13. AB = 12 cm, BC = 5 cm1 and AC = 13 cm; ǻABC
So, DE is not parallel to QR. length of BG Area of Δ EDC = (1/4) × 64 = 16 cm2
+ ‘ACB = 180° Ÿ ‘ACB = 70° is a = (2/3)
right (6.5)
angled = 4 /3 cm
triangle.
1 cm; of ADEB = 64 − 16 = 48 cm
Area 2
∴ DE ≠‘A QR + ‘B + ‘C = 180° Ÿ 50° + ‘B + 70° = 180°
Choice (2)  ?Length of the median Choice (2) (13) = 6.5
BD =(1/2)
Ÿ2 ‘B = 60° Choice (3) 1
length of BG = (2/3) (6.5) = 4 /3 cm Choice  (2) Choice (3)
14. Let the triangle be ABC, where ∠ABC =
7. ∠8 −6.∠1PD = =100°
3 cm, PQ = 3.3 cm; PE = 7 cm, PR = 7.8 cm.90° 14. Let the triangle be ABC, where ‘ABC 20.
= 90°2∠EBC + 3∠BCF = 460° → (1)
∠1 + ∠8By= the 180°basic
(sumproportionality
of exterior angles
theorem, if DE is A right angled 'le has AE the|| DF
A maximum area when∴ it∠EBC is
on the same side of thePD PE
transversal) 3 7 + ∠BCF = 180° → (2)
parallel to BC, Ÿ z isosceles. Each of the
---------------------- PQ PR 3.3 7.8 perpendicular sides =
Solving (1) and (2), we have ∠EBC = 80°
2∠8 = 280°So, DE is not parallel to QR. 12 ∠ADC = ∠EBC = 80° (exterior angle of
---------------------- cm a cyclic quadrilateral
1 2 Q is equal to the inte-
⇒ ∠8 =?140° DE z⇒ ∠1 QR= 40° Choice (2) B C rior angle opposite to it) Choice (1)
2 2
D P
1 § 12 · C
Area = u¨ ¸
7. ‘8  ‘1 = 100° 2 ¨© 2 ¸¹
‘1 + ‘8 = 180° (sum of exterior angles on the = 36 cm .
2

same side of the transversal) Choice (2)


----------------------
08_Section.indb 73
2‘8 = 280° 15. ‘BOC = 90° + (1/2) ‘A Ÿ ‘A/2 =120°  A B 10/30/2009 11:53:05 AM
© 2 ¹
C [Perpendicular from centre to chord bisects the
AC chord]
= 2 o (1) AB = 2DC and AC = 2BD ‘z = 180°  75° = 105° In 'OBP,
BD 105q In triangle AOB ‘O = 30q, ‘A = 30q, ‘B = 90q,
AB AC ?‘x = = 35° Choice (2)
ects ‘BAC, ? = o (2); Dividing 3 ? AB : OB : OA = 1 : 3 : 2
BD DC
2 2 24. Let the number of sides be N. OA 2 18
2), BD = DC Ÿ BD = DC Ÿ OA = = 6 cm. Choice (4)
2), AB = AC = BC
74  nChoice   Solution(1)
Manual N(N  3) N(N  3) OB 3 3
Number of diagonals = Ÿ =
19. In triangles ABC and DEC, ‘C is common 2 n (n2 3) 10 u 7 2 2 2
2 cm, BC = 5 cm and AC = 13 cm; ǻABC 27. The number of diagonals = 32.= BC = (24)  (16) = (40) (8) = 320
ht angled triangle. 21. ‘CDE = ‘CAB (corresponding angles)
Construct a line BX parallel to l and m 9N Ÿ N(N
28.  21)of= the
Radius 0 Ÿlarger
N = 21 circle is 5 cm Choice
2and the 2
‘ CED = ‘ CBA ( -do- ) (2) 35 16
th of the median BD =(1/2) (13) and = 6.5 cm; through B. ∠ABX = 60°
passing distance between centres is 4 cm. Choice (1) D C
of BG = (2/3) (6.5) = 4 /3 cm? Triangles
Choice (2)ABC and DEC are similar
1
Ÿ (alternate
DE = 1/2 AB. angles); ∠CBX = 50° (alternate 28. Radius (2n of the4) larger
90q 12 circle is 5 cm and the
triangle be ABC, where ‘ABC =of90° 25. Interior angle =
distance between u 90°
centres is 4 cm. = 135°
angles),
Area ∠ABC
ǻ ABC = 64=cm ∠ABX
2
; + ∠CBX = 60° nA 8 24
A right angledArea A B
ofhas
+'le50° ǻ=EDCthe= (1/4) u 64 = 16 cm2Choice (1)
110° ? Exterior angle = 360°/8 = 45°
A Choice (3) 16
maximum area Area when
of ADEBit is= 64  16 = 48 cm2 Choice (3) O1
isosceles. 22.
Each In triangle
of the ADP, AP = AD + DP (by Py-26. Let the interior angle be T;OExterior angle
2 2 2
= 180°  T Ÿ BC =
1
20. 2sides
perpendicular EBC=+ 3‘
‘thagoras BCF = 460° o (1)
theorem). 2T + 3(180°  T) = 420° Ÿ T = O 120° O Choice (2)
12 AE || DF B ⇒ BC = 8 5 cm Choice (2)
cm ? ‘ EBC + ‘ BCF = 180° o (2)
2 Q B
C Solving (1) and (2), we have ‘EBC = 80° 33. ∠XTA = ∠ABT (by alternate segment
2
D = 80°P(exterior angle of a cyclic ∴Centre of the smaller circle is in the Sol/858
1 § 12‘·ADC = ‘EBC C ?Centre of the smaller circle is in the interior of theorem)
Area = u ¨¨ ¸
¸
interior of the longer circle.
the longer circle. Join OA as O A
Join
1 OA as
2 © 2quadrilateral
¹
is equal to the interior angle
∴ ∠ABT = 35°; ∠AOT = 2∠ABT = 2 ×
2 opposite to it) Choice (1) OAs1
A O O1 = 4 cm, O1A = 5 cm and OA = 3cm
m. ‘AOO 11 = 90°; Similarly ‘5BOO 1
= 90° 35° = 70° Choice (2)
oice (2) 21. Construct a line BX parallel to l and m and
As O O = 4 cm, O 1
A = cm and OA =
3cm? AB passes through O 34. ∠AOB = 180° ⇒ ∠AOD = 90°
= 90° + (1/2) ‘A Ÿ ‘A/2passing =120°  throughA B. ‘ABXB = 60° (alternate ? AB = 6 cm. 34. ‘
Choice (2)AOB = 180° Ÿ ‘AOD = 90°
∠AOO1 = 90°; Similarly ∠BOO1 = 90° Circumference of circle S; π;
= 10 Lengthofof arc AD
angles); ‘CBX = 50q (alternate angles), ‘ABC =
Circumference of circle = 10 Length
Ÿ ‘A = 60° ‘ABX
AP2 += ‘24 CBX
2
+ 10= 60
2 q + 50q
⇒ AP2 == 110 Choice (1)
576q + 100 29.∴InAB passes through O Inradius = a 3 cm arc AD = 1/4 C = 5π/2 cm Choice (3)
(3)
an equilateral triangle, = 1/4 C = 5 S/2 cm Choice (3)
22. In A triangle cm →
P = 26ADP, AP(1);
2
= In
ADtriangle
2
+ DP2 (byPCQ, PC =
Pythagoras ∴ AB = 6 cm. 2 3 (2)
Choice 35.
2 AB2 = (BC) (BD) 2 ⇒ 8 = (4) (BD) ⇒
2
bisectors of ‘P and ‘Q meet at
theorem). the
DC2 – DP point
= 15 2– 10 = 5 cm Ÿ a = 6cm. 35. AB = (BC) (BD) Ÿ 8 = (4) (BD) Ÿ BD = 16 cm
R. AP2 = 24 102 Ÿ AP 29. InSide an ofequilateral
'PQR = 6 triangle, Inradiusof ='PQR = BD = 16 cm ⇒ DC = BD − BC = 12 cm
P As +triangles ADP= 576 and+ 100
QCP are similar, cm Ÿ Perimeter Ÿ DC = BD  BC = 12 cm Choice (4)
AP Q = 26 cm o (1); 18
a cm. Choice (4)  Choice (4)
x y In PC DP = 3 cm
triangle
= DP = 15 – 10 = 5 cm
PCQ, 36. r1 + r2 > d
x yPC = DC 30.2‘AOC 3 = 2‘ABC = 2 u 60° = 120°
QC –DA ‘ADC = ‘ABC = 60° (angles in the same
34. ‘AOB = 180° Ÿ ‘AOD = 90°
PC DP a = 6cm.are equal).
⇒segment Choice (3) 36. r1 + r2 > dof circle = 10 S; Length of arc AD
Circumference
As triangles 5 ADP
10 and QCP are similar, = 1/4 C = 5S/2 cm Choice (3)
= QC
⇒ QC = 12 cm → (2) DA Side of ∆PQR = 6 cm ⇒ Perimeter
___ of ___
5 QC 10 24
2
centre of the circle and OB A 35. = (BC) (BD) Ÿ 82 = (4) (BD) Ÿ BD = 16 cm
x y
S= 2x and ‘Q = 2y; PS = QR =Ra cm. Ÿ QC = 12 cm
31.∆PQR
Let O =be18
the
cm. If rAB
PQ 1 + r2 > d then two intersecting circles are
T o (2) Ÿ DCIf= rBD+ r2BC
> d= then
12 cmtwo intersecting
Choicecircles
(4)
In triangle
QC 24 PCQ, PQ2 = PC2 + QC2 = 52 +  Choice (4) formed.areHence two common tangents can be
1
§ ___
· ___
12 ;=PQ = 13 → 36. r1 + r2 > d formed. Hence two common tangents
PQ(3)= PC + QC = 5 + 12 ; drawn. Choice (1)
2 2 2 2 2 2
= ‘PTS ¨' PQ || SR ¸ ; ‘InPQT triangle PCQ,
‘QTR
© ¹ PQAQ = 13 = AP + PQ = 26 + 13 = 39 cm
o (3)
30. ∠AOC = 2∠ABC = 2 × 60° = 120° can be drawn. Choice (1)
___
· AQ[From
= AP + (1)PQand
= 26 + 13 = 39 cm Choice (4)
(3)] ∠ADC = ∠ABC = 60° 37. OB = OA = radius; Given AB 2 radius Ÿ OAB
O (angles Pin the
|| SR ¸ [From (1) and (3)] Choice (4) X 37. OB = OA = radius; Given AB 2 radi-
¹ same segment are
30° equal). 60°30° is an
If r1isosceles righttwo
+ r2 > d then triangle Ÿ‘OBE
intersecting = 45°
circles are
23.
23. Clearly,
Clearly, 3∠x
3‘x = ‘z = ∠z 30° formed.us ⇒ OABtwois an
Hence isosceles
common right triangle
tangents can be
he figure, PS = QR = ST = TR = a  Choice (3) Choice (3)
= SR = 2a A B 60°60° B drawn.⇒∠OBE = 45° Choice (1)
90° A 38. Radiusmakes a right angle with the tangent.
ter of PQRS = 13.5 cm (given) O 31. Let O be the centre of the circle and Choice (3)
y As XY = PQ Ÿ XA 37. OB = OA = radius; Given AB 2 radius Ÿ OAB
13.5 cm Ÿ a = 2.25 cm. ChoiceX(2) x Y Q= PA. Ÿ
Y ‘AXP = ‘APX = 30q
___ ___
is38. Radius makes
an isosceles a rightŸ‘
right triangle angle
B OBEwith the tan-
= 45°
75° OB PQ= § PQ · = 9 cm.
and⊥BP Choice (3)
A congruency, the areas of both the ¨ ¸ gent.
s are equal. © 2 ¹
38. Radius makes a right angle with the tangent.
each triangle = 1/2 (Area of quadrilateral)
C [Perpendicular from centre to chord bisects the
36) = 18cm
2
Choice (1) chord] A B O
‘z = 180°  75° = 105° In 'OBP,
∠z PCq − 75° = 105°
PA =105
180°
?‘x =
allel to SR. In triangle PSR, =
o35° Choice (2)
In triangle AOB ‘O = 30q, ‘A = 30q, ‘B = 90q, C
O A O
PS =PR3 105° X : OB : OA = 1 : 3 : 2 P
? AB Hence ABO is a right angled triangle.
∴∠x = 35° Choice (2) 30° 60° 30°
3 of sides be N. OA 2 30° 18
Choice (right angled at B).
24. Let the number C
basic proportionality theorem, APC is OA60°
Ÿ60° = B = 6 cm. (4)
24. Let '
the number of sides
N(NbeN.
3) N(N  3) OB 3 3 ABHence
u BOABO is a right angled triangle.
= 12
to 'SPR. PA : PS = 2 : 5 Number of diagonals = Ÿ = A (right angled at B). is a right angled triangle.
triangle APC : Area of triangle 2
SPR = 2of: diagonals = 2 N(N − 3)2⇒ 32. BC2 = (24)2  (16)2 = Y(40) (8) = 320 12 = AB Hence
u 3 ŸABOAB = 4
Number Q AB u BO = 12
25 9N Ÿ N(N  21) = 0 Ÿ N = 21 2 Choice (right angled at B).
12
2 = AB u2 3 Ÿ AB 2=4 2 2
'APC : Area of SACR = 4 (2)
: 21 D
16 AB + BO AB ×=BO AO= 12 AO = 3  4 = 5.
Ÿ
N(N − 3)
Choice (4) = 9N ⇒ N(N − 21) = 0 ⇒ N As XY = PQ ⇒ XA = PA. ⇒ C ∠AXP = AB2 + BO2 = AO2 Ÿ AO = 32  42 = 5.
12 = AB × 3 ⇒ AB = 4
(2n  4) 90q 12 Choice (2)
25. Interior 2angle = u 90° = 135° 24 Choice (2)
Sol/857
= 21
n 8
Choice (2) A  PQ B 39. 39.
(RQ) (RP)
AB2 ==+(RS)
(RQ) (RP)
(RS) =(RT)
BO2(RT)
AO2 ⇒ AO = 3 + 42 = 2
?Exterior angle = 360°/8 = 45° Choice (3) ∠APX = 30° and BP16 =   = 9 cm.
 2  Choice (2) (8)(8) 5.++QP) (8) (8
(8) (8  4)  4)
(2nT; −Exterior
26. Let the interior angle be 4) 90°angle
12= 180°  T Ÿ BC = 8 5 cm (RQ
(RQ QP) ==(6)
(6)(RT).
(RT). RT RT
25. Interior angle = = × 90°  6 6 Choice (2)
2T + 3(180°  T) = 420° Ÿ T n = 120° Choice
8 (2) 33.[Perpendicular
‘XTA = ‘ABT from centre segment
(by alternate to chordtheorem)
bi-
Ÿ RT = 16 cm Choice (1)
= 135° ? ‘the
sects = 35°; ‘AOT = 2‘ABT = 2 u 35° =Ÿ70°
ABTchord] RT =(RQ)
39. 16 cm
(RP) = (RS) (RT) Choice (1)
∴Exterior angle = 360°/8 = 45° Choice40.
(2)PX .PY = PQ.PR; PX = PQ Ÿ PY = PR
In ∆OBP, 40. PXŸ.PY = PQ.PR;
(RQ PX
+ = PQ
QP) Ÿ PY =
PX + XY = PQ + QR Ÿ XY = QR = 5 cm (RT).
(8) = (6) PR
 Choice (3) Sol/858
In triangle AOB ∠O = 30°, ∠A = 30°, ∠B Ÿ PX + XY = PQ + QR Ÿ XY = QR = 5(2)cm
Choice
26. Let the interior angle be θ; Exterior angle = 90°, (8)(8 + 4) Choice (2)
= 180° − θ = RT
6
2θ + 3(180° − θ) = 420° ⇒ θ = 120° ∴ AB : OB : OA = 1 : 3 : 2
⇒ RT = 16 cm Choice (1)
 Choice (2)
OA 2 18 40. PX .PY = PQ.PR; PX = PQ ⇒ PY = PR
27. The number of diagonals = = ⇒ OA = = 6 cm.
OB 3 3 ⇒ PX + XY = PQ + QR ⇒ XY = QR
n (n − 3) 10 × 7
= = 35  Choice (4) = 5 cm
2 2  Choice (2)
 Choice (1) 32. BC2 = (24)2 − (16)2 = (40) (8) = 320

08_Section.indb 74 10/30/2009 11:53:06 AM


6. Considering the shaded region enclosed between per cent Choice (3)
four adjacent circles, and joining the centres of
the four circles, a square of side 10 cm is formed. 14. Area of four walls = 2h (l + b) = 2 u 3.5 (8 + 5) =
2
The shaded region = (Area of square) – (Area of 91 m .
2 2
circle)= (10) – S (5) = 100 – 25S Area of two doors = 2 u 2 u 1.5 = 6 m2.
There are four such shaded regions Area of the two windows = 2 u its height u its
= 4 (100 – 25S) cm .
2
Choice (1) breadth = 2 u 1 u 0.75 = 1.5 m2.
2
Area of the other window = 0.75 u 1.2 = 0.9 m .
7. ? Required area = Area of quarter circle with Solutions forpaper
Area of the Mathematical Skills 
required to cover n  of75
four walls the
radius 7 cm room = 91 – (6 + 1.5 + 0.9) = 91 – 84 = 82.6 m2.
1 2 1 22 77 2 Choice (4)
(A) = Lateral
8. = u 7=u perimeter
u area
S(7) surface 7 = = 38.5 m x
of base 15.
Mensuration 4 4 7 2
height = (5 + 6 + 7) × 10 = Choice
180 cm(4)
. 2 15.
Volume = Area of base × height = h 25 m
Practice Exercise 8. Lateral surface area = perimeter of base x height 25 m 20 m
= (5 + 6 ×+ 7)su(s10 = 180 2 7m
− a) (s −cm . − c)
b) (s = 10 ×
Solutions for questions 1 to 40:
A B C
1. When the centres of the three circles are 9 × 4 × 3 × 2 = 60 6 cm3.Choice (3)
joined with one another an equilateral tri- 9. Let the length and the breadth of the AB = 252 − 7 2 = 24 m and Sol/858
BC =
angle with side, (7 + 7) = 14 cm is formed. field be l and b respectively. Distance he
Area of the shaded portion = (Area of the would have walked using the direct route 252 − 202 = 15 m
equilateral triangle with side 14 cm) – Width of the road = AC = AB + BC = 39 m
= l2 + b 2 . Distance he would have
(3 × area of one sector with 60° angle) =  Choice (3)
[( 3 /4) × 14 × 14] – 3 × (60° / 360°) × π walked if he walked along its edges = l + b
× 7 × 7 = 7.87 cm2. Choice (3) 2 l 15 16. Sum of parallel sides = 36 − 16 = 20 cm
l + b – l = l2 + b 2 ⇒ = Area = (1/2) × 20 × 5 = 50 cm2Choice (1)
5 b 8
2. Area of ABG = 1/2 × 100 × 30 = 1500;
Area of CBE = 1/2 × 50 × 30 = 750  Choice (2) 17. Length of the cuboid so formed = 40 – 12
Area of DCEF = 25 × 30 = 750; = 28 cm
10. Area of 1/4 of the circle of radius 2 m =
Area of AFD = 1/2 × 25 × 30 = 375 Breadth = 32 − 12 = 20 cm; Height of the
(1/4) × π × (2)2 = π
Adding all the above areas, we get the to- cuboid = 6 cm
Area of 3/4 of the circle of radius 6 m =
tal area = 3375 cm2. Choice (3) ∴ The required volume = 28 × 20 × 6 =
(3/4) × π × (6)2 = 27π
3360 cm3  Choice (1)
3. Number of square tiles the total area of the field in which the cow
grazes. 18. Number of bricks =
Area of the floor ∴π + 27π = 28π = 28 × (22/7) = 88 m2
= Volume of the wall
Area of each square tile  Choice (2)
Volume of each brick
Area of the floor 11. Area of the park = 240 × 180 = 43200
= ; Number of square m2. Area of the path running parallel to 30 ×100 × 2 × 100 × 20
(side of each tile) 2 n= = 2000;
length= 40 × 240 m2 12 × 50 × 10
tiles is minimum when the side of each Area of the path running parallel to
square tile is maximum. Maximum side Cost = 2000 × 5 = Rs 10,000Choice (2)
breadth = 30 × 180 m2
= HCF(4.8, 3.6) = 1.2 m 19. Area of the chess board = (64) (5.25) =
Total area of the path = 40 × 240 + 30 × 180
∴ Minimum number of square tiles = 336 cm2
– 1200 = 13800 m2
(4.8)(3.6) Area of the park excluding the paths = Let the length of the rectangle be l cm. ⇒
= 12 Choice (1) (l)(16) = 336 ⇒ l = 21 cm Choice (4)
(1.2) 2 43200 – 13800 = 29400 m2 Choice (4)
4. Area of floor = 122 = 144 m2. Length of 12. Outer side of the photograph = 6 + 2 × 20. Volume of the wall = 24 × 15 × 1 = 360 ft3.
width = 6 + 2 × 1 = 8 cm Volume of mortar = (25 / 100) × 360 =
144
carpet required = = 240 m Area of the frame = Outer Area – Inner 90 ft3.
(60 / 100) Volume of Bricks = Volume of wall – Vol-
area = (8 × 8) – (6 × 6) = 28 cm2
Cost = 240 × 305 = Rs 73200.Choice (2) Required ratio = 36 : 28 = 9 : 7 Choice (3) ume of mortar = 360 – 90 = 270 ft3.
5. Radius of the cricket field with the pave- Volume of one brick = l × b × h
13. Let the side of the original cube be a cm. = (12 × 8 × 5) ft3. inches
ment = 80 + 8 = 88 Total surface area of the original cube = 6a2
22 270 × 12 × 12 × 12
Area of path (pavement) = π(R2 – r2) = cm2. ∴ Number of bricks =
7 12 × 8 × 5
Total surface area of each part = 2a2 + 4(a)
22 = 972 Choice (4)
(R + r) (R – r) = × 168 × 8 = 4224 a
7  
2 21. Let the speed of the water flowing through
Total cost = 4224 × 8.45 = Rs 35692.8
= 4a2 the aperture be x m/s. Volume of water
 Choice (3) The sum of the total surface area of the 40 3
6. Considering the shaded region enclosed two parts = 8a2 filled in the tank = 360 × 144 × m
100
between four adjacent circles, and joining 8a 2 − 6a 2 → (1);
∴ Required percentage =
the centres of the four circles, a square of 6a 2 Volume of water flown through the tank =
side 10 cm is formed. (100) = 33 /3 per cent
1
Choice (3) 40 30
The shaded region = (Area of square) – × × x × 8 × 60 × 60 → (2)
(Area of circle)= (10)2 – π (5)2 = 100 – 14. Area of four walls = 2h (l + b) = 2 × 3.5 (8 100 100
25π + 5) = 91 m2. 40 30 40
There are four such shaded regions = 4 Area of two doors = 2 × 2 × 1.5 = 6 m2. ⇒ × × x × 8 × 60 × 60 = ×
100 100 100
(100 – 25π) cm2. Choice (1) Area of the two windows = 2 × its height
144 × 360 ⇒ x = 6 Choice (1)
× its breadth = 2 × 1 × 0.75 = 1.5 m2.
7. ∴ Required area = Area of quarter circle
Area of the other window = 0.75 × 1.2 = 22. Let the height be 5x cm and radius be 2x
with radius 7 cm
0.9 m2. cm
1 1 22 77 Area of the paper required to cover four Volume of the cylinder = 21560 cm3
(A) = π(7) 2 = × ×7×7 =
4 4 7 2 walls of the room = 91 – (6 + 1.5 + 0.9) = ⇒ πr2 h = 21560 ⇒ (22/7) × 2x × 2x × 5x
= 38.5 m2 Choice (4) 91 – 84 = 82.6 m2. Choice (4) = 21560 ⇒ 2x = 14cm

08_Section.indb 75 10/30/2009 11:53:07 AM


76  n  Solution Manual

Curved surface area of the cylinder = 2π rh 29. Given weight of the box = 0.243 kg = 243 36. Volume = πr2h = (22/7) × 7 × 7 × 12 = 154
= 2 × (22/7) × 14 × 35 = 88 × 35 = 3080 cm2 gm. Let the thickness of the box be x cm × 12 = 1848 cm3
 Choice (3) ∴ The outer dimensions of the box are (12 If it is wrongly taken as; height = 14 cm.
23. The largest cone is formed if the length + 2x) (9 + 2x), (6 + 2x); volume × density Diameter = 12 cm; Radius = 12/2 = 6 cm
and height of the cuboid are the diameter = weight of box Volume of the cylinder = πr2h = (22/7) ×
and height of the cone. ⇒ [(12 + 2x) (9 + 2x) (6 + 2x) − 12 × 9 6 × 6 × 14 = 1584 cm3;
Radius of cone = 15 m; Height of cone × 6] × 0.25 = 243. Solving, x = 1.5 cm Difference = 1848 – 1584 = 264
= 42 m  Choice (4)  Choice (1)
1 30. Diameter of the sphere = 400 3 ; Side of
∴ The volume of cone = πr2h 37. Area of rectangular field = 50 × 30 = 1500
3
this cube = diagonal / 3 = 400 3 / 3 = m2
1 22 400 m Let the depth of the trench be d m.
= × ×15 × 15 × 42 = 9900 m3
3 7 Again a smaller sphere is fitted in it. ⇒ Volume of trench = 15 × 10 × d = 150
 Choice (1) m3
 iameter of the sphere = 400 m. Side of
D
n Area of the trench = 15 × 10 = 150 m2
24. Area of a regular polygon of n sides = the smaller cube = d / 3 = 400 / 3 m
4 ∴Remaining area of the field = 1500 – 50
Volume of bigger cube : volume of smaller
180 = 1350 m2
a cot
2
where a is side of polygon. cube = (400 × 400 × 400) : [(400/ 3) ×
n Volume of trench
(400/ 3) × (400/ 3) ] = 1 : (1 / 3 3) = ∴ Raise in level =
10 180 Area of field
Area of decagon = 102 cot = 250
4 10 3 3 : 1 Choice (1)
cot 18°. Choice (1) 2 150 d
31. Let r1 and r2 be the radius of cylinder and ⇒2 = ⇒ d = 20 m
hemisphere. 9 1350
25. Volume of cylinder = Volume of sphere
Volume of 72 cylindrical cans = 72 πr12h  Choice (4)
4 3
⇒ π(R2 – r2)h = πr ⇒ (R + r) (R – r)h = 72 πr12 (2r1) = 144 πr13 cm3. Volume of
3 1 38. It radius of the sphere is doubled, then
4 2 3 new radius R = 2 × r (r = original radius)
= (4)3 hemispherical bowl = πr ⇒ 144πr13 =
3 3 2 Original volume = (4/3) πr3. Increased vol-
4 3 16 2 3 ume = (4/3)πR3 = (4/3) × π (2r)3 = (4/3) ×
⇒ (R + r) (2) (8) = (4) ⇒ R + r = πr ⇒ r2 : r1 = 6 : 1 Choice (2) π8r3 = (32 / 3) πr3
3 3 3 2
36 × 18 × 11 Increased volume = (32 / 3) πr3 – (4/3)πr3
cm Choice (4) 32. Number of lead balls =
(4 / 3) × π (1.5) 2 = (28 / 3) πr3. per cent increase = (28/3)πr3
26. Let radius of cylinder be ‘r’ and height be / (4/3)πr3) × 100 = 700 per cent
‘h’ = 504 Choice (2)  Choice (4)
⇒ length of the box = 12r; Breadth of the 33. l = (R − r) 2 + h 2 ⇒ l =
box = 6r 3 2
Height of the box = 2h; Volume of 36 cyl- 39. that is, a = 49 3 ⇒ a2 = 49 × 4 ⇒ a
(18 − 13) 2 + 122 4
inders = 36 × πr2h
⇒ l = 13 cm = 14 cm
Volume of box = 12r × 6r × 2h = 144r2h;
Also given length of the side of the trian-
TSA of bucket = π(R2 + r2 + l (R + r)) =
144r 2 h − 36πr 2 h gle = diameter of circle that is, d = 14 cm
Empty space = × 100 =
144r 2 h 22
(324 + 169 + 403) = 2816 cm2 3
7 Thus the required area = × (πr 2 ) =
21.5 per cent Choice (2)  Choice (1) 2
27. Volume of water flowing through the 3 22
34. Volume of cylinder : Volume of hemi- × × 49 = 231
2 2 7
4 sphere : Volume of cone = πr2h : (2/3)πr3 :
circular pipe per second = π   (0.4) ∴ The required area is 231 cm2.
2 (1/3)πr2 h
= π h2 × h : (2/3) πh3 : (1/3) π h2 × h  Choice (3)
(100) = 160π cubic cm
160π(3600)
Raise in the water level = = = 1 : (2/3) : (1/3) = 3 : 2 : 1  Choice (2) 40. Volume of 1 lead shot = (4/3) π (2)3 = 32π /
π(402 )
35. Volume of ice cream = 63 × 55 × 14 cm 3 3 cu mm
360 cm = 3.6 m Choice (3) Volume of 20,000 lead shots = 20,000 ×
Volume of ice cream distributed to each
28. The solid so formed is a cone. length of person = volume of cone + volume of 32π/3 mm3
the arc of cut portion = (216°/360°) × 2 × The water level in the beaker is raised
1 2 2 1 when 20,000 lead shots are dropped in it.
π × 5 = 6π hemisphere = πr h + πr3 = πr2 (h
Radius of the cone is 2πR = 6π ⇒ R = 3 3 3 Volume increased = 20,000 × 32 × π/3 ⇒
3 cm 1 2 6 πr2 h = 20,000 × (32/3) × π × (1/10) × (1/10)
From the figure AO = 52 − 32 = 4 cm; + 2r) = πr × 6r = πr3 × (1/10) cm3
3 3
h = 4 cm radius of cylinder = 16/2 = 8 cm ⇒ r2h =
∴ The number of persons =
Volume of the cone = (1/3) × π × 32 × 4 = 20 × 32/3 ⇒ 8 × 8 × h = 20 × 32/3 ⇒ h
12π m3. 63 × 55 × 14 = 3.33 cm
 Choice (4) 22 = 1440 Choice (3)  Choice (4)
2× × (1.75)3
7

08_Section.indb 76 10/30/2009 11:53:08 AM


Solutions for Mathematical Skills  n  77

35
21. of [(2 + 7 − 5) (48 − 39 − 5)] ÷ 49] × 7
(1.4 + 1.2 ) − (1.4 + 1.2 )
3 3
Numerical Aptitude 9.
2.63 − 0.23
=
12
3 (1.42 )+ 1.22 3 (1.42 )+ 1.22 35 35 16
= of {[(4) (4) ÷ 49] + 7} = × ×
Practice Exercise (a + b) – (a – b) = 6a b + 2b = 2b (3a +
3 3 2 3 2 12 12 49
Solutions for questions 1 to 50: b2)
20
7=  Choice (4)
(a + b ) − (a − b )
3 3
1. 1.212 + 3.612 – 3.63 × 3.61 3
∴ = 2b = 2(1.2) = 2.4
= 1.212 + 3.612 – 3(1.12) (3.61) 2
3a + b 2
8 × 76 ÷ 38 × ?− 50 30
= 1.212 + 3.612 – 2(1.21) (3.61) – (1.21)  Choice (3) 22. = ⇒
(3.61) ?× 7 × 12 ÷ 3 − 32 108
3 4
= (3.61 – 1.21)2 – (1.12 × 1.92) = 2.42 – 10. (240) – (210) = 24 Choice (3)
5 7 8 × 2 × ?− 50 30 16?− 50 30
(2.09)2 = ⇒ =
?× 7 × 4 − 32 108 28?− 32 108
= (2.4 – 2.09) (2.4 + 2.09) = (0.31) (4.49) 11. 20032 – 52 + 9 = 20032 – (25 – 9) = 20032
 Choice (3) – 42 = (2003 – 4) (2003 + 4) = (1999) ⇒ ? = 5 Choice (4)
2 1 2 20 21 2 2 (2007) Choice (1)
2. 6 × 4 +1 = × + 1 = 29 23. 4[5 + {7 + 58 + 105} ÷ 5 – 3] = 4 [5 +
3 5 3 3 5 3 3 12. The cube root of the given number is 1.03. {170} ÷ 5 – 3] = 4[39 – 3] = 4[36] = 144
 Choice (2)  Choice (2)  Choice (1)
3. Let a = 3.2, b = 2.1 and c = 4.3;
(251) + (241)
2 2
the given expression equals = 251 241 24. 100 + 68 × 2 − 17 = 240 − ? ⇒ ? = 240 −
13. + = = [(251)2 219 = 21 Choice (3)
a + b − c + 3 (a )(b )(c )
3 3 3 241 251 (241)(251)
+ 2412 – 2 (251) (241) + (2) (251) (241)] 25. 14 − 7 + 12 + 23 − 9 + 15 = ? + 24 ⇒ 48 =
a 2 + b 2 + c 2 − ab + bc + ca
1 ? + 24
a 3 + b3 + (−c ) − 3 (a )(b )(−c )
3
×
= =a+ (241)(251) ∴ ? = (24)2 = 576 Choice (1)
a 2 + b 2 + c 2 − ab − b (−c )− (−c )a
(251 − 241)
2
b – c = 1 Choice (2) 26.
= + 2 that is > 2
4. 5.2 – 4.8 = (5.2 + 4.8) (5.2 – 4.8) = (10)
2 2 (241)(251) 26(20 + 2) 20 + 1.414 21.414
(0.4) = 4 Choice (2) ≅ = = 36.5
 Choice (4) 26(2 − 2) 2 − 1.414 0.586
1 1 1 1 1 4 14. Tens digit of the given number is the tens
+ of +  Choice (3)
5. 3 3 3 = 3 9 = 9 = 2 Choice (1) digit of 24 + 31 + 31 – 68 – 60 = 86 – 128
1 1 1 1 1 2 = 58.
− of − 27. 2 + [2 + {2 + (2 + (2 ÷ 23)}] = 2 + 6 + 1/4
3 3 3 3 9 9 ∴ Tens digit is 5. Choice (3)
= 33/4 Choice (2)
6. 1.442 + 2.562 + 1.44 × 2.56 = 1.442 + 2.562 15. As per BODMAS rule, 60 + 3 × 6 + 23 + 1
+ (2) (1.44) (2.56) – (1.44) (2.56) = (1.44 × 57 – 39 = 60 + 18 + 23 + 57 – 39 = 158 28. By applying BODMAS rule.
+ 2.56)2 – (1.22 × 1.62) = 42 – ((1.2) (1.6))2 – 39 = 119 Choice (2) 51  19 49  19 51
= (4 – (1.2) (1.6)) (4 + (1.2) (1.6)) = (4 – × ÷ ÷ = ×
16. Units digits of the given number would be 4  17 6  3 4
1.92) (4 + 1.92) = (2.08) (5.92)
the units digit of 32 × 63 × 94 – 9 = 7
 Choice (2) 51  19 6  3 27
 Choice (2) × × × =  Choice (2)
7. Each number is the form a + b where a 4  17 49  19 98
17. 10 per cent of 20 per cent of 30 = 10 per
+ b is constant. cent of 30 per cent of 20 = 20 per cent of 300 400 3 4
29. + +1 = + + 1=
( a + b)
2
 = a + b + 2 ab . As a + b 10 per cent of 30 = 20 per cent of 30 per 700 300 7 3
cent of 10;
is constant, the greatest number’s square ∴ required value = 1 Choice (1) 9 + 28 + 21 58 60
= = = 3 Choice (4)
will have the greatest 2 ab that is, the 21 21 20
18. The units digit of given number would be
greatest ab; ab for 2 + 11, 3 + 10 , the units digit of 3 × 7 × 9 × 3 × 3 = 1701 30. (2.03 + 7.06 − 1.052) × (8.35 − 2.15)
∴ Units digit is 1 Choice (4) (9−1) × (8−2) = 8 × 6 = 48. Answer is
5 + 8 and 7 + 6 are 22, 30, 40 and
greater than 48. Choice (4)
42 respectively. The greatest of these is
19. 31. 25 per cent of 18 ÷ [56 + 44 + 5] = 25 per
42. Choice (4)
2 (1.12 +1.22 +1.32 +1.1×1.2 +1.1×1.3 +1.2 ×1.3) 1 18 3
3.75 + 3.25 2 2
3.75 + 3.25 2 2
cent of 18 ÷ [105] = × =
2 1.1 + 1.2 + 2 (1.1)(1.2 )+ 1.1 + 1.3 + 2 (1.1)(1.3)+ 1.2 + 1.3 + 2 (1.2 )(1.3)
8. = 2 2 2 2 2 2
4 105 70
(3.75 − 3.25) + (3.75 + 3.25)
2
0.52 + 7 2
=  Choice (4)
3.752 + 3.252 3.752 + 3.252
= 2 (1.12 + 1.22 + 1.32 + 1.1×1.2 + 1.1×1.3 + 1.2 ×1.3) 20
2
0.5 + 7 2
(3.75 − 3.25) + (3.75 + 3.25)
2 2
32. of [{−3 + 13) + 5(6 − 2(1))} + 5] =
2 (1.1 + 1.22 + 1.32 + (1.1)(1.2 )+ (1.1) (1.3)+ (1.2 )(1.3)+ (1.2 )(1.3))
2
100
3.752 + 3.252 = 1 Choice (2) 20 20
of [{10+20} + 5] = of [30+5]
3.752 − 2 (3.75)(3.25)+ 3.252 + 3.752 + 2 (3.75 )(3.25 )+ 3.252 20. Following the BODMAS rule 100 100
⇒ {30%of (52.5 × 2 )}÷ 63 × 27 20
3.752 + 3.252 1 = × 35 = 7 Choice (3)
= =  Choice (1)
2 (3.752 + 3.252 ) 2 ⇒ [31.5 ÷ 63] × 27 = 13.5 Choice (1) 100

08_Section.indb 77 10/30/2009 11:53:09 AM


78  n  Solution Manual

33. 53/4 + 101/2 + 151/4 + 201/8 = 50 + 15/8 = 63 93 101 40 131 3

43. , , , , 2a ⋅ 22a + 2 ⋅ 23a − x


515/8 (iii) 
103 188 233 63 238 2a −1 ⋅ 22a − 2 ⋅ 23a

Choice (3)
1 63 40 131
34. of 216 + Each of , , is > 0.5, so we     = 2a + 2a + 2 + 3a − 3 − a + 1 − 2a + 2 − 3a = 22
6 103 63 238  Choice (3)
93 101 (iv)  [22 + 4] –1/3 = (23)–1/3 = 2–1 = ½
1 have to compare and numera-
1 7    188 233  Choice (2)
 of 72 −  of 64 +  of 50  −1 − 1
3 4 5    tor is increasing by less than 10 per cent −1 −1
(v)   1   1 − x2 − 1 
= 6 + [24 − {16 + (70) − 1} −1] while denominator is increasing by ap- 1 − 1 −  = 1− 
 1 − x2  2
 1− x 
= 6 + [24 − {85} − 1] = 6 + [24 − 86] =
93
−56 Choice (3) proximately 25 per cent and hence 1 − x2 1
188     = 1 + = 2  Choice (3)
35. [{4+101 − (5 × 29 × 8} + 17] ⇒ [{105 is the fourth largest.  Choice (4) x2 x
− 1160} + 17] = [−1055 +17] = −1038
44. 3
0.000126 × 0.000735 = 0.05 × 0.09 = 2. If a + b + c = 0, a3 + b3 + c3 = 3abc → (1)
 Choice (3)
a3 b3 c3
1 1 0.0045 Choice (4) The given expression is 2 abc 2 abc 2 abc
36. of 343 + [ of 108 − {109 + 48 − 7}
7 2
− 4] 1 53 a 3 + b 3 + c3
3
abc
45. 1 + =  Choice (4) =2 abc
2 abc
= 23 = 8 Choice (2)
1 43
= 49 + [54 − 150 − 4] = 49 − 100 = −51 4+
1 a b
 Choice (2) 2+  x  y
1 p +  p − 
37. (27.89)2 + (12.09)2 + 2 × (27.89) × 1− q  q
4 3. Given  a b
(12.09)  x  y
46. 3 50 + 2 80 = 5 35 + 3(?) q +  q − 
= ? × 79.94  p  p
⇒ (28)2 + (12)2 + 2(28) (12) = ? × 80 ⇒ 3 49 + 2 81 = 5 36 + 3(?)
(pq + x ) (pq − y)b
a
pa × pb
⇒ (28 + 12)2 = ? × 80 ⇒ ? = 400  3(7) + 2(9) = 5(6) + 3(?) ⇒ ? = 3
⇒ = a b
×
 Choice (2) (pq + x) (pq − y) qa × qb
 Choice (2)
2625
38. 52.1 x 22.1 x 51.5 x 21.5 x 2−0.6 x 50.4 = 47. × 7 + 15 = ? × 3 ⇒ 750 = ? × 3 ⇒ q
− (a + b)

25 =   ; so t = – (a + b) Choice (4)


? = 250 Choice (1) p
3⋅ 0 4⋅0
2 ×5 = 52 × 2 2 = 50 2 1 1

 Choice (2) 48. [{2 − (5 + 21 + 5} − 18] of 100  (32)0 ⋅ 2 + (81)0 ⋅ 25 


4.   =
(25 )5 + (34 )4
[{2 − 31} − 18] of 100 = [−47] of 100 =  (256 ) − (121) 
0 ⋅5 0 ⋅5 1 1
39. Let 815 + ? be k –4700 Choice (4)   (162 )2 − (112 )2
815 + ? 815 × 14 × 6 49. 160 × 300 + 4002 − 402 = 48000 + 160000
× 28 × 3 − = 1680 2+3 5
100 100 − 1600 = 206400 Choice (3) = = =1 Choice (3)
16 − 11 5
84 k 815 × 84 50. [10122 − 1092 + {−1 + 4} + 17] = [10122
⇒ − =1680 ⇒ 84 5. 2100 = (24)25 = 1625; 350 × 525 = {32 × 5}25
100 100 − 1092 + 3 + 17] = [10142 − 1092] = 9050
= (45)25; Therefore 350 × 525 is larger.
 Choice (3)
 k 815   Choice (2)
 −  = 1680
 100 100  33
6. Given 3 3 , (33)33, (333)3, 3333.
k − 815 Indices (33 )33 < (33)33 < (34 )33 and (35)3
⇒ = 20 ⇒ k = 2815
100 < (333)3 < (36)3
k = 815 + ? = 2815 ⇒ ? = 2000
Practice Exercise
∴Ascending order is (333)3, (33)33, 3333, 3
 Choice (1) Solutions for questions 1 to 25: 3
33
 Choice (4)
40. 3
27400 + 3 64100 + 3 7980 ≅ 30 + 40 + 1. (i)  he given expression can be written
T 7. The numbers can be written as (320)1/140,
20 = 90. Choice (3) as [5/3] 2x –3/2 × [43/53] × 52/3 (428)1/140, (235)1/140, (528)1/140
   [1/a-1 = a] [expressing 25 and 9 By comparison of the numbers in brack-
ets, it is obvious that 528 is the largest ⇒
41.
to base 5 and 3 respectively] = 51/5 is the greatest number. Choice (4)
(807 + 289 )2 + (807 − 289 )2  
 = ( ) ( ) 
2 807 + 289 
2 2
 33 43 52
× × = 5–4 . 32 . 43 x
x−y − (x − y )
x
x−y
y
807 2 + 289 2 (807 ) + (289 ) 8.   +  = 2⋅ 
2 2
( ) ( )  53 53 3 Choice (2) y x y
= 2 Choice (3) (ii)  The given expression can be 2−4
2
= 2⋅  = 8 Choice (2)
(x −5 ⋅ y−6 ) ⋅ (x 6 ⋅ y−7 )
−4
42. Applying BODMAS rule, 4
7 + ½ of [8 – 4 ÷ 2 × 3 – 2 + 6 – 2 × 2] written as
x −1 ⋅ y 6 9. 7 x + 2 = 7 6x – 15 ⇒ x + 2 = 6x – 15
7 + ½ of [8 – 2 × 3 – 2 + 6 – 2 × 2]
= 7 + ½ of [8 – 6 – 2 + 6 – 4] = 7+ ½ of = x24 + 6 + 1. y24 − 7 − 6 ⇒ 5x = 17
⇒ x = 17/5 Choice (3)
[2] = 8 Choice (3)     = x27.y11  Choice (3)

08_Section.indb 78 10/30/2009 11:53:10 AM


Solutions for Mathematical Skills  n  79

2 1 Alternate method:
x+
10. Given, 33x-1 + 27x = 5 3 – 5 3 . Substi-
x−
Given p + q + r = 0 ⇒ r = –p – q ⇒ qr Surds
tuting the choices in the above equation it = –pq – q and pr = –p – pq ⇒ r = p
2 2 2 2
can be seen that only choice (3) satisfies Practice Exercise
+ q2 + 2pq
it. Choice (3)
11. (2) (x + 5)/2 = (2) (3x – 5)/4 ⇒ 3x – 5 = 2x + 10 p2 q2 r2 Solutions for questions 1 to 25:
+ +
⇒ x = 15 Choice (3) qr − pr pr − q 2
pq − r 2

12. 2x + 3y = 17 ⇒ 2x = 17 – 3y → (1) ⇒ 2x+1 + 1 1− 2


p 2
q2 p 2 − q 2 + 2pq 1. = = 2 –1
3 = 43 = 2 (2 ) + 3(3 ) = 43
y+1 x y
= + 2 + 1 + 2 (1 + 2) (1 − 2)
− pq − q 2
− p 2
− p − pq − q 2
pq − p 2
− q 2
− 2pq 
3 = 9 ⇒ y = 2; From 2 = 8 ⇒ x = 3 and
y x
Choice (4)
x + y = 5 p2
Choice (2) q 2 p 2 − q 2 + 2pq
+ + 7 1
13. 3x × 5y = 75 = 3 × 52 →−pq (1)− q 2 − p 2 −p 2 − pq − q 2 pq − p 2 − q 2 − 2pq 2. 3 × 7 3 − −
3 3 3 3
3 × 5 = 45 ⇒ 3 × 5 = 9 × 5
y x y x
p 2 + q 2 + p 2 + q 2 + 2pq 2(p 2 + q 2 + pq) 21 × 3 × 3 − 7 − 1 181
⇒ 3y × 5x = 32 × 5 → (2) = 2 2
= = =
–2 −(p + q + pq) −(p 2 + q 2 + pq) 3 3 3 3
For both the equations (1) and (2); We get
x = 1 and y = 2 Choice (1)  Choice (2)
181 3
=  Choice (1)
14. Let 2x = 4y = 8z = k ⇒ 2 = k1/x; 4 = x1/y; 8 = 9
p (p − q )
−4

( ) ( )
−4
k 1/z
 pq − q 
4  1/ 4 1/ 4 1/ 4
 2 2

21.  =  1/ 4 1/ 4  13 + 11 + 13 − 11
 p − 4 pq   p ( p − q1/ 4
)  3.
1 1 5  
( ) ( )
2 2
k1/z × k1/y = k5/x ⇒ + = → (1);   13 − 11
z y x
1/ 4 −4 1/ 4 4
1 1 1 ⇒ (q / p )  = (p / q )  ⇒ p/q 2(13 + 11)
    = = 24
Given + + =4  13 − 11
2x 4y 4z Choice (3)
 Choice (3)
2+5
( )
2 2
⇒ = 4 ⇒ 7 = 16x ⇒ 7/16 = x  1  1 4. 2 + 24 = 2 + 24 + 2 2 24
22. Given  x −  = 64 ⇒ x − = ± 8;
 4x Choice (3)  x x
= 26 + 2 48
1   
2
15. Given 2x = a; 2y = b and 2z = c 1  1
x3 – 3 =  x −   x −  + 3 
x    
( )
2
Given expression = 2x – y + z – 2; x  x  Similarly, 6 + 8 = 14 + 2 48
= ± 8(67) = ± 536
2 x × 2z ac
y 2
=  Choice (1)  Choice (4)
( 3 + 4)2 = 19 + 2 48 ; ( 12 + 2)2 =
2 ×2 4b
23. 3 × 3 × 3 × .… 10 terms
1/2 1/4 1/8

( )
2
1 2 = 31/2 + 1/4 + 1/8 ..… 16 + 2 48 ; 2 + 24 > ( 3 + 4)2 >
16. y = 6 + 6 + 6 ; Using (a – b) = a –
3 3 3 3
Clearly, powers are in G.P.
3a2b + 3ab2 – b3 and (a + b)3 = a3 + b3 + ( 12 + 2)2 > ( 6
1 / 2 1 − (1 / 2)10 
3ab(a + b)) ∴S10 = = 1 – 1/2 10

∴ y3 – 18y2 + 90y = 150 Choice (2) 1/ 2 ∴ 6 + 8 is the least. Choice (2)


17. (102 × 98) (102 − 400) = {(100+ 2) (100
2 ⇒ 3 (1 – 1/1024) 
Choice (3)
5. a2 = 15 + 2 44 ; b2 = 15 + 2 26 ; c2 = 15
− 2)} {(100 + 2)2 − 400} = (1002 − 22) (1002
+ 22) = 1004 − 24 + 2 54
24. Let x = 42 + 42 + 42 + .........

Choice (2) ∴ comparing all the three we can say that
1 1 1 b < a < c. Choice (2)
+ + 1 1 1 ∴ 42 + x = x; Squaring on both sides, 42
18. abc = (abc) p q r ⇒ + + = 1
p q r + x = x ⇒ (x – 7) (x + 6) = 0
2

⇒ x – 7 = 0 or x + 6 = 0 that is, x = 7 (3 + 2 3) (7 + 4 3)
⇒ qr + pr + pq = pqr Choice (2) 6. =a+b 3;
or –6 (7 − 4 3) (7 + 4 3)
19. For the product to be more than 1000, the But x cannot be negative (any square root
decimal should be placed between 4 and must be positive) x = 7 Choice (1) 45 + 26 3 = a + b 3
5 that is, the decimal should be moved Equating the similar terms; a = 45; 26 3
seven places to the right hence the value 1 1 = b 3 ⇒ b = 26 Choice (4)
25. x + = 6 ⇒ x2 + 2 + 2 = 36;
of k is 7. Choice (3) x 2 x
 1 1
20. Suppose p = q = 1; p + q + r = 0; ∴ r = 2
 x −  = x + 2 − 2 = 34 – 2 = 32 7. Given surd 98 − 80
–2  x  x

p2
+
q2
+
r2
would x2 −
1  1 1
= x +  x −  = 6 ×
x2  x x
= (7 2 − 2 20 ) = 2 ( 5 − 2)
1/4

qr − p 2 pr − q 2 pq − r 2 
 Choice (2)
have a value of –2 for the above values 32
= 24 2 1 1 3+ 8
8. = × =3+ 8
of p, q and r. It can be seen that taking any 2 3− 8 3− 8 3+ 8
1 
 2
( )
2
other combination of values of p, q and r  x − 2  = 24 2 = 1152 1
 x  Similarly = 8+ 7,
the given expression has a value of –2. Choice (2) 8− 7

08_Section.indb 79 10/30/2009 11:53:12 AM


80  n  Solution Manual

2 ( 3 −1) 2 ( 3 − 1)
1
= 7+ 6,
1
= 6+ 5 =
( 3 + 1) ( 3 − 1)
=
3 −1 17.
(
2 3− 7 ) =
2(3 − 7)
7− 6 6− 5
(3 + 7 )(3 − 7 ) (3) 2 − ( 7) 2
1
and = 5+2 = 3 −1
5−2
= −1 + 3 = A + B ⇒ (A, B) 2 (3 − 7)
∴Given expression = =3– 7
9−7
= (−1, 3) Choice (2)
= (3 + 8) − ( 8 + 7) + ( 7 + 6) − ( 6 + 5) + ( 5 + 2) x2 − 6x + 2 = (3 − 7 )2 − 6(3 − 7 ) + 2
8+ 2 ( )( 3 − 2 ) = 18 − 18 − 6 7 + 6 7 = 0Choice (3)
+ 8) − ( 8 + 7) + ( 7 + 6) − ( 6 + 5) + ( 5 + 2) = 5  Choice (2) 14.
3+ 2 ( )( 3 − 2 )
15
( 3 + 2)
2
9. x = 5 + 2 6 =

+
(8 − 2 )( 3 + 2 ) 18.
10 + 2 5 + 2 10 − 5 − 4 5

x +
1
=
x +1
=
5 + 2 6 +1 ( 3 − 2 )( 3 + 2 ) 5 5( 10 + 5)
x x 3+ 2 = =
10 − 5 ( 10 − 5)( 10 + 5)
8 3 + 6 −8 2 −2
6+2 6 =
= =2 3  Choice (3) 3−2 = 10 + 5  Choice (3)
3+ 2
8 3 +8 2 − 6 −2
+
3−2 19. The conjugate of 12 − 80 is 12 + 80
10. 16 – 2 20 – 2 28 + 2 35
= 16 3 – 4 = 16(1.73) – 4 = 23.68.  12 + 80 = 12 + 4 × 20
=7+5+4–2 4× 5−2 7 4+2 7 5  Choice (3)

( a+ b− c )
2

15. x =
( 3 + 2 )( 3 + 2 ) = = 12 + 2 20 = (10 + 2 )
2

=a+b+c–2 a b −2 a c +2 b c ( 3 − 2 )( 3 + 2 ) = 10 + 2= 2 ( 5 + 1)
∴ a = 7, b = 5, c = 4  Choice (1)
( 3 + 2)
2

∴ The square root is 7+ 5– 2 20. Applying componendo dividendo rule,


(3 − 2)
 Choice (3)
x + 1 ( a + 2b + a − 2b) + ( a + 2b − a − 2b)
11. Let the second surd be x + y. =3+2+2 6 =5+2 6 =
x − 1 ( a + 2b + a − 2b) − ( a + 2b − a − 2b)
1
( ) ( )
3 3
1 + 12 2 + x + y x3 + = 5 + x2 +61 = +( a5 +−2b
2 +6 a − 2b) + ( a + 2b − a − 2b)
=5+9 2 x3
2 x − 1 ( a + 2b + a − 2b) − ( a + 2b − a − 2b)

⇒x+ y =9+6 2  (5 + 2 6 + 5 − 2 6 )3 − 3 × (5 + 2 6 )
⇒ 2
 x + 1  a + 2b
  = ; Again ap-
(5 − 2 6 ) (5 + 2 6 + 5 − 2 6 )  x −1 a − 2b
9 + 2 18 = 6+ 3 Choice (4)
= (10)3 − 3{52 − (2 6 )2} (10) = 1000 − plying componendo dividendo
30 = 970 (x + 1) 2 + (x − 1) 2 a + 2b + a − 2b
4 =
12.  Choice (1)
2+ 3+ 5 (x + 1) 2 − (x − 1) 2 a + 2b − (a − 2b)
16. (x2 – y2) = (x + y) (x – y)
4( 2 + 3 − 5) x2 + 1 a
=  5+ 2 5 − 2  5 + 2 5 − 2 = ⇒ b(x2 + 1) = ax Choice (4)
( 2 + 3 + 5)( 2 + 3 − 5) = +   −  x b
 5 − 2 5 + 2   5 − 2 5 + 21.
2  As the original method involves more
4( 2 + 3 − 5) steps, back substitution is the shorter
=  5+ 2 5− 2  5 + 2 5 − 2
2 6 method.
= +   − 
 5 − 2 5+ 2   5 − 2 5 + 2  Put x = ± 2
1
= (2 3 + 3 2 − 30)  Choice (4)
 2(5 + 2)   4 10  14 4
3 = = × 10
( ) ( )
[( ± 2 )2 −1] [( ± 2 )2 −1]
2   ∴ 2+ 3 + 2− 3
13.  3   3  3 3
4
16 + 12 + 2 16 × 12
14 × 4 56 10
( ) ( )
2 −1 2 −1
3(x 2 − y 2 ) = 3 × 10 = = 2+ 3 + 2− 3 =4
9 3
2
(2 + 3 ) + (2 − 3 )
−1 −1
= If x = 0, = 4
( 16 + 12 )  5+ 2  5− 2
2
4 ⇒ xy =     = 1  Choice (1)
 5− 2  5+ 2
2 2( 3 − 1)
2 2 3x2 – 3y2 + 4xy = 3(x2 – y2) + 4xy = 22. =
⇒ = 1+ 3 ( 3 + 1) ( 3 − 1)
( 3 + 1)
2
4+2 3 56 10
+ 4 Choice (1) 2( 3 − 1)
3 = = 3 –1
( 3) 2 − 12

08_Section.indb 80 10/30/2009 11:53:14 AM


Solutions for Mathematical Skills  n  81

2 Logarithms 3
Similarly = 5– 3
−2
= 96 × 6log6 8 = 96 × 8−2 =  Choice (2)
3+ 5 2
2 2 Practice Exercise 8. log (p + q) = logp + log q = log (p) (q). p + q
= 7– 5;
5+ 7 7+ 9 = (p) (q) ⇒ pq – p – q = 0
Solutions for questions 1 to 25:
Adding 1 on both sides, pq – p – q + 1
=3– 7 ; log 26 6 log 2 =1
1 (i) log 2 = = =4 p (q – 1) – 1(q – 1) = 1 ⇒ (p – 1)
2 2
log 23/ 2 3 / 2 log 2
= 19 – 17 (q – 1) = 1 Choice (1)
17 + 19  Choice (2)
∴ The given expression = 3 – 1 + 5 log 73 3log 7 3 1 1 1
(ii) = =  Choice (2) 9. + +
log 7 2 2 log 7 2 log a a + log a bc log b b + log b ca log c c + log c ab
– 3 + 7 – 5 + 3 – 7 +......+ 19 –
(iii) log10 (200 × 500) = log10 (105) = 5
17 1 1 1
 + Choice (3) +
log a a + log a bc log b b + log b ca log c c + log c ab
∴ Required value = 19 – 1 Choice (1)
2 4
2. Given logab = , logcb = and 1 1 1
23. x = 14 + 6 5 3 3 = + +
log a abc log b abc log c abc
x
x = 14 + 6 5 = 3 + 5 → (a); logca =
8 = logabca + logabcb + logabcc = logabc
2 3 x abc = 1
1 1 3− 5 4 (logab) (logbc) (logca) = . . ⇒1
= = ; =3− 5 3 4 8  Choice (1)
x 3+ 5 4 x
→ (b) 10. Given that a2 + b2 = 7ab; a2 + b2 + 2ab = 7ab
x
= + 2ab
4 16
x+ = 3+ 5 + 3 − 5 = 6 ⇒ x = 16 Choice (2) (a + b)2 = 9ab ⇒ a + b = 3 ab ;
x
 Choice (3) 3. (i) log(x2 – 1) = log2 (since loga + logb =
log ab). (a + b) a +b 1
= ab ⇒ log   = log ab
Therefore, x2 – 1 = 2, ⇒ x = 3 3  3  2
4 3 4 3(2 + 2)
24. = =4 3+2 6  Choice (4) 1
2− 2 2 = (log a + log b)
(ii) (5x + 2)/7 = (x – 2) ⇒ 5x + 2 = 7x – 14 2
30 30(4 3 + 3 2)  Choice (1)
= ⇒ 2x =16 ⇒ x = 8 Choice (3)
4 3 −3 2 48 − 18
11. log4 (log3 (log2x)) = 0 ⇒ (log3 (log2x)) =
30(4 3 + 3 2) (iii) Given, log10x + log 10 x + log 3 100 x 40 = 1
⇒ =4 3+3 2 = 27
30 ⇒ log2x = 3 ⇒ x = 23 = 8 Choice (3)
Let log10x = p; Given expression
3 2 3 2(3 − 2 3) log a log b log c
= 3p 12. Let = = = r ; log a = r
3+ 2 3 9 − 12 becomes p + 2p + = 27 ⇒ b−c c−a a−b
2
−3 2(3 − 2 3) (b – c) ;
⇒ = (−3 2 + 2 6) 9 log b = r (c – a); log c = r (a – b)
3 p = 27 ⇒ p = 6
2 aloga + blogb + clogc = a[r(b – c)] + br
4 3 30 3 2 log10x = 6 ⇒ x = 106 Choice (4) [c – a] + cr [a – b] = 0
= − −
2− 2 4 3 −3 2 3+ 2 3 a log a + b log b + c log c = 0; logaa × bb × cc
4. logba = 4 ⇒ a = b4 → (1); log2b 64a = 5
= log 1 ⇒ aa × bb × cc = 1 Choice (4)
(4 3 + 2 6) − (4 3 + 3 2) − (−3 2 + 2 6) = 0 ⇒ 64a = (2b)5 = 32b5
64b4 = 32b5 ⇒ b = 0 or 2; b represents a (log a x .log y a) (log x a .log y a)
 Choice (1)
base in the first equation given. Any base 13. =
1 1 log a y + log a x
must be positive and other than unit b ≠ +
99
1 0; b = 2, that is, b can assume only one log a x log a y log a x .log a y
25. Given expression ∑
x =1 x + (x + 1) value. Choice (1)
 1 1 
1 1 1 5. log2 (x + 4) . (x – 4) = log2 48 ⇒ x² – 16 =  .  (log a x .log a y )
= log x log a y
=
+ + ..... + a
1 + 1+1 2 + 2 +1 99 + (99 + 1) 48
log a xy
1 1 1 ⇒ x² = 64 ⇒ x = 8 (∵ log is not defined
⇒ + + ....... +
1
1+ 2 2+ 3 99 + 100 for negative numbers) Choice (2)
= = logxya Choice (3)
6. logpq – logpp = a ⇒ logpq = a + 1. log a xy
1 1 1
= + + ...... +
2 +1 3+ 2 100 + 99 p 1 14. We know that log15210 < log15225 and
Now logq   = logqp – logqq = –1 log13180 > log13169
After rationalizing denominators of each q a +1
fraction, we have ∴log15210 < 2 → (1), ∴log13180 > 2 →
  (2)
⇒ 2 − 1 + 3 − 2 + ….+ 100 a 1
=– ∵ = log q p   Choice (1) From (1) and (2), log15210 < log13180
a +1  log q 
− 99  p  ∴ 2log15210 < 3log13180; ∴Both (1) and (2)
7. 36 × 36 1/6
½ log 8
× 36 36 ; 6 × 16
log 16 are true.
⇒ 100 − 1 = 10 – 1 = 9 Choice (3) × (62) – log68  Choice (3)

08_Section.indb 81 10/30/2009 11:53:15 AM


82  n  Solution Manual

15. 3 = 3 log10 y – log 10 x ⇒ 3 = log 10 (y3/x) 25. log2 8 [0.3 + 0.03 + 0.003 + …..)] A
(y3/x) = 103 ⇒y3 = 103x ⇒ x = y3/1000 = log2 23 × [1/3] = 3log22 × [1/3] = 1
 Choice (4)  Choice (2)
30° 60°
16. Given log5x = a and log2y = a ⇒ x = 5a, y B1 C B2
= 2a
Trigonometrical ⇒ B1C = 45 3 m
2a 4a
(10a ) 4 (xy) 4
1002a – 1 = 100 = 10 = − Ratios and
100 100 100 100 AC
Heights & Distances In ∆AB2C, tan60° =
 Choice (4) B2 C
⇒ 3 = 45/B2C
log a log b log c Practice Exercise
17. = = = k ⇒ a = 104k; 45
4 3 5 Solutions for questions 1 to 25: ⇒ B2C = = 15 3
3
b = 103k; π
c = 105k ⇒ a2 = (104k)2 = 108k = bc 1. (i)  We know that 1° = Radians. B1B2 = B1C + CB2; 45 3 + 15 3 = 60
180
 Choice (4) 3
π 2πc
18. If a, b and c are in continued proportion,   120° = 120 × = .Choice (3)
180 3 distance 60 3
Speed = ; Speed = m/s
⇒ b2 = ac; ∴ 4x = (logxa) log x ; (22)x = (ii) We know that πc = 180° time 15
a

1 5πc 5 72 3
2 (∵ logxa logax = 1) ⇒ 2x = 1 ⇒ x = = × 180° = 300°. Choice (4) = km/hr Choice (4)
2 3 3 5
 Choice (1)
2. Since θ is not in first quadrant and cosθ 8. Let AB = h2 m, CD = h1 m be the heights
19. log10 + log (x2 + 5x) = 2 log 60 ⇒ 10(x2 40 of the two towers and CM = MA = x
+ 5x) = 60 ⇒ x2 + 5x – 6 = 0 ⇒ x = 1 or – = , θ must belong to fourth quadrant.
41 D
6 Choice (2) A
B
41 h1
A 2 − B2 m 9 h2
20. log10 = log102 (∵ loga  75° 15°
(A − B) 2 n C M A
B 40 C AB
= logam – logan) In ∆MAB, tan15° = ⇒
AM
−9 −9 h
From the table tanθ = , sinθ = . 2 −9 3 = 2 CD h
2
A −B 2
A 40 From41 the table tanT = , x In 'CMD, tan75q = Ÿ 2  3 = 1 Ÿ (2
= 2 (∵ if logca = logcb, a = b); 40 CM x
(A − B) 2 −941  −9  9
∴ tanθ – sinθ
9 = –   sin = –T = 9 .. ⇒ (2 − 3 )x =h2 3 )x = h1
A  40T  41  1640 41
A + B = 2(A – B) ⇒ 3B = A ⇒ =3 ?h1 : h2 CD = (2  3 )x : (2  3 )x = 7  4 3 : 1
B B 40 C Choice (2) 9 ∆CMD, tan75° =
In ⇒ 2 + 3 Choice (1)
 Choice (1) ? tanT – sinT = – CM
3. sin4x + cos4x + 2sin2x cos2x = (sin2x + 40 h
C
21. 2 logx + log(1+2/x) = log3 ⇒ log (x² cos2x)2 = 1 § 9 · 9 = h1 ⇒ (29.+ In3 ')xABD, tanD =
200
= – . Choice (2) = h1
(1+2/x)) = log3 ¨ ¸ x AC
© 41 ¹ 1640 50
 Choice (1) In 'CDA, tan2D =
x(x + 2) = 3 ⇒ x = –3, 1. Hence, 1 is 4 4 2 2 2 2 ∴h2 1 : h2 = (2 + 3 )x : (2 − 3 )x AD
= 7 + 4
the positive value of x which satisfies the 4. x2 + y2 3.
= (asin
sinθx ++cos x + 2sin
b cosθ) 2 x cos
+ (a cosθx =– (sin
b x + cos x) = 1 2 tan D h  50 B
Choice (1)
3 : 1 Ÿ Choice (1) n
given equation. sinθ)2 = a2 (sin2θ + cos2θ) + b2 (cos2θ + 1  tan 2 D 200 h
 Choice (4) h D
sin2θ) 4. x2 + y2 = (a sinT + b cosT)2 + (a cos T –9. b sinT)2 =
In ∆ABD, tanα = 2h D Ap
2 2 2 2 2 2 D
22. P = log10 (x2 + y2); Q = log10 (x + y) ∴x2 + y2 = aa2 +
(sin
2
b2T2 + cos
2
T) + bChoice
2
(cos T (3)
+ sin T) 200
200 h  50 200 m
?x + y = a + b 1 Choice (3) ;
2Q – P = 2 log10 (x + y) – log10 (x2 + 5. cosθ + secθ = 2 ⇒ cosθ + =2 h2 C 200
1
y2) cos θ 1 200
2
5. cosT  secT = 2 Ÿ cosT  =2
⇒ cos2θ − 2cosθ + 1 = 0 ⇒ (cosθ −cos1)T2 3 2 50 2 2
(x + y) 2 (x + y) 2 h  50h  h(200)  (200) 50 = 0 Choice (3)
log102 = log10 2 2
⇒ 2 =2 =0 Ÿ cos2T  2cosT  1 = 0 Ÿ (cosT  1)2 = 0 B
x +y x + y2  40
cosT==11; secT = 1
⇒ cosθ = 1;Ÿsecθ 4 4 4 4 10. In 'CAB, tan h D= Ÿ x = 40(cotD – 2)
? cos T  sec T = (1)  (1) = 2. Choice (3)  80  x
⇒ x2 + y2 + 2xy = 2x2 + 2y2 ⇒ (x – y)2 ∴ cos4θ + sec4θ = (1)4 + (1)4 = 2.Choice (3) D
 A 
200 m AB B
=0⇒x=y 6. BC = Wall; AB = Ladder
6. BC = Wall; AB = Ladder In 'DAB, tan2D = ;
 Choice (1) AC x
B
In ∆CDA, tan2α = ⇒
2 tan
AD D 40
23. logb a(1 + a) = 0 ⇒ a(1 + a) = b0 ⇒ a + a2 BC 20
40 T  tan 2 D 40 cot D  2
h +1 50
cosT = ; cosT = ;
=1 20 AB 240
tan α
2
=
⇒ a2 – 1 = –a Choice (3) 1 1 − tan α 200 2
tan D  4tanD  1 = 0
A C cosT = Ÿ T = 60q D 2D
24. Number of digits in a number expressed 2 2h 4 r 16  4 C m 80 oD m x o A
BC 20 1 Choice (2) htan
+ D50=
in base n equals 1 more than the integer cosθ = ; cosθ = ; cosθ = ⇒ θ 200 = ; 2
part of its logarithm to the base n. log(370) = 60° 7.ABIn 'AB C, tan30
40 AC 2 1 45 h2 200
tanD = 2  3 , or tanD = 2  3
 1 q= Ÿ 1 −
= 70log3 = 70 (0.4771) = 33.397 BChoice
1C
(2)
3 B1C (200 )
2
Since D and 2D are always acute. ? tanD = 2
Integer part of log(370) = 33; ∴ 370 has 34 Ÿ B1C = 45 AC 3 m 1 45 A h3 + 50h2 + h(200)  3 Ÿ2 D−= (200)
15° 2 50 = 0 Choice (1)
digits. Choice (4) 7. In ∆AB1C, tan30° = ⇒ =
B C AC3 B C  Choice (3)
In 'AB2C, tan60q =
1 1 230  x
B2 C 11. From ' ABC, tanE = o (1)
AC
Ÿ 3 = 45/B2C 30q x
60q From ' DEB, tanD = o (2)
45 B1 C B2 DE
Ÿ B2C = = 15 3
3 B
08_Section.indb 82 B1B2 = B1C  CB2; 45 3  15 3 = 60 3 (1) = 10/30/2009 11:53:17 AM n
(2)
36 E BC BC
AC In ෙ AED, tanD = Ÿ A
In 'CDA, tan2D = 50 AE D 1Ÿ AB = BC
AD In 'DEA, sinD B
36  AB BD 1 BDh h 500 m
B 17. In 'ABD,
tanD = tan30 q = Ÿ
(2)
oAB = Ÿ BD
2 tan D h  50 30 3 50 m 10 3 300 D Let, AD = BE =
Ÿ n = Ÿ sinD = BC m; In 'DEC, ta
1  tan 2 D 200 h 50 5 40 300 (1) C B
D 50 m E
= = 100 3 h C DE
2h D Ap AE = 1 Ÿ 30 =
0
D In ADE, tanD = ; 3 (2) 36  h
200 h  50 200 m
AD D T CE
h2 200
;
3 30
D C A 2hSolutions
= 36 Ÿ h =for Mathematical
18 m Skills 200n  3 831 500
1 ; AD = 40 m BC Choice D (1)Ÿ
In ෙ ABC, tanD = 3 500  x
200
2
4 AD AB
13. Let AB pole, AC Choicebe (2)the shadow of h h
h  50h  h(200)  (200) 50 = 0 Choice 40 D
3 2 2 2
(3) 16. tanD200 = 3and  100htan
2 E =
3
10. In ∆CAB, tanα = ⇒x the pole andofθthe be pole
the angle of elevation Ÿ tanD = 91 = 4 30o D Required spe
80 + x 13. Let AB pole, AC be the shadow and T 300 36 A B
40 = 40(cotα – 2) be the angle of elevation of theofsun.the sun. h h 300 m
10. In 'CAB, tanD = Ÿ x = 40(cotD – 2)
80  x ?Given AC = 3 AB B = 3
Ÿ D = 60q
tan D .tan E =
(∵ α,
.
9 β4 are D
Choice (3)
h
E
= 25 3 1 m/s
AB B 2complementary) A B
In 'DAB, tan2D = ; AB h C
In 'ABC tanT = (จ Do , E are D
x AC 18. In ෙ 1ADE, =
36tan 30
2 tan D 40 h = 6 m. Choice (4) 20. In 'ABC, tan
AB DE complementary)
1  tan 2 D 40 cot D  2 40 Ÿ tanT = T =
AE h = 6 m. Choice (4)
x In 'ABD, tan 45
3AB C A
1 x 30o BD 1 BD AB
2 Ÿ = 17. In ∆ABD, tan30° = ⇒E = = Ÿ AB =
tan D  4tanD  1 = 0 D 2D InŸ tan'TDEA,
1 sin D∴Given AC = 3 AB A BD
= Ÿ T = 30 q B 3 AE BD 1 60 BD o
AB 3 300
4 r 16  4 C m 80 o D m x o A In ' DEA, sin D B 17. In ' ABD, tan30 q BD
= Ÿ 1 BD
= Ÿ BD 10 BD
30 3 3 17. In Ÿ'ABD, 300
tanD = 30
= Ÿ sinD 3= 50 m 10 AE = 3tan30 x ⇒ q = AB
BDAB =
10 m =
Ÿ
=
3 300 300 Ÿ BD C
2 = 50 Ÿ sinD = Choice 5 (3) 50 m AB 10 A 3 100 3 C
In 5 'DEA,In ∆ABC tanθm = E E 4040 B In300 ෙAEF, tan 60 3 B BD
o
50 sinD 50 300
= = 100 3 1 BDC 200 3
tanD = 2  3 , or tanD = 2  3 AB 3 AE 50 m AC BC
Since D and 2D are always In ∆DAB, tan2α
acute. ? tan= D = 2; 14.InGivenADE,sin tan T30D== AE ; 3 10
= 317.
EF = 100 In 3'ABD, tan30q =
In ∆ ABC, tanα =
AB
Ÿ = C Ÿ BD
3 30010+xD
x In ADE, tan =D = Ÿ 5 sin
AD ; D = D T 50 m = 3 . AB
2 tan α 40 50 AD 5 AB T 200 3
D D A AE 300 200 3
 3 Ÿ D = 15° = Choice (1) 3 30 3 ⇒ D tanθ = C C T50Am E 40
=  10 = BC 3
100 D C
1 − tan 2 α 40 (cot α − 2 ) 3 Ÿ30 tanT; AD=; AD = 40 m
= 40 m
AE 3AB
C 20m B In ෙ ABC, 10
3 = tanD3 = AB tan D  BC
= x
 F
D
230  x 4 AD In 4ADE, tanD = ;
T In ෙ ABC, Ÿ 3x 30°
11. From ' ABC, tanE = o (1) 4 AD AD D
1 ABChoice (2) 3x AB A B 200 3
ABtanθ =3 D A
ACα − 4tanα + 1 = 0 tanα
tan 2
ChoiceC(2) o D 300 m
In 'ABC3= tan 30T = ⇒ Ÿm = ⇒ θŸ=AB 30°= 15 Choice Ÿ =(3)20 D
tan x= 200
 200 Ÿ x3 = 3 10

 100
100 m 3 3BC 30
oD D
13. Let AB pole, AC be the ; AD = 40
shadow 3
4of of the pole and T D = In ෙ ABC, tantower
D = =200 30
x 4 ± 16 − 4 13. LetbeAB pole, 4 ACof ADbe theBC shadow 20
the pole and T Ÿ tan ? The height of300the AB x A 103 +=300100
20 m m 3 B
From ' DEB, tanD = = o (2) the angle
m. angle of elevationChoice elevation of the sun.
(2) Choice (2) = 3
300
Choice⇒ tanα (1) = A 300 m = 3
B
DE be the of the sun.
2 ?Given AC = 14. 3 ABGiven sinθ = 3D B
B = 3 200 3  100 3 300 30o D
B ? Given AC = 3 AB 19. Ÿ Let D =
the 60 Ÿ q tan
altitude D =at which the aeroplane Choiceis (3)
flying be
15. In ෙ13. ABC, Let AB pole, AC be the shadow
AB 5 of the pole and TŸ D = 60q ⇒ α = 300 60° ChoiceA (3)300 m ChoiceB(3)
(1) =
tanα = 2 + 3 , or tanα n = 2 − 3In 'ABC be tanthe = angle of elevation of the sun.
T AB AB.
(2) In ' ABC tan hT = 18. In ෙ ADE, tan 30 o D
Since α and 2α are always acute. tan
x ∴ Dtanα = ?Given o ACAC = 3 AB 3 B18. In Inෙ 'ABC, =
ADE, tan
3tan45°30 o A x D D
AC(1) DE Choice (3)
4 D
D
E
p In BCAB sinD⇒ tanθ = 4
'DEA, B DE AB Ÿ D = 60In
AB
18. q ∆BD ADE, 1tan 30
BD o
=
5 230  x = 2 − 3 ⇒ α = 15° Choice
Ÿ tanT(1) =AB ABC tanT = AB T 17. In DE ='=ABD, Ÿ tan30q == Ÿ = Ÿ AE BD
= Ÿ tan 30T = In ' 3 DE C 36 D = AB BC xD
5 x =In ෙ Ÿ sin
AED, D3AB= D = In Ÿ
tan AC
T 50Em 10 A AB A⇒ 3 =AE AE 18. In ෙBCADE,AB 1 tan 30 3 o 300
x x
12 50 3AB 5 AE ∆ABC
C = tanθ = A D o 500
230 m E BC40 300
4 = Ÿ20 1 Ÿ 1 AB x
= BC ⇒= = 30
AB 3DE
o
230m+ x
230
Ÿ1tan
50 m 1= 100 m=x 3 AE A 30 o C E
48 230  x 11. From ∆ ABC, tanβ = →Ÿ(1) tan T =136  AB T AE
T= = 30
h T h Ÿ 3500 = =AE A E
DŸ q 3AD 60 x
E AC Ÿ In tan
tan ADE,
TD == tanŸ 3
=
T AD= 30 o;3AB
q⇒(2)
AB D= 15 m.
T C D A Choice Let,3
(2) AE = AE BE = x 30°
60 o
10
25 x 3 BC D A Ÿ m; AE In = ' 3
DEC, x 1 ⇒x AE 10
tan = 10 3mx
45° 200o10 3
30
C A 3 30 Choice Choice1(3) C C B Ÿ AE = 3Ÿx BC m
x ; AD =h 40(3)
A = C AB Dx o C E E
25 u 230
From ∆ DEB, tanα = → (2) 4(1) AD Ÿ = tanT = 1ŸŸT = 30q
m A In ෙInABC, 0 ෙAEF, DE
tan D = 3tanAE60 o
o
B
B 60C
DE (2) 36 3  h 3 In 30ෙAEF, = tan
AB 60 10
x=
23
= 250 14. Given sinT =3 Choice (2) EF CE Ÿ AE = 3 x 10 m
10+x
B 14.13.
Given2h =sin T=
36 Ÿ h 5= 18 m Choice (3) A Ÿ tan EF .200
= 3  100 In 3
∆AEF, 30 oD
10+x C
1 500
In 'DEB, sinD =
BE
Ÿ
5 250
; BD = 650 m 
Let AB pole,5AC be the shadow of the pole
betan theTangle 3 of elevation3 of the sun. T TChoice (1)
=3Given
and T = AE
Ÿ
AE
D .= In ෙAEF, tan 60
310 500
300 Ÿ x = 500
10  x
A o
1mB
3300 B
BD 13 BD x ŸŸ tan 14.
T = AC4 = 3 AB 5 sin T = C 20m B = 3 3 10 EF
= =10  x. Ÿ 3xo EF F 10+x
C B B
Choice
D
(4)
E

16.
? Given
tan D = 4h and tanE = h
9 AB 3AB4 3 AB
20 mD
T
3=
Ÿ D = 60q 3x
Required
AE3x Ÿtan3x60 500
speed
=
=
AE 3 .Choice
 1
m/s
(3)
F

In
In ' ABC tan
'ABC = tan T =T ==ABIn ∆Ÿ
tan 3 AB = Ÿ AB = 15 = 20  x Ÿ x 10 = 10 mo10m x
In 'ABC =Ÿ tanT T=15. Ÿ ABC, = Ÿ ABC = 15 20m18. BIn= 20 ADE,x Ÿ x3tan =
= 10 Ÿ= 20
D F
12. ?In 'ACB, sinT =
40
sinTm=
Ÿ 230
4 h AC hBC 4BC 4 4 2020 ?ෙ The height of30 the tower 10 3x
+x10 + 10x= 20 m
50 5 230 m m.tan D .tan E = .
m.Ÿ tanT = AB9 4 Choice (2)ABT D 3 AB E
In 'ABC = tanT =
Choice (2)
Ÿ =
h
Ÿ AB = 15 =
DE
? The height of
= 25 3 Choice the
1Choice
m/s.(1)
3x
tower
3(1)
= 20  x Ÿ x = 10 m 3x
= = x 10 = 20 m
⇒Choice3x = 20 (2) + x
x
 Sol/868 15. In ෙ ABC, h2
3AB C BCA D 4 CA 20 B 19.AE Let the altitude ? Theatheight at which the tower
of the aeroplane = xis is 10flying
= 20 be
15. In ෙ1 ABC, = m. (จ D, E are Choice (2)
D 19. Let 1
AB.the altitude
x which
⇒ xChoice=the10 aeroplane
m 30o flying bem
C A 1 ŸAB. = A 30 (1) 30 E
36
h 20.'ABC, InAE 'ABC, tan∴60° The = height Ÿ of the=A tower x 10 = x 3D+ 10
ŸD tan=Th= Ÿ (1)T = 30q In tan 45° 60BCo =
tan
complementary) o D In '3ABC, 19. tan Let the 45° BC A 3x10 isD flying
tanD = 15.BC Ino 3ෙ ABC,
(1) AB 3 AB altitude
= 2010 matm which the aeroplane Choice be (1)
h = 6BC m. Choice (3) Choice A (4) Ÿ AE =AB = x
Ÿ
In 'ABD, tan 45° AB
AB. =
(1) 4 48 230 +ෙ xAED,tan hDE 36 D = BCŸ In tan
ෙAEF, BC
'19. = tan
ABC, Leto
the
45° Baltitude at which A C
the
A aeroplane
x D is
= 5 230 + x = = In In tan D D= DE
= oŸ(1)36 E E DD A In BC AB BC 60
(2) 5 = x 25 14. ෙx AED, tanD 3= BC AEŸ D
A
EF 1=Ÿ AB Ÿ = AB BCAB == AB
flying be AB. 500
500
12
Given sinT = AE 1 Ÿ
500 AB
. m BD= = BC = Ÿ = 10+x
36  AB 5 DE h h 36 E h h D =500 m AD = BE BC = x BC
25 × 230 tanD =36  In3AB ෙ AED,otan (2)D = Ÿ T D A AE BD
Let, 30° AB AB
x= = 250 tanD =
Ÿ tanT =BC BC o (2) AE C D D Let, AD
m;10In 10
= BE
1
'DEC, Ÿx AB = x
tanIn= ∆ABC,
BC 45° 45° 30°
= tan 45° = ⇒7.5500 = 1
23 C B B F x
4 h 36  AB C
20m h B h
3
m; = In ' DEC, 500 m Ÿ
tan 3x C B BC BC
E
(1) DE
3x C 45° B60° x E
BE 5 250 (1) (2)= =tanhD = 1 AB 1 Ÿ o3 (2)AB D 30
0
=DE Let, AD = ⇒BE AB == x BC = 500 m30°
In ∆DEB, sinα = ⇒ = In(2) ;'ABC 36 = 36 tanTh= BC Ÿ Ÿ = Ÿ AB = 15 =30 0
20 =x ŸCE x = 10 ' mDEC, tan D x C 45° Sol/869
B
BD 13 BD
h BC
h
4 20 C B? The height CE m;ofIn Let, AD
the tower = x =10 BE C
= 20= xmm; In ∆DEC, B x E
2h = 36 Ÿ (1)h = 18= mChoice hŸ 1 Choice 500 DE

2hm. = 36 Ÿ h = 18 m tanα = 1
(2) → Choice
(1) Ÿ1 30 = (1)
500 0
Ÿ x = 500 3 1
BD = 650 m Choice (4) (2) 36  h BC D Choice (1)
(1)
Ÿ 3 500 500  xCE Ÿ x = 500 3  1
DE is flying be
15. In ෙ ABC,2h = 36 Ÿ h = 18 m 19. Let the3 altitude atx which tan the
300aeroplane
=

h h 1 500 500 CE
12. ∴In ∆ACB, sinθ =
40
⇒ 16.
16. tan D
D4=D = 9
sinθtan=tan
=h and tan E =h
h tanE = In ∆ AED, tanα =
and 4
D DE
D
Choice (1)
⇒ tanα In 'ABC,
AB.
Ÿ
tan 45°3speed
Required 500 =500 x
Ÿ x 3 1
3 =1A500m/s x 3 1
D
9 BC o (1) 4 AE Required speed = 20 m/s
50 5 AB AB
h hh h D = Ÿ = 20 500 3 1

tanD.tan 16. Etan =hD h=. DE and tanE = hD
B tanInD.tan
ෙ AED,E = tan.9D =49 =Ÿ36 −36AB 4 E D→ h(2) AE BC25 3BC
=
=Ÿ25AB 3=BC
1Required
1 m/s.
m/s. speed = Choicem/s
Choice (2)
(2)
9 4 AE BC D D E 1 = 500
20
10 2 h h C B
50 m 2h tan A

50 m
E 40 1 tan
1 =D h= 36
= 36 BC
 D(AB
จ.tanDE, =E are
(จ D, oE (2)are (1)
.
9 4 =
A h h C hh B
D = 1D ⇒ 2h = 36E⇒
500 m
Let, AD = BE
hIn 'InDEC,
20.

= 25
= x 3 1 m/s.
'ABC, tan 60° 45°
3030°
=30 Ÿ BC =30
30 Choice (2)
= 10 3
36 (2)
complementary) h h
2 36C − h A B C m;
20. B In ' ABC, tan
tan 60° =
C BC BC Ÿ BC B = 3 E 3
= 10
  h(1)
complementary)
= 6 m.=1 = 1 (Ÿจ D, E are Choice (4) DE 3x
= 18 m Choice
30
0
In
= (1)
' ABD, tan 45° 30 30
D C A h = 6 (2) m. 36  h36 Choice (4) In 'ABD, CE 20.
tan In 45° 'ABC, tan 60° = Ÿ BC =A = 10 3
1 500BC
30 3
complementary)
2h = 36 Ÿ h = 18 m
h = 6 m. h hChoice (4)Ÿ=
=
1 AB
AB
BD
500Ÿ AB =⇒
Ÿ InAB Ÿ=x =tan
'ABD, 50045°
=
3 3500 1 + x
(
⇒ x = 500
A
) 3 3 −1
In ∆DEA, sinα = ⇒ sinα = 16. tanα = andChoice
tanβ =(1) 3BD500  x AB
BD
A
50 5
AE 3 3016. tanD = and tanE =
h h
9
D
4 BD = Ÿ

BD 500 3 1
AB

=
( 500 ) m/s 7.5
3 −
7.5 1
In ADE, tanα = ; = ; AD 9 4 h h speed =Required45°
RequiredBD speed
m/s =60°
tanα.tanβ = . 2045° 60° 20
AD 4 AD h h 9 h4 Sol/869 7.5
= 40 m
tanD.tanE =
Choice (2)
.
9 4 D E = 25
3 1 m/s. (
D x C
25 x3 −C
= D 1 Choice
m/s.
45° )
(2) B
B
Sol/869
60° Choice (2)
h2 A C B D x C Sol/869
B
1 = (จ D, E are 30 30
36 20. In 'ABC, tan 60° = Ÿ BC = = 10 3
complementary) BC 3
h = 6 m. Choice (4) In 'ABD, tan 45°
A
AB
08_Section.indb 83 = Ÿ AB = 10/30/2009 11:53:20 AM
BD
Let, AD = BE = x 30° 22. Let the height of the B
45°
B m; In 'DEC, tan tree be AB + BC = 3 h  450 = h Ÿ h = 225( 3  1) m
C B x E
DE
0 30 m. Choice (2)
30 = Let AB = x m; xm
CE
BC = (30  x) m 30o Exercise  21
1 500
(1) Ÿ
3 500  x
Ÿ x = 500 3 1 In ෙ ABC, sin30q = C A (Inequalities)
AB 1 x
500 3Manual
1
Ÿ Solution for questions 1 to 25:
84  n  Solution BC 2 30  x
Required speed = m/s Ÿ x = 10 m Choice (3) 1. 5x + 9 > 3x + 19 Ÿ 5x – 3x > 19 – 9 Ÿ 2x > 10
20
Ÿx> 5. Interval notation is (5, f). have
Choice
no(3)
E = 25 3 1 m/s.
20. In ∆ABC, tan 60° =
30 (2)
Choice
⇒ BC =
30 23. Let CD be the distance covered
seconds.In ∆ABC, tan60o =
ABby the car in 10
⇒ 3=
150
2.
Hence the given
13x +lution. Choice
inequalities
4 < 5x + 20; 12x + 5 > 9x + 14
so-
(4)
B BC 3 A BC BC 13x – 5x < 20 – 4 Ÿ 8x < 16 Ÿ x < 2 o (1)
= 10tan360° = 30 30 ⇒ 50 3 m 12x – 9x > 14 – 5 Ÿ 3x > 9 Ÿ x > 3 o (2)
= 10 3 (xof
− 6)
(1)(x − 2)
20. In 'ABC, Ÿ BC =
BC 3 The
3. intersection
f (x) = and (2) that
< 0 is, (x <multiply
. We 2) ˆ (x >
(4) AB 1 150 3) is empty. (2x − 1) (x + 4)
In 'ABD, tan 45° 150 m
A In ∆ABD, tan30° = ⇒ = Hence the given inequalities have no solution.
AB 60° BD
30 3 BD and divide the expression, with Choice (2x –(4)1)
= Ÿ AB = 6 3 q
BD ⇒ BD
B = 150
C 3 D (x + 4)
BD (x  6) (x  2)
3. f (x) (x − 6)(x − 2)(2x − 0 . We multiply and
7.5 Now, CD
In 'ABC, tan60 =
o BD − BC =150
=AB
Ÿ 3=
100 3 m
Ÿ 50 3 m
(2x  1) (x  4) 1)(x + 4) < 0
2 2
45° 60° BC BC distance (2x −1) (xwith
divide the expression, + 4)(2x – 1) (x + 4)
∴ The speed
AB of 1the 150
car = = (x ⇒
D x C B In 'ABD, tan30q = Ÿ = time 6) (x
(x–2)6)(2x
(x–1)2)
(x (2x
 4) – 1) (x + 4) < 0 the
 0 Ÿ (x – 6) (x – 2)
2 2
BD 3 BD (2x  1) (x  4)
denominator is positive.
100 3
AB Ÿ BD = 150 3 = 10 3 m/sec = 62.35 kmph (2x – 1) (x + 4) < 0 the denominator is positive.
In ∆ABD, tan 45° =
BD
⇒ AB = BD  10 Choice (1) + | – | + | – | +
Now, CD = BD  BC = 100 3 mXY  –4 b 1/2 c 2 d 6 e
AB = BC + CD ⇒ 10 3 + x = 30 24. In ∆BXY, tan 45° =
distance
BY = 100 3 = as weas want the the
we want expression to take
expression a “negative
to take a “neg-
? The speed of the car = sign”, the solution set is the union of sections (b)
50
⇒ x = 10 3 3 −1 m Choice (2)( ) ⇒1=
10 3 m/sec BY
time
⇒ BY = 50 m
10 ative
and (d).
The sections
sign”, the solution
(b) andis(d).
interval notation
set is the
(–4, 1/2) ‰ (2, 6)
union of

AB = BC + CD Ÿ 10 3  x 30 XY X
The interval notation is (–4, 1/2) ∪ (2,(2)
6)
21. ∴CB = BA = x (say) (∵∠ACB = 45°)tan 45q == 62.35 kmph X Choice
BY 4. As (x + 1)2 is always positive the Choice
given(2)in
50 Choice equation becomes (x + 4)(x + 6) > 0
BC
Ÿ x = 10 3 ∆ BCF, tan60°Choice
3 1In m =
BF
(2)
Ÿ 1 = (1) Ÿ BY = 50 m
BY
24. In ෙBXY, 50 m
50 m 4. As (x + 1)2 is always positive the given
x  [–6, –4] จ (x –D) (x – E) > 0. Ÿ x  [D , E]
in equation becomes
where D < E Ÿ R – [–6, –4] Choice (4)
A In 'AXY, 30o 45
o
(x + 4)(x + 6) > 0
21. ?CB = BA = x (say) A 30 45 B
XY 1 A 50 Y B x ∉ [–6, –4] ∵ (x –α) (x – β) > 0. ⇒ x ∉
(จ‘ACB = 45°) tan 30q = Ÿ = Y = 50
Ÿ AY 3m Sol/870
x h AY In ∆AXY,
3 AY [α , β] where α < β ⇒ R – [–6, –4]
BC
In ' BCF, tan60q = ? AB = AY + BY = 50( 3 + 1) m Choice (1)  Choice (4)
BF 45°
AB = BC + CD Ÿ 10 3  x C 3060° B XY XY 1 50 B (x + 1) (x + 3)
10 tan 45q =
x AB = BC + CD Ÿ 10 3  xD 30
E 25. In 'ABD, BYtan 30° =XY ⇒ = ⇒ AY = 5. < 0 ; x ≠ –1
3 tan 45q = AY 3 AY (x + 1) (x + 2)
50 Ÿ AB BY
x  20
B1 h tan45°
Ÿ 1 == Ÿ50 AD 3 50 mm
= (x  1) (x  3)
Ÿ x = 10 3 3 1 m
Choice (2) AD
BY
BY =
50 5. x + 3  0 ; x z –1 (x + 3) (x + 2) ? The solutio
Ÿ 20 + x = 3 Ÿx x = 10 3 3 1 m
Choiceh(2)
Ÿ1=
∴ ABBY = AY Ÿ BY = 50 m
+ BY = 50( 3 + h1) m (x  1) ⇒(x  2) < 0 ⇒ <0
Ÿ x = 10( 3  1)
21. ?CB = BA = x (say) A InIn'ACB,
'AXY,  Choice (1) x  3 x + 2 (x  3) (x  (x2)+ 2)²
F AB In ' AXY, 45q 60q Ÿ
(x + 0 Ÿ
2)² being a perfect  0 is always
square (x  1) (x 
A XY 1 50 x  2 (x  2)² 15.
(จ‘ACB =21. 45°)?CB = BA = x (say) tan 30=q =25. InŸ∆ABD,
tan60° = D Ÿ AY = 50 3 m A
x AC AYtan 30q3= AY XY 150 3 m = 1 C 50 greater than zero. (x  3) (x 
(จ‘ACB = 45°) x h
AB Ÿ Ÿ AY = 50 3 m (x + 2)² being a perfect square is always greater
AC 10 q2(= BC
2 x =tan60
In =' BCF, 3  1) 3 = x + 20Choice (1) x h AB tan45° = AY ⇒ AD 3 = AY
h ⇒
than zero. (x + 3) (x + 2) < 0 (x  1) (x
BF tan60q = 45° BC Ÿ? AB 3 = AY + BY = 50( AD 3 + 1) m Choice (1) –3
In ' BCF, C AC ? AB = AY + BY = 50( 3 + 1) m Ÿ (x + 3) (x + 2) < 0
BF B Choice (1) (x  3)
22. Let the height of the ⇒ 20 + x 10 = 360° x BDC 45° B In 'ABD, In ∆ACB, B +ve ve +ve
x E 60° 25. +ve
tree3 be AB + BC = 10 B –3 –2
x D 3 h  45025. AB= hIn Ÿ'hABD, = 225( AB 3  1) m
30 m. x  20 3 ⇒ x = 10( 3 + 1) E
B1 h tan45° = Ÿ AD==
tan60°
AD tan45° = AB Choice (2) ?x (∴x 3, –2) ∈(−3, –2) Choice
Choice (3)(3)
? x  (–3, 1]
Let AB = x m; x  20 AC Ÿ AD =
Ÿ 20 + x = 3 x o
xm B1 hh AD h 2 2
BC = (30  x) m AC = 2 30
x = 10 2( 3 + 1)  Choice (1) 6. (x 6.
– 2)(x+–16 2) ++
2 3< 16 0;+(x3 –<2) 0; +
(x 19
– < 0 + 19 < 0
2) 2
= 3x Exercise AB
 21 16. 8 – 4x = r4 Ÿ
InŸෙx ABC, 3 Ÿ1)q20= + x C
= 10( sin30 AF
In 'ACB, h
⇒ 3=
(Inequalities) ? For h no value of ‘x’ the quadratic expression
In ' ACB, AC 
∴2 For no value of ‘x’ the quadratic ex- or 4x = 12 Ÿ
22. = 10(
Ÿ x Let the3 height
 1) of the tree be AB + BC AB 45q 60 q (x – 2) + 19 is less than zero.
x{1, 3}
AB 1 x F tan60° = ABD 45q A 60q pression
? Solution set =(x { – }2) + 19 is less Choice
2
than zero.
(1)
Ÿ = 30 m. Solution for questions AC tan60° 1 to= 25: 150 3 m C
BC 2 30  x 3 h − 450
AC = h ⇒ h = 225(
D 3 + 1) m A ∴ Solution set = { } Choice (1) 17. |x| = 2 or |x| =
AC = 2 x = 10 Let 2( 3AB  1)= x m; Choice (1) AB 150 3 m C 2
7. x + 5x – 24 < 0 Ÿ (x + 8) (x – 3) < 0
Ÿ x = 10 m AC =BC 2=x(30 = 10− x)2(m3Choice
 1) (3) 1. 5x
Choice Ÿ+ 9 3> 3x + 19 Ÿ 5x – 3x > 19 – 9 Ÿ 2x > 10 Choice (2)
(1) ? x{–3, –2,
AC Ÿnotation AB x 7. x
(–8,
2
+
3) 5x – 24 < 0 ⇒ (x + 8) (x – 3) < 0
Ÿ x > 5. Interval 3 is (5, f). Choice (3) ? Number of
23.
22.Let
LetCDthebeheight
the distance
of the covered by the carBin 10 AC x ∈ (–8,
? Solution set {x3): 8 < x < 3}. Choice (4)
seconds.
tree be AB 22. + BC = 2. B 13x + 4 < 5x + 20; 12x + 5 > 9x + 14 ∴ Solution set {x : −8 < x < 3} 18. ~x+ 8~ t 0. S
A Let the height of the 3 h  450 = h Ÿ h = 225( 3  1) m
30 m. tree be AB + BC = Inequalities
13x – 5x < 20 – 4 Ÿ 8x < 16 Ÿ x < 2 o (1)
3 h  450 = h Ÿ h = 225( Choice 3 (2)1) m 8. Given .
2x  3
–1 > 0 Ÿ
2x  3  (x  2)
Choice
>0 (4)
Let AB = x m; 30 m. xm 12x – 9x > 14 – 5 Ÿ 3x > 9 Ÿ x > 3 o (2) x2 x2
Choice (2) x 4
BC = (30  x) Let m AB = x m; 30 xThe
m intersection ofExercise (1) and (2)  21 that is, (x < 2) ˆ (x >
2x + 3 –1 > 0 ⇒ 2x + 3 − (x + 2) > 0 19. –2>0
BC = (30  x)
C m 30 o 3) is empty. P ractice E xercise x
8.  1
Given x 4
In ෙ 150
ABC, m sin30 q = A (Inequalities) Exercise  21 Ÿ > 0; ?
x + (x
2 + 1) (x + 2) > 0 x + 2
AB 1 Inx ෙ ABC, sin30q = C A Hence the given inequalities have no solution.
(Inequalities) x2
(x  12)(x
60° 30 q AB 1 x Solution for questions 1 toChoice 25: (4) Ÿ
Ÿ
AB 6In ∆1ABC, 3 sin30° = ⇒ = Solution for questions 1 to 25: When x  [–2, –1] the above inequation is
x +1
BC 2 B 30  xŸ C x D BC 2 30 − x (x Solution for questions 1 to 25: not ⇒
true. > 0; ∴ (x + 1) (x + 2) > 0 x  4
Ÿ x = 10 m BC 2 30  x (x)+ 9 > 3x 1. 6) (x +2)9 > 3x + 19 ⇒ 5x – 3x > 19 – 9
5x
⇒ x = 10 m Choice (3) 3.1. f 5x
Choice (3) + 19 Ÿ 5x – 03x. > We 19 – 9 multiply
Ÿ 2x > 10 and Hence thex +number 2 of integral values of x the Ÿ x  (–12 ,
AB 150 (x+2x94)> 10 +is 19
o
In 'ABC, tan60Ÿ=x = 10Ÿ m 3= Ÿ 50 3 m Choice (3) Ÿ x > (2x 1. 1)⇒
5. Interval 5x notation > 3x (5,Ÿ f).5x –Choice
3x > 19(3) – 9 Ÿ 2x > 10 above inequation is not true is 2. Choice (2)
23. Let CD be the distance BC CDcovered by the car in 10
BC ∞). (3) When x ∈ [–2, –1] the above inequation 20. |2x + 3| < 5 Ÿ
23. Let be the distance covered by the divide
car the expression,Ÿ x x> 5.
⇒ >with 5.(2x
IntervalInterval1) (x +notation
– notation 4)is (5, f).is (5, Choice
seconds. 23. Let CD be the distance covered by the car 2. in13x 10 + 4 < 5x + 20; 12x + 5 > 9x + 14 9. |x + 2|is+not | x true.
– 2| = 0. ? There is no value of ‘x’ Ÿ 8 < 2x < 2
AB10 seconds.
in
Aseconds. 1 150 (x  6) (x  2) (2x  1) (x  4) Choice (3)
In 'ABD, tan30q = Ÿ = 13x – 5x <2.20 13x
–4+ Ÿ4 8x< 5x< 16 +20;Ÿ0Ÿ x12x
<(x2– +o5 >(x9x– +2)14
6)(1) satisfies the above equation Choice (4) ? Required in
BD A
3 BD (2x
12x – 9x  1) 2
(x  4) 2 H
 ence the number of integral values of
>2. 14 13x13x
–5– Ÿ+5x 43x<<>5x20 9Ÿ –
+ 420;
xŸ>3 8xo<(2)
12x +165 Ÿ > 9xx <+2 14o (1)
The intersection
(x + 4) 12x < 0ofthe
–(1) and
> 14(2)– that is,
3x(x > <9 2) ˆ (x> 3>o (2) 10. We x
know the above
that when inequation
c > 0; a < is
b not
then true
ac < is 2.
bc. 21. x = 2 or x = 1
Ÿ BD = 150 3 (2x – 1)
13x –9x5x < 20 5–Ÿ4 is
denominator
8xŸ
⇒positive. < x16 ⇒x<2 Statement (1) is true. x = 2 satisfies
150 m
3) is +empty. –The intersection
+ –of (1) and+ (2) that is, (x < 2) ˆ (x >  Choice (2)
|
Hence the given →is(1)
| | |
inequalities have no solution. a b ? Solution se
a –4 b3) 1/2 empty. c 2 d 6 e !val-.
Now, CD = BD  BC 60°=150100m 3 30 mq 12x
Hence – 9x the> 14given – 5 ⇒ 3x > 9
Choice
inequalities ⇒ x
(4)
have> 3 →
no (2)
solution. We9. know|x + that2| + when| x – 2| c < 0; ∴
= 0. a <There
b then is no
6 3 c c 22. |3x – 2| d 6 Ÿ
60° D 100 30 ue of ‘x’ (2) satisfies
is false. the above equation
distance 33q The Choice
is, (4) ?Statement
intersection
2)is the unionof
B C 6 = as we want the expression to take (1) and (2) that
a “negative Choice (1)
? The speed of the car =B = D sign”, the
(x  6)
solution
(xset of sections (b) 4
C
time150 10 3. f (x) (x (x
< 2) ∩ 
(x(x 0
 6) .
> 3) We multiply
(x is2)empty. and  Choice (4) Ÿ dxd
o AB and (d). (2x3.  1)
f (x)  4)  0 . We multiply 11.
and 6x² + 5x – 6 t 0 Ÿ (3x – 2) (2x + 3) t 0 3
In 'ABC, tan60 = Ÿ 3 = AB Ÿ 50 3 m
10 3 m/sec
In 'ABC,BC tan60oX= BC Ÿ 3 = 150 Ÿ 50 The minterval
3divide notation is (2x
the expression, with1/2)
(–4, 1) (x–‰1)(2,
(2x 4)(x6)+ 4) § 3 2 ·
= 62.35 kmph Choice x ¨ , ¸ Ÿ x  R – (–3/2, 2/3) Choice (2) 23. (x – 3) (x – 2)
AB 1 BC
150 BC (x  6) (x  2)divide (2x 1)the (x expression,
 4) with (2x(2) – 1) (x + 4)
In 'ABD, tan30q = Ÿ = AB 2  0 Ÿ (x – 6) (x – 2) © 2 3¹ ? x  R – (2,
Choice In 'ABD,BD tan303q = BD Ÿ 1 = 150 4. As (x + (2x1) 1) 2is(x 6)
(xalways 4)(x 2) (2x 1)the
2  positive (x given
4) in
 0 Ÿ (x – 6) (x – 2)
BD equation becomes (x
(2x + 4)(x
1) 2 + 6) >2 0
(x  4) 24. Clearly ‘y’ is e
(1)
Ÿ BD = 150 3 3 BD (2x – 1) (x + 4) < 0 the denominator is positive. (x  3) (x  2)
08_Section.indb 84
50 m x  [–6, –4] จ (2x (x ––D1) ) (x(x–+ E4)) > 0.the x  [D , E] is positive.
Ÿ denominator 12. < 0 Ÿ (x + 2) (x – 1) (x – 2) 10/30/2009 (x – 3) < 11:53:23 ‘x’
AM
and meets
24. In ෙBXY, Ÿ BD = 150 3 + | – | + | < 0
– | + (x  2) (x  1) ? The curv
~x+~8x+
18. 18. ~ t8~0.tSo
0. ~
So
x +~x8~+ >8~–4> for
–4 all
forxall
 xR. R.
2x  3 2x  3 2x  3  (x  2)
2x  3  (x  2) Choice
Choice (2) (2)
8. 8. Given
Given x –1
–1 > 0 Ÿ
2 >0Ÿ x2 > 0
>0
x2 x2 x 4  x 12
x 4  x 12
x 1 x 1 19. 19. x – 42 –> 20 >Ÿ0 Ÿ x  4> 0 > 0
Ÿ Ÿ x >20;>?0;(x?+(x1)+(x1)+(x2)+>2)0 > 0 x4 x4
x2 (x  12)(x  4)
(x  12)(x  4)  0 Ÿ (x + 12)(x + 4) < 0
When  [–2,
x x [–2, –1] –1]
the the above inequation
is is Ÿ Ÿ 2  0 Ÿ (x + 12)(x + 4) < 0
x  x4  4
When above inequation 2
not not
true.true. Solutions for Mathematical Skills  n  85
Hence the number of integral values of x the the
Hence the number of integral values of x  x(–12
Ÿ xŸ  (–12 , –4),
, –4), Choice
Choice (4) (4)
above
above inequation
inequation is not
is not truetrue
is 2.is 2. Choice
Choice
(2) (2)
20. |2x + 3| < 5 Ÿ 5 < 2x + 3 < 5
10. We know that when c > 0; a < b then ac 21. |2xxŸ
20. +=3|2 <or5xŸ= 1.5 The < 2xnatural
+ 3 < 5numbers are x = 5. A word that can be read from left to right
|x +|x2|+ +2|| +
9. 9. | x2|– =2|0.= ?
0. ? There is value
no value of ‘x’ Ÿ 8 <82x< <2x2 <Ÿ2Ÿ 4 <4x < x < 1,
< bc.xStatement
satisfies

(1)There is no
is true. of ‘x’
1? and x = 2 satisfies x:<(–4,
|x1,− 1)
3| = 2. or right to left remains the same is known
satisfies the the
aboveabove equation
equation Choice
Choice (4) (4) Required interval
? Required interval : (–4, 1) Choice
Choice (1) (1)
aac <b bc. ∴ Solution set is {1, 2}. Choice (3) as palindrome.
We We
10. 10. Wknow
know thatthat
e know when
that
when c > c0;c><a0;0;
when <aab<
<then
bb then
then
ac < >bc. 21. x = 2 or x = 1. The natural
21. x = 2 or x = 1. The natural numbers are x = 1 and numbers are x = 1 and
   In a five letter palindrome only first
Statement
Statement (1)true.
(1) is is true. c c 22. x =|3x
x =– 2| ≤ 6x⇒
2 satisfies
2 satisfies |x –6
x 3| ≤= 3x
|x  2. – 2 ≤ 6; –4 ≤ 3x ≤
3| = 2.
? Solution three letters are different and the other
. ∴Statement (2) iscfalse.  bChoice
a b(1) a b ? 8Solution set set
is {1,is 2}.
{1, 2}. Choice
Choice (3) (3)
We We know
know thatthat when
when c < 0;< a0; <a b< then
then ! c !. c . −4 8 two letters are same as first two letters
c ≥ c0 ⇒– 2|–d2|6≤dŸx6–6
|3x |3x
22. 22.
≤Ÿ d–63x Choice
d 3x – 2 d 6; –4 d 3x d(2) 8
11. 6x² + 5x – 6 ≥ 0 ⇒ (3x – 2) (2x + 3) 3 3 – 2 d 6; –4 d 3x d 8 in the respective places. The first three
?Statement
?Statement (2)false.
(2) is is false. Choice
Choice (1) (1)  4 8
23. Ÿ(xŸ 4– 3) (xd –x8d2) ≥ 0 ⇒ x ∉ (2, 3) Choice (2) blanks can be filled in 6 × 6 × 6 ways
−3 2
5x6 –t 60 tŸ,0(3x
Ÿ (3x
d3 x d Choice (2)
x+ –∉
6x² 6x²
11. 11. + 5x ⇒ 2)∈(2x
– 2)–x(2x +R –3)0(–3/2,
3)+t t0 2/3) ∴3x ∈ R – 3(2, 33). Choice (3) ∴ Total number of palindromes possible
 2 3
§ 3§ 32 · 2 ·¸ Ÿ x  R – (–3/2, 2/3) Choice (2) 23. –(x3)–(x3)–(x‘y’
2)–t2) 0 tŸ 0xŸ  x(2, (2,
 3) 3) is 216. Choice (4)
x  x¨ ¨ ,2 ¸, Ÿ Choice
3 x  R – (–3/2, 2/3) Choice (2)(2) 23.
24. (x Clearly is entirely positive for any
© 2© 3 ¹ ¹ x? xRof
? value –R(2,– 3).
(2, 3). Choice
Choice (3) (3)
6. Each letter can be posted in anyone of the
(x − 3)(x + 2)
12. (x  3) (x  2) < 0 ⇒ (x + 2) (x – 1) (x – 2) 24. 24. ‘x’
Clearly and
Clearly‘y’ meets x-axis
is‘y’entirely
is entirely the point
positive
positive for(–1,
for any any 0)
value
value of of five boxes in 5 ways. Hence, 8 letters can
(x  3) (x 
12. − 2)
2)(x −1)
< 0 Ÿ (x + 2) (x – 1) (x – 2) (x – 3) < ‘x’The
and meets x-axis the point
(–1,(–1,
0) 0) by y
12. < 0 Ÿ (x + 2) (x – 1) (x – 2) (x – 3) < ‘x’∴and meetscurve can
x-axis be
thebestpoint represented be posted in 58 ways. Choice (2)
(x (x (x2)(x
2)(x  1)
– 3)1)< 0. ? =? The
|x +
The curve curve
1| Choice can canbe be
bestbest represented
represented (3) by by
0. 0. y = y|x=+|x1|+ 1| Choice
Choice (3) (3) 7. The word ‘NEIGHBOUR’ contain 5 con-
(1) (1)(2) (2)(3) (3)(4) (4) (5) (5) 25.
25. Clearly
Clearly yy is
is negative
negative and xxcan
and cantake
takeboth
bothpositive sonants and 4 vowels. 3 consonants and 2
x x— — x x — — x x 25. Clearly y is negative and x can take both positive
andpositive
and and
negative
negative negative
values. values.
values. vowels can be selected in 5C3 . 4C2 ways
f f2 2 1 1 2 2 3 3f f Y Y = 6 × 10 = 60. These 5 letters can be ar-
? ∴ Solution
Solution is(2)
is region region
(2) and(2)region
and(4)region
(4) (4) ranged among them selves in 5! that is,
? Solution is region and region O O
? Solution
set set
∴ Solution
? Solution is
set(–2,
is (–2, 1)(2, (2,∪
‰ 1)
is ‰(–2,
1) 3) 3)(2,Choice
3) Choice
(4) (4) X X 120 ways
 Choice (4) ∴ The total number of required words is
We We
13. 13. know
know thatthat
||x| ||x| – |y||
– |y|| d |xd –|xy|– isy|always
is always
true.true. 60 × 120 = 7200
13. We know that ||x| – |y|| ≤ |x – Choice Choice
y| is (2) (2)
always
Ÿ⇒ yŸ =yy–|x|.
== –|x|.
–|x|.
ChoiceChoice
Choice(3)(3) (3)  Choice (4)
|3x |3x
14. 14.
true.  Ÿ –3x2 –> 24 >or4(3x
– 2|–>2|4 >Ÿ43x or (3x Choice (2)
– 2)–<2)–4< –4
Exercise
22 22 8. Out of the 11 letters A, I and N appear
14. x >|3x – 2|2>42⇒ 3x – 2 > 4 or (3x – 2) < –4 Exercise
x > 2 or2xor< x < 3 (Permutations
(Permutations andand Combinations)
Combinations) twice each. Required total number of ar-
−2 10!
x > 2 or3 x < Permutations and rangements is Choice (3)
3 2! 2!
−2 Combinations Sol/871 Sol/871
9. Consider 3 blanks . _ _ _
∴ The solution set is (–∞, )
3 Case 1: The number contain exactly
∪ (2, ∞). Choice (4) Practice Exercise one‘1’
This ‘1’ can be placed in any of the 3
(x − 1) (x − 2) Solutions for questions 1 to 25:
15.  2
≤ 0 ; x ≠) 2‰ (2, f). blanks and the remaining 2 blanks can
? The (xsolution
+ 3) (x set− 2)is (–f, 1. The number of arrangements that can be be filled with 9 numbers each. The to-
3
Choice (4) made from a group of n objects taken r at tal number of number of ways that 1
(x − 1)
0 (x ⇒1) (x  2) ≤ 0 x ≠ –3 (x  1) a time is nPr exactly occurs only one time is 3 × 9
15. ( x + 3) d 0 ; x z 2 Ÿ d0 xz ∴ Required number of three letter words × 9 = 243
always greater
(x  3) (x  2) x  3
(x − 1) (x + 3) = 7P3 Choice (3) Case 2: The number contain exactly two
(x  1) (x  3) ≤ 0 (x – 1 ) (x + 3) ≤ 0
–3 (x + 3)² d 0 (x – 1 ) (x + 3) d 0 2. There are 3 consonants and 4 even places ‘1’s. Two blanks can be filled with ‘1’ in
(x  3)²
+ve and the number of ways of arranging them
3
C2 ways. Third blank can be filled in 9
+ve ve +ve
is 4
P . There are 5 vowels and 5 places. ways.
–3 –2 3
Choice (3) Number of ways of arranging them is ∴ The total numbers of times that ‘1’ oc-
? x  (–3, 1] Choice (3)
∴ x ∈ (–3, 1] Choice (3) cur twice is = 2 × 3 × 9 = 54
<0 5!. Hence, total ways are 4P3 x 5! = 2880
16. 8 – 4x
16. 8 –=4x r4=Ÿ ±48 ⇒– 4x8 –= 4x
4 or= 84 –or4x
8 –= 4x
–4 =
Ÿ–44x⇒=4 All the three digits contain 1 in one way.
ratic expression
or 4x = 12 Ÿ x = 1 or x = 3
 Choice (4)
4x = 4 or 4x = 12 ⇒ x = 1 or x = 3 3. Consider 7 blanks _ _ _ _ _ _ _ ∴ The required number of times that 1
x{1, 3} Choice (4)
Choice (1)
x∈{1, 3} Choice (4) The first blank is always filled with T it can occurs = 243 + 54 + 3 = 300 Choice (3)
17. |x| = 2 or |x| = 3 Ÿ x = r 2 or x = r 3
<0 ? x
be done in one way. 10. Given digits are 0, 1, 2, 3, 4, 5, 6, 7, 8.
17. |x|{–3,
= 2–2,or 2,
|x|3}= 3 ⇒ x = ± 2 or x = ± 3    Since the word never ends with L. so
? Number of solutions = 4 Choice (3) Total number of 5 digits numbers formed
Choice (4) ∴ x∈{–3, –2, 2, 3} it can be filled by the remaining 5 letters without repetition
18. ~x+ ∴ 8~Number
t 0. So ~of x +solutions
8~ > –4 for all x  R.
= 4 Choice (3)
x  2) Choice (2) in 5 ways. Now, we have 5 letters and 5 _ _ _ _ _
>0 blanks. These can be filled in 5! ways. ↓ ↓ ↓ ↓ ↓
18.
x  x+
4 8 ≥ 0. xSo 12x + 8 > –4 for all The total number of words that can be 8 × 8 × 7 × 6 × 5 = 13440
19. –2>0Ÿ >0
x  x4 ∈ R.  x4 Choice (2) formed as required is 5 × 5! = 600  Choice (3)
(xx −12)(x
4  4) − x(x−12  Choice (4)
ve inequation is Ÿ
19. – 22> 0 ⇒  0 Ÿ + 12)(x
>0 + 4) < 0 11. If the sum of the digits of a number is di-
x + x4 4 x+ 4 4. Total number of letters are 6 of which 3 visible by 3, then the number is divisible
values of x the Ÿ x  (–12 , –4), Choice (4) are vowels and 3 are consonants. As there by 3
Choice (2) (x + 12)(x + 4)
20. |2x +⇒3| < 5 Ÿ 5 < 22x + < 3< 05 is no condition on the consonants, these ∴ The number of three digits num-
s no value of ‘x’
(x + 4 )
Ÿ 8 < 2x < 2 Ÿ 4 < x < 1, can be arranged first in 3! ways. bers formed with (1, 2, 6) and (1, 5, 6)
Choice (4) ⇒ (x + 12)(x
? Required interval+:4) < 01)
(–4, Choice (1) __ C __ C __ C __ is 2 × 3! = 12 and the number of three
b then ac < bc. 21. x = 2⇒orx x∈=(–12 1. The, –4), Choice
natural numbers are x = 1(4) and    Now there are 4 places for three vow- digit number formed with (0, 1, 2) and
x = 2|2x
20. satisfies
+ 3| <x5|x⇒  3|
−5= <2. 2x + 3 < 5 els. These can be placed in 4P3 ways (0, 1, 5) = 2 × 4 = 8
a b ? Solution set is {1, 2}.
⇒ −8 < 2x < 2 ⇒ −4 < x < 1,
Choice (3) Total ways are 3! × 4P3 = 3! × 4! ∴ Required answer = 12 + 8 = 20
b then ! .
c c 22. |3x –∴2|Required
d 6 Ÿ –6 interval
d 3x – 2 :d(–4,
6; –41)d Choice
3x d 8 (1)  Choice (3)  Choice (2)
Choice (1) 4 8
Ÿ dxd Choice (2)
3) t 0 3 3

3) Choice (2) 23. (x – 3) (x – 2) t 0 Ÿ x  (2, 3)


? x  R – (2, 3). Choice (3)
08_Section.indb
24. 85
Clearly ‘y’ is entirely positive for any value of 10/30/2009 11:53:24 AM
86  n  Solution Manual

12. 15 persons who refuse to go to upper deck 21. Man can invite one friend or two friends 21× 5
have to be sent to the lower deck and the or all friends. Hence the required probability is 26
C2
10 others who refuse to be in the lower ∴ Numbers of ways that he can invite at
21
deck are sent to the upper deck. Having least one friend = 5C1 + 5C2 + 5C3 + 5C4 = Choice (1)
taken care of these 25, of the remaining + 5C5 65
85 people 40 have to be sent to upper = 5 + 10 + 10 + 5 + 1 = 31 Choice (2) 5. Total number of outcomes = 10C5 = 252;
deck and 45 in the lower deck and the 22. Each question can be attempted in 4 ways. Number of favourable cases = 5C3 × 5C2 +
number of ways of doing this is 85!/40! Hence 10 questions can be attempted in 5
C4 × 5C1 + 5C5 = 100 + 25 + 1 = 126
45!  Choice (2) 410 ways. Choice (3) 126 1
∴ The required probability = = .
13. If 5 letters are placed in corresponding en- 23. 9 beads can be arranged in a necklace in 252 2
velopes, then the 6th letter is also placed  Choice (3)
(9 − 1)! 8!
into corresponding envelope. The number ways that is . Choice (4)
of ways = 0. Choice (1) 2 2 6. P (both are kings or both are queens) =
24. Since no two teachers are together it 4
C2 + 4 C2 6+6 2
14. Let the sides of the polygon be n. The = = Choice (1)
means there is a student present in be- 52
number of diagonals in it are n(n − 3)/2 C2 26 × 51 221
tween any two teachers first arrange 5
= 2n ⇒ n = 7. Choice (4)
students. This can be done in 4! ways. 7. The total number of four digit numbers by
15. The team can be selected in the following Now the 5 teachers can be arranged in the using 1 to 5 is 5P4. The number of even
ways 5 places in 5! ways. Hence total ways are numbers which can be formed is = 2(4P3)
4! 5!. Choice (3) = 2 × 4!.
Bowlers Batman No. of ways of 2 × 4! 2
(6) (10) selecting team
25. 1st book can be given to any of the 4 stu- ∴ Required probability = =
dents, that is, in 4 ways. Similarly 2nd, 5! 5
1 10 6
C1 . 10C10 3rd, 4th and 5th books can be given in 4,  Choice (2)
2 9 6
C2 . 10C9 4, 4 and 4 ways respectively. 8. The probability of 5 being pink and 3 being
∴ Total number of ways = 4 × 4 × 4 × 4 ×
3 8 6
C3 . 10C8 6
4 = 45 Choice (2) C5 × 8C3 16
orange is 14
=  Choice (1)
C8 143
 Total number of ways of selecting the

team is 6C1. 10C10 + 6C2. 10C9 + 6C3. 10C8 = 6 9. 5 boys can be arranged in 5! ways. E be
+ 150 + 900 = 1056  Choice (1)
Probability
the required event E be the event that the
16. Total number of ways of selecting 8 boys Practice Exercise two boys are always sit together. n( E )
and 4 girls from 15 boys and 10 girls is
15
C8 × 10C4. Solutions for questions 1 to 25:
4! 2! 2
A, B are together in the representation in = 4! 2!; P( E ) = = . ∴P(E)
1. The favourable outcomes are (1, 1), 5! 5
14
C7 × 9C3 ways. Hence, they are never to- (2, 2), (3, 3), (4, 4), (5, 5) and (6, 6).
gether in 15C8 × 10C4 – 14C7 × 9C3 ways. 2 3
Total outcomes = 36. = 1 – P( E ) = 1 – =
 Choice (3) 5 5
6 1  Choice (1)
∴ Required probability = =
17. From the given data, the remaining 5 per- 36 6 10. P (room is lighted) = P (at least one good
sons can be selected from 10 persons in  Choice (3) bulb is selected) = 1 – P (all bulbs chosen
10
C5 ways that is 252 ways. Choice (1) are bad)
2. Two adjacent letters can be selected in 25
6
18. From 4 ladies and 5 men we need to se- ways. Of these 25 ways, there are 9 ways C4 32
= 1− = Choice (4)
lect 5. Exactly 2 ladies and 3 men are se- where two adjacent letters are a vowel
12
C4 33
lected in 4C2 × 5C3 = 6 × 10 = 60 ways and a consonant. The remaining 16 ways 11. Total number of outcomes = 8C3 = 56.
 Choice (1) are the favourable ways. Hence the re- Number of favourable cases = 5C1 × 3C2 +
19. quired probability 16/25. Choice (3) 5
C2 × 3C1 + 5C3 = 15 + 30 + 10 = 55
Numbers of girls (8) Number of boys 10) 3. The total number of outcomes = 15C2 55
∴ The required probability = 
= 105; Number of favourable outcomes 56
1 7
= 10C2 = 45  Choice (4)
0 8 45 3
∴ The required probability = =
105 7 12. There are 1296 rectangles and 204
∴ The required number of ways =
 Choice (4) squares on a chess board. Hence required
8
C1 × 10C7 + 10C8 = 960 + 45 =1005 4. (i) Two letters can be picked up in 26C2 204 17
probability is = Choice (2)
 Choice (1) ways. Now, the two letters should 1296 108
20. Total number of questions are 10. Each be vowels which can happen in 5C2 13. The number of squares in a 8 × 8 chess
ways. Hence the required probabil- board = 204.
question can be handled in 3 ways – 5
either not attempt it, or attempt choice A ity is C2 = 2  Choice (1) The number of squares of the size 2 × 2 in
or choice B. Total ways are 310 of which
26
C2 65 a chess board = 49.
we remove the case of not attempting any (ii) One is a vowel and one is a conso- 49
∴ The required probability =
of the questions. Hence, required ways nant - this can happen in 5C1 × 21C1 204
are 310 − 1 Choice (4) 
ways. Choice (2)

08_Section.indb 86 10/30/2009 11:53:25 AM


Solutions for Mathematical Skills  n  87

14. To be able to get a maximum amount, one


should have selected all five rupee coins. ( ) () () ( )
21. P X ∪ Y = P X + P Y – P X ∩ Y 1 ≤ a ≤ 9 that is, 1 ≤ 9c ≤ 9 ⇒
1
≤ c ≤ 1.
9
The probability of this being
5
C4 1 ; = P (X ) + P (Y )– P (X ) P (Y ) = 0.1 +
20
= ∴ c = 1 is the only possibility. B and C
C4 969 0.2 − 0.1 × 0.2 are unique.
∴odds in favour are 1 : 968 Choice (1) = 0.28 Choice (2)
∴ Column A = 1. Column B: Let the
15. Total number of outcomes = 33C2 = 22. P(exactly one of the reaches the summit) numbers be denoted by abc. a = 2b and b
33× 32 = 2c ⇒ a = 4c
= P (A J M or A J M or A J M) 1
2 ×1 1 ≤ a ≤ 9 that is, 1 ≤ 4c ≤ 9 that is, ≤ c ≤
4
1 4 3 2 1 3 2 4 1 13 9
= 33 × 16 = × × + × × + × × = ∴ c = 1 or 2. abc has two possibilities.
For the sum to be odd, one number 3 5 4 3 5 4 3 5 4 30 4
 Choice (1) ∴ Column B = 2
should be even and other should be odd.
23. A leap year has 366 days every day re- ∴ Column A < Column B. Choice (2)
Favourable possible outcomes = 17 × 16;
peated 52 times with 2 days left. The
17 × 16
Required probability = = 17/33. two days may be {(Sun, Mon), (Mon, Tue),  5Any natural number 
33 × 16 4. ∴ Rem   = 1.
…….. (Sat, Sun)}. ∴n(s) = 7  4 
 Choice (2) The year have 53 Mondays, of the two
Column A < Column B. Choice (2)
16. The probability selecting first bag and days left one of the day must be Monday.
drawing a different colour balls from it = ∴Favourable cases = {(Sun, Mon), (Mon, 5. Let the time taken by A and B to complete
8 4
Tue)}. n(E) = 2 job X be a days and b days.
1 C1 C1 ∴P(E) = 2/7 Choice (1)
×
2 12 C2 a.b
24. The required probabilities Column A = ;
The probability of selecting second bag = P(6heads) + P(5heads) + P(4heads) a+b
and drawing a different colour balls from
6 1a +b
C 6 6 C5 6 C 4 Column B =  ;
1 7 C1 5 C1
it = × ; ∴ required probability = 26 + 26 + 26 2 2 
2 12 C2
Column B − Column A
1+ 6 +15 22 11
1 8C 4C 1 7C 5C 67 = = = Choice (2)
= × 121 1 + × 121 1 = 64 64 32
2 C2 2 C2 132 1a +b a.b
25. When n coins are tossed the probability of =  −
Choice (3) n
2 2  a + b
C
getting ‘r’ heads = n r . The probability
(a + b ) − 4ab = (a − b )
2 2
17. Let number of boys be x and the number 2
=
x
C2 7 of getting at least one tail = 1 – prob- 4 (a + b ) 4 (a + b )
of girls be x = = ⇒ x = 15; 6
(2x )
C2 29 C6 I f a = b, this is 0 that is, column A = col-
ability of getting all heads = 1 – =
∴ The number of boys = 15. 26 umn B. Otherwise, this is positive, that is,
The strength of the class is 15 + 15 = 30 1 63 column B > column A.  Choice (4)
1– = .
 Choice (2) 26 64 Choice (4)
6. Let x be 10a + b. 10a + b = 81 + a + b; 10a
18. The probability that the three balls are of + b = 81 + a + b ⇒ a = 9
same colour P(YYY or WWW or BBB) Column B: Let y be 10p + q; Column B =
5
Data Comparison q − 1; q ≤ 9
C3 + 4 C3 + 3C3 3
= 12
= Choice (3) ∴ Column B ≤ 8. ∴ Column A > Column
C3 44 Practice Exercise B. Choice (1)
19. Three members can be selected from 20 Solutions for questions 1 to 50: 7. Let the middle number be a and the com-
members in 20C3 ways. Number of ways mon difference be d where d > 0. Largest of
1. Column A: Let the roots be p and q where
of selecting one teacher, one boy and one the numbers = a + d. Smallest of the num-
p and q where p > q
girl is 6C1 × 10C1 × 4C1. Required prob- bers = a − d. a + d + a + a − d = 30 ⇒ a = 10;
p + q = 22; p > q ⇒ p + q < 2p ⇒ p > 11
6 × 10 × 4 4 Column A = a + d > 10
ability = = Choice (1) Column B: Let the roots be c and d where Column B: (a + d) (a) (a − d) = 1000 ⇒ (a
20
C3 19 c>d + d) > 10. ∴ Columns A and B cannot be
c + d, = −b, cd = 121 (The sum of the roots compared uniquely.  Choice (4)
20. Let p and q denotes the probability of suc-
is negative and product of the roots is posi-
tive, both the roots are negative.) Column 2 (a )(b )
cess and failures respectively. ∴p = 1 , 8. Column A: Average speed = ;
4 A > Column B  Choice (1) a+b
q= 3 Column B: Average speed
4 2. Let the angles of the triangle be a, b and
The probability of B winning the game c, where a > b > c. a + b + c < 3a ⇒ 180° b
= qp + q3p + q5p + …. = qp(1 + q2 + 2 (2a )  
< 3a ⇒ a < 60°.  2  = 2ab
q4 + ……. ) =
If a = 61, column A < column B. If a = 62, b b
2a + 2a +
column A = column B 2 2
3 1  Choice (4)
.
=
qp
= 4 4 = 3 Choice (1) b
If a + b = 2a + ⇒ Column A = column
1− q 2
9 7 3. Column A: Let the numbers be denoted 2
1− by abc. a = 3b and b = 3c. a = 3(3c) = 9c B;
16

08_Section.indb 87 10/30/2009 11:53:26 AM


88  n  Solution Manual

9. If there are three consecutive natural


b 10
If a + b > 2a + , column A < column B numbers, one of them would be divisible + ;
2 by 3, another will be 1 more than a num- +b
 Choice (4) ber divisible by 3 and the third will be 2 As R is not a square, ℓ > b.
more than a number divisible by 3. 10 10
∴ 10 + > 10 + = 15 per cent
 +b 2
(Any natural number divisible by 3) = 0
3

Remainder of ⇒ Column A > 15 per cent. It can be simi-


9 larly shown that column B < 15 per cent
Column A > Column B. Choice (1)
(Any natural number which is 1 more than a number divisible by 3)
3

Remainder of =1 21. Column A: x = ± 15 + 224 ; If x > 0,


9
column A = column B; If x < 0, column A
(Any natural number which is 2 more than a number divisible by 3)
3
< column B Choice (4)
Remainder of =8
9 22. Let the sides of the triangles be a, b and c
∴ Required remainder = 0 + 1 + 8 = 9 which is effectively equal to 0. Column A = Column B. where a < b < c.
 Choice (3) a + b + c = 24; Let a < b < c ⇒ a + b + c
< 3c, ∴ c > 8
10. If x = 10 then the number of even fac- Also a + b > c; ∴ a + b + c > 2c; ∴12
1 1
tors and odd factors are the same. If x = 16. If x = then column A = and column B > c; If c = 9, A = B; If c = 10, A > B.
100 then the number of even factors is 4 2
 Choice (4)
more than the number of odd factors. 1
= 23. As the polygon is regular, each interior
 Choice (4) 8 angle ≤ each exterior angle = 180° −
11. Column A: Let the number be 10a + b ⇒ A > B; If x = 4, then column A = 2 and (each interior angle)
10a + b a + b + 9a 9 column B = 8 ⇒ A < B Choice (4) ∴ Each interior angle ≤ 90°. Let the num-
= =1+ . This is
a + b a + b b 17. Column A: Let the length, breadth and ber of sides in the regular polygon be de-
 1+
a height of the cuboid be l, b and h respec- noted by N.
9
tively. Surface areas of three adjacent 180° (N − 2 )
minimum when is minimum that is, faces are ℓ b and l.h and b.h. (l.b) (l.h) Each interior angle = ;
b N
1+ (b.h) = (6) (12) (18)
a (l.b.h)2 = (6) (6) (2) (6) (3) = (62)2 = 362; 180° (N − 2 )
≤ 90° ⇒ N ≤ 4; Minimum
b ∴ l.b.h. = 36. ∴ column A = column B.  N
when is maximum that is, b is maxi-
a  Choice (3) value of N must be 3. (1) ⇒ N = 3 or 4. ∴
mum (9) and a is minimum 1, 18. If x = 1, x2 = 1. Column A = 9 and Column Column A = 2;
∴ Column A = 1.9. Column A < Column B = 1. ∴ Column A = Column B. Choice (3)
B. If x = 5 then x2 = 25, in this case column
22
 Choice (2) A = column B. Choice (4) 24. If N = 1, = 4 in this case A = B. If
12
12. Column A: Let the angle be x. Its comple-
22 + 42 + 62 + .......(2N )
2
ment = (90° − x°) R 
3
 N ≥ 2, A = .
Given that x° + 2 (90° − x°) + 3(9° − x°) 19. Column A = Rs P 1 +  ; 12 + 32 + 52 + .......(2N − 1)
2

= 180°  100 
Column B = Rs P The numerator is less than 22 + 62 + 102
⇒ x = 67.5. ⇒ Column A = Column B.
+…(2(2N − 1))2 = 22(12 + 32 + 52 + …..+
 Choice (3)  R−r R  R+r
1 +  1 +  1 +  (2N − 1)2)
 100   100   100  ∴Column A < 4; ∴ Column A < Column
1
13. Let logyx = z ⇒ logxy = 2 B. Choice (4)
z  R   R−r R+r
1 +  > 1 +  1 +  25. Let the number be represented as
2 2  100   100   100 
 1 1  1 10a + b;
A =  z +  = z2 + 2 + 2 ; B =  z −  because (100 + R)2 > (100 + R)2 − r2 ⇒
 z z  z a a a
A > B Choice (1) If = Prime, clearly must be < 10.
1 b b b
+ 4 = z2 + 2 + 2 .∴ Column A = Column 20. Suppose R has an initial length ℓ and an
z = 2, 3, 5 or 7
B. Choice (3) initial breadth of b. Initial perimeter =
a
14. If N = 1, column A = 1 and column B = 13 2(ℓ + b) If = 2, 10a + b = 21 or 42 or 63 or 84; If
20  b
= 1. In this case, column A = column B. 
Column A: New length = ℓ 1 + 
If N = 2, column A = 1 + 2 = 3 and col-  100  a
= 3, 10a + b = 31 or 62 or 93
umn B = 13 + 23 = 9. In this case, column b
 10 
A < column B.  Choice (4) = 1.2ℓ; New breadth = b 1 +  a a
 100  If = 5, 10a + b = 51; If = 7, 10a +
15. Column A: Any two digit prime number is b b
one more than or one less than a number = 1.1b; New perimeter = 2(1.2ℓ +
1.1b); Percentage increase in perimeter b = 71;
divisible by 6. ∴ Column A = 9; ∴ Column A = Col-
∴ It will leave a remainder of either 1 or 5 2 (1.2 +1.1b ) − 2 ( + b ) umn B.
when divided by 6. ∴ Column A = 2. = (100) = 10
2 ( + b )  Choice (3)
∴ Column A = Column B. Choice (3)

08_Section.indb 88 10/30/2009 11:53:27 AM


Solutions for Mathematical Skills  n  89

26. x and y are whole numbers. ∴ 2x = 1 or 2 I f it is 32, column A = column B. If it is >


or 4 or 8 or 16 or ….. 32, column A > column B. Choice (4) ( 5 + 3 )2 < ( 7 + 2 )2. ∴ B > A.
2x + 1 = 2 or 3 or 5 or 9 or 17 or.  Choice (2)
33. The vowels are a, e, i, o and u, positions of
2x + 1 = 3y ⇒ if 2x + 1 = 3, x = y = 1 and if 39. Column A: (x – 2)2 = 9 ⇒ x = 5 or – 1;
a, e, i, o and u are 1, 5, 9, 15 and 21.
2x + 1 = 9, x = 3 and y = 2. Column B: 2x + 3 = 11 ⇒ x = 4
∴Three vowels have a composite posi-
∴ x has at least two possibilities. ∴ The given two values cannot be com-
tion and one vowel has a prime position.
Column B: (x, y) = (5, 0) is the only so- pared. Choice (4)
lution. 1 3
Column A = , Column B = ;
∴ Column B = 1; ∴ Column A > Col- 5 5 −12 1
40. Column A: (35 )7 = (35 )7 = (243)7; Col-
umn B. ∴ Column A < Column B. Choice (2) 7
 Choice (1) 34. Column A: Let the number be 100 initial- umn B: 71
45
= 71 = 7, A > B Choice (1)
27. Column A − Column B =  20   20  41. Column A: The number of ways in which
ly. Final value = 100 1 +  1 + 
2 (a 2 + b 2 + c 2 ) − (ab + bc + ca )  100   100  the letters of the word CRICKET can be
2 = 144 7! 5040
arranged is = = 2520. Column
Effective percentage increase = 44 per 2! 2
1
= [(a − b)2 + (b − c)2 + (a − c)2]; a, b and cent; B: The number of ways in which the
2 ∴ A = 44 per cent. Column B: Let the letters of the word HOCKEY can be ar-
c are distinct. number be 100 initially, Final value = 100 ranged is 6! = 720.
∴ (a − b)2 + (b − c)2 + (a − c)2 > 0. ∴ A > B. Choice (1)
 30   20 
∴Column A > Column B. Choice (1) 1 −  1 −  = 56. Effective per-
 100   100  42. Column A: Let the number be 100 initially.
28. Let the length and the breadth of rect- centage decrease = 44 per cent;  20   20 
angle be ℓ and b respectively (initially). ∴ B = 44 per cent Choice (3) Its final value = 100 1+  1− 
Initial area = ℓ.b.  100   100 
Column A: Final area = ℓ.b + 8 = ℓ (b + 2) 35. Let the selling prices of the articles con- = 96 ⇒ A = 4 per cent
⇒ ℓ = 4; Column B: Final area = ℓ.b. + 8 sidered in each column be Rs 100 each. Column B: Let the number be 100 initially.
= b(ℓ + 2) ⇒ b = 4 Choice (3) Column A: C.P. = Rs(100 + x)
 30   30 
Actual loss percentage Its final value k= 100 1−  1+ 
29. The possible values of ABB are 100, 144,  100   100 
400, 900. ∴ Column A > column B. x 100x = 91 ⇒ B = 9 per cent
= (100) = per cent
 Choice (1) 100 + x 100 + x  Choice (2)
30. Column A: Let the C.P. be Rs 100 ⇒ MP = Loss percentage calculated on the selling 43. Column A: 2x + y = 9; xy is a two digit
Rs 150 number.
Let the discount percentage be x per cent x ∴ x ≤ 4.5. ∴ maximum value of x = 4;
and the profit/loss percentage be y per price = (100) = x per cent; ∴ Column
100 Column A = 4
cent. Column B: X – Y = 5; XY is a two digit
100x
 x  A= per cent + x per cent number
Selling price = 150 1 −  100 + x
 100  ⇒ y ≥ 0. ∴ X ≥ 5; Column B ≥ 5
Column B: Cost price = Rs(100 − x); Ac-  Choice (2)
 y  100x 44. Let the usual speed of Ajay be s kmph.
= 100 1 ±  ⇒ 100 = 3x + 2y or 3x tual profit percentage = per cent
 100  100 − x d
Usual time = hours
2y Profit percentage calculated on the selling s
− 2y = 100 → (1); Also x = ;
3 d
x Column A: Present time =
 2y   2y  price = (100) = x per cent; ∴ Col-  20 
(1) ⇒ 100 = 3   + 2y or 3   − 2y 100 s 1− 
 3   3   100 
= 100 100x
umn B = per cent + x per cent 5d 1 d
y = 25 or y = 100, which is not possible. 100 − x = hours. Column A = × hours.
4s 4 s
∴ y = 25; ∴ Column A = Column B 100 − x < 100 + x.
 Choice (3) ∴ Column A < Column B. Choice (2) d
Column B: Present time =
31. If y is odd, 9 ends with 9 and 4 ends with  20 
y y
36. Column A: 1123 under base 4 = 91; Col- s 1+ 
4. In this case, column A = 5. If y = 0, 9y umn B: 101111 under base 2 = 47.  100 
= 4y = 1 ∴ Column A > Column B Choice (1)
5 d 1 d
In this case, column A = 0. If y is even = hours; Column B = hours;
and positive, 9y ends with 1 and 4y ends 80 6 s 6 s
37. Column A: 80 per cent of 95 = × 95 =
with 6. 100 Column A > Column B Choice (1)
In this case, column A = 5. In any case, 25 45. The number of factors of 50 is 6. (1, 2,
Column A < Column B. Choice (2) 76; Column B: 25 per cent of 304 =
× 5, 10, 25 and 50); The number of prime
100
factors of 50 is 2. (2 and 5)
32. Let the numbers be a and b. Column A: a 304 = 76 Choice (3) Column B = 3(2) = 6, ∴A = B.Choice (3)
+ b = 8; a2 + b2 = (8 − b)2 + b2 = 2b2 − 16b
+ 64 38. Column A: 8 + 2 15 = ( 5 + 3 )2 = 46. Since the initial lengths of the squares are
= 2(b − 4)2 + 32 not known the quantities cannot be com-
(3.9)2; Column B: 9 + 2 14 =
(b − 4)2 ≥ 0; ∴ a2 + b2 ≥ 32. pared.
( 7 + 2 )2 = (4.0)2  Choice (4)

08_Section.indb 89 10/30/2009 11:53:28 AM


7
?The possible values of a + b |c – d| are D
stream (u) = km/hr
78 78 18 18 5
, , or that is the possible values are
220 4 220 4 uv
39 39 9 9 Speed of the stream = =
, , or . From the choices, it is 39 : 2. 2
90  n  Solution Manual 110 2 110 2 D D
Choice (4) 
5 7 = 2D = D km/hr
47. By BODMAS rule, Value in column (A) 4. Let AB be the height of the building. 2 u 35 D35
22D
3. Ratio of profits if A, BLet and
CDCbeisthe10,000
height of 12 pole.
u the : BC = = = km/hr
8 35 the help of the current that is
2 × 35 with
A raft is moving
= 8 + 64 – = 8 + 64 – 15,000
1 = 71 u 12 : 20,000 u AE,6=A:B:C
8
Ÿ A : B : C = 20 : 30 : 20 AB== 2CE
: 3and
: 2CD = 36 m ⇒ ED = 36 − EC A raft isDmoving with the help of the cur-
8 A’s share = C’s share= 36 at a speed of km/hr.
? = Rs
− AB5000 Choice (4)
rent that35
Value in column (B) = ×8+8–8 D
8 is at a speed of km/hr. ‘D’
The time it takes to cover 35 distance ‘D’ km
the
= 16 – 8 = 8; A > B.4. Let AB be the height of D
 the building.
Choice (1)
The time Dit takes to cover the distance
from A to B = = 35 hours
D
Choice (2)
Let CD be the height of
48. Given that x2 = a2 ⇒ x = the
± a; pole.
If x = a, A= km fromDA to B = = 35 hours
BC = AE, D
B;  35
AB = CE and CD = 36 m 35
If x = − a, then A ≠ B. Choice (4) 9. The total number of outcomes = 62.
Ÿ ED = 36  EC = 36 
Favourable cases are (1, 1), (1, 2),Choice (1, 4),(2)
(1, 6)
2 AB D
49. If x = y, y + 2 = y ⇒ y = 6; If y < 6, say E A (2, 1), (2, 3), (2, 5), (3, 2), (3, 4)
9. The total number of outcomes = 62.
3 In ෙ ABC, D (4, 1), (4, 3), (5, 2),cases
(5, 6),
Favourable are (6, 1), (1,
(1, 1), (6,2),
5) (1, 4),
3, x = 4; If y > 6, x = 6; If y > 6,AB
say 9, (1, 6) 15 5
x = 8 tanD Choice
= o (1)
(4) ? The required
(2, 1), (2, probability
3), (2, 5), (3,=2), (3,
2 4)
=
BC D 6 12
50. There are 50 odd numbers in the first 100 (4, 1), (4, 3), (5, 2), (5, 6), (6, 1), (6, 5)
DE C Choice (3)
natural numbers. In ෙ AED, tanD = B 15 5
∴ The required probability = 2 =
There are 50 even numbers (including AEIn ∆ ABC, 10. Let the mark scored by A be 100. 6 12
zero) in the first 100 whole numbers. 36  AB
AB Then, B's
 marks = 110 and C's marks = 99
Choice (3)
∴ A = B.  Ÿ tanD = (3)
Choice (2) =
o tanα → (1) C's marks is 1 per cent less than A's marks
BC BC 10. Let the
Choice (3)mark scored by A be 100.
ABIn ∆36  AB DE tanα = 36 − AB
AED, tanα = AB ⇒
Then, B’s marks = 110 and C’s marks =
PRACTICE TESTS From (1) and (2), BC→ (2) BC Ÿ AE = 36 
BC
11. Volume 99
of water raised = volume of the spheres
C’s
2 marks 4is 1 per 3cent less than A’s
AB Ÿ AB = 18 m Choice h = (60) S (3.5) Ÿ h =Choice
Ÿ S(14)marks 17.5 cm
(3)
(1) AB 36 − AB 3
TEST PAPER 1 From (1) and (2), = ⇒ AB
BC BC ? The
11. water
Volumelevelof water raised= =23
initially volume
4
 17.5 of = the5.5 cm
1. C.P = Rs 540, MP =5.540Let + 15there be of
10 parrots =and
36 −18 cows.
⇒ ABIf =three
18 m cows spheres ⇒ π(14)2h = (60) π (3.5) Choice ⇒ (2)
3
per cent AB Choice (1)
escape, the ratio of parrots to cows is 2 : 3. Let h = 17.5 cm 3
540 = Rs 621
5. Let there be 10 parrots and 18 cows. ∴ The water level initially = 23 − 17.5 =
S.P = Rs 496.80, Discount = 621 – 496.80
If three cows escape, the ratio of par-
= 124.2
rots to cows is 2 : 3. Let there be 11
5.5 cm Choice (2) Sol/877
Discount per cent = 124.2/621 × 100 = 20
parrots and 17 cows. If 1 parrot and 12. Given P = 42 + 42 + 42 + . . . . ∞
per cent Choice (3)
2 cows escape, the ratio of parrots to cows
2. Let the HCF of a and b be H then HCF of c is again 2 : 3. But from this data alone ⇒ P = 42 + P
and d = H (number of birds escaping and ratio of the
On squaring both sides we get P2 =
a b c d birds) we can’t uniquely determine the
given that = 77 and = 221 42 + P
H H H H initial ratio of the birds. (4) Given P = 2 42  42  422  . . . . f
Choice12. 19. Pa
⇒ P – P – 42 = 0 ⇒ P – 7P + 6P – 42
The possible values of 6. Each letter can be posted in 4 ways Ÿ P = = 42 0 P
a b a+b c d ⇒ P(Pboth– 7) sides
+ 6(P –we 7)get
= 0 P⇒2 (P + 6) (P Pa
, , and , and the ∴ The five letters can be posted in 4 × 4 × On squaring = 42 + P
H H H H H 4 × 4 × 4 is 45 ways Choice (3) Ÿ P2 – P – 7) = 0 2
– 42 = 0 Ÿ P – 7P + 6P – 42 = 0
⇒ P = 7 or –6 Pa
c d  |c−d|
difference and  i.e. 7. Let the length of the parallel sides of the Ÿ P(P –But 7) + 6(P – 7) = 0 Ÿ (P + 6) (P – 7) = 0
 are the value of the given expression can
H H H  trapezium be 5x and 9x cm respectively. Ÿ P = 7never or –6
be negative
But the value of the given expression can never be Ti
tabulated below. Given distance between parallel sides = ∴ P = 7. Choice (2)
13 cm, negative
a b a+b c d |c−d| =
∴ Area = 273 sq. cm ? P13.
= 7.
Let AB be the tower and C be the Choice (2)
position
H H H H H H 13. Let AB ofbeman.
the tower and C be the position of
1 77 78 1 221 220 1 co
⇒ (13) (5x + 9x) = 273 ⇒ x = 3. man. A
7 11 18 13 17 4 2 20. Le
∴ The lengths of the parallel sides are 15 Q
∴ The possible values of a + b |c – d| are Q
cm and 27 cm. Choice (3) h
78 78 18 18 =
, , or that is the possible 8. Let the distance be ‘D’ km.
220 4 220 4 45° 1
B C
values are 20 m
D
39 39 9 9 Speed up stream (v) = km/hr; Speed
, , or . From the choices, it is 7 In 'ABC, In tan45°
∆ABC, tan45°
= h/20= h/20 21. Li
110 2 110 2 h/20 = 1h/20or h= =1 20
or hm= 20 m ChoiceChoice
(3) (3)
D Af
39 : 2. Choice (4) down stream (u) = km/hr
5 14. By observation,
14. By observation,
the sum the sum 0
of 2 of
1
, 22 ,, 2
0 12
2 ,is2 a
2
is multiple
a pe
3. Ratio of profits if A, B and C is 10,000 × of 7. multiple of 7.
12 : 15,000 × 12 : 20,000 × 6 = A : B : C D D Similarly, the sum of the next 33 powers, Se
− Similarly, the sum of the next powers, and
u−v
⇒ A : B : C = 20 : 30 : 20 = 2 : 3 : 2 Speed of the stream = = 5 7 every one and every
of the one17 of the 17 sets
sets of threesuccessive
of three suc-
∴ A’s share = C’s share = Rs 5000 2 2 powers cessive powersof
is a multiple is 7.
a multiple of 7. R
 Choice (4) 3
[In 1 + 2 + 2² + 2 + ………. + 2 , there are 51
50 Th
terms, or 17 sets of three successive powers] Am
?The number is a multiple of 7 or there is no =
remainder. Choice (1) To
3 2 3
Pr
08_Section.indb 90 15. Required number of ways = C1 u 10/30/2009
C1 + C11:53:29
2 = 9 AM
Solutions for Mathematical Skills  n  91

[In 1 + 2 + 2² + 23 + ………. + 250, there 18 − 0.9x 20


(100) = 80 ⇒ x =  1 1
are 51 terms, or 17 sets of three succes- 20 9  +  =1
sive powers] x y
∴The number is a multiple of 7 or there is  Choice (3)  
no remainder. Choice (1) 1.2 0.8 x 2  2 3
21. Listed price of car = Rs 50,000 = ⇒ = ⇒  + 2 = 1
15. Required number of ways = 3C1 × 2C1 + After successive discounts of 20 per cent y x y 3  2 y y 
3
C2 = 9  Choice (1) and 25 per cent 3 
Selling price = 50000 × ⇒ y = 12 Choice (2)
16. Let the distance between A and B be x.
Then the distance travelled by walk = x/4
(100 − 20 )(100 − 25) = Rs 30,000 26. Let the sum be Rs 100
Remaining distance = 3x/4 100 ×100 Amount after 17 years = Rs 300; Amount
∴Distance travelled by cycle = 3x/20 This is the cost price for Suresh after 34 years = Rs 900. Amount after 51
Remaining distance = 3x/5 Amount spent on repairs = 20 per cent of years = Rs 2,700; Amount after 68 years
Distance travelled by cycle = x/5 30,000 = 6000 = Rs 8,100
2x Total cost price = Rs36,000 ∴ The amount becomes 81 times itself in
Remaining distance = ; Profit = 20 per cent 68 years. Choice (3)
5
∴ 490 = 2x/5 ⇒ x = 1225 m 120 27. The 6 men can be arranged in 6! ways.
Selling price for Suresh = × 36,000
Alternatively, if d is the total distance, 100 There are 7 gaps. The seven girls can be
3 4 2 = Rs 43,200 Choice (2) arranged in these 7 gaps in 7! ways.
then × × × d = 290 ⇒ d = 1225 m ∴ Total number of arrangements possible
4 5 3 48 12
 22. Given, + =8 = 6! 7!. Choice (2)
Choice (2) 12 − b 3
4 (12 − b)
48 16 28. Required probability = Probability (the
17. Let the length of the cloth required be l m. ⇒ + =8
12 − b 12 − b card being a spade) + probability (the
Area of the cloth = 50 ℓ sq.m
card being a king) – probability (the
Area of cloth required for the conical 64
⇒ = 8 ⇒ b = 4 kmph Choice (4) card being a spade as well as a king) =
tent. 12 − b
Curved surface area of cone = π rℓ and ℓ = 1 1 1 4
+ − = Choice (3)
23. Part of the tank filled byAand B in a period of 4 13 52 13
r2 + h2
1 1 4 2
2 hours = + = = 29. Marks obtained ∝ (Number of hours)2 that
= π (28) 282 + 962 = 2800π ⇒ 50ℓ = 10 30 30 15
M1 H12 450 9 5
2800π Part of the tank filled by A and B in a pe- is = ⇒ = ⇒ H2 =
⇒ ℓ = 56 π m Choice (3) M2 H22 625 / 2 H 2 2 2
14
riod of (2 × 7), that is, 14 hours = = 21/2 hours Choice (2)
18. Total number of games played in the tour- 15
14 1 30. (1 + x ) + (1 + x ) − x
a −1 b −1 a+b
5(4) remaining work = 1 − =
nament = 5C2 = = 10; ∴ Sum of the 15 15 1+ x b +1+ x a
1(2) = − x a +b = 1 − 1 = 0
points in all the matches = 10 1
th  (1+ x a )(1+ x b )
{For a win, one player gets one point. For Now, A can fill of the tank in
15 Choice (1)
a draw, two players get ½ a point each. So
for each match the total score of all the 1 1 2 31. Given, P( A ) : P(A) = 3 : 5
÷ hours = hours. ∴The tank will
players increases by 1} Choice (2) 15 10 3
3 5
2 P (A) = and P(A) = ⇒ P(B):
1 be filled in 14 hours. 8 8
19. Part of the job done on the first day = 3
4 P( B ) = 5 : 7
Choice (3)
Part of the job done on the second day = 5 7
24. Fresh coconuts have 12 per cent solid. P(B) = , P( B ) =
2 1 12 12
= Dry coconuts have 78 per cent solid.
4 2 Any one of them solve the problem, the
Quality of fresh coconut solid = Qual-
Part of the job to be completed = 1 – problem will be solved.
ity of solid in dry coconut. Quantity of
1 1 1 12 P(A ∪ B) = 1 – P( A ∩ B)
 + = solid in 65 kg fresh coconut = (65) =
4 2 4 100 = 1 – P( A ) . P( B )
Time in which this will be completed = 7.8 kg,
rd 3 7 25
1 3 1  100  =1– . = Choice (1)
÷ = of the third day. ∴ The job total quality of dry coconut = 7.8   8 12 32
4 4 3  78 
will be completed in 21/3 days.Choice (1) 32. Part of the job completed in 6 days =
= 10 kg. Choice (4)
 1 1 1  13
20. Let the quantity to be withdrawn be x 25. Let the periods of time taken by P and Q 6 + + =
litres.  20 30 40  20
to complete the job be x days and y days
Quantity of milk in it = 0.9x litres. respectively. 7
Quantity of milk remaining = (0.9) (20) ∴ Remaining part =
2 3 4 6 20
– 0.9x Job =  +  2. So + = 4.8
= (18 – 0.9x) litres. x y x y This would be divided among Q and R in
the ratio of their capacities, that is ratio of

08_Section.indb 91 10/30/2009 11:53:31 AM


92  n  Solution Manual

1 1 22 per cent of total expenditure Their total cost = Rs (10x + 12y + 16z)
daily work done = : =4:3 = Rs 2200
30 40 12.10
Rs 8700 Cost price of mixture per kg =
10
4 7  1 1+
∴ Q would do   = of the total Total money spent = Rs 8700 Choice (1) 100
7  20  5
38. Total consumption for 10 days = 50 × 10 = Rs 11 per kg
work. 10x + 12y + 16z = 11(x + y + z); x = y +
th
× 1.5
3 7  3 Consumption for 4 days = 50 × 4 × 1.5 5z only choice (1) satisfies this condition.
R would do  = of the total
7  20  20  Choice (1)
Consumption for 5 days = 60 (x) (5)
The required ratio =6 50 × 10 × 1.5 = 50 × 4 × 1.5 + (60 × x × 5) 5. Total number of points = 3 + 4 + 5 =12
+ 50 = 300 + 300x ⇒ 300x = 450 ∴ The number of triangles formed by
 1  1 1  1  3
 :6   + :6   + = 3 : 4: 3 x = 1.5 kg Choice (3) these points is 12C3
 20   30  5  40  20 But 3, 4 and 5 points are collinear hence
 Choice (1) 39. Let the present ages of the person and
they do not form any triangle
his wife be 6x years and 5x years. The
a c e ratio of their ages 25 years hence = ∴ The total number of triangles formed
33. From the given condition, , and = 12C3 − 3C3 − 4C3 − 5C3 = 220 − 1 − 4 − 10
b d f
6x + 25 6(x + 5) − 5 = 205 Choice (4)
1 3 5 =
are , and respectively. Only 5x + 25 5(x + 5)
2 4 6 6. Only xy z is necessarily an odd natural
choice (2) is true. Choice (2) 6 5 number. xy + z and xy – z are even, xy/z
= −
5 5(x + 5) may not be an integer. Choice (4)
34. Quantity of wine in mixture drawn from
20
∴ This ratio is always less than 6 : 5. 7. Let the present ages of A, P, Q and R be a,
P= × 5 = 1 litre Only Choice (4) satisfies this condition. x, y and z respectively.
100
Choice (4) From the given data, a = x + y + z → (1) and
Quantity of wine in mixture drawn from
40. From 0 to 100 (first hundred and one a − 11 = 2 (x − 11 + y − 11 + z − 11) − 5
30
Q= (4) = 1.2 litres. Quantity of wine whole numbers), we have a total of 12 ⇒ a = 2 (x + y + z) − 60 → (2)
100 zeroes. Choice (3) Solving (1) and (2), we get x + y + z =
40 60 → (3)
in mixture drawn from R = (2) = 0.8
100 It is also known that, y = z − 5 → (4) and
TEST PAPER 2 x = z − 10 → (5) Solving (3), (4) and (5),
litres. Total quantity of wine = 1 + 1.2 +
Solutions for questions 1 to 40: we get z = 25 and y = 20 ∴ Present age of
0.8 = 3 litres
Required concentration 1. Expenditure of Raju in the first 6 months = Q = 20 years Choice (2)
3 300 (4500) (6) = Rs 27000
= (100)% = % Choice (3) 8. Time taken to meet for the first time any-
5+4+2 11 His expenditure in the next 4 months =
Length of the track
(3500) (4) = Rs 14000. where on the track =
35. 1.030301 = 1 + 0.03 + 0.0003 + Relativespeed
His expenditure in the next 2 months =
0.000001
(4000) (2) = Rs 8000. 600
= 1 + 3(1)2 (0.01) + 3(1) (0.01)2 + 10–6 = = 24 sec. Choice (2)
= 1 + 3(1)2 (0.01) + 3(1) (0.01)2 + (10–2)3 Total expenditure of Raju = Rs 49000. 10 +15
= 1 + 3(1)2 (0.01) + 3(1) (0.01)2 + (0.01)3 = Total income of Raju = 49000 + 5000 =
(1 + 0.01)3 Rs 54000
54000 9. 32 = 16.2 = 4 2
(∴ a3 + 3a2b + 3ab2 + b3 = (a + b)3) = Average monthly income = = Rs
12 18 = 9.2 = 3 2
1.013
4500 Choice (4)
∴ 1.030301 = 1.01
3
12 = 4.3 = 2 3
For a number greater than 1, the cube root 2. Consider four blanks __ __ __ __Thou-
sands digit can be filled in 4 ways, hun- 27 = 3.9 = 3 3
is (or any other root) less than the number.
Among the choices, only 1.01 is less than dred’s digit can be filled in 5 ways, ten’s 2+ 6
digit can be filled in 5 ways and unit’s ∴
the given number. Choice (2) 32 − 18 − 12 + 27
digit can be filled in 2 ways.
36. Let the speed of the first boy = 3x kmph
∴The speed of other boy = 4x kmph
∴ Total number of our digit even num-
=
2 ( 2 + 3 )= 2 Choice (2)
bers can be formed = 4 × 5 × 5 × 2 = 200
Given (3x + 4x).5 = 210 ⇒ (7x).5 = 210 2+ 3
Choice (1)
210
⇒x= = 6 kilometres per hour 3. 3x² − 8x + 4 = 0 ⇒ 3x² − 6x − 2x + 4 = 0 10. Let the cost of the gold chain be Rs x and
35 the service charge be Rs y.
∴Speed of faster boy = 24 kilometres/ ⇒ x = 2 or 2/3
Common difference = Lesser root = 2/3 x
hour Choice (4) ⇒ Cost of gold ring = Rs .
First term = Greater root = 2 2
37. 20 per cent of amount spent on machines From the given data, x + y = 3200 → (1)
∴T10 = a + 9d = 2 + 9 × 2/3 = 8
= Rs 1300. Amount spent on machines = x 3x
Choice (1) x + y + + y = 5000 ⇒ + 2y = 5000
100 2 2
1300 × = Rs 6500 4. Let the quantities of sugar priced at Rs 10
20 ⇒ 3x + 4y = 10000 → (2) Solving (1) and
per kg, Rs 12 per kg and Rs 16 per kg be x
65 per cent of total expenditure = Rs (2), we get y = Rs 400 Choice (3)
6500 kg, y kg and z kg respectively.

08_Section.indb 92 10/30/2009 11:53:31 AM


Solutions for Mathematical Skills  n  93

11. Let the speed of the second train be x 17. In a non-leap year, everyday repeated 52 x + y + z = 22 → (1)
kmph. times with one odd day. To have 53 Fridays 10x + 20y + 100z = 1000 ⇒ x + 2y + 10z
138 + 262 400 ×18 in a year, that odd day should be Friday. = 100 → (2)
= 32 ⇒ = 32 (2) − (1) ⇒ y + 9z = 78
5 (135 − x)5 1
(135 − x) ∴ The required probability =
400 u18 18 1 7 78 − y 6−y
= 32 Ÿ = 32 ? The required probability = Choice (3) z= =8+ → (3)
⇒ 32 (135 – x) = 80 × 18 ⇒ 135 – Choice (3) 9 9
(135  x)5  7
x = 45 Since y and z must be positive integers.
18. Let the speed of the train be s m/sec.
⇒ x = 90 18. LetChoice (4) of the train
the speed ∴6–y=0⇒y=6
x) = 80 × 18 Ÿ 135 – x = 45 Let itsbe s m/sec.
length be Lm
12. IfChoice
the two(4) particular players Letare its not
length
in- be Lm In this case, y = 6, z = 8 and x = 8.
5
cluded the number of selections = 14C11. L = 10 s; L + 300
5 = 30(s – (36) ( 18 )) 6−y
L = 10 s; L + 300 = 30(s – (36) ( )) When = –1 ⇒ y = 15
articular players are notIfincluded
one alone the
is included 14C10 x 2 18s – 300 ⇒ 20 s = 600 9
10s + 300 = 30
elections = 14C11. Total selections = 14C11 + 14C 10s x 2
10 + 300 = 30 s – 300 z = 7 but x = 0
s =Ÿ30 20
∴ Ls==10s600= 300 Choice (3)
is included 14C10 x 2 Choice (1) Since Mayur have at least one note for
14 14 s = 30 ? L = 10s = 300 Choice (3)
19. Let AB represent the tower and the points
ons = C11 + C10 x13. 2 The Choice (1)
total number of outcomes = 250 each denominators. This not possible.
50 The number of favourable Let AB represent theCtower
19. cases and Dand
be the
theinitial andCfinal
points andpositions ∴ y = 6, z = x = 8. Choice (2)
mber of outcomes = 2 D be the initial and of Rohit.
final positions of Rohit.
50 = 50 C1 + C3 + …..+ C49 = 24. Let the required men be x.
50 50 50
of favourable cases = C1  C 3  …..
AB 44 men completed 55 per cent of the work
22 · n
50 n

In 2'ABC, = 249tan45° = BC
n
n n n  C1 + n n
49 C3 + ..... C r ..... =  A in 20 days.
C1  C3  ..... C r ..... 2 ¸ = 2 . 2  .
2 ¹ (44 + x) has to complete 45 per cent of the
Ÿ AB = BC = x work in (30 − 20) = 10 days
249
2 49
∴The required probability = 50 AB
ed probability = 50 Choice (2) In2'ABD, tan30° = x 44 × 20 (44 + x )10 m1 d1 m 2 d 2
BD = [ = ]
2  Choice (2) 55 45 w1 w2
14. Let the speed of Kumar beŸx kmph BD 30° 45°
d of Kumar be x kmph and the speed AB =and that is, 44 + x = 72 ⇒ x = 28 Choice (1)
the speed of Pavan be y kmph. 3 D 1 km C x B
y kmph. 25. Let v kmph be the initial speed. The 30
120 120 120 120
120 120 = +1 ; = − 2 x 1 1 km stretch was covered at a speed of (3/4)
1; 2 y x y x / 2x = or x = AB v. If it was covered at v, the time taken
y x/2 3 3  1In ∆ABC, tan45° = BC ⇒ AB = BC = x would be 10 min less.
120 120 120 120
120 120So x + 1 = x / 2 − 2 ⇒ x / 2 3− x1 = 3
=
120
x/2
2Ÿ
x/2 x
 =3 =
2
km = 500 In 3 ∆ABD,
 1 mtan30° =
AB
BD
⇒Choice
AB =
BD
3
that is,
30

3 / 4v v
30 1
= 1/6 [(10 min) in hr]
6
240 120
0 ⇒ − =3 (2) x +1 1 40/v − 30/v = 1/6 ⇒ v = 60 kmph.
=3 x x x= or x =  Choice (3)
20. The number of rectangles 3in a 8 u 83chess −1 board
120 9 9
10 1
is Choice
C2 u C (3)2 = 1296. 3 +1
Ÿ x = 40 kmph

x
= 3 ⇒ x = 40 kmph
Choice (3) The number of squares in
= ( )
km = 500 3 + 1 mChoice (2)
2 a 8 u 8 chess board is
26. Part of the job completed by A = =
30 3
15. Part of the work completed by Pavan in
8.9.17 20. The number of rectangles in a 8 × 8 chess
2 10 1
1 1 ¦8 = = 204; Part of the job completed by C ==
one hour
work completed by Pavan in one hour = = 6 board is 9C2 × 9C2 = 1296. 60 6
20 × 6 120
1 ? The probability The
that thenumber
selectedof squares in a is
rectangle 8 ×a 8 chess Part of the job completed by B = 1 –
Part of work completed by Kalyan in one
20 204 17 board is ∑8 =
8.9.17
= 204; 1 1 1
 + =
2
1 1 square = . 6Choice (1)
completed by Kalyan in hourone= hour == 1296 108 3 6 2
18 × 5 90
∴ The probability that the selected rect- 900
Work done by Pavan 21. and Kalyan
Let theinpartsone of the job completed by a man, a ∴ B’s share = = Rs 450 Choice (2)
204 17 2
1 1 7 woman and a machine day be= m, w= and . c
angleinisaa square
hour = + = 1296 108
by Pavan and Kalyan in one 120 hour 90 = 360 respectively.  1 1
Jobby = (2m + 3w + 8c) 12 = (6w + 28c)6 Choice (1) 27. CP = and SP =
7 Part of the work completed Pavan 6 4
24m + 36w +96c = 36w
21. Let +the168 c of the job completed by a
parts
360 1
and Kalyan in 5 that ism = 3c.
36
Job= = (m + c)man,
hours 45 =a 180c 1 1
woman and a machine in a day be −
work completed by Pavan and Kalyan 7 7
? A machine takes 180 m, wdaysand ctorespectively.
complete it. ⇒ Profit = 4 6 × 100 = 50%
36 7 36 17 36 1 Job = (2m + 3w + 8c) Choice (3) + 28c)6
12 = (6w 
1
hours = u = × =
6
7 360 7 10 360 7 10 2
22. x = k(y + 2) 36 = k(18)
24m + 36w +96c = 36w + 168 c
do the work in 10 days.So, Choice (4) do the workk =
they can in 10 days. m = 3c. Job = (m + c) 45 = 180c Choice (2)
2 Now, x = 2(122 ∴ + 2) Ÿ x = 2(146) = 292
A machine takes 180 days to complete
 Choice (4) 28. Volume of cylindrical block = π(5)2(8) =
y = Rs 100; A’s salary = 20 per cent it.
Choice (4)
16. Let B’s salary = Rs 100; A’s salary = 20 200π cm3
= Rs 80 23.80Let the number of RsChoice 10, Rs(3) 20 and Rs 100 notes
per cent less than B = Rs 1 2
30 per cent more than B = Rs 130 with than
Mayur Volume of cones carved = 2( πr2h) =
C’s salary = 30 per cent more B =be x, 22.
y andx =zk(y
respectively.
2
+ 2) 36 = k(18) 3 3
is less than C’s salary by x  y  z = 22 o (1)
Rs 130 k = 2 Now, x = 2(122 + 2) ⇒ x = 2
00 = 38.5 per cent 400
∴ A’s salary is less than C’s10x  20y
salary by  100z = 1000 Ÿ =x 292
(146)  2y  10z = 100 o (2)
Choice (4) π(5)2(4) = π ⇒ Wastage = 200 π –
= 50/130 (2)  (1) Ÿ y  9z = 78
× 100 = 38.5 per cent 3
0 = 40 per cent, the answer is slightly 23. Let the number of Rs 10, Rs 20 and Rs
(As
52
× 100 = 40 per cent,z the78  y 6 100
y notes with Mayur be x, y and z re- 400 200
130 = answer is 8  o (3) π = π
per cent.) 9 9spectively. 3 3
2) slightly less than 40 per cent.)Choice (2)
Since y and z must be positive integers.
p year, everyday repeated 52 times ?6–y=0Ÿy=6
d day. To have 53 Fridays in a year, In this case, y = 6, z = 8 and x = 8.
should be Friday. 6y
When = –1 Ÿ y = 15
08_Section.indb 93 9 10/30/2009 11:53:33 AM
30 30 1 31. Value of the first instalment when the last
 = 1/6 [(10 min) in hr] 2
instalment was paid = 1000 (1.1) = Rs 1210
3 / 4v v 6 Value of the second instalment when the last
40/v  30/v = 1/6 Ÿ v = 60 kmph. Choice (3) instalment was paid = 1000 (1.1) = Rs 1100
Required value = 1210  1100  1000 = Rs 3310
10 1 Choice (1)
. Part of the job completed by A = Manual
94  n  Solution
30 3 32 Let the price be Rs x initially.
10 1 Price after the first increase = (1.2) (x) = Rs 1.2x
Part of the job completed∴ Clearly
by C = the percentage of wood wasted Final price = (0.8) (1.32x) = Rs 1.056x
60 6 Price after the second increase = (1.1) (1.2x) =  R 
2
 R 
is 66.66 per cent. Choice (4) 1.056x = Rs 21.12 x = Rs 20 Choice (1) P 1 +
Rs 1.32x  − P 1 +  = 3600
29. Let ABCD be the 1 1 · 1tank and the
§rectangular  100   100 
Part of the job completed by B = 1 – ¨  ¸ 33. price
Final Marks= (0.8) (1.32x)
secured = Rs= 1.056x
by Mohit 60 per cent;
goat is tied to the© 3
corner
6 ¹A. 2The area the 1.056x Pass
= Rsmark
21.12
= 45xper
= Rscent20 Choice (1)  R  R 
P 1 +  1 + − 1 = 3600;
goat can graze is the sector MPN. Difference between the marks secured  100   100 
900 33. Marks secured by Mohit = 60 per cent; Pass mark
? B’s share = = Rs 450 Choice (2) and
= 45 per centpass mark = (60 – 45) per cent = 15
2 D C  12   1 
between the marks secured and pass P  10   5  = 3600 ∴P = Rs 15000
per cent
Difference

1 1 M mark =Given
(60 –that
45)Mohit got 45
per cent = marks
15 permore
centthan Choice (4)
. CP and SP 7m Given the
thatpass marks.got 45 marks more than the
Mohit
6 4 A 7 mN B Let the maximum marks be x.
pass marks. 37. Let the price of sugar and its consumption
1 1 7m Let the⇒ maximum marks be x. 45 × 100 be initially Rs p/kg and Rs c/kg respec-
 15 per cent of x = 45 ⇒ x = tively.
Ÿ Profit = 4 6 u 100 50% Choice (2) Ÿ 15 per
4515u 100
= 300 Its initial expenditure = (p) (c) = Rs pc.
1 P = 300cent of x = 45 Ÿ x = 15
∴ The maximum marks in the examination  100 +10 
6 Final price = (p)  
270 3 22  100 
∴ Area = × πr2 = × × 49 = ? The =maximum
300. Choice (2)
marks in the examination = 300.
= Rs 1.1p/kg
. Volume of cylindrical block = S(5)360 (8) = 200S4cm 7
2 3
34. Perimeter = (1 + 2 + 3 + -------- n) cm Choice (2)  100 − 20 
115.5 sq. m. Choice (4) Final consumption = (c)  
1 2 2 2 n(n +1)+ 2 + 3 + -------- n) cm  100 
Volume of cones 30.
carved = 2( Sof r h) = S(5) (4) 34. Perimeter= = (1 cm
Let the sides3 the square
3 and the triangle 2 = 0.8c kg
be A and E respectively. Let the radius of = n(n  1) cm Final expenditure = (1.1p) (0.8c) = Rs
400 400 200 Among the given choices, only choice (4)
= S Ÿ Wastage =the 200circle
S –be R. S = S 2 0.88pc
3 n(n +1)
3 23 3 Amongcannotthe given choices,
be expressed only
in the choice
form (4). cannot Expenditure has decreased. ∴ per cent
A2 = E = πR2; S = 4A, T = 3E and 2
? Clearly the percentage 4of wood wasted is n(n  1) 1− 0.88
66.66 per cent. be expressed
 in the form . Choice
Choice (4) (4) decrease = (100) = 12 per cent
C = 2πR 2 1
Choice (4) S2 = 16A2, T2 = 9E2 and C2 = 4π2R2 35. Let each side of the square be ‘a’ cm. 
35. Let each side of the square be ‘a’ cm. Choice (3)
. Let ABCD be the rectangular tank
3 and
2 the goat is 2 A B
S = 16 (
2
E ), T = 9E and C = 4π2 
2 2
38. (15000) (12) : (20000) (x) = 1 : 1; x = 9
tied to the corner A. The area the4 goat can graze
is the sector MPN.  Choice (2)
DA 2
C
( ) that is, S2 = 4 3 E2, T2 = 9E2 and 39. Cost of white washing = Rs 1568
π 2 ⇒ Area of four walls × cost per square
M §a·
C27=m 4π D C S ¨ ¸2 feet = Rs 1568
A 37 m2 N B Area of the shaded region = a2 – π  a  = 476
©2¹ 2h(l + b) × cost /feet2 = 1568
( E ) = π 3 E2⇒ C2 < S2 < T2
2 2[14(20 + b)] × 1.75 = 1568 ⇒ b = 12 feet
7m 4 Area of the shaded region = a2 –  
∴ C < S < T Choice (1)
2
8a  Sa 2 2  Choice (4)
Ÿ 8a 2 − πa 2
= 476
= 476 ⇒ = 476 40. Area of rectangular field = 50 × 30
P 31. Value of the first instalment when the last 8 8 = 1500 m2
instalment was paid = 1000 (1.1) = Rs 2
 22  Let the depth of the trench be d m.
1210 8− 7  8× 7 ⇒ Volume of trench = 15 × 10 × d
Value of the second instalment when the ⇒ a 2
  = 476 ⇒ a =
2
× 476 Sol/882
 8  34 = 150 m3
last instalment was paid = 1000 (1.1) =
  Area of the trench = 15 × 10 = 150 m2
Rs 1100
= (28/17) × 476 ⇒ a = 28 cm Choice (2) ∴Remaining area of the field = 1500 – 50
Required value = 1210 + 1100 + 1000 =
= 1350 m2
Rs 3310 Choice (1) 36. The difference between the compound
interest for the 3rd year and the 2nd year Volumeof trench
32. Let the price be Rs x initially. ∴ Raise in level =
is equal to the simple interest on the com- Area of field
Price after the first increase = (1.2) pound interest of the 2nd year.
(x) = Rs 1.2x 2 150d
R ⇒2 = ⇒ d = 20 m
Price after the second increase = (1.1) ∴3600 × = 720 ⇒ R = 20 per cent 9 1350
(1.2x) = Rs 1.32x p.a. 100
∴ The trench was dug by 20 m deep.
 Choice (4)

08_Section.indb 94 10/30/2009 11:53:34 AM


Solutions for Data Analysis
and Data Sufficiency 4
Data Analysis 7. Pawan 800 increased from 15 to 20 = = 111/6 = Rs 18.50 lakhs and it is closest
5/15× 100 growth = 331/3 per cent. Jindica to 2 profit in 2002  Choice (2)
there is no change. 15. As given in the solution for question
Practice Exercise 1
Pantro increased from 14 to 20 =6/14× number 12, the expenditures in 2000,
Solutions for questions 1 to 5: 100 = 426/7 per cent. For Bord there is a de- 2001 and 2002 are equal. Choice (3)
1. By observation the number of Swing as well crease in the sales.
∴ Pantro showed maximum percentage Solutions for questions 16 to 20:
as comfort branded cars sold in each year
is less than the Beatle, Kite and Vayu increase. Choice (1) 16. Total seats available = 80. Number of
branded cars. seats admitted by IT graduates = 162 ×
8. Revenue made by Pawan 800 in 2003
Though the sales of Swing branded cars 80/360 = 36
= 15 × 200 = Rs 3000 × 106
in 1998 and 1999 are more than that of At least one each by Engineering, sci-
Jindica = 18 × 290 = 5220 × 106
comfort branded cars, as comfort brand- ence, Arts and commence = 4; Total seats
Pantro = 14 × 310 = 4340 × 106
ed cars sold in 2000, 2001 and 2002 are filled = 40
Bord = 12 × 300 = 3600 × 106
much more than swing branded cars. Maximum possibility for Maths graduates
∴ Jindica made the maximum revenue
So on the whole, the sales of Swing to do HR specialization = 40 Choice (3)
 Choice (4)
branded cars is the least.  Choice (1) 17. Number of IT graduates admitted to op-
9. As sales volume and price both in- erations =20/360× 162 = 9 and
2. Total number of cars sold in 1998 =776.
25 per cent of 776 = 1/4 × 776 = 194 creased by maximum for pantro, its that to marketing = 80/360× 162 = 36
Vayu = 226 > 194 and Beatle = 195 > 194 revenue changes by maximum per cent. The difference = 27 Choice (3)
 Choice (4)  Choice (4)
60 − 45
3. By observation the number of cars sold 10. As there is no information on manufac- 18. Required percentage = ×100
45
in successful years, we find that the num- turing cost, given question cannot be an-
= 331/3 per cent Choice (1)
ber of cars of Comfort brand, in 1999, swered from the given data. Choice (4)
is twice the number of cars of the same 19. Number of Arts graduates in the course
brand in 1998 that is, 152/76 = 2 and it is Solutions for questions 11 to 15: 720 70
= × 35 = × 100 = 7 per cent
the greatest. Choice (1) 11. Profit per cent of Glasgowe limited only 360 1000
4. In the year 2000, 133 1/3 per cent of Vayu in 2004 was 23/45 × 100 > 50 per cent, all ∴ 93 per cent Arts graduates qualified but
branded cars sold = 4/3 × 324 = 432 other years it is less than or equal to 50 did not get admission. Choice (4)
Number of Swing branded cars and per cent. Choice (4)
20. Total number of seats available 41. Num-
Kite branded cars in the year 2000 = 154 12. Expenditure in 1999 = 35 – 7 = 28 ber of IT graduates = 60/360× 162 = 27
+ 278 = 432 Choice (3) In 2000 = 40 – 15 = 25; In 2001 = Seats filled by Commerce graduates = 41
5. If the ratio of the number of cars sold in the 50 – 25 = 25 − 27 = 14.
year 2002 to the number of cars sold in the In 2002 = 45 – 20 = 25; In 2003 = ∴ Required percentage = 14/98× 100 =
year 1998 is the greatest, the percentage 50 – 21 = 29 142/7 per cent Choice (2)
increase will be the greatest for that car. and In 2004 = 45 – 23 = 22 Choice (4)
Solutions for questions 21 to 25:
For Swing 206/128 < 2; For Kite 431/151 13. If the profit in 1998 is 100 then profit in
< 3: 21. Required percentage is the least be in the
1999 is 140. Since the profit in 1999 is Rs
year 2000 at 15/60 × 100 per cent be 25
456 7 lakhs
For Vayu < 3; and For Comfort per cent. Choice (1)
226 Profit in 1998 = 100/140 × 7 = Rs 5 lakhs.
305/76 > 4 Similarly income in the year 1998 = 22. In the year 2004, the number of books by
 Choice (4) 100/125 × 35 = 28 lakhs Indian authors = 90 – 33 = 57
Solutions for questions 6 to 10: Expenditure in 1998 = 28 – 5 = Rs 23 Number of books by Indian authors is
lakhs  Choice (3) greater than that by foreign authors by
6. Total sales of all the brands together in the
57 − 33
year 2004 = 20 + 18 + 20 + 9 = 40 + 27 = 7 + 15 + 25 + 20 + 21 + 23 × 100 per cent = 72.72 per cent
67 thousand cars. Choice (2) 14. Average profit = 33
6 Choice (1)


08_Section.indb 95 10/30/2009 11:53:34 AM


96  n  Solution Manual

23. Let the average cost of a book be Rs 4x. 8. Trade value in the year 2000 = 400 + 300 15. Required answer is 282 – (148 + 128) = 6
Then the average cost of a book authored = 700  Choice (1)
by an Indian author is x. In 2001 = 900; In 2002 = 1200; In 2003 =
Solutions for questions 16 to 20:
Total cost of all the books in 2003 = 80 × 4 x 1300 and In 2004 = 1300 Choice (2)
= Rs 320x 16. Angle represented by the population in
9. The required percentage increase the northern region = 28/100 × 360 =
Total cost of books by Indian authors
= 50 × x = Rs 50x 1300 − 1200 100.8° Choice (3)
= × 100 = 1/12 × 100 = 8.33
Total cost of books by foreign authors 1200 17. Percentage of female population in north-
= 320x – 50x = Rs 270x per cent Choice (1) ern region = (24/100 × 2) ÷ (28/100 × 5)
Ratio of the total cost of books by foreign 10. Average imports = 2900/5 = Rs 580 crore. = 24/28 × 2/5 < (1 × 2/5)
authors to that by Indian authors = 270x : The imports in two years is less than the In the East zone = 13/12 × 2/5 > 1 × (2/5)
50x = 27 : 5 average imports per year. Choice (2) In the South zone = 38/36 × 2/5 > 1 ×
 Choice (2) (2/5)
Solutions for questions 11 to 15: In the West zone = 25/24 × 2/5 > 1 x (2/5);
24. There is no relationship between the aver-
age costs or total costs in the year 2001 251 250 13/12 > 19/18 > 25/24 Choice (2)
11. Required percentage = (100) ≃
and 2002. 499 500 18. Female population in the western region
Hence the answer cannot be determined. (100) = 50 per cent Choice (2) = 25/100 × 2/5 × 576000 × 100/24 =
 Choice (4) 240000 Choice (3)
12. Required number = 251 + 499 + 174
25. Let the average cost of a book by an In- + 148 + 282 + 120 + 98 + 128 = 1700 19. Female population in the East zone
dian author or a foreign author be Rs 1  Choice (3) = 13/100 × 2x = 26x/100
Total cost of books by foreign authors in Male population in the East zone
2002 = 25 × 1 = Rs 25 13. Maximum of the numbers of residents = (12/100 × 5x – 13/100 × 2x)
Total cost of books in 2003 = 80 × 1.1 who prefer only one brand = 499 and that = 34x/100.
= Rs 88 of two different brands = 901 ∴ Required ratio 17 : 13 Choice (4)
Total cost of books by Indian authors in
2003 = 50 × 1.2 = Rs 60
Required percentage 499 ( 501 )
× 100 ≃ 20. Female population in the South zone
55.55 per cent Choice (2) = 38/100 × 2/5x = 152000 (where x is the
Total cost of books by foreign authors in
total population) ⇒ x = 1000000
2003 = Rs (88 – 60) = Rs 28 14. 156 residents have their first preference Population in the South zone = 36 per
28 − 25 as Milk Bikis and second preference as cent of × 1000000 = 360000. ∴ Male
Required percentage = × 100 per
25 Tiger. 148 residents have their first pref- population = 360000 – 152000 = 208000
cent = 12 per cent. Choice (4) erence as Tiger and second preference as  Choice (1)
Milk Bikis. Solutions for questions 21 to 25:
Practice Exercise 2 Required percentage = 8 ( )
148
× 100 ≅ The following table can be drawn to show the
number of people who read different News papers
Solutions for questions 1 to 5: 5.33 per cent Choice (2) of different localities.
1. During the last 5 days the average sales,
of sales of D was the greatest, as its sales
increased by 60 units going to be the News papers →
The Times of India The Hindu Hindustan Times
maximum. Choice (2) Localities↓

2. Observing the given data as the sales of A 20 40 40


A 5600 × = 1120 5600 × = 2240 5600 × = 2240
and C is more than that of B and D on 6th 100 100 100
day and as their prices are also more, on
6th day we can say that combined revenue
30 40 30
of A and C is more than that of B and D. B 7800 × = 2340 7800 × = 3120 7800 × = 2340
 Choice (2) 100 100 100

3. In the third part the total sales was 483 40 20 40


units which is the maximum. Choice (3) C 7000 × = 2800 7000 × = 1400 7000 × = 2800
100 100 100
4. The least profit of the trader on 15th day =
2400 × 6 + 1800 × 16 + 1800 × 16 + 1320 × 13 D 50 30 20
6400 × = 3200 6400 × = 1920 6400 × = 1280
= Rs 89,160/- Choice (4) 100 100 100
5. For none of the days the sales is double
the previous day’s cumulative sales. E 20 20 60
5000 × = 1000 5000 × = 1000 5000 × = 3000
 Choice (2) 100 100 100
Solutions for questions 6 to 10:
6. Ratio of imports to exports is the greatest
in 2002. at 700/500 that is 7/5 Choice (2)
21. In the localities A, B, C and E, more 22. Required ratio =
7. The percentage increase in imports from than 1500 people read Hindustan Times.
2001 to 2002 = 200/500 × 100 = 40. (2800 + 3200 ) = 6000 = 5 : 2. Choice (2)
 Choice (4)
 Choice (4) (1400 + 1000 ) 2400

08_Section.indb 96 10/30/2009 11:53:35 AM


Solutions for Data Analysis and Data Sufficiency  n  97

23. In the next year in locality D, Number of Number of male literates 18. If the pass percentage in a particular year
people, who read The Times of India = 240000 – 98000 = 142000 is the greatest, then the fail percentage
Number of males = 400000 – 210000 = will be the least in that year. By observa-
3200 × 120 190000 tion only in 2003, the pass percentage is
= = 3840
100 Literacy rate of males = (142/190) × 100= crossing 80 per cent and hence it is the
74.7 per cent Choice (2) greatest in 2003. Choice (3)
Number of people, who read The Hindu =
9. We can see that only town B has more 19. Number of boys failed = 439 – 349 = 90
1920 × 130 males than females. So, the ratio of males Number of girls appeared = 923 – 439 =
= 2496
100 to females in town B is greater.Choice (2) 484
and that of Hindustan Times = 10. Number of male literates in town D The required ratio = 90 : 484 = 45 : 242
1280 × 140 = (65/100) × 190 × 103 = 123500  Choice (1)
= 1792
100 Those in town E = (70/100) ×240 × 103 20. Ratio of boys passed to the girls passed in
∴ Total number of people = 8128. = 168000 2001 = 195/213 < 1
 Choice (2) Overall male literacy in both the towns In 2002 = 228/263 < 1; In 2003 = 301/
24. (2240 + 3120 + 1400 + 1920 + 1000) = 9680. 291 > 1
 1235 + 1680 
=  × 100 = 67.7 per cent In 2004 = 349/359 < 1 Choice (3)
 Choice (4) 1900 + 2400 
25. ∴ Required per cent = Solutions for questions 21 to 25:
 Choice (3)

(11660 − 10460 ) × 100 Solutions for questions 11 to 15: 21. Number of cars of model R sold
= 22/100 × 10400 = 2288 Choice (3)
10460 11. In the year 2001 =
1200 22. Required percentage = 14/22 × 100 ≃ 64
= × 100 = 10.29% . Choice (1) 1000 × 30
11660 = 300 − 80 = 220 per cent
100
 Choice (1)
In 2002 = 400 − 100 = 300
Practice Exercise 3 In 2003 = 200 − 130 = 70 23. The ratio of average value of brands S
In 2004 = 300 − 120 = 180 and P cars = 17/32 : 22/16= 17 : 44
Solutions for questions 1 to 5:  Choice (2)
Hence, 220 + 300 + 70 + 180 = 770 stu-
140.50 140.50 dents received interview calls but could 24. Assume in 2006, 100 cars were sold and
1. Average net profit = ≃
419172 42 ×104 not get selected. Choice (2) the total revenue was Rs 100
=Rs 0.033 lakhs /car Choice (1) 12. Since, we do not know that how many ∴Cost of each in 2006 = 100/100= Re.1
2. Required increase students were studying in institutes B and and that of 2007 = 125/130 = 25/26
C in 2004, therefore we cannot answer ∴ Required percentage decrease
3855.60 − 2850.50 = 1/26 × 100 ≅ 4 per cent Choice (4)
= (100) ≃ 35.3 % this question. Choice (4)
2850.50
13. Required ratio = 25. The required difference (32 − 30) per
 Choice (2)
cent, that is 2 per cent of 10400 = 208.
3. Projected total expenditure (110 + 120 + 100 + 140 ) = 470 = 47 : 84 .  Choice (2)
3400.5 180 + 190 + 240 + 230 840
= × 3400.5 ≃ Rs 5585 crores.  Choice (4)
 2070.5 Choice (3) Practice Exercise 4
14. Required number = 100 + 90 + 110 + 110
140.50
4. Required percentage = (100) ≃ = 410. Choice (3) Solutions for questions 1 to 5:
1850.36
7.6 per cent Choice (3) 1. Average monthly expenditure on food
15. Required per cent = 310 × 100 = 83.78% .  20 20 20 
5. Observing the given table, as net profit 370 24000 × + 28000 × + 25000 × +
1 100 100 100 
for the quarter ending 30/6/04 is more and = 
 Choice (2) 6  30000 × 15 + 15000 × 20 + 32000 × 25 
 
the number of cars is not so high, profit/  100 100 100 
car would be the maximum during this Solutions for questions 16 to 20:
≈ Rs 5150 Choice (4)
period. Choice (4)
16. In 2002 the number of girls appeared
2. Total expenditure on rent =
Solutions for questions 6 to 10: = 650 – 330 = 320; Girls failed = 320
– 260 = 60
6. Overall female literacy for towns B and 15 18 20
fail percentage of girls appearing 24000 × + 28000 × + 25000 ×
C, put together = (80 + 95)/(190 + 180) × 100 100 100
= (60/320) × 100 = 18.75 per cent 15 18 15
100 = 47.29 per cent Choice (1) +30000 × + 15000 × + 32000 × = 25640.
 Choice (2)
100 100 100
7. Number of males = total population – 17. Number of students passing in 2003 ≅ 290
number of females. For town A, number + 300 ∴ Average monthly expenditure on rent
of males = 450 – 230 = 220 = 590; Number of students passing in 2004 = Rs 4273
In this way the total male population ≅ 360 + 350 = 710 ∴ Required answer is 4. Choice (3)
= (220 + 210 + 170 + 190 + 240) that Percentage increase
is10.30 lakhs  Choice (3) 16 × 2800
710 − 590 3. Required percentage = × 100
= × 100 = 20.33 per cent 10 × 32000
8. Number of literates in town D 590
= (60/100) × 400000 = 240000  Choice (3) = 140 per cent. Choice (4)

08_Section.indb 97 10/30/2009 11:53:35 AM


98  n  Solution Manual

10. Required angle = 15/100 × 360° = 54° 18. In colony C, the total number of persons
4. Required percentage =
(1700 ) × 100 Choice (3) in 2000 = 167 + 155 = 322
4500 The total number of persons in 2004 =
≃ 37.7 per cent. Choice (2) Solutions for questions 11 to 15: 184 + 173 = 357; Percentage increase =
5. Observing the expenditures, E’s is the 11. Only U.P. has more than 20 per cent of the 35/322 × 100
least and his percentage, on bills is the total number of employees, as 20 per cent 10 per cent of 322 = 32.2 and 1 per cent
least, hence his expenditure on bills is of 360° is 72°. Choice (1) of 322 = 3.22 ; 11 per cent of 322 = 35.42
the least. Choice (4) per cent. 10 per cent < Ans < 11 per cent
12. Given, 60 per cent × (Actual share of  Choice (1)
Solutions for questions 6 to 10: Punjab) = 60°; Actual share = 100°; Total
share is 360 + (100 − 60) = 400° 19. In the year 2001, the number of new persons
6. Production of steel by company C in 2003 settling in A = 310; B = 274; C = 334; D =
Percentage share of the employees from
= 12/100 × 860 = 103.2 lakh tones 352; E = 212
Gujrat
 Choice (2) In two colonies, C and D the number of
= 45/100 × 100 = 11.25 per centChoice (3)
7. Among 190/500, 90/690, 260/600 and new persons settling is more than 320.
80/860, only 80/860 < 10 per cent 13. Given, 15°/360° of total employees = 840  Choice (3)
The percentage change is more than 10 ⇒ Total employees = 20,160 Choice (4)
20. The number of new persons in 2004 is
per cent only in 3 years.  Choice (3) 90° 360 more than that in 2003 for A = (178 – 163)
14. Required difference = × ×1680
8. Total production of steel except in 2003 = 360° 30 + (156 – 142) = 15 + 14 = 29
2570 = 5040 Choice (1) For B = 17; for C = 13; for D = 15; for
Product of steel by company A from 2000 E = 6; Only for two colonies A and B, in
15. Required ratio =30°/60° = 1 : 2 Choice (3)
to 2004 except 2003 = 18/100 × 2570 = 2004 it is more than 15 compared to 2003.
462.60 lakh tonnes. Solutions for questions 16 to 20:  Choice (1)
Production of steel by company A in the Solutions for questions 21 to 25:
16. Every year the number of new males and
year 2003 = 22/100 × 860 = 189.2
females settling in colony D is more than Number of students in each school in as follow
Total production of steel by company A
the other colonies. So the number of per- School A – 30 per cent of 12000 = 3600
in the given five years = 462.60 + 189.2 =
sons settling in colony D is the highest. School B – 25 per cent of 12000 = 3000
651.8 lakh tonnes  Choice (4)
 Choice (4) School C – 21 per cent of 12000 = 2520; and
9. Average production of steel per year School D – 24 per cent of 12000 = 2880
= 3430/5 = 686 lakh tonnes 17. In 2003, the number of males are less than
The production in 2001, 2003 and 2004 females in colony D only. The following table can be drawn to show the
is greater than the average production. So the ratio of males to females is the least number of students who opted different subjects
 Choice (3) for colony D.  Choice (2) in the different schools.

Schools →
A B C D
Subjects ↓
Science 25 per cent × 3600 = 900 25 per cent × 3000 = 750 37.5 per cent × 2520 = 945 37.5 per cent × 2880 = 1080

Art 25 per cent × 3600 = 900 50 per cent × 3000 = 1500 25 per cent × 2520 = 630 12.5 × 2880 = 360

Commerce 50 per cent × 3600 = 1800 25 per cent × 3000 = 750 37.5 per cent × 2520 = 945 50 per cent × 2880 = 1440

21. Required answer = 900 + 750 + 945 + 1080 24. Average number of science students in Practice Exercise 5
= 3675. Choice (2) the four schools = 3675/4 ≈ 918
∴In schools C and D, the number of sci- Solutions for questions 1 to 5:
22. Required ratio = 360°/630° = 4 : 7.
 Choice (1) ence students are more than the average. Let us calculate the total number of votes cast
 Choice (2) in different localities in different years.
23. Required per cent = (150/900) × 100 =
16.67 per cent. Choice (3) 25. By observation. Choice (4)

Year A B C D E

60,000 × 45 per cent 65,000 × 60 per cent 80,000 × 75 per cent 70,000 × 40 per cent 60,000 × 55 per cent
1990
= 27000 = 39000 = 60,000 = 28000 = 33000
70,000 × 50 per cent 72,000 × 64 per cent 85,000 × 80 per cent 80,000 × 60 per cent 65,000 × 70 per cent
1994
= 35000 = 46080 = 68000 = 48,000 = 45500
75000 × 65 per cent 80,000 × 85 per cent 90,000 × 60 per cent 86,000 × 50 per cent 75,000 × 80 per cent
1998
= 48750 = 68000 = 54000 = 43000 = 60,000
78000 × 70 per cent 84000 × 72 per cent 98000 × 65 per cent 100000 × 45 per cent 104000 × 60 per cent
2002
= 54600 = 60480 = 63700 = 45000 = 62400

08_Section.indb 98 10/30/2009 11:53:36 AM


Solutions for Data Analysis and Data Sufficiency  n  99

1. Required answer = 48750 + 68000 + Solutions for questions 6 to 10: of 175000 – 22 per cent of 15000) = 22
54000 + 43000 + 60000 that is 2,73,750 6. Amount invested in PPF and pension plan per cent of 25000 = Rs 5500 Choice (4)
 Choice (2) in 2004= (28/100) × 175000 = Rs 49000 10. Investment of the Goels in 2004 in
2. In 1994, required average number of votes Choice (1) Banks
cast 7. (Investment in banks/investment in = (7/100) × 175000 = Rs 12250
= (35000 + 46080 + 68000 + 48000 + shares) × 100 = (7/28) × 100 = 25 per cent Interest on Rs 12250 for one year
45500) ÷ 5 = 48516 Choice (1)  Choice (2) = (6/100) × 12250 = Rs 735 Choice (4)
3. In 2002, 464000 − 286180 = 177820 peo- 8. Ratio of the annual incomes in 2003 and Solutions for questions 11 to 15:
ple did not cast their vote. Choice (2) 2004 = 425/500 = 17/20 = 0.85
4. In the year 1994, maximum number of Ratio of annual spending = 275/325 = 11/13 Let us calculate all the values of revenue and ex-
voters cast their votes in constituency C. = 0.84; Ratio of annual savings = 150/175 penditure and represent this information in the
= 6/7 = 0.857 Choice (3) form of the table.
 Choice (2)
5. Required ratio = 164000 : 200900 = 1640 9. From 2003 to 2004, the increase in the in-
: 2009 Choice (3) vestment in the real estate = (22 per cent

Revenue

1999 – 2000 2000 – 01 2001 – 02 2002 – 03 2003 – 04 2004 – 05 2005 – 06


20 per cent 25 per cent 40 per cent 25 per cent 20 per cent 10 per cent
360 432 540 756 945 1134 1247.4

Expenditure
1999 – 2000 2000 – 01 2001 – 02 2002 – 03 2003 – 04 2004 – 05 2005 – 06
60 per cent 25 per cent 20 per cent 50 per cent 10 per cent 10 per cent
240 384 480 576 864 950.4 1045.44

11. The profit in the year 2002–03 was 756–576 17. The required percentage = Solutions for questions 21 to 25:
= Rs 180cr Choice (3) 21. Runs scored by player B through 1’s = 30
31800 − 24600
12. The average revenue = 5414.4 ÷ 7 ≈ × 100 % = per cent of 7500 = 2250
 24600
773.5. ∴ In 3 years, the revenue was Runs scored by player A through 1’s = 21
more than the average revenue. 29.3 % Choice (1) per cent of 6000 = 1260
 Choice (1) Required difference = 2250 − 1260
18. The number of motor cycles of Luxor
= 990 Choice (2)
13. Profitability for the year 2003 – 04 brand to be sold in 2005 = 1.20 × 24600 =
= 81/945 ≈ 0.085. Choice (2) 29520
2250 −1200
The number of RVS brand motor cycles 22. Required percentage = ×
14. By observing the table of expenditure, we 1200
sold in 2004 = 19800 100
can conclude that the increase in the value
RVS sales should increase (29520 – 19800) = 87.5 per cent Choice (4)
of expenditure is the highest for the year
in 2005 = 9720 23. Required ratio = 1440 : 1500 = 24 : 25
2003–04. Choice (3)
Required percentage = 9720/19800 × 100 Choice (2)
15. The value of profitability is the highest = 49 per cent Choice (4)
for the year 1999–00. Choice (1) 24. As there is no information on the num-
19. Let the price per unit of Road King brand ber of occasions where C was out and
Solutions for questions 16 to 20: the number of balls where no runs were
motor cycle in 2003 and 2004 be Rs x and
16. Let the number of RVS brand motor bikes Rs y respectively. scored, the question cannot be deter-
be 100 in 2003 then the sales of motor bikes Given that 100x = 105y ⇒ x/y = 21/20. mined. Choice (4)
in 2004 = 110 The price in 2004 is less than that in 25. Total number of fours scored by D and G
Sales of motor bikes RVS sold in 2003 2003 by 1/21 × 100 = 4.76 per cent
 Choice (1) 8400 + 4200
100 × 19800 = = 3150.
= = 18000 4
110 20. In 2004, the average sales of the given
Total number of fours scored by A and B
companies = 117600/5 = 23520
Similarly the number of units sold in 2003 ⇒ 90 per cent of 23520 = 21168.
20% of 6000 24% of 7500
for Luxor = 20500 This value is nearest to the number = + = 750
for Swing =24000; for Road King = of units sold for Road King brand. 4 4
20000 Choice (3)  Choice (4) ∴ Required difference = 2400.Choice (3)

08_Section.indb 99 10/30/2009 11:53:36 AM


100  n  Solution Manual

Practice Exercise 6
Solutions for question 1 to 5:

Total Number of Male employees Part time Wages of Part time Number of days part time
Employees Employees employees employees worked
Hyderabad 600 360 200 120 18
Bangalore 300 200 60 180 18
Mumbai 120 45 20 150 24
Delhi 60 30 10 150 24
Jaipur 90 30 30 225 15
Mohali 90 18 30 225 20
Chennai 150 120 75 240 10

1. Required difference = 200 − 75 = 125. Solutions for question 6 to 10: 8. Percentage increase in the production of
 Choice (2) 6. Average production rice only in 2003 is more than 25 per cent
 Choice (4)
2. Total wages of all the part-time employees = 50 + 60 + 72 + 84 + 75 + 96 / 6 = 72.8
at Mohali: 225 × 30 × 20 = Rs 1,35,000 The production of rice in 2001, 2002 9. Exports to Kenya in 1999 = 18/100 × 25/100
 Choice (2) and 2003 is greater than the average pro- × 60 = 2.7 million tonnes.
duction.  Choice (3)  Choice (2)
3. Required ratio = 200:60 = 10 : 3
 Choice (3) 7. Total quantity of rice exported in 2003 10. The percentage of increase in the produc-
= 20/100 × 96 × 106 × 103kg tion of rice from 1998 to 1999 and that
4. Total wages paid to all the employees at
Value of exports in 2003 = Rs 24000 from 1999 to 2000 are equal.
Chennai = 75 × 300 × 30 + 75 × 240 × 10 =
crore In the year 2003, the value of rice exported
Rs 8,55,000. Choice (1)
Cost of rice exported per kilogram to Iran is more than that exported to Iraq by
5. As the number of part-time employees at 4 per cent points but not by 4 per cent.
24000 × 107
Hyderabad is more than twice that at any = = Rs.12.50 In the year 2003, the value of rice export-
20
other city, the total amount is the highest × 96 × 106 × 103 ed to Kenya = 12/100 × 24000 = Rs 2880
at Hyderabad. Choice (3)  100 Choice (1) crore. This statement is true.  Choice (3)

Solutions for questions 11 to 15:


GDP (Rs Crores) Exports (Rs Crores) Imports (Rs Crores) National Income (Rs Crores) Percapita income (Rs) Population
2002-03 20,000 1,000 2,000 21,000 40,000 50,00,000
2003-04 24,000 2,400 2,400 24,000 40,000 60,00,000
2004-05 30,000 4,500 3,750 29,250 50,000 60,00,000
2005-06 30,000 6,000 4,500 28,500 48,000 62,50,000
2006-07 36,000 9,000 7,200 34,200 50,000 72,00,000
2007-08 45,000 9,000 11,250 47,250 60,000 75,00,000

11. Except in 2005-06, in every other year na- parable is 2006-07. But when compared 19. Assume in 2003, 100 motorcycles were
tional income increased when compared to 2006-07, national income in 2007-08 sold and the total amount was Rs 100 that
to the previous year. Choice (1) increased by at least 30 per cent where is average cost = Re.1
12. Required percentage as population increased by close to 5 per In 2004, it is = 120/125 = Rs 24/25.
cent. Hence in 2007-08, per capita in- ∴ per cent decrease in value = 1/25 × 100
72,00,000 − 50,00,000 come based on national income will be = 4 per cent
= × 100 = 44 per
50,00,000 the highest. Choice (3)  Choice (4)
cent Choice (3)
Solutions for questions 16 to 20: 20. Measure of the motor cycle C, for the mar-
13. By observation, only in 2003-04, the ex- ket share by value = 24/100 × 360° = 86.4°
ports were more than twice that in the pre- 16. Number of brand C motor cycles sold
 Choice (3)
vious year. Hence, the greatest percentage = 21/100 × 25200 = 5292 Choice (1)
change is in 2003-04. Solutions for questions 21 to 25:
17. By value, the sales of brand D motor
 Choice (4) cycles is more than brand A motor cycles 21. The weighted average price for the year
14. As the population did not decrease in any by (27 – 21)/21 × 100 = 29 per cent 2001
year, the required number of years is zero.  Choice (2)
= 105× 20 +120 × 30 +115× 20 +130 × 30 
 Choice (4) 18. The required ratio = 28/25 : 21/18 = 24: 100 100 100 100 

15. The national income is the highest in 25
2007-08 and the only year that is com-  Choice (2) = (21 + 36 + 23 + 39) = 119

08_Section.indb 100 10/30/2009 11:53:37 AM


Solutions for Data Analysis and Data Sufficiency  n  101

The weighted average price for the year 2002 4. Observing the given data only in 2003, the 14. Total population of state A = 225 + 210 =
 20 30 20 30  percentage increase crosses 15 per cent. 435: That of state D = 350 + 325 = 675
= 120 × +125× +120 × +135×   Choice (4) The required ratio is 435 : 675 = 29 : 45
 100 100 100 100 
 Choice (3)
5. In 2002, the required ratio = 2981/
= (24 + 37.5 + 24 + 40.5) = 126
1105= 2.7 Choice (3) 15. Existing number of literates
126 − 119 = 100 + 80 = 180lakhs.
∴ Inflation in year 2002 = × Solutions for questions 6 to 10:
119 Literacy target of state C = 60 per cent
100 6. Given profit of P in 2004-05 = Rs 60 of 375 = 225 lakhs that is 45 lakhs more.
= 5.88 per cent Choice (2) lakhs; Profit percentage of P in 2004-05  Choice (2)
22. Weighted average price for year 2003 = = 30 per cent
Solutions for questions 16 to 20:

60 lakhs
20 30 20 30  Given, 30 = × 100 16. Production cost / unit =
130 × +130 × +125× +140 ×  exp enditure
 100 100 100 100 
∴ Expenditure of P in 2004-05 = Rs 200
(1,80,00,000 ) × (120 + 60 ) = Rs 150.
= (26 + 39 + 25 + 42) = 132 Choice (4) lakhs. Hence, its income in 2004-05 = 60,000 (360)
23. The weighted average price for 2000 Rs 260 lakhs. Choice (1)
= 100; The weighted average price for 40% of 3,00,00,000
7. Let the profit of Q and R in 2004-05 be Sales price / unit =
2001 = 119 Rs 12x. Then, using the graph, we have 80%of 60,000
The weighted average price for 2002
12x 12x = Rs 250. ∴ Required difference = Rs 100.
= 126 =0.3 and = 0.4
exp Q exp R  Choice (3)
The weighted average price for 2003
= 132 ⇒ Expenditures of 17. Sales income from North = 30°/360 × 40
As the prices of all the types of articles in Q and R are in the ratio 4 : 3. per cent of 3,00,00,000 = Rs 10,00,000
2004 is more than that in 2003, weighted Incomes ratio = 52x : 42x = 26 : 21 Interest = 15/360 × 1,80,00,000 = Rs
average price will also be more than 124.  Choice (3) 7,50,000
10,00,000
∴ For the year 2002, 2003 and 2004 the ∴ Required percentage = ×
8. As the profit is 30 per cent of expenditure, 7,50,000
weighted average price is more than 124.
we have income = 130 per cent of expen- 100
 Choice (3)
diture of R in 2006-07 = Rs 390 lakhs. =1331/3 per cent Choice (1)
24. Weighted average price for the year 2000 ∴ Expenditure of R in 2006-07 = 18. Administrative expenses in 2007-08
= 100 Rs 300 lakhs. Also, 150 per cent
Average price for 2002 = 500/4 = 125 30
of expenditure of R in 2007-08 = 130 per cent of × 1,80,00,000 =
∴ Required ratio = 100/125 = 4/5 Rs 19,50,0000 360
= Rs 435 lakhs
 Choice (2) ∴ Expenditure of R in 2007-08 = Rs 290 19,50,000
Sales in 2007-08 = =
25. The average price for 2004 = 142.5. lakhs. 0.2
∴ Required percentage per cent increase in profit of R from 2006- Rs 97,50,000.
07 to 2007-08 = 55/90 × 100 = 61.11 per Sales in 2006-07 = 40 per cent of
142.5 −119 3,00,00,000
= × 100 ≈ 20%  Choice (3) cent Choice (3)
119 = Rs 1,20,00,000
9. Given, 20 per cent of P : 10 per cent of Q
∴ Required percentage
Practice Exercise 7 : 30 per cent of R = 4 : 2 : 3 that is, P:Q:R
= 20:20:10 = 2 : 2 : 1 Choice (2) 1, 20,00,000 − 97,50,000
= × 100 =
Solutions for questions 1 to 5: 1, 20,00,000
10. As the income of P in 2007-08 is not
1. During the year 2001, the total number of 18.75 per cent Choice (4)
known, the ratio cannot be determined.
students in the government schools  Choice (4) 19. Income from property and royalty
= 461 + 302 + 596 + 423 + 756 + 526 = charges = 25 per cent of 3,00,00,000 =
3064 Choice (2) Solutions for questions 11 to 15: Rs 75,00,000.
2. From 2000 to 2001, the number of girl 11. Male literacy rate for E = 275/400 × 100 75,00,000
Now, × 360 = 150°. But
students has decreased. = 68.75 per cent Choice (3) 1,80,00,000
Hence statement (I) is not true. none of the expenses form more than
12. Female literacy rate in A = 90/210 = 3/7 =
Statement (II) cannot be inferred because 150°. Choice (1)
42.84 per cent
there is no data pertaining to the number
In B = 125/275 = 5/11 = 45.45 per cent; 20. Change in expenditure on administrative
of schools.
In C = 80/200 = 40 per cent expenses, maintenance and manufacturing
From statement (III) it can be inferred
In D = 150/325 = 6/13 = 46 per cent expenses from 2006-07 to 2007-08 = 20 per
that the number of students in govern-
In E = 225/375 = 60 per cent Choice (3) cent of 105 = 21.
ment (urban) schools is more than the
Change in raw material and wages from
number of students in the private schools 13. Number of illiterates in B = (330 + 275) – (150 2006-07 to 2007–08 = – 20 per cent of
every year.  Choice (3) + 125) = 330 195 = −39
3. During 2003, the total number of girls That in C = (175 + 200) – (100 + 80) = Change in all other expenses = 30 per cent
= 361 + 478 + 593 + 544 = 1976 195 of 60 = + 18. ∴ Net change = + 21 − 39 +
The required percentage = 1976/4726 Those in B and C = 330 + 195 = 525 lakhs 18 that is, 0 per cent
× 100 = 41.8 per cent Choice (1) = 5.25 crores.  Choice (4)  Choice (2)

08_Section.indb 101 10/30/2009 11:53:38 AM


102  n  Solution Manual

Solutions for questions 21 to 25: 5. Let the number of males in 2000 and 2001 of students in 2003 is also less than that
21. Amount of trade in 2000 = 120 + 80 = be x and y respectively. Given that, 40 in 2004. ∴Total fees in 2003 will be less
200; in 2001 = 140 + 100 = 240 per cent of x = 44 per cent of y ⇒ x : y = than that in 2004.
in 2002 = 90 + 110 = 200 11 : 10. Choice (2) There is no need to find the total fees in
that is in 2000 and 2002. Choice (3) 2003. Total fees in 2001 = 125 × 500 =
Solutions for questions 6 to 10: 62500
22. In the year 2000, 2001 and 2004 there is 6. Interal marks of Alok in maths That in 2002 = 200 × 350 = 70000
trade surplus, in the remaining two years That in 2004 = 180 × 475 = 85500
there is trade deficit. The required ratio =
(49 + 96 140 ) × 40 = 38  Choice (4)
is 2 : 3 Choice (4) 300
12. Spending towards the maintenance
23. Required percentage: 8 per cent/20 per In the samyeway internal marks of = (20/100) × 75 × 400 = 6000 Choice (3)
cent × 100 = 40 per cent Choice (1) Bavesh, Chandan, Deepak and Gautham
13. Average number of students per year
24. In the year 2000, let the trade surplus are 38 each.
per employee as well as the number of ∴ Total is 190. Choice (1) 400 + 500 + 350 + 450 + 475
= = 2175/5
employees be x each. Given that x2 = 7. As all the five students got equal marks 5
40crores. in internal exams of maths, Deepak will 0= 435
⇒ x = 20000 Choice (3) get the least marks as he got the least in
external exam. Choice (2) The number of students in 2001, 2003 and
25. In which ever year the trade value is the
2004 are greater than 435.  Choice (2)
highest, that will be the required answer. 8. Aggregate marks in II language of Chan-
 Choice (3) dan is the highest as he got 38 and 48 14. Total fees collected in 2004 = 180 × 475
in the internals externals respectively. Let the number of students in 2005 be x.
Practice Exercise 8  Choice (3) Given that, 1.25 × 180 × x = 180 × 475 ⇒
x = 380 ∴ Number of students who left =
9. As all the students got 38 marks each in
Solutions for questions 1 to 5: 475 – 380 = 95 Choice (4)
mathematics and as their external marks
1. Let the number of females and males be are greater, their aggregate total in maths 15. Using the values of total fees in 2003 and
1200 and 1300 respectively. is the highest. Choice (2) 2004, the required percentage increase
Overall literacy 10. All the five students will get scholar-
=
(85500 ) − (140 × 450 ) × 100
30% of 1200 + 40% of 1300 ship in maths. In science Alok, Bavesh 140 × 450
= × 100
(1200 + 1300 ) and Gautham will get the scholarship.
∴ Scholarship amount received is 22500
= 35⋅2 per cent Choice (3) = × 100
8 × 5000 = Rs 40,000 Choice (4) 63000
2. In the year 2002, let the number of fe- = 35.7 per cent Choice (4)
males and males be x and y. Solutions for questions 11 to 15:
Given that, 35 per cent of x = 49 per cent Solutions for questions 16 to 20:
of y ⇒ x/y = 7/5. 11. By observation, fees per student in 2003 Let us calculate the values and represent it in
∴ Required percentage = 2/5 × 100 = 40 is less than that in 2004. And the number the table.
per cent.
 Choice (2)
3. per cent increase in male literates from Dainik Bhaskar Dainik Jagaran TOI
2000 to 2001 A 60% 24000 that is = 14400 20% = 4800 20% = 4800
 (44) (1.2)  B 50% =9000 20% = 3600 30% = 5400
=  − 1 [100] = 32. Choice (1)
 40  C 40% = 12800 10% = 3200 50% = 6000

4. Let the number of males and female in D 20% = 5000 40% = 10,000 40% = 10,000
2002 be 700 and 800 respectively. E 40% = 6400 40% = 6400 20% = 3200
Overall literacy rate F 30% = 6000 50% = 10,000 20% = 4000
 35 49 
 × 700 + × 800 
 100 100 
= × 100 = 42⋅5
700 + 800 16. In localities D and F more people read ties A, B and C, the required ratio is less
per cent. Dainik Jagaran than Dainik Bhaskar. than 1.
 Choice (2) 4800
Similarly overall literacy in 2003 Locality A = = 0.333.
14400
 38 50  17. Total number of people who read Times
 × 900 + × 1100  of India = 43,400 Choice (1) 3600
 100 100  × 100 = 44.6 For Locality B = = 0.4 and
= 9000
900 + 1100 18. In localities C and D, more people read
per cent. Times of India than Dainik Bhaskar. In 3200
For the Locality C = = 0.25.
others it is the reverse. Choice (4) 12800
The increase is 44⋅6 − 42⋅5 = 2⋅1 per cent
points. Choice (2) 19. By observation, we can say that in locali- ∴ The ratio is the least for locality C.
 Choice (3)

08_Section.indb 102 10/30/2009 11:53:38 AM


Solutions for Data Analysis and Data Sufficiency  n  103

20. Required per cent = 3. Minimum amount required = 40 × 15 × D = (2700 − 5347) × 1 + 132 × 2
[35 + 30 +25 + 40] = 78000 Choice (2) + 185 × 5
4800 + 5400 +10000 + 3200 = − 2647 + 264 + 925 = − 1458
× 100 4. Demand of the products A, B, C and D at
14400 + 9000 + 5000 + 6400 Choice (1)
Q are 32, 24, 36 and 28 respectively.
23400 ∴ Only W is in a position to transport Solutions for questions 16 to 20:
117 × 2
= × 100 ≅ × 100 ≅ 662/3 to Q. Choice (1)
34800 117 × 3 16. Value per tonne of sugar exported in 2000
per cent 5. As W and X are in a position to supply = 144/12 = 12; In 2001 = 165/15 = 11
 Choice (2) atleast one product to each of P, Q, R In 2002 = 130/10 = 13 and in 2003 =
and S, we can say that these are optimum 156/12 = 12.
Solutions for questions 21 to 23: suppliers. Choice (3) In 2004 = 230/20 = 11.5 and in 2005 =
21. The required percentage increase 275/22 = 12.5. It is the greatest in 2002.
Solutions for questions 6 to 10:
 Choice (1)
10 − 4 6. Ratio of estimated imports of edible oils
= × 100 = 150%  Choice (2) 17. The percentage increase in the exports of
4 to the actual imports of edible oils
sugar (by value) in 2001 = 21/144 ≈ 1/6.8
22. By observation the car moves at 8 km in 1996 = 30/23 = 1.30; in 1997 = 40/30 =
≈ 14 per cent
per minute when the time is 60 seconds. 1.33; in 1998 = 45/27 = 1.6 and in 2001 =
In 2003 = 26/130 ≈ 1/5 ≈ 20 per cent
 Choice (3) 80/60 = 1.33 Choice (3)
In 2004 = 74/156 = 1/21 ≈ 50 per cent and
23. When the time is 10 seconds, speed is 7. The required percentage = (121/295) × 100 In 2005 = 45/230 = 1/5.1 = 20 per cent
1km per minute. If we substitute k = 10, = 41 per cent Choice (2)  Choice (1)
we get speed = 10/10 = 1 kmpm 8. The percentage increase in the value of 18. The average quantity for the given period
When time is 20 seconds speed = 20/10 = estimated imports in 1997 = (10/30) × 100
2 kmpm. = 33.33 per cent 12 + 15 + 10 + 13 + 20 + 22
=
∴ The required function is k/10 in 1998 = (5/40) × 100 = 12.5 per cent 6
 Choice (1) in 2000 = (10/60) × 100 = 16.66 per cent
92
and in 2001 = 10/70 × 100 = 14.28 per = = 15 ⋅ 33
Solutions for questions 24 and 25: 6
cent Choice (1)
24. Profit is earned in the years 2001, 2002 and In the years 2004 and 2005, the quan-
2004. 9. During 1998, the value of imports from tity exported is greater than the average.
The profit earned is Rs 2559484, Rs Malaysia = Rs 1400 crore. When con-  Choice (2)
2991386 and Rs 2609540 respectively. verted into
19. Value of sugar exported in 2001 = 165/15
 Choice (2) US $ = 1400/40 = $35 crore and Cost of
= Rs 11, in 2004 = 230/20 = Rs 11.50
5 litres = $4
The difference is Rs 0⋅50. Choice (4)
25. The sales has decreased from 2002 to
Number of litres for $35 crore =
2003. Statement (Ι) is not true. 1 ⋅ 4 × 275
35 × 107 × 5 20. Value per tonne in 2006 = =
In the year 2000 and 2003 the company kl 1 ⋅ 25 × 22
does not make profit. Statement (ΙΙ) is not 4 × 103 Rs 14. Choice (4)
true. = 437500 kl Choice (4)
The percentage increase in the sales for Solutions for questions 21 to 25:
10. By observation the required ratio is 4 : 2
the given years approximately is or 2 : 1 Choice (4) 21. As A, B, C, E and G have higher share
8864 − 5326 in 2000 than in 1990, their population
× 100 = 66% Solutions for questions 11 to 15:
5326 increased. Only for F there is decrease,
11. B, C, D, H, N and O have the batting av- since 11 × 2 > 3 × 3 Choice (1)
Conclusion (C) can be drawn from the
erage above 45 Choice (3)
given data.  Choice (3) 22. In the year 2000 the population of Meher
12. B, E, F, I, J, K, L, N, O and P have a bowl- Nagar
ing averages less than 25. Choice (2) = (20 + 5) per cent = 25 per cent
Practice Exercise 9
13. Only A and D have their batting and bowl- Similarly the population of Sai nagar in 2000
Solutions for questions 1 to 5: ing averages more than 25. Choice (4) (25 + 10) per cent = 35 per cent
The difference is 10 per cent of 75 lakh =
1. From godown x, the supply of at least 14. Only D, I, J, K, M, N and O have the strike 75,000 Choice (3)
three of the products A, B, C and D meets rate more than 20 and the no. of catches
the demand at R and hence can be trans- taken per match more than 0.8 Choice (4) 23. In 1990, the population of the tribal peo-
ported to R. Choice (2) ple = 25/100 × 12/100 × 50= 1,50,000.
15. Net Runs = (Runs Scored − Runs conceded) ×  Choice (2)
2. Total demand of A: 35 + 40 + 40 + 35 = 1 + Catches × 2 + wickets × 5
150 24. In 2000, the required percentage =
A = (13,510 − 3423) × 1 + 196 × 2 + 121
New demand for all the products ×5
= 50 per cent of 150 + 145 + 140 + 140 15 − 12
= (10087 + 392 + 605 = 11084 × 100 = 3/12 × 100 = 25 per cent
= 500 B = (7,202 − 1237) × 1 + 142 × 2 + 66 × 5 12
Total supply of all the products = (5965) × 1 + 284 + 330 = 6579  Choice (3)
= 125 + 135 + 125 + 115 = 500 P = (11323 − 1726) × 1 + 247 × 2 25. The required ratio is 20/100 × 50 : 20/100
∴ Excess of demand = 0 tonnes. + 70 × 5 × 75 = = 2 : 3 Choice (4)
 Choice (2) = (9597) + 494 + 350 = 10441

08_Section.indb 103 10/30/2009 11:53:39 AM


104  n  Solution Manual

Practice Exercise 10 10. ∴ Required ratio = 40/100 : 198/360 = 8 : 11. 14. Required percentage = (300/100) × 100 =
 Choice (2) 300 per cent Choice (4)
Solutions for questions 1 to 5: 15. Let the cost of each oven of company A
Solutions for questions 11 to 15: be ‘n’. So, the cost of the oven of com-
1. Ratio of returns on the investment of B
during the first and the second year = 28 11. By observation, the total production by pany B = n + 500
per cent of 45 : 40 per cent of 30 = 21 : 20 the given companies was the greatest in Required difference = (n + 500) × 400 – n
 Choice (2) the year 2004. Choice (4) × 350
= 50n + 200000. But ‘n’ is not known.
2. Return on investment of E in the first year 12. Required average Choice (4)
75 per cent of 60 = 45
200 + 150 + 250 + 400 + 400 Solutions for questions 16 to 20:
During the second year = 35 per cent of = = 280 .
20 = 7 5 In 2003 25 per cent of the two wheelers pro-
Overall = 62 per cent of 100 = 62 The production of A in 2000, 2001 and duced are sold in India⇒ Remaining 100 − 25 =
Return on investment in the third year 2002 is less than 280. Choice (3) 75 per cent, 2 wheelers are exported. Like wise
= 62 – (45 + 7) = 10 (100 − 55) = 45 per cent of 4 wheelers are ex-
Return on investment as percentage 13. Since the total production of C is 1650 is ported
= 10/20 × 100 = 50 per cent Choice (2) the greatest, the average production will Let us represent the percentage of vehicles ex-
also be the greatest for C.  Choice (3) ported in a line graph
3. During the first year, investment by E is
the greatest and return on investment also
is the greatest.  Choice (3)
100
4. Ratio of investments of D and F during 90
the second year = 10 : 30 = 1 : 3 Choice 80 75
(4) 65 62
70 60 65 4 - Wheelers
5. Since the total amounts invested by the 60 63
persons are equal for the given three 50 55 3 - Wheelers
50 58
years, whoever gets the least percentage 40 50 2 - Wheelers
45
of return will also get the least amount of 43
30
return. By observation, B’s return is the
20 30
least.  Choice (2)
10
Solutions for questions 6 to 10: 0
6. As there is no individual split-up of stu- 2003 2004 2005 2006 2007
dents from different places, data is not
sufficient to answer this question.Choice
(4) 16. By comparing the graphs they are exhib- Solutions for questions 21 to 25:
iting inverse relationship. Choice (1)
7. Total number of students from 21. Total number of shoes manufactured by
Hyderabad = 900; Number of BE students 17. Required ratio = 42 per cent of 25000 : 45 the four companies A, B, C and D are 942,
from Hyderabad per cent of 50000 472, 776 and 1035 respectively. ∴ D > A
= 1200 × 25/100 = 300 = 7 : 15 Choice (2) > C > B Choice (2)
300 18. Total no of 3 wheelers produced in India 22. The number of shoes manufactured by
∴ Required percentage = × 100 =
900 = 35 + 65 + 45 + 60 +40 = 2,45,000 company B in quarter I = 21 + 30 + 37 +
33.33 per cent Choice (3) Total number of 3 wheelers exported = 15 = 103.
(.63 × 35,000) + (,43 × 65,000) + (0.3 In quarters ΙΙ, ΙΙΙ, and ΙV are 108, 119 and
8. Number of students from BBA back 142 respectively. By observation, only
× 45,000) + (0.55 × 60,000) + (0.65 .
ground who took marketing and are from from quarters ΙΙΙ to ΙV, there is an increase
40,000) = 1,22,500
Mumbai of approximately 20 per cent, whereas
= 60 per cent of 20 per cent of 3000 × 1, 22,500 other cases it is clearly less than that.
20/100 = 72. Required percentage = ×100 =
2, 45,000  Choice (2)
∴ Required difference = 120 – 72 = 48
 Choice (2) 50 per cent Choice (3) 23. The total number of shoes manufactured
19. Observing the above graph as the exports by company C in Q ΙΙΙ in 2005
9. Total number of students who are from
of 4 wheelers highest at 70 per cent and = 48 + 62 + 78 + 27 = 215
Delhi in 2006 = 20 per cent of 3000 =
their products on is also the highest dur- Number of shoes manufactured in Q ΙΙΙ
600
ing that year, only in 2005 is the percent- In 2004
In 1999, number of students from Delhi
age is more than 50 per cent Choice (2)
215
20%of 20. Total units produced in India = 7,42,000 = = 172  Choice (1)
= = 500 1 ⋅ 25
120% Total Number of units exported
∴ Total number of students in 2005 = 1,22,500 + 1,63,400 + 1,01,550 = 24. The total no. of shoes for male and female
= 500/20 per cent = 2500 3,87,450 segments
∴ Increase in intake = 500 Choice (1) ∴ Required answer = 74200/38450 ≃ 2 = (219 + 296) + (95 + 134) + (173 + 221)
 Choice (3) + (220 + 325) = 1683 Choice (4)

08_Section.indb 104 10/30/2009 11:53:39 AM


Solutions for Data Analysis and Data Sufficiency  n  105

25. By careful observation, we can find c = d = –1/2and (a + b) (c + d) is an Ratio of their present ages
that no company satisfies the condition. integer and a = b = 1/2; c = 1/3; d = 5x − 2 5 1 / 2
 Choice (1) 1/4 and (a + b) (c + d) is not an integer. = = + which is more than
4x − 2 4 4x − 2
 Choice (4)
5/4. B is sufficient. Choice (1)
9. To have the maximum number of items,
DATA SUFFICIENCY one must purchase more of cheap items. 16. Using both the statements, if the HCF
∴a = 1. Hence statement A is sufficient. of two numbers is 1 then the numbers
Practice Exercise 1 To have minimum number of items one are relative primes. Let a = 7, b = 2 ⇒
must purchase more of costlier items. ∴ a–b=5
Solutions for questions 1 to 25: a = 4. Let, c = 5, d = 3 ⇒ c – d = 2
Either of the statements are sufficient. (a – b) and (c – d) are relative primes
1. Let the cost of each chocolate and each
Choice (2) Let us take another example. But for a
biscuit be Rs c and Rs b respectively.
= 7, b = 2, c = 11, d = 6
4c + 8b = 20; ∴c + 2b = 5-(1) 10. From statement Ι, we know that x2 > y2. Here, (a – b) and (c – d) are not relative
Using statement A, c ≤ 2 From this we cannot say whether x9 > y7 primes.
If c = 1, b = 2, c ≠ 2 as b = 1.5. A is suf- or not since x may be positive or nega- Hence, the question cannot be answered.
ficient. tive.  Choice (4)
Using statement as B, b ≤ 2, we have From statement ΙΙ, we know that y is a
different possibilities for c. B is not suf- positive integer. Again this alone is not 17. Let the present ages of Prakash and Rake-
ficient. Choice (1) sufficient, since we do not have any infor- sh be p years and r years respectively.
2. Combining both the statements we get, a mation about x. Prakash was r years old (p − r) years ago.
+ b – c + d = (a + b) – (c – d) = even – odd Using both the statements also we cannot Age of Rakesh, then = r − (p − r) = (2r −
= odd Choice (3) say whether x9 > y7or not since we do not p) years = p/2⇒ 3p/4
know whether x is positive or negative. Using statement A, p = 80. ∴r = 60 A is
3. Let the number of days the maid was sufficient.
 Choice (4)
present be x. 50x − (30 − x) 10 > 500 ⇒ Using statement B, r > p/2. B is not
× > 15 11. It is given 2x + 3y = 17
sufficient. Choice (1)
From statement A: number of days the Statement A is not sufficient to answer the
maid worked is greater than 15, State- question, as possible values of x and y are 18. From statement A, we know that the high-
ment A is sufficient. (4, 3) (7, 1) (1, 5). est power of 2 contained in k! is 19.
Statement B not sufficient to answer the Statement B alone is sufficient, as x = 7 If k = 22 or 23 then the highest power of
question as the number of days worked and y = 1 is the only solution. Choice (1) 2 in k! is 19.
may be 16, 17, 18 etc., Choice (1) To find the number of zeroes at the end
12. If the HCF of two numbers is 1 then the of k! we need to find the highest power
4. From statement A, we cannot say whether two numbers are relative primes (co- of 5 in k!
x21 > y21 or not as x may be positive or primes). So statement A alone is suffi- If k = 22 or 23 the highest power of 5
negative. cient. Choice (1) in k! is 4.
Statement B alone is not sufficient to find
n So the number of zeroes at the end of
whether x21 > y21 as we do not have any 13. Let the fraction be , Using statement k! is 4. Choice (1)
information about y. d
Using both the statements, as x2 > y2 and n +1 3 n 3 19. Let A be 10x+y.
x is positive, we can say that x21 > y21. A, = ⇒ < <1 Using statement A, 10(x + y) − (10x + y)
d +1 4 d 4
 Choice (3) =9
n −1 5 n 5 ⇒ y = 1 A is sufficient.
5. By combing both the statements also, Using statement B, = ⇒ >
d −1 7 d 7 Using statement B, 10x + y − (x + y) = 9
we cannot answer the question as y > z or  Choice (2) ⇒ x = 1 B is not sufficient. Choice (1)
y < z. Choice (4)
20. From statement A: 3p + 2Q = 35 so the posi-
6. If a number has only 3 factors, then the 14. Product of x and y = (LCM of x and y) ×
tive pairs of P and Q are (1, 5) and (3, 3).
number is always a perfect square. (HCF of x and y). Using both the state-
So P can be 1 or 5 and hence a unique
So statement I alone is sufficient. ments, product of x and y = 2 × 2 = 4
value of P is not possible.
 Choice (1)  Choice (3)
From statement B: 3p + 2Q = 89. The pos-
7. Let the number of pens, pencils and eras- 15. Using statement A, Let the age of the man sible pair of P and Q is (4, 3). So P = 4.
ers he bought be x, y and z respectively. two years ago be 13x years and wife be  Choice (1)
Total expenditure = Rs (4x + 3y + 2z) 10x years. 21. Let the numbers of chocolates with Ramesh
Either of the statements are not sufficient Ratio of their present ages = and Suresh be r and s respectively.
to answer the question: 13x + 2 0.6 both statements, s = r. Choice (3)
Using both statements, x + y + z = 6 = 1.3 − which is less
10x + 2 10x + 2 22. From statement A, we have z = y –1.
and x, y, z ≥ 2 that is, x + y + ≥ 6. ∴ x =
than 1.3. ∴xyz = (y + 1) (y) (y – 1)
y = z = 2, Both statements taken togeth-
er are required to answer the question. We cannot say whether this is more than As the product of 3 consecutive integers
 Choice (3) 5/4 or not. A is not sufficient. is always divisible by 6, xyz is also divis-
Using statement B, Let the age of the man ible by 6.
8. Either statement alone is not sufficient. two years hence be 5x years and wife be ∴Statement A alone is sufficient.
By combing both the statements, a = b = 4x years.  Choice (1)

08_Section.indb 105 10/30/2009 11:53:40 AM


106  n  Solution Manual

23. Let the costs of a pen, an eraser and a From this, we cannot say whether ab = 10. Combining (A) and (B): 1200 = P + 120,
sharpner be Rs p, Rs e and Rs s respec- cd or not. from which ‘P’ can be found.
tively. b d  R 
5

Using both statements, 2p = 3e + 1 From statement B, a= c ⇒ ab = From 1200 = P 1+  , R can be


100 100 found.  100 
and 2e
cd. Hence B alone, is sufficient. Choice
= s + 6. Hence 4p = 3s + 20 Hence, amount can be found out at the
 (1)
Both statements even when taken end of 4th year. Choice (3)
together are not sufficient to answer 5. Let the numbers of apples, bananas and
the question as we can say p/s > 3/4. mangoes purchased by Ram be a, b and m 11. Given a + b + c = 90-------(1), where a, b
 Choice (4) respectively. and c are the respective weights of A, B
Total expenditure = Rs (10a + 12b + and C.
24. From statement A, we know that x2y > 0 From A, we have a + b − c = 70-------(2)
15 m)
This is possible only when y is positive (1) − (2) gives c = 10. From (1) and (2),
Using statement A, we have 10a =
(as x2 is always positive and x2y should a + b = 80
12b = 15 m
be positive). We have only one equation with two un-
From statement B we have a + b +
∴Statement A alone is not sufficient to say knowns a and b.
m = 30.
about x. ∴ b cannot be found. A is not sufficient.
Using both statements we have b = 5/6
From statement B, we know that x3y2 < 0 Using statement B, we have a + c = b − 30
a and m = 2/3a
⇒ x is negative as y2 is positive 90 − b = b − 30 ⇒ b = 60. B is sufficient.
Substituting them in a + b + m = 30, we
∴Statement B alone is not sufficient to  Choice (1)
get a = 12
say about y.
Total expenditure can be found.Choice (3) 12. From statement A, we have the units digit
Using both the statements we can say that
x is negative and y is positive ⇒ xy < 0 6. Let the total value of the consignment be of x3 = 2
 Choice (3) This is possible only when the units digit
x 3 2x 6
Rs x. ∴Total profit = × + × of x is 8.
25. Given, e + g + h = 120 (1) where e, g 3 100 3 100
∴Statement A alone is sufficient
and H are the numbers of marbles with But we should know the total profit to From statement B, we have the units digit
Eswar, Ganesh and Harish be e, g and h find x, which is given as Rs 2000 in state- of xn is 9. This is possible when the units
respectively. ment B digit of x is 3 or 7.  Choice (1)
Using statement A, g + h = 90. As g is ∴Using both the statements we can find
unknown, h cannot be found. 13. Let x be 10a + b. Using statement A, we
the value of x. Choice (3)
Using statement B, e + g = 70. have (10a + b) + (10b + a) = 11
7. Let the costs of each idli, dosa and each ⇒ 10a + b + 10b + a = 11 ⇒ a + b = 1
Hence h = 50 from (1) B is sufficient.
puri be Rs i, Rs d and Rs p respectively. ∴ a = 1 and b = 0 is the only possibility.
 Choice (1)
Using statement A, we have ∴ab = 10. A is sufficient.
8i + 9d + 10p = 222 --------- (1) Using statement B, we have
Practice Exercise 2 Using statement B, we have 10a + b − (10b + a) = 9 ⇒ a − b = 1
16i + 17d + 20p = 432 --------- (2) The number has more than 1 possibility.
Solutions for questions 1 to 25:
We have only one equation with three un- B is not sufficient. Choice (1)
1. Both statements even when taken to- knowns i, d and p, ∴ d cannot be found
14. From statement A, 2 or any prime number
gether are not sufficient to answer the from any statement.
more than 2 will satisfy the given condi-
question as they provide only information Combining both the statements and (2) –
tions.
about ratio of incomes and expenditures 2x (1), gives d= 12  Choice (3)
∴A alone is not sufficient.
not their actual values.
8. From statementA, we know thatA’s savings From statement B, (n – 1) is odd and hence
 Choice (4)
= 16 per cent of his salary and B’s savings (n – 1) is not an even number. ∴State-
2. To calculate the simple interest we need = 20 per cent of his salary. ment B alone is sufficient. Choice (1)
the sum, the rate of interest, and the peri- But we do not know whether their sav-
15. Given x and y are natural numbers that
od of investment. As the period of invest- ings are equal or not, therefore statement
satisfy 2x + 3y = 13
ment is not known, the question cannot be A alone is not sufficient.
By trail and error, the possible solutions
answered. Choice (4) From statement B, we know that savings
are (2, 3) and (5, 1).
3. Given, A α 1/d2 that is a point nearer of A and B are equal.
From statement (A), both (2, 3) and (5, 1)
to the source will have more intensity of ∴Using both the statements we get, 16
are possible.
light. per cent of A’s salary = 20 per cent of B’s
From statement (B), as x ≮ y (5, 1) is the
From statement A, we have A/B = 4/3. salary.
only possibility. Choice (1)
∴ A will have less intensity of light. ⇒ A’s salary : B’s salary = 5 : 4. ∴A’s
salary is more than B’s salary. Choice (3) 16. From statement A, if x is a positive inte-
Statement A is sufficient.
ger then x < x2 and if x is a negative inte-
∴ Statement B is not sufficient as 9. Let the present ages of Mohan and Sohan
ger then x < x2.
distance of A from the source is given, be m years and s years respectively.
If x = 1 then x = x2. Hence we can say
but the position of B is not known. Using statement A, m = 2(s − 10) + 30 that
that x is never greater than x2. So A alone
 Choice (1) is, m = 2s + 10. ∴ m > 2s A is sufficient.
is sufficient.
a b Using statement B, m – 10 = 3s – 30 that
4. From statement A, ×c = ×d ⇒ Statement B alone is not sufficient as x
100 100 is, m = 3s − 20.
> x2 for x = 1/ 2 and x < x2 for x = 2.
∴ m < 3s B is not sufficient. Choice (1)
ac = bd  Choice (1)

08_Section.indb 106 10/30/2009 11:53:40 AM


Solutions for Data Analysis and Data Sufficiency  n  107

17. Given, E = A + nF, where E = Total ex- 22. Statement A alone is not sufficient as Percentage increase in his savings
penditure. A = Entry fees; F = Cost of there is no information about principal or
(x + 0.2x − y) − (x − y) (0.2x)
food and n = number of persons. interest. = = × (100) > 20% .
x−y x−y
A From statement B alone we know that CI
∴ Expenditure per member = + F ; for the first year is 10 per cent less than If his expenditure increased by 20 per
n
that for the fourth year. cent, then increase in his savings =
A If the rate of interest is r per cent then
From statement A we have 150 = +F (0.2x − 0.2 y)
10 = 20% .
 r 
4
x−y
 -------(1) 9 1 +  = 10 . So, r can be found out
From statement B, we have  100  Similarly if we take any amount of increase
2500 A between 0 to 20 in expenditure. The saving
= + F -----------(2)  r 
2

CI = 1 +  − 1 100 will be greater than 20 per cent.
30 30
 100   ∴ In either case it will be ≥ 20 per cent.
Combing both the statements and solving ∴ Statement A is sufficient.
(1) and (2) A = Rs 1000 and F = Rs 50  10  Using statement B, increase in monthly ex-
=  − 1  × 100% .
∴ E = 1000 + 1000 = Rs 2000. Choice  9  penditure = Rs 0.2y. Suppose his monthly
(3) income increases by 20 per cent. Increase
Thus, required percentage difference can in his monthly income = Rs 0.2x.
18. From statement A y can be either even or
be found  Choice (1) Percentage increase in his savings
odd. The same conclusion can be drawn
from statement B also. Hence, the ques- 23. From statement A as he had, as maximum = (x + 0.2x) − (y + 0.2y) − (x − y) ×100 = 20% .
possible coins he must have minimum x−y
tion cannot be answered even after com-
bining both the statements. Choice (4) possible number of coins of Rs 5. ∴ The If his monthly income increases by more
number of Rs 50 ps coins should be 44 than 20 per cent, his percentage savings
19. Let the present ages of Raja, his wife and ∴ Number of Rs 1 coins are not more will be more ∴ the percentage increase in
his son be x years, y years and z years re- than 1. Statement A is sufficient. his savings will be more than 20 per cent.
spectively. From statement B, as Ravi had minimum B is sufficient. Choice (2)
x + z = 70 --------- (1) number of coins he must have 2 half ru-
Using statement A, when Raja’s wife 2. Using both the statements also, we cannot
pee coins and 2 one rupee coins and 5 five find the average as we do not know the six
would attain Raja’s age, she would be x rupee coins. Statement B is also sufficient
years old. consecutive numbers. Choice (4)
to answer the question. Choice (2)
She would attain Raja’s age (x − y) years 3. Let Raj’s monthly sales in this month be
from now. 24. From statement A, as the selling price of
Rs x. His monthly commission in this
Raja’s age then= x + x − y = (2x − y) 15 articles > cost price of 10 articles, s.p.
month = Rs 0.2x.
years. of 1 article may or may not be greater
Using statement A, 0.2x ≤ 2000, x ≤
3x − y = 140 --------- (2) than that of c.p. of article, hence we can-
10000 A is sufficient.
Using statement B, when Raja’s son not answer the question.
Using statement B, 0.2x ≥ 2000, x ≥
would attain Raja’s wife’s age, he would From statement B, as the selling price
10000 B is not sufficient. Choice (1)
be y years old. of small number of articles are more
than the cost price of greater number of 4. Let the first term of A.P. be ‘a’ and com-
He would attain Raja’s wife’s age (y − z)
articles hence the trader makes profit. mon difference be ‘d’. Given a = even
years from now.
 Choice (1) From statement A, we have 8th term of
Raja’s age then will be (x + y − z) years.
the progression is even that is a + 7d =
Given, x + 2y − z = 150 --------- (3) 25. Let the numbers of chicken and sheep in
even.
Using both statements, we have three un- the farm be c and s respectively. c + s =
Already as a is even, hence 7d should be
knowns and three different equations and 29 --------- (1)
even that is d is even, hence we can find
solving them we can find y. Using statement A, total number of legs
that all the terms are even.
 Choice (3) = 2c + 4s = 88.
From statement B, as 13th term is even
∴c + 2s = 44--------- (2); (2) − (1) ⇒ s =
20. Statement A alone is not sufficient since that is, a + 12d = even. Anyway as 12d is
15. A is sufficient.
the value of (a or b) and (c or d) is not even, we can not find whether d is even or
Using statement B, c − s = 1 or s − c = 1
known. not. Hence we cannot find whether 16th
If c − s = 1, from (1), s = 14 or if s − c
Statement B, alone is sufficient since it is term is even or not. Choice (1)
= 1, from (1), s = 15. B is not sufficient.
known that the numbers are in increasing
 Choice (1) 5. Let the numbers of boys and girls in the
order that is,
class be b and g respectively. Required
d > c > b > a.
Practice Exercise 3 b
Hence ( a + b )< ( c + d ). Choice
Solutions for questions 1 to 25:
per cent =
b+g
(100) per cent

(1) Using statement A, b + g = 70. As b is
1. Let the initial monthly income and that ex-
21. From statement A, to have the maximum penditure of Ram be Rs x and Rs y respec- unknown. ∴ required percentage cannot
number of items, a should be y. Statement tively. His saving Rs (x − y). be found. A is not sufficient.
A is sufficient. Using statement A, increase in monthly Using statement B, b/g = 3/4. Substituting
From Statement B, to have minimum income = Rs 0.2x. these values, required percentage can be
number of items, a should be y. B is suf- Suppose his monthly expenditure does calculated.
ficient. Choice (2) not increase. B is sufficient. Choice (1)

08_Section.indb 107 10/30/2009 11:53:40 AM


108  n  Solution Manual

6. Using both the statements the question 12. From statement A, by knowing the prod- 19. A is not sufficient as we do not know
cannot be answered as the milk is sold uct of the roots alone we cannot say what percent of poor students travel to
at cost price or not is not mentioned. whether the sum of the roots is greater school by car, so, we can not answer the
 Choice (4) than the product of the roots or not. State- question.
7. Let the number of employees in the of- ment A alone is not sufficient Using statement B, number of students
fice be x. Number of married employees From statement B, we have the quadratic 27
who travel to school by car = ×
= 0.4x. equation as 5x2 –3x + 1 = 0, hence we 27 + 73
Using both statements, number of married can find whether sum of the roots is
male employees = 0.75x0.4x = .3x. Num- greater than product of the roots or not. 100 that is 27 per cent. Hence B is suf-
ber of these who have atleast one child  Choice (1) ficient. Choice (1)
= 0.8 (0.75x) = 0.6x. Number of married 20. The question can be answered from state-
13. Let the share of Praveen be Rs p. Share of
female employees who have a child (4x × ment A alone, as in an equilateral triangle
Rakesh = Rs (60000 − p).
0.25) × 0.6 = .06x the perpendicular bisectors pass through
Using statement A, we have
0.24x + 0.66x p = 60000 − p − 12000 ⇒ p = 24000 the opposite vertices.
Required per cent = x100%
.4x Using statement B, as ratio of profits = Statement B alone is not sufficient as we
= 15 per cent. ratio of investments for these two per- do not know what type of triangle is ABC.
Both statements taken together are re- sons, we can find p as 2/5 (60,000). B is  Choice (1)
quired to answer the question.Choice (3) sufficient.  Choice (2) 21. Using statement A, given k+1 = 4 ⇒
8. Statement A alone is not sufficient as we k=3
14. Combining both the statements, it can be
do not know the cost of cheaper variety. 4 −3
concluded that B is more efficient than C, per cent increase = × 100 = 331/3 per
From statement B alone, since the cost of as the work done by B in one day is more 3
the cheaper variety itself is Rs 18, so the than that of C by 1/36. Choice (3) cent. A is sufficient
cost of the mixture is always more than Using statement B, k is known. ∴ per-
15. Using statement A, extra interest = (1000)
Rs 18 per kg. Choice (1) centage increase can be found. B is suf-
9. Let Ram’s cost price be Rs x. His profit  R  ficient. Choice (2)
  (2) = 200 that is R = 10. A is
percentage = x per cent  100  22. Using both the statements we can find
Using statement A, x + x per cent of x that is sufficient. their time taken as shown below
= 39, x2 + 100x − Using statement B, extra interest = (P) Time taken by X to complete one round = 2/10
3900 = 0 ⇒ x = 30 or – 130. Since x > 0, ∴ x × 60 = 12 min and that of y = 2/8× 60 =
= 30. A is sufficient.  10 
  (2) = 200 that is P = 2000. B is 15 min
Using statement B, x per cent of x = 9  100 
Therefore, they meet at the starting point
⇒x2 = 900 not sufficient. Choice (1) once every 60min. (L.C.M of 12 and
⇒ x ± 30. Since x > 0, ∴ x = 30 B is 15). Hence we can answer the questions.
16. Using both the statements, as we know
sufficient.  Choice (3)
the time taken by the train to cover 130 m,
 Choice (2)
we can find the time taken by it to cover 23. Using statement A, let the target people
10. The ratio of the 7th terms is
250 m. Choice (3) be P, Total expected money = 3000 × P
t7 a1 + (7 −1)d1 a1 + 6d1
= = collected money = 5000 × P × 69/100 =
t1 7 a 2 + (7 −1)d 2 a 2 + 6d 2 17. Let the monthly salaries of Raja,
3450P which is more than 60 per cent of
Suresh and Pavan be Rs r, Rs s, Rs p
Using statement A, alone: total expected money. A is sufficient
respectively.
13 From statement B, the trust collected
s13 [2a1 + 12d1 ] 3 r + s + p = 1,00,000 --------- (1)
only 31 × 20P = 620P which is not 60
= 2 = Using statement A, we can find r but not
per cent of the total expected money. As
s113 13 2a + 12d 2
[ 2 2] p. A is not sufficient.
we do not have any information about
2
Using statement B, p = r + s the other 69 per cent of people. so, we can-
a1 + 6d1 3 = 1,00,000 − p.
⇒ = not determine. Statement B not sufficient.
a 2 + 6d 2 2 p = 50,000 B is sufficient. Choice (1)  Choice (1)
Statement B alone is not sufficient as 18. From statement A, we know that the tri- 24. From statement A, ABCD may be a square
there is no information about the values angle ABC is a right-angled triangle. To or a rhombus, but since every square
that a and d take. Choice (1) find its area we should know at least two is a rhombus, therefore statement A
11. Using statement A, as we have discount of its sides. ∴Statement A alone is not alone is sufficient to answer the question.
per cent is less than profit per cent. But sufficient.  Choice (1)
as discount per cent is calculated on M.P., From statement B, we know that the
25. Let the cost price of A be Rs y.
which is normally more than CP, we may triangle ABC is the largest triangle.
Using statement A, decrease in profit =
have discount less than or more than or But to find its area we should know its
10/100 y
equal to profit. radius.
= 12 ⇒ y = 120. A is sufficient.
Using statement B, we have SP = (MP + ∴Using both the statements we can find Using statement B, increase in profit
CP)/2 that is SP – CP = MP – SP ⇒ dis- the area of the triangle as its base will = 20/100y
count = profit. be diameter and height will be equal to = 24 ⇒ y = 120. B is sufficient.
B is sufficient. Choice (1) radius of the circle. Choice (3)  Choice (2)

08_Section.indb 108 10/30/2009 11:53:41 AM


Solutions for Data Analysis and Data Sufficiency  n  109

Practice Exercise 4 By knowing only sum or difference of the From statement (B), x can be 3, 6, 9, 12
roots, we cannot find the quadratic equa- ……
Solutions for questions 1 to 15: tion. Hence we require both statement statement (B) alone is not sufficient
1. If a number is divisible by 25 and 3 then (A) and (C), which give sum as well as From statement (C), x is not divisible by
it is divisible by 75 difference of the roots. Choice (3) 2, so x is an odd number. Statement (C)
From statement (A), we have x is not di- alone is sufficient.
6. Considering statements (A) and (B), as
visible by 15. As 15 is a factor of 75 defi- Using statements (A) and (B), x is a prime
A’s speed is 20 kmph, B’s speed is 20 +
nitely it is not divisible by 75. Statement number and divisible by 3 so x = 3. Us-
10 = 30 kmph (that is, B’s speed is 50 per
(A) alone is sufficient. ing statements (A) and (B) we can answer.
cent more than A).
Statement (B) alone is not sufficient as we  Choice (3)
Considering statements (A) and (C)
do not know whether x is divisible by 3 or (B) and (C) speeds of A and B are
or not. 12. Statement (A) alone is not sufficient as
20 kmph and 30 kmph. we do not know the principal.
Statement (C) alone is not sufficient as we ∴any two of the three statements are re-
do not know whether x is divisible by 25 From statement (B), let sum
quired to answer the question.Choice (3) = Rs x, then amount = 2x
or not.
Using statements (B) and (C), x is divis- 7. Let shares of P, Q and R be x Rs x, Rs y PTR
Using I = , we have (2x − x)
ible by 25 and 12 so x is divisible by 25, and Rs z. Then x + y + z = 1200 100
3 and 4. So x is divisible by 75. Using From statement (B), we have y = 1/3 (x
x.4.R
statements (B) and (C) we can answer the + z) ⇒ y = 300 Statement (B) alone is =
sufficient 100
question. Choice (4)
From the above it is clear that, we can find ⇒ R = 25 per cent p.a. So B alone is suf-
2. Let Raju’s, Sonu’s and Bheemu’s ages be that we can find the share of the person ficient
R, S and B respectively. whose share is mentioned in terms of the From statement (C), by knowing only
None of the given statements is indepen- other two. Hence using both (A) and (C), principal we can not find the rate of in-
dently sufficient as they give only partial we can find the shares of P and R respec- terest.
information. Combining all the three tively using which we can Q’s share. Using Using statements (A) and (C), as we
statements, we have statements (A) and (C) also we can answer. know sum, interest and time period we
S + 10 = 2(B + 10) ------------------(1),  Choice (2) can answer the question. Choice (2)
R − 10 = 2(S − 10)------------------(2) and
R = 5B, ------------------ (3) 8. Cost of levelling the path = Area of path × 13. Let speed of boat in still water and speed
Solving (1), (2) & (3) we get ∴R = 50 cost per m2 of stream be x kmph and y kmph respec-
yrs, S = 30 yrs and B = 10 yrs Using statements (A) and (B): Area of path tively.
Hence, all the three statements (A), (B) = π(82 − 62) = 88 m2. Hence, cost of levelling From statement (A), we have x − y = 8
and (C) are required to solve the problem. = 88 × 5 = Rs 440. From statement (B), we have 5 (x − y) =
 Choice (4) Similarly, using statements (B) and (C) 4 (x + y)
only or (A) and (C) only, the problem can ⇒ x = 9y
3. Any one of the statements alone is not be done.  Choice (4) From statement (C), we have
sufficient as we do not have the informa-
9. Length of the diagonal of a cuboid d d
tion about all the three persons in one : =5:4
statement. Using statements (A) and (C), x−y x+y
= l2 + b 2 + h 2
working together (A) and (B) can do the None of the statements is independently ⇒ x + y : x − y = 5 : 4 ⇒ x = 9y
work in 12 days but (C) alone can do the sufficient as they do not give values of l, Combining statements (A) and (B) or
work in 10 days. So (C) is more efficient. b and h. (A) and (C), we can answer the question.
Using statements (A) and (C) we can an- Combing (A) and (B) we have ℓ + b + h =  Choice (3)
swer. Using statements (A) and (B) OR 26 and 2(ℓb + bh + hℓ) = 281. 14. Let first term and common difference of
(B) and (C), we can not answer the ques- As (ℓ + b+ h)2 = ℓ2 + b2 + h2+ 2(ℓb + ℓh + the progression be a and b respectively.
tion as we do not have the information hℓ), we can find ℓ2 + b2 + h2. Then 8th term = a + 7d
about (C) or (A) respectively. Choice (2) But combing (A) and (C) or (B) and (C), From statement (A), we have a + 5d = 17
4. Using statement (A), Pass mark as we can not find b and h values, ℓ2 + b2 + From statement (B), we have (a + 2d) +
P = 40T/100 + 10 ------------------ (1) h2 cannot be determined. Choice (2) (a + 4d) =
Using statement (B), Pass mark 10. M1 = 80; D1 = 40; H1 = 6; W1 = 1. M2 = ?, if 22 ⇒ 2a + 6d = 22 ⇒ a + 3d = 11
P = 50T/100 − 50 ------------------ (2) W2 = 31. From statement (C), we have (a + d) +
From (1) & (2), T = 600 and P = 250 Considering statement (A), H2 = 9 (a + 13d) = 46 ⇒ 2a + 14d = 46 ⇒ a +
Using statement (C), First rank marks In order to have D2, consider both the 7d = 23
= P + 100 per cent of P, 240 + 240 = 480 statements (B) and (C). D2 < 51 and D2 > So from statement (C) alone we can find
Hence, all the three statements are re- 49 ⇒ D2 = 50 8th term.
quired to solve the problem. Choice (2) ∴80 × 40 × 6/11 = M2 × 50 × 9/31 Combining statements (A) and (B), we
5. From statement (B), product of roots ⇒ M2 = 128 Men. have a + 5d
is 17. So the roots must be 17 and 1 as All the three statements are required to = 11----- (1) and a + 3d = 11----- (2)
the roots are positive integers. We know answer the question. Choice (4) Solving the two equations we can find 8th
the roots so we can frame the quadratic 11. From statement (A), x can be 2, 3, 5 or term. Choice (4)
equation. any other prime number. Statement (A) 15. Let speed of train be x m/s and length of
Statement (B) alone is sufficient. alone is not sufficient. the train be y m. Then

08_Section.indb 109 10/30/2009 11:53:41 AM


110  n  Solution Manual

200 + y ∴ Any one statement can be dispensed 22. Statement (A): Expenditures of A and B
x= ⇒ 28x = 200 + y ----- (1) with. Choice (4) are not given hence the given question can
28  not be answered.
18. From statement (A), we know the relation
Statement (B) in this statement we do not
From statement (A), x = y/8 ⇒ 8x = y between A and B, B and D so we can find
have information about B.
 ----- (2) the relation between A and D. So we can
∴ Statement (C) alone is not sufficient to
Solving equations (1) and (2) we get the find by what percent A’s salary more/less answer the given question as there is no
value of x. than D’s salary. A alone is sufficient information on incomes.
So statement (A) alone is sufficient. Either statement (B) or (C) is not sufficient When (A) and (C) are combined the ques-
y + 350 as we tion can be answered. Hence only (B) is
From statement (B), x =
43 do not have the information about A. (B) redundant. Choice (3)
⇒ 43x = y + 350  --- (3) and (C) are redundant. Choice (3)
23. To make the comparison of x and y, either
Solving equations (1) and (3) we get the 19. Statement (A): let the number of students (A) should be combined with (C) or (B)
value of passed and failed be respectively 3x and with (C).
x statement (B) alone is sufficient. 2x. From (A), as x can be positive or nega-
From statement (C) as we do not know As we do not know the value of x the tive and the same is true for y from (C).
the speed given Hence we can not find which is greater.
of the person so we can not find speed of question can not be answered. From (B) and (C), we have x3 > 64 ⇒ x
train. Choice (3) Statement (B) alone is not sufficient as > 4. But as y3 < 27 ⇒ y < 3. ∴ x > y.
there is no  Choice (1)
Solutions for questions 16 to 25:
information about failed. 24. None of the statements are independently
16. From statement (A), x = 54k + 40 ; Where When (A) and (B) are considered, we have sufficient to answer the given question.
k is quotient when x is divided by 54.
3x +12 21 When all the three statements combined,
When 54k + 40 is divided by 18 the re- = . From this we can calculate then we have S = 2P; Q = 2T and S =
mainder is equal to the remainder when 2x −12 10
30 + T. Solving them we can find whose
40 is divided by 18. (as 54k is divisible x. is older between P and Q. P – Q = 3
by 18) So statement (A) alone ∴ All the three statements are required.
is sufficient. (C) alone is not sufficient. As there is  Choice (4)
From statement (B), as 12 is not a mul- no information about passed or failed; Even
tiple of 18, we cannot answer the question if we combine (A) and (C) or (B) and (C), 25. As statement (A) gives a = b and (B)
from this. we cannot find the number of students gives r = 2, combining (A) and (B), we
passed. can find ar7.
From statement (C), 2x = 18P + 8 where
 Choice (4) Using statements (B) and (C), we have r =
P is the
2 and ar2 = 24. a (2)2 = 24 ⇒ a = 6
quotient when 2x is divided by 18.
20. From question statement, we know the Using statement (B) and (C) we can
P  P −1 time taken by A even A and C. Hence we answer the question.
So x = 18   + 4 or x = 18   + 13
2  2  can find the time taken by C. So statement (A) or (C) is redundant.
So we can not get the remainder when x From statement (A), as we know time  Choice (3)
is divided by 18. Statement (C) alone is taken by (B) and (C), we can find the
not sufficient. time that (B) takes.In the same way from
Using statements (B) and (C), when x is statement (B) as well as from (C), we can Practice Tests
divided by 12 the remainder is 4 so x is answer the question. Choice (4)
even number.
21. When the roots are α, β then the TEST PAPER 1
Using both the statements we can answer
quadratic equation is given by x2 − x
the question
(α − β) +αβ = 0
So either statement (A) or both (B) and Solutions for questions 1 to 5:
From (A), we have α + β = 11;
(C) are redundant.  Choice (2) 1. The number of distinctions increased by
From (B), we have α − β = 11;From (C),
three times from 2002 to 03, and in all
17. Statement (A) alone is not sufficient ef- we have αβ = 18
other years, it is not even twice. In the
ficiency of Ram is not mentioned Using statements (A) and (B), we have
year 2003 there is maximum percentage
Statement (B) is not sufficient as α + β = 11 --- (1) and α − β = 7--- (2).
increase. Choice (4)
work share of Ram and Shyam is not Solving the two equations we get α and β.
mentioned. So we can frame the quadratic equation 2. In 2004 the number of students got a per-
Statement (C): Efficiency of Ram is not Using statements (A) and (C), we have α + β centage between 51 per cent to 70 per
mentioned = 11 and αβ = 18 So we can frame the qua- cent = 155 – 20 = 135
Combining (A) and (B); let Shyam takes x dratic equation. ∴ Required percent = 135/265 × 100 ≅ 51
days to complete the whole work. 1/20× 10 Using statements (B) and (C), we have α per cent Choice (2)
+ 1/x× 14 = 1 − β = 7 and αβ = 18. 3. Number of students appeared for the
⇒ x = 28 days ∴ (α + β)2 = 72 + 4 (18) = 121⇒ (α + β) = examination in the year 2001 = 180
∴ work done by Ram in 10 days can be +11 or −11 Number of students appeared for the
calculated So unique value of (α + β) is not possible, so examination in the year 2005 = 270.
Combining, (B) and (C) or (A) and (C) we can not get unique quadratic equation. ∴ Average annual percentage increase
the part of work completed By Ram can So statements (B) or (C) is redundant. = 50/4 per cent that is 12.5 per cent
be calculated. Choice (3)  Choice (4)

08_Section.indb 110 10/30/2009 11:53:42 AM


Solutions for Data Analysis and Data Sufficiency  n  111

4. Total number of students who got less ∴ Required average = 3500/7 = 500 19. Let us assume that the number of persons
than 51 per cent marks in five years  Choice (3) recruited in 2003 is 100 in 2004, it is
= 80 + 70 + 100 + 170 + 120 = 480 13. The ratio of number of students who 2000.
∴ Their correct value = 480/120 × 100 passed the exam in first and third class is 30 −10
∴ Required average = 400/5 = 80 ∴ Required percentage = × 100
220:250 10
 Choice (1) ∴ Number of students who passed the = 200 per cent Choice (1)
5. Required percentage less = 10/80 × 100 exam in third class = 250/220 × 1100 =
= 12.5 per cent. Choice (3) 1250 Choice (1) 20. According to the given data, the number
14. Let us assume that the total number of of persons recruited in 2003 = 10000
Solutions for questions 6 to 10: Required ratio = 2000/1800 = 10 : 9
students passed from each city is 100.
6. Only for Company B the ratio of annual ∴ The number of students who passed  Choice (4)
profit to annual expenditure is crossing the exam in second class from cities A, Solutions for questions 21 to 24:
1.5, for all others it is less than that. ∴ it C, D and E
is maximum for company B Choice (2) = 50 + 30 + 20 + 40 = 140 21. Number of students failed in schools A

(156 −125) The number of students passed the exam  66 44 


and D = 2200 ×  +  that is, 22
7. Required percentage = × third class in cities A, C and D = 30 + 20  100 100 
100 125
+ 60 = 110
× 110 = 2420.  Choice (2)
= 24.8 per cent ≃ 25 per cent Choice (3) 140 −110
∴ Required percentage = × 100
8. Let us calculate Bonus as a percentage of 110 22. Required ratio = 8 × 52 : 13 × 42 = 16 : 21
profit. = 27.27 per cent Choice (1)  Choice (1)
Company A = 34/152 × 100 = 22.36 per 23. Let us assume that the no. of students ap-
cent; Company B = 52/225 × 100 = 23.11 15. Required ratio = 20 : 30. i.e 2 : 3 Choice (4) peared in exams in 2006-07 in school C =
per cent Company C = 35/168 × 100 = 120 ∴ In previous year = 100
Solutions for questions 16 to 20:
20.83 per cent; Company D = 31/148 Number of students failed in 2006-07 in
× 100 = 20.94 per cent Company E = 16. Let us assume that the total number of
people recruited in 2003 is 1000. ∴ in 70 × 120
23/118 × 100 = 19.49 per cent school C = = 84
∴ It is highest for company B.Choice (2) 2004 it is 1500 100
∴ Required percentage
9. Required percentage = 22/148 × 100 =  Number of students passed in 2006-07

14.86 per cent Choice (4) 150 +150 = 120 − 84 = 36
= × 100 = 12 per centChoice (2)
2500 ∴ Number of students passed in 2005-06
10. Required percentage for Company B
= 125/350 × 100 = 35.71 per cent. Which = 30, ∴ Number of students failed = 70
is less than 40 per cent for all others it (375 −150) ∴ Required ratio = 70 : 84 = 5 : 6
17. Required percentage = × 100
is atleast 40 per cent. ∴ It is lowest for 150  Choice (4)
company B. Choice (2) = 150 per cent Choice (4) 24. Suppose, the total number of students ap-
Solutions for questions 11 to 15: 18. Let us assume that the total number of peared in the given six schools is 10000.
Let us represent the given data in the form of students recruited by Infosys in 2003 is Number of students in school B = 10000
the table. 100. × 20/100 and failed from B = 2000 ×
∴ in 2004, it recruits = 120. 52/100 = 1040.
A B C D E F G
∴ Total number of students recruited Similarly, number of students failed in F
First class 20 40 50 20 30 20 40
in 2003 and 2004 is 1000 and 2400 re- = 2500 × 55/100 = 1375 ∴ Required ratio
Second class 50 20 30 20 40 40 30
spectively. 1040
Third class 30 40 20 60 30 40 30 = = 0.75 Choice (1)
1375
11. Let us assume that 100 students passed 2400 −1000
∴ Required percentage =
from each city. 1000 Solutions for questions 25 to 28:
× 100
∴ Number of students who passed Let us represent the given information in tabular
= 140 per cent Choice (3) form
in exam in first class in all the cities put
together = 220
Number of students who passed in exam A B C D E F G H
second class in all the cities put together
Total income 20000 24000 22000 18000 16000 26000 22000 16000
= 230
Required per cent = 220/230 × 100 = House hold expenses 14000 12000 12000 10000 6000 16000 10000 8000
95.65 per cent Choice (3) Education expenses 4000 8000 6000 2000 8000 6000 8000 4000
12. It is given that, The number of students Savings 2000 4000 4000 6000 2000 4000 4000 4000
who passed in exam in second class in
city D = 280
25. By observation of the above table, we
∴ Total number of students who passed 4000
can say that the ratio of savings to educa- Required ratio for C = = 0.67
from city D = 1400, which is the same for 6000
tion expenses is one or more than one for
all the cities. 2000 4000
families D and H. It is 0⋅5 for families A,
∴ Total number of students passed the E= = 0.25 ; F = = 0.67
B and G. 8000 6000
exam in third class = 3500

08_Section.indb 111 10/30/2009 11:53:42 AM


112  n  Solution Manual

 For families C, D, F and H the ratio is


∴  sing statement A, x2 and y have the
U
3
more than 0⋅6. Choice (1) same sign. x2 > 0 which means x could be x> y +3
2
positive or negative.
26. Savings per cent of
∴ > 0. A is not sufficient. 3
4000 ⇒∴x> y ⇒ x > y ---------- (1)
B= × 100 = 16.67% Using statement A, x3 and y2 have the 2
24000 same sign. y2 > 0 which means y could be Since we do not know the relation be-
Savings per cent of positive or negative. tween x and z. A is not sufficient to an-
4000 ∴ x3, y2 > 0 that is x > 0. B is not suf- swer question.
H= × 100 = 25% ficient.
16000 From Statement B, 5y > 3z + 1y; y >
Using both statements, x, y > 0. Both
3 1
Required difference = 25 − 16⋅67 = 8⋅33 statements taken together are required to z +
per cent. Choice (1) 5 5
answer the question. Choice (3)
2
27. Required per cent =
(6000 − 2000 ) × 100 33. Using statement A, B (B − 1) = 0. ∴ B = ⇒y>z−
5
z
2000 0 or 1
In any case, B3 = B. A is sufficient. Us-  ince we do not know the relation be-
S
4000 ing statement B, if B > 0 ⇒ BB > B and tween x and y, ∴ statement B alone is not
= × 100 = 200% . Choice (1)
2000 if B < 0 sufficient.
⇒ BB > B or BB < B. By combining both the statements also
28. By observation, we can say that the per-
∴ B3 > B only when B > 0 and B3 < B we cannot answer the question. Choice
centage of household expenses for families
when B < 0 (4)
B, E, G and H is 50 per cent or less than 50
∴ B alone is not sufficient. Choice (1)
per cent of their respective incomes. Solutions for questions 37 to 40:
House hold expenses as a percentage of 34. There are more white balls than black
37. Ι. 2x + 3y = 8 → (1);  ΙΙ.  5x + 4y = 13
14000 balls in a total of 12 balls.∴ There are
income for A = × 100 = 70% → (2)
20000 more than 6 white balls and less than 6
2x + 3y = 8 → (1) × 5; 5x + 4y = 13 →
black balls. Let us say there are w white
(2) × 2
12000 balls and b black balls.
C= × 100 = 54.54% The above equation will become
22000 w 2
Using statement A, ≤ ⇒w≤8 10x + 15y = 40 → (3); 10x + 8y =
10000 12 3 26 → (4)
D= × 100 = 55.55%
18000 ∴ w = 7 or 8; ∴ b = 5 or 4 On solving (3) & (4), we get y = 2; x = 1;
b 1 y > x Choice (2)
16000 Using statement B, ≥ ∴ b ≥ 4;
F= × 100 = 61.53% 12 3 38. Ι. Factorising x + 14x + 40 = 0
2
26000
∴ b = 4 or 5 x2 + 10x + 4x + 40 = 0
∴ For families A, D and F, it is more than are more U
34. There white
singballs
boththan
theblack balls in awe get b =
statements x(x + 10) + 4(x + 10) = 0
55 per cent. Choicetotal
(2)of 12 balls.?
4 There
or 5. are
When more thanthe
both 6 white
statements com-
39. ,. x = ( 7 ) / 3 + 10) (+
5(x
; x=
5
7)(x
;+ 4)
x ==70;
2
; x =x49 = – 10 or
balls and less than 6 black balls. Let us say there ( 7 )3
bined together
are w white balls and b black balls.also we can not answer the x = – 4
Solutions for questions 29 to 31: 1 2 1
question.
w 2 Choice (4) ,,. y = 2–5; ΙΙ. Factorising
y=
1
= 3 y 2 += 15y + 50 ,y ==0 ;
1
Using statement A,
29. For Congress, No. of votes cast in 1999- d Ÿwd8
12 3 y2 + 10y25 + 2 5yu+250 =80u 4 32
35. Since angle of elevation is not given in y = 0.03125;
2000 = 4,88,000 × 30/100 ? w = 7 or 8; ? b = 5 or 4 y(y + 10) + 5(y x > y+ 10) = Choice
0 (1)
statement A.
No. of votes cast in 1998-99 b 1 (y + 10) + (y + 5) = 0; y = – 10 or y = – 5
146400 × 100
Using statementSince
B, t ? boft the
heights
12 3
4; ? b = 4 are
towers or 5not given
1x = y 1 y > x;
4 y ≥2 x.
 4  Choice
2
(4)
=
120
= 122000 Statement we
Using both the statements B alone is4also
get b =
40. ,. x =
or 5.not sufficient. 4 2 4 2
 =

4 2 4 2
that is B combined
When both the statements are combined,
togetherwe alsoget AB = (7) 5

∴ Required difference = 146400 − not answer


we can the question. Choice (4) 39.
 2 2Ι. x = 2 (7) ;
5/3
x= ; x = 72;
80m; = = (7)3
14 7
122000 = 24400 Choice (2) CD = 62.5m. ∠EDB
and in = 45°.
35. Since angle of elevation is not given statement A. 0
x = 49
,,. y = 3 = 1; y > x. Choice (2)
30. No. of valid votes cast for BJP Since
= heights of the towers are not given
Statement B alone is also not sufficient.BE dATA ANALYSIS–5ANd SuFFICIENCY 1 1 1
24 × 4,88,000 In ∆ DEB,
that is B are combined, we get 45°= =80m; ⇒ DE = BE
tanAB ΙΙ. y = 2 ; y = = 3 =
= 117120 Choice (4)
CD = 62.5m. and ‘EDB = 45°.
DE SOLuTIONS FOR TEST PAPER
25 2 × -
2 2
2
8 × 4
100
In ' DEB, tan 45° B Solutions for questions 1 4:
31. Number of votes cast for BSP in 1999-=
BE
Ÿ DE = ,y = 1 to
; y = 0.03125; x > yChoice
DE 1. S had the lowest
32
percentages of profit growth
4,88,000 D 45° (1)
2000 = × 100 = 48,880 BE x
E
80 each year over the given period. ? The
100 = AB – AE = 80 – percentage growth over the 1 given period
1 must be
62.5 = 17.5 62.5 40. for
Ι. S. x = −
∴ No. of votes cast for BSP in 1998-99 = the lowest
4+ 2 4− 2
Choice (3)
? The distance
A
4,88,000 × 100 between two C 2. S’s profit in 2004 = 13.915
(1.15) (1.1) (1.1)
80
= 61000 towers is 17.5.
When combined=both AB the
– AE = 80 – 62.5
statements = 17.5
we can
=
(
= $10million. 4 − 2 − 4 + 2 Choice (2) )( )
∴ Total votes cast in 1998-99 = 6,10,000

answer the question.
Choice (3)
Choice two
∴ The distance between
17.5.
(3) towers is
3. The profits of P and 4 +R in
2 2005 ( )(
4 − are2 in the ratio )
25 25
3 (22) : (23) = 22 : 23
36. From statement WA;hen
2x !combined ! ythe
3y  6; x both  3statements we100 100−2 2
Solutions for questions 32 to 36: 2 − 2
can answer the question. Choice Ratio of their= profits in =2007
(3)
3 14 7
Ÿ ? x ! y Ÿ x ! y ---------- (1)
32. When the product of two numbers is posi- § 15 · § 30 ·
236. From statement A; 2x > 3y + 6; 22 ¨¨1 ¸¸ ¨¨1 1000¸¸
tive, both must have the same sign. Since we do not know the relation between x and © 100 ¹ © ¹
ΙΙ. y = 3 = =1; y
= 143>: x.
150 Choice (2)
z. A is not sufficient to answer question. § 20 · § 25 ·
23 ¨¨1 ¸¸ ¨¨1 ¸¸
3 1 © 100 ¹ © 100 ¹
From Statement B, 5y ! 3z  1y; y ! z 
5 5 Choice (1)
2 4. Let the profit of S in 2005 be Rs x million. Its
Ÿy!z z
5 § 10 ·
08_Section.indb 112 10/30/2009 11:53:43 AM
Since we do not know the relation between x and profit in 2006 = x ¨¨1  ¸¸ = Rs 1.1x million
Solutions for Data Analysis and Data Sufficiency  n  113

Total supply of all the products  otal population of the states in 1998 –
T
TEST PAPER 2 99 = 5x, Required percentage = 3.2x/5x
= 125 + 135 + 125 + 115 = 500
∴ Excess of demand = 0 tonnes.  Choice (100) = 64 per cent
Solutions for questions 1 to 4: (2)  Choice (2)
1. S had the lowest percentages of profit Solutions for questions 13 to 16:
8. Minimum amount required = 40 × 15 ×
growth each year over the given period. 13. Using statement A, let Dilpreet and Man-
[35 + 30 +25 + 40] = 40 × 15 × 130 =
∴ The percentage growth over the given preet received
78000 Choice (2)
period must be the lowest for S. Choice 10,000+0.5x and x votes. ⇒ 10,000 +
(3) Solutions for questions 9 to 12: 0.5x + x
2. S’s profit in 2004 = 9. Let the total population of the states in = 100000 ⇒ x = 6000
1998 – 99 be T. Total population of the ∴ Manpreet received 60,000 votes and
13.915 states in 1999 – 00 = T. Dilpreet received 40,000votes.
(1.15) (1.1) (1.1)
Percentages of literates did not change in Statement B is not sufficient as there are
= $10million. Choice (2) each state from 1998 – 99 to 1999 – 00. no details about the votes received by
3. The profits of P and R in 2005 are in the ∴ if the population of any state in- Manpreet.
creases from 1998 – 99 to 1999 – 00,  Choice (1)
25 25 the number of literates in it would in- 14. Let the cost price of A be Rs y.
ratio (22) : (23) = 22 : 23 crease from 1998 – 99 to 1999 – 00.
100 100 Using statement A, decrease in profit =
Ratio of their profits in 2007 3 4 2 3 1 2 4 5 (10/100)y = 12, y = 120.
T= T, T < T, < T, T > T Using statement B, increase in profit =
15 20 15 20 15 20 15 20
(20/100)y = 24, y = 120.
 15  30  5 6 Either of the statements is sufficient.
22 1+ 1+  and T> T.
 100  100  = 143 : 150 15 20  Choice (3)
=
 20  25  ∴ The populations increased for 2 states. 15. Part of the job completed by P = 6/20
23 1+ 1+ 
 100  100   Choice (2) = 3/10;
Remaining part = 7/10
 Choice (1) 10. Let the total population of the This will be completed by Q and R. Parts
5 states be x. Required ratio = of the job completed by Q and R will be
4. Let the profit of S in 2005 be Rs x million.
1 1
70  20  50  30  in the ratio : =y:x
 x :  x x y
 10  100  100  100  100 
Its profit in 2006 = x 1+  = Rs 1.1x Using statement A, x/y = 4/3 ⇒ Parts of
 100 
= 14 : 15  Choice (1) the remaining part done by Q and R can
million be found. A is sufficient.
11. Let the population of S in 1998 – 99 be x. Using statement B, y is unknown.
 15  Population of T in 1998 – 99 = x ∴ Parts of the remaining job done by Q and
I ts profit in 2007 = 1.1x 1+  = Rs R cannot be found. B is not sufficient.
 100 
 20   Choice (1)
Population of S in 1999 – 00 = 1+ 
1.265x million. ∴ Its profit increased by  100  16. Let the speeds of Manish’s boat in still
 20  water and the river be x kmph and y kmph
1.265x − x 1+  x
(100) = 26.5 per cent  100  respectively.
x
over these two years. ∴ average percent- = 1.2x; Population of T in 1999 – 00 = Average speed =

26.5%  20  (12)(2) (x + y)(x − y)


age growth = = 13.25 per cent 1+  x = 0.8x. =
2  100  12 12 x
 Choice (1) +
x+y x−y
 umber of illiterates in S in 1999 – 00
N
Solutions for questions 5 to 8: x
= 25/100 (1.2x) = 0.3x and that in T = Using statement A, = 2 i.e x = 2y
5. As W and X are in a position to supply 35/100 (0.8x) = 0.28x y
atleast one product to each of P, Q, R and Required ratio = 0.3x : 0.28x = 15 : 14 (2y + y)(2y − y)
S, we can say that these are optimum sup- ∴ average speed =
 Choice (1) 2y
pliers. Choice (3) 3 y kmph
12. Let the population of each state in 1998 – =
6. From godown x, the supply of at least 2
99 be x. y is unknown
three of the products A, B, C and D meets Total number of literates in the 5 states in
the demand at R and hence can be trans- ∴ average speed cannot be found A is not
ported to R. Choice (2) sufficient.
70 60 50 Using statement B, x = 8, y is unknown.
1998 – 99 = x+ x+ x+
7. Total demand of A: 35 + 40 + 40 + 35 = 100 100 100 ∴average speed cannot be found. B is not
150. sufficient
75 65
New demand for all the products x+ x Using both statements, y = x/2 = 8/2 =
100 100
= 50 per cent of 150 + 145 + 140 + 140 4. The average speed can now be found.
= 500 = 3.2x  Choice (4)

08_Section.indb 113 10/30/2009 11:53:44 AM


114  n  Solution Manual

Solutions for questions 17 and 18: ∴ Required ratio 45 × 80 × 75 per cent × 5k :  hich is more than 200. ∴ There must be
w
17. R is not a square. ∴ Its length is more 180 × 75 × 80 per cent x 8k = 5:32 at least 38 dropouts.  Choice (1)
than its breadth. Let the side of S be a. Let  Choice (4) Solutions for questions 33 to 36:
the length and the breadth of R be ℓ and b Solutions for questions 24 to 27:
33. The amount spent on education in 1998 =
respectively. 24. In none of the years is the domestic con-
105 per cent of 90 = 94.5°.
Using A, a2 = ℓ b < ℓ2. ∴ a < ℓ. sumption more than the total of Import
The percentage decrease in entertainment
1 1 and Export.
+ = 4.5/20 × 200 = 22.5 per cent ∴ x = 22.5
1 a a  Choice (4)
Using statement ΙΙ, = l b ⇒ + , per cent Choice (4)
∴ a < ℓ. a 2 l b 25. The per cent change in exports is the
34. Savings of Amol in the year 2006 9/360 × x
highest in 2003 as the change is half of
= 1,35,000 ⇒ x = Rs 5,40,000
∴ Either statement is sufficient. Choice that of 2002 that is, 50 per cent increase.
(4)  Choice (2) 50 40
The required difference = x− x
18. A quadrilateral is cyclic only if, any pair 26. Domestic consumption for 2002: 50 + 25 360 360
of its opposite angles are supplementary. − 30 = 45 million tones. For 2003 : 45 + 10x 10
∴ ABCD is cyclic if ∠A + ∠C = ∠B 50 − 45 = 50 million tones. that is = x(5,40,000) = 15,000
 360 360 Choice (4)
+ ∠D ∴ The percentage change in domestic
Using statement A, suppose ∠A = ∠C = consumption from 2002 to 2003 35. Amol’s savings in 2006 = 90°
60o and ∠B= ∠D = 120o. Then ∠A + ∠D = 5/45× 100 =111/9 per cent Choice (1) Entertain amount in 2006 = 50°
= ∠B + ∠C 27. Domestic consumption is more than ∴ New savings = 150 per cent of 50° = 75°
but ∠A + ∠C ≠ ∠B + ∠D. In this case Production in each of the years in which ∴ Present savings should be reduced by
ABCD is not cyclic, If ∠A = ∠C = 90; ∠B Imports are more than Exports. This hap- 15°
= ∠D = 90 then ABCD is cyclic. ×100 = 16.66 per cent Choice (3)
pened in 2001 and 2003. Choice (2) 90°
A is not sufficient. B is sufficient. Choice (1)
Solutions for questions 28 to 32: 36. There are only two items shown increase
Solutions for questions 19 to 23: 28. As there are no drop outs, total number of from 2006 to 2007, they are food and edu-
19. Seating capacity of all the airlines = 100 students in that batch from 2003 to 2007 cation.
[50 × 180 + 45 × 80 + 50 × 180 + 60 × 180 = 37 + 25 + 17 + 15 + 4 = 98 But as food’s rate of increase is more,
+ 75 × 180 + 30 × 80 + 40 × 80 + 30 × 80 ∴ The maximum number of students who food will have more percentage increase.
+ 20 × 180] = 57,50,000 Choice (3) can take admission in class V in that batch =  Choice (3)
20. Let Rs x be the amount charged by 200 − 98 = 102  Choice (4)
Solutions for questions 37 to 40:
each of the airlines on business class in 29. Observing the table, the number of stu- 37. Ι. Factorising 26x2 – 43x + 15 = 0
2006-07. dents admitted in class VΙ has increased ⇒ x = 15/13 or x = 1/2
As the Indian airlines and Kingfisher car- by maximum of 10 in 2007 from 2006 ΙΙ. Factorising 2y2 + y – 3 = 0
ried 10 per cent and 20 per cent of the and it is 10/36 × 100 > 25 per cent, and ⇒ y = 1 or y = −3/2; Choice (4)
total traffic, the ratio of revenues cannot all others are less than this.
be determined as the ratio of charges for  Choice (3) 38. Ι. Factorising 2x2 + 13x + 18 = 0
economy and business class is not known. ⇒ x = –2 or x = −9/2;
30. Total number of students who took ad-
 Choice (4) ΙΙ. Factorising y2 + y – 12 = 0
mission in, 2003 = 150 + 37 + 23 + 20 + ⇒ y = 3 or y = – 4; Choice (4)
21. The total aircraft in 2006-07 = 400/ 10 + 15 = 255
0.8 = 500 2004 = 156 + 40 + 25 + 15 + 9 + 7 = 252 5
5 5 
5
2005 = 163 + 41 + 22 + 17 + 2 + 6 = 251 53 × 52
∴ Average number of people carried per 39. Ι. x =   =  5  = 3
2006 = 169 + 36 + 29 + 23 + 15 + 9 = 4 4  4 × 42
aircraft
281 125 × 25 3125
2007 = 190 + 46 + 22 + 14 + 4 + 4 = 279 = = = 3.0517
110% of 110% of 6 Crores 64 × 16 1024
= ∴ In the year 2005 the number of
500 × 200
students who took admission is minimum.
ΙΙ. y = 30 + 2 ⇒ 30 > 4.
11 × 11 × 6 × 105  Choice (2)
= = 726 Choice (3) ∴ y > 4 Choice (2)
31. Percentage decrease in total number of
105
students in year 2003 to 2004 = 3/255 ×
22. Let the total air traffic in 2003-04 be x 100 per cent = 1.17 per cent 1 1
∴ Total air traffic in 2006-07 = (1.15) 40. Ι. x= − ,
Percentage decrease in total number of 7+ 2 7− 2
(1.1) (1.1) x students in year 2004 to 2005 = 1/252 ×
∴ Required percentage 100 per cent = 0.397 per cent
Again percentage decrease in the total x=
(7 − 2 )− (7 + 2 )
=
x
× 100 ≃ 72 per cent
number of students in 2006 to 2007 = (7 + 2 )(7 − 2 )
(1.15) (1.1) (1.1) x 2/281 × 100 per cent
= 0.712 per cent Choice (3) 7− 2 −7− 2 −2 2
= = ;X < 0
 Choice (2) 49 − 2 47
32. The number of students, who took admis-
23. As the number of trips is same, revenue sion in the batch who took admission in −
1
−1
 64  3  216 
depends on number of aircraft, seating class V in the year 2003 = 150 + 40 + 2 + ΙΙ. y =   ×   y>0 y>
23 + 3 = 238 x.  27   8 
capacity, occupancy ratio and fares. Choice (2)

08_Section.indb 114 10/30/2009 11:53:45 AM


Solutions for Intelligence
and Critical Reasoning 5
Series 15. The series consists of successive prime 30. The product of the digits in each number
numbers. The prime number after 61 is is added to the number to get the next
Practice Exercise 67. Choice (4) number in the series. Choice (3)
16. 5×1+1, 6×2+2, 14×3+3, 45×4+4, 184×5+5, 925 31. 253−18, (235)−16, 219−14, 205−12, (193)−10,
Solutions for questions 1 to 72:
 Choice (1) 183−8, 175 Choice (1)
1. The given series can be written as the
17. 21+2, 23×2, 46+2, 48×2, 96+2, 98×2, 196 32. 121+2, 123−3, (120)+5, 125−7, (118)+11,
squares of consecutive primes.
 Choice (3) 129−13, 116 Choice (2)
 Choice (4)
18. 2×2+3, 7×2+3, 17×2+3, 37×2+3, 77×2+3, 157 33. 121+(1+2+1), (125)+(1+2+5), 133+(1+3+3),
2. The differences are consecutive Odd
 Choice (2) (140)+(1+4+0), 145+(1+4+5), 155+(1+5+5), 166
numbers Choice (2)
19. 13×4−1, 51×4−1, 203×4−1, 811×4−1, 3243×4−1,  Choice (4)
3. The given numbers are the alternate
12971 Choice (2) 34. 5×2, 10+3, 13×2, 26+3, (29)×2, 58+3, (61)
prime numbers. Choice (2)
20. 5 , 6 , 12 , 15 , 60 , 65 , 390
+1 ×2 +3 ×4 +5 ×6  Choice (1)
4. 11+2, 13+2, 15+2, 17 is the odd number
series.  Choice (4) 1 1 1
× × ×
12−2, 10−2, 82, 6 is the even number series 21. (4)3, (6)3, (8)3, (10)3, (12)3, (14)3 = 2744 35. 12×5, 60 2
, 30×5, 150 2
, (75)×5, 375 2 ,
in reverse order series. Choice (1)  Choice (4) (187.5) Choice (1)

5. 12, 23, 32, 43, 52, 63, 72, 83 = 512 36. Every number starting with the third
22. The given series can be written as n2+1,
 Choice (3) number is the sum of its previous two
where n is a prime number.
numbers. Choice (1)
6. 124+16, 140+25, 165+36, 201+49, 250+64, 22+1, 32+1, 52+1, 72+1, 112+1, 132+1, 172
314+81, 395 +1 = 290 Choice (4) 37. 3×5,15+6, 21×7, 147+8, (155)×9, (1395)+10, 1405
where 16,25, 36, 49 and 64 are the per-  Choice (1)
23. 930 = 302 + 30, 812 = 282 + 28,702 = 262
fect squares of the consecutive numbers. + 26, 600 = 242 + 24, 506 = 222 + 22 38. 23−(2×3), 17+(1×7), (24)−(2×4), 16+(1×6),
 Choice (2)  Choice (1) (22)−(2×2), 18+(1×8), 26 Choice (3)
7. 17×2, 34×3, 102×4, 408×5, 2040×6 = 12240 24. (1)4, (2)4, (3)4, (4)4, (5)4, (6)4 = 1296. 39. 103+23, 126+23, (149)+23, 172+23, 195+23,
 Choice (2)  Choice (4) (218)+23, 241 Choice (3)
8. 43+1, 44+4, 48+9, 57+16, 73+25, 98 25. 6 = 22 + 2, 12 = 32 + 3, 30 = 52 + 5, 56 = 73 40. 8×3, 24+13, 37×3, (111)+13, 124×3, 372+13, (385)
 Choice (4) + 7, 132 112 + 11, 182 = 132 + 13  Choice (3)
9. The differences are perfect cubes in de- Prime numbers of the form of N2 + N.
41. M +3, P+3, S+3, V+3, Y+3, B+3, E Choice (1)
creasing order. Choice (1)  Choice (1)
10. The consecutive natural numbers are giv- 42. C+5, H+5, M+5, R+5, W+5, B+5, G Choice (1)
26. (2)1, (3)2, (4)3, (5)4, (6)5 = 7776. Choice
en in the numerators and their squares are (3) 43. The first letters are related as follows:
given in the denominators. Choice (2) B+1, C+2, E+3, H+4, L+5, Q.
27. 2 =1 + 1, 30 = 3 + 3, 130 = 5 + 5, 350 =
3 3 3

11. 13+1, 14+8, 22+27, 49+64, 113+125, 238+216, 73+7, 738 = 93+9 The second letters are C +1, D+2, F+3, I+4, M+5, R
454 The given numbers are in the form  Choice (2)
1 = 13, 8 = 23, 27 = 33 and so on of N3+N. Where N is an odd number. 44. The series consists of pairs of correspond-
 Choice (4)  Choice (2) ing letters from the two halves of the
12. 5×1, 5×2, 10×3, 30×4, 120×5, 600×6, 3600 28. The series consists of successive prime alphabet. So the next pair is ER.
 Choice (3) numbers. The prime numbers after 83 are  Choice (1)
13. 7×3, 21+1, 22×3, 66+1, 67×3, 201, 202Choice 89 and 97. Choice (3) 45. Series 1: M +2, O+3, R+5, W+7, D+11, O
(2) 29. The digits in each number are reversed Series 2: S+1, T+2, V+3, Y+4, C+5, H
14. 24×0.5, 12×1, 12×1.5, 18×2, 36×2.5, 90×3, 270 and the number so formed is added to the Series 3 : O+3, R+5, W+7, D+9, M+11, X
 Choice (4) original number. Choice (1)  Choice (3)

08_Section.indb 115 10/30/2009 11:53:46 AM


116  n  Solution Manual

46. The given series has two alternate series. 59. The 1st letters are in the following patter: Solutions for questions 73 to 76:
The first letters are S+3, V+3, Y+3, B T+3, W+3, Z+3, C+3, F If the alphabet is taken as it is, then from any
The second letters are A+3, D+3, G+3, J The pattern for the second letter: letter in the right ward direction place value in-
The other series is EH, OX, ST, EH A+3, D+3, G+3, J+3, M creases and on in left ward direction, the place
E×3, O×3, S×3, E The pattern for the third letter: value decreases.
H×3, X×3, T×3, H Choice (2) N+3, Q+3, T+3, W+3, Z Choice (3) 73. The place value of F is 6. The required
47. AX2, BX2, DX2, HX2, P 60. The first letters are consecutive vowels: letter is
J+2, L+2, N+2, P+2, R A, E, I, O, U 6 + 5 – 4 = 7 = G. Choice (4)
X−3, U−3, R−3, O−3, L Choice (4) The pattern for the second letter: C+3, 74. The place value of H is 8. The required
F+3, I+3, L+3, O
48. The first series is B , F , R , B×3, F.
×3 ×3 ×3
letter is 8 − 5 + 12 = 15 = O. Choice (4)
The pattern for the third letter: D+4, H+4,
The second series is D +3, G+3, J+3, M+3, 75. The place value of O is 15. The required
L+4, P+4, T Choice (2)
P. letter is 15 − 7 + 3 = 11 = K. Choice (3)
The third series is E×3, O×3, S×3, E×3, O. 61. The first series is: C , F , I , S , Z , Z
×2 +3 ×5 +7 ×11

The fourth series is H+3, K+3, N+3, Q+3, T. The second Series is D×2, H+3, K×5, C+7, 76. Yis third letter to the right ofV. Required letter
 Choice (3) J×11, F and the third series is F ×2, L+3, O×5, = 22 – 7 – 6 = 9 = I. Choice (3)
W+7, D×11, R Solutions for questions 77 to 80:
49. A+1, B+2, D+3, G+4, K
Thus the next term is ZFR. Choice (2) In the reverse order of the alphabet, the place
C+2, E+3, H+4, L+5, Q
D+3, G+4, K+5, P+6, V 62. This is the series of consonants in the re- values decrease in the rightward direction and
B+4, F+5, K+6, Q+7, X Choice (2) verse order. So, the letter in the blank is increase in the leftward direction.
‘s’. Choice (1) 77. The place value of P is 16. The required
50. The first series is I, O, U, A, E, I is a series
of consecutive vowels. 63. P8B ⇒ P ÷ B = 8; J2E ⇒ J ÷ E = 2; T5D letter is 16 – 4 + 2 = 14 = N. Choice (3)
The second series J, K, L, M, N, P is a ⇒ T ÷ D = 5; O5C ⇒ O ÷ C = 5 78. The place value of K is 11. The required
series of consecutive consonants.  Choice (2) letter is 11 + 1 − 5 − 1 = 6 = F.Choice (1)
The third series is also a series of con- 79. The place value of X is 24. The required
64. Series 1: B+2, D+2, F+2, H+2, J
secutive consonants. Choice (1) letter is 24 – 5 + 1 = 20 = T. Choice (1)
Series 2: (1)3, (2)3, (3)3, (4)3, (5)3
51. It is a series of vowels written in ascend- Series 3: A+4, E+4, I+4, M+4, Q 80. The place value of Y is 25. The required
ing first and then in descending order,  Choice (2) letter is 25 − 17 − 2 = 6 = F. Choice (1)
hence ‘e’ comes next. Choice (1) 65. In the given series, the place value of the
52. B+3, E+3, H+3, K+3, N letter is written on either side of the let-
J+3, M+3, P+3, S+3, V ter.
Analogies
The number at the middle represents the 1K1 ⇒ K = 11, 1L2 ⇒ L = 12
place values of the two letters.Choice (2)  Choice (2) Practice Exercise
53. B ,+1
C ,
+1
D ,
+1
E , F
+1
66. (i) B , C , A , B , Z , A , Y
+1 −2 +1 −2 +1 −2 Solutions for questions 1 to 30:
D−2, B−2, Z−2, X−2, V (ii) D+3, G−4, C+3, F−4, B+3, E−4, A 1. 756+234 : 990 : : 1238+234 : 1472
F+3, I+3, L+3, O+3, R (iii) The numbers in each term is square  Choice (3)
H−4, D−4, Z−4, V−4, R Choice (3) of the sum of the position values of
the letters. Choice (2) 2. Next prime number to 83 is 89. Simi-
54. The pattern for the first letter: H–4, D–4, A–4, larly, the next prime number to 67 is 71.
W–4, S 67. Series 1: B+2, D+4, H+6, N+8, V  Choice (2)
The pattern for the second letter: O–4, K–4, Series 2: (3)1, (3)2, (3)3, (3)4 (3)5
G–4, C 4, Y Series 3: Z−3, W−3, T−3, Q−3, N 3. 928–345 : 583 : : 1523–345 : 1178
The pattern for the third letter: V–4, R–4,  Choice (4)  Choice (2)
N–4, J–4, F Choice (4) 4. 1225 = 35 and 1225 − 35 = 1190
68. ab bc c cd d d d Choice (1)
Similarly, 1089 = 33 and 1089 − 33 = 1056
55. B , D , H , N , V , F
+2 +4 +6 +8 +10
69. a b cb c ac a ba b c  Choice (1)
A+1, B+3, E+5, J+7, Q+9, Z The three letters a, b and c are taken in
D+1, E−2, C+3, F−4, B+5, G Choice (2) 5. The given pair of numbers can be written
cyclic order. Choice (3)
as (n)2 : (n + 1)3 Choice (1)
56. T + 2 = V; T – 2 = R; O+ 2 = Q; O – 2 = 70. a b c a a b b c c a a a b b b c c c.
M; P + 2 = R; P –2 = N 6. (6)3 : (6)² : : (11)3 : (11)² = 121
By observation we can say that each of a,
Hence, TOP : VRQMRN and SET :  Choice (1)
b and c are taken once, twice, thrice re-
UQGCVR. Choice (4) 7. 405÷3 : 135 : : 729÷3 : 243 Choice (3)
spectively. Choice (3)
57. The first letters are E +2, G+4, K+6, Q+8, 8. The digits in the given numbers are reversed.
71. a a b ca b b ca b c c
Y+10, I  Choice (3)
The first 3 letters are taken but in the
The second letters are F+3, I+6, O+9, X+12, 9. 35×7 : 245 : : 65×7 : 455 Choice (1)
first group ‘a’ is given twice, in the sec-
J+15, Y ond group ‘b’ is given twice and so on. 10. 15 x 6 = 90 and 17 x 6 = 102
The third letters are: G+4, K+8, S12, E+16,  Choice (4)  Choice (1)
U+20, O Choice (1)
72. a ab b bc cd d de ef f f 11. The given pair of numbers can be written as
58. P–2, N–2, L–2, J–2, H The letters are taken twice and thrice al- n : n2 Choice (3)
L+2, N+2, P+2, R+2, T ternately Choice (4)
J–2, H–2, F–2, D–2, B Choice (3) 12. 8² : 83 : : 10² : 103 Choice (4)

08_Section.indb 116 10/30/2009 11:53:47 AM


Solutions for Intelligence and Critical Reasoning  n  117

13. Chennai is the capital city of Tamil Nadu, 32. The place value of each letter is multi- 9. Except ‘Dam’ in all the other water sourc-
similarly Jaipur is the capital city of Raj- plied by two Choice (3) es, water flows. Choice (2)
asthan. Choice (2) 33. For each vowel in the first term its pre- 10. All except ‘Football’ require a stick or a
14. Food is related to hunger, in the same way vious vowel is given in the second term. bat other than a ball. Choice (2)
water is related to thirst. Choice (3)  Choice (2)
11. Except ‘Tripura’ all are the capital cities
15. Doctor works in Hospital, similarly Chef 34. b c d b c d of the various states. Choice (3)
works in kitchen. Choice (1) +2 +3 +4 Similarly, –2 –3 –4
12. Except ‘9’ all the numbers can be written
d f h z z z
16. 23 × 7 : 161 : : 19 × 7: 133 in nn + 1 form. Choice (1)
 Choice (1)
 Choice (4) 13. All except ‘46’ are multiples of ‘7’.
35. In each term the position value of the let-
17. 6 – 6 : 4 – 4 : : 10 – 10 : 8 – 8 = 504
3 3 3 3
 Choice (1)
ter is equal to the product of the digits on
 Choice (2) either sides. Choice (3) 14. Except 312 all are the odd numbers.
18. Sum of the digits in the given numbers 36. 3 G 3  Choice (4)
are subtracted from the numbers. +1 +1 15. Except “Flapping” all are the sounds
→ 7 + 8 + 6 = 21 = 786 − 21 = 765 4 R 4 33rd letter is G and the 44th let- made by various animals. Choice (4)
 Choice (2) ter is R.
16. Except ‘Sorrow’ all are the synonyms.
19. The given pair numbers can be written as n Similarly, 6 N 6
 Choice (4)
: n2 + n Choice (3) +1 +1
7 7 66th letter is 17. 1728 = (12)3 but 1726 is given; 512 = (8)3;
20. (20)2 and (20)2 + 20 = 420
N and the 77th letter is Y. Choice (2) 216 = (6)3; 1000 = (10)3 Choice (1)
Similarly, (31)2 and (31)2 + 31 = 992
 Choice (2) 37. Pen is used to write, similarly needle is 18. Except ‘66’ all are perfect cubes.
used to sew. Choice (4)  Choice (1)
21. Successive prime numbers are given.
Next prime number to 5 is 7 and for 7 is 38. J is the 4th letter from the left and Q is 19. The digits in each set represent the posi-
11. the 4th letter from the right of the alpha- tion of the given letter in the English al-
Similarly, for 1113 the missing numbers bet. So, J and Q are corresponding letters. phabet.
are 1317. Choice (1) Similarly, M and N are the corresponding 2W3 ⇒ 23 = W; 1Q7 ⇒ 17 = Q; 1M3 ⇒
letters. 13 = M; 1R9 ⇒ 19 = S but not R.
22. (5)² + (8)² = 25 + 64 = 89 Hence, JM : QN : : CT : XG. Choice (4) Hence 1R9 is the odd one.
Similarly, 63 = (6)² + (3)² = 36 + 9 = 45
39. Each letter is shifted backwards by two  Choice (4)
 Choice (2)
positions. Choice (1) 20. Except in Choice (1) in all the groups the
23. 4 → 1/4 = 0.25
40. Each letter is shifted backwards by four place value of the letter is equal to the
5 → 1/5 = 0.2 Choice (3)
positions. Choice (1) product of two numbers on either of its
24. 3 and (3)4 = 81. Similarly, 7 and (7)4 = 2401 sides. Choice (1)
 Choice (1) 21. In all the pairs, the sum of the digit and
25. We get bread from bakery, similarly we Odd-Man Out the place value of the letter is equal to 5
get milk from Dairy. Choice (2) except in (1). Choice (1)
26. Calorie is the unit of heat, similarly Bar is Practice Exercise 22. In each group the number between the
the unit of pressure. Choice (3) Solutions for questions 1 to 30: two letters is equal to the sum of the posi-
27. Ammeter is an instrument to measure the tion values of letters on either side of it.
1. All except ‘21’ are prime numbers. But this pattern is not followed in choice
strength of an electric current, similarly
 Choice (1)
Barometer is an instrument to measure (3). Choice (3)
atmospheric pressure. Choice (4) 2. Except ‘1001’ in every number the num-
23. In every pair the second term is the young-
ber of zeros between two digits is equal to
28. Canada is in the continent North America, er-one of the animal except in choice (4).
the value of the digit. Choice (1)
similarly Sudan is in Africa. Choice (2)  Choice (4)
3. Except ‘243’, all others are even num-
29. We get wood from trees, similarly we get bers. Choice (2) 24. Except ‘Dynamics’ all are the various
wool from sheep’s. Choice (3) parts of the mathematics. Choice (1)
4. Except ‘903’ all are the multiplies of 11.
30. Cardiology deals with heart, similarly  Choice (1) 25. In each group of letters the second letter
botany deals with plants. Choice (1) is the corresponding letter from the other
5. Except ‘Dog’ all are aquatic animals.
 Choice (3) end of the alphabet and the third letter is
Solutions for questions 31 to 40: the corresponding letter with respect to
If the 1st half and the 2nd half of the English al- 6. All the numbers except ‘158’ are in the two halves of the alphabet. This process
phabet are reversed, then a new series is formed form of ‘n² − n'. Choice (4) is followed in all the groups except in
as follows. 7. Except in second option in every group ‘SHG’. Choice (2)
M L K J I H G F E D C B A the third number is the product of first
26. Except ‘O’ all are the consonants.
Z Y X W V U T S R Q P O N two numbers. Choice (2)
 Choice (2)
31. Here, each consecutive pair of the letters 8. Pairs of prime numbers in descending or-
27. Except ‘April’ all are the months which
in the first term is reversed to get the next der are taken and written one next to the
consists of 31 days. Choice (3)
term. Choice (4) other. Choice (3)

08_Section.indb 117 10/30/2009 11:53:47 AM


118  n  Solution Manual

28. Except ‘Lotus’all other are terrestrial plants. 12. 3544554 is the code for the word MON- 25. The word and their codes are as follow
 Choice (2) SOON. Choice (1) In these words ‘E’ and R coded as 1 and 5
but which stands for what is not known.
29. Except in the fourth option in all the pairs Solutions for questions 13 to 16: A is coded as 6.
the place value of one of the letters from
left to right is same as that of the other 13. Word  :  G R A D U A T E Hence the code for EAR is 156 or 165 or
from right to left. Choice (4) Logic  :  +1 −1 +2 −2 +3 −3 +4 −4 any combination of 1, 5, 6. Choice (3)
Code  :  H Q C B X X X A 26. Turmeric is yellow in colour and in the
30. Except in the pair GE in every pair a con- Similarly, ‘;LABORATORY’ is coded as code yellow is called orange.
sonant is followed by it’s immediate next MZDMUXXKWT. Choice (3) Hence, in that code turmeric is orange in
vowel in the alphabet. Choice (1)
14. A Pen is used for writing and in the code colour. Choice (4)
duster means pen. 27. Let us number the letters in the word
Hence, duster is used for writing.
Coding-Decoding  Choice (1)
S I G N A L as 1 2 3 4 5 6
These letters are arranged in the coding as
Practice Exercise 15. In the given word P, R, D, T are even GAS I N L3 5 12 4 6
positioned letters and all the remain- Similarly, H U R D LE is coded as RLHUDE.
Solutions for questions 1 to 6:
ing are with odd positioned letters.  Choice (3)
1. Letters are arranged in the increasing or- Thus 112122212 is the code of the word Solutions for questions 28 to 30:
der of their place values. Choice (4) PREDICATE. Choice (3) In the word PAGEANT, the letter A is written
2. The word is divided into two halves and 16. The first 3 letters are shifted for- twice and so the code z. Hence the code for A
each half is reversed. Choice (1) ward by 3 position, the last 3 let- is z.
ters are shifted backwards by 3 letters Similarly, from the 2nd word the code for E is v.
3. In the word, the vowels are written in the
 Choice (1) From the 5th word the code for N is m. Compar-
increasing order followed by the conso-
ing the 2nd and 3rd words, the letter M and the
nants in the increasing order. Choice (2) Solutions for questions 17 to 19: code n are common. So, the code for M is n.
4. In this, alternate letters starting from the It is given that the code for A and Z is 1, B and Y Similarly by comparing the words, the code for I
first are written followed by the remain- is 2, C and X is 3, D and W is 4 and so on. is r, R is I, L is o, P is k, T is g, G is t and S is h.
ing letters. The letters and their corresponding codes are as The letters and the codes are as follows
So, SIGNIFICANT is written as SGIIAT- follows: Letter A E P M S N I L R T G
INFCN. Choice (2) Letter: A B C D E F G H I J K the 5thz word
Code  v the
k code for Nm
n h is m.
r Comparing
o i g thet 2nd 6. As 55 stu
Code: 1 2 3 4 5 6 7 8 9 10 11 and 3rd words, the letter M and the code n are
5. By comparing (1) and (2) we have Sip common.
28. ormvzi is the implies tha
So, the codecode
for Mtoisthe
n. word LINEAR.
Letter: L M N O P Q R S T U V
is b, ∴ The code for tip is either g or h. Similarly
 by comparing the words, the code for (1)
Choice I is r,
Code: 12 13 13 12 11 10 9 8 7 6 5 Solutions for
 Choice (4) R is I, L is o, P is k, T is g, G is t and S is h. The
Letter: W X Y Z letters
29. and
orgrtzgv is the are
the codes codeasoffollows
the word LITIGATE. From the giv
6. Each letter is shifted backwards by 2 diagram, whe
Code: 4 3 2 1 Letter  A E P M S N I L Choice
R T (2)
G
Siemens and N
positions Choice (4) Code z v k n h m r o i g t
17. 9137591398891213 is the code of the 30. krtnvmg is the code to the word PIGMENT. P = 14
28. ormvzi
 is the code to the word LINEAR.Choice (3)
Solutions for questions 7 to 9: word INTERMISSION. Choice (4)
Choice (1)
The letters and their corresponding codes are 18. 958791913 is the code of the word RE- 29. orgrtzgv is the code of the word LITIGATE.
follows: STRAIN. Choice (3) Choice (2)
Letters: P M N I L E A R T S Venn Diagrams
Codes: 5 4 9 3 6 0 7 2 1 8 19. 78613459 is the code to the word 30. krtnvmg is the code to the word PIGMENT.
THUNDER. Choice (4) Choice (3)
7. 57278310 is the code of the word PARA- Practice Exercise
Exercise 5 i. 22 families
SITE. Choice (4) Solutions for questions 20 to 22: (Venn Diagrams) ii. 10 families
Solutions for questions 1 to 3:
Word : E Q U A T I O N iii. 35 families
8. 395713091 is the code of the word INPA- As per thefor
Solutions given data, we
questions get3:the following dia-
1 to
iv. 25 families
Logic 1 : The letters in the word are reversed
TIENT. Choice (1) gram:
As per the given data, we get the following diagram: Siemens.
N O I T A U Q E
Total
Total = 150
= 150 v. 15 familie
9. 52340 is the code of the word PRIME. Logic 2 : +1 +1 +1 +1 +1 +1 +1 +1 Panasonic
 Choice (2) Code : O P J U B V R F vi. 23 familie
a b G = 65 not Nokia.
H = 45 r
Solutions for questions 10 to 12: 20. Choice (3) 35 10 55 vii. All the 20
The letters and their corresponding codes are as one compa
21. Choice (1)
follows. 7. 35 familie
Letter: A B C D E F G H I J K 22. Choice (4) H → History G → Geography
Code: 1 2 3 4 5 1 2 3 4 5 1 H o History G o Geography
1. 55 take only Geography. Choice (3) 8. 25 familie
Letter: L M N O P Q R S T U V Solutions for questions 23 to 27: 1. 55 take only Geography. Choice (3) and Nokia
2. 2. 35 35 students take only History. Choice (2) Panasonic
Code: 2 3 4 5 1 2 3 4 5 1 2 23. In this the place value of the letters are
students take only History. Choice (2)
Letter: W X Y Z given as their codes. Choice (1) 3. 3. At most one subject = µ – r = 150 – 10
At most one subject = P – r = 150 – 10 = 140.= 140.
Code: 3 4 5 1  Choice
Choice(4)
(4) 9. Exactly on
24. Word  :  C O N C E P T Nokia + O
10. 15353145 is the code for the word POR- Solutions for questions 4 to 6: families.
Logic  :  +1 +2 +3 +4 +5 +6 +7 Solutions for questions 4 to 6:
TRAIT. Choice (4) Code  :  D Q Q G J V A Since, 60 students
students failed
failedininMaths. ∴165– –6060= =105
Maths.?165 10. Neither Pa
students passed in Maths similarly, 65 students failed So, 10 f
11. 113411435 is the code for the word PUR- Similarly, EXECUTION is coded as 105 students passed in Maths similarly, 65 stu-
in English. Panasoni
SUANCE. Choice (1) FZHGZZPWW. Choice (4) dents
? 165failed
– 65 =in100
English.
students passed in English.
Solutions for
The respective Venn – diagrams is as follows which
shows the number of students who passed the According to t
subject. P = 165 will be as follo
? a = 42 per
Maths = 105 English = 100 cent
08_Section.indb 118 10/30/2009 11:53:47Similarly,
AM
Panasonic. Given 100 students36were + 378 = 414 who read novels
surveyed (people
H o History G o Geography Choice (2) ,
vi. 23 8. families
25 families
use both usePanasonic
mobile-phones of of
and Siemens Christie
both but or Doyle or both.
Panasonic ? 142 p
1. 55 take only Geography. G = 65 Choice (3) and Nokia but not Siemens (Region Fromcommon the given 13. Atleast one category
to table we get the information that = m – n = 600 or 600 – 36 = 100
564 m
a r b not Nokia. L Q
H = 45 40 females read students
Doyle and 70 students read Christie. Choice (3)
2. 35 students35take10only 55
History. Choice (2) vii. All thePanasonic 200 families and useNokia but not Siemens).
mobile-phones of at least 20. It is give
one company. Further,
Choice (1) 14. (10 + 9 + 8) per cent of 600 = 162 people belong U
3. At most one subject = P – r = 150 – 10 = 140. 1. 37 per cent of the total students read both that is 37 P X
7. 359. Exactly
families use one company
mobile-phones = Only
of only Panasonic
Siemens. + Only to exactly any two of the categories. Choice (1)
Choice (4) students. SolutionsTfo
Nokia + Only Siemens = 22 Choice + 2.10 The + 35ratio
(2) = 67 ofSolutions
males and forfemales
questions is 115: to1. 17:
that is
HSolutions
 Historyfor questions 4 to 6: families. Choice (2)  whon read119 O
8. 25 families use mobile-phones ofSolutions for Intelligence and Critical Reasoning
G  Geography number of males = number of females = 50. The given di
both Panasonic Given 100 students
3. 50 per cent of the females that is 50 per cent of were surveyed novels,V
1. 55 Since, 60 students
take only Geography. failed in Maths.Choice ?165(3) – 60 = 105 10. Nokia
and Neither butPanasonic
not Siemens nor Siemens implies only
(Region common to Nokia.of Christie or Doyle or both. 10,000 m ,
students passed in Maths similarly, 65 students failed 50 = 25 females read both. 100
Panasonic So, 10 and families
Nokia butuse mobile-phones of neitherFrom the given
not Siemens). Females table we get the information that
2. 35 ∴ 165
students
in English. – 65 =
take 100
only students
History. passed in
Choice English.
(2) 10. Neither Panasonic
Panasonic nor Siemens.
nor Siemens implies
Males Females
(3) 40 females read Doyle and 70 students read21. The letter ‘X’ represe
ChoiceChoice
(1) Doyle Christie.
3. At The
? most165respective
–one 100Venn
65 =subject – –diagrams
students
=  passed
r = 150 in – is as
English.
10 follows
= 140. only Nokia. Doyle Christie
Further, Christie participate in all the fou
9. Solutions
Exactly one for questions
company = Only 11Panasonic
to 14: + Only
which shows the
The respective number
Venn of students
– diagrams aswho
isChoice (4)passed
follows which So, 10 families use mobile-phones of nei- 1. 37 per cent of the total students read both thatcommon is 37 to all the four c
Nokia
According + Only Siemens
to the given = 22 + 10 + 35 = 67 15 25
the
shows
Solutions subject.
forthe number 4ofto 6:
questions students who passed the ther Panasonic
families. nor information,
Siemens. Choice the Venn-diagram
Choice 15(3) 12
(2) 23 students. 10 22. Athletes who participat
subject. P = 165 will be as follows: 2. The ratio of males and females is 1 : 1. that are isrepresented by the
number of males = number of females = 50. that is, the region com
Since, 60 students failed in Maths.
Maths English
165 – 60 = 105
= 100 Solutions
10. Neither for
? a = Panasonic questions
42 per cent – 11
(8 +
nor Siemens to 414:+ 10)only
implies per Nokia.
cent = 20 per ,V
students passed in Maths= similarly,
105 65 students failed cent
So, 10 families use mobile-phones 3. With
50 per the above
cent of information,
the females we
that isget,
50 per cent of
According to the given information, of theneither
Venn-
With the above 50 information,
=Doyle
25 females we get,
read both.
and the region common 10,000
in English. Similarly, nor Siemens.
Panasonic Choice (3) So, = 27 So, Doyle = 40 (3)
diagram Females
 165 – 65 = 100 students b = 43will perbecentas –follows:
(8+ 4 + 9) per cent = 22 per So, cent,
Doyle Males
= 27 So, Doyle = 40
a passed 45 in English.
b Solutions = 35 Christie = 35 Christie
Christie Christie
= 35 = 35 21. The let
∴ ac== 42 44forperquestions
cent ––(10
cent (8 11
++ 44to++14:
9)10)= 21perpercent
cent= 20 Doyle Christie Doyle 23.Christie
Athletes who participate
participa
The respective Venn – diagrams is as follows which n = 100 per given
cent –information,
(20 + 21 + the 22 +Venn-diagram Both
10 + 9 + 8 + 4) per
= 12 Both = 12 Both Both
= 25 = 25 represented bycommon the lette
shows the number of students who npassed the per cent
According to the
Now after fillingNow afterin the
filling
tablethe gaps 15 in the 25 table
will becent = 6 per cent
as follows: the gaps we get: the region common to e
subject.  = 165 Similarly, 15 12 23 10
45 students passed
passed in inMaths
Mathsasaswell wellasasEnglish.
English. 22. Athletes
45 students  a =?6 per
percent
cent=–36 people. we Christie
get:
? aa ==Maths
∴ 105–=–45
105 105
45= =60English
60 = 100
students
students passed
passed onlyonly in cent
in Maths 4342per
b = Hence, cent
600 – (8
people(8++ 4 + 10) per cent = 20 per
4 +the
are 9)members
per cent the = 22 club.per Doyle Both Total
(1) are repr
Male 27 35 12 50 that is,
and b =and
Maths 100 b – 45 = 55 –students
= 100 45 = 55 passed
students only passed cent,
English. Similarly, Female 40 35
Doyle Christie Both
25 50
Total
With the above information, we get, 24. The letter ‘W’ andreprese
the
bc==43 44per per cent
cent –
– (8+ (10 +
4 + 9) 4
per+ 9)
cent = =21
22Sportspersons
per cent
per cent, Total 67 70 37 100
only
Number English.
of students passed in at least one subject =
a 45 b Postgraduates Male 27 35 12 50 participate in all
(3)the thre
c = 44100per per cent – (10–+(20 4 + 9) 43 per cent So, Doyle = 27 So, Doyle = 40
60 + 55+ 45
Number of =students
160 passed in at least one sub- nn==100 42 percent
cent + =2121+per 22cent
+ 10 + 9 +128 Males + It is the region commo
per cent – (20 +a21 + 22 + 10b+ 9 + 8 +15. 4) per read
Femalethe books 40 by=both
Christie 3535 the authors. 25 Christie
50 = 35 23. Athletes
Hence, n = 165 – 160 = 5 students
ject = 60 + 55+ 45 = 160 n failed in both. 4)
cent per
= 6 cent
per =
cent6 per cent 8 Both = 12 Choice (2) Both = 25IV and excluding ,.
represen
∴6 per cent = 36 people. 4 per Total 67 70 37 100
4. a +passed
Hence,
45 students bn ==60in
165+Maths
55– =160115
as =students
well 5 English.
as passed
students in in 6 per cent = 36 people. 10
exactly
failed 9 per 16. The number of Now students after whofillingread
the gaps
booksinby theonly
table weSolutions
get: for questions
the regio 25
105one
 a =both. – 45subject.
= 60 students passed only in Maths Choice (3) Hence,Hence,600 600 people
people are are the members
the members the club.
the club. Christie is 23 15. + 10 12= Males33. Doyle read Christie
theChoice
books (3)by both theAlgebra and Geometry
c Both Total
25.
and b = 100 – 45 = 55 students passed only English. (1)
5. 5 students failed in both the subjects.
4. a + b = 60 + 55 = 115 students passed
Politicians
Sportspersons 17. 4415 females readauthors. Male by only
books 27 Doyle. 35 12 Choice 50 (2)other but they constitu
Number of students passed in at least one subject Choice
= (4) Postgraduates per cent n = 36 Female 40 35Choice 25
(4) 50 Mathematics. 24. The lett
43% 16. The
60 + 55+ 45 = exactly
160 in one subject. 42% Sol/913 Totalnumber 67of students 70 who 37read100 books participa
Hence, n = 165  – 160 = 5 students failed in both.Choice (3) a 8% b Solutions for questions by only 18 to 20: is 23 + 10 = 33.
Christie 26. Ganga, Godavari and I
15. 12 Males read the books by both the authors. It is the
they are different from e
4. a + b
4% 
From the given information we will get the following Choice (2) Choice (3) IV and e
5. = 60
5 +students
55 = 115 failed
studentsinpassed
both exactly
the subjects.
in 10% 9% Venn-diagram. P = 150
one subject . Choice (3) 27. Some females are Doct
Solutions fo
 Choice (4) c 17.
16. The 15 number
femalesofread books
students whobyread onlybooksDoyle.
byIntelligent.
only Some Docto
Star Movies = 52 Christie + x Discovery = 55 + x
is 23 + 10 = 33. Choice
5. 5 students failed in both the subjects. Politicians 44%  Choice (4)(3) 25. Algebra
6. As 55 students passed only Choice in (4)
English
n = 36 17. 15 females
27 x 36 read books by only Doyle. other bu
which implies that 55 students failed only Sol/913 Solutions for questions 18 to 20: Solutions for questions Mathem 28
8 Choice (4)
in Maths. Choice (1) 11. 600 people are there in the club. From17the given 11 information we will get the28. fol-City is the part of stat
Comparing the 2nd Solutions for questions 18 to 20: 26. Ganga,
6. As 55 students passed only in English which Choice (3) lowing Venn-diagram. y country. they are
the code n are Solutions implies forthatquestions
55 students 7 tofailed
10: only in Maths. From the given information we will get the following
μ = 150
From the given data, we get the following Choicethe(1) 12. At most one = none + exactly one = 36 + Venn-diagram. P = 15029. A soldier can 27.beSomea paintfe
the code for I is r, Sony TV = 36 + y
t and S is h. The
Solutions
diagram, for questions
where P S and M7 standsto 10: for Panasonic, (20 + 21 +22) per cent of 600 = 36 + 63 n=0
Star Movies = 52 + x Discovery = 55 + x Intelligen
Siemens
From the andgiven
Nokiadata, we get the following the per cent of 600 = 36 + 378 = 414 From the above venn diagram we get x + y = 51. 30. Red and yellow are two
diagram, where P S and M stands
Totalfor= Panasonic,
200  Choice (2) 27 x 36 are two parts of the rain
L R T G Solutions fo
Siemens and Nokia Total = 200 8
o i g t
P = 140
13. Atleast one category = m – n = 600 or 600 – 17 11 28. City is t
N = 120
LINEAR. 36 = 564 students Choice (3) y country.
Choice (1)
22 25 10 14. (10 + 9 + 8) per cent of 600 = 162 people 29. A soldie
LITIGATE. 70 Sony TV = 36 + y
23 15 belong to exactly any two of the catego- n=0
Choice (2) 18. It is given that, x = 2y ? 3y = 51 or y = 17 ? x = 34
35 S = 143 ries. 11. 600 people are there in Choice
the club. (1) From Choice the
From the (3) above venn diagramwewegetget
above venn?diagram The people whoxwatch
x + 51.
y=
+ y = sonny TV = 3630. + 17Red = 53.and
PIGMENT. 51. Choice are (4)two
Choice (3) 12. At most one = none + exactly one = 36 + (20 + 21
Solutions for questions 15 to 17:
+22) per cent of 600 = 36 + 63 per cent 18. It is =given that,
of 600 x = 2y who
19. People ∴ 3ywatch
= 51 at or most
y = 17two channels = 150 –
i. 22 families use only Panasonic Phones. Given 100 students 36 + 378were= 414 surveyed who read
) ii.i. 10
22 families
familiesuse useonlyonly Panasonic
Nokia phones.Phones. ∴ x = 34 (people who watch all the three) Ÿ 150 – 8 = 142.
novels of Christie or Choice Doyle or (2)both. ? 142 people watch
iii.
ii. 35 10 families
familiesuse useonlyonlySiemens phones.
Nokia phones. 13. Atleast ∴ The people who watch sonny TVat=most
36 +two channels.
iv. 25 families use both Panasonic and Nokia but notFrom the given table weone getcategory = m – n = 600
the information that or 600 – 36 = 564 Choice (1)
iii. Siemens.
35 families use only Siemens phones. students Choice17 (3)= 53. Choice (4)
llowing diagram: 40 females read Doyle and 70 students read 20. It is given that, x = 15 ? y = 51 – 15 ? y = 36.
al = 150
iv. 15
v. 25 families
familiesuse useboth
bothNokia
Panasonic and Nokia
and Siemens but notChristie. 14. (10 + 9 + 8) per cent of 600 = 162 people 19. belongPeople who watch at most two channels = Choice (2)
but not Siemens.
Panasonic. to exactly any two of the categories. Choice150 (1) – (people who watch all the three) ⇒
vi.
Further, Solutions for questions 21 to 24:
G = 65 v. 23 15 families
familiesuse useboth
bothPanasonic
Nokia and and Siemens
Siemens but
Solutions for questions 15 to 17: 150 – 8 = 142.
not Nokia. The given diagram is as follows.
vii. Allbutthenot200Panasonic.
families use mobile-phones of at least
1. 37 per cent of the total students read both
Given 100 students were surveyed who read novels ∴ 142 people watch at most two channels.
vi. one 23 families
company. use both Panasonic and Sie- that is 37 of students.
Christie or Doyle or both. , ,,
 Choice (1)
From the given table we get the information that 100 m 400 m
mens but not Nokia.
7. 35 families use mobile-phones of only Siemens. 2. The ratio of males and females is 1 : 1. that L Q M
vii. All the 200 families use mobile-phones of at(2) 40 females read Doyle and 70 students read 20. It is given that, x = 15 ∴ y = 51 – 15 ∴ y
Christie.
Choice is number of males = number of females =
Further, U V(2)
least one company. = 36. Choice X
8. 25 families use mobile-phones of both Panasonic 50. 1. 37 per cent of the total students read both that is 37 P R
Choice (3) students. T W
7. and 35 Nokia
families but use
not Siemens
mobile-phones(Region of only to3. 50 per cent
common of the females that is 50 per
2. The ratio of males and females is 1 : 1. that is
Solutions for questions 21 to 24:
Panasonic and Nokia but not Siemens). cent of 50 =number
25 females read both.
Choice (2) Siemens. Choice
Choice(2)(1) of males = number of femalesThe = 50.given diagram is as follows. O
,V
S N
,,,
150 – 10 = 140. 3. 50 per cent of the females that is 50 per cent of 10,000 m
9.8. Exactly
25 families
one use
company mobile-phones
= Only Panasonic of both
+ Only 50 = 25 females read both.  
1500 m
Choice (4) Males Females
Panasonic
Nokia + Onlyand Nokia= but
Siemens 22 not + 10Siemens
+ 35 = 67 Males 100 m 400 m
families. L21. TheQ letterM ‘X’ represents those athletes who
(Region common to Panasonic andChoice Nokia(2) Doyle Christie Doyle Christie participate in all the four races that is, the region
?165 – 60 = 105 but not
10. Neither Siemens).
Panasonic nor Siemens implies Choiceonly (1)
Nokia. common
U V to all the four circles. Choice (2)
65 students failed P X R
So, 10 families use mobile-phones of neither 15 12 23 15 25 10
9. Panasonic
Exactly one company = Only Panasonic 22. Athletes
T who participate in at least three races
nor Siemens. Choice (3) W
are represented by the letters U, V, W, T and X
n English. + Only Nokia + Only Siemens = 22 + 10 O that is, S theNregion common to the three circles
Solutions for questions 11 to 14: ΙV
s as follows which + 35 = 67 families. Choice (2) With the above information, we get, and the region common  to all four circles. Choice
who passed the According to the given information, the Venn-diagram 10,000 m (3) 1500 m
will be as follows: So, Doyle = 27 So, Doyle = 40
165
Christie = 35 Christie = 35
? a = 42 per cent – (8 + 4 + 10) per cent = 20 per 23. Athletes who participate in exactly two races are
100 Both = 12 Both = 25
cent represented by the letters P, Q, R and S that is,
Similarly, Now after filling the gaps in the table we get: the region common to exactly two circles.
b = 43 per cent – (8+ 4 + 9) per cent = 22 per cent, Choice
Doyle Christie Both Total
c = 44 per cent – (10 + 4 + 9) = 21 per cent (1)
08_Section.indbn 119 Male 27 35 12 50 10/30/2009 11:53:49 AM
= 100 per cent – (20 + 21 + 22 + 10 + 9 + 8 + 4) per
120  n  Solution Manual

21. The letter ‘X’ represents those athletes 33. Number of elements in B ∩ (A ∪ C) = 15 40. Number of elements in A’
who participate in all the four races that + 8 + 9 = 32 Choice (1) that is, the elements which are not in A =
is, the region common to all the four cir- 12 + 9 + y + r = 21 + y + r Choice (1)
cles. Choice (2) Solutions for questions 34 to 36:
By observing the data entries of any city, we can
22. Athletes who participate in at least three
Solutions
say for
thatquestions
some people31 toare33:
also included in the to- 38. Number of elements in C = 40
races are represented by the letters U, The
V, given
tal, who
diagram = 100 phones.
do not use any of thePmobile
Clocks
It is given that the number of elements in A = 40
W, T and X that is, the region commonistoas follows. A = 41 Number of elements in only A
For Lucknow, we have (250 –B25) = 40
+ (153 – 25) x = 40 – (8 + 12 + 7) Ÿ x = 40 – 27 Ÿ x = 13
the three circles and the region common + 25 + n = 428 12
15 8 Practice Exercise Choice (4)
to all four circles. Choice (3) ∴ n = 428 – 378 = 50 6 8 9
39.Solutions
It is givenfor questions
that 1 toof25:
the number elements in all the
23. Athletes who participate in exactly two [Here ‘n’ represents the people 12 who do not use
C = 35
sets are equal.
N=? Hence,
1. In A1= hour the hour-hand moves 360/12
races are represented by the letters P, Q, any of the two mobile phones] B = C = 40
Now, = x= 30°.
40 – (12 + 8 + 7) Ÿ 40 – 27 = 13
R and S that is, the region common to ex-N = P –Similarly,
(A ‰ B ‰ C) For Jaipur, n = 320 – [(160 – 10) +
Now, Hence,
y = 40 –in
(12
actly two circles. Ÿ N = (140
Choice (1) 100 ––(12 10)++8 10]
+ 12⇒ n =+30.
+ 15 6 + 9 + 8)
10+ minutes
8 + 9) Ÿ it
40cover
– 29 =30°/6
11 = 5°
Now, (A ‰ B ‰ C) = 13 + 11 + 12 + 7 + 8 + 9 + 12
Ÿ N = For
100Ahmedabad,
– 70 Ÿ N = n30= 445 – [(225 – 35) + (200 – 35) = 72
 Choice (4)
Choice (2)
24. The letter ‘W’ represents those athletes
+ 35] ⇒
31. Number n = 445 –in390
of elements = 55
B complement
who participate in all the three races, ex- B’ = 12 + 6 + 12 + N, B’ = 12 + 6 + 12 + 30 Ÿ B’ 40. Number
2. As ofweelements
know that
in A'hour hand moves half a
For Dehradun, n = 480 – [(270 – 40) + (215 – degree
that is, per minute.
the elements which are not in A = 12 + 9 +
cept in 100 m. It is the region common to = 60 OR B' = 100  40 = 60 Choice (3)
40) + 40] = 480 – 445 = 35 y + r =∴21It +will
y + move
r Choice (1)
24° in 48 minutes.
the circles ΙΙ, ΙΙΙ and IV and excluding Ι.
32. Number of elements in A ‰ B ‰ C = 12 + 8 + 12 Since, minute hand 6moves by 6° in one
 Choice (4) + 15 34. 35 people do not use anyChoice
+ 9 + 6 + 8 = 70
of the(4) mobile Exercise
phones in Dehradun. Choice (2) minute. (Clocks)
Solutions for questions 25 to 27: 33. Number of elements in B ˆ (A ‰ C) = 15 + 8 + 9 Solutions∴for
It will move 48
questions × 625:
1 to = 288° in 48 minutes.
35. 153 – 25 = 128 people use Choice
= 32 only Panasonic
(1)  Choice (4)
25. Algebra and Geometry are different from in Lucknow. Choice (1) 1. In 1 hour the hour-hand moves 360/12 = 30q.
Solutions for questions 34 to 36:
each other but they constitute two differ- Hence,
3. In in50
10 seconds
minutes itthe
cover 30q/6 =hand
second 5q covers
ent areas in Mathematics. 36. People
By observing
Choice (4) the datause onlyofPanasonic
entries in can
any city, we Lucknow
say : 300°. Choice (4)
that some people
peoplearedoalso
notincluded
use anyinofthe
thetotal,
twowho mobile
do not use any of the mobile phones. 2. As weInknow60 that
seconds
hour the
handminute handa degree
moves half covers
26. Ganga, Godavari and Indus are three riv- phones in Jaipur. per minute.
For Lucknow, we have (250 – 25) + (153 – 25) + 25 +
ers and they are different from each other. 300/60 = 6°.
n = 428 = 128 : 30 = 64 : 15. Choice (3) ? It will move 24q in 48 minutes.
 Choice (3) Hence, in 50 seconds it covers 5°.
? n = 428 – 378 = 50 Since, minute hand moves by 6q in one minute.
 move 48 u 6 = 288q in 48 minutes.
? It will Choice (2)
27. Some females are Doctors and some fe- Solutions
[Here ‘n’ representsforthe
questions
people who37 to
do40:
not use any of
the two mobile phones] Choice (4)
males are Intelligent. Some Doctors are
Similarly, For Jaipur, n = 320 – [(160 – 10) + (140 – 11
Intelligent. Choice (2)
10) + 10] Ÿ n = 30. A B 3. In4.
50 As we know
seconds that θ =
the second hand m − 30h here,
2 covers 300q.
For Ahmedabad, n = 445 – x[(2258– 35)y + (200 – 35) + 35] In 60 seconds the minute hand covers 300/60 = 6q.
we have h = 5 and m = 35 Choice (1)
Hence, in 50 seconds it covers 5q. Choice (2)
Solutions for questions 28 to 30: Ÿ n = 445 – 390 = 55 12
For Dehradun, n = 480 – [(2707 – 940) + (215 – 40) + 5. The angle between the hands at 3 hours
28. City is the part of state, which is the part
40] = 480 – 445 = 35 11
12 4. As we25know
minutes
that Tis= m  30h here, we have
of a country. Choice (1) 2
34. 35 people do not use N = rany of theCmobile
= 40 phones in θ = 11/2 m – 30 h
h = 5 and m = 35 Choice (1)
29. A soldier can be a painter or a singer or both. Dehradun. Choice (2) Here, h = 3 and m = 25  Choice (4)
 Choice (4)
35. 153 Number
– 25 =of128 elements
peopleinuseonly C =Panasonic
only 40 – (7 + in 12 5. The angle between the hands at 3 hours
+ 9)
Lucknow. Choice (1) 256. minutes
Here, iswe have h = 7 and m = 10
30. Red and yellow are two distinct colours ⇒ 40 – 28 = 12 θ =m30– 30
T = 11/2 × 7h− 11/2 × 10 = 210 − 55 =155°
36. People use only Panasonic in Lucknow : people Here,h = 3 and m = 25 Choice
but both are two parts of the rainbow do not use any of the two mobile phones in Choice(4)(3)
colours. 37. It is given that the number of elements in
Choice (3) Jaipur. 6. Here, we have h = 7 and m = 10
7. θ = 11/2 m – 30h
= 128 : 30B=is64twice
: 15. that of number ofChoiceelements(3) in C, T = 30 u 7  11/2 u 10 = 210  55 =155q
Solutions for questions 31 to 33: that is, B = 80. Here it is given that θ = 0 Choice and h (3)
= 6
Solutions for questions 37 to 40:  Choice (1)
The given diagram is as follows. Then y = [(80 – (8 + 9 + 12)] = y = (80 7. T = 11/2 m – 30h
µ = 100 – 29)A B Here
8. Here, we that
it is given 4, θ h==0°6 Choice (1)
haveT h= =0 and
A = 41 B = 40 that is yx = 518 y Choice (3) 0 =11/2 m − 4 × 30 ⇒ m = 219/11
8. Here, we have h = 4, T = 0q
15 8 12
12 38. Number of 7 elements in C = 40
9 0 =11/2 m  4 u 30 Ÿ m = 219/11 Choice(1)(1)
Choice
8
6 9 It is given that12 the number of elements in
9. North
12 A =N40 12
N=? C = 35 =r C = 40 11 1
Number
Number of elements in of
onlyelements
C = 40 –in(7only
+ 12A+ 9) 10 2
N = µ – (A ∪ B ∪ C) Ÿ 40 – 28 = 12 x = 40 – (8 + 12 + 7) ⇒ x = 40 – 27 ⇒ x = 13
⇒ N = 100 – (12 + 8 + 12 + 15 + 6 + 9 + 8) 9. West 9 3 East
37. It is given that the number of elements in B Choice is twice (4)
⇒ N = 100 – 70 ⇒ N = 30 that of number of elements in C, that is, B = 80.
39. It is given that the number of elements in 8 4
Then y = [(80 – (8 + 9 + 12)] = y = (80 – 29)
31. Number of elements in B complement that is y =all51the sets are equal. Choice 7
6
5
B’ = 12 + 6 + 12 + N, B’ = 12 + 6 + 12 +(3) Hence, A = B = C = 40 South
30 ⇒ B’ = 60 OR B' = 100 − 40 = 60 Now, x = 40 – (12 + 8 + 7) ⇒ 40 – 27 It is given that the minute hand is Sol/915
pointing
= 13 towards South, then the hour hand must be
 Choice (3)
Now, y = 40 – (12 + 8 + 9) ⇒ 40 – 29 towards Southeast. Choice (1)
32. Number of elements in A ∪ B ∪ C = 12 = 11
10. It is given that θ = 180°, h = 3
+ 8 + 12 + 15 + 9 + 6 + 8 = 70 Now, (A ∪ B ∪ C) = 13 + 11 + 12 + 7 + 8
θ = 11/2 m – 30h, 11/2 m = θ + 30h
+ 9 + 12 = 72 Choice (2)
 Choice (4)  Choice (2)

08_Section.indb 120 10/30/2009 11:53:50 AM


Solutions for Intelligence and Critical Reasoning  n  121

11. 12:00 − 8:53 = 3:07 Choice (1) The watch loses (12 + 20) = 32 minutes. ∴ The watch loses 5 min in 5/15 x 54 =
The watch loses 32 minutes in 110 hours. 18 hours
It12. (12:00that
is given – 7:15)
the = 4: 45.hand
minute is Choice
pointing (4) 18. Time from 4:00 a.m. on 6th of this month to 6:00
towards South, then the hour hand must be ∴10th
p.m. on The of
watch loses 12
this month minutes
= 110 hours.in 12/32 x 110 18 hours from 3:00 p.m. Sunday is 9:00
towards
13. The exception time for the
Southeast. condition
Choice (1) θ The watch
= 41loses
hours(1215+ minutes
20) = 32 minutes. a.m. on Monday, which is when the clock
= 180° The watch loses 32 minutes in 110 hours.
10. It is given that Tis= 6 O’clock.
180q, h = 3 From 4:00 p.m. on Hence 41 hours 15 minutes from 4:00
? The watch loses 12 minutes in 12/32 x 110 shows the correct time. Choice (4)
Tuesday
T = 11/2 m – 30h,to 4:00
11/2 m =p.m. on Wednesday,
T + 30h Choice (2) it a.m. 15
= 41 hours onminutes
6th is 9:15 p.m. on 7th.
11. 12:00 occurs
8:53 = 22 times. Hence 41 hours 15 minutes from 4:00 a.m. on 6th (1)
Choice
3:07 Choice (1)
is 9:15 p.m. on 7th. Choice (1)
From 4:00 p.m. Wednesday to 4:00 a.m.
12. (12:00 – 7:15) = 4: 45. Choice (4) 19. From 2:00 p.m. on 2nd of a month to 2:00 Calendars
on Thursday it occurs 11 times. 19. From 2:00 p.m. on 2nd of a month to 2:00 p.m.
13. The exception
From 4:00timea.m.for the
to condition T = 180q
11:00 a.m., is 6
it occurs on thep.m. 4th ofon thethe
same 4thmonth,
of theitsame occursmonth,
44 times. it oc-
O’clock. From 4:00 p.m. on Tuesday to 4:00 p.m. From curs 2:00 44 p.m.times.
to 1:00 a.m. they coincide each Practice Exercise
6 times. Hence
on Wednesday, it occursboth the hands will be
22 times. other for 10 times.
From 2:00 p.m. to 1:00 a.m. they coincide
From opposite
4:00 p.m.for (22 + 11 +to6)4:00
Wednesday = 39a.m.
times.
on ? Hence, they coincides for, 44  10 = 54 times. Solutions for questions 1 to 25:
Thursday it occurs 11 times. each other for 10 times. Choice (2)
 Choice (2)
From 4:00 a.m. to 11:00 a.m., it occurs 6 times.
20. In a
∴ Hence,
correct clock,
they
the
coincides
hands of a
for,
clock
+ 10 = 54
44coincide 1. 100 days = 7(14) + 2. Hence, there are 2
Hence
14. Let both
thethe
givenhands will be opposite
information for
be represented every times.
65
5
/11 minutes. But in this case they Choice are (2) odd days in 100 days. Choice (2)
(22  11  6) = 39 times. Choice (2)
as follows: together again after 66 minutes, hence clock
14. Let the given information be represented as
20.
loses In
time. a correct clock, the hands of a clock 2. Number of odd days in 382 days. ⇒
follows: 50 minutes ago Loss in coincide
66 min =every (66 – 65 655/511/11) =minutes.
6/11 minutes.But in this 382/7 = 54 complete weeks + 4 odd
Present time
x case they are together60again
Loss in 24 hours = 6/11 x
u 24 after 66 min-
= 1440/121 days. Choice (4)
2 O’clock 50 minutes 5 O’clock
109
utes, hence clock loses66 time. 3. October have 31 days and 31 = 7(4) + 3
4x = 11 Loss /121 in 66 min = (66 – 655/ ) = 6/11 min-
180 minutes
The clock loses 11 /121 minutes in
109 11 24 hours.  Choice (3)
Let theLnumber
et the number of minutes
of minutes from 2toO’clock
from 2 O’clock fifty utes. Choice (3)
60 × 24 4. Number of odd days in 426 days = 426/7
minutes ago be x.
to fifty minutes ago be x. 21. The
Loss inof 24
number hours
hours
from
=10:00
6/11 p.m.x on a day=to
? The number of minutes from present time to 66 = 60 complete weeks + 6 odd days.
5 O’clock ∴ is The
4x. number of minutes from present 8:00 a.m. on the next day
1440/121 = 11lost
109
= 10 hours.
/=12150  10 = 40 minutes. 6th day after Sunday is a Saturday.
In 10 hours, the clock
? x  4x time to=5180
 50 O’clock is 4x. The clock will show the correct
109 time, at the moment  Choice (1)
Let the number of minutes from 2 O’clock to fifty The clock loses 11 / minutes in 24 hours.
∴ x + 4x + 50 = 180
minutes ago be x. it gains 11 hours 50 minutes =121 710 minutes.
 u 10 = 177.5 hours = 7 days 91/2 hours. Choice (3) 5. There are 6 odd days in 125 days. Hence,
? TheLnumber et the number
of minutesoffrom
minutes from 2time
the present O’clock
to 710/40
1
5 O’clock to fifty
is 4x.minutes ago be x. 7 days 9
21. The number of hours from 10:00month
/2 hours from 8a.m. on 3rd of the p.m. on the six days to Friday is Saturday.
is 30 minutes past 5 p.m. on 10th. Choice (3)
?x  4x  50 = 180 Ÿ x = 26
∴ The number of minutes from the pres- a day to 8:00 a.m. on the next day = 10  Choice (3)
? The present time is 3:16. Choice (1) 22. Let thehours. time be 5 hours x minutes.
ent time to 5 O’clock is 4x. Minute hand reaches 6 at 5 hours 30 minutes and 6. Odd days from 2005-2008: 1 + 1 + 1 + 2.
15. The duration from 4:00
∴x + 4x + 50 = 180 ⇒ x = 26a.m. on Sunday to hour hand In hours, the clock lost = 50 − 10 = 40
106 O’clock. Hence, 1st January 2009 is 5 days after
12:00 p.m. on Wednesday = 80 hours.
∴ The present time is 3:16. Choice (1) ? From minutes.
the answer choices the present time which Saturday, that is, Thursday.
In 80 hours, the clock gained 7  9 = 16 minutes. satisfied the given condition is 5 hours 15 minutes.
But to show the correct time, the clock has to gain The clock will show the correct time, at the  Choice (1)
715. The duration from 4:00 a.m. on Sunday to Choice (1)
minutes. moment it gains 11 hours 50 minutes = 710
Hence,12:00 p.m. on
after ?7/16 Wednesday
u 80 = 35 hours = from80 Sunday
hours. 23. Difference in minutes between the two clocks in 7. Total number of odd days from 1st Janu-
3 p.m. Iitnshows the correct time. gained 7 + 9 = 16
minutes.
80 hours, the clock one hour = 3 minutes. ary to 23rd February is ⇒ Jan(2) + Feb(2)
35 hours from 4 am on Sunday is 3:00 p.m. on Number 710/40
of hours × 10from=8:00 177.5 a.m.hours = 7noon
to 12:00 dayson91/2
Monday. minutes. But to show the correct Choicetime,
(2) the = 4. Hence, the 4th day from Monday is
that day hours.
= 4 hours.
clock has to gain 7 minutes. The two clocks1 differ by = 3 x 4 = 12 minutes. Friday. Choice (1)
16. Time from 6:00 a.m. Monday to 7:00 p.m. on 7 days 9 /2 hours from 8a.m.Choice on 3rd(3)of the
Thursday Hence, hours.∴7/16 × 80 = 35 hours from
= 85 after month is 30 minutes past 5 p.m. on 10th. 8. A century year is a leap year only if it is
The watch Sunday 3 p.m. it shows the correct time. 24. Let the present time be x O’clock.
gains (4 + 6) = 10 minutes in 85 hours divisible by 400. In the given years, only
? The35watch  Choice (3)
hoursgains
from4 4minutes
am oninSunday [4/10 x is85] 3:00 ? 3(10  (x  4)) = 10  x 2800 is divisible by 400. Hence, 2800 is a
= 34 hours. (shows correct time) 3(6  x) = 10 x
 time
34 hours p.m. = 1on Monday.
day 10 hours. Choice (2) 22. Let the be 5 hours x minutes. leap year. Choice (3)
18  3x = 10  x
1 day 10 hours from 6:00 a.m. Monday = 4:00 p.m.
Tuesday.
16. Time from 6:00 a.m. Monday to 7:00 8 = 2x,Mxinute = 4 hand reaches 6 at 5 hours 30 min- 9. If a year starts with Friday, then that year
Choice (2) p.m. ? Theutes present andtimehouris hand
4 O’clock. 6 O’clock. Choice (2)
on Thursday = 85 hours. will have 53 Fridays and 53 Saturdays.
17. The number of hours from 8:00 p.m. on Thursday ∴ From the answer choices the present
The watch gains (4 + 6) = 10 minutes in 25. Time from 3:00 p.m. Sunday to 9:00 p.m. Tuesday All the remaining days occur for only 52
to 8:00 a.m. on Monday = 84 hours. = 54 hours.time which The watch satisfied
loses (5 the+given
10) = 15condition
minutes is
85 hours
In 84 hours, the clock gained 12 minutes. times. Choice (1)
But to ∴ show
5
in 54 hours. hours 15 minutes. Choice (1)
Thethe correct
watch time,4 the
gains clock has
minutes to gain
in [4/10 x 85] ? The watch loses 5 min in 5/15 x 54 = 18 hours 10. The leap year occurs for every four years.
5 minutes. 23. Difference in minutes between the two
= 34 hours. (shows correct time)
? 5/12 u 84 = 35 hours.
18 hours from 3:00 p.m. Sunday is 9:00 a.m. on So, 2096 + 4 = 2100 but 2100 is a century
Monday, clocks which is when
in one hour the clock shows the
= 3 minutes.
35 hours 34 from hours = 1p.m.
8:00 dayon10Thursday
hours. is 7:00 a.m. correct time. Choice (4) year which is not divisible by 400. Hence,
on Saturday.
1 day 10 hours from 6:00 a.m. Monday = Choice (1) N
 umber of hours from 8:00 a.m. to 12:00 2100 is not a leap year.
Exercise
noon on that day = 4 hours. 7
4:00 p.m. Tuesday. Choice (2) 2100 + 4 = 2104 is the next leap year after
The two clocks differ by = 3 x 4 = Sol/916 12 minutes. 2096. Choice (3)
17. The number of hours from 8:00 p.m. on
 Choice (3)
Thursday to 8:00 a.m. on Monday = 84 11. As we do not know whether the year is
hours. 24. Let the present time be x O’clock. a leap year or not, the day of the week
In 84 hours, the clock gained 12 minutes. ∴ 3(10 − (x + 4)) = 10 − x on 14th July in that year cannot be deter-
But to show the correct time, the clock 3(6 − x) = 10 − x mined. Choice (4)
has to gain 5 minutes. 18 − 3x = 10 − x 12. Number of odd days from 1st January,
∴ 5/12 × 84 = 35 hours. 8 = 2x, x = 4 2012 to1st January, 2016 are 5. 5th day
35 hours from 8:00 p.m. on Thursday is ∴ The present time is 4 O’clock. after Sunday is Friday. Choice (1)
7:00 a.m. on Saturday. Choice (1)  Choice (2)
13. The number of days from 10th April,
18. Time from 4:00 a.m. on 6th of this month 25. Time from 3:00 p.m. Sunday to 9:00 p.m. 1963 to 23rd August, 1963
to 6:00 p.m. on 10th of this month = 110 Tuesday = 54 hours. The watch loses (5 + Month: April + May + June + July +
hours. 10) = 15 minutes in 54 hours. August

08_Section.indb 121 10/30/2009 11:53:50 AM


of 2022. Choice (2) of his siste
25. Number of days from 3rd April, 1995 to 1st 7. Ravi’s fath
October, 1995 in-law is
Month: April + May + June + July + August + Paternal g
September + October mother is
Odd days = 6 + 3 + 2 + 3 + 3 + 2 + 1 = 20 days that is, Ra
= 6 odd days siblings. H
122  n  Solution Manual 6th day after Monday is a Sunday.
Hence, 1st October, 1995 is a Sunday. 8. My mothe
I can meet my friend on the 1st, 8th, 15th, 22nd, aunt’s son
Days: 20 + 31 + 30 + 31 + 23  umber of odd days from 1st January, 1978
N 29th of October.
father also my co
= 135 to 18th July, 1978 : Choice (2) (2)
23rd August 1963 is 2 days to Wednesday Month: Jan + Feb + Mar + Apr + May + brother
Exercise
father8
mother (Blood Relation) 9. Given Shil
that is, Friday Jun + July Only sibling sister-in-la
The number of odd days form 23rd Au- Odd days: 3 + 0 + 3 + 2 + 3 + 2 Solutions for questions 1 to 9: her sister-i
gust 1963 to 23rd August 1959 is five. + 4 = 17 me
1. My father’s brother’s only sibling is my father, husband is
Hence, 23rd August 1959 is five days Total number of odd days = 1 + 96 + 17 = whose
 father-in-law is the father of my mother.
Choice (2) Solutions for
114 days father
back to Friday is Sunday. Choice (1)
Number of odd days in 114 days = 114/7 2. My brother’s grandfather is my grand- 10. P{ }Q[ ]R>S
14. 20th January, 2000 is a Thursday. father either paternal brother R is the au
= 16 weeks + 2 odd days. 2nd odd day is father or maternal. My and S is
20th January, 1997 is 3 days before Thurs- mother
grandfather’s only sonOnlyhassibling
one child
a Tuesday. mother of S
day, that is, Monday. Hence, 18th July, 1978 is a Tuesday. only. Therefore he has to be my maternal
The number of odd days from 20th Janu-  Choice (3) uncle only.meMy uncle’s son is my cousin.
Choice (2) 11. S z T < U >
ary, 1997 to 26th February, 1997 is 2. (As S is the s
2. My brother’s grandfather is my grandfather
Choice either
(3)
calculated in question 13) 20. The method of calculation is as shown in father of V
paternal
3. Ravi’sormother’s
maternal.brother’s
My grandfather’s only son
father is Ravi’s
2nd day after Monday is Wednesday. question 19. has one child only. Therefore he has to be my
There are 5 odd days from 1st January 1 mother’s father that is Ravi’s maternal
maternal uncle only. My uncle’s son is my cousin.
 Choice (3)
AD to 24th may 2019. The 5th day from grand-father. His only daughterChoice
is Ravi’s
(3)
15. The number of odd days from 12th Janu- Sundays is Friday. mother,
3. Ravi’s whose
mother’s daughter
brother’s fatherisisRavi’s sister.
Ravi’s mother’s
ary in 2007 to 22nd February 2007 ⇒ Hence, 24th May 2019 is Friday.  that is Ravi’s maternal grand-father.
father Choice (2)His
Jan(5) + Feb(1) = 6 odd days.  Choice (2) only daughter is Ravi’s mother, whose daughter 12. X z Y ( ) Z
4. Fromsister.
is Ravi’s the given data, we get the following
Choice (2)
Hence, 22nd February 2007 is 6 days Thus X is t
21. The number of odd days is 35 days diagrams.
from Friday that is Thursday. 4. From the given data, we get the following
13. A ( ) B { } C
35 days have ‘0’ odd days. diagrams.
From 22nd February 2007 to 22nd Febru- A C who in tu
Hence, 26th March, 2023 is a Sunday. B
ary 2008, number of odd days is 1. (husband) (wife) Thus A is t
 Choice (1)
Hence, 22nd February 2008 is one day Solutions for
C
from Thursday that is Friday. 22. A non leap year starts and ends with the
Given,
 Choice (1) same day. Hence, the year is a non leap
S T means S
year. Choice (1) E (daughter)
16. The number of odd days from 2nd De- brother of T
Clearly, B is the grandmother of E. Choice (2) S y T means S
cember 2008 to 2nd December 2009 = 1 23. The year 2008 is a leap year and a leap Clearly, B is the grandmother of E. the father o
odd day. year repeats it self after 28 years (7 x 4 = 5. My mother’s brother’s wife’s son is Choice
my maternal
(2)
Hence, 2nd December of 2008 is one day 28). uncle’s son. My father’s father-in-law is father of 14. A + C = B
2008 + 28 = 2036 5.
my My mother’s
maternal uncle,brother’s wife’s
whose son son is of
is grandson mymy
to Sunday that is Saturday. father’s father-in-law.
Hence, 2036 will have the same calendar maternal uncle’s son. My father’s father-
 Choice (4)
as that of 2008. Choice (4) in-law is father of my maternal uncle,
17. The number of odd days from 31st January whose son is grandson of my father’s
1995 to 30th July, 1995 is 5. (As calculated 24. Year 2022+23+24+25+26+27+28+29+30 father-in-law.
in question 13). +31+32+33
5 odd days after Tuesday is Sunday. Odd days :
The number of odd days from 30th July, 23. The1 year
1  2008
2  1 is a1 leap1  year
2  1 
and1 a leap
1  2 year1 Father

1995 to 30th July, 1993 is 2. As the


repeats number
it self after 28ofyears
odd(7days
x 4 =from
28). 2022 to
2008 + 28is=14
2033 2036
⇒ 14/7 = 0 odd days.
2nd day before Sunday is Friday. Hence, 2036 will have the same calendar as that of
 Choice (3) H

2008. ence, 2033 will have the same calendar
Choice (4)
Father Mother brother Wife

as that of 2022. Choice (2) Spouse


24. Year
18. The number of odd days from 18th April 2022+23+24+25+26+27+28+29+30+31+32+33
2007 to 16th July in 2007 is 5. (As calcu- 25. Number of days from 3rd April, 1995 to brother Me Son
Odd days :
lated in question 13) 1 1st1 October,
2 1 1995 1 1 2 1 1 1 2 1
As M
theonth:
numberApril + May
of odd + June
days from 2022+toJuly
2033+isAu-
14  Choice(1)(1)
Choice
Hence, 16th July 2007 is 5 days from Fri- Ÿ 14/7
day. that is, Wednesday. gust=+0September
odd days. + October
6. 6. TheThe
man’s sister’s
man’s husband
sister’s is the is
husband man’s brother-
the man’s
Hence, 2033 will have the same calendar as that in-law. Brother-in-law's wife is his sister. Daughter
Odd days = 6 + 3 + 2 + 3 + 3 + 2 +Choice
of 2022. 1 = 20 (2)
days brother-in-law. Brother-in-law’s wife is
The number of odd days from 16th July, of his sister is his niece. Choice (2)
= 6 odd days his sister. Daughter of his sister is his
2004 to 16th July, 2007 is 3. Hence, 16th 25. Number of days from 3rd April, 1995 to 1st 7. Ravi’s father’s wife is Ravi’s mother. Her mother-
6th day
October, 1995after Monday is a Sunday. niece.
July 2004 is three days to Wednesday that Month: April1st
+ October,
May + June + July + August +
in-law is Ravi’s father’s mother that is Ravi’s
Hence, 1995 is a Sunday. 
Paternal grand mother. Ravi’s paternal Choice (2)
grand
is Sunday. Choice (1) September + October mother is also Shyam’s maternal grand mother
I
 can meet my friend on the
Odd days = 6 + 3 + 2 + 3 + 3 + 2 + 1 =1st, 8th,
20 15th,
days
19. 18th July 1978: 7. thatRavi’s father’s
is, Ravi’s wife
father andis Shyam’s
Ravi’s mother.
mother Her
are
= 622nd, 29th of October.
odd days Choice (2) siblings. Hence, RamisisRavi’s
Shyam’sfather’s
cousin. mother
mother-in-law
1600 + 300 + 77 + number of days from 1st 6th day after Monday is a Sunday. Choice (3)
January 1978 to 18th July 1978.
Hence, 1st October, 1995 is a Sunday. 8. Mythat is Ravi’s
mother’s sister’s Paternal
husband’s grand mother.
only son is my
I can meet my friend on the 1st, 8th, 15th, 22nd, Ravi’s
aunt’s son paternal grand
that is cousin. My mother
cousin’s is also
sister is
1600 years has ‘0’ odd days. Blood Relation
29th of October.
Choice (2)
also my cousin.
Shyam’s maternal grand mother Choice that is,
300 years has ‘1’ odd day. (2)
Ravi’s father and Shyam’s mother are
In 77 years, there are 19 leap years and 58 Exercise
Practice 8
Exercise 9. Given Shilpa Hence,
is the only child
(Blood Relation) siblings. Ram is ofShyam’s
her parents. Her
cousin.
non-leap years. sister-in-law is the sister of her husband. Hence if
One leap year has ‘2’ odd days and a non- Solutions forquestions
Solutions for questions1 1toto
9: 9: her sister-in-law is the daughter of Rahul Choice (3)
then her

leap year has ‘1’ odd day. 1.


1.
My father’s brother’s only sibling is my father,
Myfather-in-law
father’s brother’s only 8. husband is the son of Rahul.
My mother’s sister’s husband’sChoice (1)
only son
whose is the father of sibling is my
my mother.
Number of odd days in 77 years = 19 x 2 father, whose father-in-law is the father of
father
is my
Solutions for aunt’s son 10
questions that
to is
13:cousin. My cous-

+ 58 x 1 = 96 my mother. 10. P{ in’s sisterPisis also


}Q[ ]R>S, my cousin.
the mother of Q. Choice (2)
brother R is the aunt of Q. Thus R is the sister in law of P
father and S is the nephew of R. Thus P is aunt or
mother
Only sibling mother of S. Choice (4)

me Choice (2) 11. S z T < U > V


S is the sister of T, U is the son of T, U is the
2. My brother’s grandfather is my grandfather either father of V. Sister
08_Section.indb 122 T S 10/30/2009 11:53:51 AM
paternal or maternal. My grandfather’s only son
2 1 So, isXAisisthe
that thegrandfather
uncle of B. of Y and Choice Solutions for questions
Emanuel 22 to 25: (wife)
to 2033 is 14 Choice (1) that(2) is A is(2) Xthe= Zuncle xYŸ ofX B.father of Z mother Choiceof Y Solutions for questions 22 to 25:
From
From the givendata,
the given data, wewe getget the following
the following tree diagram.
tree diagram.
(2)
6. The man’s sister’s husband is the man’s brother- 15. X =15. Z =XSo, Y ZŸ
= X= X Y the
is father of Zof father
father
Xgrandfather Z
of Y. fatherof ofY Choice Y (3)From the given data, we get the following tree diagram.
ndar as that (husband)
(husband) FredaFreda
in-law. Brother-in-law's wife is his sister. Daughter 15. X =So, Z =16. XYis Ÿ
So, X father
X isgrandfather
the of Z offather
the grandfather of
Y Yand andof Y (wife)
Choice (2) Q y P x R Ÿ Q daughter of P mother of R Anna
(husband)
Emanuel
Emanuel Freda(wife)
of his sister is his niece. Choice (2) Brian (son)
So, XX=isZthe xX Y X X father
= ZŸx Y father
grandfather ofofY and of
Z mother Z mother of ofY Y (daughter-in-law)
Emanuel (wife)
1995 to 1st So, P is the mother of Q and R Choice (2)
7. Ravi’s father’s wife is Ravi’s mother. Her mother- X = Z x Y Ÿ So,XX isfather of Z ofmother
the grandfather of Y. of Y Choice (3)
in-law is Ravi’s father’s mother that is Ravi’s So, X is the grandfather Y. Choice (3)
17. P R + Q Ÿ P sister of R brother of Q
Solutions friend   n  123
+ August + Paternal grand mother. Ravi’s paternal grand 16. So, Q 16.the
X yis  P x R  Q ofdaughter
Q grandfather Y. of Choice P mother (3) offorR Intelligence Anna and Critical BrianReasoning
(son)
P xSo, RŸ P isQthe daughter
sister of Q.of P mother of (4)
Choice R Anna
(daughter-in-law) Brian (son) Gerhard
mother is also Shyam’s maternal grand mother So, P is the mother of Q and R Choice (2) Dimitri Celina
1 = 20 days 16. Q y P x R Ÿ Q daughter of P mother of R Anna (daughter-in-law)
(son)
that is, Ravi’s father and Shyam’s mother are So, P is the mother of Q and R Choice (2) (daughter)
Brian (son)
siblings.
9. Given Shilpa Solutions for
(1),questions 18 to R 21: 25. Anna is the daughter-in-law of Emanuel.
Hence, Ramis is the
Shyam’s child of her par- 17.
onlycousin.
So, PP isRthe
PFromR+Q
mother
P sister of
ofsister
Q and R
brother o f Q
Choice (2)
(daughter-in-law)
ents. Her sister-in-law is the sister Choiceof (3)her17. These + Q Ÿ P of R brother of Q 22. Freda
 is the mother offriend
Brian. Choice (1)
Choice (2)
So, P isquestions
the sister of Q. are based onChoice the(4) following Dimitri Celina Gerhard
. 17. R +information. Siblings
sister of R( )brother Sisters Q
ofChoice friend
h, 15th, 22nd,
8. My mother’s
husband.sister’sHencehusband’s
if her sister-in-law
only son is is my theP So, PQisŸ AtheP sister Siblings
of
A for questions 18 Q. ( )to 21: Sisters (4)
BB
23.
(son)
Dimitri Gerhard is the
(daughter)friend
Celina Gerhard and
of Celina and
Gerhard
there is no
aunt’s son that is cousin. My cousin’s sister is Solutions relationship betweenfriend Dimitri.
daughter of Rahul then her husband is theSo, P isFrom the sister
(1), of Q. Choice (4) (son)
Dimitri (daughter)
also my cousin.
(2) son of Rahul.
Choice Solutions
Choice (1) These forquestions
questionsare18 based to 21: on the following
(son) Direction Sense
Celina
22. Freda is the mother of Brian.
(daughter)
Gerhard
Choice (2) Choice (4)
Solutions information. Child
Child 22. Freda is isthe mother of Celina
Brian.and there Choice no (2)
These for questions
questions A are18 to
Siblings 21:
based ( ) on Sisters the following B
23.24.
Gerhard
Emanuel theis friend of
the grandfather of Celina. isChoice (4)
relationship between Gerhard and Dimitri.
9.Solutions
Given Shilpa for questions 10 toof13:
is the only child her parents. Her information. Siblings I 22. Freda
23. is
Gerhard the mother
is theP of
friendBrian. E
of Celina Choice (2)
These questions From (1), are based
A
I
( ) on the following Sisters ractice
25. Anna is the daughter-in-law of Emanuel. xercise Choice (4) is no
and there
sister-in-law is the sister of her husband. Henceinformation. if B relationship between ofGerhard and Dimitri. Choice
is no(1)
10. P{ }Q[ ]R>S,
her sister-in-law is theP daughter
is the mother
of Rahul Q. her FromA (1), Siblings
of then ( ) Sisters
Child
B
23. Gerhard
24.
is isthe
Emanuel for
Solutions
friend
thequestions
grandfather1ofto
CelinaCelina.
and there
ChoiceChoice
19:Dimitri. (4) (4)
From (2) and (3) E(f) relationship between Gerhard
Exercise 9 and
is my father, husband is the son of Rahul. Choice (1)From (1), I
R is the aunt of Q. Thus R is the sister in Child 25. Anna is the daughter-in-law
24. Emanuel is the grandfather (Direction of Emanuel. Choice
of Celina. Choice (4)
Sense) (4)
my mother. oA Choice (1)
Solutions for questions 10 to 13: (D)(f) Child
law of P and S is the nephew of R. Thus I(D)(f) ( )
(G)24. Emanuel
25. Solutions
Anna
is the
is the
grandfather
questions 1oftoCelina.
fordaughter-in-law 19: Choice (4)
From (2) and (3) E(f) 10
Exercise 9 of Emanuel.
10. P{ }Q[P]R>S, is aunt or mother
P is the mother of ofQ.S. Choice (4) I 1.1. is the daughter-in-law
25. Anna (Direction Sense) oA
of Emanuel. Choice (1)
other R is the aunt of Q. Thus R is the sister in law of P B
From (2) From
(D)and(f) (3) (3) J F (G) Choice (1)
11.
and SS ≠is Tthe <U > V of R. Thus P is aunt or
nephew (D) E(f)
(f) ( ) Solutions for questions 3 Exercise
1 to 19: 9 10
2 2 (Direction Sense)
sibling mother of S. Choice (4) From (2) and (3) E(f) Exercise B 9o A
S is the sister of T, U is the son of T, U is(D) son son
1.
(f) From (3) J (G) Solutions for5(Direction questions Sense)
1 to 3
19:
the father of V. (D)(f) F ( ) 2 2 10
Choice (2) 11. S  T < U > V (D)(f) (G)
S is the sister of T, U is the son of T, U is the (D)(f) (H)( (f)) Solutions
1. for
L
 etquestions
A and B B 1be to the
19: initial
oA and the final
dfather either son son 5
father of V. Sister From (3) J grand daughter 1. points
Let A and respectively
B be the inA the
2 3oinitial journey.
and the final points
her’s only son T S F 2 10
, I, respectively in the journey.
as to be my Mother From (3) F Jhas two sons who cannot
(H) be,
F J thus A and C are sons of E and F. any one among Distance he is away from theplace:
original
G, E, F, H and (f)
Distance he isBaway 5 from10 the original
is my cousin. U B,whoseI,son E, F, Hgrand
G, daughter and daughter son
is B.J From thus A (1)and I is C theare sons ofof
daughter place:
LetVertically
A and B o be 10 thekm initial and the final points
Choice (3) FC.(from
Eson
has two3),
and
sonsG iswho the cannot
wife of be,A,any and oneB among
is the(1)widow
B, I, and respectively inBthe 2o journey. 2= 83km
F,F.Hchild
whose daughter isson B. From I is the Vertically →from
Horizontally 510+ 3km Choice (1)
Father G,
J isE, the and Jofthus C and A and D. C are sons of E and F. Distance he is away the original place:
3
daughter ofgrand
C.(from (H) 3), G (1)isI the wife of A, and
of B 2 2
vi’s mother’s whose daughter is B. From
daughter
(f) is the daughter
2. Horizontally
Vertically
The 
path B 10 km
travelled → 5 5 +
byinitial3 = 8
Rajan is asthe km follows :
V Choice (3) husband Let A and
d-father. His isC.(from
thesons 3), G is the
widow (H)wife
and J(f) isofthe A, and B is the widow
child of C and m D. and Male Horizontally  5be + the kmI and and final
(1) bepoints
F has two grand who C(E)
cannot
of daughter be, any (F)one(m) among B, I,
 the initial
Let 53 journey.
point = 8be Choice
the final Choice
point (1)
F.
J is the child and
Femal Let respectively in the
(f) D.
ose daughter 12. X  Y ( ) Z. X is sister of Y who is the father Choice
of Z. (3)G, E, F, H and J thus A and C are sons of E
f
and F.
A and B be
Distance he is
the
awayI initial and the final points
from the original place:
Choice (2) ThusX X is F has two sons who cannot be,husband any one among m B, I, Male e 2. The path
2. The
respectivelypath in thetravelled
travelled journey. by Rajan is as fol-
by Rajan is as follows :
12. ≠ the
Y (aunt
) Z. ofXZ.is sister of Y who Choiceis (1)
theG, E,whose
F, H daughter
and J thus is (E)
AB.and From C are (1) sonsI(F)is(m)the
of Edaughter of Let the initial
Verticallyhe isoaway point
10 km be I and the final point be F.
fand F.and Distance lows: from the original place:
(f)
the following C.(from 3), G is the wife of A, and B is the widow Female I
13. A ( ) Bfather
{ } C of Z.AThus
 D. X isofthe
is father auntis of
B, who Z. whose
mother of daughter is B. From (1) I is the daughter of Horizontally
Vertically o 10 o
km 5 + 3 = 8 km Choice (1)
J is the
C.(from 3), child
daughter
G is of Cwife
the andofD.A,son and B is the son
widow and
LetF the initial point be I and the final point
C who  in turn is the sister of D. Choice (1) wife Horizontally o 56 + km3 = 8 km Choice (1)
Choice (4)J is the child of C 2. The be path F. travelled by Rajan is as follows :
Thus A is the grandfather of D. (B)and(f)
D. husband
(C)(m) (D)m(f) (A)(m)Male(G)(f) Fthe initial point7be kmI and the final point be F.
13. A ( ) B { } C ≠ D. A is father of B, who daughter (E)(f) husband son (F)(m) son 2. TheLet path travelled
6 km by Rajan is as follows :
Solutions for questions 14 to 17: daughter m f Male Femal
wife
Let the initial pointI be I and the final point be F.
is mother of C who in turn is the sister of (E)(B)(f)daughter
(f) (F)
(C)(m) (D)
sister
(f) (A) (m) e (G) (f)
7 km 4 km
(m)
f Femal I
Given, D.
(H)(f) daughter (I)(f) (J) e
S  T means S is sister of T, S + T means S is the daughter 4 km
Thus A is the grandfather of D.
brother of T, S  T means S is the son of T, daughter
18. (C, D) is a couple. sister Choice (4)
F
(H) (f) son (I) (f) son (J) 6 km 3 km
Choice (2) S  T means  S is the daughter of T, S = T means Choiceis (4)
Sdaughter 19. (I, J)
18. (C, D) isson is a pair
a couple. of siblings. son
wife
Choice (3) F ? Finally he is facing towards West. Choice (2)
(B) (C)(m) (D)(f) (A)Choice (4)
(G)(f) 6 km
the father of T, S x T means S is the mother of T. (f) wife
(m) 7 km 3 km
my maternal Solutions for questions 14 to 17: 18.
(B)(f) 19.20. (I,As (C, we
J) is D)
a pair is
do of a couple.
notsiblings.
know (D) the(f)gender
(A)(m) Choice
Choice
of J(G) it (3)
cannot (4) be 3. Finally he is facing 10 towards West. Choice (2)
(C)
daughter
(m)
(f)
7 km
w is father of 14. A + C = B  A brother of C father of B 19. determined, whether J is the niece or nephew of xo A
Given, daughter
20. B.(I,
As weJ) doisnot
daughtera pairknowofthe siblings.
sistergender of J it cannot Choice
Choice (3) 3.
be (4) 42 km
andson of my 10 B
S ∗ T means S is sister of T, S + T means S is the daughter (H)(f) determined, whether (I)(f)J is the(J)niece or nephew of
sister 15 o A 4 km
20.
21. B
B. As
is we
the do
mother not of know
H. the gender of J
Choice
Choice (4) it (3) 13
2
brother of T, S − T means S is the son of T,18.(H)(C, (f) D) is a couple.
(I)(f) (J) Choice (4) 15 10 B
cannot be determined, whether J is the
S ÷ T means S is the daughter of T, S = TSol/918 means
18. (C, D) 21. is aBcouple.
is the mother of H. Choice Choice(4)(3) ? Finally he is 13
3 km
19. (I, J) is aniece pair oforsiblings.
nephew of B. Choice Choice (3) (4)
∴ Finally 10 facing
he is facing
towards West. Choice
3 km towards West.
(2)
Sol/919
S is the father of T, S x T means S is19. the(I, J) is a pair of siblings. Choice (3) ? Finally he is facing towards West. Choice (2)
mother of T. 20. As we do not
21. B is the mother of H. know(husband) the gender of J it cannot
Choice (3)
Freda be 3.  10 Sol/919
Choice (2)
20. As determined,
we do not know whether theEmanuel J is the of
gender niece or nephew
J it cannot be of 3.
(wife) xo A
(husband) Freda 10 2
14. A + C = B ⇒ A brother of C father of determined, B.Solutions whetherfor questions
J is the niece
Emanuel 22 to or 25:nephewChoiceof
(wife)
(4) xo A B
3. 15 2
B B. From the given data, we get the
21. B isSolutions
the motherfor of questions
H.
Choice (4)
following
Choice (3) tree B
that is A is the uncle of B. Choice 22 to 25: 15 13
(2) that is A is the uncle of B. Choice (2)
21. B is the diagram.mother of H. Choice (3)
10
13
From the given data, we get the following tree diagram. 10
15. X = Z = Y ⇒ X father
15. X = Z = Y Ÿ X father of Z father of Y of Z father of Sol/919
Y (husband)
(husband) Freda
Freda
L
 et A and B be the initial and the Sol/919
final points
So, X is the grandfather of Y and Emanuel (wife)
X = ZSo, x YXŸisXthe grandfather of Y and
father of Z mother of Y (husband) Emanuel Freda (wife) in the journey. The vertical distance between
mother of Emanuel (wife) A and B is (15 − 13) = 2 m Choice (4)
So, XXis=the Y ⇒ X father
Z 3grandfather of Y. of Z Choice (3)
Y 4. The path travelled by Reema is as
16. Q y P x R Ÿ Q daughter of P mother of R Anna Brian (son)
So, X is the grandfather of Y. (daughter-in-law) follows:
So, P is the mother of Q and R Choice
Choice (2)
(3) Let I and F be the Initial and final point re-
17. P R + Q Ÿ P sister daughter of
of R brother of Q spectively.
16. Q ÷ P 3 R ⇒ P P
friend
So, Pmother of ofRQ.
is the sister Choice (4) Dimitri Celina Gerhard 4 km
(son) (daughter)
So,forP questions
Solutions is the mother of 21:
18 to Q and R

These questions are based on the following
Choice (2) 22. Freda is the mother of Brian. Choice (2) 4 km
7 km
information. 22. Freda is is
thethe
mother
friendofofBrian.
Celina Choice (2)is no
+ Q ⇒ P sister
17. AP ∗ RSiblings ( ) of Sisters
R brother of 23. Gerhard and there
F
B relationship between Gerhard and Dimitri.
From (1), Q 23. Gerhard is the friend of Celina and there
Choice (4)
So, P is the sister Child
of Q. Choice (4) is no relationship between Gerhard and 
5 km
24. Emanuel
Dimitri.is the grandfather of Celina.
Choice (4) (4)
Choice
Solutions for questionsI 18 to 21:  he direction in which Reema started
T
25. Anna
24. is the is
Emanuel daughter-in-law of Emanuel.
the grandfather of Celina.
These questions are based on the following in- Choice (1) her journey is 900 in anticlockwise from
formation.  Choice (4) south that is east.
From (2) and (3) E(f) Exercise 9
(Direction Sense)
(D)(f) (G)
(D)(f) ( ) Solutions for questions 1 to 19:
1. oA
From (3) J F 10
08_Section.indb 123 10/30/2009 11:53:54 AM
B
Let I and F be the Initial and final point respectively. Let O and H be office and house respectively.
4 km 4 km The path traversed by Ramesh is shown 2 in above 2 2
Let A and B be the initial and the final 7 km points in the figure by solidWe line. know that, (OH) = (BH)  (OB) and
journey. The vertical distance between A and B is Let O and H be BH = 60
office and–30 house = 30, OB = 80 + (70 –50) = 100
respectively.
4 km
(15  13) = 2Fm
2 2 2
7 km Choice (4) We know that, (OH) = (BH) 2 (OB) and 2
30, OB== 8030  100
–50) = 100 10900 = 10 109
Hence, at
Let A and B be the initial and the final points in the BH = 60 –30 =?OH + (70
the North-E
journey. 4. The Thevertical
path , Fxtravelled
distancebybetween Reema A is as andfollows
B is :
?OH = 30 ?
2
 100 The house
2
10900 =is 10 towards
109 South-west from the
The   =Let direction inthe which5 kmand Reema started her journey
0   ,Solution
(15124 13) n2m I and F be Initial
Manual final
Choice point (4)respectively. 18. Given, the
isThe90direction x
in anticlockwisein which 5 kmReema 4 km from south
started herthatjourneyis east. ? The The house office.
path traversed is towards by Ramesh
South-west is shown from intheabove Choice (4) points towa
4. The path travelled 0 by Reema is as follows : office. figure by solid line. Choice (4)
Let I? She started Initialher journey towards east.
is 90 in anticlockwise from south that is east. N, E, S, W, NE, SE, NW and SW are the pointers
and
? She
F ∴bestarted
the started
She 4 km
and
her journeyher final point respectively.
journey
towardstowards east. east. ∴OH Let=O9.and 302 +H100 2
be office= 10900
and house = 10 NE respectively. of the the compass
compass which is showing point west North,thus East, allSouth,
the that is
4 km Choice (4)
The
9. path We traversed by
that,NERamesh 2 is shown 2 in above
2
 7Choice
km
Choice (4)
(4) 109 know
figure by solid line.
(OH) = (BH)  (OB) and West, remainingNorth-east, pointers South-east,
also point North-west
as shown and
BH = 60 –30 = 30, OB = 80 + (70 –50) = 100 South-west respectively. But now the pointer NE
5.5. F 20 20 Let O and H be10 office and house 10
10 respectively. 10 above. Thus the pointer E(east) points
4 km ∴ The house = 30 is
2 towards
2
 10022 (OB)South-west
10900 from of the compass is showing west thus all the
We know?OH and = 10 109
2
7 km that, (OH) = (BH) SE SE North-west. Hence, Atul astravelled to- Hence, at 6:
5. 55 , x the office. Choice (4) from the remaining pointers also point shown above.
10
5okmReema10 5
started her5journey BH = 60NW –30
? =
The 30, OB
house = 80
is + (70
towards –50) A = 100
South-west
House the North-Ea
The direction in which
F0 NW ThusAwardsHouse
the North-west.
pointer E(east) pointsChoice North-west.(4)
Let A is and 90B 8 in
beanticlockwise fromo south that theeast. ?OH
in is
office.
9. = 30  100 2 2
10900 = 10 109
Choice (4) Hence, Atul travelled towards North-west. Similarly,
18. Given, the c
journey. ? She vertical 8the Binitial
started her
and
A the
B journey A
final
towards
points
east.is 10 NE 10 13. Given that a compass was damaged Choiceand (4)its toward Sou
The (15 Let A, xThe
direction andin B5 be
which km
distance
the initial
Reema
between
and her
started theA and
finalB points
journey ? The 9. house is towards NESouth-west
10 from the 10 points toward
LAet=and
 13) A2 mand B be the initial Choice andandthe (4) finalfinal
Choice (4) E,needle turned SE,inNW such a way thattheit showed
is 90Let in B be the
southinitial the points
0respectively thefrom
journey. office. Choice (4) 13. N,Given S,that
W, NE, a compass wasand SW are
damaged and pointers
its needle
in anticlockwise that is east. SW
4. She
? He ispoints
respectively
5. started
The path 20travelled
her respectively
(8 +in10)by
journey theReema
in
2journey.
= towards
km the
20isaway
journey.
aseast.from the
follows : starting From 10 the given 10 description 10 it is clear that the ofturned
10 theNorthcompass for East
in such awhich
wayandthat so on.
point North, East,
it showed North South,
for East
that is
point.
Let I and H
 e Fisbe 20 the− (8 +
Initial 10)
and = 2
final km
point away from
Choice
respectively. the
(4) 9. person startsNE from the same SEplaceSW SE and reaches West, and So,so North-east,
the
on. original South-east,
directions North-west and Hence, at 6:0
He is 20 5  (8 + 10) = 2 km Choice
away (4)
from the starting NW
the starting
From the given
A House description it is
South-west clear that
respectively. the But now the pointer NE
the North-Eas
6. The starting path traversed by the person10 is as follows: 5 The path NW point.
traversed by Ramesh Hence, the distance
is shown A House
in above between W
the sameofSo, the original directions
point. 4 km
5. point. o 2 km Choice (4) Choicefigure (4) the 10 line.Station
by Railway
solid person 10 starts
and his house from is 0 km. place
the compassand reaches is showing west thus all the 18. Given, the clo
F 20
8 A
B by the A person is as follows: the andstarting 10 respectively.
SE point. Hence, (2) the distance
remaining N pointers between also point as shown W above.
6. The 56. The
path
41 km path
km traversed
traversed by the person is as fol- Let O and H10 be office
210
house 10 Choice points toward
Let A and B be the 10 initial 5and2the final points We NWknow that, (OH)the = (BH)
2
 (OB)
2
A Station
House
and Thus N, E,is the
S,0W, pointer
NE, SE, E(east)
NW and points SW are North-west.
the pointers
lows:
Frespectively o journey. 7 km km A BH 10.= 60 –30 = 30, OB SW= 80 Railway
Post Office and his house km. became
Similarly, a
in the +
SW (70 –50) = 100 Hence,
of theAtul travelled
compass towards
which point North-west.
North, East, South, that is
8
He1iskm 20 FB (8 +3A10) km
= 2 km away from the starting ?OHF=rom From the2 given 10 o
description itclear
is clear that the Choice
WWest, North-east, E
(2)South-east, S Choice N
North-west (4)
and toward Sout
3 km 2 km 10
30 the
2
 given
100 description
10900 = 10 it is
109 that
Let A and Fpoint. B be the initial and the final points AChoice (4) person starts from the same place and reachesSouth-west respectively. But now the pointer NE 19. Let N, E, W
4 km , x the
? The house person 10. starts
is towards from the same place Post and Office
13. Given that a compass was damaged and its needle
respectively
The
6. direction
The in theinjourney.
1 path
km traversed 5 kmReema
which started her is journey the starting SW point.South-west Hence, thefrom the
distance betweenof the compass is showing west thus all the
He is 900in(8anticlockwise
is 20 + 10) = 2 km from3by
away km the
southfrom
person
thatthe
as follows:
starting A
2is km
east. office.reaches
From thethe giventhe30starting
Railway description
Station
30 Choice
point.
it isHence,
and his clear
house the
(4)
thatisdis- o
0the
km. turned in Ssuchpointers
remaining a way that also itpoint
showed North
as shownE for East
above.
compass
F and so on.the pointer E(east) points North-west. West respe
point. ? She started3her km journey towards east. Choice (4) person tancestarts from thethe
between sameRailwayplace Station
and reaches Choice (2)
and Thus
1 km 3 km 9. NEHence, the distance between A
So,Hence,A person
person
the walked
original walked towards
directions
Atul travelled towards
towards
West, West, which is
which
North-west.
is actually atW
As the poi
Similarly,
6. The path traversed 4 km 3 kmby 2the person is as Choice follows:(4) the starting point.
his North. Choice(4) (4)
km 2 km 10. house
the Railway Stationis 0 and km.his house Post Officeis 0Choice
km. (2)
30 actually North. Choice
Choice (4) showing
toward E
South
F 4 km A 10 N W
5. 20 3 km 10 o
30 Choice (2) 14. InGiven
the thatevening as the sun is in and the its West the
1 km 10. 20 SE 13.
14. In the eveninga compass becameaswasthedamaged
sun is in the West
needle
3 km shadow
B5 1 4 kmC 2 D 10 2 5 E
10. NWJatin is walking Posttowards
Officethe East.
A House Choice (4) turned infalls suchtowards
a way that East. As shadow
it showed NorthofforShyamEast
o the shadow falls towards East. As shadow
We know that3 km 30 Wisand falling
so on. to his left and his
E S left is east. ? N He is
(AB)23=km
8
(BE)2  B(AE) 2A 11. The path traversed 30 o10by Sameer is as follows:
So,ofthe
facing Shyam
south andisdirections
falling
Shaan is tofacing
his left and his left
North. W
2 km 2andkm where 10H is the house and O is the Office.
original
N, E, S
19. Let West
Let
from A and B be the initial
4 kmfigure, BE = BC  CD  DE = 1  2  2 = 5 and the final points Choice (2)
respectively in the journey. 4 km 20
H is east.N ∴ He is facing south and W Shaan is compass w
km B SW 30 S E
He 1 + 10) C = 22 km 2 km D 2 the starting E From the given 30 description it is clear that the 15. Given facingthatNorth.
– Piyush becameand Ravi were facing each West respec
AE Bis=20F B (8
point.
1= 4 km C away from
2 D Choice 2 (4) E person starts from the same 20
Jatin is walking place and reaches
towards the East. A other.
personPiyush's Choicetowards
walked (4)
shadow West,
fell towhich is actually
his left (that is As the poin
6. The We know
We know
AB B=traversed
? path that 2 that 2
51  by4C the 2person 41D km.is as 2follows: Choice the starting Npoint. Hence, towardsthe distance betweenChoice (4)towards  North.
W West as E it was morning). S So,Choice
Choice
Piyush N(4)(2)
was showing Eas
2 2 2 km 2 2 2 2 E Jatin is
11. walking
The path the
traversed East. by Sameer is as follows:
(AB) F (AB)
We = (BE)
know = (BE) that + (AE)and
(AE) 2 and
km A the Railway Station and 7his km house is 0 km.
is
14. 15.
facing North, and Ravi was facing South. Choice
In thethe Given
evening that –
asPiyushthe sun and isRavi
in the were Westfacing the 19. Let N, E, S
(2) 11. The path where
traversed H byis the
Sameer house
Choice is as
(2) and
follows: O (2) Office. ? As sha
fromfrom km figure,  +CD CD+DE +22= compass wh
2
BEBE
2
1figure,
(AB) = (BE) =(AE) = BC
BC
2
and DE == 11  W5 E 20 shadow eachfalls towards East.shadowAs shadow ofhisShyam
where towardsH Post
is theOffice
house and O Choice is the Office. Sother. HPiyush’s fell
E to left compass
West respecth
7.B km 12 6+ 2C
from figure,
= 5 2km BE D =9BC 2 CD EDE = 1  2  2 = 10.5 Jatin Jatin
is walking
is walking
the
towards
East.
the East.
(4)
H16.is4 At km
falling
5:15 to his
thewalked left towards
minute and
hand hisis left
at theiswhich
east.
houris? He is 3
division
We knowkm that 3 km o 4
4 km km A (that
person is towards West as
West, it was morning).
actually As the pointe
=AE F2 =B F(AE)
=B4=2km 4 km facing
thus south
we canand sayShaanclock is as facing
shown North.
below.
(AB)AE 2
6= (BE) AE 3 km and 6 11. The path  traversed S 4 bykmSameer is as follows: Choice (4) North.
So, Piyush wasSE facing North,Choice
Choice (4)
and Ravi showing
WestEast
S
4 km 6 = F B = 4 km2 9
2 = 1  2  210
where H is the house and O is the Office. O
(2) Solutions for
from figure, BEo= BC 22 CD  22DE =5
? AB ∴
? AB== 5 5( 5) + 44( 4 ) = 4141
=
AB 41km.4 km.
km. Choice 11.
Choice The path 1 traversed
Nkm 30by N Sameer
4 km is as H fol- 14. In the
was evening
facing as
South. the sun is in the West the
20. As per the
km A 30 15. Given shadow thatfalls– Piyush
towardsand 12 Ravi
East. As were
shadow facing
of each
Shyam
AE = F B (2) = 4 km Choice (2) lows: 7 km 7 km  other. Piyush's
is falling to hisshadowleft and fell his leftto his left ?
Choice
is east. (that
He (2)is
is
(2) B
where
W H is
3 km
the house and O is the Office. towards
facing West
south as
and it Shaan
was morning).
is facing North. So, Piyush was
? AB = 5 6 4 2 km 41 km.
Let A and 2 B be2 the initial and the final points
Choice N Office isW E
7.
respectively in the journey.9
? towards South-west E with respect to
16.
facing AtNorth, NE
5:15 and 9
the minute
Ravi was hand 3
facing is at SW the
South. hour
Choice Choice di-
(2) West S
6 9 house. 720km 4 km (4) (2)
Choice
7.(2) H vision
4 km 3 –thus we can say clock isfacing
as shown ? As shan
AB 7. 2
B =6 19  4C = 210 km
2
D (approx)29 E 6
Choice (2) W Jatin is walking E S
towards 4 km
the East. 4 km
Choice (4) 15. Given that Piyush and Ravi were each compass he
6 12. North below. 6 isfell
7. We
6 6 know that o 92 6 10 11. The path traversed by Sameer 44 km
S is as follows: O 16. At other.
5:15 Piyush's
the minuteshadow hand at theto his hourleftdivision
(that is 3 R
8. (AB)2 = (BE) 6 2  (AE) A and 9
O 4 10 E 1 km SE km thus
O
towards
we can Westsayas it was
clock NWis asmorning).
shown So, below.Piyush was
o = BC  CD  DE6= 1  2  2 = 5 where SH N
is the house
4 km and O is the Office. S facing North, and Ravi was facing South. Choice Solutions for qu
6 from figure, BE ?(2)At 9 O’clock theSE hour hand will at 9 and points
km6 o 30 m
A
9 10B 4 7 3kmkm 4O 1kmkm H towards North-east. Choice (2)
? As shanta
Let A and B be the initial
4 and the final points 20. compass
As per the
hegi
AE = L FAetB =A4 and km B be the initial and the final W 1?km Office E is towards South-west with respect 16. to At 5:15 the minute 12 hand is at the hour division 3
respectively in the journey. B
50points N house.
West NE 3 kmSW East Choice17. (4) thus we
5 respectively
B 42 be in km.
the
B 80journey. N can say clock is as shown N
2 2
below.
? AB
Let
Let A and
A=mBand
AB = be92the 70
 4 initial
= 10 mthe41
km initial
and (approx)
m and
the final points
Choice
the final
Choice (2) points 3 km
7kmkm OfficeNorth
? South-west is with
4 km
towards South-west ི with NE respect 9ཱུ SEto 3 SW Solutions for is
So, he quo
2 2 ? 12. Sis towards
Office 4 respect to
respectively
(2) ABin=the9journey. in+ 4the=journey.10 km (approx) Sol/920 ི ཱུ
respectively
8. O Choice (2) Whouse. E house. SEO Choice (4) Choice (4)12 20. Aswith respe
per the giv
7. =AB92=6 49 22
= 104km 2A (approx)
9 C 1 km E
W S W E 6
figure.
30=m10 km (approx) Choice
12. (2)
AB 60 m Choice (2) N 4 km W E
North NW R
6 8. 6 S 4 12.
km North
NE 9 NW 3 SW 21. A walks fro
8. 6 9 O 10 3 km SE S-W
8. E O route).
o H B O ∴ Office km is
South
E
S SEre- East ཻ ? At∴ 9 O’clock
At 9 O’clock theླྀ hour thehand
hour ཻwill at 9 andླྀ
hand will atpoints 9
50
30 Am m 4 1West NEtowards South-west with SW S
70 m 80 m towards S North-east. 6 S Choice (2)
30 m B spect to3 km house. Choice (4) and points towards North-east. R
(i) At 12 O' clock (ii) At 6 O' clock
Let A and B be the initial and the final points ? Office
Sol/920
17.  N NW N Choice (2)
50respectively
m in the journey.A 80 m West
12. NEis towards South-west with SW respect Eastto
? At 9 O’clock the hour hand will at 9 and points
70 m C house. N Choice (4)
W ི ཱུ So, he is or
60 m
AB 50 = 9m 2
 42 = 10 km (approx) West NE NW towards SWNorth-east. East ི Choice
ཱུ (2) with respect
70 m Choice 80 m (2)
12. North Sol/920 17. figure.
A H CO B South W 17. N E W N E
8. SE
60 m NE S W ི Sol/920 ཱུ 21. So, he is from
A walks orig
NW
30 m S-W ི ཱུ with respect t
A ཻ ླྀ route).
H 60Bm C South N ཻW W ླྀ E W E
figure.
S S
50 m 80 m
West NE SW East NW 21. A walks from
70 m (i) At 12 O' clock (ii) At 6 O' clock
S-W route).
B South ཻ ླྀ
H ཻ ླྀ
A S S
The path traversed 60 m by Ramesh is shown
C N W
(i) At 12 O' clock (ii) At 6 O' clock 9
NW
in above figure by solid line.  ence, at 6:00 p.m., the hour hand points
H
H H be officeB and house respec- South
Let O and toward the North-East. Choice (3)
tively.  , E, S, W, NE, SE, NW and SW are
N
the pointers of the compass which point 18. Given, the clock is so placed that the min-
We know that, (OH)2 = (BH)2 + (OB)2
North, East, South, West, North-east, ute hand points towards North-West when
and
South-east, North-west and South-west re- it is 3:00 p.m.
BH = 60 –30 = 30, OB = 80 + (70 –50)
spectively. But now the pointer NE of that is
= 100

08_Section.indb 124 10/30/2009 11:53:56 AM


Hence, at 6:00 p.m., the hour hand points toward
the North-East. Choice (3)
Solutions for Intelligence and Critical Reasoning  n  125
18. Given, the clock is so placed that the minute hand
Hence, at 6:00 p.m., the hour hand points toward
points towards North-West when it is 3:00 p.m.
the North-East. Choice (3)
W are the pointers N-W N So, A is facing North originally which  Choice (2)
orth, East, South, 18. that
Given, is the clock 12is so placed that the minute hand is North-West as per the given figure.
North-west and Hence,
points towardsat 6:00 p.m., North-Westthe 3hour when hand it ispoints toward
3:00 p.m. Solutions for question 3:
 Choice (1)
ow the pointer
W are the pointers NE the North-East. W N-W xN E Choice (3) The given information can be summarised in a
west
orth, thus East, all the
South, that is 12 22. A is standing in the South-East direction.
18. Given, the clock is so placed table as follows.
,as North-west
shown above. and 3 that the minute hand
points towards North-West S when it is 3:00 p.m.  Choice (3)
owoints the North-west.
pointer NE x hour Ehand points toward Air Rail Road
Wwest are the
orth-west. thuspointers all the Hence,
Similarly, at 6:00
Similarly, atWN-Wp.m.,
9:00
at 9:00Nthe
a.m., a.m., the the hour hourhand hand points
rth, East,
Choice South,
(4) that
the is South-West,
North-East.
toward 12 as shown below. Choice (3) Solutions for questions 23 to 25:
as shown above. points toward South-West, as shown be- According to the given information, we get the Kolkata ↔ Kolkata ↔ Kolkata ↔
North-west
oints North-west. and NS 3
18. Given, low.the clock is so placed that the minute hand Delhi Delhi Delhi
wged theand
orth-west. pointerits needle
NE Similarly, at 12
W 9:00 xa.m., the E hour hand pointsfollowing diagram.
wedest Norththus for East
all the points towards North-West when it is 3:00 p.m.
Choice (4) toward South-West, as shown below. Chennai ↔ Chennai ↔ Chennai ↔
sare the pointers
shown above. N-W N N
that is W 12 x E A 40 m Delhi Delhi Delhi
th,
intsEast,
aged North-west.
and South,
its needle 12 S E
North-west
rth-west.
wed North forand East Similarly, at 9:00 9 a.m., 3 the hour hand points
W Nagpur ↔ Nagpur ↔ Raipur →
the Choice
pointer (4) NE toward South-West, W S-W xas shown E below. 30 m Mumbai Mumbai Nagpur
est thus all the W NSx E
ged and
s shown above. its needle 12 50 m Nagpur ↔ Raipur ↔ Hyderabad ↔
9 Choice (2)
ed North
nts WNorth-west. forNEast S-W S C B
Hence, at 6:00 p.m., the hour hand points toward Raipur Kolkata Chennai
th-west. 19.Similarly, at S
LettheN,North-East.
E,W
9:00 anda.m.,
xS W are the the hourpointers
E below.
hand points
Choice (3) of the 20 m 40 m
Choice (4) toward
compass South-West,
which points as shown North, East, South and F Kolkata ↔ Chennai → Delhi →
E 18.  Given, the 9
clock N
is so placed that the Choice
Choice
minute (2)
hand
(2) D
N West respectively. 15 m Hyderabad Hyderabad Nagpur
ed W
and its 12
, which is needle
actually 19.
19. AsLetLet
points towards
theN,N,pointer
E, S
E,
S-W
S and
North-West when it is 3:00 p.m.
which
and WNWwasarearethe pointers
showing
the North
pointers of the
isof now
the
edand North
SW are for the
East N-W S 20 m Hyderabad ↔ Nagpur → Mumbai →
Choice (4)pointers showing
that is East,
compass which these
12points points oriented
North, East, as South
follow. and H
pointENorth, East, South, compass which points North, East, South
Choice (2)
G
Mumbai Delhi Hyderabad
uth-east, W
West respectively. x North3 E
in the North-west
WestN the and and West respectively. 23. B is standing towards North with respect to F
t,But which is actually
now the pointer NE As the pointer which Wx was E showing North is now Chennai ↔ Hyderabad →
shadow
owing W Choice
of Shyam
west thus (4)all the 19. Let AsN,theE,East,
showing S W9
pointer and
these Wpoints
which are was theshowing
oriented pointers of the
North
as follow.  Choice (2)
is east. ? He above.
is compass which points S-W North, East, South and Raipur Mumbai
point as shown is now showing East,
North these points orient-
E S
gast) North.
in the
points West the
North-west. West respectively. W
S 24. Since, we do not know the distance be-
which
shadow
wards isof
Choice
North-west. actually
(2)
Shyam As ed
West
Similarly,
the
S at 9:00 a.m.,
aspointer
follow. which was the showing N East
hour hand
Choice
North points
is(2)now ↔ – indicates two way connection.
N toward South-West, as shown below. tween D and I, we cannot say the position
t is east. Choice Choice
? (4)
He is (4) showin thes ints oriented as follow. → – indicates one way connection.
were facing each
g North. and its needle 19. Let N, E, S and W
12 North
N are the pointers of the of I with respect to D.
as
oindamagedtheleft
his West theis
(that compass West which S points N East ∴ It cannot be determined. Choice (4) 3. There is only one such possibility.
t itEshowed Choice North(2)for East WE North, East, South and
g).hadow So, Piyushof Shyam was West respectively. Mumbai – Hyderabad – Chennai –
ngsiswere
which South.
east. facing
is ?Choice
Heeach
actually is W South
As the pointer which was showing North is now 25. C is towards North-west with respect
x E
? As East, shantan is points
going orientedtowardsasWest as per
Raipur
oNorth. hisChoiceleft (that (4) is showing these 9 follow. to H. Choice (1)
W West She is actually
compass E N EastNorth. ∴ She needs to travel through two cities
g). So, Choice
Piyush (2)was S-W Northgoing towards
menhethe hourWest divisionthe 3 SSouth Choice (3) to reach Raipur. Choice (2)
ng South. Choice W
wn below.
were
hadow facing
of Shyam each ? As shantan is going towards Choice West (2) as per
S left (that isN Solutions for questions 20 to 21:
shis
. So,
he
North.
east.
hour
? He is
Piyush was 3
division
compass he is actually
19. Let N, E, S and E W
going towards North.
are the pointers Routes and Networks
of
Choice
the
(3)
Solutions for questions 4 to 7:
20. As West per the
compass S givenwhichdata, points we get the following figure.
gwn South.
below.
Choice Choice
E (2) SouthNorth, NEast, EastSouth and 4. The root one has to follow to reach P from
West
? As shantan respectively. is going T towards West as per
Solutions for questions 20 to 21:
s West, which is actually
ere facing each compass
As the pointer which was showing North is now
heEast,
is actuallypoints goingoriented
towards North. Practice Exercise R is as follows.
SW Choice (4) 20. As ∴showing
perAsthe shantan
giventhese is going
data, we get towards West as
as follow.
the following
e
hishour left division
(that is3 North Choicefigure.
(3) 1 hr 1 hr 1 hr
wn sun
So, below.
is
Piyushin thewas West the per compass he isWactually E
T going towards Solutions for question 1 and 2: R S Q P
ast. As shadow of Shyam Solutions for questions South20 to 21: (4:00 (6:00 (7:00 (8:00
South. Choice North.R Choice (3)
SWleft is east. ? He is
his ? As shantan is going towards P West as per 1. Let us represent the given information in a.m.) a.m.) a.m) a.m.)
is facing North. 20. As per the given data, we get the following figure.
compass
Solutions forhequestions
West is
S actually20 going
to 21: towards North. the form of a diagram. ∴ one takes 4 hours to reach P from R if
e hour
will and Choice
at 9division 3 (2)
points T CN East Choice (3)
nd below. Choice (2)
Ravi were facing each 20. As per the given data, we R get the P follow- he starts at 4 am.
SW Stops Delhi Indore
w fell to his left (that isSolutions for questions 20 to 21: But if starts at 7 O’clock.
morning).
N So, Piyush was ing figure. EQ
C
will at 9 and points 20. As per the given data, we get the following figure. He can reach P as follows
as facing South.(2) Choice South
Choice So, he is originally towards South-West direction
Stops

? AsR shantan is Tgoing towards
withcompass
respecthetoisP, but itgoingis South P West as per
as North.
per the given 1 hr 1 hr 1 hr
actually Q towards Ahmedabad R S Q P
SW
d is Nat the hour division 3 figure. Choice
Choice (3) (3)
llasatshown
9 andbelow. E
points So, he is originally towards South-West direction
C (7:00 (8:00 (9:00 (10:00 a.m.)
Choice ཱུ (2) 21. Solutions
A walks from P to Q along
for questions the diagonal (shortest a.m.) a.m.) a.m)
with respect to P, but20 toStops
it is 21:
South as per the given
ླྀ route).
N E
figure.
20. As per R the given Q
data, we getP the following
P Choice
figure.(3) ∴ One takes 3 hours to travel to P from R.
S T  Choice (2)
So, he is originally towards
21. A walks from P to Q along the diagonal (shortest South-West direction
)l Atat 396 O'and
ཱུ SW clock
points with respect to90° P, but it is South C as per the given Calicut Bangalore
ླྀ(2) route).
Choice figure. PMidwayChoice (3)
Stops
5. The shortest route that a person can fol-
S E Midway Sol/921 T
 he possible routes from Delhi to Indore low from R to Q is R – S – Q
R P to QQalong the diagonal
)NAt 6 O' clock 21. A walks from 90° P (shortest are as follows: R ______ S ______ Q
ླྀ route).
So, he is originally towards South-West direction (i) Delhi − Ahmedabad − Indore
P Midway (10:00 (11–12 (1:00
withSrespect
o, he istooriginally it istowards
South asSouth-West
C
hand ཱུ will at 9 and points P, butMidway per theSol/921
given
S Choice (2) Q (ii) Delhi − Ahmedabad − Calicut − In- a.m.) a.m) p.m.)
figure. direction with respect to P, but itChoice Stops is South (3)
At 6 O' clockE dore
N 21. A walks as perfrom the90°Pgiven
to Q figure.
Q
along the diagonal Choice (3)
(shortest
1 hr wait
Midway (iii) Delhi − Calicut − Indore.
ི ླྀ ཱུ route).
21.
So, he is originally
Awith walks from to P,Pbut
towards South-West direction
Midway
to itQisP Q as
along Sol/921
theperdiagonal
As he reached Q at 1 pm he should start from
respect South the given (iv) Delhi – Ahmedabad – Bangalore – S at 12 noon but no train reaches from R at
figure. Choice (3)
WS E (shortest route). Calicut – Indore.
At 6 O' clock 21. A walks from
12 noon thus he must reach S at 11 a.m.
90° P to Q along the diagonal (shortest ∴ Four different routes are possible.
ཻ ླྀ ro Q Midway only.
S
Midway P Sol/921  Choice (4) ∴ He would have started at 10:00 a.m.
(ii) At 6 O' clock 90° 2. AQPB and APQB can be eliminated be- from R. Choice (1)
Midway
Midway Sol/921 cause P and Q cannot send the data to B. 1 hr 1 hr 1 hr 1 hr
Q The paths AQB and APB do not violate 6. R S Q P T
any conditions and hence can be possible (4:00 (5:00-6:00 (7:00 (8:00 (9:00
Q a.m.) a.m.) a.m.) a.m.) a.m.)
routes to send data from A to B.

08_Section.indb 125 10/30/2009 11:53:58 AM


126  n  Solution Manual

 He has to wait only at station S for 1


∴ 15. The longest possible route from G to B is 21. From Q to P, he can reach by boat.
hour, because he reaches there at 5:00 as follows: From P to R also he can travel by boat.
a.m. and the first train departures at 6 (ii) G − H − F − D − E − A − B Or So, a person should travel by boat to reach
O’clock. Choice (4) (iii) G – F – D – E – H – A – B R from Q without changing the mode of
∴ The longest route from G to B contains transport. Choice (1)
7. The route is_____
as follows:
_____ _____ _____ five cities in between. Choice (2)
P T Q R S 22. If a person wants to visit all the places and
(10 (11:00 (12:30 2:00 (5:00 Solutions for questions 16 and 17: again return to P, then he can go in the
a.m.) a.m.) p.m.) p.m.) p.m.) order of P→ Q → T → R → S and then
16. Q gets filled in first 21/2 hours.
Next train Next train Next train S → R → P (OR)
P needs 800/200 hours that is, 4 hours to
at 11:30 11:00 p.m. at 4:00 P → R → S and then S → R → T → Q
get filled completely.
p.m. p.m. →P
If no water flows from Q to R, R fills in
∴ He will reach ‘S’ at 5 O’clock. He must visit R twice. Choice (2)\
1200/300 that is, 4 hours.
 Choice (3) 23. One has to travel between any of the two
But after 21/2 hours, water flows from Q
Solutions for questions 8 to 10: to R. cities with a restriction that if there is
∴ R gets filled completely before P gets more than one possible route, he has to
8. By road only, one can go to D from G by go by the least number of cities enroute.
following the route given below: filled.
∴ P never gets filled as water flow stops It is better to take the pair of cities given
G − H − A − C − E − D. Choice (3) in each of the choices.
in the pipes when R gets filled complete-
9. The possible route from G to F where one Choice (1) Q and S: A person can go from
ly. Choice (3)
can visit maximum cities by opting for Q to T to R to S (or) Q to P to R to S or
exactly two modes of transport that is rail 17. As Q gets completely filled in first 21/2 vice-versa. In both the routes, there are
and road is as follows: hours, in that time 21/2 (300) = 750 litres two cities enroute. We can calculate the
G−H−A−C−B−D–E–F of water flows from S to P. number of options in the entire route by
 Choice (4) ∴ P will have 750 litres in it by the time multiplying the options available in each
Q is completely filled. Choice (4) segment of the route.
10. C − A − H − G – F is the route which one
has to take so as to visit maximum cities, Solutions for questions 18 and 19: For the route Q-T-R-S, the number of op-
by travelling through only one mode of The following diagram is obtained from the tions = 1 x 2 x 2 = 4
transport that is road. Choice (2) given information: For the route Q-P-R-S, the number of op-
tions = 2 x 1 x 2 = 4
Solutions for questions 11 to 13: Choice (2) P and R: The route between P
1100 500
Let us represent the given information through E C H and R has only one mode of travel, that
diagram, which is given as follows: 300 200
is boat.
A A Choice (3) P and T: A person can go by
700 500 400
400
PQT in either way or by PRT in either
H way. If it is by PQT, then the options are
1000 300 600 B
D D G F
boat-air or rail-air that is two ways. If it
is by PRT, then the options are boat-rail
18. If the person travel along the route HACF-
C B or boat-bus in either way. Hence, they are
F GE, then he can travel through maximum,
only two options.
number of cities with in the given limit of
Choice (4) Q and R: If a person travels
E G 1700 km. Thus, he can travel through a
between Q and R, then he can go by QTR
maximum of five cities.
or by QPR. If he goes by QTR, then the
11. As it is clear that to travel from H to any  Choice (3) options are air-rail or air-bus, that gives
city, one has to travel via B. Choice (2)
19. The minimum distance from D to H is two options or if he goes by QPR, then
12. The longest route from C to D is as fol- the options are rail-boat or boat-boat, that
2300 km which is along the route DECH.
lows. will again give us two options.
 Choice (2)
C → H → B → A → E → G → F → D. So, it is very clear that Q and S have
One can visit all the cities by travelling Solutions for questions 20 to 23: maximum number of travel options
from C to D. Thus it is 6 cities.
air
available between them. Choice (1)
 Choice (4) T
20. boat, rail Q
13. From the diagram it is cleared that one Solutions for questions 24 and 25:
can go to D only after crossing F thus one P According to given information, the diagram
rail, bus will be as follows:
cannot reach D before F.  Choice (1)
boat F
Solutions for questions 14 and 15: G
bus, boat R
S A
14. From the observation, it is very clear that
one cannot go to I from any city. Hence I P and T are connected through Q.
cannot be visited. Among the other cities T and S are connected through R B
E
one can go to C only through B. hence no Q and R are connected through T or P. Q
C
one can visit C before reaching B. Hence, and S are connected through R and P or
the cities, one cannot visit are I and C T. So none of the pairs in the choices are
while going from F to B. Choice (3) directly connected. Choice (4) D
H

08_Section.indb 126 10/30/2009 11:53:59 AM


Solutions for Intelligence and Critical Reasoning  n  127

24. From the diagram, it is clear that to (2) 3  5 A 10 £ 7 ∆ 5 ⇒ 3 − 5 < 10 ÷ 7 Conclusion ΙΙ, F + G ⇒ F > G does not follow
travel from A to H, one has to visit + 5; −2 < 10/7 + 5 (true) Choice (2)  Choice (4)
city E. Choice (2) 20. Given statements are as follows,
Solutions for questions 12 to 14:
25. The longest route from G to F is: T ∼ Z ⇒ T = Z; T × Y ⇒ T < Y ⇒ Y > T;
12. After conversion the expression will be, W−Y=W≥Y
G − A − B − E − H − C – F Choice (2)
7 × 4 + 12 − 10 + 15 ÷ 3 = 28 + 12 − 10 + ∴W≥Y>T=Z
5 = 35 Choice (4) Conclusion Ι, Z × W ⇒ Z < W is true.
Symbols and 13. After conversion the expression will be, Conclusion ΙΙ, W + T ⇒ W > T is also
Notations 27 ÷ 3 × 27 + 80 − 23 = 9 × 27 + 80 − 23 true. Choice (3)
= 243 + 80 − 23 = 300 Choice (3) Solutions for questions 21 to 23:
Practice Exercise 14. After conversion the expression will be, 21. 12 ∆ 7 ▼ 9 = (7 × 9) + 12 = 75 = t
Solutions for questions 1 to 3: 343 ÷ 35 × 10 + 16 × 7 − 25 = 343/35 × 10 75
∴ 8 ⊕ t ∗ 7 = 8  75 ∗ 7 = =5
+ 112 − 25 = 185 Choice (1) 8+ 7
1. X is the only element which is immedi-
ately followed by a digit and immediately Solutions for questions 15 to 17:  Choice (2)
preceded by a symbol. Choice (1) Given that the symbol 22. 40  5 # 9 = 40 × (9 − 5) = 40 × 4 = 160 = P
‘=’ means ≠; '−' means <; '+' means ≥; '÷' means ∴ 35  6  P = 35  6  160 =
2. After reversing the first ten elements the
>; '×' means = (35 × 6) − 160
8th element from the left end is 2 and the
5th element to its left would be 9. 15. Statement P + Q means P ≥ Q; Q ÷ R = 210 − 160 = 50 Choice (3)
 Choice (2) means Q > R; R − S means R < S; P ≥ Q
> R and S > R 39 ×15
3. Here in the sequence the first elements are 23. 39  45 ◊ 15 = = 13 = x
Conclusion Ι : R × S means R = S is defi- 45
the consecutive letters, the second elements
nitely false. ∴ 17  11  x = 17  11  13 = (17 × 11) −
are the consecutive digits and the third
Conclusion ΙΙ : P ÷ R means P > R is true 13 = 187 − 13 = 174 Choice (4)
elements are the consecutive symbols.
from both the given statements.
Hence the next term must be C 3 φ Solutions for questions 24 and 25:
 Choice (2)
 Choice (3) The coding pattern is based on the binary system
16. Statement : L × M means L = M; L + N means in which only two digits 1 and 0 are used.
Solutions for questions 4 and 5: L ≥ N; N − T means N < T Given that
4. T+2, ς+2, J+1, ↑+2, Q+2, L+1, 1+2, C+2, 5+1, ≠+2, implies 0 is written as M; 1is written as N; 2 is written
$+2, 2+1, B+2, £+2, 9 Choice (2) L = M ≥ N and T > N as NM
Conclusion Ι: M − T means M < T is not because
5. Eleventh element from the right is ≠. definite from both the given statements. 2 _______
2
The seventh letter to the right of ≠ is £. Conclusion ΙΙ: L × M means L = N is also
 Choice (3) 1 - 0 → 10 ⇒ NM
_______
not definitely true from the given state-
Similarly, 3 is written as NN
Solutions for questions 6 to 8: ments. Choice (4)
because
6. After conversion the expression will be. 17. Statements : A + B means A ≥ B; A × C 2 _____
3
15 ÷ 3 ÷ 13 + 2 × 3 ÷ (7 + 6) + (4 + 3 − 5) means A = C
____ → 11 NN
1−1
÷ 13 = 1 Choice (1) A = D means A ≠ D implies A < D or A
> D. 24. 8 ⇒ 1000 that is N M M M Choice (2)
7. After conversion the expression will be.
Implies C = A ≥ B and A < D or A > D 25. The given equation is
8 − 4 + 7 × 15 ÷ 3 × 2 + 4 − 2 = 8 − 4 + 7
C > B is not definitely true and hence it (M N M N M − N N M N N) that is
× 15/3 × 2 + 4 − 2 = 82 − 6 = 76.
does not follow. (1 1 0 1 0 − 1 1 0 1 1) = 30 – 27 = 3 = 11
 Choice (3)
D ÷ C means D > C is also not true because = NN Choice (2)
8. After conversion the expression will be, D > C or D < C Choice (4)
1 + 2 − 3 + 4 ÷ 2 × 5 = 1 + 2 − 3 + 4/2 × 5
Solutions for questions 18 to 20:
= 1 + 2 − 3 + 10 = 10. Choice (4) Linear and Circular
18. The given statements are as follows. Arrangements
Solutions for questions 9 to 11:
Given that Q − S ⇒ Q ≥ S ⇒ S ≤ Q; Q ∼ T ⇒ Q = T;
R+T⇒R>T⇒T<R
 → −; ∆ → +; £ → ÷; $ → ×; → Practice Exercise
=; A → <; ς → >; Combining (i), (ii) and (iii) we get.
∵S≤Q=T<R Solutions for questions 1 to 3:
Let us convert the symbols as per the given di-
∴ Conclusion Ι is R + S that is R > S is It is given John sits exactly in between Anil and
rections.
true David, John sits exactly in between Charan and
9. (1) 5 ∆ 10 $ 2 8  5 ⇒ 5 + 10 × 2 = Conclusion ΙΙ is S ÷ R that is, S ≤ R does Kamal.
8 − 5; 25 = 3 (false) not follow Choice (1) As John sits exactly between the two pairs of
(2) 5 £ 10  2 A 8 $ 5 ⇒ 5 ÷ 10 − 2 < 8 boys, he has to sit exactly in the middle of the
× 5; –1.5 < 40 (true) Choice (2) 19. The given statements are as follows
G + E ⇒ G > E ⇒ E < G; D + E ⇒ D > row. The possible arrangements are
10. (1) 7 A 3 ∆ 5 ς 6 $ 1 ⇒ 7 < 3 + 5 > 6 × 1; E; F − D ⇒ F ≥ D Kamal/Charan Anil/David John Anil/David Ka-
7 < 8 > 6 (true) Choice (1) ∴F≥D>E<G mal/Charan
Conclusion Ι, F × G ⇒ F < G does not Anil/David Kamal/Charan John Kamal/Cha-
11. (1) 3 ς 5 ς 10 ∆ 7  5 ⇒ 3 > 5 > 10 +
follow ran Anil/David
7 − 5; 3 > 5 > 12 (false)

08_Section.indb 127 10/30/2009 11:53:59 AM


128  n  Solution Manual

1. John sits exactly in the middle of the row. 7. J and K can sits on either ends of the row. Solutions for questions 14 to 16:
 Choice (1)  Choice (3) It is given that, eight persons – P, Q, R, S, T, U,
V and W sit in eight chairs alligned as shown
2. If Kamal and Charan occupy the ends 8. In case (2) L sits in between M and N.
below.
then in four different ways the boys can  Choice (1)
___ ___ ___ ___
be seated and if Anil and David occupy
Solutions for question 9: ___ ___ ___ ___
the extremes then they can be seated four
From (ii) V can not be at left extreme and, from
more ways, thus totally the five boys 9. As per the given instruction, we get the
(ii) and (iii) V and W are in same row but not
can be seated in eight different ways. following seating arrangement:
adjacent each other and from (iv), W is not at
 Choice (3) A E C any extreme end so, the possibilities are
3. If Anil sits to the immediate left of John D B F _ _ _ S ___S
and there is a boy who sits to the immedi- Hence, only statement (2) is true. _ _ W R V (or ) R W _ V
ate right of Kamal then their seating order  Choice (2) From (i) and (iii) T and Q are in same row they
is as follows can be along with S, but not adjacent to each
Kamal Anil John David Charan Solutions for questions 10 to 12:
other, P is along with V, but opposite Q thus the
Anil is to the immediate right of Kamal. It is given that six persons A, B, C, D, E and F
possibilities are
 Choice (2) stay in a six floors building where each one of
T U Q S Q U T S
them stays in a different floor.
P W R V case (1) (or) R W P V case (2)
Solutions for questions 4 and 5: From (Ι), we have Charan
It is given that three subjects are scheduled be- Raman 14. U sits opposite W. Choice (2)
tween Mathematics and Physics and Botany is Kiran 15. If R is not sitting in between V and W
scheduled exactly between English and Hindi. then the order is as that is case (2), but in
From (ΙΙ), we have Rajan
Periods Subjects Pavan any case
Case – I Case – II Shravan R sits opposite Q. Choice (1)
I Maths/Physics Chemistry Combining (Ι) and (ΙΙ), we got either Kiran or 16. If Q does not sit adjacent to S that is case
II Eng/Hindi Maths/Physics Shravan stays in the 1st floor. (2).
From (ΙΙΙ), we have Raman T sits in between S and U and P sits op-
III Botany Eng/Hindi
Kiran posite T. Choice (2)
IV Eng/Hindi Botany
V Maths/Physics Hindi/Eng Pavan Solutions for questions 17 and 18:
VI Chemistry Maths/Physics
And Rajan is below Raman and above Kiran It is given that six trains leave from a station at
Combining (Ι), (ΙΙ) and (ΙΙΙ), we get Shravan six different timings.
4. As explained above Botany is scheduled stays in the first floor, and four people stay From (i),two trains leave in between T2 and T3,
either in the IIIrd period or in the IVth pe- above Pavan. Kiran and Rajan are below Raman but from (iii) T3 leaves before T2, we have
riod. Choice (4) who is below Charan. T3
5. If Hindi is scheduled immediately after The order is as follows: –
mathematics then the arrangement is as 6 - Charan; 5 - Raman; 4 - Rajan / Kiran; 3 –
follows. -Rajan / Kiran; 2 -Pavan; 1 -Shravan T2
Periods Subjects From (ii), we have T1 > T4 > T5,
10. Pavan stays in the 2nd floor. Choice (1)
From (iii), we have T6 > T3 > T2,
I Maths 11. Either Rajan or Kiran stays in the 4th Note: T1 > T4 means T1 leaves before T4.
II Hindi floor. Choice (4)
17. If T2 is the last train to leave the station,
III Botany
12. If one person stays in between Pawan and then the order of the trains to leave the
IV English
Kiran, then the order is as follows. station is as follows.
V Physics 1 - T1 / T6
6 - Charan; 5 - Raman; 4 - Kiran; 3 - Rajan;
VI Chemistry 2 - Pavan; 1 - Shravan 2 - T1 / T6
English will be scheduled for IVth period. Rajan stays in the 3rd floor. Choice (3) 3 - T3
 Choice (3) 4 - T4
Solutions for question 13: 5 - T5
Solutions for questions 6 to 8: 6 - T2
13. From the given statements, we get the fol-
It is given that seven persons – I, J, K, L, M, N ∴ T4 is the fourth train to leave the sta-
lowing seating arrangement:
and O sit in a row. tion. Choice (1)
Where A - Anand,
From (i), we have, M ___ ___ O
B - Bhuvan, 18. If T3 leaves immediately after T6 then the
From (ii), we have, I ___ ___ ___ ___ K C A D B
C - Chander, order is as follows.
From (iii), we have, J ___ ___ L
D - Dinesh, 1 - T1 1 - T6
From (iv), we have, L ___ O
M - Martin, 2 - T6 2 - T3
Combining (i), (ii), (iii) and (iv) we have
3 - T3 3 - T1
Case (1): I J N M L K O (or) N - Newman N P M O
4 - T4 4 - T4
Case (2): J I M L N O K O - Owen
5 - T5 5 - T2
6. O sits at one end that is case (1) M sits P - Paul 6 - T2 6 - T5
exactly at the middle of the row. Hence, Chander sits opposite to Newman. ∴ It is possible only in two ways.
 Choice (2)  Choice (1)  Choice (2)

08_Section.indb 128 10/30/2009 11:54:00 AM


Solutions for Intelligence and Critical Reasoning  n  129

Solutions for questions 19 and 20: 23. If every person interchanges seat with the L .
V
.
19. person sitting opposite to him, then also V
the arrangement remains the same, but
E rotates by 180°. Hence, F is sitting to the 1 G 1G L
D B right of C. Choice (4)
24. It is given that the following persons in- Ar Ar
Rh Rh
terchange places: 3 
V
3
C A A and D, F and E, B and C. G
F
C
It is given that A is two places away to B 1 Ar
the left of E, who is adjacent to B and F
D. D is to the right of E and A is setting
L
between B and F. These conditions give A Rh
E
us the following arrangement. 3
D
∴ A is sitting opposite D. Choice (1)
Now we get the following arrangement. 28. Atul can be opposite the first ranker in
20.
(1) is false, as A is to the left of A. case ΙΙΙ only. In that case, according to
C (2) Only E is sitting between B and D, (iii) Neha will be between Atul and Vick-
F G hence false. ey. Hence, Vickey is the fourth ranker.
(3) As C and D sit opposite to each oth-  Choice (1)
B A
er hence this statement is true.
29. From (i), Neha is adjacent to the second
H (4) E and F sit opposite to each other,
E ranker. From (iii), Neha is adjacent to the
D hence this is false. Choice (3)
fourth ranker and Atul.
 s per the given conditions, if H is sitting Solutions for questions 25 to 27:
A ⇒ Atul is the second ranker.
between B and D, then H must be sitting It is given that Rohan is sitting adjacent to Vickey can be adjacent to the second
to the left of D, as already E is sitting to Pankaj who is sitting opposite Arun. ranker in case ΙΙ and also case ΙΙΙ.
the right of D. hence, we get the follow-
Hence, either Geeta or Archie can be the
ing seating arrangement. Rohan Pankaj first ranker. Choice (1)
Hence, only the statement “F” is sitting
Arun Pankaj Rohan
Choice (1) between
already E isBsitting
and C, must
to the beoftrue.
right D. hence, we get 30. The second ranker Atul can be opposite
 following seating arrangement. Choice (3)
the (or)
Arun the third ranker Rahul in case ΙΙ only.
at is case (2).
sits opposite T.
Solutions foronly
Hence, question 21: “F” is sitting between
the statement (I) (II)
B and C, must be true. Choice (3) At
Choice (2) 21. More than one arrangements are possible Now, from the above arrangements and the re- 2
18: Solutions
here. for question 21: maining information, we get the following two V N
m a station at six ∴ It than
21. More cannot
onebe determined.are possible here.
arrangements arrangements.
 It cannot be determined. 1G L
nT 2 and T 3, but Rahul (or) Rahul Veda 4
ve Shanu Rohan
Ar Ry
Ashok Vivek Vivek Ashok
Rh
Arun Pankaj 3
Raj Raj
Solutions for questions 22 to 24:
 Choice (4) Pinky Dimple
For circular seating arrangement, the data is as given Suman In this situation Lata is the fourth ranker.
re T Solutions
below for questions 22 B to 24: (I)  Choice (2)
4.
For circular seating C arrangement,
1 E the data is as Dimple
e station, then the (or)
given below 6 2
the station is as Suman Pankaj
B5
4
3 Distribution
C F 1 D Shanu Rohan
E
6 A
2
(i) Six friends – A, B, C, D, E and F. Arun Practice Exercise
3 Veda
(ii) F is to the left of5 A 4
F D Pinky Solutions for questions 1 to 3:
(iii) B is sitting between C and E and is opposite to A.
(iv) E is sitting to the right
A of D. (II) From (i), Chen and Govinda like Alpenlibe and
the station. Using statements (ii), (iii) and (iv), we get Mangobite in any order. From (iii)
Choice (1) (i)
Here,Six friends
B is between– A, B, C,
C and D, EEisand
E and F. right of D,
to the 25. Either Pinky or Shanu sits opposite Dimple.
Chen likes Mangobite.
then the order is as
thisFstatement
(ii) is to themeans
left ofthat
A E is at seat 2 and D Is sitting  Choice (4)
at seat 3. Govinda likes Alphenlibee.
(iii) B is sitting between C and E and is oppo- 26. Suman is sitting opposite Veda. Choice
22.site
E is to
sitting From (ii) and (iii)
A. to the left of B. Choice (2)
(1)
6
Parag likes Melody.
3 (iv)
23.EIf is sitting
every to the
person right of D.
interchanges seat with the person
27. In case (i) Dimple is sitting opposite Shanu. Curie likes Diary milk.
1
Using
sitting statements (ii), then
opposite to him, (iii) and (iv),arrangement
also the we get
remains the same, butErotates  Choice (2) Rajesh like Popins.
and Ebyis180°. Hence,
rightF
4
Here, B is between C and to the
2 is sitting to the right of C. Choice (4)
5 of D, this statement means that E is at seat 2 and Solutions for questions 28 to 30: 1. Parag likes Melody. Choice (3)
D 24. It is given
Is sitting that3.the following persons interchange
at seat 2. Curie lies Dairy milk. Choice (2)
Choice (2) places: From (i), (ii) and (iv) we get the following ar-
20: 22. AEandis sitting
D, F andtoE,the leftC.of B. Choice (2) rangements.
B and 3. Govinda likes Alphenlibe. Choice (2)
Now we get the following arrangement.
C
B
F

08_Section.indb 129 10/30/2009 11:54:02 AM


A
130  n  Solution Manual

Solutions for questions 4 to 6: Solutions for question 12 to 15: Solutions for questions 19 to 21:
From (ii) and (iii) The given information is as follows. The given information is as follows.
Binita is in class ΙΙΙ Student Subject Asian Countries
From (i) and above discussion
A Phy. Sc. / Eng Chi Jap Mal Eng Aus Ger Pol
Namrata is in class IV
From (iv) and above discussion C Bio / Maths A √ X x x x
Savita is in class V D Eng / Bio B X x x x
Namita is in class ΙΙ E/F Hist C X
Sunita is in class Ι F/B Geog D X x
4. Namita is in class ΙΙ. Choice (1) E Maths / Ph. Sc E x x x X X

5. Sunita is in class Ι. Choice (1) From the above table it is clear that E cannot get F x x x X x x
the highest marks in History. G
6. Namrata is in class IV. Choice (4) ∴ F got the highest marks in history.
From the above table,
Solutions for questions 7 and 8: ∴ B got the highest marks in Geography.
F is from Germany and G is from England.
From (i), (ii) (iii) and (iv) B is not wearing yel- 12. F got the highest marks in History. ⇒ C is from Australia and E is from Poland.
low, green, black and pink coloured dress.  Choice (3) ∴ B and D are from Japan and Malaysia in any
∴ He is wearing red coloured dress. order.
13. B got the highest marks in Geography.
D is wearing yellow coloured dress. Hence, the final arrangement is as follows.
 Choice (1)
A is wearing green coloured dress.
Chi Jap Mal Eng Aus Ger Pol
B is the fifth ranks. 14. C could be the person who got the highest
marks in Biology. Choice (1) A √ x x X x x x
C and D are the 4th and the 3rd ranker in any
order. B x X x x x
15. If C got the highest marks in Maths, then
C x x x X √ x x
7. B is wearing red coloured dress. E got the highest marks in Physical Sci-
ence. Choice (2) D x X x x x
 Choice (1)
E x x x X x x √
8. C’s rank can be the 3rd or the 4th. Solution for questions 16 to 18: F x x x X x √ x
 Choice (4) The given information is as follows: G x x x √ x x x
Solutions for questions 9 to 11: House Car 19. F is from Germany. Choice (3)
The given information is as follows. Red Green
20. Either B or D is from Malaysia.
Name Profession Place Ekta White
 Choice (4)
A Architect Chetan Not Yellow Pink
21. G is from England. Choice (1)
B Not Doctor Not Bangalore Pink Not Yellow
C Engineer Chennai Fahrook Not pink Red Solutions for questions 22 to 24:
D Not Professor Not Bangalore Dhiren Yellow From (i) and (ii) A is husband of P or Q
E Mumbai The colour of the dress of R and S are violet
From the above table it is clear that Amol and
and blue
Not Architect Kolkata Bhashkar has red and pink coloured house in
Not Lawyer
From (iii) A is the husband of P
any order.
B and Q are wearing green dress
The doctor from Bangalore cannot be any one White coloured car must belong to the person From (iv) C is the husband of S, who were wear-
among A, B, C, D and E. Hence, it is F. who has a pink coloured house. ing violet dress
Since D is not a professor, he is not from Delhi. So, Ekta has a blue coloured car. E is husband of F who were wearing blue dress.
⇒ B is the professor from Delhi. Fahrook has yellow coloured house. The couples and their dresses are as follow
Since, the person from Kolkata is not an Archi-
Dhiren and Chetan has blue and green coloured Couples Place
tect, A is from Hyderabad and D is from Kol-
houses in any order.
kata. Male Female
⇒ D is the Painter and E is the Lawyer. The final distribution table is as follows.
A P Red
The final distribution is as follows. Name House Car B Q Green
Name Profession Place Amol / Bhaskar Red Green C S Violet
A Architect Hyderabad Ekta White Blue D T Orange
B Professor Delhi Chetan Blue / Green Pink E R Blue
C Engineer Chennai Amol / Bhaskar Pink White 22. B and Q wears green dress. Choice (3)
D Painter Kolkata Farookh Yellow Red
23. A is the husband of P. Choice (3)
E Lawyer Mumbai Dhiren Blue / Green yellow
F Doctor Bangalore 24. S is the wife of C. Choice (3)
16. Ekta has blue car. Choice (3)
9. F is the doctor. Choice (4) 17. Farookh has yellow house. Choice (2) Solutions for questions 25 to 27:
From (i) and (ii), T and U are from the same
10. The painter is from Kolkata. Choice (3) 18. Bhaskar’s house is either pink or red in country and P and R from different country.
11. E is the lawyer. Choice (2) colour. Choice (4) ∴ Q and S are from the same country.

08_Section.indb 130 10/30/2009 11:54:02 AM


Solutions for Intelligence and Critical Reasoning  n  131

Similarly, for the colours, ∴A>G>F>D 7. Nikil got the first rank in Chemistry.
The persons wearing blue and black coloured From (i) and (iii), B is the tallest and C is the  Choice (2)
costumes are from the same country. second tallest. 8. Akil got the second rank in Chemistry.
The persons wearing green and yellow coloured From (iv), and above results, we have the following  Choice (1)
costumes are from different countries. arrangement.
So the persons wearing red and orange coloured B>C>A>G>F>E>D 9. Sunny got the third rank in Chemistry.
costumes are from the same country.  Choice (3)
1. A is the third tallest. Choice (2)
From (iii) and (iv),
Solutions for questions 10 to 12:
Q is wearing green coloured dress. As U is wear- 2. Three persons are taller than G.
From (i), P < R < S.
ing blue coloured dress, thus T and R are from  Choice (3)
From (ii), S is the third eldest.
India and wearing yellow and black dresses in
3. E is the second shortest. Choice (3) From (iii), Q is elder than fourth youngest [that
any order. P is wearing orange coloured dress. S
is wearing red coloured dress. is, fourth eldest]
Solutions for questions 4 to 6:
The final distribution will be as follows : ∴Q is the second eldest.
Let the number of chocolates with each of them
From (iv), T is the second youngest and U is the
Person Colour be denoted by the first letter of his name.
youngest.
T Yellow / black From (ii), C < S < R.
∴V must be the eldest.
India U Blue
From (iii), S > N
Also, R is the fourth eldest and P is the fifth el-
From (iv), R − C = K − R
R Yellow / black dest.
As R − C is atleast two, and no number has a dif-
Q Green ∴We have, U < T < P < R < S < Q < V.
ference of three with more than one of the given
Pak S Red numbers, 10. V is the oldest. Choice (4)
P Orange R−C=2 11. P is the third youngest. Choice (2)
25. S is wearing red costume. Choice (2) ∴K−R=2
From (ii) and (iii) 12. R is the fourth oldest. Choice (1)
26. P, Q and S are the players from Pakistan. N<C<S<R
 Choice (3) Solutions for questions 13 to 15:
As K − R = 2, From (i) and (iv), Anand got a better rank than
27. Either R or S is wearing yellow costume. A must be greater than R. atleast three persons that is, Anand’s rank can
 Choice (4) ∴ The final arrangement will be be 1 or 2 or 3.
Solutions for questions 28 to 30: N < C < S < R < A < K From (iii) Charan’s rank can be 6 or 5 or 4.
From (i), either D1 or D2 are the director for 3 4 5 6 7 8 If Anand’s rank is 3 then Charan’s rank must be
movies M1 and M3 and either D3 or D4 are the 4. Ranjit has 6 chocolates. Choice (2) 4, in this case, condition (v) is violated.
directors of the movies M2 and M4. ∴Anand’s rank is either 1 or 2.
5. Sumit has 5 chocolates. Choice (1)
From (iv) M3 is directed by D2 which impress Here we have three possibilities:
that M1 is directed by D1. 6. The difference is = 8 – 3 = 5. Choice (4)
From (ii) and (iii) (a) 1 2 3 4 5
Kavita and sonu worked together in M2, and for Solutions for questions 7 to 9: 6
D4 Kailash is the male singer, which means that The sum of the ranks can be atleast three and at Gopal Anand Deepti Hriday/ Charan
M2 is not directed by D4. Hence, M2 is directed most seven. Brihesh/
by D3 and M4 by D4. From (ii), In Physics either Bunny or Sunny got Brijesh Harish
Now the singers for M3 is Kavita and Kailash the fourth rank. (b) 1 2 3 4 5
and for M1 are Shreya and Sonu. As it is given that Bunny got the fourth rank in 6
chemistry [from (i)], ∴ Sunny got the fourth Anand Gopal Hriday/ Deepti Brijesh/
Movie Director Male Female
rank in physics. Charan
M1 D1 Sonu Shreya
Also, as the sum of the ranks of Bunny is not the Brijesh Hriday
M2 D3 Sonu Kavita
highest (that is, 7). (c) 1 2 3 4 5
M3 D2 Kailash Kavita
Bunny’s rank in physics can be either 1 or 2. 6
M4 D4 Kailash Shreya
But from (ii), Akil got a better rank than Bunny
28. D4 is the director of M4. Choice (3) in physics. Anand Deepti Hriday Gopal Brijesh
∴ In physics Akil got the first rank and Bunny Charan
29. M2 is directed by D3. Choice (2)
got the second rank. 13. It is possible in case (b) and (c).
30. Kailash and Kavitha work together in M3. ∴ In Physics, Nikil got the third rank. But only in case (b), we have one per-
 Choice (2) As only Sunny can get the sum of the values as son between Deepti and Charan that is,
7, his rank in Chemistry must be 3 otherwise Brijesh. Choice (4)
Bunny’s sum of the rank will be the highest.
Comparison 14. We have more than one possibility.
∴ In Chemistry, Nikil must get the first rank and
 Choice (4)
Practice Exercise Akil must get the second rank.
The ranks are represented in the following ta- 15. It is possibility (c), in which Brigesh got
Solutions for questions 1 to 3: ble: the fifth rank. Choice (1)
From(i), B > C, Solutions for questions 16 to 18:
Rank Physics Chemistry
From (ii), A > G > F Let the marks scored by each person be denoted
1 Akil Nikil
From (iii), C is the second tallest. by the first letter of his name.
From (iv), E is not the third tallest. 2 Bunny Akil From (ii), E > C > D. From (iv), G > H From
From (v), F > D 3 Nikil Sunny (v) D > G
From (ii) and (v) 4 Sunny Bunny Combining the above, we get.

08_Section.indb 131 10/30/2009 11:54:03 AM


132  n  Solution Manual

∴E>C>D>G>H low coloured curtain can be the costliest or, the 1 2 3 4


from (iii), 1  2  3  4  5  6  7  8 door decorated by Pearl can be the door of the
least cost. Q R P S
F
As H did not get the lowest score, either A or B If the Pearl decorated door is of least cost then OR
got the lowest score. the door decorated by gold must be the third 1 2 3 4
From (i) and above results, A got the first rank costliest. But it violates condition (vi).
The door decorated by diamond which has yellow Q S R
1 P 2 3 4
and B got the eight rank. ∴ The diamond
coloured decorated
curtain can door
be theis the costliest.
costliest or, the door Q R S P
OR
∴ The final arrangement will be as follows. As platinum is the second costliest. It can be
decorated by Pearl can be the door of the least cost.
Relation between R and S is not clear.
If the Pearl decorated door is of
only the door to the right of the door decorated least cost then the 1 In terms
2 of heights;
3 4S is shorter than Q.
A>E>C>D>F>G>H>B
door decorated by gold must be the third costliest.
by silver. Q is shorter than R.
16. Dayanand got the fourth highest score. But it violates condition (vi). Q R S P
S is not the tallest of all.
∴ Door? Thedecorated
diamond by gold must
decorated door is bethe
thecostliest.
third
 Choice (4) As platinum is the second costliest. It can be only
Relation
the
between
Position ofRPand S isknown.
is not not clear.
costliest. So, the order of heights is as follows.
17. Goutam got the third lowest score. door to the right of the door decorated by silver. In terms of heights; SRis> shorter than Q.
Now combining with (iii) we get the final ar- Q>S
 Choice (3)
? Door decorated by gold must be the third costliest. Q is shorter
But than
the R.entire order cannot be decided, as the
rangement as follows.
Now combining with (iii) we get the final arrangement
S is not the tallestofofS all.
position is not mentioned.
18. Anurag got the highest score. Choice (2) as follows.
6 Position of
26.P Data
is notisknown.
not sufficient to decide the correct order
3
Solutions for questions 19 to 21: Topaz So, the order of
ofrich people.
heights P or
is as S can be the poorest.
follows.
Gold Choice (4)
Each row or column must be filled with different Re d Violate / Blue R > Q > S
27.
But the entire order cannot be decided, as the as follows.
P is the heaviest of all as the order is
numbers among 75, 80, 85 and 90. From (iv), 4
Either C or D got 90 marks in Chemistry. position of S is not mentioned. 1 2 3 4
Pearl P S R Q
From (ii), and above, D got 90 marks in Chem- Indigo 26. Data is not sufficient to decide the cor- Choice (1)
istry. 1 2 rect28.
order of rich
P and S arepeople.
heavierP than
or S can be Q,
R and thebut only S is
Now from (iii), C got 75 marks in Physics. Diamond Platinum poorest.
definitely shorter than Q, Choice
whereas(4)
P may or may
Hence B can not get 75 marks in Physics and Yellow Blue / Violate not be shorter than R and Q. Choice (4)
hence from (iii), B got 75 marksin Biology. Also 5 27. P is the heaviest of all as the order is as
29. If P is the 3rd richest in order, then the order is as
follows.
D can get 75 marks only in Maths. Silver follows.
Now, As C got 85 marks in Biology, D can not Green 1 2 3 4
1 2 3 4
get 85 marks in Biology. Hence, D get 80 marks In each figure, the words over between and below the Q R P S
In each figure,
lines the words
represented over between and be-
as follows. P S R 2ndQrichest must be R.
in biology and 85 marks in Physics. Now, A can Then the Choice (3)
low the lines
rank represented
from cos tliest toasleast
follows.
cos t  Choice (1)
get 90 marks in Biology. Now A can not get 90 30. The heaviest is P, but the tallest is not clear, so
marks in Physics. So, B gets 90 marks in Phys- Decorated
rank from cos tliestbyto least cos t 28. P and Sthe
aretallest canthan
heavier be either
R andRQ,orbut
P. only
But the richest is
ics, which implies A get 80 marks in Physics. Colour of curtain
Decorated by Q, so the respective order can be PRQ or PPQ.
S is definitely shorter than Q, whereas PChoice (1)
Also as B, C and D got 75 marks each, A can get 22. FourthColour ofdoor
costliest curtain
is decorated by Pearl. may or may not be shorter than R and Q.
75 marks in Chemistry only. So A gets 85 marks Choice (3)  Exercise Choice
15 (4)
in Maths. In the same way, B gets 85 marks in 22. Fourth costliest door is decorated by (Selections)
23. The colour of the curtain of the door decorated by If P is the 3rd richest in order, then the
29.
Chemistry and 80 marks in Maths and C gets 90 Pearl. Choice (3) Solutions for questions 1 to 3:
Topaz can be blue or violet. Choice (4)
marks in Maths and 80 marks in Chemistry. order is as follows.
23. 24.
TheThecolour of the curtain of the door
colour of the curtain on the costliest door is
Let each person be donated by the first letter of his name.
The final table will be as follows: decorated
yellow. by Topaz can be blue or violet.Choice (3) 1 1. If2 D is selected,
3 4then some of the possibilities are:
 Choice (4) Q DEF
R P S
Maths Physics Chemistry Biology 25. Diamond decorated door and green curtain door CDE
A 85 80 75 90 24. Thearecolour
adjacent to each
of the other.
curtain Choice (1)
on the costliest ABD
Then the 2nd richest must be R.
door is yellow.
Solutions for questions 26 to 30:Choice (3) ? Any one can be selected. Choice (4)
B 80 90 85 75  Choice (3)
C 90 75 80 85 25. The given conditions
Diamond decoratedare thatand
door P, Q, R and
green S are four
cur- 2. If A is selected, then C must not be selected.
30. The heaviest
[from (i)]
is P, but the tallest is not
men. In terms of weight, we know that P is heavier
D 75 85 90 80 tain door are adjacent to each other.
than S and R is heavier than Q but is lighter than S, clear, so
? Nowthe tallest can be either
using condition (ii) theRpossibilities
or P. are:

which means that R is lighter than Choice
S. (1) But theABErichest is Q, so the respective or-
19. B got the maximum marks in Physics. So, the order of weight is as follows. der canABF
be PRQ or PPQ. Choice (1)
 Choice (2) Orderfor
Solutions of weight
questions 26 to 30: ADE
ADF
20. C got 80 marks in Chemistry. Choice (2) The given conditions1 are that 2 P,3Q, R4and S are
AEF
four men. In terms P S
of weight, R know
we Q that P
Selections
? There are 5 possibilities. Choice (1)
21. Only one person (that is C) got more In terms of poverty, it is given that Q is richer than S
is heavier than S and R is heavier than Q but
marks than A in Maths. Choice (3) and Q is the richest of all, whereas R is richer than P. 3. As B is selected, then C must not be selected.
is lighter than S, which means that R is lighter If D is selected, then E or F must be selected.
So, the order of poverty is as follows. Practice Exercise
Solutions for questions 22 to 25: than S.
Order of Poverty ? If A is selected, then D must not be selected.
So, the order of weight
1 is2 as follows.
3 4 ? To select a team of 3. E or F must be selected.
Solutions for questions 1 to 3:
Gold Choice (4)
Order of weight Q R P S
Let each person be donated by the first letter of
OR
Pearl
1 21 32 43 4 his name.Solutions for questions 4 to 6:
Q S R P 4. If Rajesh is selected then Uday must be selected
1. If D is selected, then some of the possi-
ORP S R Q [from (iii)].
bilities are:
Silver In terms of poverty, it is given that Q is richer DEF     CDE     ABD Sol/932
than S and Q is the richest of all, whereas R is ∴ Any one can be selected. Choice (4)
From (ii) and (v) we get, richer than P. 2. If A is selected, then C must not be se-
Now from (i), (iv) and (vi), So, the order of poverty is as follows. lected. [from (i)]
The door decorated by diamond which has yel- Order of Poverty

08_Section.indb 132 10/30/2009 11:54:04 AM


Solutions for Intelligence and Critical Reasoning  n  133

∴ Now using condition (ii) the possibili- 8. If P and Q are selected, then S or T may ∴ From (ii), only one of A and B is with
ties are: also be selected from the boys. So the them.
ABE boys may be PQS or PQT. If PQS form ∴ The possibilities are : M, N, K/L, A/B
ABF the team, then the team should be PQ- and L/K, C, B/A.
ADE SUW. If PQT are the boys, the teams may ∴ There are four possibilities.Choice (1)
ADF be PQTUV and PQTUW. Choice (2)
AEF Solutions for questions 15 to 19:
9. If P and S are the boys selected, then R Let us write the data in short.
∴ There are 5 possibilities. Choice (1)
cannot be selected. So the boys can be 2 boys = A and B
3. As B is selected, then C must not be se- P and S along with Q or T. that is PSQ or 2 girls = C and D
lected. PST or PSQT (three ways of selection). 2 men = P and Q
If D is selected, then E or F must be se- Then the girls U or W can be selected as 2 women = R and S
lected. V cannot go with S. So the teams may be 2 cars = Black and White
∴ If A is selected, then D must not be PSQUW, PSTUW, PSQT with U or W. Conditions are as follows:
selected. Hence, altogether there are four ways of
∴ To select a team of 3. E or F must be selection. Choice (4) (i) 2 children and 2 adults must be there in one
selected. Choice (4) car.
10. If four boys can be selected as PQRS,
(ii) A ≠ Q, that is A and Q cannot be in one
Solutions for questions 4 to 6: PQST, PQRT, PRST, QRST but P and R
car.
4. If Rajesh is selected then Uday must be cannot be together, then PQST or QRST
(iii) P ≠ D, that is P and D cannot be in one
selected is possible. In either case V cannot be
car.
[from (iii)]. selected as S and V do not go together.
(iv) R ≠ S, that is R and S cannot be in one car.
For other two persons, we have the fol-  Choice (2)
(v) 2 Males + 2 Females = Each car, that is 2
lowing possibilities: 11. Among the choices PQRVW cannot go males and 2 females must be there in each
(Srinu, Madhav) as a team, as P and R cannot be together, car.
(Srinu, Naveen) whereas the other given teams can be se- It is clear that 1 boy, 1 girl, 1 man and 1 woman
(Srinu, Praveen) lected. Choice (4) must be there in each car. As A and Q cannot be
(Srinu, Tarun)
Solutions for questions 12 to 14: in one car, it can be concluded that A and P must
(Madhav, Naveen) Choice (1)
Let each person be denoted by the first letter of be in one car. As P and D cannot be together, P
5. If Tarun is not selected then Srinu may or his / her name. and C must be in one car. So B, Q and D must be
may not be selected. in the other car, whereas R and S can be in any of
∴ The team can be 12. K and C are in the same team. the cars. So, the distribution is as follows.
(Madhav, Naveen, Uday, Rajesh) ∴ L must be in the other team. [from
Cars
(Srinu, Praveen, Uday, Rajesh) (iii)]
(Madhav, Srinu, Uday, Rajesh) If A and B are in the same team as L, then Black Or White White Or Black
(Naveen, Srinu, Uday, Rajesh)Choice (2) it violates (iv). A P C R or A P C S B Q D S or B Q D R
6. If Srinu and Uday are selected, then [from (ii)] Now let us solve the questions.
Rajesh must be selected. ∴Only one of A and B is in the same team
as L. 15. If P and A go in one car, P is a man and A
∴ The other person can be any of Mad- is a boy.
hav, Naveen, Praveen and Tarun. ∴ Using other conditions the possibili-
ties are : So, they should have two females in
 Choice (4)
(a) K, C, A, M and L, B, N their team – one woman and one girl.
Solutions for questions 7 to 11: They can be R or S and C or D. So any
(b) K, C, B, M and L, A, N
A team of 5 members with at least three boys of RD, RC, SC, SD can be with them.
(c) K, C, A and L, M, N, B
and at least one girl from five boys P,Q,R,S,T  Choice (2)
(d) K, C, B and L, M, N, A
and three girls U,V,W has to be selected. It is 16. If Q and R go together, the car must have a
(e) K, C, B, N and L, M, A Choice (4)
further known that if P is in the team, then R is boy and a girl in it. D cannot go with P. So,
not, that is 13. M and B are not in the same team.
she must go with Q and R. The boy may
From (iii), either K or L is with B.
P  ⇒  RX  So, R  ⇒  PX be A or B, but A and Q do not go together.
Now, if A is with B, then C and N must be So Q, R, D and B must go in one car.
If S is in the team, then V is not in the with M, which violates (iv).  Choice (3)
team, ∴ A is with M.
Now, atleast one of C and N is with B. 17. The car which is carrying D cannot carry
S ⇒ VX So, V ⇒ SX ∴ The teams are: B, K/L, C, N and M, P, hence P must be in the other car. Four
L/K, A members must be in each car of which
that is
B, K/L, C and M, L/K, A, N two children and two adults (two males
7. If Q, R and V form a team of three mem- and two females) must be there. So, the
B, K/L, N and M, L/K, A, C
bers, then one or two more boys or one selection may be as follows.
∴ There are six possibilities. Choice (4)
more girl has to be selected. Now P and
Car 1 Car 2
S will not be in the team. So choice 2 and 14. From (iii), M and N are in the same team.
2Children + 2Adult 2Children + 2Adult
4 is eliminated. Now choice 1 also is not One of K and L is with them.
possible as both are girls. Hence, choice ∴ C cannot be with them as it violates C A D B
(3). Choice (3) P Q
(iv).

08_Section.indb 133 10/30/2009 11:54:04 AM


134  n  Solution Manual

 and S can be in any car. But D cannot


R anything about their purchases as all the
go with P. So A, C, P cannot go with D. choices 1, 2, 3 be true always.Choice (4) Deductions
 Choice (1)
26. If Ajay buys P and Bony buys Q, then Practice Exercise
18. P, S, A and C is a correct combination. Chetan buys neither T nor U. So, Chetan
Choice (1) is not true, as it states that can buy the pair of R and S only. Choice Solutions for questions 1 to 5:
PSAC is not the correct combination but (1) 1. As the middle term is not distributed
Q, R, B, D is correct. Hence choice (2) is
27. Let us validate each choice. even once in the given premises, no
also not true for the same reason as that of conclusion can be drawn.
Choice 1: if Ajay buys R, Bony cannot
choice (1).  Choice (4)
buy P and S but he can buy Q or T or U.
P, Q, A, C cannot go in one car. But P, S,
So it is not necessary to buy Q.
A, C can go in one car. Choice (4) 2. As one of the premises is universal and
Choice 2: If Chetan buys T or U, that
affirmative and the other one is particular
19. P, S, A, C and Q, R, B, D are correct, as means Bony cannot buy Q. So this is also
and affirmative the conclusion must be
P is a man. A is a boy. false.
particular.
S is a woman. C is a girl. Choice 3: Ajay bought R, then it is not
Hence, ‘some insects are dogs and ‘some
Q, R, B, D is also correct, but all others necessary for Bony to buy T as explained
dogs are insects’ are the only two conclu-
are incorrect combinations. Choice (2) in Choice 1.
sions that can be drawn from the given
Choice (4). If Ajay buys R and Bony buys
Solutions for questions 20 to 22: premises. Moreover a negative conclu-
Q, then Chetan has to buy only P and S, as
Let each person be denoted by the first letter of sion cannot be drawn from affirmative
he cannot buy T and U. Chetan has to buy
his name. premises. Choice (4)
P and S. Choice (4)
From (iii) at least one of A, E, F and D must not 3. As both the premises are universal affir-
be selected. Solutions for questions 28 to 30: mative, the conclusions that can be drawn
From (iv), at least one of C and H must not be are universal affirmative and particular
28. If Kareem is selected, Puja cannot be se-
selected. affirmative.
lected, then Tinku and Shalu also cannot
∴ At least one of B and G must be selected. Hence, ‘some stones are papers’ and ‘all
be selected. That leaves only Ronak and
From (v) and above results, exactly one of B and stones are papers’ are the only two con-
Umesh. Hence Ronak and Umesh must
G must be selected. clusions that can be drawn from the given
be selected. Choice (3)
∴ Exactly one of A, E, F and D, and exactly one premises. Choice (2)
of C and H must not be selected. 29. It is given that P may go to A or B and
∴ From (iii) and above results A or E is not se- must be seen, whereas S goes to only A, 4. As both the premises are particular, con-
lected. and T goes to only B. It is clear from the clusion cannot be drawn. Choice (4)
∴ F and D are selected. data that O has gone back and neither 5. As one of the premises is negative, the
If B is selected then we have the following pos- (M and R), nor (U and Q) go together. If conclusion must be negative.
sibilities: A/E, F, D, B, H Q, R, S go to one director and meet him, Hence, ‘no bike is bus’, ‘no bus is bike’,
If G is selected, then we have the following pos- the other director sees any two of M, N, ‘some bikes are not buses’ and ‘some
sibilities. A/E, F, D, G, C/H U and T along with P, who must be seen. buses one not bikes’ are the conclusions
M does not go with R and U, so PNT or those can be drawn from the given prem-
20. Fattu and David must always be selected.
PUT are seen. Only B can see them as S ises. Choice (3)
 Choice (3)
goes to A. So B − PNT or PUT is correct.
21. If Chandu is selected, then Girish, Fattu  Choice (1) Solutions for questions 6 to 15:
and David must be selected. Choice (4) 6. Here both the statements are nega-
30. Choice (1): If B is selected, then A and
22. If Bharath is selected, then Harish, David tive. So no conclusion can be drawn.
E cannot be selected. The other 3 must
and Fattu must be selected. Choice (1)  Choice (4)
be from CDFG but D and F have to be
together so the team will be B, D, F and 7. The middle term B is not distributed,
Solutions for questions 23 to 27:
any one of C or G. So D must be selected hence no conclusion can be drawn.
Three persons Ajay, Bony and Chetan buy two
in the team, hence this choice is definitely  Choice (4)
each out of 6 items P, Q, R, S, T and U. If Ajay
buys R, Bony does not buy P or S or both. If TRUE. 8. Here there are three terms. The middle
Bony buys Q, Chetan does not buy U or T or Choice (2): If A is selected B cannot be term ‘flowers’ is distributed.
both. chosen. The other 3 members can be any As one of the premises is particular and
3 from C, D, E, F and G. This choice is negative, the conclusion must be particu-
23. If Ajay buys R and T, then Bony cannot not definitely TRUE. lar and negative.
buy P or S or both. So Bony buys Q and Choice (3): If B is selected, then A and E ∴ The conclusion is ‘some beautifuls are
U. Choice (2) are ruled out. The other 3 can be chosen not ugly’. Choice (4)
24. If Bony buys Q and S, then Chetan cannot from C, D, F and G. Of this D and F must
be chosen with the last member being ei- 9. There are three terms but the middle term
buy T and U. So, Ajay must buy T and U,
ther C or G. This choice is not definitely “colourful” is not distributed, hence no
as each one has to buy two each. Choice
TRUE. conclusion can be drawn. Choice (4)
(2)
Choice (4): C need not be chosen on 10. There are four terms, hence no conclusion
25. If Chetan bought P and S, Ajay and Bony
every team. If we select DFEG, this is a can be drawn. Choice (4)
have to choose any two each of Q, R, T,
valid team without C. 11. Both the statements are particular, hence
U. Ajay may have any pair of QR, QT,
 Choice (1) no conclusion can be drawn. Choice (4)
QU, RT, RU, or TU. So, we cannot say

08_Section.indb 134 10/30/2009 11:54:04 AM


Solutions for Intelligence and Critical Reasoning  n  135

12. Here there are three terms and the middle 24. (A) Here both the premises are particu- Hence, ΙΙ follows. If it is assumed that the
term A as well as particular is distributed. lar, hence is not a valid set. customers do not abide by the notice, then
As both the premises are affirmative the (B) Both the premises are negative, the notice would not have been issued.
possible conclusions are hence is not a valid set. Choice (4) Hence, Ι follow. Choice (4)
(i) Some Bs are Cs. (ii) Some Cs are Bs.
25. (A) It does not violate any condition. 4. The word ‘pity’ indicates that the state-
 Choice (2)
(B) The middle term ‘fly’is not distributed. ment expects the government to take
13. Both the statements are negative, hence  Choice (2) charge of the situation. Hence, Ι follows.
no conclusion can be drawn. Choice (4) The statement has no reference to the
Solutions for questions 26 to 30: feelings of the government towards the
14. Any light is bright ⇒ All lights are bight.
26. As one of the premises is particular and affected areas. Hence, ΙΙ does not follow.
Here there are three terms. The middle
the other one is negative, the conclusion  Choice (1)
term ‘light’ is distributed. One of the
premises is particular, hence the conclu- must be particular negative. 5. The word ‘Despite’ indicates that the
sion must be particular. As the term ‘buses’ is not distributed in statement assumes that an icon in Bol-
∴ The possible conclusions are the premise, it should not be distributed lywood will be affected by the trapping
(i) Many brights are bulbs. in the conclusion. Hence, ‘some buses are of tinsel town. Hence, Ι follows. From
(ii) Many bulbs are bright. Choice (4) not scooters’ is the only conclusion that the statement ΙΙ, it appears that becoming
can be drawn from the given premises. an icon occurs before getting affected.
15. There are three terms. The middle term
Hence, ΙΙ does not follow. Choice (1)
“intelligent” is distributed. One of the 27. As one of the premises is particular, the
premises is particular and other one is conclusion must be a particular. As both Solutions for questions 6 to 10:
negative, hence the conclusion must be the premises are affirmative the conclu-
6. The author of the statement is giving in-
particular and negative. sion cannot be negative. Hence, ‘some
formation about the grades of the listen-
∴ The conclusion is ‘some managers are televisions are computers’ ‘some comput-
ers. This implies that the listeners are
not devils’. Choice (1) ers are televisions’ are the two conclu- interested to know such information.
sions that can be drawn from the given Hence, (Ι) is an assumption. (ΙΙ) is a
Solutions for questions 16 to 20:
two premises. generalisation which cannot be assumed.
16. From the first and the second statements  Choice (1)
28. As both the premises are negative, con-
it can be concluded that ‘some camels are
clusion cannot be drawn. 7. The nuclear-haves [countries possessing
not cat’s. Using this conclusion with the
third statement it can be concluded that 29. In order to draw a conclusion we need nuclear weapons] realised certain facts
‘some dogs are not cats’. Choice (2) to have exactly three different terms but after the Indian explosions. These explo-
there are four terms in these two prem- sions also gave Indians a sense of pride.
17. From the second and the third statements,
ises. Hence, conclusion cannot be drawn. This does not imply that the nuclear-haves
the conclusion is ‘some teachers are not
would never have realised the facts other-
doctors’. Using this conclusion with the 30. As one of the premises is particular con- wise. The meaning of the phrase “sense
first statement it, can be concluded that clusion must be particular. As both the of pride” used in the statement is different
‘some teachers are not lawyers’.Choice (3) premises are affirmative the conclusion from the meaning of “proud people” used
18. As two of the given premises are negative cannot be negative. Hence, ‘some pencils in assumption (Ι). Neither (Ι) nor (ΙΙ) is an
no conclusion can be drawn. Choice (4) are erasers’ and ‘some erasers are pencils’ assumption. Choice (3)
19. From the first and the third statements it are the two conclusions that can be drawn
8. In this statement, the author concludes
can be concluded that, ‘some pens are not from the given premises. that the glittering thing is gold. His reason
pencils’. Using this conclusion with the for this conclusion is that “It glitters” This
second statement it can be concluded that points to the fact that the author of this
‘some pens are not scales’. Choice (4) Statements and statement opines that only gold glitters
20. Since two of the given premises are par- Assumptions and no other thing glitters. This opinion
of the author can also be said to be his
ticular, conclusion cannot be derived.
assumption. Hence Only (Ι) is an assump-
 Choice (4) Practice Exercise tion. (ΙΙ) is not an assumption as it contra-
Solutions for questions 21 to 25: Solutions for questions 1 to 5: dicts the author’s view. Choice (1)
21. (A) It does not violate any condition. 1. Unless the University assumes that 9. From the statement, we cannot say wheth-
(B) It does not violate any condition. some people want to do Hotel Man- er Bushkashi is a sport or not. Hence, (ΙΙ)
 Choice (1) agement course, the advertisement is not an assumption. “Everybody loves
watching Bushkashi.” From this we can-
22. (A) It does not violate any rule. would not have been given. The ad-
not say that Bushkashi is the only thing
(B) In conclusion the term hard is dis- vertisement is given with an intention
that is liked by everybody, hence Ι does
tributed, which is not distributed in of attracting the people to the Univer-
not follow. Choice (3)
the statement. sity also. Hence, both Ι and ΙΙ follow.
Hence only (A) is valid. Choice (2)  Choice (4) 10. The fact that this suggestion is given
means that the book could be useful for
23. (A) The term ‘intelligent’ which is not 2. The statement assumes existence of the the exam. Hence, assumption Ι follows.
distributed in the premises is dis- subject and the other hypothetical things The person may be preparing for the
tributed in the conclusion. Hence referred to. Hence, both Ι and ΙΙ follow. exam or he may be the publisher of the
not a valid set.  Choice (4) book who makes this statement to garner
(B) It does not violate any condition. publicity. Hence, (ΙΙ) does not follow.
3. When a notice is given, it is assumed that
 Choice (3)  Choice (1)
there is a necessity to give such notice.

08_Section.indb 135 10/30/2009 11:54:05 AM


136  n  Solution Manual

Solutions for questions 11 to 15: 14. Choice (1):  From the phrase “caught be- 20. There is no information in the statement to
tween the devil and the deep sea”, it can indicate that radio is not available to any
11. Choice (1):  It is stated that Veerappan is
be concluded that country X does not one other than the common man. Hence,
able to strike at will. It is also stated that
want to antagonise either party. Ι does not follow. Statement ΙΙ indicates
the STF personnel were led to believe
Hence, (1) is a conclusion. that there was no programme broadcast on
that Veerappan had magical powers. It is
Choice (2):  The statement does not give radio, which airs songs of listeners' choice
because that STF people are unable to as-
any clue, regarding the whereabouts of until 1980, Hence, ΙΙ does not follow.
sess as to when he would attack. Hence,
the don. The statement has no reference to various
(1) is a conclusion.
Hence, (2) is out of context. programmes that are broadcast on radio.
Choice (2):  As the STF personnel be-
Choice (3):  As FBI has sought the help of Hence, ΙΙΙ does not follow. Choice (4)
lieved that Veerappan has magical pow-
ers, it is the assumption of the STF country X, it is the assumption of FBI that Solutions for questions 21 to 25:
personnel, that one can become invisible country X can find out the whereabouts
of the don. 21. It is asserted that the quality of education
with magical powers. is a reflection of the quality of teachers,
Hence, (2) is an assumption. Hence, (3) is an assumption.
Choice (4):  As it is stated that the don is but not vice versa.
Choice (3):  It is stated that officers are As the passage advises an improvement
afraid of posting in STF and that STF per- notorious, it can be inferred that he is in-
volved in illegal activities. Hence, (4) is in the training of teachers, it is assumed
sonnel were lead to believe that Veerap- that there is scope for improvement.
pan is invisible. From all these, (3) can an inference. Choice (3)
Hence (2) is an assumption.
be inferred. 15. Choice (1):  The passage has no reference The passage states that the quality of edu-
Choice (4):  The statement has no refer- to the British. cation in the determining factor. Hence,
ence to war tactics. Hence, (1) is out of context. (3) can be concluded.
Hence, (4) is out of context. Choice (2) Choice (2):  As the Indians are using the From the tone of the statement, it can be
12. Choice (1):  Though it is stated that the Mc Mahon line, it can be concluded that it understood that there is a need to improve
villagers are locking themselves indoors accepts the border drawn by the colonial the economic and social sectors. Hence
by 3 p.m. it is not explicitly stated at what powers. Hence, (2) is a conclusion. (4) is an inference. Choice (2)
time the dacoits are raiding the village. Choice (3):  As the Chinese were not ac- 22. As the statement refers to higher incomes
But from the given context it can be in- cepting the Mc Mahon line, it can be in- and the boom in the service sector, (1) can
ferred that the dacoits raid the village af- ferred that it is not to the advantage of the be inferred from the given statement.
ter 3 p.m. Chinese. As the statement equates higher earnings
Hence, (1) is a conclusion. Hence, (3) is an inference. Choice (4) and young age of the consumers, to abil-
Choice (2):  As it is stated that they return ity to purchase higher quality accessories,
Solutions for questions 16 to 20:
from fields, it cannot definitely say that (2) is the implicit assumption.
they are farmers, they can go to the field 16. From the words ‘in the event of mid-air The statement does not indicate when
for different purposes. emergency’ Ι follows. The statement has the boom began. Hence, (3) is out of the
Hence, (2) is not a conclusion. no reference to the President. Hence, ΙΙ context.
Choice (3):  It is said that the dacoits have does not follow. Since a helicopter was As it is explicitly stated that earnings
raided the village frequently. From this it kept on high alert, ΙΙΙ follows.Choice (3) and the desire for superior quality goods
can be inferred that there is no proper po- has increased, it can be concluded that
17. The statement refers to the Indian IT
lice protection. the purchasing power has increased.
sector being taken by storm with the an-
Hence, (3) is a an inference.  Choice (2)
nouncement by IBM. It could be that such
Choice (4):  As the villagers do not return a big investment is after a long time and 23. The word ‘decipher’ in the statement in-
before 3 p.m., so it is the assumption of not necessarily for the first time. Hence, Ι dicate that the data in the hard disk is in
the villagers that the dacoits do not raid and ΙΙ do not follow. There is no reference a coded form. Hence, Choice (1) can be
before 3 p.m. Choice (4) to the status of the company. Hence, ΙΙΙ concluded from the given statement.
13. Choice (1):  As the Governor has request- does not follow. Choice (4) There is no information as to whether the
ed the centre to dismiss the government, experts belongs to the police department
18. The statement does not relate pioneers or not. Hence, choice (2) is out of con-
the statement is not true. with the ability to give international stan-
Choice (2):  As the Governor requested text.
dard flight training. Hence, Ι does not The police are trying to find out the con-
for dismissal of the government as the
follow. From the statement, it cannot be tents of the hard disk. Hence, Choice (3)
law and order broke down, it is the as-
found out what the people do not prefer. is the assumption.
sumption of the Governor.
Hence, ΙΙ does not follow. The first state- From the passage nothing can be inferred
Choice (3):  As the governor requested
ment indicates 'being pioneers' as the about the activities of Broccoli. Hence,
the centre for dismissal, under the powers
reason for the ability to understand flying Choice (4) is out of context. Choice (3)
conferred upon it, (3) can be concluded.
better. Hence, ΙΙΙ follows. Choice (4)
Choice (4):  From the given statement it 24. The statement has no reference to the
is not explicitly mentioned that the cen- 19. ‘Giving away the rights’ is called foolish. political aspect of the cabinet’s decision.
tre has asked the Governor to confirm Hence, ΙΙ follows. The reason for which Hence, Choice (1) is out of context.
the breakdown of law and order situation the rights were given away is not men- As the cabinet has taken a decision to un-
in the state. Hence, (4) can be inferred. tioned in the statement. Hence, either Ι or der the power conferred on it, it can be
 Choice (2) ΙΙΙ follow. Choice (2) concluded. Choice (2), is a conclusion.

08_Section.indb 136 10/30/2009 11:54:05 AM


Solutions for Intelligence and Critical Reasoning  n  137

 rom the decision, the intention of the


F there are special events / celebrations and at all. Hence Ι does not follow. We can
government can be inferred that it wants that readers should reserve their copies in discount conclusion ΙΙ by taking the fol-
increase its tax revenue without bur- advance. This tells us that these supple- lowing example. Let us assume that this
dening the lower income group. Hence ments have a huge demand in the market area has 100 male and 100 female mem-
Choice (3) is an inference. which could be ascribed to their covering bers. Of this 40 women are employed in
From the decision, it is clear that the cabi- the event very exhaustively. This is exact- the government service and 100 males
net is assuming that people in the income ly what conclusions Ι and ΙΙ state, hence are all employed (as 70 per cent of the
group of more than 5 lacks per annum can both follow. Choice (4) overall population is employed). From
bear the tax burden. Hence, Choice (4) is this we cannot know whether there are
3. The statement tells us that India has a num-
an assumption. Choice (4) any couples in this area or not and hence
ber of collaborations with Germany and
25. From the statement, it cannot be inferred, this has resulted in our projects receiving concluding that 40 per cent of the couples
as to whether the rest of the book is go- financial as well as technical know-how. are married is not possible. Thus, ΙΙ does
ing to hold the interest of the fans or not. The statement does not tell us the fields not follow. Choice (3)
Hence, choice (1) is out of context. in which we do have collaborations and 6. The statement has no reference to weight
The fans may be very eager or it is possible neither does it tell us that collaborations lifting. Hence, Ι does not follow. The
that they are interested only in what hap- exist in every field. What we can however words ‘there ought to be’ indicate that at
pens to Harry Potter. As every fan wants deduce is that a collaboration leads to an present there is no such law. Hence, ΙΙ fol-
to skip to the last page, it is more probable exchange of technical know-how and ex- lows.  Choice (2)
that they are eager. Hence, choice (2) is not tension of financial aid. But we have to
a conclusion but it is an inference. 7. It is a general statement made by Brian
be careful as to not generalise this trend
The reason why the book is the best one as the statement talks only of the case of Lara and so it does not necessarily imply
is not known from the statement. Hence, Indian and German collaborations but not that he plays on Saturdays and Sundays.
Choice (3) is out of context. Hence, Ι does not follow.
of collaborations everywhere in the world.
As the readers plan to skip to the last page, From his statement, it can be concluded
Thus, we can say that some collaborations
to know what happens to Harry Potter, it that, he is aware of the difficulties of play-
lead to the exchange of technical know-
is their assumption that the information ing on Saturdays and Sundays. Hence ΙΙ
how and extension of financial aid. Based
will be available in the last page. Hence, follows. Choice (2)
on this, we can eliminate conclusion Ι on
Choice (4) is an assumption. Choice (4)
account of the words ‘every field’. How- 8. The statement does not indicate whether
ever, conclusion ΙΙ states exactly what we there is any other reason that can cause
have discussed above, hence it follows. erosion of ethnic culture. Hence, neither
Statements and
 Choice (2) Ι nor ΙΙ follows.  Choice (3)
Conclusions
4. The statement tells us that a top Holly- 9. From the statement it is clear that the num-
Practice Exercise wood actor of Indian origin has given up ber of districts in the country is more than
his profession and has come back to act 602. The statement is not clear whether
Solutions for questions 1 to 15: in Indian films. This statement, however, there are more states in the country than
1. The statement tells us that a 14 year old does not give us any clue as to the reason that indicated in the statement. Hence, Ι
convict in a bank robbery case had told for which this actor has left Hollywood does not follow.
the court, when he was being prosecuted, and as such any conjecture on this aspect Hence, neither Ι nor ΙΙ follows.
that he was inspired by movies on ‘Star would not be valid. Conclusion Ι very em-  Choice (3)
Movies’–a channel banned by the cable phatically states that ‘love for the country
10. The words ‘fate decided otherwise’ indi-
TV network. It also tells us that this is the and his desire to work in Indian movies’
cate that Sabita did not pursue the profes-
latest incident of its kind, which means made him come back to India. However,
the given statement gives no proof for sion that her parents wanted her to. Hence,
that more such offences have been com-
this hypothesis, hence this is not a defi- Ι follows. The statement does not reflect
mitted by juveniles under the influence
nite conclusion. Conclusion ΙΙ tells us that Sabita’a opinion. Hence, ΙΙ does not follow.
of movies. However, the statement does
he was ‘facing a problem’ in Hollywood,  Choice (1)
not tell us that all young criminals were
inspired by Star Movies nor can we take which also cannot be concluded from the 11. The statement tells us that USA is help-
this example and pass a judgement on all given statement. Hence, both Ι and ΙΙ do ing Pakistan build up a massive military.
juveniles and any conclusion which does not follow. Choice (3) This means that USA is an ally of Paki-
so would not be a valid one. However, we 5. The statement gives us certain facts and stan and we could also infer that USA, in
can say that a few juveniles are motivated figures about a particular area. It states that terms of military, is much stronger than
or influenced by these movies which is 40 per cent of the women are in govern- Pakistan. Students should be very cau-
exactly what conclusion Ι says and hence ment service and 70 per cent of the over- tious here as to not bring in their general
it follows. Conclusion ΙΙ generalises all awareness and relate this aspect to India,
all population is in government service.
young minds as being adventurous which as the passage does not mention India
From this we cannot infer anything about
is not correct, as isolated happenings with- in any way, nor does it explicitly state
the remaining 60 per cent of the women
in a group cannot be used to categorise a the reason as to why Pakistan intends to
of that area. Conclusion Ι states that 60
particular group, hence it does not follow. build up its military. Hence, any reason
per cent of the women are employed in
Only conclusion Ι follows. Choice (1) given for the same would be pure con-
the private sector which is not supported
2. The statement tells us that magazines by the given statement, as it could be pos- jecture and not supported by evidence
publish special supplements whenever sible that these women are not employed from the passage. Conclusion Ι is rejected

08_Section.indb 137 10/30/2009 11:54:05 AM


138  n  Solution Manual

as the statement does not hint at the rea- more than Rs 5000/-, or conversely less  he fans may be very eager or it is pos-
T
son for USA helping Pakistan. In much than 100 per cent of the supervisors earn sible that they are interested only in what
the same way the sentence does not give more than Rs 5000/-. Hence, Ι is contra- happens to Harry Potter. As every fan
any reason for Pakistan building up its dictory and ΙΙ is not certain. Hence, nei- wants to skip to the last page, it is more
military. Hence, ΙΙ also does not follow. ther Ι and ΙΙ follows. Choice (3) probable that they are eager. Hence,
 Choice (3) choice (2) is not a conclusion but is an
15. The statement is a rather philosophical one
inference.
12. The statement tells us that in the past, pi- which states that the world is neither good
As the readers plan to skip to the last
geons were used to send mail and connects nor bad and that each person interprets the
page, to know what happens to Harry
it with present day Orissa, where it is still world in his/her own way. This basically
Potter, it is their assumption that the in-
being used by the police department in means that every person perceives this
formation will be available in the last
some areas. This could be because those world in his/her terms. This implies that
page. Hence, Choice (3) is an assumption.
areas are not easily accessible to the post- some people could perceive the good side
 Choice (3)
al department, or because of the remote- of the world and some others would see the
ness of the area, sending mails through bad side. But we cannot generalise and say 19. The statement has no reference to the
pigeons is more convenient. It could also that all people find the world good or that efficiency of the British Police. Hence,
be that the police department finds pigeon all find it bad. The objection in the above Choice (1) is out of context.
mail more safe and secure when com- case is to the use of the word ‘all’ which From the statement it can be understood
pared to other ways of sending mail. All would mean a sweeping generalisation that the British Police have the authority.
in all we can conclude that pigeon mail which is something that is not supported Hence, Choice (2) is an inference.
is still being used because it is superior by the given statement. However, in this The police have released them with out
in some aspect to other ways of sending case conclusions Ι and ΙΙ both follow on changes. When there are no changes, it is
mail. This is exactly what ΙΙ states, hence account of the word ‘some’. Choice (4) assumed that they are not guilty. Hence,
it follows. Just because pigeons are being Choice (3) is an assumption.
used in some areas, we cannot conclude Solutions for questions 16 to 20: As it is stated that there are no changes
that the postal department has not made 16. The UGC while making such plans it against the suspect, it can be concluded
progress in the last century. Hence Ι does made an assumption that, surely there that the police could not find any evi-
not follow. Choice (2) will be takers for these courses. If UGC dence. Hence, Choice (4) is a conclusion.
assumes that there will not be any takers,  Choice (4)
13. The statement tells us that all patients
having high blood pressure were found it would not have made such plans. Hence, 20. The statement has no reference to the
to be having some worry or the other. At Choice (1) is implicit. political aspect of the cabinet’s decision.
first glance it appears that having high As the UGC is extending scholarships to Hence, Choice (1) is out of context.
Blood Pressure is connected to worry. those who are enrolling to these courses, As the cabinet has taken a decision to un-
However, on closer inspection we can it can be concluded that the financial bur- der the power conferred on it, it can be
see that there is no definitive evidence, den is going to be lesser. Hence, Choice concluded. Choice (2), is a conclusion.
medical or otherwise, to show that worry (2) is the conclusion. From the decision, the intention of the
causes high Blood Pressure or vice-versa The passage has no reference either to the government can be inferred that it wants
design of the course to its utility. Hence, increase its tax revenue without bur-
in the given statement. It is possible that
Choice (3) is out of context. dening the lower income group. Hence
this link is purely coincidental, hence
From the act of the UGC it can be inferred Choice (3) is an inference.
any conclusion which shows a definitive
that UGC want more girls take up profes- From the decision, it is clear that the cabi-
link between the two is to be avoided.
sional courses. Hence, choice (4) can be net is assuming that people in the income
Conclusion Ι follows on account of the
inferred. Choice (2) group of more than 5 lacks per annum can
word ‘may’ which links the two terms
17. The word ‘decipher’ in the statement in- bear the tax burden. Hence, Choice (4) is
‘worry’ and ‘blood pressure’. Conclusion
dicate that the data in the hard disk is in an assumption. Choice (2)
ΙΙ, which is the converse of Ι, also fol-
lows on account of the word ‘may not’. a coded form. Hence, Choice (1) can be Solutions for questions 21 to 25:
Hence, both conclusions Ι and ΙΙ follow. concluded from the given statement.
21. Choice (1):  It is stated that taking ‘Battle
 Choice (4) There is no information as to whether the
tanks’ on to Himalayas is almost impos-
experts belongs to the police department
14. The given statement tells us that 80 per sible even today. From this we cannot say
or not. Hence, choice (2) is out of con-
cent of the employees of X Ltd earn that there are definitely no tanks on the
text.
above Rs 5000/- per month. It also tells Himalayas. There can be some tanks also.
The police are trying to find out the con-
us that 70 per cent of the employees work So it cannot be properly concluded.
tents of the hard disk. Hence, Choice (3)
as supervisors. From this we know that 20 Choice (2):  As the General has done
is the assumption.
per cent of the employees earn Rs 5000/- what is unthinkable, in more hostile con-
From the passage nothing can be in-
or less. If this 20 per cent are all supervi- ditions, it is probable that he might have
ferred about the activities of Broccoli.
sors then we can say that at least 50 per faced some opposition, or he has chalked
Hence, Choice (4) is out of context.
cent of the supervisors earn more than Rs out a plan where there is no opposition.
 Choice (1)
5000/- per month. If none of these 20 per Hence, (2) is out of context.
cent employees are supervisors, it means 18. From the statement, it cannot be inferred, Choice (3):  The passage is not revealing
that all the supervisors earn more than Rs as to whether the rest of the book is go- who the enemy is.
5000/- per month. Thus we can say that ing to hold the interest of the fans or not. Hence, (3) is out of context.
either 100 per cent of the supervisors earn Hence, choice (1) is out of context.  Choice (4)

08_Section.indb 138 10/30/2009 11:54:06 AM


Solutions for Intelligence and Critical Reasoning  n  139

22. Choice (1):  As it is asked, to dance with law and order broke down, it is the as- Chinese.
caution, it can be concluded that the situ- sumption of the Governor. Hence, (3) is an inference.
ation is such that dancing with the power- Choice (3):  As the governor requested Choice (4):  This statement contradicts
ful is necessary. the centre for dismissal, under the powers the Indian stance in case of Mc Mahon
Hence, (1) is a conclusion. conferred upon it, (3) can be concluded. line. Choice (3)
Choice (2):  The last sentence ‘dance with Choice (4):  From the given statement it 4. Choice (1):  As it is asked, to dance with
caution’ indicates, that it is not necessary is not explicitly mentioned that the cen- caution, it can be concluded that the situ-
to throw ethics to the wind, if one wants tre has asked the Governor to confirm ation is such that dancing with the power-
to dance with the devil. the breakdown of law and order situation ful is necessary.
Choice (3):  As the two events ‘dancing in the state. Hence, (4) can be inferred. Hence, (1) is a conclusion.
with the powerful’ and ‘stepping on the  Choice (3) Choice (2):  The last sentence ‘dance with
devil’s tail’ is being talked about together, caution’ indicates, that it is not necessary
it can be understood that the author as- to throw ethics to the wind, if one wants
sumes that the powerful are devils. Inferences to dance with the devil.
Hence, (3) is an assumption. Choice (3):  As the two events ‘dancing
Choice (4):  There is no reference to the Practice Exercise with the powerful’ and ‘stepping on the
powerless. devil’s tail’ is being talked about together,
Hence, (4) is out of context. Solutions for questions 1 to 10: it can be understood that the author as-
 Choice (1) 1. It is asserted that the quality of education sumes that the powerful are devils.
is a reflection of the quality of teachers, Hence, (3) is an assumption. Choice (4)
23. Choice (1):  Though it is stated that the
villagers are locking themselves indoors but not vice versa. 5. Choice (1):  Though it is stated that the
by 3 p.m. it is not explicitly stated at what As the passage advises an improvement villagers are locking themselves indoors
time the dacoits are raiding the village. in the training of teachers, it is assumed by 3 p.m. it is not explicitly stated at
But from the given context it can be in- that there is scope for improvement. what time the dacoits are raiding the vil-
ferred that the dacoits raid the village af- Hence (2) is an assumption. lage. But from the given context it can be
ter 3 p.m. The passage states that the quality of educa- inferred that the dacoits raid the village
Hence, (1) is a conclusion. tion in the determining factor. Hence, (3) can after 3 p.m.
Choice (2):  As it is stated that they return be concluded. Hence, (1) is a conclusion.
from fields, it cannot definitely say that From the tone of the statement, it can be Choice (2):  As it is stated that they return
they are farmers, they can go to the field understood that there is a need to improve from fields, it cannot definitely say that
for different purposes. the economic and social sectors. Hence they are farmers, they can go to the field
Hence, (2) is not a conclusion. (4) is an inference. Choice (4) for different purposes.
Choice (3):  It is said that the dacoits have 2. Choice (1):  It is stated that Veerappan is Hence, (2) is not a conclusion.
raided the village frequently. From this it able to strike at will. It is also stated that Choice (3):  It is said that the dacoits have
the STF personnel were led to believe raided the village frequently. From this it
can be inferred that there is no proper po-
that Veerappan had magical powers. It is can be inferred that there is no proper po-
lice protection.
because that STF people are unable to as- lice protection.
Hence, (3) is a an inference.
sess as to when he would attack. Hence, Hence, (3) is a an inference.
Choice (4):  As the villagers do not return
(1) is a conclusion. Choice (4):  As the villagers do not return
before 3 p.m., so it is the assumption of
Choice (2):  As the STF personnel be- before 3 p.m., so it is the assumption of
the villagers that the dacoits do not raid
lieved that Veerappan has magical pow- the villagers that the dacoits do not raid
before 3 p.m. Choice (1)
ers, it is the assumption of the STF before 3 p.m. Choice (3)
24. Choice (1):  The passage has no reference personnel, that one can become invisible 6. Choice (1):  The passage does not give
to the British. with magical powers. any indication of the marital status of
Hence, (1) is out of context. Hence, (2) is an assumption. Marianna.
Choice (2):  As the Indians are using the Choice (3):  It is stated that officers are
Hence, (1) is out of context.
Mc Mahon line, it can be concluded that it afraid of posting in STF and that STF per-
Choice (2):  From the expressions ‘she is
accepts the border drawn by the colonial sonnel were lead to believe that Veerap-
able to forgive’ and ‘the humanity glows’,
powers. Hence, (2) is a conclusion. pan is invisible. From all these, (3) can
it can be understood that the author as-
Choice (3):  As the Chinese were not ac- be inferred.
sumes that forgiveness is humane.
cepting the Mc Mahon line, it can be in- Choice (4):  The statement has no refer-
Hence, (2) is an assumption.
ferred that it is not to the advantage of the ence to war tactics.
Choice (3):  The forgiveness may arise in
Chinese. Hence, (4) is out of context. Choice (3)
two situations. One is that the other person
Hence, (3) is an inference. 3. Choice (1):  The passage has no reference repented or for the reason that the victim
Choice (4):  This statement contradicts to the British. does not want to nurse a grouse against the
the Indian stance in case of Mc Mahon Hence, (1) is out of context.
offender. Hence, (3) is neither an inference
line. Choice (2) Choice (2): As the Indians are using the
nor a conclusion. Choice (4)
Mc Mahon line, it can be concluded that it
25. Choice (1):  As the Governor has request- accepts the border drawn by the colonial 7. Choice (1):  As the Governor has request-
ed the centre to dismiss the government, powers. Hence, (2) is a conclusion. ed the centre to dismiss the government,
the statement is not true. Choice (3):  As the Chinese were not ac- the statement is not true.
Choice (2):  As the Governor requested cepting the Mc Mahon line, it can be in- Choice (2):  As the Governor requested
for dismissal of the government as the ferred that it is not to the advantage of the for dismissal of the government as the

08_Section.indb 139 10/30/2009 11:54:06 AM


140  n  Solution Manual

law and order broke down, it is the as- country X can find out the whereabouts that private investment that is forthcom-
sumption of the Governor. of the don. ing in a good measure investment is very
Choice (3):  As the governor requested Hence, (3) is an assumption. little. So, the inference is definitely true.
the centre for dismissal, under the powers Choice (4):  As it is stated that the don is  Choice (1)
conferred upon it, (3) can be concluded. notorious, it can be inferred that he is in- 18. In the paragraph, it is given that “A re-
Choice (4):  From the given statement it volved in illegal activities. Hence, (4) is port on the - - - - - has been tardy prog-
is not explicitly mentioned that the cen- an inference. Choice (4) ress”. But whether a committee has been
tre has asked the Governor to confirm
appointed cannot be concluded with the
the breakdown of law and order situation Solutions for questions 11 to 25:
available information.  Choice (3)
in the state. Hence, (4) can be inferred. 11. In the paragraph, it is given that while all
 Choice (4) 19. In the paragraph, it is given that a mere
the major football powers have made it
11 million out of 66 million is enough to
8. Choice (1):  Though it is not stated ex- easily to the pre-quarter finals, it is France
effect a saving of 20,000 crore on the im-
plicitly, the intention to modernize the that under performed. The fans are not all
port bill, but nothing is mentioned about
railway station to international standards, that happy which implies that the fans are the total bill. Hence, data is inadequate.
means that they are intended for interna- happy at least to some extent and by this,  Choice (3)
tional travellers. we can conclude that despite their poor
Hence, (1) is an inference. performance they were into pre-quarter 20. Any crop which has low gestation period
Choice (2):  The statement intends to finals. So, the inference is probably true. and high yielding potential will definitely
mention that the time period for mod-  Choice (2) be a profitable one to the cultivators but
ernizing the 18 railway stations is upto whether it may be very profitable or not is
2020. 12. In the question, it is given that there definitely not known. So, the inference is
Hence, nothing can be inferred about the are 8 pools with each pool consisting probably true. Choice (2)
time period for modernizing the Patna of equal number of teams and in the 1st
round, each team played all the other Solutions for questions 21 to 25:
railway station alone.
Choice (3):  The statement does not men- teams in that pool exactly once. In the 21. The passage states that Assam has the
tion anything about possibilities. Hence, paragraph, it is given that 48 matches largest plantation belt of tea in the world
(3) is out of context. were played in the first round. So, 6 and is the largest producer in the country,
Choice (4):  From the words “the first 18 matches are played in each pool and this however it does not give us any clue as to
railway stations”, it cannot be concluded happens only when there are 4 teams in the yield per hectare. Hence no inference
that there ‘will’ not be other such railway each pool. As there are 8 pools, 8 × 4 = can be made on this aspect as the data is
station. 32 teams which participated in the world inadequate. Choice (3)
Hence, (4) is out of context. Choice (1) cup. So, the inference is definitely true. 22. The passage states that the number of
9. Choice (1):  It is stated that taking ‘Battle  Choice (1) tea estates registered with the National
tanks’ on to Himalayas is almost impos- Tea board has increased with the small
13. In this, there is no specific information
sible even today. From this we cannot say growers joining it. However the passage
whether there is a pre-tournament fore-
that there are definitely no tanks on the does not specify that this statistic is for
cast for every world cup. So the data is
Himalayas. There can be some tanks also. Assam. However since Assam has the
So it cannot be properly concluded. inadequate. Choice (3)
largest plantation belt and production, it
Choice (2):  As the General has done 14. In the paragraph, it is given that “The men is likely that a majority of the tea estates
what is unthinkable, in more hostile con- in blue and white stripes - - - - - - heirs there would be held by small time grow-
ditions, it is probable that he might have have peaked too early”. The tone of the ers. Hence it is probably true as it has not
faced some opposition, or he has chalked statement reveals that it is not too good to been conclusively proved in the given
out a plan where there is no opposition. play exceptionally well in the first round passage. Choice (2)
Hence, (2) is out of context. as that may probably effect the future
Choice (3):  The passage is not revealing 23. From the passage we know that Cropped
prospects because the opposition teams area of tea = 2,69,000 hectares
who the enemy is. will have a clue to defend. So, the infer-
Hence, (3) is out of context. Yield per hectare = 1,850 kg
ence is probably true. Choice (2) ∴ Total yield in Assam = 5,13,00,000 kg
Choice (4):  As the author compares the
conditions, it is the author’s assumption 15. In the last paragraph, it is given that “most (approximately)
that it is easier to take tanks up the motor- teams have displayed a spirit of adventure This is approximately = 51.3 million kg
able roads than on mule track. unusual for the opening round”. In this ∴ The country’s yield = Approximately
Hence, (4) is an assumption. Choice (4) “most countries” do not imply all the coun- 100 Mn kg.
tries. So, the inference is definitely false. (Since Assam accounts for 53 per cent of
10. Choice (1):  From the phrase “caught be- the country’s total production)
tween the devil and the deep sea”, it can  Choice (4)
∴ The given statement is definitely false.
be concluded that country X does not  Choice (4)
want to antagonise either party. Solutions for questions 16 to 20:
Hence, (1) is a conclusion. 24. As per the passage the industry hopes to
16. In the passage, it is clearly given that
Choice (2):  The statement does not give reverse its losses by cutting down on the
“They can enlighten - - - - - - bringing
any clue, regarding the whereabouts of perks enjoyed by the managerial cadre.
down carbon emission levels”. So, the in-
the don. This means that the managerial cadre
ference is probably true. Choice (2)
Hence, (2) is out of context. perks adds significantly to the overhead
Choice (3):  As FBI has sought the help of 17. The last statement, “Unless private in- costs hence the given statement is defi-
country X, it is the assumption of FBI that vestment - - - - - - hard to build”, implies nitely true. Choice (1)

08_Section.indb 140 10/30/2009 11:54:06 AM


Solutions for Intelligence and Critical Reasoning  n  141

25. The passage directly states that the local  hen funds are allocated only to these
W 15. Here the lack of emergency medical fa-
tea industry in Assam employs 5.9 lakhs sectors it would hamper the growth in cilities.
workers, hence the given statement is other areas. Therefore, this is a negative Ι follows, as it profits the emergency medi-
definitely false. Choice (4) course of action. cal facility.
∴ Only Ι follows Choice (1) ΙΙ is based on an assumption that hospitals
8. Here as the pillion riders are receiving does not have such facilities.
Courses of action more head injuries, helmet can save them ΙΙΙ is not a proper course of action as that
from head injuries. Ι is a proper course of may cause a new problem to the vehicles
Practice Exercise action. travelling on roads. Choice (2)
ΙΙ causes a new problem, that is, injuries 16. Here the problem is extinction of various
Solutions for questions 1 to 10: to the driver. So ΙΙ is not a proper course species of animals, so providing them
1. As the problem is the students who are of action. Choice (1) their native habitat is the proper solution
under stress the solution to the above prob- 9. In the given statement it is mentioned for the problem.
lem is to find out, how to keep the students that the chefs are inadequate but it is not ΙΙ is a proper course of action, as stopping
away from stress rather than banning the mentioned whether they are not available deforestation can solve the problem to some
violent scenes on screen. Hence, Ι does not globally or they are not available in that extent.
follow. area. If they are not available globally, the ΙΙΙ is not a proper course of action as ur-
ΙΙ increases the stress on children. It is in- question of recruiting chefs from other ban forest cannot be the habitat for the
appropriate. Hence, ΙΙ does not follows. places does not arise. Hence, Ι does not wild animals. Choice (3)
∴ Neither Ι nor ΙΙ follows. Choice (3) follow.
17. As, hawkers are not the only problem,
Because the problem is with the inadequa-
2. As the cause for extinction of polar bears hence Ι does not follow.
cy of chefs shifting to other business lead
is green house gases, we should reduce As, ΙΙ is talking about all encroachments,
to death of one type of industry. Hence, it
that. So, Ι is a proper course of action. hence it follows.
is a negative course of action.
Shifting of all the polar bears is not fea- ΙΙΙ is not a proper course of action as that
∴ Neither Ι nor ΙΙ follows. Choice (3)
sible and also the environment may not is not practically possible. Choice (3)
be suitable for them. So ΙΙ is not a proper 10. As the Indian cricket team lost continuous
18. Here the problem is with the municipal
course of action. Choice (1) six finals, a proper investigation is neces-
staff who did not take any initiative though
sary. So, Ι follows.
3. A firm or an organisation runs any busi- the water got contaminated.
The statement does not give scope to as-
ness at minimum expenses whether it is Ι is a negative course of action, hence
certain the reason.
making profit or a loss. As the present does not follow.
So ΙΙ does not follow. Choice (1)
level of expenses is not known, Ι does not Initiative must be taken by municipal
follow. Solutions for questions 11 to 20: staff, not by word officers, hence ΙΙ does
Creating on alternate domestic market is not follow.
11. As the problem is with the poor umpir-
a solution to beat the exchange rate risk. ΙΙΙ is not proper course of action because
ing decisions, hence “high standing and
Hence, ΙΙ follows. it is not practically feasible for every fam-
repute” umpires should be allowed to of-
∴ Only ΙΙ follows. Choice (2) ily to go with their own bore wells.
ficiate hence Ι is proper course of action.
4. The government should set up an inquiry Hence, none follows. Choice (4)
ΙΙ is a negative course of action.
committee to find out the reason so that ΙΙΙ is not possible. Choice (1) 19. Ι is a negative course of action.
such cases do not get repeated. So Ι is ΙΙ and ΙΙΙ are feasible and also help the
12. Course of action Ι and ΙΙΙ are negative
a proper course of action. Statement ΙΙ banks to reduce their non-performing as-
courses of actions.
is something which has to be done im- sets. Choice (3)
Here the problem is regarding pests. As,
mediately after the collapse, but the fact 20. As people can be educated regarding vita-
the course of action ΙΙ says about the
that the number of death, etc are known, mins.
research which will help to produce hy-
the time for that action is long over. ∴ This a valid course of action.
brid varieties which are strong enough
 Choice (1) ΙΙ is a negative course of action.
to fight against pests, hence it follows.
5. Increased productivity would reduce the  Choice (1) ΙΙΙ is based on the assumption that such
problem of low buffer stocks and tight types of programmes were not there be-
13. Ι and ΙΙΙ are the proper courses of actions
food situation. Hence, Ι follows. fore.
as the terrorist can attack at any place.
Course of action ΙΙ takes a long time to So, we are not certain about the result.
Ι is not a proper course of action, as this
yield result. Hence, ΙΙ does not follow.  Choice (4)
cannot be done immediately. Choice (3)
∴ Only Ι follows. Choice (1)
14. Disclosing all arms to the enemy country Solutions for questions 21 to 25:
6. Course of action Ι assumes that the pres-
is not a proper course of action to avoid 21. Here the problem is the drainage system
ent education system does not teach the
threat. Hence, Ι does not follow. and the choked openings of the drains.
basic values of life. So, Ι is not a proper
Course of action ΙΙ will create a new prob- (1) directly addresses the problems.
course of action.
lem, as that will led to arms race. Hence ΙΙ (2) would cause new problems. Garbage
Course of action ΙΙ is not giving any solution.
is not a proper course of action. would get collected at the openings.
 Choice (3)
ΙΙΙ is a proper course of action as being Hence, (3) would not help in solving
7. By allocating more funds it would be alert by all means is always the solution the problem. (4) is not related to the
easier to carry out the reforms. of a threaten. Choice (2) given problem. Choice (1)

08_Section.indb 141 10/30/2009 11:54:07 AM


142  n  Solution Manual

22. Here the problem is the open drain in that Solutions for questions 6 to 15: building is fraught with risk. Hence (1) is
locality. So, covering the nala is good in 6. As the Indian Cricket League formed not a possible consequence.
many ways. Construction of wall would in opposition to BCCI and was the first The government takes immediate relief
solve only one problem. (2) is not fea- to launch the premier league, its com- measures as a first step. One among such
sible and (4) is improper as the nalas are petitor BCCI also announced their pre- relief measures is a private make shift
already overflowing. Choice (1) mier league. So, B is the cause and A is arrangements. Hence (2), is a possible
its effect. Choice (2) consequence. Relocating the people to
23. Here the problem is the lack of play-
other district is not a practical solution.
ground. 7. Increase in air pollution and sound pol- As it leads to other problems like lose
(1) is not a solution for the given prob- lution can be the effects of a common of income source etc. Hence (3), is not a
lem. cause, that is, increase in vehicular traffic. possible consequence. Choice (3)
(2) is a negative course of action. Chil-  Choice (4)
dren cannot remain in school for 19. In the statement it is given that this pro-
8. The statements are referring to different gramme is in the planning stage and the
long. Hence, (3) is not feasible. aspects of norms and guidelines. (1) re-
 Choice (4) given consequences take place after im-
fers to uses, while (2) refers to the nega- plementation of this programme. Hence
24. Here the problem is that the infrastructure tive side of it. Hence, they are effects of a none of A, B and C are the possible con-
has not developed to match the inflow of common cause. Choice (4) sequences. Choice (4)
people. (1) is a negative course of action 9. The necessity to secure a seat on security 20. As a precaution, the government may
while (2) assumes that existing rules are council is the cause for lobbying. evacuate the people from low-lying areas.
not sufficient and (3) is not a practical so-  Choice (1) Hence, (1) is a possible consequence. It
lution. Choice (4)
10. The reasons for the exit being ignomini- is not the duty of the Meteorological De-
25. Here the problem is the rift among chil- ous and his resignation are one and the partment to take up rescue operations.
dren created by elders. same. Hence, (1) and (2) are effects of a Hence, (2) is not a possible consequence.
(1) (2) and (3) are negative suggestions. common cause. Choice (4) The Government maintains a separate
(4) is a proper course of action. 11. The attacks on villages by wild animals fund to deal with natural calamities.
 Choice (4) is an effect of decrease in forest cover. Moreover, no damage had taken place
Hence (B) is effect and A is its cause. so far. Hence (3) is not a possible conse-
 Choice (1) quence. Choice (4)
Cause and Effect 21. A is a possible consequence as NAB-
12. As the opposition parties are against to
the naval exercise they staged a protest ARD has already started micro finance.
Practice Exercise against the government. Hence, B is the Nowhere in the statement the interest
cause and A is its effect. Choice (2) rates have been referred. So, B cannot be
Solutions for questions 1 to 5: a possible consequence. The increase or
13. The T.V. programmes affect the tender decrease in the income depends on how
1. Death of 90 in one month is the imme-
minds of children. Hence, (B) is the successfully the borrower invests the bor-
diate and principal cause for forming an
cause and (A) is its effect. Choice (2) rowed money. Hence, C is not a possible
emergency medical team. Choice (1)
14. The disease has no relation with people of consequence. Choice (1)
2. The two multinationals are already op-
Madras. Hence, (1) and (2) are effects of 22. As Mr. Chatrapati is a very popular fig-
erating in India as the individual play-
independent causes. Choice (3) ure and the militants have threatened to
ers, but they got the permission to work
together. Getting permission to operate 15. The decrease in cost of petrol in inter- kill him at any cost, it is possible that he
jointly is the immediate and principal national market and decrease in cost of would be provided with the best security.
cause but having individual approvals is petrol driven vehicles are effects of inde- Hence, (1) is a possible consequence.
not the principal cause. pendent causes. Choice (3) As Mr. Chatrapati is under threat, it is
Hence, A is the effect and B is not its im- possible that his movements are restrict-
Solutions for questions 16 to 25: ed. Hence (2) is a possible consequence.
mediate and principal cause. Choice (3)
16. The industry cannot recruit people. If it is found that militants are really mak-
3. As the traffic cops have been equipped Hence, (1) is not a possible consequence. ing efforts, then the question of retaliation
with a sophisticated instrument, motorists Those who are planning to join another arises. Hence, (3) is not a possible conse-
in the city will be more cautious. So, A is industry, do not go on strike. Hence, (2) is quence. Choice (3)
the immediate and principle cause and B not a possible consequence. 23. A and B are possible consequences as
is its effect. Choice (2) When the Trade Union serves a notice of the demand increases when there is a
4. The terrorists deliberately want to cre- strike, the management may call them to shortage. But, we cannot say whether the
ate havoc among the people, hence they resolve the issue. Hence (3), is a possible number of unemployed graduates will in-
perpetrate activities like bomb blasts and consequence. Choice (2) crease or decrease. Choice (1)
kidnappings. 17. As banks are flash with cash, loans may 24. When big players enter the market with
Hence (2) is the immediate and principal be more easily available and applica- huge investments, it is quite possible
cause and (1) is its effect. Choice (1) tions may be processed faster. So both that small business find the going tough.
5. The events are related. Chronologically (B) and (C) are possible consequences. Hence, (1) is a possible consequence.
A occurs before B. A can be the immedi-  Choice (3)
Purchasing smaller companies is one way
ate and principle cause for the effect B. 18. The place is located in a seismically sensi- of penetrating into the market. Hence (2)
 Choice (2) tive area. In such places, a multi storeyed is a possible consequence. When MNCs

08_Section.indb 142 10/30/2009 11:54:07 AM


Solutions for Intelligence and Critical Reasoning  n  143

enter the market with huge investments 9. Ι is strong, because it is necessary to iden- 16. Argument Ι says that the prime minis-
the possibility of large scale changes in tify the words which are unparliamentary ter must be “young enough” for some
the services is high. Hence, (3), is a pos- so that the usage of such words can be explained reasons. This implies that the
sible consequence. Choice (1) avoided. ΙΙ is strong because it cautions prime minister must not be older than
25. The basic purpose of their visit is to es- against children learning it. Children a certain age limit. This argument is
timate the loss incurred due to floods learning unparliamentary words is unde- strong.
and to submit a report to the government sirable. Choice (4) Argument ΙΙ: This says that there should
to take further decisions. So A is a pos- 10. Ι is strong because it refers to the cost as- not be an upper limit because older per-
sible consequence. B is a consequence sociated with it. sons can perform better because of the
that takes place after studying the report. ΙΙ is not strong as it is not directly rel- experience gained. This also is strong.
But it is not a direct consequence of the evant. Choice (1)  Choice (3)
given statement. We cannot say whether
the aid will be properly utilized or not. Solutions for questions 11 to 20: 17. Ineffectiveness of the existing players is
 Choice (1) an important point to be considered while
11. “Free food grain supply” implies that the
examining new proposals. Hence, argu-
neighbouring country is in trouble. Argu-
ment Ι is a strong argument. If true, even
ment Ι explains the troubles and is a strong
Strong and Weak argument ΙΙ which says that national secu-
argument. The fact that the other country
rity could be put in jeopardy, would pro-
Arguments is an enemy does not stop us from helping
vide a valid and strong argument against
them as we have already understood that
it is in trouble. Therefore argument ΙΙ is a foreign media networks being allowed in
Practice Exercise India. Choice (3)
weak argument. Choice (1)
Solutions for questions 1 to 10: 12. Argument Ι explains a valid reason to 18. One important reason to stop children
go against such marriages because it is from using kiosks is adult material avail-
1. Ι is incomplete. Hence, Ι is not strong.
wrong to encourage anything that jeopar- able on the net. Hence, argument Ι is a
ΙΙ is not strong as the relation between
team's strength and conducting the event dises the health of a person. Hence, (Ι) is strong argument. Argument ΙΙ gives an
is not established. Choice (3) a strong argument. irrelevant reason as its argument and is
Argument ΙΙ is not a valid argument be- very weak. Choice (1)
2. Ι is strong because it is relevant and states cause a custom being followed since a
that the suggested course of action is a 19. Both the arguments, if true, are valid and
long time does not necessary make it strong arguments. Choice (3)
way of achieving the desired result. good. Only Ι is strong. Choice (1)
ΙΙ is not strong because it does not directly 20. Argument Ι : The basic purpose of con-
deal with the question. Choice (1) 13. Argument Ι is pessimistic in nature be-
ducting classes is to help students. Hence,
cause it says that corruption can never
3. Ι is strong, because it assumes that reducing argument Ι is a strong argument as it con-
be eliminated, but we cannot check the
school time does not improve their natural veys this idea.
validity of the argument and we have to
talents as they do not concentrate on it dur- Argument ΙΙ : If the institute conducts
take this argument to be true, in which
ing their free hours. classes with the intention of making prof-
case it is to be admitted that argument
ΙΙ is strong because unlike regular, cur- its, then this is a valid point to be consid-
Ι is a strong argument. Argument ΙΙ is a
ricular natural talents are diverse. Hence, very strong argument because a person’s ered. Hence, argument ΙΙ is also strong.
it is not possible to pay attention to such integrity is a major consideration for giv-  Choice (3)
diverse activities. Choice (4) ing him or her a job. Both the arguments Solutions for questions 21 to 25:
4. Ι is not strong because being popular is are strong. Choice (3)
21. Knowing a person from Kerala is not a rea-
not a criteria to be a teacher. ΙΙ is strong 14. Sanctuary for endangered animals be- son for considering a person from Manipur
because subject knowledge is necessary comes necessary when it improves and
to be a teacher. Choice (2) as a foreigner. Hence (1) is not the proper
betters the living conditions of the world.
explanation.
5. Ι is strong because it states that parents But at the same time, if an economy is
Since, the colleague, who is from Kerala,
are the first people from whom a child not able to provide basic necessities to its
never interacted with a person from Ma-
learns. ΙΙ is irrelevant. Choice (1) people, then it cannot think of construct-
nipur, it is possible that he has taken him
ing sanctuaries. Therefore, the decision is
6. Ι is strong because it mentions that there as a foreigner. Hence, (2) is the proper
based on both the factors viz. requirement
are other areas of good work that need to and fund supply. Hence, both Ι and ΙΙ are explanation.
be identified. strong arguments. Choice (3) Not being from Manipur does not explain
ΙΙ is based on the assumption that more why the Keralite is not considered as a
number prized would make it worthless. 15. Argument Ι says that total cost is increas- foreigner.
Hence, ΙΙ is not strong. Contradict each ing. But this does not mention whether Choices (4) contradicts the given statement.
other. Choice (1) production is also correspondingly in-  Choice (2)
creasing or not. If production is also
7. Failure on one occasion is not a sufficient increasing, then there would not be any 22. Choices (1) and (2), strengthens the fore-
reason to stop. Hence, Ι is not strong. ΙΙ is problem. Hence, this argument is weak. cast made. From choice (3), it is clear
comparative, hence not strong.Choice (3) Argument ΙΙ : We do not know whether that an elderly person is back in the race
8. Ι is strong because good returns are a de- the employees would be removed or and except for him, no one else was in
sirable positive result. not. It is likely that the employment is a position to give a serious threat to the
ΙΙ is irrelevant as it does not talk of any not disturbed. Hence this also is a weak champion. Hence, choice (3) weakens the
loss due to addiction. Choice (1) argument. Choice (4) forecast.

08_Section.indb 143 10/30/2009 11:54:08 AM


144  n  Solution Manual

 ajor tournaments are not being conduct-


M follow. RΙΙ gives the reason why the 12. RΙ : "People's communication skills".
ed at junior level does not mean that there thieves are to be kept in Jail. Hence, RΙΙ This phrase implies "people's communi-
is no talent among the young generation. follows. Choice (2) cating ability", but not the media’s. This
Hence, choice (4) neither strengthens nor 3. RΙ is a reason because it is a well known is not a valid reason.
weakens the forecast. fact that villages in the vicinity of big RΙΙ : Even if it is true that science has cre-
 Choice (3) dams get affected. RΙΙ is not a reason as ated this medium, it is people who accept
23. Exemplary performance will not be pos- such constructions may be useful from it and make it popular. No reason is given
sible always. Hence, choice (1), strength- the employment point of view and may in RΙΙ to explain why this medium is grow-
ens the need for specialist batsman. be harmful in view of environmental as- ing. Neither RΙ nor RΙΙ is a valid reason.
The statement talks about the weakness pects. Choice (1)  Choice (4)
in the batting, but not the pressure on the 4. The purpose of having chimney is to re- 13. “Indifferent” means having no opinion.
bowlers. Hence, choice (2) is irrelevant. lease smoke at a height, but not to have an That is, these people are neither against
According to choice (3), the score put to- identity. Hence, only RII follows. nor advocating reservations. Neither
gether by batsmen need to be defended by  Choice (2) RI nor RΙΙ is explaining the reasons
the bowlers. Hence, replacing a bowler is as to why these people are indifferent.
not a good suggestion. The batting should 5. Neither the size nor the population is the  Choice (4)
be strengthened without weakening the reason for the railways being the largest.
Because, there are countries which are 14. RΙ : This explains the cause for the op-
bowling. Hence, choice (3), weakens the
bigger than India and there are countries position of the environmentalists towards
recommendation of the coach.
which have larger population than India, polythene bags and is valid.
Choice (4), would strengthen the need for
but it does not mean that they have large RΙΙ: If polythene is made using natural
a specialist batsmen. Choice (3)
railways. Hence, neither RΙ nor RΙΙ fol- products which do not harm the environ-
24. The author assumes that no other part in ment, then it would not be a problem for
lows. Choice (3)
India is as beautiful as Shillong. the environmentalists. However, it could
Choice (1) supports the assumption. 6. Because time and tide occur naturally, no
be a problem only when the natural prod-
Choice (2) does not compare the beauty one can stop them. Hence, Ι follows. ΙΙ
ucts used in the bags are themselves dan-
of the cities in which the author lived with does not follow because it is hypothetical.
gerous to earth in one way or the other but
the beauty of Shillong. Hence, (2) does  Choice (1)
this is not explained by RII. Hence, it is
not weaken the assumption. 7. The popularity or otherwise of a contes- not a valid reason. Choice (1)
The statement does not mention, if the au- tant is not the only criterion that affects
thor has seen any other city from a hilltop 15. RΙ can be a valid reason because it ex-
an election result. Choice (3) plains the process by which a child be-
or not. Hence, choice (3) does not weaken
the assumption. 8. Dearness Allowance is increased to off set comes a genius. RΙΙ is invalid because
As there are cities which are more beauti- the effect of the rise in prices of essential we cannot say that "Only a genius mind
ful than Shillong, this statement, if true, commodities. Hence, neither Ι nor ΙΙ fol- is curious. Choice (1)
would seriously weaken the author’s as- lows. Choice (3) 16. RΙ is not explaining the reasons behind
sumption. Choice (4) 9. AIDS is acquired immune deficiency syn- the increasing number of deaths. RΙΙ
25. Choice (1) has no relevance to the state- drome, it is not a disease, hence it can- is a valid reason and under the circum-
ment. not be cured. Hence, RΙ follows. Since, stances given by RΙΙ (the law is weak),
Choice (2) does not mention any achieve- cure has not yet been discovered, it re- the law-breakers may behave adamantly.
ment that the American mission to Mars mains incurable. Both RΙ and RΙΙ follow.  Choice (2)
has resulted in.  Choice (4) 17. RΙ : If only computer education is useful,
Choice (3) is only an assumption of the 10. Many books have been written on the life then more people may tend to join these
Russians. Hence, it does not weaken the of Mahatma Gandhi because of his great courses. If more people join and the
proposed Indian mission to the Mars. achievements and because his life allows existing coaching centres are not able to
When no good scientific objective can be us to learn many good things. Hence, only accommodate the new joining, then new
achieved, the proposal for the Mars mis- RI follows. RΙΙ is not a reason for books centres would be started. The reason ex-
sion would be weakened. Hence, choice being written on Mahatma Gandhi and so plained in RΙ is based on a hypothetical
(4) if true, would weaken the proposal. RΙΙ does not follow. Choice (1) situation ("no other education is useful").
 Choice (4) Moreover, it very vaguely and indirectly
Solutions for questions 11 to 20: explains the causes for the assertion
Practice Exercise 11. “Standing next to America” cannot be made. Hence, this cannot be taken as a
(Assertions and Reasons) taken as a valid reason for Indian defence valid reason.
endeavours. Any action of a government RΙΙ : This explains the cause behind the as-
Solutions for questions 1 to 10: sertion and is valid. Choice (2)
evolves out of necessity or urge for devel-
1. RΙ is a reason as the late detection of fire opment. Hence, RΙ is not a valid reason. 18. “Capital punishment” means sentencing
would render it difficult to control. RΙΙ RΙΙ is a valid reason, as India has "hos- to death. Both RI and RΙΙ give strong
is a reason as the speed with which it tile” neighbours. “Hostile” implies the reasons to abolish such a law, hence both
spreads makes it difficult to control. presence of inimical and dangerous follow. Choice (3)
 Choice (4) neighbours, which would necessitate high
level of defence preparedness. Hence, RΙΙ 19. RΙ :  The opinion of the political leaders is to
2. Having power is not a sufficient reason
is a strong and valid reason. Choice (2) create a good impression in the minds of peo-
to arrest some one. Hence, RΙ does not

08_Section.indb 144 10/30/2009 11:54:08 AM


Solutions for Intelligence and Critical Reasoning  n  145

ple (to be in the good books of people) hence, Step ΙΙΙ  :  ×3 ×3 ×3 ×3 ×3 ×3 step and neither can we find which words
they provide the facilities as mentioned in the 9 15 45 48 84 114 have been actually rearranged and which
"Assertion". Thus, RΙ is a strong reason. Step ΙV  :  Sum of the digits were initially in their correct positions.
RΙΙ : This is not valid, as the money spent 9 6 9 12 12 6  Choice (4)
on such activities belongs to the govern- Step V  :  Arrange in ascending order
6 6 9 9 12 12 Solutions for questions 11 to 15:
ment. Choice (1)
The numbers given in the output are shifted one
20. The only reason behind getting an award 1. Choice (1) position to their right cyclically in the output.
can be outstanding performance. Hence, 2. Choice (3) In Step I, the first number in the input has in-
RΙ and RΙΙ are not valid reasons be- terchanged its position with the second number.
hind Hindi films getting many awards. 3. Choice (4)
In Step II, the third number has interchanged its
 Choice (4) 4. Choice (4) position with the first number. In Step III, the
5. As sorting is done from step ΙΙ, fourth number has interchanged its position with
Solutions for questions 21 to 25:
we cannot find step Ι. Choice (4) the first number. This is followed in the steps
21. Here the given assertion is about the pref- further, till all the numbers are arranged by shift-
erence of today’s youth. Solutions for questions 6 to 10: ing by one place to the right cyclically.
RΙ is talking about the strong base of west- As we observe the output, we know that the
words in the given input are arranged in the in- 11. The given input is
ern music. Having strong base does not
creasing order of the number of letters. When Input  :  256 159 386 125 81 64 121
imply that it is good to listen.
the words of equal number of letters occur, then Step Ι  :  159 256 386 125 81 64 121
RΙΙ tells that youth like rocking music but
words are arranged in the order as they appear Step ΙΙ  :  386 256 159 125 81 64 121
there is no evidence that western music
in the dictionary. In each step, one word is be- Step ΙΙΙ  :  125 256 159 386 81 64 121
is rocking.
ing rearranged. Among words with equal num- Step IV  :  81 256 149 386 125 64 121
∴ Neither RΙ nor RΙΙ is a reason.
ber of letters, the word that appears first in the Step V  :  64 256 159 386 125 81 121
 Choice (3)
dictionary is arranged first and so on. The input Step VI  :  121 256 159 386 125 81 64
22. In the given assertion it is given that Mr. Hence, the last but one step is Step V.
is “adopted action to a stamp drafted general
P. got Nobel Prize for Peace last year operation”. In the given input, the word with  Choice (4)
which can be for his work among the minimum number of letters is ‘a’, hence it oc- 12. In each step, only one element is rear-
poor. Hence RI can be a reason while RII cupies the first position in the first step and the ranged. Hence, the total number of steps
is not a valid reason Choice (1) remaining words follow the same sequence as we require to get the final output is (n −
23. Here in the assertion, we are only talking in the input. The second position is occupied 1), where ‘n’ is the total number of ele-
about the weakness of the employees. RΙ by the word ‘to’, as it contains the next highest ments in the input.
is giving the reason for skipping break- number of letters. Similarly the other words are In this input, the total number of elements
fast, so it is not the reason. RΙΙ is also not also arranged. = 7.
a valid reason, as lack of mental exercise 6. The required arrangement of words can Hence, we require (7 − 1) = 6 steps to get
may not lead to deterioration. Choice (3) be obtained as shown below: the final output. Choice (1)
24. The interesting topic and the art of writ- Input  :  we do at is exact fund your life
13. In the output the numbers given in the in-
ing is a proper reason for the popularity of Step Ι  :  at we do is exact fund your life
put are rearranged by shifting each num-
an author. Hence RΙ is a reason. RΙΙ is not Step ΙΙ  :  at do we is exact fund your life
ber by one position to the right cyclically.
the reason, because if the subject is not Step ΙΙΙ  :  at do is we exact fund your life
Hence, to get the input from the output
good, language does not help. Choice (1) Step IV  :  at do is we fund exact your
the converse logic has to be applied, that
life
25. Here the assertion is about the collapse is, the numbers should be shifted to their
Step V  :  at do is we fund life exact your
of the newly constructed bridge, which is left by one place. Hence, the input is 94
Hence, StepVhas the required arrangement.
possible because of faulty construction or 32 51 87 13 7 23 58. Choice (1)
 Choice (4)
because of improper design. Hence, Ι can 14. Choice (4)
be reason. 7. The final output is reached as shown un-
der: 15. It is given that the first eight prime num-
RΙΙ talks about type of soil, which is out
Input : post followed after government bers are taken in ascending order as the
of context. Choice (1)
fallen nomination the of input. Hence, the input and the output will
Step Ι : of post followed after govern- be as follows.
ment fallen nomination the Input  :  2 3 5 7 11 13 17 19 Choice (3)
Input and Output Step ΙΙ : of the post followed after gov-
ernment fallen nomination Solutions for questions 16 to 20:
Step ΙΙΙ: of the post after followed gov- The words given in the input are arranged in the al-
Practice Exercise
ernment fallen nomination phabetical order in the output. Let us now analyse
Solutions for question 1 to 40: Step IV :of the post after fallen followed the input and the steps through which the output
government nomination is determined. The given input is “taking decision
Solutions for question 1 to 5: three clear expects happen next public”. In the
Hence, Step IV is the final output for the
The logic is as follows. given input. Choice (3) given input, the word “clear” comes first in the dic-
Input  :  15 17 35 7 23 11 tionary, hence it occupies the first position and the
8. Choice (4)
Step Ι  :  +1 −2 +3 −4 +5 −6 remaining words follow the same order as they are
9. Choice (3) in the input. In the second step, the second position
16 15 38 3 28 5
Step ΙΙ  :  Sort in ascending order 10. By back tracking, we cannot determine is occupied by the word “decision”. Similarly, the
3 5 15 16 28 38 the arrangement of words in the previous other words are also rearranged.

08_Section.indb 145 10/30/2009 11:54:09 AM


146  n  Solution Manual

16. The last step for the given input is as occupies the fourth place. Similarly, the other Step ΙΙ  :  7 3 8 2 6 1 5 4
shown below: words and the numbers are arranged. Step ΙΙΙ  :  8 7 5 1 2 3 4 6
Input:  products retail growth share little 21. The given input is Step IV  :  5 8 4 3 1 7 6 2
option board base Input:  sat 726 rat 534 mat 684 gate 436 Step V  :  4 5 6 7 3 8 2 1
Step Ι: base products retail growth share bite Step VI  :  6 4 2 8 7 5 1 3 and so on.
little option board Step Ι:  bite 726 rat 534 mat 684 gate 436 Hence, based on the above concept, let us solve
Step ΙΙ: base board products retail growth sat the following questions.
share little option Step ΙΙ:  bite 726 gate 534 mat 684 rat
Step ΙΙΙ: base board growth products re- 26. Choice (1)
436 sat
tail share little option Step ΙΙΙ:  bite 726 gate 684 mat 534 rat 27. As shown in the given table, for the input
Step IV: base board growth little prod- 436 sat of words 1, 2, 3, 4, 5, 6, 7 and 8 the words
ucts retail share option Hence, Step ΙΙΙ is the last step. in Step V are in the order of 4, 5, 6, 7, 3,
Step V: base board growth little option Choice (4) 8, 2 and 1. Choice (4)
products retail share
22. Choice (3) 28. It is given that Step IV of an input is “fall
Step V is the final output. Choice (2)
into hands grip doing from the view”.
17. Choice (1) 23. In these type of questions we cannot de- In the above given table for the input of
termine the input from the output, that is words 1, 2, 3, 4, 5, 6, 7 and 8, the order
18. Choice (4) working out in reverse is not possible, be- of the words in Step IV is 5, 8, 4, 3, 1, 7, 6
19. In these type of questions, we cannot find cause the initial position of numbers and and 2. Choice (3)
the input from the output. It is not pos- the words cannot be determined. Choice
sible to know the initial position of the (4) 29. In the above given table for the input of
words in the input. Choice (4) words 1, 2, 3, 4, 5, 6, 7 and 8, the order of
24. Choice (1)
the words in Step VI is 6, 4, 2, 8, 7, 5, 1,
20. Choice (3) 25. Choice (4) 3.
Solutions for questions 21 to 25: Step VI  : 6 4 2 8 7 5 1 3 (These num-
Solutions for questions 26 to 30:
As we observe the output through every step, we bers are arranged in the order
As we observe the output through every step, we
know that the words are arranged in the same or- of 3, 1, 7, 6, 4, 2, 8, 5)
find that the words in each step are arranged in
der as they are in the dictionary and the numbers Step VII  : 2 6 1 5 8 4 3 7 (These num-
the order of 3, 1, 7, 6, 4, 2, 8, 5 to get the order of
are arranged in the decreasing order of their val- bers are arranged in the order
words in the next step. Let us number the words
ue. Every word is followed by a number. In Step 3, 1, 7, 6, 4, 2, 8, 5)
of the input as 1, 2, 3, 4, 5, 6, 7 and 8. In the
Ι, among the given words in the input, “global” Step VIII  :  1 2 3 4 5 6 7 8
input the words in the positions of 3, 1, 7, 6, 4,
comes first in the dictionary. Hence, it occupies Hence in Step VIII, the same input re-
2, 8, 5 are arranged to get the words in Step Ι. In
the first position. The largest number in the in- peats as in the output. Choice (1)
Step Ι, the words of order 3, 1, 7, 6, 4, 2, 8, 5 are
put is 356. Hence, 356 occupies the second posi-
arranged to get the words in Step ΙΙ. Similarly 30. Step IV of an input is “public citizens
tion in Step I and 356 interchanges its position
the same order is followed to get the order of the health where against action further de-
with the word in the second position. In Step ΙΙ,
words in the successive steps. cided” and the order of the words in this
the next word which comes in the dictionary is
Let us represent the positions of the words and step is 5, 8, 4, 3, 1, 7, 6 and 2. The order
“higher”. Hence, it occupies the third place and
their arrangement in further steps. of the words in the Step ΙΙ is 7, 3, 8, 2, 6,
interchanges with the word/number in that posi-
Input  :  1 2 3 4 5 6 7 8 1, 5 and 4. Choice (1)
tion. In the next step, the second highest number
Step Ι  :  3 1 7 6 4 2 8 5

Decision Making
Practice Exercise
Solutions for questions 1 to 5:
Q.No. City (i) (ii) (iii) (iv) Coarse of action
Population > 20 income of atleast 50 per well connected with Average temperature
[a] cent population is ≥ 2lpa major cities of the during summer > 20°C
less than 10 per cent of [b] world
population has annual sales of refrigerators in previ-
income < 1.5lpa ous year > 5000.
×
1 Nagpur    Launch of 446ZMQ
[]
×
2 Lucknow   
[]
Launch model
no.518JTX
3 Kolkata    
486ZMQ
446ZMQ
4 Delhi  ?   Data inadequate
5 Bhopal ×    Data inadequate
[?]

08_Section.indb 146 10/30/2009 11:54:09 AM


Solutions for Intelligence and Critical Reasoning  n  147

1. Does not satisfy the basic condition (ii)  Launches models numbered 486ZMQ
∴ 4. No clear information about condition (ii).
but satisfies the alternate condition. and 446ZMQ. Choice (2)  Choice (4)
∴Launches only 446ZMQ. Choice (2)
3. Satisfies all the basic conditions. 5. No clear information about the alternate
2. Does not satisfy condition (i) but satisfies ∴All the 3 models will be launched. condition of (i). So data is inadequate.
the alternate condition.  Choice (1)  Choice (4)

Solutions for questions 6 to 10:


Q.No. Number (A) (B) (C) First & last digits are perfect squares. (D)
Seven digits At least 2 prime [E] Middle digit even [F]
digits First & last digits are even All digits are odd.

6. 1778459    
7. 1366789    
×
8. 1367574   
[×]
9. 964374 ×   ×
10. 9632578   × 
[×]

6. Satisfies all the basic conditions. 8. Does not satisfy condition (D) and also its ∴ Garbage number. Choice (4)
∴ Superior number. Choice (1) alternate condition.
10. Does not satisfy condition (C) and also its
∴ Garbage number. Choice (4)
7. Satisfies all the basic conditions. alternate condition.
∴ Superior number. Choice (1) 9. Does not satisfy the basic criteria (A) and ∴ Garbage number. Choice (4)
(D).

Solutions for questions 11 to 15:


Question (a) (b) (c) (d) Remarks
Number Land Cost < 20L Est.Cost of Const. < 30L Posh locality Loan ≥ 20L @ maxm.
18 per cent p.a.
11  ×   (b) violated
12   ? ? (c), (d) unknown
13    ? (d) unknown
14   ×  (c) violated
15 ×   × (a) and (d) violated

11. In this case, condition (b) is violated, as (g) is satisfied. Hence, the matter should (e) If condition (d) is violated, but at least one
the estimated cost of construction is more be discussed with the family members. relative of the candidate is presently stay-
than Rs 30 lakhs. As there are flats avail-  Choice (3) ing in A.P. for the last 5 years, then the can-
able in that locality on the ground floor,
15. In this case, the land cost is Rs 20 lakhs, didate can be admitted.
the respective alternate condition (f) is
whereas according to condition (a) the (f) If condition (b) or (c) is violated, but not
fulfilled. Choice (1)
cost of land should be less than Rs 20 both, and the candidate has a good sports
12. As no information is available to check lakhs. Hence, condition (a) is violated. record, then the candidate should be ad-
conditions (c) and (d), the data is inad- As there is no alternate condition for the vised to approach the Principal.
equate to take any decision. Choice (2) same, the house cannot be constructed. (g) If both the conditions (a) and (b) are vio-
13. In this case, the loan amount is not speci-  Choice (4) lated, but the candidate is ready to donate
fied, which is required to check condition Rs 1,00,000 to the college, then the can-
Solutions for questions 16 to 20:
(d). Hence, data is inadequate to take a didate should be advised to approach the
The four basic conditions from (a) to (d), given
decision. Choice (2) Secretary.
in the selection criteria, are as shown in the ta- Now let us scrutinise the applicants for the
14. In this case, condition (c) is violated as ble below. In case a basic condition is violated, basic conditions, as given in the table be-
the land is not located in a posh locality. the case is verified for the respective alternate low (a tick mark “”means that the condi-
However, as all the basic amenities are condition given. The alternate conditions are as tion is fulfilled, cross mark “X” means that
available nearby, the alternate condition given below: the condition is violated).

08_Section.indb 147 10/30/2009 11:54:09 AM


148  n  Solution Manual

Question Name of the (a) (b) (c) (d) Remarks


Number candidate SSC ≥ 80 per cent Entrance Age ≥ 16 years, Resident of A.P.
or Exam ≥ 60 per cent male candidate
CBSE ≥ 70 per cent
16 Prakash ×    (a) violated
(c) & (d) unknown,
17 Rahim × × ? (age) ?
(a) &(b) violated
18 Jahangir    X (d) violated
19 Rita   × (female)  (c) violated
20 Ranga × ×   (a) and (b) violated

16. In this case, condition (a) is violated. As dition (f) is applied. As Rita has a good in films, but has a good success rate with
there is no alternate condition given for sports record (represented state in basket- the heroine he is going to work with in the
violating condition (a) alone, admission ball), she should approach the Principal. film, then take the suggestion of the hero-
cannot be granted to Prakash. Choice (2)  Choice (3) ine. OR
20. In this case, both the conditions (a) and If condition (a) is violated, but the hero is
17. In this case both the conditions (a) and (b)
(b) are violated. Then, Ranga’s case is successful in the type of films the Producer
are violated in lieu of which the alternate
verified for the alternate condition (g). is making, then take the advice of the Pro-
condition (g) is satisfied
(that is Rahim can pay a donation of Rs As Ranga is ready to pay a donation of ducer.
1,00,000). But, no information is avail- Rs 1,00,000, he should be advised to (f) If condition (c) is violated, but the hero de-
able to check conditions (c) and (d), approach the Secretary of the school. mands a remuneration of less than Rs 50
hence we cannot take a decision as the  Choice (3) lakhs, then he can be selected.
data is inadequate. Choice (4) (g) If condition (d) is violated, but the hero can
Solutions for questions 21 to 25:
work for at least 15 days in a month, may
18. In this case, condition (d) is violated, that The four basic conditions, from (a) to (d), given
not be continuous, then the hero can be se-
is Jahangir is not a resident of A.P. Hence, in this selection criteria, are as shown in the
lected.
his case is verified for the alternate con- table below.
dition (e). Since, a close relative of Jah- In case a basic condition is violated, the respec- Now let us scrutinise all the applicants for
angir has been staying in A.P. for the last tive alternate condition is applied in order to the basic conditions (from (a) to (d)) as
10 years (that is more than the required 5 take a decision. given in the table below (a tick mark “"
years), Jahangir is selected. Choice (1) The alternate conditions are as given below: means that the condition is fulfilled and a
19. In this case, condition (c) is violated, that (e) If condition (a) is violated, and the hero has cross mark "X" means that the condition is
is Rita is a female. Then the alternate con- a success rate of greater than 50 per cent violated):

Question Name of (a) (b) (c) (d) Remarks


Number the Hero Success rate as a Time spent Remuneration Dates continuous
hero in the Industry < Rs 10 lakhs ≥ one week
≥ 70 per cent ≥ 5 yrs

21. Romeo     All conditions are satisfied

22. Ritesh ×    (a) violated

23. Roopesh ?   ? (a) & (d) unknown

(b) & (d) unknown and (a) and (c)


24. Chris × ? × ?
are not satisfied.
25. Hasmukh ×    (a) violated

21. In this case, Romeo satisfies all the con- record for all the 8 years is unknown, and as Chris’s experience in Industry and
ditions, hence he is selected as a hero. also his ability to give dates is not men- whether he can give dates continuously
 Choice (1) tioned, hence data is insufficient to take a for at least one week, is not known.
22. In this case, condition (a) is violated, then decision. Choice (4) Hence, the data is inadequate for taking a
the alternate condition (e) is tested. As decision. Choice (4)
24. In this case, condition (a) is not satisfied,
Ritesh has 90 per cent hits in the type of as the information given regarding the 25. In this case, condition (a) is violated. But
films the producer is making, Producer’s success rate is not that of his success as a as Hasmukh is very successful with the
advice should be taken. Choice (3) hero. Similarly, (c) is not satisfied, as he heroine of the film, he fulfils the alternate
23. In this case, both the conditions (a) and charged more than Rs 10 lakhs per film. condition (e). Hence, the suggestion of
(d) cannot be checked, as Roopesh’s track Conditions (b) and (d) cannot be checked, the heroine should be taken. Choice (3)

08_Section.indb 148 10/30/2009 11:54:10 AM


Solutions for Intelligence and Critical Reasoning  n  149

Solutions for questions 26 to 30:


Question (a) (b) (c) (d) Remarks
Number Every student No body watched Theatre ≤ 3 kms Every student
attended all lectures the movie earlier agrees
26  ×   (b) violated
27   ×  (c) violated
28  ×   (b) violated
29     All conditions are satisfied.
30  ×   (b) violated

26. In this case, condition (b) is violated as perpendicular plane (giving 3 x 2 = 6 Solutions for questions 9 to 11:
some students have seen the movie earli- parts) and the fourth cut along the other 9. Cutting the large cube into 27 smaller
er. As the number of these students is less perpendicular plane (giving 6 x 2 = 12 cubes will give us a 3 x 3 x 3 configura-
than half of the total number of students parts). Choice (2) tion. Out of these, if we remove all the
in the group, the alternate condition (f) is outer cubes to get the number of cubes not
3. Five cuts are distributed 2 along each
fulfilled. Hence, all of them go to watch
of the two perpendicular planes and having any face painted at all, we have to
the movie. Choice (1)
one along the third perpendicular plane. remove one layer of cubes on each of the
27. In this case, the theatre is more than 3 km Hence, we get 3 x 3 x 2 = 18 pieces. faces so that we are left with a 1 x 1 x 1
away, hence condition (c) is violated. But  Choice (3) cube which is not painted at all. Hence,
as at least two seniors are accompanying the answer is one cube. Choice (2)
them to watch the movie, the alternate 4. Four cuts can be along one plane (giving
5 parts), another four cuts along a per- 10. The cubes which are not along any edge
condition (g) is fulfilled. Hence, all of
pendicular plane (giving 5 parts) and the are the ones that have only one face paint-
them can go to watch the movie.  Choice
remaining five cuts along the other per- ed. On each face of the original cube, if
(1)
pendicular plane (giving 6 parts). Hence we do not count the faces along the edges,
28. Here, condition (b) is violated, that is 2 out the maximum number of identical pieces then we have only one face at the middle
of 10 students have already watched the which is painted only on one face. Hence,
will be 5 x 5 x 6 = 150 Choice (4)
movie earlier. But as the movie is a hit, and for six faces of the original cube, we get
more than half the total number of students 5. 12 cuts should be divided equally paral- six cubes that have only one face painted.
(that is, 8 out of 10) would be watching the lel to the three perpendicular planes - 4  Choice (2)
movie for the first time, the alternate con- parallel to each of the three planes. When
dition (f) is satisfied. Hence, all of them go there are 4 cuts parallel to the plane, 11. The cubes along the edges but not at
to watch the movie. Choice (1) there will be 5 identical pieces. Hence, 4 the corners will have two faces painted.
cuts parallel to each of the three planes Along each edge, if we remove the cor-
29. In this case, as all the conditions are sat- ner cubes, there is one cube that has
isfied. Hence, the group will watch the means there will be 5 x 5 x 5 identi-
cal pieces that is, 125 identical pieces. two faces painted. Hence for 12 edges
movie. Choice (1) of the cube, there will be 12 cubes
 Choice (4)
30. In this case, condition (b) is violated as which have only two faces painted.
some students have seen the movie earli- 6. Three cuts parallel to one face, two cuts  Choice (3)
er. As the number of these students is less in a perpendicular plane and one cut in
than half of the total number of students the third perpendicular plane. Hence, six Solutions for questions 12 to 16:
in the group, the alternate condition (f) is cuts. Choice (3) 216 cubes ⇒ 6 smaller cubes on each edge.
fulfilled. Hence, all of them go to watch
7. We need to find three factors the product 12. No red paint at all ⇒ remove the 2 op-
the movie.  Choice (1)
of which is 120 - 6 x 5 x 4. So, to get 6 posite faces each having 6 x 6 cubes, 216
pieces parallel to one plane, we need to - 72 = 144. Choice (2)
have 5 cuts; hence, the number of cuts to
Cubes and Dice get 6 x 5 x 4 pieces, we need 5, 4 and
13. Cubes having at least two different colours
are all those along the edges which is 4 x
3 cuts a total of 12 cuts along the three
6 + 8 x 4 = 56 (where 4 vertical edges
Practice Exercise parallel planes to get 120 identical pieces.
each have 6 cubes and the other 8 edges
 Choice (4)
Solutions for questions 1 to 8: have 4 cubes each). Choice (3)
8. If X has least possible dimensions and the
1. The 3 cuts can be at the middle of the cube 14. If one layer of the outer cubes is removed
small cubes (27 or 64) have integers on
in 3 perpendicular planes. First cut will on all sides, the remaining 4 x 4 x 4 (that
edges, the dimensions of X will be 12 x 12
make the cube into 2 parts. The second cut is, 64) cubes will have no face painted at
x 12 (because then when we cut it into 27,
along a perpendicular plane at the cen- all. Choice (1)
that is, 3 x 3 x 3 cubes, the edges will be 4
tre will double the number of parts to 4.
cm and when we cut it into 64, that is, 4 x 15. The common edges are four and on each
The third cut along the third perpendicu-
4 x 4 cubes, the edges will be 3 cms). If X, edge there are 4 cubes with only red and
lar plane will double 4 parts to 8 parts.
is now cut into the largest number of cubes, green.
 Choice (2)
that is, smallest dimensions, that is, 1 cm, (The eight corner cubes have blue also
2. The cuts can be two along one plane we get 12 x 12 x 12 that is, 1728 cubes. and hence not counted here). So, 4 x 4 =
(giving 3 parts) third cut along a  Choice (3) 16. Choice (4)

08_Section.indb 149 10/30/2009 11:54:10 AM


150  n  Solution Manual

16. On the face where the particular colour is 26. From the figures it can be observed that 3. As the author is making a general state-
used, if we remove the outer cubes, the the four faces adjacent to the face marked ment, we are not sure whether Ι or ΙΙ is
4 x 4 square inside will have only one with “d” are marked with “b”, “f”, “e” his assumption or not.
colour. So, 16 cubes in each face. We and “a” respectively. Ι is not implicit, as the speaker says, “If
have 2 faces for green and 2 faces for blue ⇒ “c’ cannot be on the adjacent face of suppression is there then social justice
and hence 4 x 16 = 64. Choice (1) “d”. will not be realized”. That does not mean
Choice (4) is wrong. Choice (4) that he assumes that suppression is there.
Solutions for questions 17 to 20: ΙΙ is also not implicit, here also we are not
27. By observing the given figures it can be
17. No red paint means the two adjacent faces sure whether he thinks that social justice
found out that the pairs - (α, φ), (β, θ) and
having is realized or not.
(π, r) are on the opposite faces. In choice
red have to be removed, that is, 36 + 30  Choice (3)
(3) α and φ are on adjacent faces. Hence,
= 66 cubes. 4. Ι is implicit. The assumption that 15 to
this view is wrong. Choice (3)
Hence, 216 - 66 = 150. Choice (2) 20 days of preparation is sufficient for
Solutions for questions 28 to 30: college studies certainly bridges the gap
18. At least two different colours will mean
out of the 56 cubes along all the edges 28. The cube can be drawn as between the two sentences made by the
together, we remove 4 cubes (excluding author.
the corner cubes) for each colour, which 2 4 ΙΙ is not implicit, as the assumption is not
 complete. We can only observe how the
have same colour on two faces that is, 12
5 3 speaker is comparing the CAT exam with
cubes. Hence, 56 - 12 = 44. Choice (4)  1
college exams but not with any other.
19. One face red ⇒ out of 36 + 30 = 66 cubes  Choice (1)
(on both the red faces together), we need 
to remove 6 common cubes which have 6 Solutions for questions 5 to 8:
two faces painted red. Hence, 66 – 6 = 60. Only choice (2) is not possible with this cube. 5. This is not related to the problem as it talks
 Choice (3)  Choice (2) about prosecution of intruders where the
20. There are 3 common edges giving 6 + 6 + 29. The numbers on faces of the cube is as problem is about the laws. Hence, Ι does
4 cubes which have green and red that is, given below not follow
16. Choice (3) This is a feasible course of action which
would prevent intrusion. Hence, ΙΙ fol-
Solutions for question 21: 3
lows.
1
21. 125 smaller cubes will form a 5 x 5 x 5 6  ∴ Only ΙΙ follows. Choice (2)
cube. To cover it we require a 7 x 7 x 7 4 2
 6. A firm or an organisation runs any business
cube because one additional layer of at minimum expenses whether it is making
cubes is required on all faces. Hence ad- profit or a loss. As the present level of ex-
5
ditional cubes required = 73 − 53 = 218 penses is not known, Ι does not follow.
 Choice (3) Choice (4) is violating the views of the cube. Creating on alternate domestic market is
 Choice (4) a solution to beat the exchange rate risk.
Solutions for questions 22 and 23:
30. Option (2) does not belong to the given Hence, ΙΙ follows.
22. A cannot be opposite D, B, F or E as they ∴ Only ΙΙ follows. Choice (2)
cube. Since M and X are opposite each
are adjacent to A. Hence, A must be op-
other. Choice (2) 7. As the problem is the students who are
posite C. Choice (2)
under stress the solution to the above
23. As explained above, it is better to look problem is to find out, how to keep the
at the adjacent faces. Now Yellow is ad- Practice tests students away from stress rather than ban-
jacent to White, Red, Green and Pink, ning the violent scenes on screen. Hence,
so Yellow cannot be adjacent to these Ι does not follow.
colours. Hence, Yellow should be adja- TEST PAPER 1 ΙΙ increases the stress on children. It is in-
cent to Blue colour. Choice (3) appropriate. Hence, ΙΙ does not follows.
∴ Neither Ι nor ΙΙ follows. Choice (3)
Solutions for questions 24 to 27: Solutions for questions 1 to 4:
8. In the given statement it is mentioned
24. From the figures, it can be observed that 1. Demographic profile is skewed towards
that the chefs are inadequate but it is not
the four faces adjacent to the face marked senior citizens, means that there are more
mentioned whether they are not available
with “1” are marked with “2”, “3”, “4” numbers of senior citizens. Hence, Ι is not
globally or they are not available in that
and “5” respectively. implicit.
area. If they are not available globally, the
⇒ “6” cannot be on the adjacent face of ΙΙ is also not implicit, as “we” in the state-
question of recruiting chefs from other
“1”.Choice (3) is wrong. Choice (3) ment need not be India. Choice (3)
places does not arise. Hence, Ι does not
25. From the figures, it can be observed that 2. Ι is implicit, as the author assumes that follow.
the four faces adjacent to the face marked education had some meaning. Only then Because the problem is with the inadequa-
with “t” are marked with “s”, “p”, “q” and he can say that its meaning is lost. cy of chefs shifting to other business lead
“u” respectively. ΙΙ is also implicit, as elections can be to death of one type of industry. Hence, it
“r” cannot be on the adjacent face of “t”. banned only if they are conducted. It is is a negative course of action.
Choice (4) is wrong. Choice (4) also a valid assumption. Choice (4) ∴ Neither Ι nor ΙΙ follows. Choice (3)

08_Section.indb 150 10/30/2009 11:54:10 AM


alone, as D
not wear th
? ,, alone

Solutions for Intelligence and Critical Reasoning  n  151 21. From stat
From the above diagram: persons.
alone, as D wears
Conclusion ,, affirmative, does not follow. The
alone,
not wear
perso
as Ds
the Blue
Solutions for questions 9 to 12: Solutions for questions 16 to 19: Conclusion
Conclusion ΙV, affirmative,
,,, affirmative, does not follows.
follow. among
not
? ,, alone wear the
the
is suffic
As the ,,,,
Conclusion negative
negative,conclusions
follows. follow, let 21. From
? statei
,, alone
9. Argument Ι neither brings out necessity 16. From the given statements: From Estatement
 2nd tal
Conclusion
us try affirmative,
to,V,negate them follows.
using the alternate persons.
for reduction of airfare nor it talks on eco- From the above diagram: 21. From
C > 2 state
pers
AsConclusion
the negative conclusions follow,
follow.let us try to The person who
nomic lines. Hence, Ι is not strong. MUVs SUVs From diagram.
negatethethem ,, affirmative,
above diagram:
using
does not
the alternate
persons.
amongThe person
Conclusion ,,, affirmative, does notdiagram.
follow. The
the other
four person
person
th
CVs Conclusion ,, affirmative, does not follow. From statement ,,
Argument ΙΙ brings out the negative re- Conclusion ,,,, negative, follows.
Conclusion
Imaginations among
Bytallestthe
combin
Conclusion ,V, affirmative, follows. follow.
,,, affirmative, does not E  2nd
sult of the suggested action. Hence, ΙΙ is Conclusion ,,,, negative,
As the negative follows.
conclusions follow, let us try to C > 2From
D, C,statem
persons E, F
Conclusion affirmative,
,V,using follows. Etallest.
The person 2nd tallis
who
strong. Choice (2) negate them the alternate diagram.
Ideas four persons.
C > 2 perso
As the negative conclusions follow, let us try to
Imaginations
By combining both
22. From
The state
person
10. Argument Ι supports the suggestion made negate them using the alternate diagram. D, C, four
E, F person
and Ac
‘tuts’, ‘hoy’
Imaginations
Dreams tallest.
in the statement. Statement proposes Ideas From
By stat
combini
HGVs 22. From D, cannot
C, E, beF, aa
strengthening rural economy with a pur- statement
‘tuts’, tallest.
?Byorcomb
‘hoy’ ‘sop’
pose of checking influx of people from From the above diagram, Images
Ideas
Dreams From statement
rural area to urban area. Argument Ι brings Conclusion Ι, negative, follows. 22. From
cannot statem
be “maru” “
out the bad effects of influx people from Conclusion ΙΙ, affirmative, does not fol- Images 23. ‘tuts’,
?By From‘hoy’
combing state
bot
From the above diagram,
low. From the above diagram, Dreams + left +state
From left
rural area to urban area. Hence, Ι is strong. ,,, is negated but ,, which did not follow earlier ?Initially be ,h“
Conclusion ΙΙΙ, negative, follows. ΙΙΙ isthe
follows negated
diagram butnow. Ι, which did not follow 23. Fromcannot statement
Argument ΙΙ assumes that strengthening From the above diagram, + left ?By+?Statemen
left) comb
the pe
of economy as industrialisation, which Conclusion ΙV, affirmative, follows. ,,,earlier
Hence, ,Vfollows
is negatedand but ,,the
either , diagram
whichor ,,, notnow.
Images
didfollow. follow earlier(4)
Choice From state
ΙV
?Initially he is faci
Hence,
follows the diagram and now.either Ι or ΙΙΙ follow.
is not true. Hence, ΙΙ is not strong. As the negative conclusions follow, let us ?Statement about ,the alond
Hence,
19. From  the ,V and either
the given statements: , or ,,, follow. Choice (4)
Choice (4) 23.
From From
statement
North. ,, statem
 Choice (1) negate them using the alternate diagram. From above diagram, about+the leftdirection
+ left)
isFrom
,,, From negated Integrals
but which
,, statements: Inverse
did not follow earlier
Integers
Differentials
19.
19. thethe
given statements:
given Solutions
North. ?Initially forhe
11. Argument Ι is not strong because it is a HGVs follows the diagram now.
MUVs Differentials Integrals Integers Inverse The?Statemen
Solutions uncommo
for questio
mere assertion that night clubs are sign of Hence, ,V and either , or ,,, follow. Choice (4) From statem
respectively.
CVs The uncommon digits
development. Being sign of development respectively. about the du
Similarly
19. From the given statements: andthe
Similarly North.
r. uncomm
does not support continuation of night SUVs and r.? The codes fo
Differentials Integrals Integers Inverse Solutions 5for q
clubs. There are many things which cause ? The codes
Eliminate for the and d
From the above diagram: Eliminate the digits 4 a
sleeplessness. This cannot be a reason to From the above diagram:
From the,, above
Conclusion negative, diagram:
follows.
The
terms.
terms. uncommon
ban night clubs. Hence, ΙΙ is not strong. Ι ΙΙ cannot be negated and Ι is negated but Conclusion ,, negative, follows. respectively.
Now 2Now 2 is comm
Conclusion
Conclusion
Conclusion Ι, negative,
,,, negative,
,,, negative, follows.
follows.
follows. is common in ‘
ΙΙ which did not follow earlier follows codeSimilarly
code
‘t’ ‘t’ isthe
is common. comm un
 Choice (3) Conclusion
Conclusion
Conclusion ,,,, affirmative,
ΙΙ, negative,
,,,, affirmative, does not
doesfollows. follow.
not follow. and
So r.
So the code for 2 isf
the code
now. Hence, ΙΙΙ, ΙV and either Ι or ΙΙ fol- Conclusion ,V, negative,
Conclusion ,V, negative, follows. follows.
ConclusionnegativeΙΙΙ, affirmative, does usnot fol- code? code
in The incodes
all terms. all term fo
12. The director will have many means of con- lows. Choice (3) AsAsthe thenegative conclusions
conclusionsfollow, let let
follow, try
us totry to ThereEliminate
are twoare 9’s
thein di
96
From low.
negate thethemabove
them diagram:
using the There two
negate using thealternate
alternate diagram.
diagram. x. Then
trolling various departments under him. Conclusion ,, negative, follows. x. the
terms. Then code theforcod6
17. From the given statements: Integrals
Integrals ΙV, negative,
Conclusion Inverse
Inverse follows. Similarly,
Now we
Similarly, can
2 is comm calcu
we ca
But argument Ι states that the suggestion Conclusion ,,, negative, follows.
As the negative conclusions follow, let code ‘t’ 1is comm
Digit 2 3
made in the question leads to better con- Conclusion ,,,, affirmative, does not follow. Digit
Hurrican us try to So the vcode t 1 foy
,V,negate them using the alternate
Integers Codes
Typhoons Tsunami Conclusion negative, follows.
trol, which is a strong argument. Integers codeCodes in all term v
Cyclone As the diagram.
negative conclusions follow, let us try to 24. ‘s’ is the code for 6
There are two 9
Argument ΙΙ is against such visits, be- negate them using the alternate diagram. 24. ‘s’ is the co
x. isThen
25. ‘v’ the codethe cod
for 1
cause the director’s time is valuable, Integrals Inverse Similarly,
26. ‘y’25. is we
‘v’ code theforca
co
which is also a strong argument. Hence, Differentials is the 3

both Ι and ΙΙ are strong. Choice (4) above diagram, ,, and ,V can be
From the Differentials 26.Digit
27. wyvzy ‘y’ is the
is the
1co
code
negated. Integers Codes v
From the above diagram: From the ,,above
But when diagram,
is negated and ,V
then ,,,,,follows. can, be Solutions
Hence, 27. wyvzy is th
for questio
Solutions for questions 13 to 15: negated.
and either ,, or ,,, follows. Choice (3) 24. path
28. The ‘s’ istravelled
the co
Conclusion Ι, affirmative, follows. But when ,, is negated then ,,, follows. Hence, , Solutions for
Let each girl be denoted by the first letter of her 25. ‘v’ is the co
Conclusion ΙΙ, negative, does not follow. Solutions for questions 20 to 23:
and either ,, or ,,, follows. Choice (3) 28. The path tr
name.
Conclusion ΙΙΙ, affirmative, does not fol- 20. From statement , alone, the person who wears the
Differentials 26. ‘y’ is the co
From (i) and (ii), A and B are sitting in the mid- Solutions
Blue for
shirtquestions
can be A, B 20ortoC.23:
From statement ,,
dle column.
low. FromFrom
the the diagram,,,ΙΙ and
abovediagram,
above and ,V can ΙV
can bebe 27. wyvzy is the
Conclusion ΙV, affirmative, does not fol- 20. negated.
From statement , alone, the person who wears the
negated.
From (iii), C is at the extreme left of a row. Bluewhen
shirt,,can be A, Bthen or C.
low. But is negated ,,, From statement
follows. Hence, ,,, Solutions for q2
Form (iv) and above results, D is the first row  ut when ΙΙ is negated then ΙΙΙ follows.
B
Hence, only Ι follows. Choice (1) and either ,, or ,,, follows. Choice (3) 28. The path tra
x
and E is in the second row and both of them are Hence, Ι and either ΙΙ or ΙΙΙ follows.
18. From the given statements: Solutions
 for questions 20 to 23: Choice (3)
the extreme right of the corresponding rows.
From (v) and above results, F must be in the first 20. From statement , alone, the person who wears the
Ideas Imaginations Solutions for questions 20 to 23:
Blue shirt can be A, B or C. From statement ,,
row and at the extreme left.
∴ Chameli is at the extreme left of the second 20. From statement Ι alone, the person who
row. wears the Blue shirt can be A, B or C.
∴ The final arrangement will be as follows: From statement ΙΙ alone, as D wears the
F A D Dreams Black shirt and A or C does not wear the
C B E Blue shirt, B wears the Blue shirt.
Images ∴ ΙΙ alone is sufficient. Choice (2)
13. Farheen is at the extreme left of the first row.
From the above diagram: 21. From statement Ι alone, B > D and C > 2
 Choice (1)
Conclusion Ι, affirmative, does not fol- persons.
14. Bindu is at the middle of the second row. The person who is the tallest can be B or
low.
 Choice (2) one among the other three persons.
Conclusion ΙΙ, affirmative, does not fol-
15. Devi is to the immediate right of A. low. From statement ΙΙ alone,
 Choice (2) Conclusion ΙΙΙ, negative, follows. E − 2nd tallest

08_Section.indb 151 10/30/2009 11:54:11 AM


152  n  Solution Manual

C > 2 persons 29. My only sibling’s daughter is my niece Solutions for questions 36 to 40:
The person who is the tallest is among the and her uncle may be myself or my sib- 36. AMAZON is coded as BODYMK.
other four persons. ling’s brother-in-law. Therefore, my only
By combining both the statements, as nei- sibling’s daughter’s uncle’s wife is either Here, the pattern is
A M A Z O N
alone,ther ofwears
as D D, C,the
E, F and shirt
Black A can beAthe
and or tallest,
C does my son’s mother or not related to my son.
not wearB must be the
the Blue tallest.
shirt, B wears the BlueChoice
shirt. (4)  Choice (4) +1 +2 +3 –1 –2 –3
? ,, alone is sufficient. Choice (2) B O D Y M K
22. From statement Ι alone, the code of ‘guns’ 30. 1. (23 × 32 – 108 + 200 ÷ 10) × 6
21. From can statement alone,orB‘sop’.
be ‘tuts’,, ‘hoy’ > D and C > 2 = (8 × 9 – 108 + 20) × 6 = (72 – 108 Similarly, for COURSE
persons.
F
 rom statement ΙΙ alone, the code of ‘guns’
The person who is the tallest can be B or one
+ 20) × 6 = –16 × 6 = –96 C O U R S E
ollow.
among cannot be “maru”
the other “sop” “tue” and “hoy”.
three persons. 2. 43 ÷ 8 + 108 – 16 × 15 ÷ 3 – 132 +1 +2 +3 –1 –2 –3
follow.
From ∴By combing
statement both, the code of ‘guns’ is
,, alone, 64 16 ×15 D Q X Q Q B
E  2nd tallest
‘tuts’. Choice (4) = + 108 – – 132 = 8 +
C > 2 persons 8 3 COURSE is coded as DQXQQB. Choice
w, let us try to
gram. The
23. person who is the Ιtallest
From statement alone, after turning
is among to
the other 108 – 80 – 132 = –96 (1)
four persons.
his (right + left + left) the person is facing 3. (42 + 24) × 3 – 96 – 102 + 102 ÷ 17
ions
By combining both the statements, as neither of 37. Letter / digit which is 20th from the left
D, C, towards
E, F and West.
A can be the tallest, B must be the = (16 + 16) × 3 – 96 – 102 +6 = 96 –
is E, 8th letter / digit to the left of E is 1.
tallest.∴Initially he is facing towardsChoice North.(4) 96 – 102 + 6 = –96
eas  Choice (4)
∴Statement Ι alone is sufficient. 4. (354 – 58 + 104) ÷ 4 – 4 – 64 × 3 + 1
22. From statement , alone, the code of ‘guns’ can be
‘tuts’, F‘hoy’
romorstatement
‘sop’. ΙΙ alone, we cannot say 38. My mother’s husband’s brother’s sister’s
ams 400
From anything
statementabout the direction
,, alone, he isoffacing.
the code ‘guns’ = − 4 – 64 × 3 + 1 = 100 – 4 – 192 + 1 mother is my paternal grand mother and
cannot It be
can“maru”
be South
“sop”or“tue”
North.
and “hoy”.Choice (1) 4 my mother’s mother-in-law, whose only
es ?By combing both, the code of ‘guns’ is ‘tuts’. daughter-in-law is my mother itself and
Solutions for questions 24 to 27: Choice (4) = –96 + 1 = –95 Choice (4)
her brother is my maternal uncle. Choice
23.The
Fromuncommon
statement digits
, alone,inafter
6857 and 4786
turning to his are 5
(right 31. The relation given here is a place and the (3)
and 4 respectively.
+ left + left) the person is facing towards West.
t follow earlier property of the place.
Similarly
?Initially the
he isuncommon
facing towardsletters
North. in their codes 39. In the evening the shadow fall towards
?Statement Mount Etna is a volcano and so on.
are z and r. , alone is sufficient. east. Here, as Mona’s shadow falls to her
Choice (4) From statement ,, alone, we cannot say anything But England is a place in Europe.Choice
∴about
The codes for 5 and
the direction he 4isare z andIt rcan
facing. respectively.
be South or
left, Mona’s left is towards east that is
Eliminate
North. the digits 4 and 5 and their Choice codes in(1)all
(1) she is facing south. Hence, Sona is facing
Inverse the terms. north. Choice (2)
Solutions for questions 24 to 27:
Now 2 is common in ‘2341’, ‘2376’, ‘9629’ and Solutions for questions 32 to 35:
The uncommon digits in 6857 and 4786 are 5 and 4 40. The given expression = 2 – 15 ÷ 5 + 3 –
in the code ‘t’ is common.
respectively. 15 × 5
So the the
Similarly code for 2 is t.letters
uncommon Eliminate thecodes
in their digit 2are
andz Bandra 12 km Ο
and The actual expression = 2 + 15 ÷ 5 × 3
itsr.code in all terms.
? The codes for 59’s
and + 15 – 5
There are two in49629
are zand
andhence
r respectively.
the code for
Eliminate the digits 4 and 5 and their codes in all the 15 km = 2 + 3 × 3 + 15 – 5 = 2 + 9 + 15 – 5 = 21
9
terms.is x. Then the code for 6 is ‘s’.
NowSimilarly,
2 is common we can calculate
in ‘2341’, the codes
‘2376’, ‘9629’ for
andall
in the
20 km  Choice (2)
code ‘t’ is common.
terms.
follow.
So the code for 2 is t. Eliminate the digit 2 and its
w, let us try to
code in all terms.
Digit 1 2 3 4 5 6 7 8 9
12 10 km House TEST PAPER 2
There are two 9’s in 9629 and hence the code for 9 is 5 Palam
gram.
x. Then
Codesthe code
v fort 6 isy‘s’. r z S W u x
Similarly, we can calculate the codes for all the terms. Solutions for questions 1 to 5:
Digit
24. ‘s’ is the code for 6. Choice (2) 32. Dadar from Palam is 13 km away
1 2 3 4 5 6 7 8 9 1. The given statements can be represented
gers Codes
25. ‘v’ vis thet code
y for
r 1.z S W Choice
u x (2) in the following basic diagram.
122 + 52 = 13 km Choice (1)
24. ‘s’
26. is the
‘y’ code
is thefor 6. for 3.
code Choice
Choice(2)(4) Jeeps
33. The direct distance from Palam to Dadar is
25. ‘v’ is the code for 1. Cars
27. wyvzy is the code for 73153. Choice
Choice(2)(1) 13 km and then Dadar to Bandra is 20 km.
26. ‘y’ is the code for 3. Choice (4) So total distance is 33 km. Choice (3)
Solutions for questions 28 to 31:
d ,V can be 27. wyvzy is the code for 73153. Choice (1) 34. The direct shortest route from Bandra to
28. The path travelled by Ashish is as fol-
ows. Hence, , Solutionslows
for questions 28 to 31: Palam is 19.20 km that is, 122 + 52 =
Choice (3) 28. The path travelled by Ashish is as follows Aeroplanes

30 m 19.20 and Palam to his house is 10 km, From the above diagram,
x
so the total distance from Bandra to his Conclusion Ι, affirmative, does not fol-
who wears the house via Palam is 29 km, approximately.
20 m low.
m statement ,,
40 m  Choice (1) Conclusion ΙΙ, affirmative, does not fol-
35. The shortest distance from his house to low.
20 m Sol/957 Conclusion ΙΙΙ, affirmative follows.
x Bandra is 26.62 that is, 222 + 152 = ∴ Only ΙΙΙ follows. Choice (4)
 is house is towards south–west with re-
H 26.62 and Bandra to Dadar is 20 km. So 2. The given statements can be represented
spect to his friends house. Choice (2) the total distance is 46.6 km. Choice (4) in the following basic diagram.

08_Section.indb 152 10/30/2009 11:54:12 AM


? Either , or ,, follows and ,,, follows.
alternate diagram.
AC Cars 6. Ravi has no sister but has brother.
Choice (4)
No sister m Ravi o Brother
Solutions for questions 6 to 9:
AC Cars 6. RaviWifemRavi
has no sister Brother
o but o Wife o Brother o Brother
has brother.
No sisterSisters
m Ravi o Brother Choice (3)
Motors
7. o Brother o Wife o 500
Solutions, for
In the above diagram conclusion
WifemRavi
Intelligence
does not and Critical Reasoning   o
Brother n Brother
153
Tea = 200 Lassi = 100
follows. Sisters Choice (3)
Motors
? Only ,, follows. Choice (1)
Goods Perishable 4. The given statements can be represented 7.7. 500
In the
4. above
The diagram
given conclusion
statements can ,
be
in the following basic diagram. does not in the
represented O Lassi = 100
Tea = 200
Costly follows. following basic diagram.
? Only ,, follows. Choice (1)
Hero
4. The given statements can be represented in the O
following basic diagram. Butter Milk = 150
N=?
From the above diagram, Hero
Indian
Conclusion Ι, affirmative, does not fol-
Butter Milk = 150
lows. N=?
Conclusion ΙΙ, negative , follows. Indian Sol/959
 he people who do not drink anything
T
Conclusion ΙΙΙ, affirmative, does not fol- People cannot be decided because the people
low. drinking exactly one and exactlySol/959
two is
From the above diagram, The people who do not drink anything cannot be (3) If the digit 4
Conclusion Ι, affirmative, does not fol- not given.
decided because the people drinkingChoice
exactly(4)
one the numbe
Goods, follows.
Conclusion ,,, negative Perishable People The people who do not drink anything cannot be This (3) If t
lows. and exactly two is not given. Choice (4) means
Conclusion ,,,, affirmative, does not follow. 8. decided because the people drinking exactly one they shoul the
Negative ,,,
?Conclusion conclusion,
negative ,,,, follows. To prove it
follows. Conclusion ΙΙ, affirmative, follows. 8. G R and A P exactly
E Stwo O is notY given.
P E Q Choice (4) ExcludingTht
false we have,,,,
Conclusion toaffirmative,
prove “some Goods
does are costly”,
not follow. Conclusion ΙΙΙ, negative, does not fol- should be the
through an alternate diagram. From the above diagram, +2 8.-2 + 2 -2 G +2R-2 A P -2E+ 2S2 +2 O -2Y P E Q Ex
? Negative conclusion, ,,, follows. To prove it lows. ,, affirmative, does not follows. mentioned
Conclusion sh
Goods
false we have to prove “some Goods are costly”,
Perishable +2 -2 + 2 -2 +2 -2 -2 + 2 2 +2 -2
through an alternate diagram. ∴ Only
Conclusion
From ΙΙ affirmative,
,,,
the above follows.
diagram,follows. Choice (2) I P C N G Q M A N G O 14. The given me n
Conclusion,,,,
Conclusion ,, affirmative,
negative, does does notnot follows.
follows. Choice (3) number, so
Goods Perishable 5. The ,,given  M Choice
A N (3) 14. Th
Only
?Conclusion ,,,questions
follows. affirmative, can be represented
follows. Choice (2) in I P C N G Q G O per the give
Choice (3) Hence, 567 nu
5. The the following
given
Conclusion questions basic
,,,, negative, can diagram. not follows. in the 9. 9.
be represented
does 47
pe
following basic
? Only ,, follows. diagram. Choice (2) 15. The given
Costly 9. 47 He
5. The given questions can be represented in the extremes a
Conclusion ,,, negative , follows. 23 + 24 15. Thc
the letter
∴ Negative
Conclusion conclusion,
,,,, affirmative, doesΙΙ,
notfollows.
follow. To following basic diagram. ex
IHGFED.
?prove it false
Negative we have
conclusion, to prove
,,, follows. To “some
prove it Human being 23 + 24 the
false
Goods we have to prove “some
are costly”, throughGoodsan are costly”,
alternate 12 + 11 + 13 16. The given IH
through an alternate From the above diagram,
diagram. Costly diagram. Human being
Conclusion ,, affirmative, does not follows. 12 + 11 + 13
odd numbe
16. Th
Goods Perishable the extreme
From the From
abovethediagram,
above diagram,
,, does not ΙΙ follow,
does not butfol-
, Conclusion ,,, affirmative, Manfollows. 8 + 4 + 7 + 6 coded asod ‘X
follows. low,
Conclusions Costly
, and ,,Conclusions
are contradictory Mad the
but Ι follows. Ι andto ΙΙ Conclusion ,,,, negative, does not follows.
From the above diagram, So, 23 is the missing number. Choice (2) 17. The given
each
From other.
the above diagram, ,, does not follow, but , ? Only ,, follows. Choice (2) 8 + 4 + 7 + 6 even numb co
Eitherare orcontradictory to ,,
each other. From the affirmative,
above diagram, Man not follows.
?follows. ,, follows. , and
,Conclusions Choice (3) to
are contradictory 5. Conclusion
The given ,,questions candoes be representedMad in the Solutions forSo, 23 is the 10
questions missing 13:number.
tonumber. Choice (2) extremes 17. Th s
∴ Either Ι or ΙΙ follows.
each other. Choice (3) Conclusion
Conclusion
From the ,,,
above
following basic diagram. Ι,
negative,affirmative,
diagram, follows. does not fol- So, 23 is the missing Choice (2) ev
3. The given statements can be represented in the Conclusion
Conclusion ,,,,
,, negative,
affirmative, follows.
does not follows. ex
? Either , or ,, follows.
3. The Choice (3)
given statements can be represented lows. Solutions for questions 10 to 13:
following diagram. Conclusion
Conclusion,,,,,and negative. Let us try to Solutions for questions 10 to 13:
,,, arefollows.
negative, Solutions for q
in thestatements
3. The given followingcan
Cars diagram.
be represented in the draw Conclusion
Conclusion an alternate,,,, ΙΙ, negative,
diagram
negative, follows.
to disprove them. P’sP’s
follows. sister o K Female – Advocate
sister  →  K  Female  –  Advocate oror
following diagram. Conclusion
Negation ΙΙΙ,
of ,,,,isand negative,
“Some human follows.
beings Clerk Given Soluti
that non
Human
Conclusion being ,,, are negative. Letare usmad”try to Clerk P’s o sisterL Male o K –Female – Advocate or
and K’s husband Doctor race. Also Niru
Cars
Conclusion ,,, negative , follows. AC draw Cofonclusion
anis alternate
,,, “Some mad
ΙΙ and persons
diagram
ΙΙΙ toaredisprove
are negative. men”. Let us them.
K’s Clerk →  L  Male  –  Doctor
husband 
N’s husband o M Male – Doctor Given
? Mani won the
Costly But ,,, cannot
Negation of ,, beisdisproved.
“Some human beings are mad” K’s husbando L Male – Doctor race. A
Conclusion ,,,, affirmative, does not follow. The tryofto,,, draw
andalternate
an alternate
diagram
is “Some madtopersons provediagram“Some
are
to human
men”.
dis- M’sN’s wife husband  o →  N Female
M  o – – 
Male  Clerk
Doctor– Doctor or As Niru cannot
From the
? Negative above diagram,
conclusion, ,, does To
,,, follows. notAC
follow,
prove but it , N’s husband M Male ? Man
beings
But prove
,,,are them.
mad”be
cannot Negation
is as Man
follows.
disproved. of ΙΙ is “Some Mad hu- Advocate →  ON  Male
and white.
false wefollows.
have Conclusions
to prove “some , andGoods
,, are contradictory
are costly”, to From
The man the beings
above
alternate are mad”
diagram,
diagram toand of
prove ΙΙΙ is
“Some “Some human M’sM’s
fatherwife 
M’so wife oFemale  – Retired
N –Female Clerk
– or Ad-
Clerk
_____ or
Niru used As Nir
yellow
througheach other.
an alternate diagram. From the above diagram, vocate Advocate and wh
Conclusion
beings are ,, affirmative,
mad” is as does
follows. not follows. Unmarried o P Male – Doctor Given that the
? Either
Goods , or ,, follows. Choice (3) mad persons
Conclusion are men”.
,, affirmative, does not follows. M’s father o O Male – Retired _____ Niru us
bikes finished
Perishable Conclusion
Conclusion ,,, negative, follows.
,,, affirmative, follows. M’s
The order wise father 
Unmarried
arrangement →  o can O 
P be Male 
Male –  Retired
– Doctor
as follows. Given
3. The given statements can be represented in the Human ΙΙΙ cannot
But beings be disproved. placed peson
Motors Conclusion
Conclusion ,,,, negative,
,,,, negative, follows.
does not follows. M_____
! P ! L ! K and O ! K So ‘O’ can be anywhere. green). bikes
following diagram. T
 he
Conclusion alternate
,, and diagram
,,, are to prove
negative. “Some
Let us tryhu-to The order wise arrangement can be as follows. placed
Only ,,beings
?Human follows. Choice (2) Condition 1, no man can
P  beO either– an advocate or a
From the above diagram, draw man an alternate
beings are mad”diagram
can is beastorepresented
disprove them.
follows.
Unmarried 
in the clerk thatMmeans !P!M L !→  K and Male 
! K be Doctor
‘O’ can be anywhere.As Sri used
Soadvocate. wh
Conclusion ,, Motors
negative,Carsfollows. 5. The given questions
The order wise
Condition
and
1, no
O cannot
arrangement man can
can bebe as follows.
either an advocate and
or a Manigreen)
used
Negation of
following basic diagram. ,, is “Some human beings are mad” From the condition 2, O is a widower and he is the As not
Sri
Conclusion
From the ,,, the
above affirmative,
diagram, follows. clerk As Viru did
From above diagram, AC and of ,,, is “Some mad persons Man are men”. Mad M > of
father P >M, >that
Lwho means
Kisand O >MKandSo
his son. O cannot
‘O’ canbebeadvocate.
any- and M
Conclusion
Conclusion,,,, affirmative,
,, negative, does not follows.
follows. From the 3 ocondition 2, O andis a her
widower andishe is finished
the fifth.
Conclusion Ι, negative, follows. InButthe,,,abovecannot be disproved.
diagram ,, does not follow but , From where.condition N is a female husband (1) ManiAs Vir Blu
Negative
Conclusionconclusion , follows.
,,, affirmative, To prove it false
follows. Man Mad father of M, who is his son.
Conclusion ΙΙ, affirmative, follows. The alternate
follows. , and ,, diagram
are contrary to proveto each “Someother.human a doctor but he earns more points than P who earns (3)is Raj finishe Gre
we have
Conclusionto ,,,,
show that
affirmative,“All does
cars not
are AC” In an
follows. Human being Condition From 1, no man
condition can3 o be N either
is a an
femaleadvocate
and heror husband
alternateConclusion
diagram. ΙΙΙ,, affirmative, does not fol-
?Inbeings
Eitherthe are , ormad”
above is as follows.
,, follows
diagram and,,,,,does follows.not follow but ,more than L, means N’s husband cannot be L or P.
a doctor but he earns more points than P who (5) Viru (1) Re
earns
Ma
Negative conclusion follows. To prove it false follows. , and ,, are contrary to eachChoice other. (4) From a clerk that means
condition 4 oM K’sand O cannot
husband is Lbeandadvocate.
K is the (3) Ra
we havelows.to show that “All cars are AC” In an
Costly sister more than
ofcondition
P. L, means N’s husband cannot be L or P.18. Mani wonVir
(5) th
? Either , or ,, follows and ,,, follows. From the 2, O is a widower and he is
alternate diagram. Solutions for questions 6 to 9: From condition 4 o K’s husband is L and K is the
From Negative conclusion Ι follows. To prove
the above diagram, ,, does not follow, but , Choice (4) 10. K or N is an advocate. We cannot determine who
the father of M, who
Human beings Man Mad sister of P. is his son. 19. Raj used 18. Ma gr
follows.
AC itConclusions
false weMotors , and
have ,, are that
to show contradictory
“All carstoare
Cars 6. Ravi has no sister but has brother. exactly out of these two is the advocate.
each other. Solutions
From thefor questions
above diagram, 6 to 9: From condition
10. K or N 3 is→anN advocate.
is a female Weand her hus-
cannot determine who
AC”the In above
an alternate
diagram,diagram. No sister
In the above m Ravi diagram o ΙΙBrother Choice (4) 20. Viru who 19. Ra is
? Either
From
, or ,, follows. Choice (3) Conclusion ,, affirmative, does does not follow
not follows. band is a doctor exactlybut out heof these
earns twomore is the advocate.
points than
ACConclusion ,, negative, follows. Cars 6. Ravi has no sister but has brother. bike.
but
Conclusion Ι follows. Ι
,,, negative, and ΙΙ are
follows. contrary to each 11. M is the husband of N. Choice (3) Choice (4) 20. Vir
3. The givenConclusion ,,, affirmative,
statements can be follows.
represented in the WifemRavi
No sister o Brother
m Ravi o Wife o Brother
Brother o Brother P who earns more than L, means N’s husband
other. ,,,, negative,Man
Conclusion follows. Mad
12. K gets 11.theMlowest
is the husband
points. of N. Choice (1) Choice (3) 21. Niru usedbik y
following diagram.,,,, affirmative, does not follows.
Conclusion Conclusion
Sisters
In ∴ the above
Either ,, Ιand
diagram
or ΙΙ are
,,, follows
,, negative.
does not
and Let
Choice
follow
ΙΙΙ usbuttry
follows. (3), to cannot be L or P.
Negative Motors
conclusion , follows. To prove it false WifemRavi o Brother o Wife o Brother o Brother 21. Ni
Cars draw
follows. an , and
alternate diagramto each
,, are contrary to disprove
other. them. 13. From‘KN’condition
12.the
is K pair →
gets4ofthe K’s husband
lowest
ladies. points. is LChoice
and K(2) is
Choice Solutions
(1) for q
AC  500 Choice
are (4)
we have to show that “All cars are Cars
AC” In an 7. Negation
In the alternate
above diagram.
diagram conclusion , does not ? Either of
Sisters , or,,,,isfollows
“Some andhuman beings
,,, follows. Choice mad”(3) the sister of P. Soluti
Motors and Tea of =
,,,200is “Some mad personsLassi = 100(4)
areChoice
men”. Solutions 13.for‘KN’ is the pair
questions 14of toladies.
17: Choice (2)
follows. AC 500
Solutions
7. But ,,, cannot for questions 6 to 9:
?InOnly follows.diagram conclusionChoice
the,, above , does(1) not be disproved. 10. that
Given KSolutions
or N isof an
each the advocate.
following We cannot
numbers
for questions 14 to 17: aredeter-
coded
Solutions
TheRavi Tea for
= 200
alternate questions 6 toto9: proveLassi
follows. 6. has nodiagramsister but has brother. “Some = 100 human as letters, let us apply the following rules.
mine
Givenwho that exactly
each ofout the of these two
following is the are coded
numbers
2st Twin
4. The given statements can be represented in the beings are mad” is asOfollows. only not walkin
? Only ,, ACfollows. Choice
Cars (1) 6. Ravi has no sister but has brother. Numberadvocate.
as: letters,
1 2 let3 us 4apply 5 the 6 following
7 Choice
8 rules.
9 (4) 0 2st T
following basic diagram. Motors
No sister m Ravi o Brother Letter code : A B C D E F G H I J only n
4. TheHerogiven statements can be represented in the O Number : 1 2 3 4 5 6 7 8 9 0
In the above diagram conclusion Ι does 11.
(1) TheM is the
number
Letter
husband
starting
code :A B
of C
with N.
an Dodd EdigitChoice
F should
(3)
G Hbe I J
following basic diagram.
not follows. WifemRavi
Human beings o Brother o Wife o Brother o Brother coded as ‘X', except 9. 22. He is still
Hero Motors Butter Milk = 150 12.
(2) TheK (1)gets
numberThe the lowestwith
number
starting points.
starting Choice
with digit
an even an odd (1)beshould be simply turni
digit
should
∴ Only ΙΙ follows. Choice (1) N=?
Sisters Choice (3) coded as ‘X', except 9. 22. He
From the above diagram, Motors coded as ‘Y', except 4.
Conclusion ,, negative, follows. Indian Butter 500Milk = 150 13. ‘KN’(2) The is the
numberpair of ladies.
starting with anChoice (2) should23.
even digit be He is facing sim
7. N=? coded as ‘Y', except 4.
In the
Conclusion ,,, above diagram
affirmative, conclusion , does not
follows. 23. He
Tea = 200 Man Lassi = Mad 100
follows.
Conclusion ,,,, affirmative, does notIndian
follows.
? Only ,, follows., follows. To proveChoice (1) In the above diagram ,, does not follow but ,
Sol/959
Negative conclusion it false follows. , and ,, are contrary to each other.
we 4.have
Thetogiven
People show that “Allcan
statements cars
be are AC” In in
represented anthe O and ,,, follows.
? Either , or ,, follows Sol/959
alternate diagram.
following basic diagram. Choice (4)
08_Section.indb 153People 10/30/2009 11:54:14 AM
Hero
3 1
26. This statement is evident from the first line of the
passage itself and finds support subsequently. (Afte
Hence this statement is definitely true in the light Sum
of this passage. Choice (1) Sim
in th
27. The passage talks of the winds of change
blowing across the education sector in India and S
154  n  Solution Manual also tells that massive changes are taking place
in terms of infrastructure but nowhere we can 19
find hence comparison between India and other
Solutions for questions 14 to 17: 19. Raj used green coloured bike. Choice (4) alsocountries
contributing its mite
and hence theby opening
given up cannot
statement By a
Given that each of the following numbers are 20. Viru who is the last to finish – used Red the be inferred to
education be true
sector to and neither
private can it classified
investors.
as false. Hence, the data is inadequate to pass 35. This
coded as letters, let us apply the following coloured bike. Choice (1) Theanygiven statement
judgement oncontradicts these facts
this statement. Choice (3) It is
rules. hence it is definitely false. Choice (4) We
Number : 1 2 3 4 21. Niru used yellow coloured bike. Choice 28. The passage specifies the changes man
(4) 29. Theencompassing
change beingthe craved are forsector
education employ-
in India and 4
5 6 7 8 also the courses
movesandbeing undertaken to
ment oriented thus engineer- (4 –
9 0 Solutions for questions 22 to 24: ing accommodate
and technologythese changes. The government is
courses are finding
also contributing its mite by opening up the
Sub
Letter code : A B C D mult
more seekerssector
education than toanything else. This
private investors. The given
E F G H North 1st Twin only goesstatement
on to show that these
contradicts thesesectors are
facts hence it is (14
I J 45° (not walking) definitely false. Choice (4)
comparatively generating more employ- Hen
(1) The number starting with an odd digit 2nd Twin ment
29. opportunities
The change being thancraved
other sectors.
are for The
employment (25
only not walking 15 km given statement
oriented contradicts
courses this inference
and thus engineering and 36. 289
should be coded as ‘X’, except 9. All t
andtechnology courses are
hence it is definitely finding more
false.Choice (4) seekers prim
(2) The number starting with an even digit than anything else. This goes on to show that
O
should be coded as ‘Y’, except 4. Solutions these sectors are comparatively generating more
for questions 30 to 36: Solution
employment opportunities than other sectors.
(3) If the digit 4 and 9 are both not at the ex- 22. He is still 15 km to the North because he
30. Let The
the given statement
number contradicts
of small cubes ofthiscubes
inference and 37. Acc
tremes of the number then they should be is simply turning but not walking.Choice hence it is definitely false. Choice (4)
be n be a
coded as ‘Z’. This means if 4 and 9 are at (3) their
Solutions
Then n × 9for× questions
9 × 9 = 3630 to 36:
× 36 × 36
the extremes, then they should be coded as 23. He is facing the west. Choice (3) Hen
∴n
30. Let the number of small cubesChoice
= 64 (2)be n
of cubes argu
per the letter code. Excluding these three
que
cases, the remaining digits should be coded 24. The meeting should be on any Sunday of 31. ForThen
each nletter
u 9 uthe
9 uconsecutive
9 = 36 u 36 unext
36 letter
?n = 64 Choice (2) stud
as per the letter codes mentioned earlier. October. 21st August is Thursday so total from the Alphabet are written in the fol- stro
days in August + September is 40. lowing order.
31. For each letter the consecutive next letter from
14. The given number is 56728. It starts with 40 ÷ 7 = 5 remainder the Alphabet are written in the following order. 38. , is
an odd number, so 5 is coded as X and the only
So last day of September is Tuesday and U N I V E R S E child
remaining as per the given letter-code. 1st of October is Wednesday so, 5th will
Hence, 56728 is coded as ‘XFGBH’ be Sunday. Choice (2)
 Choice (1)
24. The meeting should be on any Sunday of
Solutions for questions 25 to 29:
October. 21st August is Thursday so total days in
15. The given number is 987654. The digits
at extremes are 4 and 9. So, they are cod- 25. August + September
The passage tells usis 40. that nowadays the F V T O S J F W
40 y 7 = 5 remainder
ed as per the letter code. Hence, 987654 24. Thetrend is
meeting more towards
should be technology-based
on any Sunday of
So last day of September is Tuesday and 1st of
F A M I L Y
October.
courses21st August is Thursday so employ-
total days in
is coded as IHGFED. Choice (2) October isand courses
Wednesday which
so, 5thare will be Sunday.
August
24. The + September
meetingThe is 40.
should be on
40ment = directed.
y 7October.
5 remainder passage alsoanytells Sunday of
Choice F V T O S J F W
16. The given number is 398756. It starts (2) 21st August is Thursday so total days in F A M I L Y
Souslast
that
day this
of is the
September
August + September is 40.
transformation
is Tuesday and that
1st of
with 3, an odd number, hence 3 is cod- F MV BT JO NS J F W Choice (3)
has 40
October
Solutions taken
is
for place
Wednesday
questions in
÷ 7 = 5 remainder the
25 so,to education
5th
29: will sector.
be Sunday. Z G
ed as ‘X’. 9 is not at the extreme, so 9 Choice F A M I L Y
The So statement
last day of here tells us isthat
September learn-
Tuesday and 1st of 32. The pairs, which satisfy the given Choice (3) are AG,
criteria,
is coded as ‘Z’. So, 398756 is coded as 25.(2)
The passage
October tells
is us that nowadays
Wednesday so, 5th the trend
willthe is
be Sunday.
ing rather than employability
more towards technology-based courses and was GI and NO. Choice (2)
‘XZHGEF’. Choice (1) Solutions forinterest
questions 25 to 29:of yore 32. The
Choice Z pairs,
G M which
B satisfy
J N the given criteria, Choice (3)
area(2)
courses of which in the
are days
employment which The
directed.
17. The given number is 245376. It starts passage also tellssupported
us thatin this is the are Except
33. AG, GIGrape,
and NO. Choice
others are rhyming (2)
words.
though not directly
25. The passage tells us that nowadays the trend is the pas- 32. The pairs, which satisfy the given criteria, are AG,
Choice (4) (3)
transformation
Solutions for that has 25
questions taken
to 29: place in the ZNO. G M B J N Choice
with 2, an even number. So, 2 is coded as sage
more can
towards be inferred
technology-based from
education sector. The statement here tells us
other state-
courses and 33. Except Grape, others are rhyming words. (2)
GI and Choice
courses 34. Taking the place values of the letters in the word
Y. 4 is not at the extremes so coded as ‘Z’. ment inwhich
25. learning
that The it.
passage are employment
Hence,
rathertells thisusis
than that directed.
probably
nowadays
employability true.
was
The
the  Except
trend is33.
the 32. The pairs,others
Grape, whichare
satisfy Choice
the
rhyming given (4)
words. criteria, are AG,
passage also tells us that this is the CONFIDENCE
GI and NO.we get: Choice (2)
Hence, YZECGF. Choice (1) 
area more
of interesttowards
in thetechnology-based
days of yore whichcourses
Choice though and
(2)
transformation that has taken place in the 34. Taking the place values of the letters in (4) Choice
coursessupported
not directly which arein employment
the passagedirected.
can be The C O N F I D E N C E
Solutions for questions 18 to 21: 26. education
Thispassage
inferred
sector.also
statement
from other isThe statement
evident
tells
statement from
us here tells
thisthisus
the first
in it.that
Hence, isis the34. 33. Except
theTaking
word the Grape,
CONFIDENCE
place others
values of are
wetheget:rhyming
letters in words.
the word
that learning rather than employability was thein the CONFIDENCE we get: Choice (4)
Given that none among Raj, Viru and Sri won lineoftransformation
probably
area
true.passage that
ofinterest
the in the days
has
itselfof and
yore
taken
finds
which
place(2)
Choice
sup-
though
education sector. The statement here tells us 3 15 14 6 9 4 5 14 3 5
the race. Also Niru is the fourth to finish. portdirectly
26.not
This subsequently.
statement supported
that learning
Hence
is evident
rather
thispassage
in from
the
than
statement
the can
first line
employability
isof be
the the
was
C 34.
O Taking
N F the I Dplace
E N values
C Eof the letters in the word
inferred
definitelyfrom other
true statement
in the ofinthis
it.ofHence, this isthough (AfterCONFIDENCE we get:
adding the double digit values)
∴ Mani won the race. passagearea itself
of and
interest inlight
finds thesupport
days passage.
subsequently.
yore which
probably true. Choice Sum C of these
O Nvalues
F I isD 78.
As Niru cannot have red, blue, green (∵ 4th
Hence
 notthis statement
directly is definitely
supported in the true
Choice
passage(1)(2)light
in the can be 3Similarly
15 14 6by 9taking
E N C E
4 5the14place
3 values
5 of the letters
of thisinferred
passage.
from other statement inChoice
it. (1)
Hence,
place) and white. 26. This statement is evident from the first line of the this is in the word the word SUCCESS we get,
27. passage
The probably
passage talks
itself true.
and
of thesupport
winds of change Choice (2) (After (Afteradding
addingthe thedouble
double digit values)
digit values)
27. The passage talksfindsof the winds subsequently.
of change
Niru used yellow coloured bike. blowing across the education sector in Sum 3 these
15 14values
ofthese 6 9 is478.5 14 3 5
Hence
blowing
26. Thisthis statement
across
statement is evident
definitely
the education
is truethe
sector
from in India
in the light
first andof the Sum
line S of U C C values
E Sis 78.S
Given that the persons with red and yellow India
ofalso andthat
thistells
passage. alsomassive
tellsandthat massive
changes arechanges
Choice
taking (1)
place Similarly by adding
(After taking the place
theplace
double values of the letters
digit values)
passage itself finds support subsequently. Similarly
in the Sum by taking
word thethesethe
word SUCCESS values of the
coloured bikes finished the race one after the in
are terms
taking ofplace
infrastructure
in terms butdefinitely
of nowheretrue
infrastructure weincan 19 21 3 of
3 5 19 values
19 is 78.we get,
Hence this statement
27. The passage talks of the winds of change is the light letters in the word the word SUCCESS
other, the fifth placed peson used red coloured find hence this comparison
of across passage. between India and other (1)
Choice Similarly by taking the place values of the letters
but nowhere
blowing we
the can find hence
education
countries and hence the given statement cannot
sectorcompari-
in India and weSget,Uin CtheCword E the
S word
S SUCCESS we get, (1)
bike (∵ 3rd is green). also tells that massive changes are winds
taking place By adding the values we get 89. Choice
son
27. between
The India
passage and
talksotherof countries
the
be inferred to be true and neither can it classified andof change
As Sri used white coloured bike, he finished sec- in terms of infrastructure but nowhere wein can 35. 19This 21is S3
a 3
U 5
C
question 19
C E19
based S onSartificial values
hence
as false.the
find hence
blowing giventhe
Hence,across
comparison
statement
the cannot
education
data is inadequate
between
be
sector
India are
in-
andtaking
India
to pass
other place
and
any also
judgementtells that
on andmassive
this statement.changes It is given that 4 x 5 = 12 & 14 x 6 = 65
ond and Mani used blue coloured bike. ferred
countries toand
be true
hence theneither can itChoice
given statement classi- (3)
cannot By
in terms of infrastructure but nowhere we can Weadding
get thisthe values
value on we get 89. of Choice
account (1)
the following
As Viru did not finish 3rd, Raj finished third and be fied as
28. The findfalse.
inferred to
passage
henceHence,
be true the databetween
and neither
specifies
comparison is inadequate
can it classified
the India changes
and other 19 21 3 3 5 19 19
manipulation:
Viru finished fifth. as false. Hence, the data is inadequate to pass 35. This is a question based on artificial values
encompassing
to passcountries the education
any judgement
and henceon sector
thethis
given in India and
statement.
statement cannot By adding 4 By u the 5 values
adding we get 89. Choice
= 12values
any
alsojudgement ontothis It is given that 4 xthe
5 = 12 &we14get x 689.
= 65 Choice (1)
 betheinferred moves bestatement.
being
true and neither Choice
undertaken
Choice can (3) to
(3)it classified (1)We(4 –get1) uthis
(5 –value
1) = 12on account of the following
(1)  Mani  Blue (2)  Sri  White accommodate
false. these
28. The aspassage Hence,changes.
the dataThe
specifies is governmenttois pass
theinadequate
changes 35. This is a one
Subtracting question based
from on artificial
each number values
and
up the (3) manipulation:
and 35.
also contributing
28. encompassing
The any
passage itsonmite
specifies by opening
the changes This is aIt question based
4 x 5on= artificial
& 14val-
(3)  Raj  Green (4)  Niru  Yellow judgement
the educationthis sector inencom-
statement. Choice
India is given
multiplying
4 We u 5
that
the numbers so 12
obtained. x 6 = 65
= 12value on account of the following
education sector to private investors. The given get
u this
(5)  Viru  Red passing
also thethe education
moves sector
being in India
undertakenand to ues(4 – 1)14 u (5 – 1)
6
= 12
= 65
28. The contradicts
statement passage these specifies the it changes
facts hence is
accommodate
also the moves these changes. The government is and –manipulation:
is(14given1) that
It Subtracting u (64 5–x 1) = 12
5from
= 65 & 14 x 6
false. being
encompassing
definitely undertaken
the education tosector
accom-
Choicein (4)
India 4 u× one = 12 each number and
18. Mani won the race. Choice (1) also contributing itsmoves
mite bybeing
opening up the Hence 25
multiplying the 3
numbers so obtained.
modate these = 65 (4 – 1) × (5 – 1) = 12
sectorchanges. Theinvestors.
governmentThe is
also the undertaken to
education
29. The accommodate to private
change being thesecraved are for The
changes. given
employment
government is (25 14
– 1) uu (3 6 – 21) = =6548 from Choice (2)
Subtracting
2 one 2
121each
2 number 2 and
statement
oriented contradicts
also courses andthese
contributing itsthus facts
mite by hence
engineering up the 36. (14
it and
opening is 289–=1)17 ; u361(6=–19
multiplying 1) ; numbers
the
169
= 65= 13 ;so = 11 ; 225
obtained.
= 15
definitely
technologyfalse.
educationcourses
sectorare Choice
finding investors.
to private (4)
more seekers
The given All these numbers, except 225, are squares of
Hence 25 14 u 3× 6 = 65
than statement
anything else. This goes
contradicts theseon to show that it is prime numbers. Choice (3)
29. The change being craved are for facts hence
employment (25 – 1)
(142 –u1) (3 –×21)(6=–48 1) =265 Choice (2)
thesedefinitely
sectors are comparatively generating
false. more (4) 36. 289 = 17
Choice ; 361 = 19 ; 169 = 13 ; 121 = 11
2
; 225 = 15
2
oriented courses and thus engineering and SolutionsHence
for questions
25 × 337 to 40:
employment opportunities than other sectors. All these numbers, except 225, are squares of (2)
technology
29. given courses
The change are finding more seekers (25 – 1) × (3 – 1) = 48 Choice
08_Section.indb 154 The statementbeing cravedthis
contradicts are for employment
inference and 37.prime numbers.
According to 2,, some deserving
10/30/2009 11:54:16 AM
2 Choice
students 2(3)not
may
than anything else. This goes on to show that 2 2
Solutions for Intelligence and Critical Reasoning  n  155

 e get this value on account of the fol-


W Solutions for questions 37 to 40: ΙΙ is strong for the reason maintained
lowing manipulation: 37. According to Ι, some deserving students above. Choice (2)
4 × 5 = 12 may not be able to perform due to sick- 39. Ι is a strong argument, as the discovery of
(4 – 1) × (5 – 1) = 12 ness etc., hence their true ability does new oil reserves should result in decrease
Subtracting one from each number and not come to the fore. Hence, Ι is a strong in quantity of oil imported, thus same for-
multiplying the numbers so obtained. argument. ΙΙ is also a strong argument, eign exchange. ΙΙ is not a strong argument
14 × 6 = 65 as the examiner’s integrity can be ques- as these is no substantiation and explana-
(14 – 1) × (6 – 1) = 65 tioned; the examiner might favour certain tion as to why the existing contracts are
Hence 25 × 3 students or could be biased. Hence, both not repeatable.
(25 – 1) × (3 – 1) = 48  Choice (2) are strong arguments. Choice (3)  Choice (1)
36. 289 = 172; 361 = 192; 169 = 132; 121 = 38. Ι is not strong as the officer can be dis- 40. Question in discussion is not R & D but
112; 225 = 152 missed only if it is known that he is aware Rupees 1.5 crore. Hence neither of the ar-
All these numbers, except 225, are squares of existence of child labour and he is not guments is strong.
of prime numbers. Choice (3) taking any action.  Choice (4)

08_Section.indb 155 10/30/2009 11:54:16 AM


Solutions for Mock Tests 6
MOCK TEST 1 concluded because the total expenditures Difference = 51 – 47.25 = Rs 3.75
are not known. Choice (2)  Choice (4)
Solutions for questions 1 to 3: Solutions for questions 8 to 11: 14. Initial price of petrol in Delhi = 45; Final
price of diesel in Jaipur = 33. Required
1. The required cost per tonne 25 − 20 40 − 25 45 − 40
8. Of , , 45
20 25 40 percentage = × 100 = 1364/11 per cent
(35 + 40 ) × (135) @ 2×1 33
=  Choice (4)
(35 + 47 ) × 60 and
50 − 45 50 − 45
, is the least.
 Choice (1) 45 45
 Choice (4) 15. In Delhi, the percentage change in the
2. The costliest mode of transport must
5
have the highest cost of transportation price per liter of petrol = × 100 =
per tonne. 25 + 25 + 40 90 45
9. The required ratio = = = 100
10 40 35 15 15 30 + 45 + 45 120 = 11.11 per cent
Out of , , and , is the 9
8 35 47 10 10 3:4  Choice (1)
greatest. Hence, the costliest mode of  Choice (1)
transport is Air freight. Solutions for questions 16 to 20:
 Choice (4) 30 − 25 45 − 30 50 − 45 The contribution of manufacturing to the GDP
35 40 10 10. Of , , in 2001, 2002, 2003, 2004, 2005 and 2006 are
3. As < < , B < A < C Choice (4) 25 30 45
47 35 8 125, 165, 137.5, 208, 245 and 225 billion rupees
55 − 50 45 − 30 respectively.
and , is the highest.
Solutions for questions 4 to 7: 50 30 Total contribution of manufacturing to the GDP
4. Total expenditure of A = Rs 6000; Amount  Choice (2) in the given years = 125 + 165 + 137×5 + 208
62.5 11. The required percentage increase + 245 + 225
paid by A as rent = Rs 6000 3 =
100 = 1105 × 5 billion rupees
Rs 3750 (50 + 55) − (45 + 50) × 100 Total GDP in the given years = 500 + 600 + 500
=
 otal expenditure of C = Rs 7000;
T (45 + 50) + 650 + 700 + 750 = 3700 billion rupees.
25
Amount paid by C as rent = × 7000 = 10.5 per cent 16. In 2002 and 2006 Choice (2)
100  Choice (2).
= Rs 1750
The difference = Rs (3750 –1750) Solutions for questions 12 to 15: 1105 ⋅ 5
17. The required percentage is × 100
= Rs 2000 12. Let the regular consumption of pet- 3700
 Choice (2) rol of Mr.Manjeet be 100 liters. Ex- = 29×87 @ 30 Choice (1)
5. Total expenditure of D = Rs 9000; Amount penditure at the initial price of petrol
= Rs 4500. When price per liter of petrol 18. Total contribution of
25
spent by D on food = × 9000 = Rs is increased to Rs 50 the new consump-
100 20
tion be x liters ; \ x 3 50 = 4500 IT = × 3700 = 740 billion rupees.
2250 Choice (3) x = 90, \ Percentage decrease in petrol 100
6. Let B’s total expenditure be B and C’s total consumption = 10 per cent Choice (2) 1105.5
expenditure be C. Given, C’s expenditure = 1.49 @ 1.5 Choice (3)
13. Average price of petrol per liter initially 740
on food is 40 per cent more than B’s expen-
diture on food that is 50 per cent of C = 1.4
45 + 47 + 48 + 49 189
3 25 per cent of B ⇒C = 70 per cent of B ; = = Rs = Rs 47.25 350
4 4 19. The least GDP = × 100 = 875 billion
C is 30 per cent less than B. Choice (4) 40
Average price of petrol per litre finally = rupees.
7. B is paying 50 per cent of his total
expenditure towards rent whereas 25  Choice (4)
per cent of his total expenditure on his 50 + 51 + 51 + 52 204
= Rs = 51 20. It was the lowest in 2001  Choice (1)
food. Other two statements cannot be 4 4

08_Section.indb 156 10/30/2009 11:54:17 AM


Solutions for Mock Tests  n  157

Solutions for questions 21 to 24: B is an error. In statement D, ‘she also must have parallel structure. Since the
has apparently’ should read ‘she has also first in the list is ‘eradicated’ the others
21. In sentence A only cry can fit. In B it can
apparently’. Choice (2) must be ‘established’ and ‘lifted’.
only be ‘cry’. In C it can only be ‘weep
for joy’. In D it can be sob, wail or cry, 31. Only statement D is grammatically in-  Choice (4)
wail. Cry fits into 3 blanks and that is the correct. ‘Commandos are put through a
maximum. Choice (3) grind of all classes’ in the sentence should Solutions for question 37 to 40:
be corrected by inserting ‘the’ (definite 37. Between sentences A and C, C is a better
22. In sentence A it is ‘below’ – below is
article) before ‘classes’. Choice (3) opening sentence. ‘That universe’in B links
used for position or measurement on a
scale. In B it can be under underneath 32. Statements A and B are error-free. So A and D are a pair because they explain
or beside. In C it can only be under– option 4 is the answer. Incorrect use of why the mobile phone is unique. Hence
under is used for movement from one phrasal verbs make both sentences C and CBAD.
side below something to another. In D it D erroneous. The error ‘New Delhi was  Choice (3)
can be under or beside. Since under fits shut away from’ can be corrected in two
38. B is ideally the opening sentence be-
into three blanks, the answer is three. ways. New Delhi was shut out of Afghan
cause it is a generalized statement which
 Choice (3) affairs or New Delhi was shut off from
states the topic on which the rest of the
Afghan affairs. Both imply kept away
23. In sentence A it can only be ‘economical’. paragraph is used. C follows B by stat-
from. In statement D, ‘the situation has
Only ‘economy’ fits into B. In C it can ing how the researches were instrumen-
turned over’ is wrong. Turned around is
only be ‘economic’ and in D ‘economies’. tal in identifying the climatic shifts.
correct, which implies a situation which
As each word is used only one time, the A follows C as it caries the idea forward
changed for the better. Choice (4)
answer is one. Choice (1) by stating when the climatic changes
24. In sentence A only ‘frowned’ goes with Solutions for questions 33 to 36: took place. D follows A because ‘these’
‘upon’. In B it is grimaced. In C it can be in statement D refer to the ‘periods of
33. In choice (A), ‘in getting’ is inappropri-
frowned, ‘scowled’ or glared. In D it is climatic’ change mentioned in A. Hence
ate. The challenge the warning system
‘scowled’. ‘Scowled’ fits into 2 sentenc- BCAD is the correct order.
faces is getting the word out or ‘to get the  Choice (1)
es, as also frowned’, the answer is two. word out’. Choices (B) and (C) are ruled
 Choice (2) out due to an error in the parallel struc- 39. Sentence B is appropriate as the opening
ture. statement because it is very generalized.
Solutions for questions 25 to 28: Statement D follows B as it goes on to
In choice (C) the words’ to get the ……
25. A chance happening is ‘serendipity’. describe how Christmas fuels consum-
and making sure” make the sentence
‘Revelation’ means creating awareness erism. Statement A follows D by saying
incorrect. The sentence should read “to
‘serum’ is a thin liquid while ‘prescience’ consumerism fuels global warming which
get the word out …… and to make sure
is the art of forecasting events. in turn fuels fear, guilt, doom etc. C ide-
…….” ally sums up the para by saying that ‘all
 Choice (2)
Apart from the above mentioned errors, these acts and feelings’, that is the acts
26. ‘Flamboyant’ is used with people rather choice (D) has a tense error. ‘Will re- and feelings, referred to in the preceding
than for a work of art. ‘Picturesque’ is spond’ is not appropriate in the context. sentences, are a far cry from Christmas.
something that is attractive and interest-  Choice (2) Hence BDAC is the correct order.
ing while ‘resplendent’ means something
34. ‘Regarded’ should be followed by ‘as’  Choice (2)
that is dazzling which cannot be used to
and not ‘to be’ which makes option A
substitute the phrase. Only ‘chromatic’ is 40. From among the given choices, we un-
erroneous. Option B is correct. Option
an apt substitute. Choice (2) derstand that either B or C may open
C is erroneous because of the phrase ‘of
27. A long and angry speech is ‘a tirade’ ‘In- the paragraph. The words “It is unlikely
transmitting; which follows ‘the right of
vective’ is abuse while ‘riposte’ is an an- …….” indicate that B is a continuation
television’. It should read the right of tele-
gry retort. Choice (2) of an idea. Hence ‘C’ is the appropriate
vision ‘to transmit’ because we have the
statement to open the paragraph. Op-
28. “Allayed’ is to soothe or assuage while right to do something not ‘for something’
tion (3) is ruled out. The words ‘use of
‘revoke’ is to take back an order ‘Alien- option D is erroneous because of the use
foreign aid’ in ‘C’ find continuation in
ate’ is to become unfriendly or unsym- of ‘mass’ in place of ‘massive’. Massive
A. Hence CA go together. Further D
pathetic to someone. To ‘nullify’ is to means large whereas ‘mass’ means ‘com-
follows A. The word ‘they’ in D refers
negate or render something ‘ineffective’. monplace’ which is not the meaning in-
to the words ‘state governments’ in A.
 Choice (1) tended in this context. Choice (2)
 Choice (2)
35. Statements B and C are erroneous be-
Solutions for questions 29 to 32: cause these statements do not maintain Solutions for Questions 41 to 60:
29. Statements A and D are erroneous, mak- consistency in tense. These sentences are 41. Let b be the number of lillies and a be the
ing option 1 (B and C) the correct answer. in the past tense and hence present per- number of roses purchased. Then a + b = 50
‘In the heart of the capital Mumbai’ in fect should not be used. Statement D is
statement A should be corrected to ‘in the incorrect as ‘was to rid of the nation’ rep- Let x be the cost of one rose and y be the
heart of its capital , Mumbai’. In state- resents incorrect use of the phrasal verb cost of one lilly.
ment D, omission of article ‘a’ before ‘rid something of something’. The cor-
Then ax + by = 125 × (1) and bx + ay
daily wage earner is the error.Choice (1) rection is ‘rid the nation of the idea…..’.
 Choice (1) = 175 × (2)
30. Statements A and C are grammatically
correct. Omission of the definite article 36. The sentence says what China will have (ax + by) + (ay + bx) = 300 × (a + b) (x + y)
‘the’ before 1960s and 1980s in statement achieved by 2050. The achievements = 300; x + y = 300/50 = 6 Choice (2)

08_Section.indb 157 10/30/2009 11:54:17 AM


158  n  Solution Manual

42. Let C. P of 50 articles = S.P. of x articles = 54. Since we do not know the age of the em-
tom of pole and A is 3 times the height
1; C.P of 1 article = 1/50 S.P. of 1 article ployee who returns, the answer cannot be
= 1/x of the pole. So height of the pole cannot determined. Choice (4)
be equal to distance between points AB.
1 1 55. Let B take a days to complete the work. Then
−  Choice (4)
25
x 50 = ⇒ x = 40  Choice (3) A takes 2a days and C also takes 2a days
1 100 49. The cost of 4 pairs of glows is Rs 600. to complete the work. A works on the first
50 Hence, the cost per pair is Rs 150, which and fourth days and does 1/a. B works on
43. Work done by P + Q in one day = 1/15 is 75 per cent of Rs 200. the 2nd day and does 1/a. Hence A and B
Work done by Q + R in one day = 1/20 contribute equal amounts Choice (4)
\ It is cheaper by 25 per cent.Choice (2)
Work done by R + P in one day = 1/25 (2P 56. Let the critical population be 100; Net in-
50. The cost would be 5,00,000 crease in 1 year = 7 per cent
1 1 1
+ 2Q + 2R) one day work = + + 70 70 70 \ population after 1 year = 1.07 3 100
15 20 25 3 × × = Rs 1,71,500
100 100 100 in 3 years net increase in population =
47 (10.01)3 3 100
=
300  Choice (1)
\ net per cent increase = 22.5 Choice
51. Let us assume each man can do the work (1)
47 in x days
P + Q + R one day work = ; Work
600 57. Let the C.P of each article be Rs 100; S.P
completed in 3 days by three of them = Total work done by 5 men in 5 days = of the article which is sold at 28 per cent
47 47 5×5 profit = Rs 128
3× = =
600 200 x S.P of another article which was sold at
7×5 35 14 per cent loss = Rs 86
25/x ; Second part = ; Total
47 153 x x Total C.P = Rs 200; Total S.P = Rs 214;
Remaining work = 1 – = ; (Q +
200 200 Profit per cent = 14/200 x 100 = 7 per cent
60
work =  Choice (4)
R) can complete whole work in 20 days. x
58. Let the length of the train be L m. Let
5 men in one day can do 5/x of the work; 5
 153  its speed be S m/sec. Time taken by it to
  work can be completed by them in 60 cross the 100 m platform = L seconds;
 200  men need 12 days to complete work
x
153 153 \ 5 men need 2 more days to complete L
20 × = = 15.3 days Choice (3) = 40 ⇒ L = 40S.
200 10 the work S
 Choice (1)
44. Speed of the train = 126 km/h = 35 m/s L + 100 40S + 100
52. B’s speed = 20/2 = 10 m/s; Time taken by Required time = S; =
Total distance covered = length of train + S+5 S+5
B to complete the race = 500/10 = 50 sec
length of platform= 100 + 50 = 150 40(S + 5) − 100 100
\ A took 50 – 2 = 48 seconds to complete = = 40 –
150 the race. S+5 S+5
Time taken = = 4.28 = 4.3
35
D’s speed = 100/20 = 5 m/s; Time taken This is always less than 40. Only choice
 Choice (1) by D to complete the race = 500/5 = 100
sec (3) violates this condition. Choice (3)
45. Radius of circle = 5; \ Area of circle =
25p; Side of hexagon = 5 \ Time taken by C to complete the race 59. The actual cost per kg of the mixture =
= 100 - 20 = 80 sec
\ Area of hexagon = 6 3 / 4 × 25 = 30 × 6 + 40 × 5 380 6
\ Ratio of times taken by A and C to com- = = 34 .
11 11 11
75 3 75 3 plete the race = 48 : 80 = 3 : 5Choice (4)
; \ Area left = 25p – = 13.5
2 2 53. Let the speed of the boat in still water be
6
x kmph; Speed of stream = y kmph 35 − 34
 Choice (2) \The profit percentage = 11 × 100
Relative speed while going down 6
46. As two boys are sitting together, they stream = (x + y) kmph; Rela- 34
have to be considered as one unit. So 7 11
tive speed while going up stream
units can be arranged around a circular 5
= (x - y) kmph = ×100 @ 1.3 per cent. Choice (1)
table in 6! ways and the two boys can be 380
internally arranged in 2! ways. \ They 45 45
In1st case, + = 14 ---- (1) 60. The required number of ways is 7C4
can be arranged in 6! 3 2! ways. x+y x−y
 Choice (2) 7! 7.6.5
36 = = = 35  Choice (1)
And in 2nd case =4⇒x+y=9
4!3! 1.2.3
47. The required probability = x+y
5
C2 + 6C2 45 45 45 Solutions for questions 61 to 64:
10 + 15 From (1), + = 14 ⇒ =9
11
= = 5/11. Choice (2) 9 x−y x−y 61. Here, Australian do not play in the
C2 55
spirit of the game, therefore they need
48. As the angle of elevation of top of the pole ⇒ x – y = 5; \ x = 7 and y = 2 counseling.
from A is 30°, the distance between bot-
 Choice (2) \ Assumption I is implicit.

08_Section.indb 158 10/30/2009 11:54:19 AM


Solutions for Mock Tests  n  159

It is talked about only Australians, in the 71. Statement I indicates that sacrifice is nec- Solutions for questions 79 and 80:
statement, thus assumption II is not im- essary for one to become a leader but it 79. It is given that
plicit. Choice (1) does not specify whether sacrifice is suffi-
cient or if there are any other requirements. EAGLE – 517125
62. Since, better career ‘starts’ with good
Hence, A does not follow, but B follows. CAMEL – 3113512
English, thus it is the essential require-
 Choice (2)
ment for a good career.; \ Assumption If we observe here, the place value of the
I is implicit. 72. According to statement I, availability of letters is taken as their code.
resources will make a country rich. It does
Statement II is talking about a better ca- Value of E is 5
not necessarily mean that nothing else
reer, it doesn’t mean that there will be no
would make a country rich. But when a Value of A is 1
future without the knowledge of English.
country is not rich, it can be certainly said
\ Assumption II is not implicit. Value of G is 7
that resources are not available. Hence, B
 Choice (1)
alone follows. Choice (2) Value of L is 12
63. According to the author, to have a healthy
Solutions for questions 73 to 75: Similarly, the code for TIGER is
life one should know about good health.
2097518. Choice (2)
The statement is regarding the sequence 73. My father’s sister is my paternal aunt and
of steps towards a healthy life. Hence, her mother is my paternal grandmother 80. Milk is white in colour and white is called
neither I nor II is the assumption based and my grand mother’s only daughter-in- Blue.
on which the statement is made. law is my mother. Her only son can be Hence, milk is Blue in colour. Choice (4)
 Choice (3) myself or my brother. Choice (4)
Solutions for questions 101 to 104:
64. Lack of Marketing is hazardous for the 74. The path traversed by Richmond is as fol-
success of a movie, thus it forms an es- lows. 101. Total expenditure in Jan = Rs 1571; Mar
sential part in the success of a movie. = Rs 2048 ; Aug = Rs 2072 and Nov =
As we know that (XY) = (XO) + (YO) 2 2 2
Rs 2288
\ Assumption I is implicit.
XO = 48 – 24 = 24 km \ Expenditure in January was the least.
‘Only the Marketing’ does not guarantee
the success of a movie, thus assumption OY = 14 – 7 = 7 km  Choice (1)
II is not implicit. 102. Highest price per kg was of D in the
2 2
 Choice (1) \(XY) = (24) − 7 = 625 = 25 km.
month of January = Rs 60
Solutions for questions 65 to 68:  Choice (3)
Lowest price per kg was of A in the month
65. I is not strong because it is irrelevant. 75. In the evening, as the sun is in the West, of August = Rs 18; The required ratio
II is strong because it states that less the shadow falls towards East. As the
= 60 : 18 = 10 : 3 Choice (2)
money is required for registered mar- shadow of the Pam is falling to his left,
hence he is facing South. 103. Total expenditure in the month of August
riages and hence money can be saved.
\ Sam is facing North. Choice (1) = (26 3 18) + (4 3 44) + (14 3 42) + (4 3 51) +
 Choice (2)
(12 3 53)
66. I is incomplete. Hence, I is not strong. Solutions for questions 76 to 78: = 468 + 176 + 588 + 204 + 636 = Rs
II is not strong as the relation between F is paternal grand mother of C. \ F be- 2072 Choice (3)
team's strength and conducting the event longs to the 1st generation and C belongs 104. Total amount spent on B in the month of July
is not established. Choice (3) to the 3rd generation. Similarly, from (ii), = 45 3 3 = Rs 135
67. It is the worth of the work, but not sat- B and finance manager belong to the 3rd In none of the cases, the amount spent is
isfaction, that is criteria for awarding generation and CEO belongs to the 1st Rs 135 Choice (4)
Nobel Prize. Hence I is not strong. More generation.
prizes does not lead to lose of signifi- The females are, B, D and F and the males Solutions for questions 105 to 109:
cance. Hence II is not strong. are C, E and A. The final diagram will be 105. Total sales in 2002-03 = 105 + 96 + 208 +
 Choice (3) as follows. 165 + 73 = 647 --------- (i)
68. I is not strong because it is irrelevant. II Total sales in 2000-01 = 94 + 68 + 165 +
Wife
is strong because it states that the effect A F HR Manager 112 + 57 = 496
CEO
of the lack of trees cannot be offset with Let 647 @ 650 and 496 @ 500
green paint. Choice (2) Son Sales target in 2002-02 = 1.2 x 500 = 600
wife D m tones. Hence there is surplus in the
Solutions for questions 69 to72: Logistics E Marketing
sales target
Manager Manager
69. Statement I shows the only way to in-  650 − 600 
Son \ required percentage = 
crease the forex reserves. It does not nec- sister x
essarily mean that an increase in exports C
B  500 
would always lead to an increase in forex (Finance (Operations 100 = 10 per cent Choice (2)
reserves. Hence, neither A nor B follows. Manager) Manager)
106. Sales of Tin in 2000-2001= 0.4132 x 57 =
 Choice (3) 23.55 million tonnes
76. D and E is a married couple. Choice (2)
70. Statement I indicates every sincere per- In 2002-2003, Sales of Tin = 23.55 x 1.1
son is hard working. Hence, those who 77. CEO is the father of the logistics manager. x 1.1 @ 28.5 million tonnes
are not hard working persons are not sin-  Choice (3) Total sales = 105 + 96 + 208 + 165 +
cere. 78. Finance manager is the son of the market- 73 = 647 million tones. Required per-
ing manager. Choice (1) centage
Hence, only A follows. Choice (1)

08_Section.indb 159 10/30/2009 11:54:19 AM


160  n  Solution Manual

ther from the first second statements in- \ Value in column A > Value in column
28.5
= x 100 = 4.4%  Choice (2) dividually. By combining both the state- B. Choice (1)
647 ments 300 - 120 = 180 families read at
least one and, at most, two newspapers. Solutions for questions 121 to 124:
107. Sales of copper in 2002-2003 = 96.
The sales increased by 25 per cent in But the number of families, that read ex- 121. When the author says that we also hear
the next year, but still remained 80 per actly two news papers, cannot be found. that India can leapfrog to new technolo-
cent of the quantity available. \ Quan-  Choice (4) gies without having to deal with legacy
tity of copper available in 2003-04 114 3 < a < b < c and a, b and c are odd. Using issues, he refers to the groundwork
statement A, c < 11 that needs to be done to bring in new
96 technologies, as is clear from the rest
= x1.25 = 150 Choice (2)
0.8 ⇒ a = 5, b = 7 and c = 9 is the only pos- of para 3. Choice (3)
sibility. Statement A alone is sufficient.
108. Sales of all metals in 2000-2001 = 496. 122. ‘Leaders proclaim India’s strength lies in
Using statement B, as a < 9, a can be either its ability to bypass industrial economy
Sales of all metals in 2002-2003 = 647 5 or 7. \ Statement B alone is not sufficient. and leapfrog from an agricultural econ-
Annual percentage increase  Choice (1) omy to a service based economy’. As ag-
ricultural economy is the first sector ac-
Solutions for questions 115 to 120:
647 − 496 1 cording to the author, industrial economy
= × 3 100 % [x = 3] 115. Column A:   1123 under base 4 = 91
496 (x − 1) must be the second and services the third.
Column B:   101111 under base 2 = 47.  Choice (4)
= 15.2 per cent Choice (4)
\ Value in Column A > Value in Column 123. ‘These sectors…….. workforce’(para 5, line
109. Sales of Gold in 2002-03 = 25 per cent of B Choice (1) 3 – 4) shows that (3) is the correct an-
73 = 73/4; Sales of swer.
Silver in 2000-01 = 20 per cent of 57 = 116. Column A: 80 per cent
 Choice (3)
57 80
of 95 = × 95 = 76 124. Refer to para 4. A careful reading points
5 100 to choice 1. Choice (1)
73 5 72 5
Required ratio = × ≅ × Column B: 25 per cent
4 57 4 55 Solutions for questions 125 to 128:
18
= 125. Refer last paragraph. Choice (1)
11 25
of 304 = × 304 = 76 126. Refer paragraph 3. Choice (2)
100
@ 1.67. The nearest value is given in
Choice (3) as 1.6 Choice (3) \ Value in column A = Value in column 127. Refer paragraph 4. Choice (3)
B Choice (3) 128. Refer paragraph 2 and 4. Choice (2)
Solutions for questions 110 to 114:
117. Column A:   8 + 2 15 = ( 5 + 3 )2 Solutions for questions 129 to 132:
110. Using statement A, we do not know ei-
129. Refer to the last para, lines 3-4 (loved
ther the total number of employees or the Column B:  9 + 2 14 = ( 7 + 2 )2; the world . . . . expressed themselves.)
number of female or male employees.
( 5 + 3 )2 < ( 7 + 2 )2  Choice (2)
Hence we cannot answer the question us-
ing statement A alone. \ Value in column B > Value in column A. 130. In the passage, the author uses the words
 Choice (2) to mean a dislike of empty spaces.
Using statement B, as we know the num-
 Choice (4)
ber of female employees as a fraction of the 118. Column A:
number of male employees, we can answer 131. Only choice (2) is right – in all other op-
(x – 2)2 = 9 x – 2 = ± 3 x = 5 or – 1 tions the attributes are reversed.
the question using statement B alone.
 Choice (2) Column B:  Choice (2)
111. Either of the statements are not individu- 2x + 3 = 11 2x = 8 x=4 132. Statements A and B are true – last para,
ally sufficient to answer the question as \The given two values cannot be compared. lines 1 – 3. Choice (1)
both give only one equation with the ages  Choice (4)
of Mahendar and Karunakaran. By com- Solutions for questions 133 to 136:
bining both the statements, we have two 2
5 7 −1 5 71 133. Refer to the first sentence of Para 6.
equations in two unknowns. Hence we 119. Column A: (3 ) = (3 ) = (243) 7

 Choice (2)
can answer the question using both state- 7

ments. Choice (3) Column B:


45
71 = 71 = 7 134. The 2nd line of the 1st para “one of the
chief --- differences --- incapable of com-
112. Using statement A, we do not know \ Value in column A > Value in column plete satisfaction” makes option 1 the
whether the three houses are in a straight B Choice (1) correct choice. Choice (1)
line or not. We cannot answer the ques-
120. Column A: The number of ways in which 135. Refer to lines 3 to 6 of para 4 “These are
tion using statement A alone. Statement
the letters of the word CRICKET can be the men --- human power” which shows
B is clearly redundant.
7! 5040 that option 3 is the right choice.
Even by combining both statements, we arranged is    = 2520.  Choice (3)
cannot answer the question. Choice (4) 2! 2
Column B: The number of ways in which 136. The words in quote occur at the end of
113. The number of families who read exactly the letters of the word HOCKEY can be para 2 (third line from the end). The
two newspapers cannot be obtained ei- arranged is 6! = 720. words ‘the same characteristics’ refer

08_Section.indb 160 10/30/2009 11:54:20 AM


Solutions for Mock Tests  n  161

to the characteristics mentioned in the 148. Average wage of lala in the month is
preceding lines – men like Xerxes and 1 2 3 4 5 6 7

Wage on the first day + Wage on the last day
Newton who continued to be active even (I)  ⇒ R is in 4
after their needs were met to achieve their (ii)  ⇒ P is in 5 2
dream. Choice (4) (iii) ⇒ S can be either at 2 or 3 but if S is at 2, v
Wage on the first day = 30
Solutions for questions 137 to 140: will be at 6 then either Q or T should be at 1 Wage on the last day = 30 + 30(10) =
or 7 which contradicts statements (ii) 330
137. Since the term manic – depressive will \ S is at 3 and V is at S, U is at 1 Q and T are at 30 + 330
refers to a wider range of mentally ill pa- Average wage = = 180
2 and 6. 2
tients in Great Britain it naturally follows Arrangements is U, Q/T, S, R, P, T/Q, V
that the number of patients will be greater  Choice (3)
in Great Britain than in US. So Choice 1 141. U and V are at either end of the row. 149. From the given information,
is the best Answer.  Choice (2)
Cost : - B > D ; A>C
Choice 2 suggests that the admission rate 142. Either T or Q can sit in between P and V. Fast : - D>C
is high in the limited states. So it is not  Choice (4) The cheapest car is the slowest car. It can-
useful in explaining the situation present-
143. T is to the immediate right of P. Choice not be B or A or D.
ed in the passage. Choice 3 is irrelevant.
(1) \ It must be C. Choice (3)
Choice 4 dwells on training institu-
tions in Psychology which has no Solutions for questions 144 to 150: 150. In the group stages, in every group a total
bearing on the number of admissions. of 5C2 ie., 10 matches are played. Hence
1 44. As the given day is Wednesday, in both groups a total of 20 matches
 Choice (1) Tuesday precedes the given day. are played. In the semis, two matches
138. Choice (1) is stated and hence is not an 15th day before yesterday is Monday are played and in the final, one match
assumption. Dr. Sylvan has not made a and the 4th day after Monday is Friday. is played. Hence a total of 23 matches.
link between salinity and the assumption  Choice (3)  Choice (2)
on the impact of salinity on the Viparia. 145. Let E, W, S and N are the pointers show-
However, he has assumed that the salinity 145. Leting E, W, Solutions for questions 151 to 155:
138. Choice of(1)theisMediterranean
stated and hence is not an the Seast,
andwest,
N aresouth
the pointers
and northshowing
accord-the
has not decreased 145. Let east,
E, west,
W, S andsouth andthe
N are north according
pointers showingto the
the There are eight friends and the number of persons,
assumption.
138. Choice (1) isDr.stated
Sylvanand hashence
not madeis not a link
an ing to
compass.
the compass. The actual directions
from the Sylvan
3andppm levels east, west, The
southactual directions
and north are to
according written
the who bought same colour dress is either 2 or 3.
between
assumption. salinity
Dr. the
has not and
assumption
made hence
on con-
the
a link are written
outside theThe
compass. outside the circle.
circle.actual directions are written
impact cludes
between thatand
ofsalinity
salinity thethe
on slug will notHowever,
the Viparia.
assumption survive there.
on thehe There is only one possibility = 3 + 3 + 2
outside the circle.
impact of salinity on
has assumed thatthe the salinity
Viparia. However,Choicehe(2)
of the North The given information is as follows.
Mediterranean
has assumed has thatnot the decreased
salinityfrom of the the3 North
139. levels
ppm The assumption
Mediterranean andhas hence behind thethat
not concludes
decreased group’s
from the con-
theslug3 W Blue White Pink Name Shop
will not
ppm tentionandis
survive
levels that animals
there.
hence concludes thatthat
are shy
Choice the and
slugac-
(2) W
will not survive
tive only there.
during Choice are(2) Shoppers’
139. The assumption behind the the night
group's time contentionfeared
S
  Pradeep
Stop
and persecuted for that reason. Option West N East
is that
139. The animals
assumption that are
behind theshy and
group's active
contention only 3
West S N East Akash Arrowhead
isduring establishes
that the night time
animals thatthat areowls
are shy too
feared and
and areactive
shy and
persecutedonlyac- 
for thatthe
during reason.
tive night Option
time
at night, but3feared
are establishes
they andneither
are that owls
persecuted feared
toothat
for are reason.
shy andOption active3 at night, butthat
establishes theyowls are    Supriya Pantaloon
E
neither
too
nor
are feared
persecuted.
shy andnor
Therefore
persecuted.
active
3
at night,Therefore
is the answer.
but they 3areis E    Sindhu
the answer.
neither Though
fearedThough options 1 and
options 1 and
nor persecuted. 4 provide
4 provide
Therefore a
3 iscon-
a South
context
the text
answer. to people’s
to people's
Though attitude
attitude
options towards
1towards
and thethe
4 provide devil,
devil,
a South Not Peter
Banu followed thethedirections shown by bythe  Powel
they dothey
context notpeople's
to affect
do notthe group's
attitude
affect contention.
thetowards
group’sthe Option
devil,
contention. Banu followed directions shown England
compass with him thus the path shown
travelledby
2 is irrelevant
they
Optionto2the
do not affect theargument.
group's contention.
is irrelevant Choice
to the (3)
Option
argument.
Banu thefollowed
compass the
with directions
him thus the path by him
the
trav-    Vandana Kalaniketan
2 is irrelevant to the argument. Choice (3) is as follows.
compass with him thus the path travelled by him
140. Option (1) discusses hearts transplant inChoice general(3) is aselled by him is as follows.
follows.
and does
140. Option not say hearts
(1) discusses anything about in
transplant natural
general or    Kavita Reebok
40 m
140. does
mechanical
and Option (1)say
hearts.
not discusses
anything heartsabouttransplant
naturalinor gen- 40 m  Chandan
Option eral
mechanical (2)andseriously
hearts. does not say anythingthe
weakens aboutgivennatural
recommendation
Option or(2) because
seriously
mechanical we do not
weakens
hearts. theknow giventhe 30 m 30 m From the above table as Pradeep did
effect of any typebecause
recommendation of medicine we do onnot a mechanical
know the 30 m 30 m not buy white or Pink he bought blue

heart. Option (2) seriously
effect of any type of medicine on a mechanical weakens the given coloured dress.
Option recommendation
heart. (3) may sound goodbecause as a flawwe but thendo itnot
may be possible
sound to The remaining three persons (Akash,
Option (3) may
know the effectgood
ofinject
as a flaw
any type this but
ofimportant
then it
medicine Powel & Chandan) bought white coloured
hormone.
may be possible to inject this important 25 m
Option on
hormone. (4)a mechanical
is not discussing heart. a flaw about
25 m dress.
mechanical
Option (4) is heart.
notmay Itdiscussing
ends up discussing
a flaw the
Option (3) sound good as a about
flaw but Now he is facing West. Choice (4) As, those who bought white coloured
current lack heart.
mechanical of knowledgeIt endsof up some cardiologists.
discussing the Now he is facing West. Choice (4)
These lack
current then it
cardiologistsmay
of knowledge be possible
couldofalways to inject
some increase
cardiologists.this
theirim- dress did not go to shoppers stop or
portant hormone. 146.
NowAshethe females
is facing and couple are
West. not (4)
Choice op- CMR. Sindhu went to CMR.
knowledge
These of how
cardiologists mechanical
could always pumps
increase could be
their 146. As the females and couple are not opposite.
made to work
knowledge efficiently.
of how mechanical pumps Choice
could(2)be 146. As posite.
Father
the and Fatherandand
daughter
females daughter
should
couple should
be opposite
are not be
each
opposite. \ Chandan went to Peter England and

Solutions
made for Option
questions
to work (4) is not discussing
141 to 143:
efficiently. a flaw
Choice (2) about opposite
other.
Father eachcannot
As father
and daughterother. Asbefather
face
should North, cannot
daughter
opposite each hence, Powel went to Nike.
Solutions formechanical
questions 141 heart. It ends up discussing
to 143: faceAs
cannot
other. North,
face daughter
South.
father cannot
cannot face face South.
Choice
North, (3)
daughter
Let be chairs are numbered as follows \ The final distribution table will be as
the current lack of knowledge of some 147. cannot
The difference between the actual Choice
face South. time and
Choice (3)the
(3)
Let
1 be 2chairs 3 4are numbered
5 6 7 as follows mirror time = 3 between
hours. the actual time and the follows
cardiologists. These cardiologists could 147. The difference
1(I) Ÿ 2R is 3in 44 5 6 7
always increase their knowledge of how 147. Thetime
Actual
mirror difference
time hours.between
=–3Mirror time = 3 - the - (i) actual time
(I)
(ii)ŸŸRPisisinin45 Further,
Actual
andtime
the –mirror
Mirrortimetime = =33 -hours.
- (i) Name Shop Blue White Pink
(ii)
mechanical pumps could be made to Mirror time+ Actual time = 12 - - (ii)
Further,
(iii)ŸŸPSiscanin 5 be either at 2 or 3 but if S is at 2, v will Actual time
be Ÿ
(iii) can work
at 6S then either
be efficiently.
Q or
either at T should
2 or 3 butbe at is1 at
if S or 2,
7 which
v will From
Mirror (i) and
time+ (ii),– Mirror
Actual
time = 3 - - (i)
time = 12 - - (ii) Pradeep Shoppers stop   
 thenstatements Further,
Actual
From (i)time
and =(ii),
7:30 Choice (2)
contradicts
be at 6 either Q or(ii)T should be at 1 or Choice
7 which(2) Akash Arrowhead   
contradicts
? S is at 3statements
and V is at(ii)S, U is at 1 Q and T are at Mirror
Actual timetime+
= 7:30Actual time = 12 Choice - - (ii) (2)
148. Average wage of lala in the month is
? S isSolutions
2 and 6.at 3 and forV isquestions
at S, U is141at 1toQ143:
and T are at From (i) and (ii), Supriya Pantaloon   
148. Average wage
the of
2Arrangements
and Let
6. be chairsis U, are
Q/T,numbered
S, R, P, T/Q, V
as follows WageActual on time =lala
first day
7:30in theWage
month onisChoice
the last (2)
day
Arrangements is U, Q/T, S, R, P, T/Q, V Wage on the first day 2 Wage on the last day
141. U and V are at either end of the row.
Wage on the first day =230
141. U and V are at either end of the row. Choice (2)
Wageon
Wage onthe
thefirst
last day
day == 30
30 + 30(10) = 330
Choice (2)
142. Either T or Q can sit in between P and V. Wage on the last day
30 =330
30 + 30(10) = 330
Average wage = 30  330 = 180
Choice
142. Either T or Q can sit in between P and V. (4) 2 = 180
Average wage =
08_Section.indb 161 Choice (4) 2 Choice (3) 10/30/2009 11:54:20 AM
143. T is to the immediate right of P.
162  n  Solution Manual

Name Shop Blue White Pink 12 2


North ⇒ p + p = 20 ⇒ p = 12 ⇒ 5p = 60
3
Sindhu CMR    Choice (3)

Powel Nike   

Vandana Kalaniketan    9 3 1 2 2 2
East 166. The required part = 1 − − × = .
West 3 3 3 9
Kavita Reebok   
 Choice (1)
Chandan Peter England   
6
151. Sindhu went to CMR. Choice (3) South 167. Let the time taken by tap B to fill the
tank be x hrs. Time taken by Tap A =
1 52. Chandan went to Peter England \ At 8:45 the minute hand points West (x – 4) hr.
. Choice (3) and the hour hand points towards South- \ Time taken by C = (x + 5) hr. The ef-
153. Powel went to Nike. Choice (3) West Choice (4) ficiency of A is equal to the sum of ef-
Solutions for questions 161 to 180: ficiencies of B and C;
154. Chandan - Peter England - White is the
correct combination. Choice (4) 161. Let speed of the boat in still water be x 1 1 1
\ = + \ x = 10; Total work
155. Akash and Powel bought white dress. kmph and speed of stream be y kmph. x−4 x+5 x
 Choice (1) done when all the three taps work together
30 20
Solutions for questions 156 to 159: + = 14 ------------ (1);
x−y x+y
The given routes between the cities are repre- 1 1 1 10 1
= + + = = ; Total time taken
sented in the following diagram : 33 15 6 10 15 30 3
+ = 14 ------------ (2)
B x−y x+y 3 hrs.  Choice (2)
G
Solving (1) and (2) we get x = 4 kmph, 168. Let the ages of Amit and Bharath be 4k
E D H y = 1 kmph. Choice (1) years and 5k years respectively.
162. Numberofwaysofselecting5from7students 4k + 18 7
A = 7C5 = 21 = ⇒ 32k + 144 = 35k + 126
5k + 18 8
F Number of ways of selecting 2 from 6 stu-
⇒ 3k = 18; \ k = 6. Choice (3)
dents = 6C2 = 15; \ Total number of ways
C I = 21 3 15 = 315. Choice (2) 169. Number of hemispherical blocks
156. The possible ways from G to I are 163. Let the speed of the first train be x kmph.  42   42   70 
Distance travelled by the 1st train in 1 hr =       = 90; Volume of the
(i) G-H-A-I  14   14   7 
= x km
(ii) G-H-A-F-C-I At 9.00 a.m. the distance between the cuboid
(iii) G-H-D-A-I trains = (160 - x); Speed of the second = (42) (42) (70) c.c. = 123480 c.c.
(iv) G-H-D-A-F-C-I train = (x + 10) kmph Volume occupied by the hemispherical
(v) G-H-D-E-C-I
(vi) G-H-D-E-F-C-I 160 − x  2   22 
blocks = 90     (7)3 c.c. = 64680
(vii) G-H-D-F-C-I Now = 1 ⇒ x = 50; Speed of 3  7 
x + x + 10
There are seven possible ways. c.c;
 Choice (3) the faster train = 60 kmph Choice (4)
\ Empty space = (123480 - 64680) c.c.
1 57. The person cannot reach city E. 164. Total number of balls = 7; Number of = 58800 c.c Choice (3)
 Choice (3) ways of selecting two balls is 7C2
Number of black balls = 3; Number of 170. Since the sphere is completely submerged
158. The largest possible route is
ways of selecting 2 black balls is 3C2 in water, the increase in the level of
E - B - G - H - D - A - F - C – I ; \He can water is because of the volume occupied
go through seven cities. Choice (4) \ Probability of selecting two black balls by the sphere. Let the increase in water
159. Counting the total number of odd days level be h.
3 × 2 ×1
from 15th March to 12th October. \ The increase in total volume = p
2 ×1 = 1 
3
C
is 7 2 = Choice (2)
Month:Mar+Apr+May+Jun+Jul+Aug+Sep C2 7 ×6 7 2
+ Oct 2 ×1  16 
  h;
 2
Odd days :2 + 2 + 3 + 2 + 3 + 3 + 2 + 5 ; 165. Let the number of chocolates that each of
Total odd days = 22 ⇒ 22/7 = 1 odd day. Pinky, Chinky and Tinky can eat in one 4 8 8 8
\ p 3 8 3 8 3 h = × π× × × .
hour be p, c and t respectively. \ p – t = 3 2 2 2
1 day after Wednesday is Thursday ;
t–c 4 1
Hence 12th October is Thursday.
240 \ h = =1 cm Choice (1)
 Choice (3) ⇒ 2t = p + c; p + t + c = = 30 3 3
8
Solutions for question 160:
171. Area of four walls – area of two doors – area
t + 2t = 30 ⇒ t = 10; 3c = 2p ⇒ p + c
160. At 8:45 the two hands of the given clock of window = 2 3 5 (12 + 8) - 2 3 2 3 1 - 1
= 20
will align as follows. 3 1 = 200 - 4 - 1 = 195 m

08_Section.indb 162 10/30/2009 11:54:21 AM


Solutions for Mock Tests  n  163

= 3 : 5 : 7 Choice (3)
\ Cost of white washing = 20 3 195 = - 120 = 45.  Choice (2)
Rs 3,900  Choice (4) 178. The required number of boxes 5. Let the cost incurred by the manufacturer
172. Let the principal be Rs x. \ x(1.1)2 – x 22 be Rs x.
× (8) (8) (14) \ x (1.2) (1.25) (1.3) = 390 ⇒ x (1.95)
x × 2 ×10 = 7 = 44  Choice (2)
– = 525 ⇒ x(0.01) = 525; \ x (4) (4) (4) = 390
100 ⇒ x = 200 Choice (1)
= 52500  Choice (3)
6. The required number of months
173. Let there be 10 sparrows and 18 pi- 179. The required height = 2880
geons. If 3 birds (all pigeons) escape, = =9
2× 2× 7 4
the ratio of sparrows to pigeons is 2 8000 ×
22 100
: 3. Let there be 11 sparrows and × 7× 7 − 2× 2× 7  Choice (1)
17 pigeons. If 3 birds (1 sparrow and 2 7
pigeons) escape, the ratio of sparrows 7. 1938 is not divisible by 54 and 37.
= 14/75 metres = 18 2/3 cm Choice (1)
to pigeons is again 2 : 3. But from this \ Ignore choices 1 & 2
data alone (number of birds escaping and 180. Let the distance of each equal stretch be
1938
ratio of the birds) we can not uniquely ‘x’ km Choice : 3 = 57.
Total dis tan ce 34
determine the initial ratio of the birds. Average speed =
Total time \ The two numbers are 34 and 57
 Choice (4)
(4 × 7 = 28 and 3 × 5 = 15) Clearly 34 is
174. Since the paper is folded along the length x+x+xx the smaller. Choice (3)
=
2pr = 44, \ r = 7 x x x x 8. Suppose Johar joined the business after x
+ + +
\ Volume of cylinder = pr2h = 22/7 3 10 20 30 60 months of the start of the business.
(7)2 3 10 \ 6 × 12 : 8 × (12 – x) = 3 : 2 ⇒ 6 = 12 – x
 
= 1540 cm2 Choice (4)  4x  ⇒ x = 6 Choice (2)
=   = 20 kmph Choice (2)
175. Speeds of Ramesh, Sameer and Tarun  12 x  9. The ratio of capacities = 1/24 : 1/8 : 1/12
 
180 m 240 m 300 m  60  = 3 : 4 : 6 that is, A does (say) 3x work (out
are , and that of total of 3x + 4x + 6x = 13x work) in 60
60 sec 60 sec 60 sec
days. ⇒ A does the full work that is 13x
is, 3 m/sec, 4 m/sec and 5 m/sec respec-
 13 
tively. Required time = L.C.M. (times MOCK TEST 2 in 60   = 260 days Choice (3)
3
Solutions for questions 1 to 20: 10. The required difference =
1. Let the ages of X, Y and Z be x, y and z 380 × (60 − 40 )
taken by any two pairs to meet) = L.C.M. years. Given x + y + z = 180. = 76 Choice (3)
(Time taken by Ramesh and Sameer to 100
10 years ago, sum of their ages = 180 - 3
meet, time taken by Sameer and Tarun × 10 = 150. 10 years ago, the age of X = 11. Let the number of hours taken by the man
1/6 × 150 = 25. be x.
 600 600 
to meet) = L.C.M.  , = \ The number of hours taken by the
4−3 5− 4 \ Present age of X = 25 + 10 = 35 years.
woman is x + 10
600 seconds. Choice (1)  Choice (2) 20 10 7
\ + = ⇒ 200x + 2000
176. The number of nuts produced var- x x + 10 10
ies directly with the time for which 2. Number of literates = 300 × 70 = 700 . + 100x
the machines work and the number of 30
machines which are working. The re- 35 = 7x2 + 70x ⇒ 7x2 – 230x – 2000 = 0
Number of female literates = 700 × = 245
100 ⇒ 7x2 – 280x + 50x – 2000 = 0
14 25
quired number of nuts = 544 3 ×  Choice (1) ⇒ (7x + 50) (x – 40) = 0; x = 40 or
8 10
= 2380 Choice (1) 3. Let the C.P of the article be Rs 100. Then −50
x=
list price = C.P + 20 per cent of C.P = 7
Rs 120 But x cannot be negative. \ x = 40
177. Number of man-hours required to com-
But the list price shown by the father = 20 The required time to complete the work
plete 9/20 of the work = 120 3 6 3 30 =
21600. per cent more on list price = Rs 144 3
= × (40 + 10) = 15 Choice (1)
Number of man hours required to As there is a profit of 0.8 per cent, S.P.= 10
complete the remaining work = Rs 100.8. Discount per cent = 43.2/144 12. Let the Initial sales be px. Price is re-
21600 × 100 = 30 per cent Choice (3) duced
× 11 = 26400.
p 3p
9 4. from y + z : y - x = 6 : 1 ⇒ y + z = 6y – 6x (or) to  p −  = and number of cars sold
Number of men required to complete in z = 5y – 6x  4 4
20 days each working 8 hours a day = from z + x : z - y = 5 : 1 3 13x
z + x = 5z – 5y ⇒ x = 4z – 5y ⇒ x = 4(5y =  x + x  =
 10  10
26400 3
= 165 – 6x) – 5y ⇒ x = y
20 × 8 5 3p 13x 39px
\ Total Sales = × = .
7 4 10 40
Additional number of men required = 165 Also, z = 5y – 6x = y; Hence, x : y : z
5

08_Section.indb 163 10/30/2009 11:54:22 AM


164  n  Solution Manual

F+5, K+5, P+5, U+5 Z is one series and


1 19. Average speed in kmph = 2x
\ Effect on Sales = px = 0.025px
x x
B+5, G+5, L+5, Q+5, V is one series.
40 + Hence, ruq is the missing term in the se-
decrease 2.5 per cent decreaseChoice (3) 7.5 10.5
ries. Choice (3)
13. Min. marks for pass = x. A’s marks = 2 2(7.5)(10.5) 28. After each term, one letter is missing and
0.90x, B’s marks = 8/9 of A = 8/9 of = = = 8.75
1 1 18 the terms are consecutive letters in the
0.90x +
7.5 10.5 decreasing order. The number of letters
C = 58 /17 per cent of (A +
14
is also decreasing. ZYXWVU, SRQPO,
Choice (1)
B) = 10/17 (0.90x + 8/9 of 0.90x) MLKJ, HGF, DC
10 × 0.90x  9 + 8  20 × 0.90x 17 
20. Weight of second vehicle = 250 kg. Hence, MLKJ is the missing term.
=  = × = 1x
17  9  19 9 Weight of first vehicle = 75 × 250 = 187.5  Choice (1)
= pass mark Choice (2) kg. 29. The given series is a combination series.
th
Weight of third vehicle = 1.36 × 250 = N+2, P+2, R+2, T+2, V is one series and Q-3,
1 1 340 kg. Weight of fourth vehicle = 2.88
14. P + Q fills in (1 + 1) hour =  +  of N-3, K-3, H-3, E is one series
3 4 × 250 = 720 kg The numbers are squares. 12, 22, 32, 42, 52
the tank fills part of tank filled in 2 hours 720 − 187.5 Hence, the next term is V25E.Choice (3)
Weight of fourth vehicle is =
7 187.5
= Solutions for questions 30 to 32:
12 × 100 per cent more than weight of first 30. Z, M, J, R and E are the letters; which
Remaining part after 2 hours = vehicle = 284 per cent are not immediately followed by a sym-
7 5  Choice (2) bol but immediately preceded by a digit.
1- =
12 12  Choice (2)
P fills for the 3rd hour, so remaining part Solutions for questions 21 to 24: 31. 10th element from the right end is = D
Given that, Rina is between Ooha and Mona. So, 5th element to its left is = l
5 1 1
after 3 hours = − = Q fills this all the three will be in the same row and as Rina 9th element to the right of l = U.
12 3 12 is in the same row as Nina, we can say that Nina  Choice (3)
1 1 1 is also in the same row as the above three and
work in 4 × = hour = × 60 = 20 32. When the second half of the sequence is
12 3 3 Nina will be at one of the ends.
reversed, the new sequence is as follow
minutes \ Tina is opposite to Rina. C 8 Z 5 M A N # P per cent 9 J Z $ 2 3 R
Total time = 2 + 1 + 20min = 3 hr 20 min ___ Tina Uma ___ HFE1LWVUTD6JIlf4
= 31/3 hrs Now, 13th element from the right end
 Choice (2) Rina Nina is = L
15. From the data given in the problem, pipe As Mona is to the left of the person who is op- 8th element to its right is = 6
A was opened for 3x minutes, pipe B for posite to Sona. 6th element to the left of 6 is = V.
2x minutes and C for x minutes. Prem Tina Uma Sona  Choice (2)
3x 2x x Solutions for questions 33 to 35:
\ + + = 1 ⇒ x = 3 Choice (3) Ooha Rina Mona Nina 33. I is definitely strong, because it shows
20 15 20
21. The other persons in the same row as the percentage of children who are being
16. As we do not know the sum we cannot Uma are Prem, Tina and Sona.Choice (2) abused. Hence, there is a need to bring
find the answer. bill on this.
 Choice (4) 22. Prem and Nina are diagonally opposite to So, I is strong.
each other. Choice (3) II is not strong, because this is not the
17. Let the cost price of an article be
Rs x; 23. The person between Uma and Prem is only way to impart discipline among chil-
20 Tina. Choice (4) dren.
x× = 140; x = Rs 700 So, II is not strong. Choice (1)
100 24. If the positions of Rina and Nina are
Discount = 30 per cent. Marked price = interchanged, the person opposite to Rina 34. As the English language is the key and
is Sona. Choice (3) Mumbai is much better than its rivals,
SP × 100 840 × 100 there is every chance of Mumbai being
= = Rs 1200 Solutions for questions 25 to 29:
100 − D 70 the financial hub.
 Choice (3) 25. The given series is a combination series. So, I is strong.
18. Work done by A and B in 1 day = C+1, D+1, E+1, F+1, G is one series, Population is a hurdle for a financial hub.
S-2, Q-2, O-2, M-2, K is one series and So, II is not strong. Choice (1)
1 1 11
+ = P-2, N-2, L-2, J-2, H is one series.
10 12 60 35. I is not strong. Though the individual is
Hence, GKH is the next term. Choice (2)
responsible, some malpractices may take
 ork done by them in 4 days = 4 × 11/60
W
26. The given series is a combination series. place. So, the RBI and the government
= 11/15th part. Remaining work = 4/15.
L+2, N+2, P+2, R is one series, should intervene.
This work is finished by B in
H+2, J+2, L+2, N is one series and So I is not strong.
4 1 C+2, E+2, G+2, I is one series. II is not strong, because fall in senses
× 12 = 3 days. \ Work is com-
15 5 HENCE, RNI is the next term.Choice (4) does not necessarily imply that the stock
27. The given series is a combination series. market was a gambling den.
pleted in 4 + 31/5 = 71/5 days.
C+5, H+5, M+5, R+5 W is one series,  Choice (4)
 Choice (1)

08_Section.indb 164 10/30/2009 11:54:23 AM


Solutions for Mock Tests  n  165

Solutions for questions 36 to 40: 47. When we refer to opinion that is divided uses the expression ‘no more than’. The
36. The events are related events. Chronolog- into two distinct groups we say it is ‘po- sentence talks about the collapse of intel-
larised’. ‘Bisected’ is to divide into two ligence, which is by no means an insignifi-
ically (B) occurs after (A) and (A) is the
halves, ‘segmented’ is divided into sev- cant event. Hence the correct expression is
immediate and principal cause for (B).
eral pieces. Choice (3) ‘no less than’. This expression is used to
 Choice (2)
emphasize the importance of something.
37. (A) and (B) are related events. Chrono- 48. ‘Impersonation’ is when we pretend to
 Choice (3)
logically (B) occurs before (A). (A) is be some one else. ‘Slapstick’ is rough
and foolish comedy. “Lampoon” is strong 55. Option 1 is erroneous in the use of ‘food
the further action taken by the residents
criticism in a humorous way. ‘Burlesque’ stock’ instead of ‘food stocks’. We say
which is immediate.
is the appropriate replacement because it something arises ‘from’ something and not
 Choice (1) means a funny and exaggerated imitation. ‘through’ which makes option 2 erroneous.
 Choice (4) Option 3 is erroneous because of the use of
38. (A) and (B) are related events. Chrono-
‘if’ after the word ‘question’. ‘If’ is used to
logically event (A) occurs before (B) but Solutions for questions 49 to 52: introduce a condition while ‘whether’ ques-
event (B) is not immediate because only
49. Part 4 is erroneous as ‘in a day or two tions the existence or otherwise of a par-
decision cannot lead to a strike, those
days’ is an incorrect expression. The cor- ticular situation and should be used in this
need to be implemented first.
rection is ‘in a day or two’. Choice (4) context. Option 4 is correct. Choice (4)
 Choice (4)
50. Part 4 has an error. The correction is ‘…….. 56. Choice (2) should read “consumers’” or
39. (A) and (B) are related events. Chrono- too important to be left to chance’. Because ‘the consumer’s’, to make it a general
logically (A) occurs before (B). Death of statement. Choices (3) and (4) have tense
‘too …to’ is the correct conjunction.
more than 100 children is the immediate errors. The correct usage here would be
 Choice (4)
and principal cause for forming the inves- ‘brands have gone’. Choice (1)
tigation team. Choice (2) 51. Part 4 is erroneous because the verb ‘has’
does not agree with the subject ‘inci- Solutions for questions 57 to 59:
40. Accident is the immediate and principal 57. C is ideally the introductory statement as
dents’. The correction is ‘….which have
cause for the fractured limb and hospitali- it introduces us to the topic of the para that
occurred several decades ago’.Choice (4)
sation. Choice (2) is ‘homesickness’ and states that a home
52. The error lies in part 2. The verb ‘have’ is not the same as a house. Statement A
Solutions for questions 41 to 44:
does not agree with the singular subject which goes on to describe what a home
41. ‘Evinced’ is the right word in the blank ‘one’. The correct verb to be used here is, ideally follows C. Statement D follows
because it means ‘to show’. ‘Evaded’ is to is ‘has’. Choice (2) A, which says that home is an experience
avoid and ‘evoked’ is ‘to inspire’.Choice of the heart that includes an attachment to
(3) Solutions for questions 53 to 56:
a house, family memory etc, by carrying
42. ‘To bask in reflected glory’ is the correct 53. In statements ‘B’ and ‘D’ we find the cor- the idea forward. B sums up the para by
expression which means ‘to lie back and relative conjunction “not only ….. but saying that the heart always needs to be at
derive pleasure from.’ also” being used. The usage is correct in home. Hence CADB. Choice (2)
 Choice (1) statement ‘B’ but because of the words
58. All the options begin with B. But only
“making their people to” it is incorrect.
43. ‘Hammered’ is to be ‘badgered with’. statement D can follow B because it
that is “make” does not take “to” (verbs
Only ‘hampered’ which means ‘to prevent complements statement B, which says
like make, have, let …. do not take ‘to’).
free movement’ fits the blank. Choice (2) that Terrorism has a centre in the training
Hence choice (2) is ruled out. In state-
camps and battlefields of Pakistan and
44. ‘Wrest’ is to ‘draw’ or ‘take by force’, ment ‘D’, “offer not only …..” is the error.
Afghanistan, by stating that the Taliban
while ‘wrestle’ is to fight. The word ‘diffi- When a correlative conjunction is used in
and the al Qaeda, were formed ‘there’ that
cult’ in the sentence indicates that ‘wrest’ a sentence, each word of the correlative
is Pakistan and Afghanistan. A follows D
is the only word that suits the blank. One conjunction pair must be followed by as it carries the idea forward by talking
can ‘wriggle’ out of a difficult situation the words belonging to the same parts of about the circumstances which favoured
but a confession cannot be ‘wriggled’ out speech or by words belonging to the same terrorism in these two countries. C con-
of someone. Choice (3) grammatical form. Again in ‘D’ also we cludes the para by stating how terrorism
find the words “making their people to” spread to different parts of the globe.
Solutions for questions 45 to 48: which is erroneous. Hence choice (4) is Hence BDAC is the correct sequence.
45. ‘Unable to see eye to eye’ means to ruled out. In ‘C’ and ‘D’ “attainment of  Choice (3)
disagree on many things or to be ‘incom- those visions” is erroneous because in
the first part of the sentence, we find a 59. ‘B’ sets the tone of the passage. ‘D’ is
patible’. The words ‘incongruous’ (inap-
reference to ‘a strong vision’ (singular). followed by ‘A’, and then ‘C’ makes this
propriate), ‘insouciant’ (nonchalant) and
Hence choice (3) is ruled out. Further in paragraph a distinct unit in prose writing.
‘incorrigible’ (irredeemable) are inappro- ‘C’ cannot follow D immediately, neither
priate. Choice (1) ‘C’ “harness their energies for“ is errone-
can ‘A’ follow ‘B’. Choice (3)
ous. ‘Harness’ takes the prepositions to/
46. When something decreases suddenly and towards. Choice (1)
in large amounts, we say it ‘plummets’. Solutions for question 60:
‘Plunged’ is to jump or dive quickly and 54. Statements A and D are incorrect because, 60. Statement B follows ‘1’ because “These”
energetically. We use ‘plonk’ when we ‘the power to see things really’ is unclear in ‘B’ refers to the “cultural police” in 1.
put or drop something heavily and care- and awkward. It can be better written as Further “They” in D also refers to the cul-
lessly. ‘Fall’ is used with shares but does ‘the power to see things as they really tural police and the word “dressing” finds
not denote large amounts. Choice (3) are’. Statement B is incorrect because, it a continuation in ‘A’. that is, the word

08_Section.indb 165 10/30/2009 11:54:23 AM


166  n  Solution Manual

“costume” in ‘C’ is a continuation of the The amount spent on entertainment and


2300
idea expressed in A. Hence BDA go to- × 100 = × 100 ≃38.4 per cent ≃38 22
gether and is followed by C. Choice (2) 5990 health in 2004 = × 80000 = Rs
per cent Choice (1) 17600 100
Solutions for questions 61 to 63: Solutions for questions 68 to 73: The amount spent on entertainment,
61. Observing the markets 0.8 from ACs is 32
20 − 18 health and others in 2001 = × 50000
the largest segment and Hitachi having 50 68. The required percenta0ge = × 100 100
per cent share in that segment. 18 = Rs 16,000
\ Definitely Hitachi controlled the high- = 11×1 per cent Choice (2)
 Choice (2)
est share of the AC market in 2000-01.
30 − 8 190 − 68
\ The required percentage share 69. As > , the correct option Solutions for questions 78 to 80:
8 68
1 3 15
244000 × + 56000 × + 76000 × is (1) Choice (1) 40 120 30 130 90 120
= 2 10 100 78. Of , , , and , is
244000 + 56000 + 76000 70. Clearly, if the number of degree students 200 240 360 390 260 240
× 100 or non degree students increases, the total the greatest  Choice (1)
numbers of students also increase.
= 39.9 per cent Choice (1) 79. Average of sales of the company
\ The increase in the total number of stu-
62. We cannot calculate percentage change dents is the highest.  Choice (3)  200 + 240 + 360 + 390 + 260 + 350  7
in the market share of kelvinator as we
=  × 10
 6 
do not know its share in 0.8 tonne and 18 20 22 25 28 = Rs 300 crore
1.0 tonne ACs. The market share of LG 71. Of , , , , and and
112 124 132 148 168 In 2000, 2001 and 2003, the sales of the
in 2000-01 Hence we cannot determine as company is greater than Rs 300 crore.
to which type of A/c registered the high- 25
is the greatest. Choice (3)  Choice (3)
est percentage increase. Choice (4) 148
63. Hitachi’s market in 1.0 tonne ACs catego- 190 − 68 360
30 38 72. The required percentage = × 100 80. The required percentage = ×
ry = 56000 × + 74000 × = 44920 68 100 1800 − 360
100 100 = 179×4 per cent. Choice (2)
= 25 per cent Choice (2)
Hitachi’s market in 0.8 tonne ACs cate-
50 40 73. Of 85 − 72 , 112 − 92 , 168 − 148 and 190 − 168 ,
gory = 244000 × + 368000 × 72 92 148 168 Solutions for questions 101 to 120:
100 100
= 269200 112 − 92 101. Total selling price
is the greatest. Choice (2)
\ The required percentage is = 92
 75 150 25 130 
269200 − 44920 =  × + ×  × 80000
× 100 = 499.3 per cent Solutions for questions 74 to 77:  100 100 100 100 
44920
 Choice (3) 74. The amount spent on cloth in 2004 = = 90000 + 26000
20 \ The gain = 116000 - 80000 =
Solutions for questions 64 to 67: 80,000 × = Rs 16,000 which is
100 Rs 36000. Choice (1)
64. The required percentage increase equal to 160 per cent of Rs 10,000 and 102. Distance covered by X in 50 seconds =
=
(420 + 530 + 640 )− (320 + 350 + 380 ) is the amount spent on rent and health to- 450 m. Speed of X = 450/50 = 9 m/s.
320 + 350 + 380 gether in 2001. Choice (2) Distance covered by Y in 50 seconds =
350 m.
× 100 ≃ 51.4 per cent Choice (2) 75. The amount spent on clothes in 2001 \ Speed of Y = 350/50 = 7 m/s
65. The maximum value of total sales comes 18 Time taken by A and B to meet for the
= 50000 × = Rs 9000
from division III for each company in 100 first time in the same direction = 900/2 =
2002. 450 seconds. Choice (4)
 hich is 150 per cent of Rs 6000 and is
W
210 310 330 the amount spent on transport in 2001. 103. Let the speed of A be x m/sec. Speeds of
Of600 × 100, 620 × 100, × 100,
310  Choice (3) B and C are 2x m/sec and x m/sec respec-
540
× 100 and750 × 100, 870 × 100
1500 1680 750 76. The total expenditure spent on food tively.

is the maximum which is equal to 41.3 20 22 Let the lengths of A and C be a m and c
= 50000 × + 80000 ×
per cent Choice (3) 100 100 m respectively.
= Rs 27600
a+c
27600 Length of B = m.
540 − 390 750 − 540 870 − 720 1500 −1050 \ The required fraction = 2
66. Of , , , 130000
390 540 720 1050
= 0.21 Choice (3) a + 100 a 100
1680 −1440 1500 −1050 = 20 ⇒ + = 20 ;
and , is the  x x x
1440 1050 77. The amount spent on clothes in 2004 =
20 c + 100 c 100
highest. Choice (4) 80000 × = Rs 16,000. = 30 ⇒ + = 30
100 x x x
67. The required percentage
The amount spent on transport and food a+c
+ 100
300 + 360 + 350 + 640 + 650 32
= in 2001 = × 50,000 = Rs 16,000  equired time = 2
R
750 + 900 + 950 + 1590 + 1800 100 2x

08_Section.indb 166 10/30/2009 11:54:24 AM


© 100 100 100 100 ¹ x  31y
y u 31. ? The average cost = of vessels needed for the three varieties of oil is
= 90000 + 26000 z 20. Choice (2) Odds in fav
? The gain = 116000  80000 = Rs 36000. Choice (2)
Choice (1) 113. The curved surface area of the conical tent =
109. O is the point of intersection of the diagonals The area of cloth required.
102. Distance covered by X in 50 seconds = 450 m. Given BD = 12 ? S u 8 u 82  152 = 6 u length Ÿ The length 120. Probability
Speed of X = 450/50 = 9 m/s. C
Ÿ BO = 6 4
Distance covered by Y in 50 seconds = 350 m. 22 uSolutions
8 u17 for m n  167
Mock Tests  Choice = . Prob
‘OAB = 30° = = 71.24 (1) 10
? Speed of Y = 350/50 = 7 m/s 30° 7u6
? AO = 6 3 (OB tan 60°) 5
Time taken by A and B to meet for the first time O ball =
A
 rea of the
Area of the RhombusRhombus ABCDD = 4 B TwoFirst
115.
(Area of 114. 3 prizes
particular from
girls 15 prizes
occupy cantwo
middle be places,
given 10
a + c + 200 1 a 100 c 100 
= = 900/2 = =450 seconds.
in the same direction + + +  230° 30°
therefore the remaining
to a particular girls in
student can
15 be arranged in
C3 ways. Re- Probability
4x 4  x Choice
x (4)
x x  ABCDD =AOB)
4 (Area of ' AOB)
= 4 (1/2) (6) (6 3 ) cm remaining eight places. that is, 8! ways and two
girlsmaining 12 prizes to the remaining
in 2 ways. 5
1
103. Let the speed of A be x m/sec. Speeds of B and = 72(6)3(6cm²3 ) cm2
= 4 (1/2) Choice (1) in the middle can be arranged ball = .
? Allstudents 512 ways. toNumber
10 girlsinaccording of ways can
given condition of 10
C are 2x m/sec and 1 A
= x m/sec respectively.
(20 + 30) = 12.5 seconds.Choice (2) = 72 3 cm² distributing = 15C × 512. Total number of
be arranged is 8! u 2! ways.
3 Choice (1)
Let the lengths of A4 and C be a m and c m 12 × Avg wt. of 12 persons + ( P − Choice50) (1) ways = 615 ? Require
respectively. 110. 115. First 3 prizes from 15 prizes15can C3. 512
be given to a
12 Required
particular probability
student in C3 =ways.15 Remaining 12
15

a  c x − x = 10 ( 5 - (-5) = 10 min)
104. 12 u =Avg
Avg.wt.
wt. of 12persons
of 12 persons ( P  P50)
–0.5.Where is the 6
prizes to the remaining 5 students in 5 ways.
12
Length of B = m. 16 60
15 110. = Avg. Number of ways of distributing = Choice
15
C3 u (4)
12
5 . Solutions for q
2 weight of the new 12person. Total number of ways = 6
15
⇒ x = 40 km wt. of 12 persons –0.5. Where P is the weight of the 116. C – 1, 0 – 2, M – 1, P – 1, E – 1, T – 2, I – 2, 121. The passa
a  100  a 100 c  100 Choice (2) P – 50 = –12 × 0.5 ⇒ P = 50 – 12 × 0.5 = N–1
15
C3. 512 listing it ou
20 Ÿ  20 ; 30 new person. Required probability = Choice (4)
x x thex speed of A bex S m/sec. Let its
105. Let 44 kg. Choice (2)
P – 50 = –12 u 0.5 Ÿ P = 50 – 12 u 0.5 = 44 kg. R
 equired number 615 of arrangements 122. Refer to th
c 100 111. We can choose any 2 points from side AB 116.
Choice (2) C – 1, 0 – 2, M – 1, P – 1, E – 1,7!5! T – 2, I – 2,7!5!
N–1
Ÿ  30 L (Vowelsnumber
together)
x x length be L m. = 40 ⇒ L = 40S and a point from either AC or BC. The Required of are
arrangements
2! 2! 2!
= (Vowels
(2!)3 123. Para 3, en
S 111. We can choose any 2 points from side
number of triangles in this case would be AB and a 7! 5! 7! 5! in tune wi
a  c L + 200 point 3Cfrom  Choice
Choice(4)
× 9Ceither
= 27. AC or BC.3 The 9 number of
together) are = (4)
Required time2=  100
movement
2 1 2! 2! 2! (2!)3
Required time = S+5 = triangles in this case would be C2 u C1 = 27. 117. Let AB and CD represent the two towers.
117. Let AB and CD represent the two towers. 124. Para 2 m
2x A C means ma
40S + 200 40(S + 5) A
a  c  200 1 § a= 100 c =100 · = 40 seconds living being
¨  S +5  S ¸+ 5 x x
4x 4 © x x x x ¹ Choice (1) x 125. Cherrapun
x x
h2 wettest pla
1 x h1 maximum r
= (20  30) 106. Toseconds.
= 12.5 gain 40 per cent, the quantity
Choice (2) of wa- x x
4 ter to be added is 40 per cent of the pure 45° 30° 126. Refer to t
x x 10 milk. Hence ratio of water and milk in the x x x x
B C B X D
104.  (' 5mixture
 (5) is
= 10 min)
2 : 5.  Ÿ x = 40 kmChoice (2)
15 16 60 Choosing 2 points from BC and a third from h2 h1 1 127. Para 3 of t
C
 hoosing 2 points from
other remaining points are C2 u C1 = 48. BC4 and a 8third tan45° = Ÿ h 2 = BX. tan30° = the answer
Choice (2) 100 BX h DX 3
Finallyfrom the other
otherremaining points are5CC2 2u×7C C11 = 70.
4 8
107. Ratio of speeds of A and B = possibility gives tan45° = 2 ⇒ h2 = BX. tan30° =
DX BX 128. According
105. Let the speed of A be S m/sec. Let its length be100 − 10 = 48. Finally the other possibility
Now, a triangle can be formed using one point Ÿ h1 = gives
10 C2 × 7ofC1 = 70. Now, a triangle
AC. can be the h1 3 1
5
L = from each sides AB, BC, Hence,
40 Ÿ L = 40S 9 =
L m. C1 u C1 u C1 = 60. Hence, ratio 129. End of para
formed using one 3 from4each of 5sides
number of ways arepoint DX 3
S AB, BC, AC. Hence, the number of ways h1 DX / 3 1
If A started x m before the start line, Hence, the total triangles are 27 + 48 + 70 + 60 = = (' DX BX) Ÿ h1 : h2 = 130. Para 4; (lin
L  200 = 205.are 3C1 × 4C1 × 5C1 = 60. Hence, theChoice total (2) h 2⇒ h =BXDX 3
Required timedistance= to be covered by = him = 1 131. Only statem
S5 triangles are 27 + 48 + 70 + 60 = 205. 3
(100 + x) m. 112. Since each vessel has to be full of oil and the 1 : 3 Choice (2) lines. State
40S  200 40(S Distance
 5) to be covered by B = 100 m.  Choice (2) h DX / 3 1 is true – P
entire quantity of oil (in litres) of each variety 118. has Every year the
Hence, ratiotree
= grows
1
= by 21 per=cent is true – Pa
= 40 seconds h BX
S5 S5 A
 s A would be beaten by B in this case, 112.
to beSince each
stored, vessel
the has
capacityto beoffull of
each oil and
vessel (in ? Every six months, 2 it grows by 10 3 cent to
per
distance covered by BChoice
when A(1)finished = litres)the hasentire
to bequantity
a factor of of
oil95,
(in 133
litres)and 152.
of each 110 per cent (or to 1.1 its initial size) 132. Para 8 sa
3
After(∵ 3 DX
periods
= BX) of six⇒months, 1 : 3 to (1.1) =
h : h2 =it grows been succe
9 variety has to be stored, the capacity of
(100 + x) > 100 each vessel (in litres) has to be a factor Sol/977
of
1.331 times its initial size.1 Therefore,
height
1
as the initial and electric
10  is 100 ft, after 1 /2 years it is 133.1 ft. (2)
Choice
95, 133 and 152. Choice (3) 133. Refer to th
x > 11 /9
1
Choice (4) As we need the least number of vessels, 118. Every year the tree grows by 21 per cent ‘transitiona

108. Total fixed cost = Rs x. Total variable the capacity has to be the greatest com- \ Every six months, it grows by 10 per
mon factor, that is, 19. The number of cent to 110 per cent (or to 1.1 its initial
cost = Rs y × 31. \ The average cost vessels needed for the three varieties size)
is 95/19, 133/19 and 152/19, that is, 5, After 3 periods of six months, it grows to
x + 31y (1.1)3 = 1.331 times its initial size. There-
= 7 and 8 respectively. The total number
z of vessels needed for the three varieties fore, as the initial height is 100 ft, after 11/2
 Choice (2)
of oil is 20. Choice (2) years it is 133.1 ft.
109. O is the point of intersection of the diago-  Choice (3)
nals 113. The curved surface area of the conical
Given BD = 12 tent = The area of cloth required. 6
119. Probability of picking a black ball =
⇒ BO = 6 2 2
\ p × 8 × 8 + 15 = 6 × length ⇒ The 9
∠OAB = 30° 2 2 1
22 × 8 ×17 = ; P (B) = 1 − = ;
length = = 71.24 mChoice (1)
C 7 × 6 3 3 3
2 1
114. Two particular girls occupy middle two Odds in favour of black = : =2:1
30° 3 3
O places, therefore the remaining girls can
D B  Choice (2)
be arranged in remaining eight places.
30° 30° that is, 8! ways and two girls in the mid- 120. Probability of picking a prime-numbered
dle can be arranged in 2 ways. 4
A \ All 10 girls according to given con- ball = . Probability of picking an even-
Choice (1)
10
dition can be arranged is 8! × 2! ways. 5
 Choice (1) numbered ball =
\ AO = 6 3 (OB tan 60°) 10

08_Section.indb 167 10/30/2009 11:54:25 AM


168  n  Solution Manual

Probability of picking an even-prime been to produce Wang Wei Lin alive. 565
Their failure to do this suggests that he 144. Average exports = ≃ 47 crore. Ex-
1 12
numbered ball = . was indeed executed. Choice 2 states that
10 ports are less than average in 6 months.
the government punishes its opponents;
\ Required probability =  Choice (2)
it does not state that it executes them.
Therefore this may not lend credence to Solutions for questions 145 to 149:
4 5 1 8 4
+ − = =  Choice (1) the conjecture. Choice (3) presents a to- 145. Total number of spoiled apples with Ven-
10 10 10 10 5
tally different situation. Whereas, in the 20
Solutions for questions 121 to 136: situation mentioned in choice (3), the dor A = 200 × = 40; Vendor B = 250
100
Chinese government chose to ignore in- 30
121. The passage gives the beliefs of Deep × = 75
ternational condemnation, in the Tianan- 100
Ecology, listing it out as one, two, three.
men square episode, they did their best 15
 Choice (2) Vendor C = 160 × = 24;
to appease their critics. Thus, choice (3)
100
122. Refer to the first and last sentences of para 2. does not present an analogous situation,
 Choice (1) and therefore, it neither strengthens, nor 16.66
Vendor D = 300 × = 50;
weakens the conjecture. One cannot ar- 100
123. Para 3, end gives Gary Snyder’s feel-
ing that is in tune with the proponents of rive at a conclusion based on choice 4. 10
 Choice (1) Vendor E = 180 × = 18
Deep Ecology movement. Choice (3) 100
124. Para 2 makes it clear that the given phrase 138. The given para assumes the price of gold total number of apples spoiled
means man is not considered superior to is determined by demand and supply = 40 + 75 + 24 + 50 + 18 = 207
other living beings. Choice (1) within the country. Option 1 points out  Choice (4)
that gold price is determined at the inter-
125. Cherrapunjee is famous because it is the national level. Choice (1) 146. Weight of oranges with vendor A = 150 ×
wettest place on the earth as it receives 60 = 9000 gms;
139. The butter made from the milk of Hol- vendor B = 100 × 75 = 7500 gms
the maximum rainfall. Choice (3)
stein cows is paler means that the amount
vendor C = 80 × 50 = 4000 gms;
126. Refer to the penultimate sentence of para of colorless Vitamin A must be higher.
2. Choice (4) vendor D = 200 × 100 = 20000 gms;
Therefore, Choice 2 is the best answer.
vendor E = 240 × 90 = 21600 gms.
127. Para 3 of the passage confirms that choice Choice 1 cannot be concluded from the
Total weight of all oranges = 9 + 7.5 + 4 +
4 is the answer. Choice (4) passage as nothing is mentioned about the
amount of butterfat in the milk. Choice 3 20 + 21.6 = 62.1 kgs Choice (3)
128. According to para 4, choice 2 is the answer. and 4 are contrary to what is mentioned 147. Unspoiled Bananas with vendors: A =
 Choice (2) in the facts mentioned in the passage.
90 80
129. End of para 5 (the last three lines)  Choice (2) 600 × = 540; B = 800 × = 640;
 Choice (2) 100 100
140. Since POTA was introduced terrorist re-
130. Para 4; (lines 1 to 5) Choice (2) lated to crimes have increased. Therefore 65
C = 1000 × = 650;
POTA is ineffective Terrorists could have 100
131. Only statement 1 is not true. Para 9 – first
increased because of other factors too. 83.33
three lines. Statement 2 is true – Para 7. D = 1200 × = 1000; E = 900 ×
POTA need not necessarily be ineffec-
Statement 3 is true – Para 10, (first three 100
tive. This is best brought out by option 3.
lines). Statement 4 is true – Para 7 (lines
Option 2 only takes the opinion of people 80
2, 3 and 4) Choice (1) = 720. Vendor D has the highest
which might not be a correct judgement. 100
132. Para 8 says that the regulators seem to  Choice (3) number of unspoiled bananas.
have been successful in telecom market,
Solutions for questions 141 to 144:
gas industry and electricity companies. Alternate method:
 Choice (2) 141. Total exports in 2006 = 565 crore. Total By observation, we can say that vendor
133. Refer to the first 3 lines of the last para. D has the maximum number of unspoiled
exports in 2005 = 495 crore. Required
The ‘transitional instrument’ is ‘our rea- bananas as he is having the maximum
70
soning power’. Choice (3) per cent = × 100 = 14.14 Choice (4) number of bananas and the spoiled per-
495 centage is minimum. Choice (1)
134. Line 4, para 6 shows A to be true. The first 142. When the exports of both the years
sentence of last para shows B to be true. are considered, they were highest in 148. Unspoiled oranges of vendor E = 240
Line 4, para 1 shows C to be true. Para 2 the month of October that is 130 crore. 75
shows D to be true. Choice (4)  Choice (3) × = 180  (A)
100
135. The phrase “a hidden illimitable con- 143. Total duty paid in 2005-06 = 495 crore Spoiled bananas of vendor C = 1000
sciousness” in lines 2 and 3 of para 1
1 35
shows that option 1 is the right choice × = Rs 495 lakh. Total duty paid in × = 350  (B)
 Choice (1) 100 100
2006-07 \ A is 170 less than B.
136. Refer to the last two lines of the passage.
 Choice (3) 1 170 17
= Rs 565 crore × = Rs 565 lakh. Re- \ × 100 = × 100
100 350 35
Solutions for questions 137 to 140:
137. Choice 1 is the best answer. The Chinese quired difference = Rs 70 lakh 5 0 per cent of 35 is 17.5, so required an-
government sought to appease its critics. swer must be very close to 50 per cent but
The best way to achieve this would have  Choice (2) less than 50 per cent. Choice (4)

08_Section.indb 168 10/30/2009 11:54:26 AM


previous year to year x and hence we can find
previous 20 year th to year x and hence we can find
th
June of year x. ? Statement , alone is
20 Junesufficient. of year x. ? Statement , alone is Choice (2)
sufficient. Choice (2)
Solutions for questions 181 to 185: The
Solutions for questions 181 to 185: 5 km
Sout
Given, Given,
previous year to year x and hence we can find
Solutions
th
20 June for
of year Mock Tests 
x. ?ofStatement n   169
, alone isof G. G is the
(i) E is(i) the
previous
E is the
mother of mother
D, who
sufficient.year to year x and henceChoice is D, who
father of G.is father
G is
we can the
(2) find Solutions
brother thof brother
A. of of A. The fin
149. Weight of oranges A, B, C = 9 + 7.5 + 4 Solutions for questions 156 to 160: 20 June st year x. ? Statement , alone is
Solutionsst
Ÿ 1 generation
for
generation
Ÿ 1sufficient. ............ E(F)............ E(F) Choice (2) When
questions 181 to 185: 5 kmac
= 20.5 kgs 156. a3 b2 – a2 b3 = a2b2 (a –b). From statement South-W
sma
Given, T
Weight of Bananas with Vendor A = 600 I, a2 > b2 ⇒ when a and b are positive then Solutions (i) 2nd for mother
questions
E generation...........
is the of D, who 181 tofather
185: of G. G is the (1) the5
D(M)is
2nd generation........... D(M) Solutions fo
×x 60 = 36 kgs; Vendor B = 800 × 85 = a>b brother of A. areSp
68 kgs Given,Ÿ 1st generation ............ E(F) (2) athe
When cube c
When a and b are negative then a < b so a (i) 3Erd isgeneration the mother of D, who
rd ...........G is father of G. G is the
Vendor C = 1000 × 40 = 40 kgs; Vendor D 3 generation (M) ...........G A( ) A Solutio
smaller
corn
> b or a < b. If a > b, then a b – a b is
3 2 2 3
brother of A. (M)
(1)( )
(3) thethe eigc
= 1200 × 38 = 45.6 kgs; Vendor E = 900 × 2stnd generation........... D(M)
positive. (ii) (ii)
F isŸ Ftheis1 the mother
generation
mother of of
H,............
H,B Bisisthe E(F)mother
the mother of
of A. But
A. are
When paia
sides
50 = 45 kgs we do
(ii) F is the mother of H, B is the mother of A. But
(2) the cub
If a < b, then a3 b2 – a2b3 is negative. Hence But3not we know which generation they belong
rd do not know which generation they
we do not...........G
generation know (M)which generation A( ) they(4) belongall sm
th
Total weight of bananas = 234.6 kgs. to. corners
(1) thp
statement A alone is not sufficient. belong2ndto. generation...........
to. D(M) (3) thenot cub
20.5 Ÿ FF(F)is the mother B
(ii) of(F)H, B is the mother of A. But In the a
give
\ Required per cent = × 100 = From statement II, a3 > b3 ⇒ a > b. Hence sides a
234.6 we rd doŸnotFknow (F) which generation B(F) they belong (2) =th1
(4) all5the
a3b2 – a2b3 is always positive. to.3 generation ...........G(M) A( ) co
8.7 per cent @ 9 per cent. Choice (4) not pain
Thus the question can be answered using HŸis F B(F) 187. (3)Eighth
(ii) F (F)
the mother Aof H, B is the mother of A. But In the givensiq
Solutions for questions 150 to 153: statement II, alone.  Choice (1) H A 5 = 125
(iii) H iswe thedo not of
sister know
C and which C is generation
not the brother theyofbelong
H. (4) a
157. If x is the total mixture, then 2x/3 is milk from (iii) So (iii)to.
CHand is HtheHH is are the
sister sisters.
C Aandof
ofsister CC andthe
is not C brother
is not the brother 187.
188.ofEight
H.nc
Num
Total students = 110 Girls
the main statement. From statement I, we get Ÿ CŸ
(iii) of
H H.
(F) isFthe So sister
So
(F) CCand Hof(F)
and HCH are B(F)
are
and sisters.
Csisters.
is not the brother of H. In 3
the
= 5g
M P M P (60 × 2/3) = (60 + y)1/3, where y is the(iv) I is brother ⇒So C Ÿ and of HD.
C(F)are sisters. 5
nd C(F) H(F) H(F) 188. Numbe
b water added. Hence, y can be determined. Ÿ 2IŸ
(iv) isGeneration:
C
brother of D.IH(M) D(M) 189.3 =Num
(iv) I
(F)
is brother of
(F)
D. 187.
54.
42 20 a c d 20 e
From statement II, we know that water in (iv) I is H
Now⇒combining
brotherndof D.
2nd Generation: A
I(M) oneEo
Ÿ 2ndŸ Generation:
all statements,
2 Generation: I(M) I(M)BDD must
(M) be wifeD(M) 189. Numbe
the initial mixture is 20 litres and hence, (iii) of D, HNow iswhich
the sister
combiningimpliesof all
C Fand C is not
statements,
(F) must be
(M)
Bthemust
wifebrother
be
of I(M) of H. 190. Num
one of i
milk should be 40 litres. Now 60 litres (BecauseSoNow
wife C of Now
combining
andAD,and combining
Hwhich
areC all
sisters.
are
implies all
F(F)statements,
statements,
cousins, B must
they
must be wife
bebelong
wife Btomust be188. wife N
faces
Boys rd of D, which implies F(F) must be wife of Ibe
of water is to be added to make the ratio Ÿ
3 generation).
of C
I of
(F) D,
(Because which
AH and
(F) implies
C are F
cousins,
(F) must
they (M) wife
190. of I(M)3
Numbe
more
(Because A and C are cousins, they belong to
(M)
faces =
M P of milk and water equal to 1 : 2. Hence, (iv) Ibelong isrd brother to 3rdofgeneration).
(Because
3 generation).
D. A and C are cousins, they belong =to36
,. Ÿ 2nd3Generation:
rd E(F)I(M) D(M) more fa
statement II alone is sufficient.Choice (3) generation). 189. N
32 10 f g ,. E(F) = 36 +
158. From statement I, the number can be ei-,,. F ,. Now combining all statements, E B must be wife Solutions on
(F)
of œ D, I(M) implies D
which F (M) must œ
(F)
be B (F)
wife of I Solutions fo
ther a square of 45 or of 75, as both the ,,. F(F) œ
(F)
œ B(F)
(M)
191. 190.
Sum N
numbers end in five and the sum of the (Because A Iand (M) C are Dcousins,
(M) they belong to fa
42 = c + 32 ⇒ c = 10; 20 = d + 10 ⇒ d = rd
3 generation).
191. Sumaymoth
10; e = 20 per cent of girls ⇒ c + d + 20 = digits is a multiple of nine. Here, in 45² ,,. F(F) œ I(M) D(M) œ B(F) mother m
broth
= 2025, we have one zero and in 75² = ,. E(F) =
brother
fathe
80 per cent of girls father o
⇒ Girls = 50; e = 10 5625, we do not have any zero. ,,,. H(F) C(F)C G(M) AA( )
,,,. H(F) G(M) Solutio
Boys = 60; f = one-sixth of 60 = 10 ⇒ g = 8; \ Statement I alone is not sufficient.
(F) ( ) 50 u
,,. F(F) œ I(M) D(M) œ B(F) 192.50 u 30
192.
a = 30 and b = 18 From statement II, the number is a perfect181. 181.C is granddaughter
C is granddaughter of E. of E. Choice
Choice (4) (4) 191.
50 50 S
 30
,,,.
cube, which ends in one, which means it 181. C is granddaughter H(F) ofCE.
(F) G(M) (4)
Choice A( ) m
150. From the above information, the number 182. 182.
If there
If thereare arefourfourmales,
males, then
thenAAmustmust be be a male
male
should be a cube of 11 or 21. But 113 is 193.1717
193. perbp
of girls who qualified in both the subjects 182. If A
that
that is, there
is, are
is Anephew fourofmales,
is nephew I. I. then A must be a
of 1919 perfa
not a multiple of 9. p
= 10 Choice (1) 181. male
Choice C is(4)
Choice
that granddaughter
A is nephew ofof
is,(4) I. E. ChoiceSimilar (4)
,,,. Simi
213 is a multiple of 9 and is the number. As  183. H (F) C(F) G(M) A( )
151. The number of boys who qualified only in 183. I and I and IfF are
182. there
F are married married
are four males,Choice
couple.
couple. then (4)
Choice
Choice (1)
A must
(1) be a 192. male 5
we know the exact number, we can find the
Physics = 10 Choice (3) 183.
184.CThe
181. Iisand that
F areis,married
number
granddaughter ofApairs
is nephew
of siblingsofare
couple.
E. I. threeChoice
Choice (1)
that is, (4) Solutions 5
fo
number of zeroes in that. 184. The number of H.
pairs of (4) siblings are three that(3)
is, Solutions
184.
D I, G A, C
The number
Choice
of pairs of siblings are
Choice
three It is given th
\ Statement II alone is sufficient. 182. D I, IfGthere
A, C H. are four males, then A must Choice
be a(3) male It is193. 1
Dthat is, pair ofgiven siblin
10 5  Choice (1)185. 185. 183.
that isis, ADisI,nephew
I and FGare
brother-in-law A, C of H.
F. I.
married
of Choice
couple. (3) (4) Choice
Choice
pair of (1)sib
1
152. Required percentage = × 100 = 55 D is brother-in-law
Solutions for question of F.
186: Choice (4) arrangement
18 9 185. D Choice (4) S
159. From statement I, either C or B sits to theSolutions 184. foris question
brother-in-law
Thefollowed
number 186:of of F. Choice (4)
pairs of siblings are three
arrangem
st that is,
per cent Choice (4) 186.I The
andpath bycouple.
Manoj is Choice (1) 1 generatio
immediate right of A. The possible ar- 183. Solutions for
F are married
D I,question
G A, C186: H. Choice st (3)
1 genera
153. The number of students who did not qual- rangements are BAC and CAB. Hence,186.184. The path followed by
The number
Manojpoint
Stating
of pairsbyofManoj
is
siblings are three that is, Solutio
186. 185.The path followed is
ify in Physics = 60 Choice (2) statement I alone is not sufficient from D I, GDA,isCbrother-in-law
H.Stating point of F. Choice (3)Choice nd(4)
It2 is gen
giv
statement II, B is not at extreme left and The final distance
Solutions is 12 km towards
for question 186:South and
Solutions for questions 154 and 155: 8 km pair 2 ofg
nd
D is not at extreme right. We have no 185. D5iskm towards westofthat
brother-in-law
9 km
F. is, 13 km towards Choice (4)
arrange
35 40 55 45 25 25 information about A and C. Hence, state- Solutions 186. for
South-West.
The Cquestion
path followed 186: by Manoj is 3rd gen
154. Of , , , and , is the 8 km
B
75 90 115 80 65 65 ment II alone is not sufficient. By com-  9 km Choice (4) st rd
186. The C path
4 kmfollowed by Manoj is point
Stating 1 gen 3 g
bining both the statements the possible ar- B
least. By observation of the bar diagram, rangement is DBAC both the statements Stating point
it can be seen that only for classes IX together are sufficient. Choice (4) 4 km D 4 km
E

and X there is a significant difference in 8 km 2


the number of boys and girls. Hence one 160. From statement I, we can say that year is D 9 E
km
4 km 8 km
of them must have the least percentage not a leap year. Hence we can find 20th 9 kmC
June of previous year to year x and hence C B 3
of girls. As class IX has less boys than B
girls, it cannot have the least percentage we can find 20th June of year x. \ State- 4 km
ment I alone is sufficient. 4 km
of girls.
\ Class X has the least percentage.  Choice (2) D E
D
4 km
4 km
E
 Choice (4)
Solutions for questions 181 to 185:
155. The required percentage = Given, Solutions for questions 187 to 190:
40 − 35 (i) E is the mother of D, who is father of G. G When a cube is painted on all of it’s face and cut
×100 = 50%  Choice (3)
45 − 35 is the brother of A. into smaller cubes then,

08_Section.indb 169 10/30/2009 11:54:27 AM


147. 3 167. 1 187. 2
8. 4 28. 1 48. 4 68. 4 88. 3 108. 4 128. 4 148. 1 168. 3 188. 4
nd
From (iii), usha
9. 3 belongs 29. 2 to the 49. 32 generation
69. 2 89.and
3 109. 3 129. 1 149. 2 169. 3 189. 3
both Asha and 10. Vital
2 30. 3 to the
belong 50. 33rd generation.
70. 3 90. 2 110. 2 130. 3 150. 2 170. 3 190. 2
From (i) and11. the2 above 31.result
2 it51.can
3 be 71. 3
concluded 91.that
3 111. 4 131. 4 151. 2 171. 3 191. 1
12. 1 32. 1 52. 1 72. 3 92. 4 112. 1 132. 3 152. 2 172. 1 192. 3
Lata is the grand
13. 1
mother and Sachin is the husband of
33. 3 53. 2 73. 3 93. 3 113. 2 133. 2 153. 2 173. 2 193. 2
Usha. 14. 3 34. 2 54. 1 74. 2 94. 3 114.194.
2 Sachin 134. 1 is the 154.lightest
1 174. 4 194. 2
From (ii) and15.(iv), Usha and Sachin are of 22 kg and person. Choice (1)
170  n  Solution Manual 2 35. 4 55. 2 75. 3 95. 2 115. 2 135. 3 155. 3 175. 2 195. 4
72 kg weight16. in 4any order. 36. 1 56. 2 76. 2 96. 2 116. 1 136. 4 156. 2 176. 2 196. 1
195 Three persons are lighter than Vital. Choice (4)
From (i), Sachin
17. 3 cannot 37. be 4 the57. heaviest.
3 77. 3 97. 1 117. 1 137. 1 157. 1 177. 2 197. 4
18. 3 22 kg
(1) the eight cubes at eight corner of larger Ÿ SachinNow, we know38.that
weight and4 Usha 58. 3weighs
Piyush belongs 78.72 4 kg. 98. 2 118. 3
to the first Solutions
196.
138.
Piyush
1
for questions
is
158. 44 to 8:
Asha’s
178. 3
grandfather. 198. 4
Choice (3)
19. 2 39. 3 59. 1 79. 4 99. 2 119. 2 139. 3 159. 1 179. 3 199. 1
cube are painted on three faces. Ÿ Lata weighs
generation.
20. 342 kg. 40. 1 60. 1 80. 1 100. 2 120. 3 140. 2
Now,
4.
197.The Latafollowing
weight 42
160. 2 180.
diagrams
kg.
3 200. 3
represents the Choice (3)
(2) the cubes along each side except those at we know that Piyush belongs to the first
From (iii) and (v), it can be concluded that Piy- given statements.
generation. Solutions
the corners are painted on two of its faces ush’s
From (iii) andis(v),
weightit can is 32 be kg, Vital’s weight
concluded is 52 kgis
that Piyush’s 198.From Thediagram
difference (i): between the weights of Asha and
(3) the cubes on each face except those alongweight the isand Asha’s
32Solutions
kg, Vital’sweight
forweight is 62
questions is kg.
1 tokg
52 3: and Asha’s weight Vital is 10 kg. Choice (1)
sides are painted only on one of it’s faces.is 62 kg. Cream
The1.finalThearrangement
given series can is be
as written
follows. as Solutions for questions 199 and 200:
(4) all the cubes which do not appear out sides The final arrangement is as follows. Candy
2u1 1 ,3u 2  4 ,10u 3  9 ,39u 4 16 ,172u 5  25
are not painted at all. 199. It is given that D is the only daughter of A and B
The next
Lata number must be (172 u 5) + 25 = 885
In the given question, the larger cube is cut into 5 Hence, 885 is the next term.
Piyush Choice (3)
is son-in-lawCake of A. Hence, A's daughter's
× 5 × 5 = 125 cubes. husband is B that is, D's husband is B.
42 kg 32 kg of three series.
2. The given series is a combination Choice (4)
187. Eight cubes are painted on three of their faces. B1, C2 , EUsha3
, H 4 , L Sachin Chocolate
 Choice (3) 200. January 1st 2101 will be Saturday. Hence 31st
2 3 4 5
D ,F ,I ,M ,R
72 kg 22 kg December 2100 is Friday. Choice (3)
188. Number of cubes painted on only one face Cheese
G 3 , J 4 , N 5 ,S5 , Y
= 6 × 3 × 3 = 54. [∵ number of faces = 6]
Hence, LRY is the next term. Choice (2)
 Choice (2) Asha
Vital (i) (ii)
d hence we can find 3. The given series is a combination of three series.
tatement , alone is 189. Number of cubes which are not painted 452 kg Conclusion:
Conclusion:
4 4
B ,F ,J ,N ,R 4
62 kg
Choice (2) on any one of it’s faces = (5 – 2)3 = 33 , (negative) is true.
Ι (negative) is true.
The final distance is 12 km towards South
= 27 and (1)
Choice 194. Sachin
9, 49, 121, is the
225,lightest
361 person. Choice (1) ,, (affirmative) is false.
185: 5 km towards west that is, 13 km towards ΙΙ (affirmative)
,,, (affirmative) is true is false.
190. South-West.
Number of cubes painted on atChoice (4) of
least two D 4 , H 4persons
195. Three , L4 , P 4 , T are lighter than Vital. ΙΙΙ (affirmative)
,V (negative) is true. is true
father of G. G is the their
Solutions forfaces = Number
questions of cubes which are
187 to 190: Hence, R361T is the next term. Choice (3)
Choice (4) From (negative)
ΙVdiagram (ii): is true.
painted on 2 more faces (that is 2 and 3) = 196. Solutions
Piyush for
is questions
Asha’s 4 to 8:
grandfather. Choice (3)
Conclusion:
From diagram (ii):
F) When a cube is painted on all of it’s face and cut into , is false.
12 (5cubes
smaller – 2) then,
+8 Conclusion:
,V is false.
4. The following
197. Lata weight 42 kg. diagrams represents the given
Choice (3)
(1) the= eight
36 + cubes
8 = 44 Choice
at eight corner of larger cube (4) statements. Ι is false.
Hence, only ,,, follows. Choice (3)
M)
are painted on three faces. From difference
diagram (i): between the weights of
Solutions
(2) the cubesfor along
questions 191 except
each side to 193: those at the 198. The is false. diagrams represent the given
5. TheΙVfollowing
M) A( )
191. corners are painted on two of its faces
Sumaya’s husband’s mother-in-law is Su- Asha and Vital is 10 kg. Choice (1) Hence, only ΙΙΙ follows.
statements. Choice (3)
(3) the cubes on each face except those along the From diagram (i) :
the mother of A. But maya’s
sides mother
are painted onlyand herofmother
on one it’s faces.only son Solutions for questions 199 and 200: 5. The following diagrams represent the
neration they belong (4) allistheSumaya’s
cubes which brother whoseout
do not appear grandfather’s
sides are given statements.
not painted at all. 199. It is given that D is the only daughter of
only son can be his father or mother’s From diagram (i) : Sol/982
In the given question, the larger cube is cut into 5 u 5 u A and B is son-in-law of A. Hence, A’s ,, (negative)
5 =brother.
125 cubes. Choice (4)
daughter’s husband is B that is, D’s hus- ,,, (affirmativ
Stretches
187. Eight cubes are painted on three of their faces.
50 × 30 1500 150 75 Choice (3) band is B. Choice (4) IV
,, (affirmativ
(negative
192. = = = . Choice (2) Grounds From diagra
,,, (affirmati
not the brother of H. 200. January 1st 2101 will be Saturday. Hence Stretches
50 + 30 80 8 4 Plains Conclusion:
IV (affirmati
188. Number of cubes painted on only one face = 6 u 3 u
31st December 2100 is Friday. Choice (3) Grounds From diagra
Hence, only
3 = 54. [จ number of faces = 6] Choice (2) Fields
193. 17 per cent2 = (17 – 2)2 = 225 Plains Conclusion:
D(M) 189. Number 8. The basic
Hence, d
only
19 perofcent5
cubes =which
(193 are
– 5)
3
not
2 painted on any
= 196. Fields follows.
one of it’s faces = (5 – 2) = 3 = 27 Choice (1)
nts, B must be wife
must be wife of I(M)
Similarly 24 per cent11 = (24 – 11)2 MOCK TEST 3 8. The basic d
190. Number of cubes painted on at least two of their
usins, they belong to = 169.
faces = Number of cubes which are painted on 2
Choice (3) Lands follows.
Solutions for questions
more faces (that is 2 and194
3) = to
12 198:
(5 – 2) + 8 Solutions for questions 1 to 3:
It is =given
36 + 8 that
= 44 thee are two married Choicecouples
(4) Conclusion:
Conclusion: Lands
1. The given series can be written as , (affirmative) is false.
and one pair
Solutions of siblings.
for questions 191 There
to 193: are grandparents Ι (affirmative) is false.
,,Conclusion:
(negative) is false.
M) œ B(F) Hence the arrangement is as follows. 2×1+1 ,3× 2 + 4 ,10× 3 + 9 ,39× 4 +16 ,172× 5 + 25 ,,, ΙΙ (negative)
, (affirmative)
(affirmative) is false.
isis false.
false and
191. Sumaya's husband's mother-in-law is
From (iii), usha belongs to the 2nd Sumaya's generation ,V,, ΙΙΙ (affirmative)
(negative)
(affirmative) is false.
is true. is false and
mother and her mother only son is Sumaya's The next number must be (172 × 5) + 25
and bothbrother Asha
whose and Vital belong
grandfather's to can
only son the be3rdhisgen- Hence, ΙVonly(affirmative)
is falseisand
IV follows.
,,, (affirmative) true. Choice (2)
= 885 Conclusion
eration.father or mother's brother. Choice (4) ,V (affirmative)
Hence, only is true.
IV follows. Choice (2)
M) A( ) From 50 (i)uand the above result it can be concluded Hence, 885 is the next term. Choice (3) 6. The Hence, following diagrams represent Choice
only IV follows. the given
(2) ,, negative, i
30 1500 150 75 statements.
6. The following diagrams represent the Conclusion
,,, negative,
that
192. Lata is the grand mother
50  30 80 8
= . and Sachin is the
Choice (2) 2. The given series is a combination of three 6. The following diagrams represent the given ,, negative,
,,,, affirmativ
Choice (4) 4 given statements. Extracts
husband of Usha. series. statements. IV,
,,, negative,
negative
n A must be a male From
193. 17 (ii)
perand
cent2(iv),
= (17Usha 2
– 2) and
= 225Sachin are of 22 kg
B+1, C+2 , E +3 , H +4 , L To
,,,,prove
affirmati“S
19 per cent5 = (19 – 5)2 = 196. Extracts
Liquid
and 72 kg weight in any order. we
IV, have
negativeto
Similarly 24 per cent11 = (24 – 11)2 = 169. not
From (i), Sachin cannot be the heaviest. To possible.
prove “S
Choice (1) Choice (3) D +2 , F+3 , I +4 , M +5 , R Liquid To
weprove
have “S to
ngs are three that is,
⇒ Sachin weight 22 kg and Usha weighs
Solutions for questions 194 to 198:
Sol/982
false we hav
not possible
72 kg. +3 +4
G , J , N ,S , Y+5 +5 Juices which is not“S
Choice (3) To prove
⇒It isLata
givenweighs
that thee42 arekg.
two married couples and one Hence,
false we onlyha
Choice (4) pair of siblings. There are grandparents Hence the Hence, LRY is the next term. Choice (2) Juices
Nectar which is not
arrangement is as follows.
3. The given series is a combination of three Solutions
Hence, foronly
qu
Married Nectar
is
st
1 generation series. 9.Solutions
The problem
couple for q
nt B+4 , F+4 , J +4 , N +4 , R Fluids suggestions
Married (i) (ii) to recruitme
9. The proble
nd
2 generation 9, 49, 121, 225, 361 referring to
couple Fluids suggestions
km D +4 , H +4 , L+4 , P +4 , T From diagrams
(i) (i) and (ii)
(ii)
how the su
to recruitme
Hence, R361T is the next term. cover.
referring to
3rd generation Siblings Conclusion : , (negative) is true.
FromFrom diagrams
diagrams (i)(ii)and (ii)
(i) and ?Neither
how the , su n
 Choice (3) ,, (negative) is true.
cover.
Conclusion
,,, (negative): is true , (negative)
and is true. 10. The problem
?Neither ,
IV
,, (negative)
(negative) is is true.
true. is an extrem
Sol/981
Hence, all follow.
,,, (negative) is true and Choice (4) 10.ofThe
the problem
presen
IV (negative) is true. Creating aw
is an extrem
7. The
Hence,following
all follow. diagrams represent Choice the given
(4) Hence, ,, foll
of the prese
statements. ?Creating aw
Only ,, fol
7. From
The diagram
following(i) :diagrams represent the given Hence, ,, fo
08_Section.indb 170 10/30/2009 11:54:28 AM
statements. 11. The problem
not possible.
To prove “Some ceilings are not windows” to be
false we have to prove “All ceilings are windows”,
Juices which is not possible.
Hence, only , and IV follow. Choice (4)
Nectar
Solutions for questions 9 to 11:

9. The problem is depletion of forest cover. The Solutions for Mock Tests  n  171
Fluids suggestions made are irrelevant. One is referring
(i) (ii) to recruitment of IAS officers while the other is
Conclusion : Ι (negative) is true. referring tointourism.
portionate view of Neither of them
the present explained
problem. territory from external aggression. RΙ is
From diagrams (i) and (ii) how theΙ does
Hence, suggestion would increase the forest
not follow. not a reason as the neighbouring country
ΙΙ (negative) is true.
cover.
ΙΙΙ (negative)
Conclusion : is true and
, (negative) is true. C reating ,awareness
?Neither would mitigate
nor ,, follows. the (3)
Choice
buying a particular type of missile cannot
,, (negative) is true.is true.
IV (negative) problem. Hence, ΙΙ follows. be the reason for a country purchasing a
,,, (negative) is true and 10. ∴ TheOnly
problem is unnecessary honking of horns. , particular type of missile. Choice (2)
Hence, all
IV (negative) is follow.
true. Choice (4) ΙΙ follows.
is an extreme action. It is disproportionate in view
Hence,
7. Theallfollowing
follow. Choice (4)
diagrams represent the of the present problem. Hence, Choice (2)
, does not follow. 16. RΙ is a reason as the late detection of fire
given statements. Creating
11. The awareness
problem would
is global mitigate the problem.
warming. would render it difficult to control. RΙΙ is a
7. The following diagrams represent the given Hence, ,, follows.
From diagram (i) :
statements. Ι?isOnly
disproportionate as the suggestion can reason as the speed with which it spreads
,, follows. Choice (2)
From diagram (i) : make very little difference. The heat gen- makes it difficult to control. Choice (4)
11. erated
The problem is global
by stoves warming.as compared
is negligible
, is disproportionate as the suggestion can make Solutions for questions 17 to 20:
Brick to the present problem.
very little difference. The heat generated by 17. Since, the statement is informative and
Ι follows
Ιstoves as deforestation
is negligible as comparedand green
to the present
problem.gases are the reasons for glob- based on the facts, it is not based on any
house
Krick ,, follows as deforestation and green house assumption. Choice (3)
al warming. Hence, only ΙΙ follows.
gases are the reasons for global warming.
Grick Hence, only ,, follows. Choice (2) 18. In the statement, nothing is mentioned in
Choice (2)
the statement regarding the performance
Trick Solutions forfor
Solutions questions 1212
questions to to
16:16: of Dhoni as a Caplain. Therefore assump-
12.
12. RΙ
R, isis aa reason
reasonbecause
becauseititisisa awell
well known
known fact that
tion Ι is not implicit.
Prick fact thatin villages
villages in of
the vicinity thebig
vicinity of big
dams get affected. ∴ It is said that ‘rarely’ therefore assump-
R,, is not
dams get aaffected.
reason asRΙΙsuch constructions
is not a reason asmay be tion ΙΙ is also not implicit. Choice (3)
(i) (ii) useful from the employment point of view and
such constructions may be useful from
may be harmful in view of environmental aspects. 19. The statement is not talking about
Conclusion:
Conclusion: Ι (negative)
, (negative) is true. is true. the employment point of view andChoice may (1) those who do not love luxury and el-
ΙΙ (negative) is true. be harmful in view of environmental as- egance, therefore assumption Ι is not
ΙΙΙ (affirmative) is false and pects. Choice Sol/983
(1) implicit. Adverteses considers that the
,, (negative) is true. is false.
IV (affirmative) vehicle matches the expectations of
,,, (affirmative) is false and 13. RΙ is a reason as when a company makes
From diagram (ii):
IV (affirmative) is false. huge profits normally the employees are those who love luxury and elegance,
FromConclusion:
diagram (ii): ΙΙ is false. given a bonus. RΙΙ is not a reason, as such therefore assumption ΙΙ is implict.
Hence, only
Conclusion: follows.
,, isΙfalse. Choice (3) action would not have the effect of em-  Choice (2)
Hence, only , follows. Choice (3)
8. The basic diagram for the given statement ployees getting a bonus. 20. The given statement is not based on the
8. Theis basic
as follows.
diagram for the given statement is as  Choice (1) assumption but is based on the facts.
follows. Choice (3)
14. The popularity or otherwise of a contes-
Solutions for questions 21 to 25:
tant is not the only criterion that affects
The following table can be obtained from the
Door
an election result. Choice (3)
given information. However, students should
15. RΙΙ is a reason as missiles are one of the only calculate those values that are required to
weapons used to defend the country’s solve specific questions.
Ceiling
Walls
Window
Choice (2)
Conclusion
Conclusion Vehicle
Total Cars Bikes Scooters
esent the given Ι, negative,
,, negative, is true.
is true. City
negative,
,,, ΙΙ, negative,is false
is false
,,,, affirmative, is false. Delhi 9,00,000 62.5 % = 5,62,500 37.5 % = 3,37,500 0%
ΙΙΙ, affirmative, is false.
IV, negative, is true.
Mumbai 8,00,000 25 % = 2,00,000 25 % = 2,00,000 50 % = 4,00,000
ToIV, negative,
prove “Some iswallstrue.are not door”, to be false
weThave
o prove “Some
to prove “All’walls
wallsare
arenot door”,
doors”, to is
which Kolkata 7,00,000 50 % = 3,50,000 25 % = 1,75,000 25 % = 1,75,000
notbe possible.
false we have to prove “All’ walls are Chennai 6,00,000 50 % = 3,00,000 12.5 % = 75,000 37.5 % = 2,25,000
To prove “Some ceilings are not windows” to be
doors”,
false we havewhich is not“All
to prove possible.
ceilings are windows”, Hyderabad 5,00,000 25 % = 1,25,000 50 % = 2,50,000 25 % = 1,25,000
To prove
which “Some ceilings are not windows”
is not possible. Bangalore 4,00,000 0% 75 % = 3,00,000 25 % = 1,00,000
to beonly
Hence, false, and
we have to prove “AllChoice
IV follow. ceilings
(4)
Total 39,00,000 15,37,500 13,37,500 10,25,000
are windows”, which is not possible.
Solutions for questions 9 to 11:
Hence, only Ι and IV follow. Choice (4)
9. The problem is depletion of forest cover. The
Solutions for questions
suggestions to 11: One is referring 21. Highest number of bikes = 3,37,500,
made are9 irrelevant. 75, 000
to recruitment of IAS officers while the other is Lowest number of bikes = 75,000, Total = × 100 = 33.33%
(ii) 9. The problem is depletion 1, 00, 000 + 1, 25, 000
referring to tourism. Neither of
of forest cover.
them explained = 4,12,500. Choice (3)
howThethesuggestions
suggestionmadewouldareincrease
irrelevant.
theOne
forest  Choice (3)
is referring to recruitment of IAS officers
cover. 22. Number of car owners in Chennai =
ue.
?Neither , nor ,, follows. Choice (3) 3,00,000; Number of scooter Owners in 25. Total number of car-owners = 15,37,580;
while the other is referring to tourism.
Hyd = 1,25,000 Total number of scooter-owners =
10. TheNeither
problem ofisthem explained
unnecessary how the
honking sug- ,
of horns.
Total = 1,75,000 Choice (1) 10,25,000
gestion would increase the forest cover.
is an extreme action. It is disproportionate in view
Choice (4) Required percentage
∴Neither
of the presentΙproblem. Hence, , does not follow. 23. The total number of bike-owners for all
nor ΙΙ follows.
Creating
 awareness would mitigateChoice the problem.
(3) the six cities is 13,37,500, as can be ob- 15,37,500 −10, 25, 000
esent the given Hence, ,, follows. = ×100 = 50%
served in the table. Choice (3) 10, 25, 000
10. The ,,
? Only problem
follows. is unnecessary honking Choice of
(2)
horns. Ι is an extreme action. It is dispro- 24. Required percentage  Choice (3)
11. The problem is global warming.
, is disproportionate as the suggestion can make
very little difference. The heat generated by
stoves is negligible as compared to the present
problem.
,, follows as deforestation and green house
gases are the reasons for global warming.
08_Section.indb 171 10/30/2009 11:54:29 AM
172  n  Solution Manual

Solutions for questions 26 to 29: 2,35,950 42. CP of 1 apple = 1/4, SP of 1 apple = 1/3
Sales of Honda in 1990 =
1.1 × 1.2
profit
25 30 52 15 37 20 per cent profit = × 100%
26. Of × , × , × = 1,78,750 Sales of Ducati in 1990 = CP
100 100 100 100 100 100
2,35,950 1 1
= 1,65,000 −
51 35 37 20
and × , × is the least. 1.1 × 1.3 = 3 4 × 100
100 100 100 100 2,35,950 1
Sales of Aprilla in 1990 = =
 Choice (3) 1.1 × 1.1 4
1,95,000
27. Colleges which do not offer risk manage- 3 bikes have sales of more than 1,65,000 = 1/3 × 100 = 331/3 per cent Choice (3)
ment course may or may not offer man- in 1990.  Choice (3)
agement. Choice (4) 43. First discount = Rs 20 by value. Price af-
ter discount = Rs 180
28. The required number of B – schools. 34. Let us assume that the sales of Honda
If the second discount = x per cent, Sec-
Mag-EV in 1995 were 100
ond discount = Rs x/100 (180) by value
=  86 × 30 + 80 × 15 + 74 × 20 + 68 × 35  Projected sales of Honda Mag-EV in
100 100 100 100 100 100 100 100
 
x/100 (180) = 180 – 162 = 18; x = 10
2005
Choice (3)
× 2000 = 1528 Choice (1) = 100 x 1.2 x 1.3 = 156
∴ Honda sales are projected to be 56 44. Suppose B worked for x days. Since, A
29. The required percentage = 26/80 × 100 =
per cent more in 2005 than that of 1995. 4 x
32.5 per cent Choice (2) worked for 4 days, + = 1 ⇒ x = 8
 Choice (2) 20 10
Solutions for questions 30 to 34: Choice (2)
Solutions for questions 35 and 36:
30. Let us assume that the sales of each bike 45. The part of the tank filled in the first 12
is ‘Z’ in the year 1990; Sales of Suzuki in
2000 = z x 1.2 x 1.25 = 1.5z 35. Of 130 −110 , 122 −104 , 98 − 76 , 112 −104  1 1  12 2
minutes. = 12  +  = =
130 122 98 112  30 45  18 3
 ales of Kawasaki in 2000 = z x 1.1 x 1.2
S
= 1.32z; Sales of Honda in 2000 = z x 1.1  he part of the tank emptied per minute.
T
88 − 64 88 − 64
x 1.2 = 1.32z and , is the highest. When the tank is full
88 88
 ales of Ducati in 2000 = z x 1.1 x 1.3 =
S 1 1 1 1
 Choice (4) = − − =
1.43z; Sales of Aprilla in 2000 = z x 1.1 15 30 45 90
x 1.1 = 1.21z ∴ The time taken to empty two-third of
36. The required percentage
∴ Highest percentage increase is for Su- 2
zuki Hayabusa Choice (3) 110 + 104 + 76 + 104 + 64
= ×100 the tank = 3 = 60 minutes. Choice (2)
550 1
31. Sales of Ducati in 2002 = 42900; we 90
know sales in 1990 x 1.1 x 1.3 = 42900 458 91600
= ×100 = = 83 3/11 per cent 46. Let the fixed charges be Rs x and Rate per
42900 550 11
∴ sales in 1990 = = 30000; Pro- call be Rs y.
1.1 ×1.3  Choice (1)
jected sales of Ducati in 2005=42900x1.4 When 100 calls are made average cost per
= 60060 Solutions for questions 37 to 40:
x
∴ difference in projected sales and the call = + y = 11 ------ (1)
37. On four occasions, the total literacy 100
sales in 1990 is 60060 – 30000 = 30,060 rate increased over the previous decade.
When 200 calls are made it is
 Choice (2)  Choice (4)
x
264000 38. The total literacy rate was the maximum + y = 6 ------ (2)
32. In 1990, sales of Kawasaki = in 2001. Choice (4) 200
1.1 ×1.2
= 200000 Sales of Aprilla AS – x100 = 39. The maximum increase occurred from Solving (1) and (2), x = Rs 1000 and y
1971 to1981, that is, 78 − 62 = 16. = Re.1
544500  Choice (3) When he paid Rs 9/call
= 300000
1.1 ×1.1 ×1.5 1000
40. The highest increase in the literacy rate Let the number of calls be n, ∴
of any decade over the previous one dur- n
 atio of sales of Kawasaki to that ofAprilla is
R 1000 1000
2:3 Choice (1) ing the pre-independence period was + 1 = 9; = 8; = n; n = 125
12 per cent. The increase for the post- n 8
independence period was 16 per cent. Choice (2)
2,35,950 
33. Sales of Suzuki in 1990 = Choice (1) is true. Choice (1) 47. Cost price of the article = Rs 9. He gains
1.2 × 1.25
Solutions for questions 41 to 60: 205 gain = 1.2 × 95 = 114
= 1,57,300 Sales of Kawasaki in 1990
114 forms 80 per cent of marked price,
41. The ratio of profits of A and B
2,35,950 ∴ M.P = Rs 142.5 Choice (1)
= = 10,000× 12 : [5000 × 12 + 2000 [10 + 8 + 6
1.1 × 1.2 + 4 + 2] = 120 : 60 + 2 (30) = 1 : 1. 48. In one day, work done by A = 1/18. Work
= 1,78,750 ∴ Each will get Rs 300 Choice (2) done by B = 1/24

08_Section.indb 172 10/30/2009 11:54:30 AM


Solutions for Mock Tests  n  173

In 2 days the part of the work completed 55. The speed of the road roller per hour = means ‘secure’. Hence choices (1) and
40 (4) are ruled out. Choice (2)
1 4+3 7 × 60 = 24 . Choice (2)
= = = , 60 + 40 62. The first blank should be filled in with a
24 72 72
56. The required percentage positive word. Hence choice (1) is elimi-
Let 1 unit of time = 2 days nated. Since the word ‘priceless’ is used,
In 10 units of time (i.e 20 days) the part 90 20 the words ‘enriched’ and ‘endowed’ are
= × × 100 = 18%  Choice (2)
of the work completed = 70/72, Remain- 100 100 more suitable than ‘supplied’. ‘Hardly in-
ing work conspicuous’ means ‘evident’ which is not
57. Let the price of the first article be Rs x
= 2/72 = 1/36 in keeping with the idea that is sought to
25 per cent of x – 20 per cent of x = Rs
If A starts the work on the first day, A be conveyed.
1000.  Choice (4)
works on the 21st day.
⇒ 5 per cent of x = Rs 1000 ⇒ x = Rs
The time taken by A on 21st day 63. ‘Seductively’ is inappropriate in the con-
20,000
= 1/36 × 18 = 1/2 day text. Hence choice (4) is ruled out. A belief
Let the price of second article be Rs y
Totally in 201/2 days the work is com- can neither be ‘scurrilous’ nor ‘wanton’.
30 per cent of y – 20 per cent of
pleted ‘Scurrilous’ means ‘grossly abusive’ and
y = Rs 1000 ⇒ 10 per cent of
If the work is started by B, the time taken ‘wanton’ means ‘motiveless’. Choices (1)
y = Rs 1000 ⇒ y = Rs 10,000.Choice (3)
by B on 21st day 1/36 × 24 = 2/3 days. and (3) are ruled out. Choice (2)
58. Let the forward and the return speeds of
2 1 1 64. ‘Aphoristic’ which means ‘in the nature
The difference = − days the car be f kmph and r kmph respective-
3 2 6 ly. of a short wise saying’, evidently does
 Choice (4) 2fr f + r not fit in the context. Choice (3) is ruled
Average speed of the car = =
49. Length of the train = 300 m. Let the speed f +r 2 out. ‘….. a mindset that reflect’ is gram-
of the train be x m/s ⇒ (f + r)2 – 4fr = 0 matically incorrect. ‘Reflects’ is the cor-
Speed of the man = 18 × 5/18 = 5 m/s. f2 + r2 + 2fr – 4fr = 0; (f – r)2 = 0 ∴ f – r rect word. Hence, choice (4) is ruled out.
= 0. f = r ‘Dwell’ means to ‘brood or linger over
300
Time taken to cross the man ⇒ ∴Forward travel time = return travel time something that is best forgotten’. Hence,
x −5 the word is inappropriate. Choice (1)
= 15 = 10/2
= 5 hours. Speed for its forward journey
⇒ x – 5 = 20, x = 25 m/s = 400/5 = 80 kmph. Choice (3) Solutions for questions 65 to 68:
Distance to be travelled to cross the plat-
65. ‘Inappropriate’ means something that is
form
not suitable but cannot be used in this
= (300 + 450) = 750 metre. Time taken x1 + x 2 _ _ _x15 context ‘Inconvenient’ is ‘unsuitable’
= 750/25 59. Given That = 36;
15 though not unfortunate. ‘Inclement’ is
= 30 seconds Choice (3) x1 + x2 + _ _ _ _ x15 = 36 × 15 = 540 used with weather. Hence only ‘inoppor-
50. Number of girls = 3/7 × 560 = 240. tune’ can replace the underlined phrase.
x1 + x 2 _ _ _x 8
 Choice (3)  Choice (2)
8
51. Let the cost price be Rs x, the selling 66. Something fast and intense comes like
price be Rs 1.4x. If the C.P is 0.8x, = 32; x1 + x2 + _ _ _ _ x8 = 256 ---- (1) a flood. Hence the suitable replace-
S.P is 1.4x – 20 and profit is 0.4x x8 + x9 + _ _ _ _ x15 = 38 × 8 = 304 ment for the phrase is ‘inundated’ which
∴ 1.4x – 20 – 0.8x = 0.4x ⇒ 0.2x = 20  ----- (2) means flooded. ‘Invasive’ means something
⇒ ∴ x = 100 Choice (3) Adding (1) and (2) x8 + (x1 + x2 + undesirable which spreads very quickly.
_ _ _ _ x15) ‘Intruding’ is unsuitable because it means
52. On every 960 gms he uses 900 gms that
is, his gain is 60 gms on every 900 gms. = 560. x8 + 540 = 560; ∴ x8 = 20 something unwelcome. Choice (2)
∴ Gain Percentage = 60/900 × 100 per  Choice (1)
67. To be fastidious means to be very particu-
cent = 20/3 = 62/3 per cent Choice (1) 60. Let the weight of the full bottle be x kg. lar about accuracy and detail. The words
53. The ratio of speeds of A and B is 5: 4. It ‘diligent’ and ‘industrious’ which are syn-
means, for every 5 metre that A travels, B ∴ The weight of the liquid in the bottle onyms which mean hardworking and the
travels only 4 metres 7x word ‘squeamish’ which means having
when it is full is kg very strong moral views are not suitable
It means for every 5 metre of travel, A 10
will reduce the distance between them by in the context. Choice (2)
∴ The weight of the liquid removed =
1 m. 68. Decisions that are not based on army princi-
A needs to travel 50 × 5 metre to clear the 6x ple, plan or system can be called ‘arbitrary’
gap between them 6x 10 6 decisions. They are not ‘illegal’ or ‘unfair’
; ∴ The required fraction = =
∴ They will meet at a point that is 10 7x 7 ‘Mediatory’ means arriving or trying to ar-
250 metres from the starting point. 10 rive at a settlement. Choice (4)
 Choice (2)
 Choice (1)
54. The snow that falls from 8 a.m. to 2 p.m. Solutions for questions 69 to 72:
1 Solutions for questions 61 to 64: 69. The word ‘than’ indicates comparison.
2
Therefore the word ‘more’ would be ap-
= 6 × 2 = 5 inches. 61. The first blank must be filled in with
3 a positive word which means ‘peace’. propriate. This rules out options (1) and
∴Total snow on the ground at 2 p.m. = 5 Hence choice (3) is ruled out. The second (4) and (5) erroneous. Option (2) is gram-
+ 5 = 10 inches. Choice (1) blank demands a positive word which matically correct. Choice (2)

08_Section.indb 173 10/30/2009 11:54:31 AM


174  n  Solution Manual

70. Options (1) and (4) omit the word ‘how’, writing it is as in ‘B’. Further in ‘A’ Step IV: 9 is subtracted from each number of
which follows ‘know’ before a verb. ‘expects of them’ is erroneous because step ΙΙΙ
Option (2) has a parallelism error. Only the subject is plural. Hence it should be Step V: 4 and 3 are added to the alternate num-
option (3) follows the correct parallel ‘expect of them’. Further in A and D ‘ex- bers of step IV
construction. Choice (3) pected of ordinary citizen’ is erroneous. It 101. Input: 19 22 48 37 28 15
should be ‘the ordinary citizen’ since the
71. Options (1) and (2) are wrong because the Step Ι: 10 14 12 10 10 6
reference is being made to every citizen
pronoun ‘it’ in option (1) and the pronoun Step ΙΙ: 20 8 24 20 20 12
of the country. In ‘C’ the words “and will
‘they’ in option (2) do not have a clear Step ΙΙΙ: 8 12 20 20 20 24
expect …..” make the sentence errone-
antecedent. Option (4) has inconsistency  Choice (3)
ous. The subject ‘A’ modern democracy’
in tense. Also, the pronoun ‘it’ has no
is singular, hence it should be ‘expects of 102. Among the steps ΙΙ and IV the step ΙΙΙ is
antecedent. Option (3) is grammatically
them’. Choice (2) the rearrangement of the numbers in as-
correct. Choice (3)
cending order. Hence, the step Ι cannot be
72. It is stated that Louis Braille designed a Solutions for questions 77 to 79: determined by giving step IV.
form of communication enabling people 77. D opens the paragraph by introducing us  Choice (4)
to do something. Only the phrase ‘by in- to the subject, on which the para is based, 103. Input: 12 15 19 11 34 22
corporating’ answers the question ‘how’. that is the link between blood lots and Step Ι: 3 6 10 2 7 4
This rules out options (1) and (2) where frequent flying C further explains D and
Step ΙΙ: 6 12 20 4 14 8
the meaning is distorted. Between options mentions the name of the disease. B fol-
(3) and (4), option (3) would be appropri- lows C by carrying the idea forward. C Step ΙΙΙ: 4 6 8 12 14 20
ate because the absence of an article (‘a concludes the paragraph by talking about Step IV: –5 –3 –1 3 5 11
series’) in option (4) makes it erroneous. the seriousness of the condition. Hence Step V: –1 0 +3 6 9 14
 Choice (3) DCBA is the correct order. Choice (3)  Choice (1)
Solutions for questions 73 to 76: 78. Statement C is appropriate as the opening 104. Step Ι: 8 4 9 3 2 1
statement as it is generalized and intro- Step ΙΙ: 16 8 18 6 4 2
73. Statement A is erroneous in using ‘where’ duces us to a topic ‘friendly fire’. State-
with a period of time. ‘When’ is more Step ΙΙΙ: 2 4 6 8 16 18
ment A which describes what a ‘friendly Step IV: –7 –5 –3 –1 7 9
appropriate. Statement B uses the modal fire’ is, ideally follows C as the two state-
‘could’ in place of ‘would’. ‘Could’ Step V: –3 –2 1 2 11 12
ments are linked. Statement D talks about
is used as the past tense of ‘can’ which disease. B follows C by carrying the idea forward.
the reference to ‘friendly fire’ in non- or output:
Step ,,,: 2 4 6 8 16 18 Choice (2)
shows ability. ‘Would’, the past tense of C concludes the paragraph by talking about the Step IV: –7 –5 –3 –1 7 9
military’
seriousness terms
of the by Amartya
condition. Hence DCBASen. Choice
is the Solutions
Step for
V: question
–3 –2 105:
1 2 11 12
‘will’ to indicate a past tense is appropri-
correctBorder.
is a continuation of D. Hence ChoiceCADB.
(3) or output: Choice (2)
ate. Statement C is error free. Statement D  Choice (4) 105. Let actually N, E, S and W are the point-
uses the preposition ‘for’ instead of ‘over’ 78. Statement C is appropriate as the opening Solutions
ers for
of question
the compass105:which points North,
statement
79. as it is Bgeneralized
Statement is the and introduces
opening statement usas
which, in this context means ‘because of’ to a topic ‘friendly fire’. Statement A which East,
105. Let SouthN,and
actually E, West
S andrespectively. But the of
W are the pointers
which is appropriate. Choice (3) it introduces us to the topic of the pas- pointer
the W ofwhich
compass the damaged compass
points North, show-and
East, South
describes what a ‘friendly fire’ is, ideally follows C
as thesage two that is the Western
statements GhatsStatement
are linked. which areDa West respectively.
ing north But the pointer W of the
as show below.
74. Statement A is incorrect because the ad- ‘treasure
talks about trove of biodiversity.’
the reference Statement
to ‘friendly fire’ in non- damaged compass showing north as show below.
verb ‘dramatically’ is not immediately military’D terms
follows by BAmartya Sen. Choice
as it continues B is a
the descrip- North
followed by the adjective it is supposed continuation
tion ofof the
D. Hence
Western CADB. Choice (4) A
Ghats. Statement
to modify (that is, novel). It should be fol- 79. Statement W
followsB isD the as ‘this varied
opening mosaic’asin itA
statement
lowed by the adjective or another adverb introduces
refersus toto the
the topic of the
‘variety of passage that is
interconnected
like ‘recognizably’. Statement D is incor- the Western Ghats which
ecosystems’ are a ‘treasure
mentioned trove of
in D. Statement
rect because the adjective ‘dramatic’ and biodiversity.’ Statement D follows B as it West S N East
continuesC istheconclusive
description in of
nature. Hence BDAC.
the Western Ghats.
the adverb ‘recognizably’ are not joined Statement A follows D as ‘this varied mosaic’ Choicein(4)A E
by the conjunction ‘and’. The appropri- refers to the ‘variety of interconnected The person going South
towards the direction which is
ate structure is “something dramatically, Solutions for question 80:
ecosystems’ mentioned in D. Statement C is The by
shown person going
pointer towards
S of the theisdirection
compass West.
recognizably novel………” The two ad- conclusive in nature. Hence BDAC. Choice (4)
80. Statement ‘1’ says ‘the hospital is a He is actually
? which is showngoing
by towards
pointer West. Choice
S of the com-(3)
verbs used must be as close as possible Solutions mega for question
project’.80:‘A’ follows ‘1’, because passfor
Solutions is West.
questions 106 and 107:
to the word ‘novel’. Further, in option C, the hospital
80. Statement '1' saysis 'the
very hospital
much relevant
is a megato the ∴ He is actually going towards West.
“would soon occur” and “would change” project'.patients, and '1',
'A' follows it says that itthe
because hashospital
becomeisa 106. After conversion the expression is 25 + 15 – 10
 y 10 = 40 – 10 u 3 = 40 – 30 =Choice (3)
are erroneous because the whole sentence very muchray ofrelevant
hope totothe thepeople
patients, and it says
of Central Ker-
u 30 10.
is in the present tense, which is why a that it has become a ray of hope to the people of Choice (2)
ala. Further B, C, D tell us about the fa- Solutions for questions 106 and 107:
107. After conversion the expression is 81 y 9 + 42
part of the sentence cannot be in the past Central Kerala. Further B, C, D tell us about the
cilities
facilities provided.
provided. B givesBthegives the of
extent extent of the
the facility + 2 10 = 9 + 6 + the
106. Afteru conversion
y 7 20 =expression
15 + 20 = 35.
is 25 +
tense. Choice (2) facility whereas C gives more specific in-
whereas C gives more specific information on the 15 – 10 × 30 ÷ 10 = 40 – 10 × 3 =Choice (3)
40 – 30
75. Since the selection procedure is singular, rooms.formation
Hence B should
on theberooms.
beforeHence
C. Choice (1)
B should = 10.for questions 108 to 111:
Solutions
it is ‘an ill defined selection procedure’ Solutions be forbefore
questionsC. 101 to 104: Choice (1)  Choice (2)
Given P u W; W u S; S u Q; S u
(rules out choices 1 and 2). The choice is
By Solutions
observing for thequestions
input an101each to 104:
step we can 107. R; After conversion
T u U; the
M expression
u N; Misu81V ÷ 9 + 42
‘subject to’ not ‘subjected with’ (rules out determine rule followed in each step
choice 4). Choice (3)
By observing the input an each step we can de- 108. If÷V7 and
+ 2 ×Q10are
= 9selected
+ 6 + 20in=the
15 + 20 =then
team, 35. M
Steptermine
,: The rulesumfollowed
of digits ininthe
each step of input are
numbers  must be selected in the same team Choice
as P. (3)
76. In ‘A’ the words “unlike those that of Steptaken
Ι:  inThe
the sum
sameof order
digits in the numbers of Choice (4)
antiquity” is the first error. Here, in this Step ,,: The numbers of step
input are taken , are doubled.
in the same order Solutions for questions 108 to 111:
109. As P and W are in different teams and W and S
sentence, ‘a modern democracy’ is be- StepStep ,,,: The numbers of step ,, are rearranged in
ΙΙ: The numbers of step Ι are doubled. Given
are P × W; teams.
in different W × S; S × Q;
ascending order.
ing compared with the democracies StepStep IV: 9ΙΙΙ:  The numbers
is subtracted from eachof step ΙΙ areofrearranged
number step ,,, ?SP×and
R; S must beT in× the
U; same team.
M × N;
of the past. Hence the correct way of Step V: 4 and 3inare ascending
added to order.
the alternate numbers of M×V Choice (3)
step IV 110. If team A has 6 members, it should have W, Q,
101. Input: 19 22 48 37 28 15 R, N, V and T/U. ? Team B must have M, P, S
Step ,: 10 14 12 10 10 6 and T/U. Choice (2)
Step ,,: 20 8 24 20 20 12 111. In this case, W must be in the same team as M.
Step ,,,: 8 12 20 20 20 24 Choice (3) Choice (4) 11:54:31 AM
08_Section.indb 174 10/30/2009
Solutions for Mock Tests  n  175

108. If V and Q are selected in the team, February ⇒ 1; March ⇒ 3 Solutions for questions 121 to 140:
then M must be selected in the same team April ⇒ 2; May ⇒ 3 121. Let the speeds of the boat in still water
as P. June ⇒ 2; July ⇒ 3 and the stream be x kmph and y kmph
 Choice (4) As the number of odd days between Feb- respectively.
109. As P and W are in different teams and W ruary 1st and August 1st is zero. The day
40 40
and S are in different teams. of the week on February 1st is same as + = 24 → (1)
that on August 1st.  Choice (2) x+y x−y
∴ P and S must be in the same team.
 Choice (3) Solutions for questions 116 to 120: I f the speed of the boat in still water dou-
110. If team A has 6 members, it should have It is given that C got the least total points. bled, time taken by it to cover the down-
W, Q, R, N, V and T/U. ∴ Team B must Hence, the total points of C is either 3 (1 + 2) or 4 40
have M, P, S and T/U. Choice (2) (1 + 3) stream same journey =
2x + y
Given, A got 1 point in strength. 40 40
111. In this case, W must be in the same team < → (2) Usual time for
∴ C got 1 point in intelligence and either 2 or
as M. Choice (4)  2x + y 2(x + y)
3 in strength.
Solutions for questions 112 and 113: Case (i): - If C got 2 points in strength, E would downstream journey < usual time for up-
Let us represent the given data in the venn dia- have got 3 points and 2 points in strength and stream journey.
gram as follows intelligence respectively (∵ E got 5 points in (1) ⇒ usual time for downstream
total). 24
journey < < 12 hours. (2) ⇒
Physics Chemistry Given D got 4 points in intelligence. Hence, D 2
would have got 5 points in strength and B would 40 40 12
x g y have got 4 points in strength. < <
2x + y x + y 2
Now B cannot get 5 points in intelligence (total
2
h points of D and B will be 9).
B would get 3 points and A would get 5 points = 6 hours. Only choice (4) violates this
It is given that the number of students those in intelligence. condition. Choice (4)
passed in both the subjects is equal to the num- The assigned points are as below:
ber of students failed in both the subjects. 122. Let the quantity of the cheaper variety
∴ g = h. Strength Intelligence Total mixed be x kg
For every one student who passed in only Chem- 5 × 20 + x × 16
A 1 5 6 ∴ is the cost price;
istry in
students failed there
botharethetwo students passed in only Phys-
subjects. 5+x
B 4Strength Intelligence
3 Total 7
? g = h. ics. A 1 5 6
For every ∴ onex= 2y who passed in only Chemistry
student C B 2 4 13 7 3 100 + 16x  105 100 + 16x
  = 18.2. =
there are two
Halfstudents passed inthose
of the students only Physics.
passed in Chemistry DC 5 2 41 3 9  5 + x  100 5+ x
? x = 2y passed in Physics also D 5 4 9
Half of the students those passed in Chemistry E 3 2 5 1820
E 3 2 5
passed in ∴ g = y also
Physics 105
? g = y ∴ g = h = y = /2
x
Case Case
(ii): If (ii):
C got If 3Cpoints
got 3 in strength,
points E wouldEhave
in strength, would ⇒ x = 10 Choice (4)
? g = h =∴ y =and
x
/2 it is given that x + y + g + h = 100 got 2 points
have got in strength
2 pointsand
in 3strength
points inand intelligence.
3 points in in-
? and it is given that x + y + g + h = 100 As D telligence.
got 4 points in intelligence, B gets 4 points and 123. The part of the work completed by
x x xx x x D getsAs 5 points
x+ + x ++ =+ 100 Ÿ + 5x ==200 100 ⇒ 5x = 200 D got in strength.
4 points in intelligence, B gets 4 points Ajay and Bharath together in one day
2 2 22 2 2 Now Band D gets
cannot get55points
points in
in strength.
intelligence (total points
The final diagram is as follows:
The final diagram is as follows: of B and D will be 9 then). 1 1 5
Now B cannot get 5 points in intelligence (total = + =
μ =P 100
= 100 Hence B gets 2 points and A gets 5 points in 6 9 18
? x = 40 and points of B and D will be 9 then).
intelligence
g = h = y = 20. Physics Chemistry Hence
But total B gets
points of A2 points
and B and will Abegets
6, 5which
pointsisina in- ∴ The number of days required to com-
contradiction.
telligence plete the work = 18/5 = 3.6 Choice (4)
40 20 20 But total
Only case points
(i) is to of A and B will be 6, which is a
be considered. 124. Quantity of alcohol in first solution = 7
contradiction. l. Quantity of alcohol in second solution
116. A got 5 points in intelligence. Choice (1)
h = 20 Only case (i) is to be considered. =8l
117. A gains the third position in overall. Choice (1)
∴ x = 40 and 116. A got 5 points in intelligence. Choice (1) Concentration of resulting solution
112. 40 passed 118. As D got the highest total points, D is Mr.India.
g = h =inyChemistry.
= 20. Choice (1)
117. A gains the third position in overall.
Choice (3)
15 15
= = = 50 per cent Choice (3)
113. Students those failed in Physics = h + y = 40. 10 + 20 30
112. 40 passed in Chemistry. 119. B got 7 points and E got 5 points
Choice (1)
Choice (2)
Choice (1)
Difference = 2. Choice
113. Students those failed in Physics = h + y = 118. As D got the highest total points, (2) 125. Let the height of the tower be y the dis-
Solutions for questions 114 and 115: D is
40. Choice (2) tance between the foot of the tower and of
Mr.India. 1  2  3 Choice
4  5 (3)
114. The clock gains 15 minutes in 24 hours. Hence, 120. Average points in a category = the building be x.
to gainSolutions
(8 + 2),for questions
that 114 and
is 10minutes, 115: 16
it takes 5
119. B got 7 points and E got 5 points
hours. = 3 points only B and D got at least 3 points in
Difference = 2.  Choice (2)
114. The clock gains 15 minutes
? The clock will be 2 minutes fast at 10 pm in 24
on hours. each category. Choice (3) 60° 30°
Monday. Hence, to gain (8 + 2), Choice
that is (2)
10minutes,Solutions
120. for
Average points in a
questions 121 to 140:category
it takes
115. If 1st of February is 16 hours. the number of odd
a Sunday 80
∴ The clock will be 2 minutes fast at 121. Let the speeds of the boat in still water and the
10
days is 1+ 2 + 3 + 4 + 5 30°
February Ÿ pm1;on Monday. 
March Ÿ 3 Choice (2) =
stream be x kmph and y kmph respectively.
40 40 5
April Ÿ 2; May Ÿ3  24 o (1) y
June 115.
Ÿ 2;If 1st of February
July isŸa 3Sunday the number x  y =x 3 ypoints only B and D got at least 3 60°
of odd
As the number of days is between February
odd days pointsofinthe
If the speed each category.
boat in still water Choice
doubled,(3) x
1st and August 1st is zero. The day of the week time taken by it to cover the downstream same
on February 1st is same as that on August 1st. 40
Choice (2) journey =
2x  y
Solutions for questions 116 to 120:
40 40
 o (2) Usual time for
It is given that C got the least total points. 2x  y 2(x  y)
Hence, the total points of C is either 3 (1 + 2) or 4 (1 + 3)
08_Section.indb downstream journey < usual time for upstream
Given, A got 175
1 point in strength. 10/30/2009 11:54:32 AM
176  n  Solution Manual

80 − y 80 200 10 90
∴tan30° = → (1) tan60° = Suresh = = . Speed of Ramesh = In ∆ MBA, AB = = 90 3 m.
x x 180 9 tan 30°
80 5 × 10/9 = 55/9 m/sec Choice (3) 180 180
⇒x= 133. Let the number of men in the group be x. In ∆ NCA, AC = = = 60 3 m
3 tan 60° 3
Job = 10x man days
Substituting the value of x in equation The total number of man days ∴ BC = 90 3 + 60 3 = 150 3 m
1 80 − y 80 x(x + 1)  Choice (2)
(1), = ⇒ = 80 – y; = 1 + 2 + 3 + ………. x =
3 80 3 2
x(x + 1) Solutions for questions 141 to 144:
3 Given, = 10x
2
141. Number of employees in division A of
∴ y = 160 x = 19 Choice (2)
3 consumables section = 150/360× 1080 =
450
= 531/3 m Choice (2)  22  Number of employees in division A of du-
134.  2 × × r  × 200 = 550 .
 7  rables section = 160/360 × 1080 = 480.
126. Let the number of runs scored by the first
five players, the sixth player and the last ⇒ 2r = 875 mm. Choice (1) ∴ The difference = 30 Choice (2)
five players be f, s and  respectively. ∴f 135. Volume of cylinder = π r2h = 142 × 21. 142. Since the total number of employees in
+ s = 6 × 45 → (1)  + s = 6 × 15 → (2) Volume of cone = 1/3 π r2h = 1/3 × 212 durables section is not known, the re-
f +  + s = 11 × 30 → (3). (1) + (2) – (3) × 14 quired value cannot be determined.
⇒ s = 30 Choice (1) Ratio of volumes = 142 x 21 : 1/3 × 212 x  Choice (4)
14 = 2 : 1  Choice (3) 143. Let the total employees in the consum-
127. Distance travelled by one revolution
of the rear wheel = 2πR = 2 ×22/7× 70 136. The letters A, E, H, R and T can be ar- ables and the durable sections be 2x and
= 440 cm ranged in 5! ways that is, 120 ways. 3x respectively.
Distance travelled by the fore wheel in ∴ The required probability = 1/120 Number of employees in division D of
one revolution = 2πr = 2 ×22/7× 35 = 220  Choice (4) consumables section
cm = 40/360 × 2x = 2x/9–––– (1)
137. A and U can be placed in 3P2 ways (in the Number of employees in division
The ratio of number of revolutions middle three positions) that is, 6 ways.
= 1/440: 1/220. ∴ Required ratio = 1: 2 B of durables section = 80/360 × 3x =
The remaining three letters can be ar-
Irrespective of the distance travelled the 2x/3–––– (2)
ranged in the remaining three positions 2x
ratio of the revolutions remains same. in 3! ways.
 Choice (3) ∴ The total number of codes is 6 × 6 = 36 Required per cent = 9 × 100 = 331/3 per cent
2x
128. Let x litres of milk be replaced. Given  Choice (1) 3
2  Choice (3)
 80 − x  138. AB2 = 202 − 122 ⇒ AB = 16
  × 80 = 61.25 ⇒ x = 10 litres BC2 = 202 − 162 ⇒ BC = 12
 80  144. Let the total employees in the consum-
 Choice (4) ∴ AC = 16 + 12 = 28 cm.
ables and durable sections be 4x and 3x
129. Radius of the bigger sphere respectively.
x The number of employees in the division
 4   22 
8 ×   ×   × (7) (7) (7) 90
3  7  20 20 16 B of consumables section = × 4x
= 3 = 14 cm 12 360
4 22
× = 600
3 7
⇒ x = 600. ∴ Total employees in dura-
∴ Its surface area = 4 × 22/7 × 14 × 14 A B C x =sections
Ÿbles 600. ?=Total employees in durables
3x = 1800
= 2464 cm2. Choice (1) sections = 3x = 1800
 Choice (1)
? The number of employees in in

 The number of employees division
division D of
139. When a sum is invested to earn compound D of durables
durables section = section
70/360 u=1800
70/360
= 350× 1800
40(1 / 2) interest, the amount it becomesChoice (1) end
at the = 350 Choice
Choice (4)
(4)
130. The required time = hr = 2 hr [Lead
50 − 40 139. When a sum is invested to earn compoundfor
of a certain year becomes the principal
Solutionsfor
Solutions forquestions
questions145
145toto147:
147:
of 20 km to be covered at a relative speed thethe
interest, next year. it becomes at the end of a
amount
of 10 kmph] Choice (3) certain year becomes the principal for the next 145. The
145. The required
required percentage
percentage
year. The rate of interest =
8640 − 7200
×100 = 20 u1.3
131. The area of the triangle 7200
8640  7200 = 20 ×1.3
u 100 § 20.8 per cent Choice
The rate of interest = u100 = 20 =
100 u1.25 × 100 ≈ 20.8 per cent
Pr oduct of the sides 7200 100 ×1.25
= 20 per cent Choice (3) (1)
per cent Choice (1)
4 × circum radius 140. Choice (3) 146. As we do not know percentage increase or
144 N 146. decrease
As we doin not knowofpercentage
revenue increase
any city apart from
= = 6 3 cm 2  Choice (4)140. Bangalore, we cannot
or decrease answer
in revenue ofthe
anyquestion.
city apart
4(2 3) Choice the
(3)
from Bangalore, we cannot answer
132. Speed of Suresh = 10/2 = 5 m / sec. M
180 147. Thequestion.
required percentage = 100/27 uChoice
100 (3)
When Ramesh finishes the race, Suresh = 370.4 per cent
147. The required percentage = 100/27 × 100
would have run 200 – 10 – 10 = 180 90 30° Choice (3)
60° = 370.4 per cent
m.Ratio of the speeds of Ramesh and B C  for questions 148 to 151:
Solutions Choice (3)
A
90 148. The required percentage
In ' MBA, AB = = 90 3 m.
tan 30q =
180 180 0.84 u 50  0.92 u100  0.68 u 200  0.78 u 250 u 0.88 u150
In ' NCA, AC = = 60 3 m u100
tan 60q 3 50  100  200  250  150
08_Section.indb 176 ? BC = 90 3 + 60 3 = 150 3 m 597 u 100 10/30/2009
= = 79.6 per cent Choice 11:54:34 AM
Solutions for Mock Tests  n  177

Solutions for questions 148 to 151: 191. Conation means ‘desire or will to perform
148. The required percentage an action’.  Choice (1)
192. Para 3, lines 7 – 8 Choice (3)
0.84 × 50 + 0.92 ×100 + 0.68 × 200 + 0.78 × 250 × 0.88 ×150 597 × 100
= ×100 =
50 + 100 + 200 + 250 + 150 750 193. Para 5 gives the authors view and vali-
dates options C and D. Options A and B
= 79.6 per cent Choice (1)
are the views of ‘others’ given in para 6.
 Choice (2)
149. The required average a s a < 11 and 5 < a. a can be 7 or 9. The
second statement alone is not sufficient. 194. Para 3 shows that colonialism was an
offshoot of industrialisation, and not its
=
(66 + 84 + 78 + 77 + 83) 250  Choice (1)
‘aftermath’. Choice (2)
× = 194.
5 100 158. The first statement alone is not sufficient
as we do not know the length and breadth 195. The last paragraph deals with ‘develop-
 Choice (2)
of the aquarium. ment’ as a whole and not in the area of
150. The required percentage ‘science and technology’ alone. Choice
The second statement alone is not suf-
0.80 ×100 + 0.90 × 200 + 0.84 × 250 (4)
= ×100 ficient as we have information about the
100 + 200 + 250 length and breadth of the aquarium. Even 196. A reading of the first four paragraphs
by combining both the statements, we leads to choice 1. Choice (1)
470 × 100
= ≈ 85.5 per cent Choice (2) cannot answer the question.
550  Choice (4) Solutions for questions 197 to 200:
151. The required total = 0.72 × 50 + 0.68 × 100 159. The first statement alone is sufficient as 197. The given statement says that scale driv-
+ 0.65 × 200 + 0.83 × 250 + 0.92 × 150 we know the number of members voted en strategy will succeed. That is, a focus
= 579.5 Choice (2) for the resolution. The second statement on large scale will succeed given India’s
alone is not sufficient as 12 members demographic. All the options exemplify
Solutions for questions 152 to 156: or validate this approach except Choice
voted against the resolution, the number
152. Among the given countries in the choic- of members voting in favour depends 4 which says this approach is not suitable
es, only in India, the production of rice upon the number of members not taking for emerging markets like India. Choice
increases every year. Choice (2) a stand. Choice (1) (4)
153. The required percentage 160. The first statement alone is not sufficient 198. Choice 4 if true suggests the acid rain
172 − 115 as we do not know the number of boys or worsens the situation by significantly
= × 100 = 49 ⋅ 6% . Choice (2) increasing the amount of decaying or-
115 girls or the total number of students ad-
mitted. ganic matter. This seriously weakens the
154. The required difference argument.
The second statement alone is sufficient
(350 − 150) + (400 − 180) + (440 − 170) Choice 1 compares the acidity levels of
as we know the number of girls in terms
mountain lakes with those of other lakes
= +(480 − 190) + (500 − 200) of the number of boys. Choice (2)
5 but makes no mention of acid rain. So
Solutions for questions 181 to 196: will not weaken.
= 256 million tonnes Choice (1)
181. The first line of para 8 (--- our adoration Choice 1 talks about underestimating
155. The average production of rice over the the effects of acidity levels while the ar-
of the motor car) and para 9 (I am not
given period for China gument is about the role played by acid
free of the disease myself) makes it clear
400 + 450 + 500 + 480 + 600 that the ‘disease’ is ‘admiration of the rain in determining acidity levels. So ir-
= motor car’. Choice (1) relevant.
5
Choice 2 dwells on where acid rain is
= 486 million tonnes; 182. Para 1, line 2 makes it clear that he hates found and not on the effect of acid rain on
∴ The required percentage the take-off stage the most. Choice (4) mountain lakes.
500 − 486 183. The second sentence in para 3 “this odd Choice 3 is irrelevant to the argument
= × 100 = 2.88 per cent ≅ 3 % result is explained by the method which as it is not about the causes of acid rain.
486
 Choice (3) divides deaths by passenger miles ---“  Choice (4)
makes option II correct.Option 2 says 199. Choice (1) if true would mean that men
156. The average production of rice for all the “the inverse relationship of … miles and
given countries over the given period. spend longer periods of time on the den-
deaths …” Choice (2) tist’s chair than do women (assuming that
980 + 1130 + 1225 + 1322 + 1472 184. “The different attitude” is to regard roads serious problems take longer to rectify
= as safe. Option 1 is the correct choice. The than do the not so serious cases). This
5
first sentence of para 6, “If we regard the could explain the statistics mentioned.
 1225.8 million tonnes which is closest
= railway and the skies as carrying risk,”  Choice (1)
to 1225 million tonnes which is the total gives the answer. Choice (1) 200. The author clearly indicates that refer-
production of the given countries in the
185. Refer to para 3. Choice (4) ences about socio-economic conditions of
year 1992. Choice (2)
186. Refer to para 8. Choice (2) people are more important. But, he does
Solutions for questions 157 to 160: not say anywhere that political events or
187. Refer to para 6. Choice (2) military campaigns are not important and
157. From the first statement alone, as c < 13 and
188. Refer to para 2. Choice (4) should not be included (option 1). He also
5 < a < b < c, a = 7, b = 9 and c = 11
does not claim that existing history does
The first statement alone is sufficient. 189. Para 4, line 8 Choice (3)
not talk at all about the socio-economic
From the second statement alone, 190. Para 2, line 18. Choice (2) conditions of that time (option 2). Option

08_Section.indb 177 10/30/2009 11:54:34 AM


178  n  Solution Manual

4 also cannot be inferred from the pas- Net profit for quarter ended (QE) 31-12- Solutions for questions 31 to 35:
sage. 01 = 553.42; ∴ Average profit per project 31. The percentage change in income for dif-
The author merely compares two differ- for the QE 31-12-01 ferent years is as follows:
ent issues and this is the spirit of option 3. 553.42
 Choice (3) = ≅ 6 lacs Choice (1) 13
90 2000 to 2001 = × 100% ≅ 25% ;
67.5
27.
Other (s) Total (s) A as 37.5
MOCK TEST 4 Period 2001 to 2002 = × 100% ≅ 60%
Income Income % of B 67.5
Quarter ended 10
Solutions for questions 21 to 25: 79.23 3236.89 2.44 × 100% = 33% ;
31-12-02 2002 to 2003 =
30
21. By observation we can see that only in Quarter ended
159.37 2604.23 6.11
the case of Oriental Bank the MC has 31-12-01 19
× 100% = 29% .
increased by more than 100 per cent 9months ended
2004 to 2005 = 65
296.23 9232.38 3.20  Choice (1)
(doubled) which is not the case with any 31-12-02
other bank. The refinery with the highest 9months ended
420.35 8052.89 5.21 32. By observation, in the year 2002 the in-
growth rate is MrPL which has seen its 31-12-01
MC increase by more than 500 per cent. ∴ come as well as the profit percent were
∴ The lowest is Q for quarter ended 31-12-02 the least. As the profit = Income × profit
MC of Oriental Bank 3235
×100 = × 100  Choice (2) percentage, profit in 2002 was the least.
MC of MRPL 4200  Choice (4)
~ 77 per cent Choice (2) 28. Depreciation for the year ended 31-03-02
=799.53 lakhs (This figure is 20 per cent 33. It happened in none of the years.
22. 100 per cent growth implies that the MC less than actual)  Choice (1)
has doubled, similarly a 116.66 per cent Actual depreciation = 799.53 × 5/4 ≅ 800 34. Income in the year 2006
growth implies that the MC has becomes × 5/4 = 1000; Difference ≅ 200
more than 21/6 times the original figure. 120
This means that the net profit is actually = × 84 = Rs 100 ⋅ 8crores ; Profit
This is happening with Orchid and Lu- 2539.34 – 200 = 2339.34; ∴ actual net in 100
pin, 2 companies out of the 10 companies profit = 2339.34 lakhs the year 2005 = 40 per cent of 84 =
listed. (excluding others). Hence 8 com- Rs 33.6 crores
panies have not registered an increase of Net profit as a per cent of actual value Profit in year 2006 = 130 per cent of 33.6
more than 116.66 per cent Choice (3) = Rs 43.68 crores
2539.34 2339 200
= x 100; + ;100 per Percentage of profit in the year 2006
23. MC of all Banks, Fertiliser, Pharma and 2339.59 2339 2339
Refinery companies is Rs 280505 crores. cent + (<10 per cent) = < 110 per cent = 43.68
This is 20 per cent of the MC of all com- = ⋅ 100 ≅ 43 per cent. Choice (2)
108.5 per cent Choice (3) 100.8
panies listed on BSE.
MC of all companies listed in BSE 29. Promoters share of Equity capital for QE 35. Profit in the year 2002 = 20 per cent of 30
1979.65 × 25 = Rs 6 crores
280505 ×100 31-12-02 = ~ 494.92
= = Rs 1402525 crores Profit in the year 2004 = 30 per cent of 65
20 100
= Rs 19.5 crores; ∴Required difference =
≅ 1403 thousand crores Choice (3) Remaining equity capital = 1479.65 – Rs 13.5 crores Choice (4)
494.92 = 1484.73; Remaining Equity
24. All Banks have registered an increase of Solutions for questions 36 to 40:
more than 20 per cent in their MC that 1484.73
shares = = 148.473 lakhs Pro-
is, 6 Banks. 2 Fertiliser companies (RCF 10 36. Number of women players participat-
and Godavari) and 3 Pharma companies moters share of net profit for QE 31-12- ing in Volleyball = 10/100 x 1,10,000 =
(Orchids, Auropharm, and Lupin) have 11,0000
50
registered an increase of more than 100 02 = 836.16 x = 418.08 Total number of players participating in
100 Hockey (both male and female) = 6/100 x
per cent in their MC. ∴ The ratio is 6 : 5.
Net profit allocated to the above eq- 1,50,000 + 4/100 x 1,10,000
 Choice (3)
uity shares = 418.08 lakhs; ∴ EPS = = 6 x 1,500 + 4 x 1,100 = 9,000 + 4,400
25. MC of all Pharma companies excluding 418.08 = 13,400
others in 2000 = Rs 36590 crores → (A) = 2.815 Choice (4) 11, 000
148.47375 Required per cent age = x 100
MC of ‘others’ banks in 2000 = Rs 95000 13, 400
crores → (B) 30. Ι → Provision for tax was increased
∴ (A) as a percentage of (B) by (75/40 – 1) x 100 = 87.5 per cent. 110 55
= x 100 = x 100 ≅ 80 per cent
∴ Ι is true  134 67 Choice (4)
36590
= × 100 = 38.51%  Choice (4)
95000 2669.15 27 4
Solutions for questions 26 to 30: ΙΙ → ≅ = 25 per cent . 37. For Golf: No. of male players = x
10807.25 108 100
26. Prince’s projects for the quarter ended 31- ∴ ΙΙ is true
3
12-02 = 120 1,50,000 = 6000; No. of trainers =
Prince’s projects for the project ended x 5000 = 150 100
ΙΙΙ → Percentage increase in earnings per
31-12-01  4.23  For Basket ball : No. of female players
share =  − 1 <100% . ∴ ΙΙΙ is
120  2.82  3
= = 90 = x 1,10,000 = 3300; No. of trainers
1.33 false Choice (3) 100

08_Section.indb 178 10/30/2009 11:54:35 AM


Solutions for Mock Tests  n  179

42. A α B, A α 1/C² and A α D2/3 ⇒ A α the original area, that is, it is not remain-
5
= x 5000 = 250 BD 2/3 ing the same here and hence this is not
100  Choice (1) the correct choice. Similarly the other
C2
6000 3300 choices can be checked mentally and the
: ; 300 : 99 ≅ 3 : 1 43. For A, discounted price = 36000 – 0.08 x correct answer can be found. Choice (3)
150 250 20000 – 0.05 x 16000 = 33600
 Choice (1) For B, discounted price = 36000 x 0.93 = 48. Let the speed of the car and the bus be
33480 ; Difference = Rs 120 Choice (2) x kmph and y kmph respectively.
38. For track events : Number of Coaches
40 13 44. If ‘x’ is the number of days A, B, C to- 100 160 200 60
= × × 5000 = 260 + = 6 and + = 5.5
100 100 gether take to complete the work, then x y x y
33 22 (A + B) take (x + ½) days and (B + C)
Number of Female players = × × Solving the equations, x = 50 and y = 40
1,10,000 = 7260 100 100 take (x + 200 per cent of x) days = 3x  Choice (2)
days. (work done by A, B in one day) +
20 21 49. Given x : y = 2 : 3 and y : z = 4 : 5
Number of male players = × × (work done, B, C in one day) − (work
100 100 done by A, B, C in one day) = work done ∴ x : y = 4 × 2 : 4 × 3, y : z = 3 × 4 :
1,50,000 = 6300 by B in one day 3×5
Number of sports persons/coach = x : y = 8 : 12, y : z = 12 : 15 ; ∴ x : y : z
2 1 1 = 8 : 12 : 15
7260 + 6300 ⇒ + − = Work of B per day
≅ 52 Choice (3) 2x + 1 3x x ∴ Share of x = 8/35 × 700 = 160
260  Choice (3)
6x + 2x + 1 − 6x − 3 2x − 2
39. For Cricket : Total earnings for male play- ⇒ = 50. Suppose Original price = Rs 100. After
3x (2x + 1) 3x (2x + 1)
1st decrease, Price = 90.
 28 
ers =  × 150000  × (1,00,000) =  …… (1) After 2nd decrease, Price = 90/100 ×
 100  90 = 81. Price to be increased to get the
Rs 42 x 108  ’s share of total earnings = 400/1800
B original Price = 100 − 81 = 19.
= 2/9 …(2) 19

Total earnings for female players = 20/100 Required percent increase = × 100 per
B’s share of earnings as a fraction of the total 81
× 1,10,000 x 50,000 = Rs 11 x 108 1900
work of B per day cent = per cent Choice (2)
Total earnings for cricket players = earnings = 81
53 x 108 work of all persons per day 51. Time taken to meet
For Lawn Tennis : Total earnings for male
players =6/100 × 1,50,000 × 5,00,000 = 2 2x − 2 1 Initial dis tan ce between them
Hence = ⇒ x = 4 =
Rs 45 x 108  9 3x (2x + 1) x Choice (2) Re lative speed
Total earnings for female players = 8/100
× 1,10,000 × 200,000 = Rs 17.6 x 108 175
45. A and B work together for 5 days. ∴ the =
Total earnings of Lawn Tennis players 27.5 + 22.5
15 + 10
62.6 − 53 work done = 5/10 + 5/15 = = 5/6 = 3.5 hours
62.6 x 108 =Required % = × 30
53 Remaining work = 1-5/6 = 1/6. Time taken  o they will meet 31/2 hours after 6:00
S
100 ≅ 18 per cent  Choice (2) by A to complete the remaining work = 1/6 a.m. that is at 9:30 a.m. Choice (3)
40. Given that Chess players = 1/2 Wrestlers × 10 = 5/3 days = 12/3 days Choice (2) 52. Let the number of benches in the class be
Also Chess players + Wrestlers 46. Consider the following two cases: x. When 3 students are sitting per bench,
Case1 Men Women Case 2 Men Women total number of students = 3x + 3 ---- (1)
1 4 3  No. 3x 2x No. 2x 3x When four students are sitting per bench,

= × 150000 + × 110000  Average a b Average a b
2 100 100  age age
total number of students = 4(x – 3)
 ----- (2)
--------------------------
10150 By equating (1) and (2) 4x – 12 = 3x + 3;
= 10150; Chess players = ≅ 3380
3 3ax + 2bx 2ax + 3bx x = 15 Number of students = 4(12) = 48
= 26 = 24  Choice (1)
Now, no. of female basket ball players 5x 5x
3 53. New ratio = 3 × 1.75 : 4(1.60) : 5(1.90)
= x 110000; Required percentage =  a + 2b = 130
3 ------- (1) = 5.25 : 6.40 : 9.50 = 105 : 128 : 190
100 2a + 3b = 120  ----------(2)  Choice (4)
3380 Solving (1) & (2) a = 30 and b = 20 Re-
× 100 ≅ 100 per cent Choice (2) quired difference = 30 − 20 = 10 years 54. A’s investment = 80 per cent of 10,000 =
3300 Rs 8,000 ; B’s Investment = 50 per cent of
 Choice (4)
32,000 = Rs 16,000
Solutions for questions 41 to 60: 47. The fastest way is to check from the an- Let C’s investment = x per cent of 36000
41. S.P of each type of fan = Rs 12,000 ; swer choices. For example, taking choice = Rs 360x ; Ratio of profits = 1 : 1 : 1
C.P of ceiling fan = 12000 x 100/120 = (1), that is, x = 25, the length is decreased ⇒ 8000 × 12 : 16000 × 6 : 360 × x4
Rs 10,000 by 25 per cent and breadth by 100 per =1:1:1
As the C.P’s are in the ratio of 5 : 6, C.P of cent. So, the new length is 0.75 times the ⇒ 6x = 400 ⇒ x = 66.66 Choice (4)
table fan = 6/5x10,000 = Rs 12,000 original length and the new breadth will
be twice the original breadth. Hence the 55. Let speed of John be x kmph. Wind speed =
As C.P of S.P, of table fan are equal, there
new area will be 0.75 × 2 that is, 1.5 times 5 kmph
is neither profit, nor loss. Choice (3)

08_Section.indb 179 10/30/2009 11:54:36 AM


180  n  Solution Manual

 1 = distance covered in t sec when going


S speed while going upstream = (x − y) kmph (A) concludes. Choice (4)
against the wind. Case 1 68. The passage talks of common colds. (A)
S2 = distance covered in t sec with the Time taken for the entire journey would be appropriate as the opening state-
wind.
16 24 ment. (D) follows, elaborating (A). CB
Since S2 = 2S1 ⇒ 2(x – 5)t = (x + 5)t + = 6 ------- (1) and in 2nd
x − y x +y go together, (C) talking about structural
(since t ≠ 0) x = 15 kmph. Choice (1)  problems and (B) giving an example.
b a+b a 12 36  Choice (3)
56. = = --- (1) Consider case. + = 6 ------- (2)
a −c c b x−y x+y 69. The passage talks about the Indian auto
b a+b  x = 8 kmph y = 4 kmph ∴Speed of the
∴ components industry. (B) introduces the
= boat in still water = 8 kmph. Choice (3) subject. DC then follow, (D) elaborating
a −c c
(B) and (C) giving in example for (D).
⇒ bc = a2 – ac + ab – bc ⇒ 2bc = a2 +ab –ac 60. Let the number of one rupee coins with
(E) then follows, showing that in spite of
 --- (2) Karan be x. Number of two rupee coins
all this, there are still problems. (A) con-
Now, a(a – c) = b2 ⇒ a2 –ac = b2 with him = 30 – k
cludes. Choice (1)
 ------ (3) x + 2(30 – x) = 52 ⇒ x = 8
 Choice (1) 70. (A) introduces the topic. (C) follows, for
Substituting equation (3) in (2), we get
the ‘it’ in (C) refers to the agreement men-
Solutions for questions 61 to 65: tioned in (A). BC go together, (C) elabo-
2bc = b2 +ab. ------ (4)
61. Choice (3) is the right answer. ‘Cower’ rating on (B). Choice (2)
a a+b
But = ⇒ ac = ab +b2 --- (5) and ‘grovel’ do not fit in the second blank,
b c Solutions for questions 71 to76:
ruling out choices (1) and (2) respective-
Substituting (5) in (4), we get 2bc = ac ly. In choice (4) ‘conjecture’ would not be 71. Choice (4) is the best answer. In
a appropriate. Choice (3) choice (1), ‘of being’ us not idiomatic. In
⇒ =2 choices (2) and (3), ‘is owed much’ is not
b 62. Choice (3) is the right answer ‘Avoid’
idiomatic. Choice (4)
2 2 would not be appropriate in the first
a − ac a ac 72. Choice (3) is the best answer. Choice (1)
From (3), =1 ⇒   −   =1 blank, ruling out choice (1). In choice (2),
b2 b bb ‘penetration’ does not fit in. ‘Defy’ would is verbose and awkward. In choices (2)
c c 3 not be appropriate in this context, ruling and (4), ‘more’ is inappropriate. It should
⇒ (2)2 – 2 =1 ⇒ = ; ∴ a : b : c = 4 out choice (4) Choice (3) be ‘most’ Choice (3)
b b 2
: 2 : 3 Choice (1) 63. Choice (2) is the best answer. In choice 73. Choice (3) is the most appropriate answer.
(1), ‘despondent’ and ‘systematic’ do Choices (1) and (4) contain misplaced
57. Let the work done by each man and each not go together. An expression cannot be modifiers and seem to suggest that the
woman in a day be ‘m’ units and ‘w’ units ‘planned’, ruling out choice (3). In choice ‘notes’ were asked to speak at the conven-
respectively. (4), ‘sober’ and ‘desultory’ do not go to- tion. Similarly, choice (2) also suggests
The total work = (3m + 4w) 10 = (11m gether. Choice (2) that the convention members were asked
+ 3w) 4 to speak. Choice (3)
⇒ 2w = m. 64. Choice (3) is the most appropriate answer
‘Incentive’ and ‘service’ do not fit in the 74. Choice (2) is the best answer. Choice (1)
Now, the total work = [3(2w) + 4w] × 10
second blank, ruling out choices (1) and is verbose and awkward. Choice (3) is
= 100 w. ∴ The time taken by 24 men
(2) respectively. In choice (4), ‘milestone’ also awkward in construction – the two
is not appropriate. Choice (3) –ing forms have to be separated with a
and 2 women to complete the work =
conjunction. Choice (4) is grammatically
100 w 100 w 100 w 65. In this context, the word that has to go
= = 2 incorrect because the conjugated verb
= into the first blank has to be more or less
24 m + 2 w 24 (2w) + 2 w 50 w ‘screamed’ immediately before the verb
synonymous with ‘separation’. The only
days Choice (3) ‘were’ is not possible. Choice (2)
word that is appropriate is ‘division’. All
58. Let the distance between Hyderabad and the other choice are ruled out. 75. Choice (3) is the best answer. ‘Gas’ is not
Mumbai be x km. Time taken by each  Choice (2) a feature, ruling out choices (1) and (4).
In addition, ‘air condition’ in choice (4)
train to travel x km is 14 hours. Speed of Solutions for questions 66 to 70: is inappropriate. In choice (2), it should
x 66. The passage talks about the problems be ‘safety devices’ and not ‘safe devices’.
each train = kmph. Let both the trains caused due to the different response time  Choice (3)
14 in computer systems. (D) would be appro- 76. Option 1 is erroneous because the to
meets t hours after 10:00 a.m. priate as the introductory sentence. (A) +verb + ing form structure is erroneous.
x x follows, giving one more instance where The to infinitive cannot go with the ‘ing’
(t + 2) + t=x⇒t=6 problems are caused. DA having been form. Option 3 is erroneous because the
14 14 about slow response time, (B) follows, word ‘avoiding’, that is the ing form of
 o both the trains would meet 6 hours
S talking about fast response time. (C) then the verb, is not in keeping with the pre-
after 10:00 a.m. that is at 4:00 p.m. talks about the optimum response time ceding verbs (explain and understand etc)
 Choice (4) and (E) concludes. Choice (2) which are all in the simple present. Op-
59. Let the speed of the boat in still water be x 67. Choice (4) is the best answer. (D) opens tion 4 is incorrect because the word ‘im-
kmph. And, the speed of the stream (water the passage by talking about the central provement’ which is a noun is incorrect
flow) = y kmph. budget. BC go together – (B) talking about as it is not in keeping with the preceding
Relative speed of the boat while going growth and (C) bringing out the obstacle verbs. Therefore only choice 2 is correct.
down stream = (x + y) kmph. relative for it. (E) follows elaborating on (C) and  Choice (2)

08_Section.indb 180 10/30/2009 11:54:37 AM


Solutions for Mock Tests  n  181

Solutions for questions 77 to 80: 4: tar war far jar bar car;  5: car tar jar far
Spoons
Hence, only Conclusion ,, ,, and ,V are true in
77. Part 3 is erroneous. The word ‘have’ war bar
87. Here, we need to follow the reverse Choice (3) of
process both the diagrams. Choice (3)
rearrangement that is number the words as Plates
does not agree with the subject ‘relation- 89. 6 By1 continuing
4 3 2 5 the series given in the 92. The following diagrams can be drawn for the
ship’ which is singular. The correction is data,
and rearrange them as given statements:
Tumblers
‘The relationship between ---------- has 1 3: 2pet wet
3 4bet 5 let get
6 set
Spoons
changed ----------‘.  Choice (3) for4:the
setprevious
pet let bet code.
wet get
3: hit sit fit bit kit wit; 2: sit kit bit fit wit hit
78. The error lies in part 3. The verb ‘give’ 5: get set bet let pet wet
1: kit wit fit bit hit sit Choice (2)
BottlesPlates

does not agree with the subject ‘control’. Here the 5th and the 1st are the same.
88. 2:Hence,
bar car far
thejarpassword
tar war; 3:will
war bar
be jar far car tar
repeated Tumblers (ii)
The correction is ‘control over intangible 4: tar war far jar bar car; 5: car tar jar far war bar (i)
assets -------- gives companies -------’. after the fourth person logs in. Choice (3)
 Choice (1) From the above diagrams:
 Choice (3) Bottles
89. By continuing the series given in the data, Conclusion: Ι (affirmative) is true and
79. The word ‘apotheosis’ should be followed 90. 3:3:pettinwet
binbetkinletsin
getpin
set win;  4: win tin sin ΙΙ (affirmative) is true.
4:kin
set bin
pet pin
let bet wet get (ii)
by the preposition of. Something is an From diagram(i)(i)
5:5get
: pin setwin
bet kin
let pet
sinwet
tin bin;  6: bin pin sin From the above diagrams: is false and
‘apotheosis’ of something. Hence part 2 Here the 5th and the 1st are the same. Hence, Conclusion: ΙΙΙ (affirmative)
kin win tin Conclusion: , (affirmative) is true and
has an error. Choice (2) the password will be repeated after the fourth ΙV,,(negative) is true.
(affirmative) is true.
Alternate
person logssolution:
in. Choice (1) From diagram
80. Part 4 has an error. The word ‘stringent’ From diagram(ii)
(i)
Here, the passwords for the sixth and the Conclusion:
Conclusion:ΙΙΙ,,,is(affirmative)
true and ΙVisisfalse
false.
and
should be followed by the preposition 90. 3: tin bin kin sin pin win; 4: win tin sin kin bin pin
5:second
pin win persons are
kin sin tin the6:same.
bin; bin pin sin kin win tin Hence, either ΙΙΙ
,V (negative) or ΙV follows and Ι and
is true.
‘in’. Choice (4) 3: tin bin
Alternate kin sin pin win ; 2: bin pin sin
solution: From diagram (ii)
ΙΙ follow. Choice (4)
kin win
Here, tin
the passwords Choice
for the sixth and (3)
the second Conclusion: ,,, is true and ,V is false.
Solutions for questions 81 to 85: persons are the same. 93. The following
Hence, either ,,,diagrams represent
or ,V follows the,, follow.
and , and
81. E belongs to T. Choice (1) Solutions for kin
3: tin bin questions 91 ;to2:95:
sin pin win bin pin sin kin win tin given statements. Choice (4)
Choice (3)
91. The basic diagrams for the given state- 93. The following diagrams represent the given
82. In case (1) D is sitting at the extreme right statements. Translation
Solutions for questions 91 to 95:
ments is Translation
end, where A who is from P is sitting at
the extreme left end. Choice (2) 91. TheAccording to the above
basic diagrams for the diagram:
given statements is Speech
Speech
According
Conclusion:to the above diagram:
Ι (negative) is true.
83. In case (2) the student from S can sit at Conclusion: , (negative) is true.
ΙΙ (affirmative) is true, hence it is always
Essay
Essay
the extreme right end, where A is sitting to ,, (affirmative) is true, hence it is always true.
,,,true.
(negative) is true
the immediate right of the student from Q. (negative) is true
,VΙΙΙ (negative) is true.
 Choice (3) ΙV (negative) is true. Review
Review
84. In either case B can be from Q.
Stars (i) (ii)
∴ We cannot determine the position of E. (i) (ii)
 Choice (4) Clouds Planets From
From figure(i):
figure (i):
Conclusion: , (affirmative) is false.
Conclusion: Ι (affirmative) is false.
85. In case (2) exactly one person is sitting ,, (negative) is true.
ΙΙ (negative)
,,, (negative) is is
true.
true.
between A and E, where A is sitting between
(negative)
ΙΙΙ,V (negative)isistrue.
true.
the students from S and Q. Choice (1)
(negative)
ΙVFrom is true.
figure (ii):
Solutions for questions 86 to 90: From figure (ii):
Conclusion: , (affirmative) is true.
The code for each of the next person is obtained Conclusion:
,, (negative) Ι (affirmative)
is true. is true.
by rearranging the words as shown below. Satellites ,,, (negative)
ΙΙ (negative) is is false.
true.
First person: get  set  bet  let  pet  wet The following alternate diagram can be drawn for ΙΙΙ,V (negative)isisfalse.
(negative) false.
the given statements. Conclusion ,, is always true.
1   2   3   4   5   6 The following alternate diagram can be ΙV (negative) is false.
Stars Conclusions , and ,V are complementary pair, if
Second person: wet  get  let  bet  set  pet drawn for the given statements. Conclusion
one is trueΙΙthenis always true.
the other is false. Choice (2)
6   Conclusions Ι and ΙV are complementary
Stars 94. The following diagrams represent the given
1   4   3   2   5 pair, if one is true then the other is false.
statements.
Clouds
 Navy Choice (2)
86. 1: cat sat fat hat rat mat;  2: mat cat hat
fat sat rat Satellites 94. The following diagrams represent the
Clouds given statements.
3: rat mat fat hat cat sat;  4: sat rat hat fat Army
mat cat Choice (4) Planets Navy
Satellites
From the above diagram.
87. Here, we need to follow the reverse pro- Conclusion , is true.
cess of rearrangement that is number the Planets Defence
Conclusion ,,, is false and Conclusion ,V is true. Army
words as
6  1  4  3  2  5 From the above diagram. Air Force
Conclusion Ι is true. Sol/998
and rearrange them as
Conclusion ΙΙΙ is false and Conclusion ΙV (i) Defence (ii)
1  2  3  4  5  6
for the previous code. is true.
3: hit sit fit bit kit wit; 2: sit kit bit fit wit Hence, only Conclusion Ι, ΙΙ and ΙV are Air Force

hit true in both the diagrams.


(i) (ii)
1: kit wit fit bit hit sit Choice (2)  Choice (3)
From figure (i) and (ii)
88. 2: bar car far jar tar war;  3: war bar jar 92. The following diagrams can be drawn for Conclusion: Ι (negative) is true. ΙΙ (affir-
far car tar the given statements: mative) is true.

08_Section.indb 181 10/30/2009 11:54:37 AM


100. Among the given statements, A and C are
182  n  Solution Manual adjacent to each other in both (i) and (ii).
Choice (4)
From figure (i) and (ii) ΙΙΙ (negative) is true and ΙV (affirmative) Solutions for questions 101 to 120: in which Atal speaks after Sonia is
Solutions for questions 101 to 120:
Conclusion: , (negative)is istrue.
true. ,, (affirmative) is 101. Boric acid Water 6!
Hence, all follow. Choice101.
(4) Boric acid Water = 360  Choice (1)
true. 2!
95.
,,, (negative) is true andThe
,V following diagrams
(affirmative) represent the giv-
is true. 76 0
en statements. Choice (4) 109. There are 26 black cards, 4 kings and 2
Hence, all follow.
black kings.
Stem ∴ By Addition theorem, Required
The following diagrams represent the given
Probability P(A ∪ B) = P(A) + P(B) − P
statements. 56 20
Stem Root 3 26 4 2 28 7
Let x cm of water be added. Then, (A ∩ B) = + − = =
Let x cm of water be added. Then,
3
52 52 52 52 13
Branch 76% of 84 56
Root = Ÿ x = 30 Choice (4)  Choice (3)
Tree 26% of 76% x 84 44= 56 ⇒ x = 30
84  of
26% of 84 + x 44  110. Let the number of green balls be x When
Branch 102. Let the total number of goods be G. Number of a ball is drawn at random, the probability
 Choice (4)
Tree (ii) G 5 G 10
(i) second quality
102. Let goods
the total u be G.
sold isof goods
number of getting a red ball =
5 6 6 10 + x
From diagram (i): Number of second quality goods sold is
G The probability of getting a green ball
Conclusion: Ι (affirmative) is false.
ΙΙ (affirmative) is true. G 5 G 6 1 x x  10 
Required × = = . Given =2  ∴
(i) ΙΙΙ (affirmative)(ii)
is false and 6 6 = 2G G4 .
5 fraction Choice (1)
10 + x 10 + x  10 + x 
From diagram (i): ΙV (negative) is true. 1
Required fraction3= 6 = .Choice (1) x = 20 Choice (3)
Conclusion: , (affirmative)From is diagram
false. (ii): 2G 4
,, (affirmative) is true. Conclusion: Ι is true. 103. Let the original strength in 3 each section be ‘x’.
111. From ∆PTB
ΙV is false. Let theLetaverage weight of theinfirst T
,,, (affirmative) is false and 103. the original strength eachsection
section be 3k
ΙΙΙ is true
,V (negative) is true. kgs andbethat
‘x’. of
Letsecond section
the average weightbeof2k first Given tan60° = h
thekgs.
Hence either Ι or ΙΙΙ and ΙΙ follows. 3kx – 2kx = 150 kg. Ÿ kx = 150 kg. x
h
From diagram (ii): section be 3k kgs and that of second sec-
 Choice (4)
Conclusion: , is true. As k istionunknown,
be 2k kgs. we
Givencannot find= 150
3kx – 2kx the kg.
average
h = 3 x …… (1) A 30° 60° P
,V is false. Solutions for questions 96 to 100: weight ⇒ of kx
the=two
150 sections.
kg. Choice (4) 50 B x
,,, is true From (1) and (2), we get, As k is unknown, we cannot find the aver-
104. To cover 39 km time taken by the
age weight of the two sections.
train (without
Hence either , or ,,, and ,, follows. Choice (4) h
(i) _____ B/D ___ C ____ D/B ____ ____  60 From ∆PTA, tan 30° =
stoppages) = u 39 = 52 min Choice (4) 50 + x
utions for questions
(ii) 96 to 100:
_____ ____ B/D C ____ D/B ____ 104. To cover45
39 km time taken by the train
m (1) and (2), we get, The train takes 60 min to cover 39 km when it 1 h
Also given that E and F sit together. 60 = ⇒x= 3 h − 50 …… (2)
(without
stops. So stoppages)
the train = for× 39
stopped 60=–5252min
= 8 min 3 50 + x
_____ B/D ___ C ∴ The D/B
____ above____
arrangement
____ becomes 45
in one hour. Choice (2)
_____ ____ B/D C ____ D/B ____ The train takes 60 min to cover 39 km From equations (1) and (2) we get
(i) B/D ___ C ____ D/B E/F F/E when it stops. So the train =stopped
105. We know that, correct mean wrongfor 60 + ¦
mean h 2h
o given that E and (ii)
F sit E/F F/E B/D C ____ D/B ____
together. – 52 = 8 min in one hour.
(correct observation – wrong observation)Choice (2) = 3 h − 50 ⇒ = 50 ⇒ h = 25
he above arrangement  3 3
From becomes
(3) and (5), we get the above possible 105. We
(17 know
15) that,
(7 correct
23) mean 2= wrong 3 metres Choice (1)
cases as follows. = 32 + – wrong
= 32 + mean + ∑ (correct observation = 32 + 0.05
B/D ___ C ____ D/B E/F F/E 40 40 112. As the sum triples to Rs 1200 in four
E/F F/E B/D C ____ D/B ____ observation)
(i) G D A C B E/F F/E = 32.05 Choice (2) years at C.I., ⇒ Sum = Rs 400
(17 + 15) − (7 + 23) 2
(ii) the
m (3) and (5), we get E/Fabove
F/E D C A B cases
possible G as = 32 + = 32 +  ow Rs 1200 is the principal. It amounts
N
106. Edge of the cube =405 cm. Thickness 40 of the
ws. to Rs 3600 in another four years. ⇒ ∴
96. From the given condition, we get:- smaller=slice32 + =
0.051 cm. l = 5 cm, b =Choice
= 32.05 5 cm (2)
Interest for first eight years
G D A C B E/F F/E (i) G D A C B E F Area of each small slice, which is in the form of a
106. Edge of the cube = 5 cm. Thickness of the = 3600 − 400 = Rs 3200 Choice (4)
E/F F/E D C A B G
(ii) F E D C A B G cuboid = 2h(l + b) + 2lb= 2 x 1 (5 + 5) + 2 x 5 x 5
in both the cases, the number of persons = 20 + smaller 50 = 70slicecm= 1 cm. l = 5 cm,Choice
2 b = 5 cm
(1) 113. A regular polygon with fixed perimeter
between A and E is two. Choice (3) A
 rea of each small slice, which is in the
From the given condition, we get:- has a maximum area if it is a regular poly-
(i) G D A C B97. E F Fcan be adjacent to B in (i) hence,107. Area ofform of a cuboid
the room = 2h(lto+the
available b) +students
2lb= 2 x 1=(5(18  2)
gon. Among all these regular polygons of
we += 5)224
+ 2 xm52x 5
(ii) F E D C A B G get (16  2) the fixed perimeter, greater the number of
As each = 20 + 50 = 70
student cm2 1 m2 area,
requires Choicea(1)total of
in both the cases, G theD number
A C B of F Epersons sides, greater the area. Choice (4)
between A and E is two.Here A is third from Choice 224 students can be accommodated in the
left.(3) Choice (1) 107. Area of the room available to the students =
classroom.
(18 − 2) (16 − 2) = 224 m2 Choice (1) 114. When A had run 500 m B would have run
F can be adjacent to 98.
B inFrom (i) hence, we only
(i) and (ii), get B, D and F can be 485 m. When B had run 500 m C would
As ways
each student requires the
1 m2speeches
area, a totalare 6!.
G D A C B F E adjacent to E. Choice108.
(2) The total of 224
of arranging
students can be accommodated
have run 480 m.
Here A is third from left.In (i), the number Choice (1) between D Of these, in half the cases, Atal speaksinbefore When D had run 500 m C would have run
99. of persons Sonia the andclassroom.
the other half, after Choice Sonia.(1)So the 475 m. When D had run 500 m A would
From (i) and (ii), only B,and DG andis ‘0’ but inbe
F can (ii), it is ‘3’.Choice (4) number
adjacent 108. The oftotal
ways,
waysin which Atal
of arranging speaks after
the speeches have run
to E. 100. Among the givenChoice statements,(2) A and C are are6!6!. Of these, in half the cases, Atal
adjacent to each other in both (i) and (ii). Sonia is 360 Sonia and the other
speaks before Choice
half, (1)  500 500 475 
= 500  × ×  = 490.10 = 490.
In (i), the number of persons  between D and G isChoice (4) 2! Sonia. So the number of ways,
after  485 480 500 
'0' but in (ii), it is '3'. Choice (4)
Sol/999

08_Section.indb 182 10/30/2009 11:54:39 AM


Solutions for Mock Tests  n  183

So A beats D by 10 m approximately.  otal Kerosene consumed by India


T 147. The student obtained the least in Botany
 Choice (2) (Quantity) = 150 lakh kilolitres. with 30 marks. Choice (3)
∴ Value of 1 litre of Kerosene =
115. The number of seconds after which 148. The maximum difference between the
they toll together is LCM (6, 7, 8, 9, 12) = 7, 200 × 100 (in lacs) marks obtained by student and average
; Value of 1 Litre
(23 × 32 × 7) sec 150 ×1000 marks is 90 – 20 = 70 for mathematics.
The number of times they toll together in  Choice (1)
of Kerosene = Rs 4.8 Choice (2)
149. The convergence between the marks ob-
60 × 60 142. The graph here shows the consumption of
1 hour = = 7.14 ~ 7 Choice (3) tained by student and average male marks
23 × 32 × 7 electricity by India but does not state any-
exists in commerce. Choice (2)
thing about the production of electricity.
116. Perimeter of Rectangle = 2 (l + b) = 4 × Hence the amount of electricity generated 150. There is a difference between the marks
Side of square. ⇒ 2 (l + 15) = 4 × 20 in India cannot be estimated.  Choice (4) obtained by student and average female
4 × 20 143. Price of 1 Barrel of crude oil imported marks for all values. Choice (4)
∴l = −15 = 25.
2 151. The marks obtained by the student is not
60, 000 × 0.3
by India = (250 million = closest to either average male marks or
∴ Area of Rectangle = l ×2 b = 25 × 15 = 15 25
Area of Square a 20 × 20 16 average female marks. Choice (4)
2500 lacs = 25 crores)
 Choice (1) Solutions for questions 152 to 156:
 Price of 1 barrel of Crude oil imported

117. It is given that the radius and depth of the by India = Rs 720. 152. Let the original cost of each book and
conical vessel are 6 cm and 8 cm respec- Price of 1 barrel of oil that Oman sells to original number of books be denoted by
tively. India = Rs 720. c and b.
Volume of the water over flown = Volume Price of Crude oil in Oman (domestic) is From statement Ι, (c – 10) (b + 20)
4 25 per cent more than Rs 720 = Rs 900. = c . b. From statement ΙΙ, (c + 15)
of the sphere = π(3)3. Required ratio  Choice (1) (b – 20) = c . b.
3
Neither of the statement is independently
4 144. Current Domestic Production of Crude
π(3)3 sufficient to find the value of c as there
3 3
= =  Choice (1) 25 are two unknowns and one equation. Us-
1 8 oil = 250 × = 62.5 million barrels
π(6) 2 (8) 100 ing both the statements, the C.P of each
3 book can be found.  Choice (3)
16 per cent of TRR = 62.5 million
118. For any values of x and y, two successive
discounts of x per cent and y per cent is Barrels.; ∴ TRR = 153. Statement Ι gives the total amount of debt.
always equal to two successive discounts 100 Statement ΙΙ gives the period in which the
62.5 × = 390.625 million Barrels .
of y per cent and x per cent and is less 16 debt has to be repaid. But no information
than or equal to a single discount of (x + regarding the rate of interest is given in
 e know that 100 per cent more implies
W either statement. Hence the question can-
y) per cent.
twice. Similarly 200 per cent more im- not be answered. Choice (4)
But two successive discounts of x per
plies thrice.
cent and y per cent is more than (for 154. Since the direction in which A and B are
In the scene every 4000 per cent more im-
x ≠ y) or equal to (for x = y) two succes- travelling is not known, the distance be-
plies 41 times. ∴ TER are 41 times the
x+y tween them cannot be found. Choice (4)
sive discounts of per cent each. TRR.
2 ∴ TER = 41 × 390.655 ~ 16000 million 155. From statement Ι, 8n + 5n is divisible by 8
 Choice (4) barrels.  Choice (4) + 5 = 13. This is possible for only odd
119. Since the vertical angle of the right circular
145. Total value of Crude oil consumed in In- values of n. Hence the nature of n can be
cone is 90°, Its radius = Its height ⇒ h
dia = 60,000 × 0.3 = 18,000 crores. found out. Choice (1)
=r=7
Domestic production = 250 × 0.25 = 62.5 156. Using both the statements also the ques-
Volume of the solid = volume of cylinder
million barrels tion cannot be answered as neither the
1 Crude oil from sources other than Domes-
+ volume of cone = π (7)2 ×10 + π(7)3 cost price nor the list price (or the selling
3 tic = 187.5 Mn barrels. Assuming 1 barrel price) is mentioned. Every thing is men-
7 22 37 from other sources costs Rs X, then the tioned in terms of percentages. Hence the
= π (7)2[10 + ]= × 49 × = 1899 price of 1 barrel of Domestic Crude costs
3 7 3 question cannot be answered. Choice (4)
1 3X
cubic centimetre Choice (4) = Solutions for questions 157 to 160:
3 4
3X 157. x + y = 12 ----(1)
 187.5 X + 62.5 ×
∴ = 18000.
120. Let the price be Rs x ∴ The increased 4 x – z = 10 ----(2)
price will be Rs (1⋅3)x. As x is an integer, ⇒ 234.375 X = 18000 ⇒ X = y + z = 2 (subtracting (2) from (1)) y + z
the increased price must be a multiple of = 2 is less than 3.  Choice (2)
18, 000 × 100 (in lacs)
1⋅3. Among the given choices, 7.00 is not 158. By cross-multiplying, x2 = 36 ⇒ x = +6 or
234.375 ×10 (in lacs)
a multiple of 1⋅3. Choice (3) ⇒ x = –6
⇒ X = Rs 768 Choice (2
Solutions for questions 141 to 145: ) If x = +6, the values in both the columns
Solutions for questions 146 to 151: is equal. If x = –6, the value in column
141. Total value of Kerosene consumed = (A) is less.
12 146. The student obtained maximum marks in ∴ The relationship cannot be determined.
60,000 × = 7200 Crores mathematics with 90. Choice (2)
100  Choice (4)

08_Section.indb 183 10/30/2009 11:54:39 AM


184  n  Solution Manual

159. m1d1 = m2d2; (x – 6) x 10 = (y + 3) x 15 is required before training the legisla- Solutions for questions 181 to196:
⇒ 2x = 3y + 21 tion but not after. Ι is definitely not the 181. Refer to the penultimate sentence of para
Since x and y are positive integers 2x > course of action, because unless there 3,– ‘ Taken together the work of Kohler
3y. Choice (1) is a keen study, one can not pass any and Tolman….laid the foundation for…
legislation.
160. Let the cost price of the item be $x, then the cognitive approach.  Choice (3)
So, Ι is not a proper course of action.
the selling price = $1.5x. ∴ Profit = 0.5x 182. Nothing is said in the passage about the
Operationalisation of legislation is signi-
= 40 use or non use of practical experiments by
fied by taking action under the legislation.
∴ S.P of the item = 1.5x = $120. the behaviourists. Choice 1 is supported
Hence, ΙΙ is a proper course of action.
 Choice (1) by para 2, line 2. Choice 3 is validated
 Choice (2)
by the first para. Choice 4 is true because
Solutions for questions 161 to 164: 163. Ι is definitely the course of action, because the behaviourist used only external events
161. Ι is not the correct course of Action, be- by doing so, the threat for secularism will (para 4, trial and error) whereas the cogni-
cause we do not know whether the Army be reduced to a greater extent. tivists believed in insight.  Choice (2)
are citizen-friendly or not. So, Ι does not ΙΙ does not follow, because this is against
secularism. Choice (1) 183. Refer to the penultimate sentence of the
follow.
last para.  Choice (4)
ΙΙ definitely follows, because this will im- 164. Ι is definitely a course of action, because
prove the relation between the Police and this will help to have a broad idea about the 184. Thinking belongs to cognitivism.
Citizens. Choice (2) PMS and its pros and cons.  Choice (1)
162. According to the statement the legisla- ΙΙ is not the course of action. 185. The author associates remoteness with
tion is already passed, hence survey  Choice (1) the phrase ‘in his living immediacy’. This
is implied in the first para of the passage.
 Choice (3)
Solutions for questions 165 to 169:
186. The last sentence of para 1 states that our
(A) (B) (C) inherent property refers to the ability to
Even number First or last digit is perfect Middle digit is prime exchange experiences. Choice (1)
Q. No. Number [ 1 ] Second or square number
second last digit [ 2 ] Both first & last digits [ 3 ] Sum of the digits is 187. According to the 3rd, 4th and 5th sentenc-
is even are even prime es of para 2, choices 1, 2 and 4 are true.
165. 2875649 × [ ]   Choice 3 is not true. The 5th sentence
of para 2 states that it was anything but
166. 5123876  ×[×] 
experience which was shared. Refer to
167. 1647326    the line ‘what ten years later was poured
168. 6863728  × []  ……..’. Choice (3)
169. 9329312   × [ ]
188. Lines 6, 7 and 8 of para 2 state that
‘contradictions’ reveal contrast between
the abilities to convey experiences.
 Choice (2)
165. Special number Choice (2) Solutions for questions 177 to 180:
189. Refer to paragraph 2, line 2 ‘but the ef-
166. Scrap number Choice (1) 177. C’s sibling’s father is C’s father as well.
fect is _ _ _ firstborns’ (2) is false. Refer
B is C’s mother. C’s father’s sister is
167. Super number Choice (1) to sentence (1) of paragraph 2 as it says
C’s aunt, B’s (C’s mother) sister-in-law.
168. Subordinate Choice (3) ‘birth order contributes more’. (3) is not
 Choice (2)
true as conflict causes some first-borns to
169. Unconventional number Choice (4) 178. Between 2007 and 2001, the number of behave like laterborns, according to the
years is 6, of them only one year is a leap last line of paragraph I, it also ‘disrupts _
Solutions for questions 170 to 174:
year. Hence the number of odd days is −(1 _ _ strategies.’ Choice (1)
170. A is true but R is false. Choice (3) × 2 + 5 × 1)
190. Refer to paragraph 2. the lines ‘Freud con-
171. A is false but R is true. Choice (4) ⇒ −(7) ⇒ 0 odd days
vinced himself. . .’ and ‘For a young child
Hence, 2001 5th September also falls on
172. Both A and R are true and R is the correct . . . view’. Hence 2 is true, Option 4 is not
Thursday. Choice (2)
explanation of A. Choice (1) true because, though they differed in their
179. When the normal clock runs for 180 ideas, it is not mentioned that this formed
173. A is true but R is false. Choice (3) minutes, the clock in question runs for the foundation of the psychoanalytic theory.
174. Both A and R are true and R is the correct 179 minutes. As the clock runs for 179 Darwin studied about ‘birth order influence’
the explanation of A. Choice (1) hours (24 × 7 + 11) the normal clock and Freud studied about parent-offspring
runs for 180 hours. that is, it shows conflict. The ‘link between’ the two was not
Solutions for questions 175 and 176: 4:00 PM. analysed by them. Hence (3) is ruled out.
175. The given series is  Choice (2) Option 1 gives the opposite idea.Choice (2)
9×2 + 2, 20× 2 + 4, 44×2 + 6, 94× 2 + 8, 196× 2 + 10, 180. At 11hr 30min, if the minute hand points 191. Refer to paragraph 5. (2) is not true as it
402× 2 + 12, 816. Choice (2) towards west then at 2’0 clock the hour cannot be concluded that they do not have
hand will point towards South east. independence. Option (3) is false as the
176. Except 38 all the other numbers can be
 Choice (2) sentence says they pay a higher price, in
written in the form of n2 −2. Choice (4)

08_Section.indb 184 10/30/2009 11:54:40 AM


Solutions for Mock Tests  n  185

comparison. Hence, this conclusion is far- 200. The second sentence of the para clearly 7. B begins the paragraph by posing a query
fetched. (4) is true. Conformists - people states that the Indian security structure whether corporate prediction markets be-
who occupy a favoured niche pay higher has not taken the spirit of democracy into come a powerful tool by really spotting
price – independence, hence, they are at a consideration. Hence option 1 is support- broader industry trends. A states there
disadvantage. Even though (1) can be in- ed by the passage. Choice 4 can be ruled have been some attempts to find out. Find
ferred that is not what the sentence means. out because it is not stated in the passage out what? The answer for the question
 Choice (4) that the freedom fighter fought for demo- posed in A. D offers an explanation by
cratic rule in India. Choice (1) citing an example. C which carries for-
192. Only (3) is consistent with the idea – if
ward the idea follows D. Hence BADC is
they are convinced that their parents are
the correct sequence. Choice (2)
worth emulating, they are likely to accept
parental authority. Refer to the line ‘when MOCK TEST 5 8. Statement A which is a general statement
parents are impoverished _ _ _ emulation’ ideally begins the paragraph. C is a con-
(last para). Choice (3) Solutions for questions 1 to 4: tinuation of A. ‘That’ in C refers to the
193. Refer to the third, fourth and fifth sen- scientists being worried about the changes
1. The sentence conveys that the others in the ocean currents. Hence A and C are
tences of para 4. It says that the ‘parent- did not approve of his suggestion and
offspring conflict is minimised…style’. linked. This AC combination is seen only
strongly disagreed with it. Although all in 4. Further, statement B follows B as B
 Choice (4) the choices seem probable in the first is a continuation of C. D is conclusive in
194. The river Mithi is 100 ft wide at its point blank, only ‘vehemently’ would be ap- nature. Hence ACBD. Choice (4)
of origin but its width gets reduced by 40 propriate in the second blank, considering
percent as it progresses further. This is due the tone of the sentence Solutions for questions 9 to 12:
to the encroachment and reclamation of  Choice (1) 9. Rather than ‘as well’ we need a conjunc-
land along the river. Refer to para 4 of the tion that connects “many individuals” and
passage. Choice (4) 2. The context of the sentence says that his
outward behaviour was different from “some groups”. “As well” makes sentence
195. The first para of the passage states that what he really was. In such a context, only (1) to be ruled out. In Choice (2) the sin-
many residents of Mumbai think that the ‘facade’ would be appropriate.Choice (3) gular verb ‘has’ makes it wrong. Choice 4
Mithi was a drain. They are unaware of has a wrong structure. Only option (3) is
the fact that the Mithi is a river. This is 3. The sentence conveys that John and his the correct sentence. Choice (3)
one of the probable reason why it is called wife had contradictory characteristics.
The phrase ‘life and soul of all parties’ 10. We use the ‘remaining’ when we can
‘the invisible river’. Choice (1) measure the quantity. “Rest” is used when
conveys that John was a very sociable
196. According to the passage all the given it is uncountable or unknown. Therefore
person, an extrovert and therefore ‘gre-
factors are responsible for turning the Choice (3). Choice (3)
garious’. ‘Garrulous’ and ‘loquacious’
Mithi into a drain. Choice (4)
can be ruled out. ‘truculent’ does not con- 11. The phrase is “last but not the least”.
Solutions for questions 197 to 200: vey the intended meaning in the second Therefore option (1) is wrong. The adverb
197. It is obvious from the passage that rus- blank. Choice (4) ‘thoroughly” should precede the verb dis-
tling (stealing cattle or sheep) is on the 4. The sentence conveys a sense of contrast. cuss. Therefore 4 is correct. Choice (4)
rise in Texas because of the beef prices The universe was once considered to be 12. “Admit” is always followed by ‘to’ and
which are very high. Choice 1, 2 and 4 are peaceful but it is now thought to be in a ‘attention’ cannot be ‘received’ (accept-
not stated. Choice (3) state of complete confusion and disorder. ed). Attention is given. Therefore option
198. According to the passage money cannot Only choice (2) conveys the intended (4) (the original sentence) is correct.
be made easily through illegal means but meaning. Choice (2)  Choice (4)
making money illegally is much easier 5. When compared to statement A, which Solutions for questions 13 to 16:
than making money legally. Hence choice appears to be a continuation of an ear-
1 is not supported by the passage. It is not lier statement, B is better as the opening 13. The noun ‘programmes’ either in singular
stated in the para that a successful man statement as it introduces us to the topic or plural does not alter the meaning. “Ei-
is always wealthy, a prudent person, with on which the rest of the para is based. ther” refers to a singular noun. We can say
good management skills manages to re- D contrasts with what is stated in B by “either candidate” or “either of the can-
main prosperous and such a person is a stating that it was Galileo who laid the didates”. Option (3) is ruled out. Further,
successful person. Hence choice 3 is sup- foundations of modern mechanics. Hence since the subject is singular, the pronoun
ported by the passage. Choice 4 is not D follows B, C is a continuation of D. A ‘his’ is correct. This rules out (2). We say
stated. Choice (3) concludes the paragraph. Hence BDCA. split “into” something and not ‘onto’.
 Choice (4) The correct structure would be in the past
199. Variations are bound to occur while eval- perfect. ‘Could offer’ in (4) rules it out.
uating essay type questions because un- 6. ‘Within these processes’ in statement C Hence (1) Choice (1)
like in objective type questions where dry refers to the exchange of values, of arts,
facts and figures are being tested, in essay of science and technology etc, mentioned 14. The phrase “Local causes or ------” is
type of questions it is the cogency, clar- in B. Hence C is a continuation of B. This connected to two parts, i.e, ‘explained’
ity and the manner of presentation which BC combination is seen in options 1 and and ‘attributed’ hence both should be of
determine the number of marks awarded. 4. Option 4 can be ruled out because D the same strucure and should be followed
Hence there is no objective standard for has to follow A as D is a continuation of by a preposition. The correct form is “ex-
measurement in essay type questions. A. This A D combination is not seen in plained by” and “attributed to” as seen
 Choice (2) option 4. Choice (1) in option (3). This rules out (1) and (4).

08_Section.indb 185 10/30/2009 11:54:40 AM


Technologies and Enterprise Solutions. 5
33. that is,
Choice (3) 30
watch any of
Solutions for question 30:
Choice (1)
30. The man in the photograph is the lady’s only
Solutions for qu
sibling’s wife’s mother’s husband that is, her
sibling’s father-in-law. Thus, the man in 34. The wins of
186  n  Solution Manual photograph is her brother’s son’s grandfather. The teams a
Choice (4) the number o
If a team wins
As the sentence is of a general nature, the are few people who take advantage of the Solutions for for
Solutions questions
questions31 to
31 33:
to 33: Total 80
Total 80
teams with a
auxiliary verb ’can’ is correct.Choice (3) loopholes in the system. It is not said that 5
every person is taking advantage. Hence, Football Cricket 4
15. The phrase “should have been” in 1 and 4
it depends on the individuals, not just the
2 is totally irrelevant in the context and 4
existence of loopholes. Hence, B is the a e c 4
so is ruled out. In option (3), the adverb g
effect but A is not its immediate and prin- 4
‘tentatively’ is wrongly placed and so is b f d
cipal cause. Choice (4) h 4
incorrect. Option (4) is the correct answer 4
choice. Choice (4) Solutions for questions 24 to 26: 4
In the above
In the abovediagram,
diagram, in each area,area,
in each let thelettop
theletter
top 3
16. The “if – condition” is an unreal / imagi- 24. Govind’s mother’s husband’s sister’s
be the number of boys and the bottom letter be the 3
nary situation. So if one was ….” is in- mother is Govind’s paternal grandmother letter be the number of boys and the bottom let-
number of girls. It is given that,
ter who
be the number From the las
formal so incorrect. It should be “if one whose husband, Aravind is Govind’s Boys watch only of girls. =It students
football is given that,
who watch
teams with th
were……”. Option (1) is therefore ruled grandfather, hence Govind’s brother’s is noneBoys who who
= 2(girls watch only
watch football = students who
both)
three wins w
out. The object of the verb is ‘men’ which grandson of Aravind. Choice (2) a =watch
g + h =none
2f = =2x2(girls
(say) who watch both)
not enough.
50 per cent of the students who watch only football
is in plural and therefore, it should be 25. The path followed by Rajesh is as shown areagirls= g that
+ h is,
= 2fa ==b2x = 2x(say) However, fou
‘their’. This rules out option (2) “Cus- in the following diagram. 50 50perper cent
cent of of
thethe students
students whowho watch
watch only foot-
football also
toms of the world” in (3) is erroneous and Starting position 10 km watch
ballcricket thatthat
are girls is, ais,
+a b= = eb += f2x
= 4x 35. From the pre
it should be “customs in the world”. Only As f = x, e = 3x; The number of girls who watch wins and still
He is now facing south. 50 per
Choice (3) football is cent
sameofasthe thestudents
numberwho watch
of girls whofootball
watch
statement (4) is correct. Choice (4) also watch cricket that is, a + b = e + f = 4x 36. The top team
26. The angle between the hands at 11.20 is cricket. that is,
Solutions for questions 17 to 20: b =Asd =f 2x;
= x,50e per
= 3x; The
cent of number
boys watch of girls whoŸwatch
football a+e first stage, 2
11 11 = cfootball
+ g, As ais+ esame= 2x +as3xthe = 5xnumber
Ÿ c + g of=5xgirls who ? The amoun
17. A, B and C are grammatically incorrect. θ= m − 30h = × 20 − 30 ×11 Total boys = 10x; 70 per cent of boys watch cricket = (5 u 50,000
The word ‘advancing’ in A makes no 2 2 watch cricket. that is, b = d = 2x; 50 per cent of
that is, c + e = 7x = 7,50,000
sense. The correction is ‘recent advance c =boys
4x and watch
g = xfootball
Ÿ h = x ⇒ a + e = c + g, As a + e =
= 220 2x + 3x = 5x ⇒ c + g =5x Solutions for qu
in orchid cultivation’. In B the error is in Football Cricket
As angle must be less than 180°, the angle Total boys = 10x; 70 per cent of boys watch 37. The given se
the tense. The word ‘ago’ is used hence 5, 12, 38, 160
is (360° – 220°) that is, 140° Choice (4) cricket that is, c + e = 7x
it should take the simple past tense. The 2x 3x 4x The logic is a
correction is ‘scientists started……’ Solutions for questions 27 to 29: c = 4x and g = x ⇒ h = x 5x2+2, 12x3+3,
In ‘C’ the correction is ‘from an orchid 2x x 2x 33859
Given, Here, 38 and
shoot’. Choice (2) Football Cricket
IM was organised on Thursday x
number which
18. In B the words ‘through the world’ make Also TB was not organized on first and last day; actual numbe
2x 3x 4x x
HR and ET were on consecutive days; PD and Hence, the re
the sentence incorrect. The correction Now, 2x + 2x + 3x + x + 4x + 2x + x + x = 80 Ÿ x
is ‘throughout the world’ or ‘across the ES were on consecutive days ⇒ TB was on = 5 2x x 2x
world’. ‘Damages’ is not the appropriate Wednesday The final diagram is as follows : x Solutions for qu
word in C, ‘damages’ means ‘the com- As ET was organised after PD, PD and ES 38. Word : C E
were organized on Monday and Tuesday respec- Football Cricket x
pensation to be paid’. The correction here
is ‘skin damage’. The correction in D is tively. Now, 2x + 2x + 3x + x + 4x + 2x + x + x
‘the sale’. Choice (2) As HR and IM were not organised on consecu- 10 20 5
= 80 ⇒ x =15 5 Logic: – 2 +
tive days, ET and HR were organised on Friday The final
19. A and D are grammatically incorrect. The 10 diagram
5
is
10 as follows5:
and Saturday respectively.
correction in A is ‘the young’. The words
Football Cricket
‘on a risk’ are incorrect in D. The correc- Monday Product Development
tion is ‘at risk’. Choice (4)
Thuesday Enterprise Solutions 10 15 20 5
20. B is erroneous because it should be ‘a
Wednesday Telecom Billing
powerful panacea’. D is also incorrect be- 10 5 10 5
cause of the usage of the preposition ‘to’, Thursday Infrastructure Management
the correction is ‘possibility of curing’. Friday Emerging Technologies
 Choice (4)
Saturday Human Resources 31. 15 boys watch both the games.Choice (2)
Solutions for questions 21 to 23:
27. ‘Enterprise Solutions’ was organized on 32. The number of boys = 50 and the number
21. The events A and B are related events of girls = 30; ∴ The required ratio = 5 : 3
and B occurs after A. The price rise is the Tuesday. Choice (2)
 Choice (4)
principal cause, but the government’s in- 28. ‘Human Resources’ was on the last day.
action is the immediate cause. Hence, B is  Choice (4) 5
the effect but A is not in immediate cause. 33. that is, 16.66 per cent of the girls did
30
 Choice (4) 29. Two seminars were organized between
Emerging Technologies and Enterprise not watch any of the two games.
22. The events A and B are related events and Solutions. Choice (3)  Choice (1)
a occurs before B. As the water of the
people in slums consume is not protected, Solutions for question 30: Solutions for questions 34 to 36:
it leads to water born diseases. Hence, A 34. The wins of different teams can be as fol-
30. The man in the photograph is the lady’s
is the effect and B is its immediate and lows. The teams are arranged in descend-
only sibling’s wife’s mother’s husband
principal cause. Choice (1) ing order of the number of wins.
that is, her sibling’s father-in-law. Thus,
23. The given events are related events and the man in photograph is her brother’s If a team wins 4 matches, there can not be
A occurs before B. It is given that there son’s grandfather. Choice (4) 3 other teams with a better performance.

08_Section.indb 186 10/30/2009 11:54:41 AM


Solutions for Mock Tests  n  187

5 4 3 2 1 0 40. Except 24824, all are palindromes.  ow applying formula M1D1H1 =


N
 Choice (1) M2D2H2, we have 32 × 6 = 54 × d2; d2 =
4 4 4 2 1 0
4 4 3 2 2 0 Solutions for questions 61 to 80: 32
× 6 = 35/9 days. Choice (2)
4 4 3 2 1 1 61. 4x + 8y = 12 ⇒ 8x + 16y = 24 54
4 4 2 2 2 1 8x + 16y = a ⇒ if a = 24, the given 67. Let the distance from P to Q be x km and
4 3 3 3 2 0 equations will be the same. ∴They have
that from Q to R be y km. Average speed
4 3 3 3 1 1 infinite solutions.  Choice (1)
x+y
4 3 3 2 2 1 62. Inflow of tourist in 2000 = 4000, Inflow =
x y
4 3 2 2 2 2 of tourists in 2001 = 4800, Inflow of tour- +
ists in 2002 = 6000 20 35
3 3 3 3 3 0
3 3 3 3 2 1 Total increase in tourists from 2000 to 140 (x + y) x 4
2003 = 65 per cent; that is Inflow of tour- = 27.5 ⇒ =
 rom the last row we see that there are
F 7x + 4y y 7
ists in 2003 = 6600
five teams with three wins each. So, two Percentage increase in tourists form 2002 ⇒ x : y = 4 : 7 Choice (3)
teams with three wins will get eliminated. to 2003 = 600/6000 x 100 = 10 per cent
So, three wins are not enough.  Choice (3)
However, four wins are enough. 68. 3x = 4y = 6z = k (say ); x = k/3; y= k/4
63. As the number is divisible by 72 it is di- z = k/6
 Choice (2) visible by 8 and 9 also. The sum of the
digits 44x8y is 16 + x + y. For the number k k k 2
35. From the previous question, a team can x : y : z = : : = 4 : 3 : 2; z = × 36
score 3 wins and still get eliminated. to be divisible by 9, the least value of x + 3 4 6 9
 Choice (4) y should be 2. As the number is divisible = 8 dozen. Choice (3)
by 8, the last three digits have to be divis-
36. The top team can win at the most 5 games 69. The total distance = (3)(20) + (4)(15) +
ible by 8. ∴ x8y cannot be 082 as it is not
in the first stage, 2 in the second and 2 in (3)(30) = 210 km; The average speed =
divisible by 8 but it can be 280. that is x =
the third. 210
2 and y = 0. ∴ x + y = 2. Choice (3)
∴ The amount won by the top team in
64. The rest of the tank to be filled is the part 3+ 4 + 3
rupees
= (5 × 50,000) + (2 × 1,00,000) + (2 × of the tank which all the taps can fill in 1 = 21 kmph Choice (1)
1,50,000) = 7,50,000 Choice (4) hour. 70. The required remaining work =
The part of the tank which can filled in
Solutions for question 37: one hour by the three taps together = 4 3 1 9
1− − × = . Choice (1)
37. The given series is: 1 1 1 15 7 7 4 28
5, 12, 38, 160, 805, 4834, 33859 + + =
6 12 16 48 71. 24 men +12 women can do the work in
The logic is as follows.
15th 6 days. 18 men + 24 women can do the
5x2+2, 12x3+3, 39x4+4, 160x5+5, 805x6+6, Time taken by the taps P and Q to fill work in 9/2 days
4836x7+7, 33859 48
∴ 144 men + 72 women = 81 men +108
Here, 38 and 4834 are the wrong num- 15
women ⇒ 63 men = 36 women ⇒ 7 men
bers. The number which differs by a 1
of the tank = 48 =1 hourChoice (1) = 4 women
greater margin with the actual number is 1 1 4
4834 (-2). + Now, 24 men + 12 women = 24 men +
6 12 21 men can do in 6 days working 6 hours
Hence, the required wrong number is
65. x + 3z = 25 ------------------ (1) a day.
4834. Choice (2)
2y + z = 10 ------------------ (2) that is, 45 men can do the work in 6 days
Solutions for questions 38 to 40: x–y=5  ------------------ (3) working 6 hours a day
38. Word : C  E  N  T  U  R  Y Subtracting equation (3) from (1); we get we have M1D1H1 = M2D2H2 ; 45 × 6 × 6
↑  ↑  ↑  ↑  ↑  ↑  3z + y = 20 ------------- (4)
Multiplying the equation (4) by 2 and 45 6 15
Logic: – 2  + 2  – 2  + 2  – 2  + = 24 × 9 × D2; D2 = 6 × × = days
2  – 2 subtracting it from equation (2), we get z 24 9 2
↓  ↓   ↓   ↓   ↓   ↓   = 6, y = 2, ∴ x = 7  Choice (4)
Code: A  G  L  V  S  T  W ∴ 2x + y – z = 2 x 7 + 2 – 6 = 10 Choice 72. The distance between the first post and
Similarly, (2) the eighteenth post = 17 x m.
Word: S    A  C  H  I  N The distance between the eighteenth post
66. As the work in same in both the case, the
↑  ↑  ↑  ↑  ↑  ↑  number of man days in both the cases is and the thirty fourth post = 16x m.
Logic: – 2  + 2  – 2  + 2  – 2  + 2 also the same. 306
↓  ↓   ↓   ↓   ↓   Let the work done by a boy and girl per Time taken to cover 16x m = × 16x
Code: Q  C  A  J  G  P; ∴ QCA- 17x
day be B and G respectively.
JGP is the code for ‘SACHIN’. (8B + 16G)6 = (16G + 4B)8 ⇒ B = 2G; = 288 sec Choice (3)
 Choice (3) Work done by 8 boys and 16 girls = 8 × 73. Let the strength of the class be x. ∴60x
39. Ravi’s father’s paternal uncle’s only neph- 2G + 16G = 32G = 10 × 80 + (x – 10) × 50 ⇒ 10x = 10 ×
ew is Ravi’s father. His sister’s mother is Also work done by 17 boys and 20 girls (80 – 50)
Ravi’s grand mother. Choice (3) = 17 × 2G + 20G = 54G ∴ x = 30 Choice (2)

08_Section.indb 187 10/30/2009 11:54:41 AM


188  n  Solution Manual

74. Given data can be represented as a dia-  1 – (0.6) (0.7) (0.8) (0.9) = 1 – 0.3024 =
= 84. The first statement alone is not sufficient
gram as shown. 0.6976 Choice (4) as we do not know whether town B is be-
If BA be represented by ‘x’, then BD 12 tween towns A and C or not. The second
78. Share of P = × 1350 = Rs 675; Share
= x. 24 statement is not sufficient as it gives no
information about the distance. Even by
12
C E of Q = × 1350 = Rs 450 combining both the statements, we cannot
36 answer the question, because the locations
Share of R = 1350 – 675 – 450 = Rs 225 of A, B and C are not known. Choice (4)
D  Choice (3)
85. Either statement alone is not sufficient as
150 they contain partial information. Com-
79. The speed of boat in still water = (8)
100 bining both the statements, Let x be the
B A number of employees. ∴ The number of

24 employees having cars = 0.32x ∴ The
Median on to the hypotenuse of a right- = 12 kmph; Downstream time =
angled triangle, is half the hypotenuse; 8 + 12 number of employees having a car and
that is BD = 1/2 AC, ⇒ AC = 2BD = 2x = 1.2 hours owning a house = 0.32x (0.6) = 0.192x ,
that is, in ∆ABC, right angled at B, 24 which is 19.2 per cent of x. Choice (3)
Upstream time = = 6 hours; Aver-
Hypotenuse AC = 2x = 2 AB, ⇒ ∠BCA 12 − 8 86. Either statement alone is not sufficient as
= 30° and ∠BAC = 60° . . . (1) we do not known the weight of Neeru.
Given CE is perpendicular to CB. 24(2) 2 By combining both the statements, we
age speed = = 6 kmph
Hence, angle ECA = 90° − 30° = 60° . 1.2 + 6 3 can answer the question as we have the
 Choice (3) sum of their weights and the ratio of their
. . (2)
Given that AC is the bisector of the angle 80. Quantity of mangoes purchased = 40 kg; weights. Choice (3)
BAE, ⇒ ∠BAC = ∠CAE = 60° . . . (3) Rate of mangoes per kg = Rs 30
Solutions for questions 87 to 91:
From (2) and (3), ∠CEA = 60°, ⇒ ∆AEC Total cost price = Rs 1200
is equilateral; Transportation = Rs 200 87. Profit from steel division = 21 per
⇒ AE = EC = CA = 2x . . . (4); I n Total cost = Rs 1400; Expected profit cent of (20 per cent of 35) crores =
∆ABC, AC = 2x, AB = x, hence, BC = = 20 per cent 21 20
Total expected selling price × × 35crore = 1.47 crores.
3x . . . (5) 100 100
Choice (4)
1400 × 120
Ratio of BC to AE = 3x : 2x = 3 : 2 = Rs = Rs 1680; Quantity of 
100 88. Only for Pharma, Steel and Software de-
 Choice (3)
25 velopment division, the profit percent is
75. A’s capital = Rs 5000; B’s capital = Rs mangoes spoiled = × 40 = 5 kgs
200 more compared to the percentage of reve-
8000; A’s time period = 12 months nue distribution. So highest profit percent
B’s time = (3 + 3) = 6 months; C’s cap-  otal selling price of spoiled mangoes
T is from these sectors only.
ital = Rs14,000; C’s time = 12 – 3 = 9 (5kg) = 5 × 21 = Rs 105 Profit percent from Pharma division =
months Remaining amount = Rs 1680 – 105 = Rs
The ratio of profits of A, B and C = 5000 1575; Quantity of 9×7
× 100 = 30 per cent
× 12; 8000 × 6; 14000 × 9 = 10 : 8 : 21 mangoes = 35 kg 6 × 37
10 parts of total profit = Rs 1400; 8 parts
1575 Profit percentage from Steel division =
8 ×1400 Required selling price = Rs =
of total profit = = Rs 1120 35 21 × 7
10 × 100 = 231/3 per cent
 Choice (4) Rs 45 Choice (4) 18 × 35
Profit percent from Software develop-
76. Let the price per unit be Re.1, Sales be Solutions for questions 81 to 86: 36 × 7
ment division = × 100 = 24% 
100, Revenue = Rs 100 ------- (1) 30 × 35
When the price is decreased by 30 per 81. Either statement alone is not sufficient as
cent. The new price = Rs 0.7, Sales both of them give a + c = 2b, from which  Choice (4)
= 120 we can not say that a, b and c are consecu-
89. Revenue from FMCG division = 24 per
Revenue = 120 × 0.7 = Rs 84 tive. Even by combining both the state-
cent of 35; Profit from Software develop-
Decrease in the revenue = Rs 16. Percent- ments, we cannot answer the question.
ment = 36 per cent of 20 per cent of 35.
age decrease in the revenue = 16 per cent  Choice (4)
Required ratio = 24 per cent of 35 :
 Choice (1) 82. Clearly, either statement alone is not suf- 36 per cent of 20 per cent of 35 = 10 : 3.
77. The probability of the missiles (A, B, C ficient.  Choice (1)
and D) to hit the target is 0.4, 0.3, 0.2 and Even by combining both the statements, 90. Revenue from Cement division
0.1 respectively. we cannot answer the question.
22
The probability of a missile hitting the  Choice (4) = × 35 = 7.7 crore
100
target = 1 – [Probability of none of the
83. From the first statement above, as both
missiles hitting the target] 14
Mahendar and Shilpa are standing to- Profit from Cement division = × 20%
= 1 – [ P(A) × P(B) × P(C) × P(D) ] gether, the ratio of their Shadows is the 100
Now the probability of missiles hitting of 35 = 0.98 crore.
same as their respective ratio of heights.
the target Expenditure on Cement division =
Second statement is redundant.
= 1 – [(1 – 0.4) (1 – 0.3) (1 – 0.2) 7.7 − 0⋅98 = 6.72 crores.
(1 – 0.1)]  Choice (1)  Choice (2)

08_Section.indb 188 10/30/2009 11:54:42 AM


Solutions for Mock Tests  n  189

91. Revenue from Steel division = 18 per cent Value of Share holding of Morgan Stan- 102. Let the present ages of father, mother,
of 35 = 6.3 crores; Profit from Steel divi- son and daughter be f, m, s and d respec-
10
sion = 21 per cent of (20 per cent of 35) = ley = 294 × = Rs 29.4 Crores; ∴ tively.
1.47 crores 100
f +m+s+d
Revenue from FMCG division = 24 per Difference in the value of the share hold- Given = 24; 5 f = 4(m + s)
cent of 35 = 8.4 crores 4
Profit from FMCG division = 20 per cent ings of Birla Sunlife and Morgan Stanley 5f
and d = 6 ⇒ f + + 6 = 96; ⇒ f =
of (20 per cent of 35) = 1.4 crores = 35.25 – 29.4 = Rs 5.85 Crores 4
Profit percent on Steel division  Choice (3) 40 years Choice (1)
1.47 1 103. Time taken to meet for the first time at the
= × 100% = 23 % 98. Value of Share holding of Metlife in June
6.3 3 starting point
23.5 10 = LCM of
Profit percent on FMCG division 2003 = 1000 x × = Rs 23.5
100 100
1.4 2 Crores  Length of track Length of track 
= × 100% = 16 % ; Required  , 
8.4 3 Value of Share holding of FI’s in March  Speed of A Speed of B 
 1 2 2 6.9
percentage points =  23 − 16  = 6 2003 = 1000 × = 69 Crores  18 18 
 3 3 3 100  LCM of  ,  = LCM of (6, 3) = 6
=
 Choice (4) Value of Share holding of Metlife in June  3 6
hours. Choice (3)
2003 as a percentage of that of FI’s in
Solutions for questions 92 to 95: March 2003. 104. t1 = 1; t2 = 2; t3 = t1 + 2t2 + 5 = 1 + 2(2) + 5
1200 − 1100 1300 − 1200 1400 − 1300 = 10;
92. Of , , 23.5
1100 1200 1300 = × 100 = 34.05 per cent  t4 = t2 + 2t3 + 5 = 2 + 2(10) + 5 = 27;
1500 − 1400 1200 − 1100  69 Choice (4) t5 = t3 + 2t4 + 5 = 10 + 2(27) + 5 = 69
and , is the  Choice (2)
1400 1100 99. The maximum percentage change would
greatest. mean either an increase or a decrease in 105. Let the length of the train be x m. Its
the share holding value 5
 Percentage increase will be the highest
∴ speed = 72 × = 20 m/s
Others (Increase of 0.5 percentage points) 18
from 2001 to 2002. Choice (2)
0.5 x = 20 × 22.5 = 450 m. Time taken by
93. Of 20 − 10 , 30 − 20 , 45 − 30 and 50 − 45 , 20 −10 = × 100 = 10.20 per cent
10 20 30 45 10
4.9 450 + 350
Public(Decrease of 0.4 percentage points) it to cross the tunnel = = 40
is the greatest. Choice (4) 20
0.4 seconds Choice (2)
= × 100 = 4.08 per cent
94. Of
10 20 30 45 50 50 9.8
, , , and , 106. From choices.
1100 1100 1300 1400 1500 1500 FI’s (Decrease of 1.6 percentage points) =
is the greatest. Choice (3) 1.6 (1) ( 5 + 1 − 2 )2 = 8 + 2 5 − 2
× 100 = 23.18 per cent
10 20 30 45 50 6.9 10 − 2 2
95. As < < < < , the
1100 1200 1300 1400 1500 FII’s (Increase of 2.8 percentage points) = (2) ( 5 + 2 − 1)2 = 8 + 2 10 − 2 5
correct answer is (3). Choice (3) 2.8
× 100 = 10.52 per cent; FI’s witness −2 2
26.6
Solutions for questions 96 to 100:
the highest percentage change. (3) ( 5 + 2 + 1)2 = 8 + 2 10 + 2 5
1000
96. Total shares of Zee Telefilms =  Choice (3)
100 +2 2
= 10 crores
Number of shares held by Indian promot- (4) ( 5 – 2 − 1)2 = 8 – 2 10 − 2 5
100. per cent Share holding of FII’s in March
23.5 ’03 = 26.6 per cent; per cent Share hold- +2 2 
ers = 10 × = 2.35 Crores  Choice (2)
100 ing of ‘others’ = 5 per cent of 26.6 per
 umber of shares held by Pru ICICI =
N cent = 1.33 per cent 107. When A, would have run 1000 m B would
Sunshine investments is 10 per cent of have run 900 m and C would have run
2.35 × 25 850 m.
= 0.5875 crores = 58.75 Lakhs 1.33
100 13.3 = 1.33 = × 100 = 0.133% So in a 900 m race, B beats C by 900 –
Choice (2) 1000 850 = 50 m. In a 2250 m race, B beats C
 of total shareholding of Zee Telefilms.
2250
97. Value of share holding of Indian pro- Choice (2) by = × 50 = 125 m Choice (2)
900
23.5 
moters = 1000 × = Rs 235 Crores
100 108. Sum of the angles in a triangle is 180°
Solutions for questions 101 to 120:
Value of share holding of Birla Sunlife ⇒ x + 2k + x + x + 3k = 180°; ⇒ 3x +
15 2x 2 (40 )(60 ) 3x
= 235 × = Rs 35.25 Crores 101. Average speed = = 5k = 180°, k = 36 – . The largest pos-
100 x x 40 + 60 5
+  3x 
Value of Share holding of FII’s = 1000 40 60 sible angle = x + 3k = x + 3  36 − 
 5 
29.4 = 48 kmph 4x
× = Rs 294 Cr = 108 –
100  Choice (3) 5

08_Section.indb 189 10/30/2009 11:54:44 AM


Choice (3) (i) B/D ___ C ____ D/B E/F F/E
(ii) E/F F/E B/D C ____ D/B ____
116. 2r = 14 Ÿ r = 7 and h = 20. ? Volume of the
cylinder = S u 7 u 7 u 20 = 3080 cm
3
From (3) and (5), we get the above possible cases as
? Volume of the wood wasted = 14 x 14 x 20 – follows.
3080 = 840 cm Choice (4) (i) G D A C B E/F F/E
(ii) E/F F/E D C A B G
117. Let the width of the room be x m
190  n  Solution Manual (4 u x + 4 u 3x) u 2 = 96 Ÿ x = 3 m ? The 121. The statement has no reference to weight lifting.
volume = 9 u 3 u 4 = 108 m3. Choice (3) Hence, , does not follow. The words ‘there
ought to be’ indicate that at present there is no
 hen x = 5, maximum angle
W 118.=Let
108 AB
– 4 be
= wall and AC A 121. The statement has no reference to weight
such law. Hence, ,, follows. Choice (2)
104°. be ladder
Choice (1) lifting. Hence, Ι does not follow. The
Given, AB : AC = 1 : 2, 122. It is a general statement made by Brian Lara
words ‘there ought to be’ indicate that at
109. Let the length of the race be x m. Let AB the1 and so it does not necessarily imply that he plays
Ÿ present there is no such law. Hence, ΙΙ fol-
4y on Saturdays and Sundays. Hence, , does not
speed of B be y m/s. Speed of AAC = 2 90° follow. lows.  Choice (2)
3 ABC, sinC =
In triangle
From his 122.
statement,
It is it can
a be concluded
general statement that,
made by Brian
m/s. A and B finish the race simultane-
AB 1 B C he is aware of the difficulties of playing on
ously. So time taken by A to cover, sinC x m= Lara and so it does not necessarily imply
AC 2 Saturdays and Sundays. Hence ,, follows.
is the same as that taken by BC to cover
= 30°; ‘A + ‘B + ‘C 180° that he plays on Choice Saturdays (2) and Sundays.
(x − 120) m. ‘A = 180° – (‘B + ‘C)
AB 1 Hence, Ι does not follow.
In=180° – 120°,
triangle ABC,‘A = 60°
sinC = ,123.
sinCThe
= statement does not indicate whether there
Angle between wall and ladder is 60° AC 2
is any other reason From thathiscanstatement,
cause erosionit canofbe concluded
x x − 120 3x x −120
= ⇒ = ⇒ 3x C = 30°; ∠A Choice
+ ∠B +(3)∠C 180° that, he is aware
ethnic culture. Hence, neither offollows.
, nor ,, the difficulties of play-
4y / 3 y 4y 119.yFor the 1st year, sum borrowed by Mr. Sanjay =
 Rs 25000 ∠A = 180° – (∠B + ∠C) =180° – 120°, ing on SaturdaysChoice and Sundays.
(3) Hence ΙΙ
∠A = 60° 124. From the statement it is clear that the number of Choice (2)
follows.
= 4 (x – 120) ⇒ x = 480 Choice
Amount (2)at the end of 1st year =
states in the country is at least 12. But the
§ 12 · Angle28between wall and ladder is 60° statement is 123. The statement
not clear whether there doesare notmoreindicate whether
110. The total area covered 25000 ¨ 1  ¸ = 25000 u = 28000
© 100 ¹  25 Choice (3) in the country
states therethan that other
is any indicated
reasonin the
that can cause
22 Amount statement. Hence,erosion
, does not follow. culture.
,, is out Hence,
of
= 10 × 2 × × (0 ⋅ 7 ) × 4 = 176 m2 paid = 119. Rs 8000.
For the Principal for the
1st year, sum2ndborrowed by Mr.
context. Hence, neither , nor
of ethnic
,, follows.
neither
7 year = Rs 20000 Ι nor ΙΙ follows. Choice (3) Choice (3)
Sanjay = Rs 25000
Amount at the end of the 2nd year = 20000 u
 Choice (2) Amount at the end of 1st year = 25000 124. From the statement it is clear that the
28 125. The words ‘fate decided otherwise’ indicate that
= 800 u 28 = 22400
25  12  28 number
Sabita did not pursue theofprofession
states in the
thatcountry
her is at least
111. The favourable combinations are A, 1+  = 25000 × = 28000 parents wanted her
Amount paid at the end of the 100second
 year = Rs
25 12. to.
But Hence,
the , follows.
statement is Theclear whether
not
statement does not reflect Sabita’a opinion.
A R A, A R A R A, ….. and8000. Principal for the 3rd year =
so on. there are more states in the country than
Rs (22,400 – 8000) Amount paid = Rs 8000. Principal for the ,, does not follow.
Hence, Choice (1)
∴ The required probability is that indicated in the statement. Hence,
= 14,400. Amount to 2nd yearat= the
be paid Rs 20000
end of the Solutions for questions 126 to 129:
1 Ι does not follow. ΙΙ is out of context.
28
1 1 1 2 23rd year to clear the debtAmount
= 14,400at uthe end
= 144 Givenyear =
ofu the 2nd Hence, neither Ι nor ΙΙ follows.
+ + +−−−− = =  25  Choice (3)
2 8 32 1 34 u 28 = Rs 16,128 28 Choice (4) A B C D E F
1− 20000 × = 800 × 28 = 22400
Choice (3) 4 25 Shirt 125. The words
Green ‘fate decided otherwise’ indi-
120. Let PC = x and BC = h
In triangle PBC Trouser cate that Sabita did not pursue the pro-
Black
Amount paid at the end of the second year fession that her parents wanted her to.
22 h h
tan 45°2= ;1= Ÿ = xRs = h8000.
--------Principal
(1) for the 3rd yearE=have no colourHence,
C and in common
Ι follows. The statement does not
112. The area of the path = (21 – 14 ) =x
2
x Rs (22,400 – 8000) Ÿ B, C, D and E
7 reflect Sabita’a opinion. Hence, ΙΙ does
300 cannot wear green; Ÿ A
22 Now, in triangle PAC; = 14,400.
tan60° = Amount to be paid atA the end of
wears green. not follow.  Choice (1)
× 35 × 7 = 770 m2 x
the 3rd year to clear the debt = 14,400 × 300
7 Solutions for questions 126 Sol/1011
B to 129:
28
∴ The cost of gravelling the path = 770 = 144 × 4 × 28 = Rs 16,128 Given
25 h
× 5 = Rs 3850 60°
 Choice (4)  Choice (4) A B45° C D E F
P C
113. When three coins are tossed, sample space 120. Let PC = x and BC = h
Shirt Green
has 23 = 8 outcomes, of which (HHH) is In triangle PBC
Trouser Black
the only favourable case. Required prob-
ability = n(E)/n(S) = 1/8 Choice (1) h h
tan 45° = ; 1 = ⇒ x = h -------- (1) C and E have no colour in common
x x
114. The required number of coins ⇒ B, C, D and E cannot wear green; ⇒ A wears
300 green.
π (6 ) (20)
2 Now, in triangle PAC; tan60° =
= = 600 x Given E did not wear Blue. But blue shirt should
π (2 ) ( 0.3)
2 300 be to the right of white shirt.
x= ------ (2)
 Choice (2) 3  ⇒ A and B (or) E and F should wear white and
blue shirt respectively.
300
115. The required probability = Equating (1) and (2), h = = 100 3
3
A
 1  1  1  1
    =  Choice (3) m = 173.2 m Choice (4)
6 6
    6 216
Solutions for questions 121 to 125: B 300
116. 2r = 14 ⇒ r = 7 and h = 20. ∴ Volume of From (1) and (2), we get,
the cylinder = π × 7 × 7 × 20 = 3080 cm3 60° h
(i) _____ B/D ___ C ____ D/B ____ ____
∴ Volume of the wood wasted = 14 x 14 (ii) _____ ____ B/D C ____ D/B ____ 45°
x 20 – 3080 = 840 cm Choice (4) P C
Also given that E and F sit together.
117. Let the width of the room be x m ∴ The above arrangement becomes Case (i): If A and B wear white and Blue shirts
(4 × x + 4 × 3x) × 2 = 96 ⇒ x = 3 m ∴ The (i) B/D ___ C ____ D/B E/F F/E respectively, D wear Black shirt. As the person
volume = 9 × 3 × 4 = 108 m3. Choice (3) (ii) E/F F/E B/D C ____ D/B ____ who ears Red shirt should wear white trousers,
From (3) and (5), we get the above possible it can be only E.
118. Let AB be wall and AC be ladder
cases as follows. ⇒ F wear s Indigo shirt, D wears Blue trousers,
AB 1 (i) G D A C B E/F F/E
Given, AB : AC = 1 : 2, ⇒ = C wears Indigo trousers and B wears Red trou-
AC 2 (ii) E/F F/E D C A B G sers.

08_Section.indb 190 10/30/2009 11:54:45 AM


Solutions for Mock Tests  n  191

Similarly case (ii) can be operated. (But it con- If R is selected, then C and S cannot be selected 144. According to the first sentence of para 8,
tradicts the conditions given) and one among B and M is not selected (which choice (3) is the answer. Choice (3)
It can be tabulated as follows. is a contradiction since two students must be 145. The penultimate sentence of para 1 states
selected). that hoice (4) is the answer to this ques-
A B C D E F
Shirt White Blue Green Black Red Indigo ∴ R is not selected. tion. Choice (4)
Trouser Green Red Indigo Blue White Black G is selected (from (1)) and C is selected 146. It is stated in the last sentence of para 2
 (from (4)). that the European Commission has the
126. Eswar is wearing a red shirt. From (5) either both A and B are selected or both power to impose its own fines.Choice (1)
 Choice (3) are rejected.
147. The first sentence of para 4 states that 3 is
127. Amar is wearing green trousers. If A and B are selected, then A, B, C and G are the answer. Choice (3)
 Choice (1) the persons who are selected (which satisfy all 148. The seventh para of the passage states
128. Dinesh is wearing a black shirt. conditions). that choice (2) is the answer. Choice (2)
 Choice (2)
If A and B are not selected, P and M are se- 149. Para 6 of the passage states that the phrase
129. Amar is wearing a white shirt and green lected and as P is selected, S should be selected ‘the very cradle of life on earth’ implies
trousers. Choice (4)  (from (3)). that oceans are a place from where life
Solutions for questions 130 to 134: originated. Choice (1)
But here none among C and R is selected
130. From the words ‘in the event of mid-air  (from (4)) 150. Refer to para 8 of the passage. The line
emergency’ Ι follows. The statement has Hence A, B, C and G are selected where B is a – ‘well over 60 per cent of the marine
no reference to the President. Hence, ΙΙ senior student, C is - junior student, G is a junior world….’ Indicates that choice 2 is the
does not follow. Since a helicopter was faculty and A is a senior faculty. answer. Choice (2)
kept on high alert, ΙΙΙ follows.Choice (3) 151. Refer to para 4 of the passage which states
B and G are assigned to a project and A and C
131. The statement refers to the Indian IT that the rate at which we exploit nature is
are assigned to the other.
sector being taken by storm with the an- in excess of restoration effects. (refer to
nouncement by IBM. It could be that such 135. Chandu is selected. Choice (1) the line –) it is a pace of change that has
a big investment is after a long time and 136. Bimal and Govind are assigned a project. outstripped our institutions and conserva-
not necessarily for the first time. Hence, Ι  Choice (3) tion efforts. Choice (4)
and ΙΙ do not follow. There is no reference 152. The UN said humankinds exploitation
to the status of the company. Hence, ΙΙΙ 137. Only Govind is a junior.  Choice (1)
of the deep rear and oceans was ‘rapidly
does not follow. Choice (4) 138. Bimal and Surya are both senior students. passing the point of no return’. (refer to
132. The statement does not relate pioneers  Choice (3) the first para of the passage). This implies
with the ability to give international stan- 139. Mayur is junior student is the only true that the damage to the environment is ir-
dard flight training. Hence, Ι does not statement. Choice (4) redeemable.  Choice (3)
follow. From the statement, it cannot be
Solutions for question 140: 153. According to the 1st para of the passage
found out what the people do not prefer.
fuel cells, at present are being used for in-
Hence, ΙΙ does not follow. The first state- 140. Word : S U M M E R creasing the life of batteries. (refer to the
ment indicates 'being pioneers' as the Pattern: +1 +1 +1 +1 +1 +1 penultimate sentence of para 1)
reason for the ability to understand flying Code: T V N N F S  Choice (2)
better. Hence, ΙΙΙ follows. Choice (4) Similarly, the code for WINTER is
XJOUFS. 154. According to the 1st sentence of para 3,
133. ‘Giving away the rights’ is called foolish.
 Choice (3) choice 3 is the answer. Choice (3)
Hence, ΙΙ follows. The reason for which
the rights were given away is not men- Solutions for questions 141 to 156: 155. Refer to the third and fourth sentences of
tioned in the statement. Hence, either Ι or para 2 which state that choice 1 is the an-
141. Refer to the first two sentences of para swer. Choice (1)
ΙΙ and ΙΙΙ follow. Choice (4)
1 which state that the supply of uranium
134. There is no information in the statement to stopped when India used the plutonium 156. Refer to the first sentence of the last para.
indicate that radio is not available to any generated in reactors for making a bomb.  Choice (4)
one other than the common man. Hence,  Choice (1)
Ι does not follow. Statement ΙΙ indicates Solutions for questions 157 to 160:
142. According to the second sentence of para
that there was no programme broadcast 157. The industrialised nations had agreed
5 choice (1) is true. Choice (2) is true
on radio, which airs songs of listeners' to lower pollution level through climate
according to the last and penultimate
choice until 1980, Hence, ΙΙ does not fol- policies. According to the paragraph the
sentences of para 5. para 6 and 7 imply
low. reduction in pollution levels would sug-
that choice (4 is true. Refer to the last
The statement has no reference to various gest that the industrialised nations were
line, of para 7- ‘Once this gets estab-
programmes that are broadcast on radio. on their way to fulfil their promise. But
lished___________’.
Hence, ΙΙΙ does not follow. Choice (4) (3) shows that the reduction was not due
Choice (3) is not-true because it is stated
Solutions for questions 135 to 139: to implementation of climate polices but
(in the last line of para 7 that India’s ener-
Let us represent the people with the first letter due to a political development. Hence (3)
gy independence is likely to be achieved
of their names. is the correct answer. (1) is incorrect as
around 2050. Choice (3)
the argument is not about the number of
Now a project must contain a senior faculty 143. It is stated in some places in the passage, factories (2) is wrong as it is about non-
member and a junior student. The other project particularly the last line of the passage industrialised nations.(4) is incorrect as
must contain a junior faculty member and a se- that the author feels obliged to the U.S. in the question is only about the result and
nior student. one way or the other. Choice (3) not the means of achieving it. Choice (3)

08_Section.indb 191 10/30/2009 11:54:45 AM


192  n  Solution Manual

158. As the economic growth has reached only 185. Since, the market index is showing a
a small fraction of the population the positive return on investment, atleast one 1.02 × 107
income of this small fraction must have of the 500 different companies making up Bonus per engineer = = Rs
2040
increased substantially. The incomes of the index should record a positive return
the poor have not increased significantly.  Choice (2) 50,000. Choice (2)
Hence economic growth has actually wid- 193. Choice (3)
ened the gab between the income level (4) Solutions for questions 186 to 190:
is correct  Choice (4) 194. Zen → 500 cars/day; Profit/ car = 1.5
186. Total sales of A and B in 1990
lacs/car; City = 600 cars/day; Profit/car =
159. According to the passage after every ter- = (320 + 290) = 610
0.8 lacs/car
rorist attacks the investigating authorities Total sales of A and B in 1993
Total profit for Zen = 750 lacs/day; Total
go into the details. But this has not been = (500 + 420) = 920
able to prevent future attacks. If (2) were Hence percentage increase 750
profit for City = 480 lacs/day ≅ 156
true it is bound to happen as the authori- 480
ties won’t be prepared for the innovative (920 − 610)
= x100 = 310/610 x 100 = per cent Choice (2)
attack (2) is correct. Though (1) may give 610
Manufacturing cos t M
the reason why terrorism continues, if 31/61 x 100 ≅ 50 per cent Choice (3) 195. = is least
does not explain the reason for the fail- Total Cost M+A
ure (3), (4) is only the reason why ter- 187. Total sales of A = (320 + 380 + 370 + 500)
rorist attacks are generally successful. = 1570; Total sales of B = (290 + 300 + M+A A
⇒ is highest ⇒ 1 + is highest
 Choice (2) 500 + 420) = 1510 M M
As the sales of A and B are given for same ⇒ A is per cent of M is highest, this is
160. Inflation causes an erosion of money val- number of years, the required percentage
ue. This would require the poverty line to true for Zen. ∴ Least for Zen Choice (2)
can be calculated on totals. Hence per-
be raised. So if old standards are applied
1570 − 1510 Solutions for questions 196 to 200:
some of those who are really poor may be centage increase = × 100
deemed out of poverty line. So this might 1510 196. By squaring the inequality, we get 81 <
have decreased the percentage of the poor = 60/1510 x 100 ≅ 4 per cent  x2 – 10 < 100 ⇒ 91 < x2 < 100
and not the measures taken by the govern-  Choice (1)
ment as claimed by it. So (1) is the correct I f x2 > 91; x > ± 91 ; If x > 91 then
answer. 188. Clearly by looking at the graph the maxi- x > 6.
(2) If anything can only strengthen the mum increase between any two consecu- If x2 < 110 ⇒ x < 9. ∴ x may be less than
government claim (3) and (4) are not rel- tive years is for B from 1991 to 92. Also by 6
evant to the argument.  Choice (1) looking at the graph no where the sales of A ∴ The relationship cannot be determined.
or B increased by more than half except for  Choice (4)
Solutions for questions 181 to 185: B in 1992. Hence percentage increase will x z pxz
181. By observation, we can find that the high- be maximum for B at = 200/300 x 100 = 2/3 197. x xp= ;
100 100 10, 000
est return could be in 5th week for com- x 100 = 66.66 per cent  Choice (3)
pany A or B, or in 8th week for company 189. Sales of A in 1991 = 380 p x pxz
x xz=
A, or in 3rd for company B. The increase Sales of A in 1992 = 370 100 100 10, 000
in the prices for each of the above three The two quantities are equal. Choice (3)
instances is Rs 150. To get the highest re- Hence percentage decrease x
turn, the denominator should be the low- 198. If = 0 x is definitely equal to 0. If y is
(380 − 370) y
est which is there in case for company A = × 100 = 10/380 x 100
in the 4th week, so it has to be the high- 380 positive y > 0, x < y; If y is negative, x > y
est. = 1/38 x 100 = 2.63 per cent Choice (2) The relationship cannot be determined.
150  Choice (4)
∴ The return is x100 = 60% 190. Sales of A in 1993 = 500; Sales of A in
250 1994 = 500 x 1.2 = 600 40 2
 Choice (1) 199. x = x x1330 = 152; y
Sales of B in 1993 = 420; Sales of B in 100 7
182. A outperformed C in the following weeks 1994 = (420 − 420/3) = 420 − 140 = 280
1st, 3rd, 5th, 6th, 8th  Choice (3) 1 1 4 247 4
Required percentage = 280/600 x 100 = = 82 x x xx = x xx
3 100 5 300 5
183. The weeks are 3rd, 4th and 7th, Totally, 3 280/6 ≅ 46 per cent Choice (3)
weeks Choice (3)  ince 247 < 300 and 4 < 5 ⇒ 247 x 4 <
S
Solutions for questions 191 to 195: 300 x 5. ∴y < x
184. In a span of 8 weeks, Stock of A goes up Since x is positive. We can say that x > y.
by 50 per cent (300 → 450) 191. We need the car for which the ratio of
 Choice (1)
Stock of B goes up by 36.36 per cent(330 M
→ 450) is least by observation, Zen (50 per 200. Let the population in 1992 be 100. ∴
A Population in 1997 = 120 (20 per cent in-
∴ Value of Stock A is 9900 (1.5) =
14850 cent) and Versa (66 per cent) are amongst crease on 100)
the last and the correct answer is Zen. Also population in 2002 = 140 (20 per
 450   Choice (4) cent increase on 100); ∴ Percentage in-
 alue of Stock B is 9900 
V =
13500  330  crease in population from 1997 to 2002 =
192. Profit = 3000 x (9.3 – 7.6) lacs = 1.7 x 105
∴ Total worth of the stock = Rs (14850 + x 3000 = 51 crores; 2 per cent of 51 crores 20
13500) = Rs 28,350. Choice (2) = 1.02 crores x100 = 16.66 per cent Choice (1)
120

08_Section.indb 192 10/30/2009 11:54:46 AM

You might also like